You are on page 1of 358

INTRODUCTION/CLASSIFICATION OF PRIVATE CORPORATIONS

[G.R. No. L-64013. November 28, 1983.]

UNION GLASS & CONTAINER CORPORATION and CARLOS PALANCA, JR., in his capacity as
President of Union Glass & Container Corporation, petitioner, vs. THE SECURITIES AND
EXCHANGE COMMISSION and CAROLINA HOFILEÑA, respondents

SYLLABUS

1. ADMINISTRATIVE LAW; ADMINISTRATIVE AGENCY; SEC; NATURE AND PRINCIPAL FUNCTION. — The jurisdiction
of the SEC is delineated by Section 5 of PD No. 902-A. This grant of jurisdiction must be viewed in the light of the nature
and function of the SEC under the law. Section 3 of PD No. 902-A confers upon the latter "absolute jurisdiction, supervision,
and control over all corporations, partnerships or associations, who are grantees of primary franchise and/or license or
permit issued by the government to operate in the Philippines . . ." The principal function of the SEC is the supervision and
control over corporations, partnerships and associations with the end in view that investment in these entities may be
encouraged and protected, and their activities pursued for the promotion of economic development.
2. ID.; ID.; ID.; JURISDICTION OVER A CASE; WHEN COGNIZABLE. — In order that the SEC can take cognizance of a
case, the controversy must pertain to any of the following relationships: (a) between the corporation, partnership or
association and the public; (b)between the corporation, partnership or association and its stockholders, partners, members,
or officers; (c) between the corporation, partnership or association and the state in so far as its franchise, permit or license
to operate is concerned; and (d) among the stockholders, partners or associates themselves.
3. ID.; ID.; JURISDICTION OVER A CASE; WHERE ISSUES INVOLVED LACKED INTRA-CORPORATE RELATIONSHIP,
COGNIZABLE BY R.T.C. — The fact that the controversy at bar involves the rights of petitioner Union Glass who has no
intra-corporate relation either with complainant or the DBP, places the suit beyond the jurisdiction of the respondent SEC.
The case should be tried and decided by the court of general jurisdiction, the Regional Trial Court. This view is in accord
with the rudimentary principle that administrative agencies, like the SEC, are tribunals of limited jurisdiction and, as such,
could wield only such powers as are specifically granted to them by their enabling statutes.
4. REMEDIAL LAW; CIVIL PROCEDURE; SUPPLETORY APPLICATION OF THE RULES OF COURT IN PROCEEDINGS
BEFORE SEC SUBJECT TO RULES REGARDING JURISDICTION, VENUE AND JOINDER OF PARTIES. — Petitioner
Union Glass is involved only in the first cause of action of Hofileña's complaint in SEC Case No. 2035. While the Rules of
Court, which applies suppletorily to proceedings before the SEC, allows the joinder of causes of action in one complaint,
such procedure however is subject to the rules regarding jurisdiction, venue and joinder of parties. Since petitioner has no
intra-corporate relationship with the complainant, it cannot be joined as party-defendant in said case as to do so would
violate the rule on jurisdiction.
5. ID.; ID.; PREJUDICIAL QUESTION; CASE AT BAR. — Hofileña's complaint against petitioner for cancellation of the sale
of the glass plant should therefore be brought separately before the regular court. But such action, if instituted, shall be
suspended to await the final outcome of SEC Case No. 2035, for the issue of the validity of the dacion en pago posed in the
last mentioned case is a prejudicial question, the resolution of which is a logical antecedent of the issue involved in the
action against petitioner Union Glass. Thus, Hofileña's complaint against the latter can only prosper if final judgment is
rendered in SEC Case No. 2035, annulling the dacion en pago executed in favor of the DBP.
TEEHANKEE, J., concurring:
1. REMEDIAL LAW; CIVIL PROCEDURE; JOINDER OF PARTIES; FOR LACK OF JURISDICTION, DISALLOWED. —
Justice Teehankee concurs in the Court's judgment setting aside the questioned orders of respondent SEC and ordering
that petitioner Union Glass be dropped from SEC Case No. 2035 for lack of SEC jurisdiction over it as a third party
purchaser of the glass plant acquired by the DBP by dacion en pago from Pioneer Glass, without prejudice to Hofileña filing
a separate suit in the regular courts of justice against Union Glass for recovery and cancellation of the said sale of the glass
plant in favor of Union Glass.
2. ID.; ACTION; VALIDITY OF THE "DACION EN PAGO" IN THE CASE AT BAR; A PREJUDICIAL QUESTION. — He
concurs also with the statement in the Court's opinion that the final outcome of SEC Case No. 2035 with regard to the
validity of the dacion en pago is a prejudicial case. If Hofileña's complaint against said dacion en pago fails in the SEC, then
it clearly has no cause of action against Union Glass for cancellation of DBP's sale of the plant to Union Glass.
3. ID.; ID.; FAVORABLE JUDGMENT SECURED FROM SEC NOT CERTAIN TO PROSPER IF BROUGHT BEFORE
REGULAR COURTS OF JUSTICE; CASE AT BAR. — A favorable judgment secured by Hofileña in SEC Case No. 2035
against the DBP and Pioneer Glass would not necessarily mean that its action against Union Glass in the regular courts of
justice for recovery and cancellation of the DBP sale of the glass plant to Union Glass would necessarily prosper. It must be
borne in mind that the SEC has no jurisdiction over Union Glass as an outsider. The suit in the regular courts of justice that
Hofileña might bring against Union Glass is of course subject to all defenses as to the validity of the sale of the glass plant
in its favor as a buyer in good faith and should it successfully substantiate such defenses, then Hofileña's action against it
for cancellation of the sale might fail as a consequence.
AQUINO, J., dissenting:
1. REMEDIAL LAW; ACTION; LACHES AND NON-EXHAUSTION OF REMEDY; PRESENT IN THE CASE AT BAR. —
Although a jurisdictional issue is raised and jurisdiction over the subject matter may be raised at any stage of the case,
nevertheless, the petitioners are guilty of laches and non exhaustion of the remedy of appeal with the Securities and
Exchange Commission en banc.
2. ID.; ID.; REVIEW OF THE DECISION OF THE SEC; COGNIZABLE BY THE IAC. — Section 9 of the Judiciary
Reorganization Law returned to the Intermediate Appellate Court the exclusive jurisdiction to review the ruling, order or
decision of the SEC as a quasi-judicial agency. The same Section 9 granted to the Appellate Court jurisdiction in certiorari
and prohibition cases over the SEC although not exclusive. In this case, the SEC seems to have adopted the orders of the
two hearing officers as its own orders as shown by the stand taken by the Solicitor General in defending the SEC. If that
were so, that is, if the orders of the hearing officers should be treated as the orders of the SEC itself en banc, this Court
would have no jurisdiction over this case. It should be the Appellate Court that should exercise the power of review.
3. ID.; ID.; JOINDER OF PARTIES, PROPER; SEC NOT DIVESTED OF JURISDICTION. — There is no question that the
SEC has jurisdiction over the intra-corporate dispute between Hofileña and the DBP. both stockholders of Pioneer Glass,
over the dacion en pago. Certainly, the joinder of Union Glass does not divest the SEC of jurisdiction over the case. The
joinder of Union Glass is necessary because the DBP, its transferor, is being sued regarding the dacion en pago. The
defenses of Union Glass are tied up with the defenses of the DBP in the intra-corporate dispute. Hofileña's cause of action
should not be split. It would not be judicious and expedient to require Hofileña to sue the DBP and Union Glass in the
Regional Trial Court. The SEC is more competent than the said court to decide the intra- corporate dispute. The SEC, as
the agency enforcing Presidential Decree No. 902-A, is in the best position to know the extent of its jurisdiction. Its
determination that it has jurisdiction in this case has persuasive weight.

DECISION

ESCOLIN, J p:

This petition for certiorari and prohibition seeks to annul and set aside the Order of the Securities and Exchange
Commission, dated September 25, 1981, upholding its jurisdiction in SEC Case No. 2035, entitled "Carolina Hofileña,
Complainant, versus Development Bank of the Philippines, et al., Respondents."
Private respondent Carolina Hofileña, complainant in SEC Case No. 2035, is a stockholder of Pioneer Glass Manufacturing
Corporation, Pioneer Glass for short, a domestic corporation engaged in the operation of silica mines and the manufacture
of glass and glassware. Since 1967, Pioneer Glass had obtained various loan accommodations from the Development Bank
of the Philippines [DBP], and also from other local and foreign sources which DBP guaranteed.
As security for said loan accommodations, Pioneer Glass mortgaged and/or assigned its assets, real and personal, to the
DBP, in addition to the mortgages executed by some of its corporate officers over their personal assets. The proceeds of
said financial exposure of the DBP were used in the construction of a glass plant in Rosario, Cavite, and the operation of
seven silica mining claims owned by the corporation.
It appears that through the conversion into equity of the accumulated unpaid interests on the various loans amounting to
P5.4 million as of January 1975, and subsequently increased by another P2.2 million in 1976, the DBP was able to gain
control of the outstanding shares of common stocks of Pioneer Glass, and to get two, later three, regular seats in the
corporation's board of directors. cdrep
Sometime in March, 1978, when Pioneer Glass suffered serious liquidity problems such that it could no longer meet its
financial obligations with DBP, it entered into a dacion en pago agreement with the latter, whereby all its assets mortgaged
to DBP were ceded to the latter in full satisfaction of the corporation's obligations in the total amount of P59,000,000.00.
Part of the assets transferred to the DBP was the glass plant in Rosario, Cavite, which DBP leased and subsequently sold
to herein petitioner Union Glass and Container Corporation, hereinafter referred to as Union Glass.

On April 1, 1981, Carolina Hofileña filed a complaint before the respondent Securities and Exchange Commission against
the DBP, Union Glass and Pioneer Glass, docketed as SEC Case No. 2035. Of the five causes of action pleaded therein,
only the first cause of action concerned petitioner Union Glass as transferee and possessor of the glass plant. Said first
cause of action was based on the alleged illegality of the aforesaid dacion en pago resulting from: [1] the supposed
unilateral and unsupported undervaluation of the assets of Pioneer Glass covered by the agreement; [2] the self-dealing
indulged in by DBP, having acted both as stockholder/director and secured creditor of Pioneer Glass; and 13] the wrongful
inclusion by DBP in its statement of account of P26M as due from Pioneer Glass when the same had already been
converted into equity.
Thus, with respect to said first cause of action, respondent Hofileña prayed that the SEC issue an order:
"1. Holding that the so-called dacion en pago conveying all the assets of Pioneer Glass and the Hofileña
personal properties to Union Glass be declared null and void on the ground that the said conveyance
was tainted with.
"A. Self-dealing on the part of DBP which was acting both as a controlling
stockholder/director and as secured creditor of the Pioneer Glass, all to its advantage and to
that of Union Glass, and to the gross prejudice of the Pioneer Glass;
"B. That the dacion en pago is void because there was gross undervaluation of the
assets included in the so-called dacion en pago by more than 100% to the prejudice of
Pioneer Glass and to the undue advantage of DBP and Union Glass:
"C. That the DBP unduly favored Union Glass over another buyer, San Miguel
Corporation, notwithstanding the clearly advantageous terms offered by the latter to the
prejudice of Pioneer Glass, its other creditors and so-called 'minority stockholders.'
"2. Holding that the assets of the Pioneer Glass taken over by DBP and part of which was delivered to
Union Glass particularly the glass plant to be returned accordingly.
"3. That the DBP be ordered to accept and recognize the appraisal conducted by the Asian Appraisal Inc.
in 1975 and again in 1978 of the asset of Pioneer Glass." 1
In her common prayer, Hofileña asked that DBP be sentenced to pay Pioneer Glass actual, consequential, moral and
exemplary damages, for its alleged illegal acts and gross bad faith; and for DBP and Union Glass to pay her a reasonable
amount as attorney's fees. 2
On April 21, 1981, Pioneer Glass filed its answer. On May 8, 1981, petitioners moved for dismissal of the case on the
ground that the SEC had no jurisdiction over the subject matter or nature of the suit. Respondent Hofileña filed her
opposition to said motion, to which herein petitioners filed a rejoinder.
On July 23, 1981, SEC Hearing Officer Eugenio E. Reyes, to whom the case was assigned, granted the motion to dismiss
for lack of jurisdiction. However, on September 25, 1981, upon motion for reconsideration filed by respondent Hofileña,
Hearing Officer Reyes reversed his original order by upholding the SEC's jurisdiction over the subject matter and over the
persons of petitioners. Unable to secure a reconsideration of the Order as well as to have the same reviewed by the
Commission En Banc, petitioners filed the instant petition for certiorari and prohibition to set aside the order of September
25, 1981, and to prevent respondent SEC from taking cognizance of SEC Case No. 2035. LLphil
The issue raised in the petition may be propounded thus: Is it the regular court or the SEC that has jurisdiction over the
case?
In upholding the SEC's jurisdiction over the case Hearing Officer Reyes rationalized his conclusion thus:
"As correctly pointed out by the complainant, the present action is in the form of a derivative suit instituted
by a stockholder for the benefit of the corporation, respondent Pioneer Glass and Manufacturing
Corporation, principally against another stockholder, respondent Development Bank of the Philippines,
for alleged illegal acts and gross bad faith which resulted in the dacion en pago arrangement now being
questioned by complainant. These alleged illegal acts and gross bad faith came about precisely by virtue
of respondent Development Bank of the Philippine's status as a stockholder of co-respondent Pioneer
Glass Manufacturing Corporation although its status as such stockholder, was gained as a result of its
being a creditor of the latter. The derivative nature of this instant action can also be gleaned from the
common prayer of the complainant which seeks for an order directing respondent Development Bank of
the Philippines to pay co-respondent Pioneer Glass Manufacturing Corporation damages for the alleged
illegal acts and gross bad faith as above-mentioned.
"As far as respondent Union Glass and Container Corporation is concerned, its inclusion as a party-
respondent by virtue of its being an indispensable party to the present action, it being in possession of
the assets subject of the dacion en pago and, therefore, situated in such a way that it will be affected by
any judgment thereon." 3
In the ordinary course of things, petitioner Union Glass, as transferee and possessor of the glass plant covered by the
dacion en pago agreement, should be joined as party-defendant under the general rule which requires the joinder of every
party who has an interest in or lien on the property subject matter of the dispute. 4 Such joinder of parties avoids multiplicity
of suits as well as ensures the convenient, speedy and orderly administration of justice.
But since petitioner Union Glass has no intra-corporate relation with either the complainant or the DBP, its joinder as party-
defendant in SEC Case No. 2035 brings the cause of action asserted against it outside the jurisdiction of the respondent
SEC.
The jurisdiction of the SEC is delineated by Section 5 of PD No. 902-A as follows:
"Sec. 5. In addition to the regulatory and adjudicative function of the Securities and Exchange
Commission over corporations, partnerships and other forms of associations registered with it as
expressly granted under existing laws and devices, it shall have original and exclusive jurisdiction to hear
and decide cases involving:
a] Devices and schemes employed by or any acts, of the board of directors, business associates, its
officers or partners, amounting to fraud and misrepresentation which may be detrimental to the interest of
the public and/or the stockholders, partners, members of associations or organizations registered with the
Commission;
b] Controversies arising out of intra-corporate or partnership relations, between and among stockholders,
members or associates; between any or all of them and the corporation, partnership, or association of
which they are stockholders, members or associates, respectively; and between such corporation,
partnership or association and the state insofar as it concerns their individual franchise or right to exist as
such entity;
c] Controversies in the election or appointments of directors, trustees, officers or managers of such
corporations, partnerships or associations."
This grant of jurisdiction must be viewed in the light of the nature and function of the SEC under the law. Section 3 of PD
No. 902-A confers upon the latter "absolute jurisdiction, supervision, and control over all corporations, partnerships or
associations, who are grantees of primary franchise and/or license or permit issued by the government to operate in the
Philippines . . .'" The principal function of the SEC is the supervision and control over corporations, partnerships and
associations with the end in view that investment in these entities may be encouraged and protected, and their activities
pursued for the promotion of economic development. 5
It is in aid of this office that the adjudicative power of the SEC must be exercised. Thus the law explicitly specified and
delimited its jurisdiction to matters intrinsically connected with the regulation of corporations, partnerships and associations
and those dealing with the internal affairs of such corporations, partnerships or associations. llcd
Otherwise stated, in order that the SEC can take cognizance of a case, the controversy must pertain to any of the following
relationships: [a] between the corporation, partnership or association and the public; [b] between the corporation,
partnership or association and its stockholders, partners, members, or officers; [c] between the corporation, partnership or
association and the state in so far as its franchise, permit or license to operate is concerned; and [d] among the
stockholders, partners or associates themselves.
The fact that the controversy at bar involves the rights of petitioner Union Glass who has no intra-corporate relation either
with complainant or the DBP, places the suit beyond the jurisdiction of the respondent SEC. The case should be tried and
decided by the court of general jurisdiction, the Regional Trial Court. This view is in accord with the rudimentary principle
that administrative agencies, like the SEC, are tribunals of limited jurisdiction 6 and, as such, could wield only such powers
as are specifically granted to them by their enabling statutes. 7 As We held in Sunset View Condominium Corp. vs.
Campos, Jr.: 8
"Inasmuch as the private respondents are not shareholders of the petitioner condominium corporation,
the instant cases for collection cannot be a 'controversy arising out of intra-corporate or partnership
relations between and among stockholders, members or associates; between any or all of them and the
corporation, partnership or association of which they are stockholders, members or associates,
respectively,' which controversies are under the original and exclusive jurisdiction of the Securities &
Exchange Commission, pursuant to Section 5 [b] of PD. No. 902-A. . . ."

As heretofore pointed out, petitioner Union Glass is involved only in the first cause of action of Hofileña's complaint in SEC
Case No. 2035. While the Rules of Court, which applies suppletorily to proceedings before the SEC, allows the joinder of
causes of action in one complaint, such procedure however is subject to the rules regarding jurisdiction, venue and joinder
of parties. 9 Since petitioner has no intra-corporate relationship with the complainant, it cannot be joined as party-defendant
in said case as to do so would violate the rule or jurisdiction. Hofileña's complaint against petitioner for cancellation of the
sale of the glass plant should therefore be brought separately before the regular court. But such action, if instituted, shall be
suspended to await the final outcome of SEC Case No. 2035, for the issue of the validity of the dacion en pago posed in the
last mentioned case is a prejudicial question, the resolution of which is a logical antecedent of the issue involved in the
action against petitioner Union Glass. Thus, Hofileña's complaint against the latter can only prosper if final judgment is
rendered in SEC Case No. 2035, annulling the dacion en pago executed in favor of the DBP. LexLib
WHEREFORE, the instant petition is hereby granted, and the questioned Orders of respondent SEC, dated September 25,
1981, March 25, 1982 and May 28, 1982, are hereby set aside. Respondent Commission is ordered to drop petitioner Union
Glass from SEC Case No. 2035, without prejudice to the filing of a separate suit before the regular court of justice. No
pronouncement as to costs. SO ORDERED.
||
| (Union Glass & Container Corp. v. Securities and Exchange Commission, G.R. No. L-64013, [November 28, 1983], 211
PHIL 222-236)

[G.R. No. L-63558. May 19, 1987.]

SPOUSES JOSE ABEJO AND AURORA ABEJO, TELECTRONIC SYSTEMS, INC., petitioners, vs.
HON. RAFAEL DE LA CRUZ, JUDGE OF THE REGIONAL TRIAL COURT (NATIONAL CAPITAL
JUDICIAL REGION, BRANCH CLX-PASIG), SPOUSES AGAPITO BRAGA AND VIRGINIA BRAGA,
VIRGILIO BRAGA AND NORBERTO BRAGA, respondents.

[G.R. Nos. L-68450-51. May 19, 1987.]

POCKET BELL PHILIPPINES, INC., AGAPITO T. BRAGA, VIRGILIO T. BRAGA, NORBERTO


BRAGA, and VIRGINIA BRAGA, petitioners, vs. THE HONORABLE SECURITIES AND EXCHANGE
COMMISSION, TELECTRONIC SYSTEMS, INC., JOSE ABEJO, JOSE LUIS SANTIAGO, SIMEON A.
MIRAVITE, SR., ANDRES T. VELARDE AND L. QUIDATO BANDOLINO, respondents.

DECISION

TEEHANKEE, C.J p:

These two cases, jointly heard, are jointly herein decided. They involve the question of who, between the
Regional Trial Court and the Securities and Exchange Commission (SEC), has original and exclusive jurisdiction over
the dispute between the principal stockholders of the corporation Pocket Bell Philippines, Inc. (Pocket Bell), a "tone and
voice paging corporation," namely, the spouses Jose Abejo and Aurora Abejo (hereinafter referred to as the Abejos)
and the purchaser, Telectronic Systems, Inc. (hereinafter referred to as Telectronics) of their 133,000 minority
shareholdings (for P5 million) and of 63,000 shares registered in the name of Virginia Braga and covered by five stock
certificates endorsed in blank by her (for P1,674,450.00), and the spouses Agapito Braga and Virginia Braga
(hereinafter referred to as the Bragas), erstwhile majority stockholders. With the said purchases, Telectronics would
become the majority stockholder, holding 56% of the outstanding stock and voting power of the corporation Pocket Bell.
With the said purchases in 1982, Telectronics requested the corporate secretary of the corporation, Norberto
Braga, to register and transfer to its name, and those of its nominees the total 196,000 Pocket Bell shares in the
corporation's transfer book, cancel the surrendered certificates of stock and issue the corresponding new certificates of
stock in its name and those of its nominees.
Norberto Braga, the corporate secretary and son of the Bragas, refused to register the aforesaid transfer of
shares in the corporate books, asserting that the Bragas claim pre-emptive rights over the 133,000 Abejo shares and
that Virginia Braga never transferred her 63,000 shares to Telectronics but had lost the five stock certificates
representing those shares.
This triggered off the series of intertwined actions between the protagonists, all centered on the question of
jurisdiction over the dispute, which were to culminate in the filing of the two cases at bar.
The Bragas assert that the regular civil court has original and exclusive jurisdiction as against the Securities
and Exchange Commission, while the Abejos claim the contrary. A summary of the actions resorted to by the parties
follows:
A. ABEJOS' ACTIONS IN SEC
1. The Abejos and Telectronics and the latter's nominees, as new majority shareholders, filed SEC Cases Nos.
02379 and 02395 against the Bragas on December 17, 1982 and February 14, 1983, respectively.
2. In SEC Case No. 02379, they prayed for mandamus from the SEC ordering Norberto Braga, as corporate
secretary of Pocket Bell to register in their names the transfer and sale of the aforesaid 196,000 Pocket Bell shares (of
the Abejos 1 and Virginia Braga 2 , cancel the surrendered certificates as duly endorsed and to issue new certificates in
their names.
3. In SEC Case No. 02395, they prayed for injunction and a temporary restraining order that the SEC enjoin the
Bragas from disbursing or disposing funds and assets of Pocket Bell and from performing such other acts pertaining to
the functions of corporate officers.
4. Pocket Bell's corporate secretary, Norberto Braga, filed a Motion to Dismiss the mandamus case (SEC Case
No. 02379) contending that the SEC has no jurisdiction over the nature of the action since it does not involve an
intracorporate controversy between stockholders, the principal petitioners therein, Telectronics, not being a stockholder
of record of Pocket Bell.
5. On January 8, 1983, SEC Hearing Officer Joaquin Garaygay denied the motion. On January 14, 1983, the
corporate secretary filed a Motion for Reconsideration. On March 21, 1983, SEC Hearing Officer Joaquin Garaygay
issued an order granting Braga's motion for reconsideration and dismissed SEC Case No. 02379.
6. On February 11, 1983, the Bragas filed their Motion to Dismiss the injunction case, SEC Case No. 02395. On
April 8, 1985, the SEC Director, Eugenio Reyes, acting upon the Abejos' ex-parte motion, created a three-man
committee composed of Atty. Emmanuel Sison as Chairman and Attys. Alfredo Oca and Joaquin Garaygay as
members, to hear and decide the two SEC cases (Nos. 02379 and 02395).
7. On April 13, 1983, the SEC three-man committee issued an order reconsidering the aforesaid order of March
21, 1983 of the SEC Hearing Officer Garaygay (dismissing the mandamus petition SEC Case No. 02379) and directing
corporate secretary Norberto Braga to file his answer to the petitioner therein.
B. BRAGAS' ACTION IN SEC
8. On December 12, 1983, the Bragas filed a petition for certiorari, prohibition and mandamus with the SEC en
banc, SEC Case No. EB #049, seeking the dismissal of SEC Cases Nos. 02379 and 02395 for lack of jurisdiction of the
Commission and the setting aside of the various orders issued by the SEC three-man committee in the course of the
proceedings in the two SEC cases.
9. On May 15, 1984, the SEC en banc issued an order dismissing the Bragas' petition in SEC Case No. EB
#049 for lack of merit and at the same time ordering the SEC Hearing Committee to continue with the hearings of the
Abejos and Telectronics SEC Cases Nos. 02379 and 02395, ruling that the "issue is not the ownership of shares but
rather the non-performance by the Corporate Secretary of the ministerial duty of recording transfers of shares of stock
of the corporation of which he is secretary."
10. On May 15, 1984 the Bragas filed a motion for reconsideration but the SEC en banc denied the same on
August 9, 1984.
C. BRAGAS' ACTION IN CFI (NOW RTC)
11. On November 25, 1982, following the corporate secretary's refusal to register the transfer of the shares in
question, the Bragas filed a complaint against the Abejos and Telectronics in the Court of First Instance of Pasig,
Branch 21 (now the Regional Trial Court, Branch 160) docketed as Civil Case No. 48746 for: (a) rescission and
annulment of the sale of the shares of stock in Pocket Bell made by the Abejos in favor of Telectronics on the ground
that it violated the Bragas' alleged pre-emptive right over the Abejos' shareholdings and an alleged perfected contract
with the Abejos to sell the same shares in their (Bragas) favor, (1st cause of action); plus damages for bad faith; and (b)
declaration of nullity of any transfer, assignment or endorsement of Virginia Bragas' stock certificates for 63,000 shares
in Pocket Bell to Telectronics for want of consent and consideration, alleging that said stock certificates, which were
intended as security for a loan application and were thus endorsed by her in blank, had been lost (2nd cause of action).
12. On January 4, 1983, the Abejos filed a Motion to Dismiss the complaint on the ground that it is the SEC that
is vested under PD 902-A with original and exclusive jurisdiction to hear and decide cases involving, among others,
controversies "between and among stockholders" and that the Bragas' suit is such a controversy as the issues involved
therein are the stockholders" alleged pre-emptive rights, the validity of the transfer and endorsement of certificates of
stock, the election of corporate officers and the management and control of the corporation's operations. The dismissal
motion was granted by Presiding Judge G. Pineda on January 14, 1983.
13. On January 24, 1983, the Bragas filed a motion for reconsideration. The Abejos opposed. Meanwhile,
respondent Judge Rafael de la Cruz was appointed presiding judge of the court (renamed Regional Trial Court) in place
of Judge G. Pineda.
14. On February 14, 1983, respondent Judge de la Cruz issued an order rescinding the January 14, 1983 order
and reviving the temporary restraining order previously issued on December 23, 1982 restraining Telectronics' agents
or representatives from enforcing their resolution constituting themselves as the new set of officers of Pocket Bell and
from assuming control of the corporation and discharging their functions.
15. On March 2, 1983, the Abejos filed a motion for reconsideration, which motion was duly opposed by the
Bragas. On March 11, 1983, respondent Judge denied the motion for reconsideration.
D. ABEJOS' PETITION AT BAR
16. On March 26, 1983, the Abejos, alleging that the acts of respondent Judge in refusing to dismiss the
complaint despite clear lack of jurisdiction over the action and in refusing to reconsider his erroneous position were
performed without jurisdiction and with grave abuse of discretion, filed their herein Petition for Certiorari and Prohibition
with Preliminary Injunction. They prayed that the challenged orders of respondent Judge dated February 14, 1983 and
March 11, 1983 be set aside for lack of jurisdiction and that he be ordered to permanently desist from further
proceedings in Civil Case No. 48746. Respondent judge desisted from further proceedings in the case, dispensing with
the need of issuing any restraining order.
E. BRAGAS' PETITION AT BAR
17. On August 29, 1984, the Bragas, alleging in turn that the SEC has no jurisdiction over SEC Cases Nos.
02379 and 02395 and that it acted arbitrarily, whimsically and capriciously in dismissing their petition (in SEC Case No.
EB #049) for dismissal of the said cases, filed their herein Petition for Certiorari and Prohibition with Preliminary
Injunction or TRO. The petitioner seeks the reversal and/or setting aside of the SEC Order dated May 15, 1984
dismissing their petition in said SEC Case No. EB #049 and sustaining its jurisdiction over SEC Cases Nos. 02379 and
02395, filed by the Abejos. On September 24, 1984, this Court issued a temporary restraining order to maintain the
status quo and restrained the SEC and/or any of its officers or hearing committees from further proceeding with the
hearings in SEC Cases Nos. 02379 and 02395 and from enforcing any and all orders and or resolutions issued in
connection with the said cases.
The cases, having been given due course, were jointly heard by the Court on March 27, 1985 and the parties
thereafter filed on April 16, 1985 their respective memoranda in amplification of oral argument on the points of law that
were crystallized during the hearing.
The Court rules that the SEC has original and exclusive jurisdiction over the dispute between the principal
stockholders of the corporation Pocket Bell, namely, the Abejos and Telectronics, the purchasers of the 56% majority
stock (supra, at page 2) on the one hand, and the Bragas, erstwhile majority stockholders, on the other, and that the
SEC, through its en banc Resolution of May 15, 1984 correctly ruled in dismissing the Bragas' petition questioning its
jurisdiction, that "the issue is not the ownership of shares but rather the non-performance by the Corporate Secretary of
the ministerial duty of recording transfers of shares of stock of the Corporation of which he is secretary."
1. The SEC ruling upholding its primary and exclusive jurisdiction over the dispute is correctly premised on, and
fully supported by, the applicable provisions of P.D. No. 902-A which reorganized the SEC with additional powers "in
line with the government's policy of encouraging investments, both domestic and foreign, and more active public
participation in the affairs of private corporations and enterprises through which desirable activities may be pursued for
the promotion of economic development; and, to promote a wider and more meaningful equitable distribution of wealth,"
and accordingly provided that:
"SEC. 3. The Commission shall have absolute jurisdiction, supervision and control over all corporations,
partnerships or associations, who are the grantees of primary franchise and/or a license or permit issued
by the government to operate in the Philippines; . . .
"SEC. 5. In addition to the regulatory and adjudicative functions of the Securities and Exchange
Commission over corporations, partnerships and other forms of associations registered with it as
expressly granted under existing laws and decrees, it shall have original and exclusive jurisdiction to hear
and decide cases involving:
a) Devices or schemes employed by or any acts, of the board of directors, business
associations, its officers or partners, amounting to fraud and misrepresentation which may be
detrimental to the interest of the public and/or of the stockholder, partners, members of
associations or organizations registered with the Commission.
b) Controversies arising out of intracorporate or partnership relations, between and
among stockholders, members, or associates; between any and/or all of them and the
corporation, partnership or association of which they are stockholders, members or associates,
respectively; and between such corporation, partnership or association and the state insofar as it
concerns their individual franchise or right to exist as such entity;
c) Controversies in the election or appointments of directors, trustees, officers or
managers of such corporations, partnerships or associations." 3
Section 6 further grants the SEC "in order to effectively exercise such jurisdiction," the power, inter alia, "to
issue preliminary or permanent injunctions, whether prohibitory or mandatory, in all cases in which it has jurisdiction,
and in which cases the pertinent provisions of the Rules of Court shall apply."
2. Basically and indubitably, the dispute at bar, as held by the SEC, is an intracorporate dispute that has arisen
between and among the principal stockholders of the corporation Pocket Bell due to the refusal of the corporate
secretary, backed up by his parents as erstwhile majority shareholders, to perform his "ministerial duty" to record the
transfers of the corporation's controlling (56%) shares of stock, covered by duly endorsed certificates of stock, in favor
of Telectronics as the purchaser thereof. Mandamus in the SEC to compel the corporate secretary to register the
transfers and issue new certificates in favor of Telectronics and its nominees was properly resorted to under Rule XXI,
Section 1 of the SEC's New Rules of Procedure, 4 which provides for the filing of such petitions with the SEC. Section 3
of said Rules further authorizes the SEC to "issue orders expediting the proceedings . . . and also [to] grant a
preliminary injunction for the preservation of the rights of the parties pending such proceedings."
The claims of the Bragas, which they assert in their complaint in the Regional Trial Court, praying for rescission
and annulment of the sale made by the Abejos in favor of Telectronics on the ground that they had an alleged perfected
pre-emptive right over the Abejos' shares as well as for annulment of sale to Telectronics of Virginia Braga's shares
covered by street certificates duly endorsed by her in blank, may in no way deprive the SEC of its primary and exclusive
jurisdiction to grant or not the writ of mandamus ordering the registration of the shares so transferred. The Bragas'
contention that the question of ordering the recording of the transfers ultimately hinges on the question of ownership or
right thereto over the shares notwithstanding, the jurisdiction over the dispute is clearly vested in the SEC.
3. The very complaint of the Bragas for annulment of the sales and transfers as filed by them in the regular
court questions the validity of the transfer and endorsement of the certificates of stock, claiming alleged pre-emptive
rights in the case of the Abejos' shares and alleged loss of the certificates and lack of consent and consideration in the
case of Virginia Braga's shares. Such dispute clearly involves controversies "between and among stockholders," as to
the Abejos' right to sell and dispose of their shares to Telectronics, the validity of the latter's acquisition of Virginia
Braga's shares, who between the Bragas and the Abejos' transferee should be recognized as the controlling
shareholders of the corporation, with the right to elect the corporate officers and the management and control of its
operations. Such a dispute and case clearly fall within the original and exclusive jurisdiction of the SEC to decide, under
Section 5 of P.D. 902-A, above-quoted. The restraining order issued by the Regional Trial Court restraining Telectronics
agents and representatives from enforcing their resolution constituting themselves as the new set of officers of Pocket
Bell and from assuming control of the corporation and discharging their functions patently encroached upon the SEC's
exclusive jurisdiction over such specialized corporate controversies calling for its special competence. As stressed by
the Solicitor General on behalf of the SEC, the Court has held that "Nowhere does the law [PD 902-A] empower any
Court of First Instance [now Regional Trial Court] to interfere with the orders of the Commission," 5 and consequently
"any ruling by the trial court on the issue of ownership of the shares of stock is not binding on the Commission" 6 for
want of jurisdiction.
4. The dispute therefore clearly falls within the general classification of cases within the SEC's original and
exclusive jurisdiction to hear and decide, under the aforequoted governing section 5 of the law. Insofar as the Bragas
and their corporate secretary's refusal on behalf of the corporation Pocket Bell to record the transfer of the 56% majority
shares to Telectronics may be deemed a device or scheme amounting to fraud and misrepresentation employed by
them to keep themselves in control of the corporation to the detriment of Telectronics (as buyer and substantial investor
in the corporate stock) and the Abejos (as substantial stockholders-sellers), the case falls under paragraph (a). The
dispute is likewise an intra-corporate controversy between and among the majority and minority stockholders as to the
transfer and disposition of the controlling shares of the corporation, falling under paragraph (b). As stressed by the
Court in DMRC Enterprises v. Este del Sol Mountain Reserve, Inc., 7 "Considering the announced policy of PD 902-A,
the expanded jurisdiction of the respondent Securities and Exchange Commission under said decree extends
exclusively to matters arising from contracts involving investments in private corporations, partnerships and
associations." The dispute also concerns the fundamental issue of whether the Bragas or Telectronics have the right to
elect the corporate directors and officers and manage its business and operations, which falls under paragraph (c).
5. Most of the cases that have come to this Court involve those under paragraph (b), i e. whether the
controversy is an intra-corporate one, arising "between and among stockholders" or "between any or all of them and the
corporation." The parties have focused their arguments on this question. The Bragas' contention in his field must
likewise fail. In Philex Mining Corp. v. Reyes, 8 the Court spelled out that "an intra-corporate controversy is one which
arises between a stockholder and the corporation. There is no distinction, qualification, nor any exemption whatsoever.
The provision is broad and covers all kinds of controversies between stockholders and corporations. The issue of
whether or not a corporation is bound to replace a stockholder's lost certificate of stock is a matter purely between a
stockholder and the corporation. It is a typical intra-corporate dispute. The question of damages raised is merely
incidental to that main issue." The Court rejected the stockholders' theory of excluding his complaint (for replacement of
a lost stock [dividend] certificate which he claimed to have never received) from the classification of intra-corporate
controversies as one that "does not square with the intent of the law, which is to segregate from the general jurisdiction
of regular Courts controversies involving corporations and their stockholders and to bring them to the SEC for exclusive
resolution, in much the same way that labor disputes are now brought to the Ministry of Labor and Employment (MOLE)
and the National Labor Relations Commission (NLRC), and not to the Courts."
(a) The Bragas contend that Telectronics, as buyer-transferee of the 56% majority shares is not a
registered stockholder, because they, through their son the corporate secretary, appear to have refused
to perform "the ministerial duty of recording transfers of shares of stock of the corporation of which he is
the secretary," and that the dispute is therefore, not an intracorporate one. This contention begs the
question which must properly be resolved by the SEC, but which they would prevent by their own act,
through their son, of blocking the due recording of the transfer and cannot be sanctioned. It can be seen
from their very complaint in the regular courts that they with their two sons constituting the plaintiffs are all
stockholders while the defendants are the Abejos who are also stockholders whose sale of the shares to
Telectronics they would annul.
(b) There can be no question that the dispute between the Abejos and the Bragas as to the sale and
transfer of the former's shares to Telectronics for P5 million is an intracorporate one under section 5 (b),
prescinding from the applicability of section 5 (a) and (c), (supra, par. 4) It is the SEC which must resolve
the Bragas' claim in their own complaint in the court case filed by them of an alleged pre-emptive right to
buy the Abejos' shares by virtue of "on-going negotiations," which they may submit as their defense to
the mandamus petition to register the sale of the shares to Telectronics. But asserting such pre-emptive
rights and asking that the same be enforced is a far cry from the Bragas' claim that "the case relates to
questions of ownership" over the shares in question. 9 (Not to mention, as pointed out by the Abejos, that
the corporation is not a close corporation, and no restriction over the free transferability of the shares
appears in the Articles of Incorporation, as well as in the by-laws 10 and the certificates of stock
themselves, as required by law for the enforcement of such restriction. See Go Soc & Sons, etc. v. IAC,
G.R. No. 72342, Resolution of February 19, 1987.)
(c) The dispute between the Bragas and Telectronics as to the sale and transfer for P1,674,450.00 of
Virginia Braga's 63.000 shares covered by Street certificates duly endorsed in blank by her is within the
special competence and jurisdiction of the SEC, dealing as it does with the free transferability of
corporate shares, particularly street certificates, 11 as guaranteed by the Corporation Code and its
proclaimed policy of encouraging foreign and domestic investments in Philippine private corporations and
more active public participation therein for the promotion of economic development. Here again, Virginia
Braga's claim of loss of her street certificates or theft thereof (denounced by Telectronics as "perjurious"
12 ) must be pleaded by her as a defense against Telectronics' petition for mandamus and recognition
now as the controlling stockholder of the corporation in the light of the joint affidavit of General Cerefino
S. Carreon of the National Telecommunications Commission and private respondent Jose Luis Santiago
of Telectronics narrating the facts and circumstances of how the former sold and delivered to Telectronics
on behalf of his compadres, the Bragas, Virginia Braga's street certificates for 63,000 shares equivalent
to 18% of the corporation's outstanding stock and received the cash price thereof. 13 But as to the sale
and transfer of the Abejos' shares, the Bragas cannot oust the SEC of its original and exclusive
jurisdiction to hear and decide the case, by blocking through the corporate secretary, their son, the due
recording of the transfer and sale of the shares in question and claiming that Telectronics is not a
stockholder of the corporation — which is the very issue that the SEC is called upon to resolve. As the
SEC maintains, "There is no requirement that a stockholder of a corporation must be a registered one in
order that the Securities and Exchange Commission may take cognizance of a suit seeking to enforce his
rights as such stockholder." 14 This is because the SEC by express mandate has "absolute jurisdiction,
supervision and control over all corporations" and is called upon to enforce the provisions of the
Corporation Code, among which is the stock purchaser's right to secure the corresponding certificate in
his name under the provisions of Section 63 of the Code. Needless to say, any problem encountered in
securing the certificates of stock representing the investment made by the buyer must be expeditiously
dealt with through administrative mandamus proceedings with the SEC, rather than through the usual
tedious regular court procedure. Furthermore, as stated in the SEC order of April 13, 1983, notice given
to the corporation of the sale of the shares and presentation of the certificates for transfer is equivalent to
registration: "Whether the refusal of the (corporation) to effect the same is valid or not is still subject to
the outcome of the hearing on the merits of the case." 15
6. In the fifties, the Court taking cognizance of the move to vest jurisdiction in administrative commissions and
boards the power to resolve specialized disputes in the field of labor (as in corporations, public transportation and public
utilities) ruled that Congress in requiring the Industrial Court's intervention in the resolution of labor-management
controversies likely to cause strikes or lockouts meant such jurisdiction to be exclusive, although it did not so expressly
state in the law. The Court held that under the "sense-making and expeditious doctrine of primary jurisdiction ..the
courts cannot or will not determine a controversy involving a question which is within the jurisdiction of an administrative
tribunal, where the question demands the exercise of sound administrative discretion requiring the special knowledge,
experience, and services of the administrative tribunal to determine technical and intricate matters of fact, and a
uniformity of ruling is essential to comply with the purposes of the regulatory statute administered." 16
In this era of clogged court dockets, the need for specialized administrative boards or commissions with the
special knowledge, experience and capability to hear and determine promptly disputes on technical matters or
essentially factual matters, subject to judicial review in case of grave abuse of discretion, has become well nigh
indispensable. Thus, in 1984, the Court noted that "between the power lodged in an administrative body and a court,
the unmistakable trend has been to refer it to the former. 'Increasingly, this Court has been committed to the view that
unless the law speaks clearly and unequivocably, the choice should fall on [an administrative agency.]'" 17 The Court in
the earlier case of Ebon vs. De Guzman, 18 noted that the lawmaking authority, in restoring to the labor arbiters and the
NLRC their jurisdiction to award all kinds of damages in labor cases, as against the previous P.D. amendment splitting
their jurisdiction with the regular courts, "evidently, . . . had second thoughts about depriving the Labor Arbiters and the
NLRC of the jurisdiction to award damages in labor cases because that setup would mean duplicity of suits, splitting the
cause of action and possible conflicting findings and conclusions by two tribunals on one and the same claim."
7. Thus, the Corporation Code (B.P. No. 178) enacted on May 1, 1980 specifically vests the SEC with the Rule
making power in the discharge of its task of implementing the provisions of the Code and particularly charges it with the
duty of preventing fraud and abuses on the part of controlling stockholders, directors and officers, as follows:
"SEC. 143. Rule-making power of the Securities and Exchange Commission. — The Securities and
Exchange Commission shall have the power and authority to implement the provisions of this Code, and
to promulgate rules and regulations reasonably necessary to enable it to perform its duties hereunder,
particularly in the prevention of fraud and abuses on the part of the controlling stockholders, members,
directors, trustees or officers." (Emphasis supplied)
The dispute between the contending parties for control of the corporation manifestly falls within the primary and
exclusive jurisdiction of the SEC in whom the law has reserved such jurisdiction as an administrative agency of special
competence to deal promptly and expeditiously therewith.
As the Court stressed in Union Glass & Container Corp. v. SEC, 19 "This grant of jurisdiction [in Section 5]
must be viewed in the light of the nature and functions of the SEC under the law. Section 3 of PD No. 902-A confers
upon the latter 'absolute jurisdiction, supervision, and control over all corporations, partnerships or associations, who
are grantees of primary franchise and/or license or permit issued by the government to operate in the Philippines . . ..'
The principal function of the SEC is the supervision and control over corporations, partnerships and associations with
the end in view that investment in these entities may be encouraged and protected, and their activities pursued for the
promotion of economic development.
"It is in aid of this office that the adjudicative power of the SEC must be exercised. Thus the law explicitly
specified and delimited its jurisdiction to matters intrinsically connected with the regulation of
corporations, partnerships and associations and those dealing with the internal affairs of such
corporations, partnerships or associations.
"Otherwise stated, in order that the SEC can take cognizance of a case, the controversy must pertain to
any of the following relationships: [a] between the corporation, partnership or association and the public;
[b] between the corporation, partnership or association and its stockholders, partners, members, or
officers; [c] between the corporation, partnership or association and the state in so far as its franchise,
permit or license to operate is concerned; and [d] among the stockholders, partners or associates
themselves." 20

Parenthetically, the cited case of Union Glass illustrates by way of contrast what disputes do not fall within the
special jurisdiction of the SEC. In this case, the SEC had properly assumed jurisdiction over the dissenting
stockholders' complaint against the corporation Pioneer Glass questioning its dacion en pago of its glass plant and all
its assets in favor of the DBP which was clearly an intra-corporate controversy dealing with its internal affairs. But the
Court held that the SEC had no jurisdiction over petitioner Union Glass Corp., impleaded as third party purchaser of the
plant from DBP in the action to annul the dacion en pago. The Court held that such action for recovery of the glass plant
could be brought by the dissenting stockholder to the regular courts only if and when the SEC rendered final judgment
annulling the dacion en pago and furthermore subject to Union Glass' defenses as a third party buyer in good faith.
Similarly, in the DMRC case, therein petitioner's complaint for collection of the amounts due to it as payment of rentals
for the lease of its heavy equipment in the form mainly of cash and part in shares of stock of the debtor-defendant
corporation was held to be not covered by the SEC's exclusive jurisdiction over intracorporate disputes, since "to pass
upon a money claim under a lease contract would be beyond the competence of the Securities and Exchange
Commission and to separate the claim for money from the claim for shares of stock would be splitting a single cause of
action resulting in a multiplicity of suits." 21 Such an action for collection of a debt does not involve enforcement of
rights and obligations under the Corporation Code nor the internal or intracorporate affairs of the debtor corporation. But
in all disputes affecting and dealing with the interests of the corporation and its stockholders, following the trend and
clear legislative intent of entrusting all disputes of a specialized nature to administrative agencies possessing the
requisite competence, special knowledge, experience and services and facilities to expeditiously resolve them and
determine the essential facts including technical and intricate matters, as in labor and public utilities rates disputes, the
SEC has been given "the original and exclusive jurisdiction to hear and decide" them (under Section 5 of P.D. 902-A) "in
addition to [its] regulatory and adjudicative functions" (under Section 3, vesting in it "absolute jurisdiction, supervision
and control over all corporations" and the Ruler-making power granted it in Section 143 of the Corporation Code,
supra). As stressed by the Court in the Philex case, supra, "(T)here is no distinction, qualification, nor any exemption
whatsoever. The provision is broad and covers all kinds of controversies between stockholders and corporations."
It only remains now to deal with the Order dated April 15, 1983 (Annex H, Petition) 22 of the SEC's three-
member Hearing Committee granting Telectronics' motion for creation of a receivership or management committee with
the ample powers therein enumerated for the preservation pendente lite of the corporation's assets and in discharge of
its "power and duty to preserve the rights of the parties, the stockholders, the public availing of the corporation's
services and the rights of creditors," as well as 'for reasons of equity and justice .. (and) to prevent possible paralization
of corporate business." The said Order has not been implemented notwithstanding its having been upheld per the SEC
en banc's Order of May 15, 1984 (Annex "V", Petition) dismissing for lack of merit the petition for certiorari, prohibition
and mandamus with prayer for restraining order or injunction filed by the Bragas seeking the disbandment of the
Hearing Committee and the setting aside of its Orders, and its Resolution of August 9, 1984, denying reconsideration
(Annex "X", Petition), due to the Bragas' filing of the petition at bar.
Prescinding from the great concern of damage and prejudice expressed by Telectronics due to the Bragas
having remained in control of the corporation and having allegedly committed acts of gross mismanagement and
misapplication of funds, the Court finds that under the facts and circumstances of record, it is but fair and just that the
SEC's order creating a receivership committee be implemented forthwith, in accordance with its terms, as follows:
"The three-man receivership committee shall be composed of a representative from the commission, in
the person of the Director, Examiners and Appraisers Department or his designated representative, and a
representative from the petitioners and a representative of the respondent.
"The petitioners and respondent are therefore directed to submit to the Commission the name of their
designated representative within three (3) days from receipt of this order. The Commission shall appoint
the other representatives if either or both parties fail to comply with the requirement within the stated
time."
ACCORDINGLY, judgment is hereby rendered:
(a) Granting the petition in G.R. No. 63558, annulling the challenged Orders of respondent Judge dated
February 14, 1983 and March 11, 1983 (Annexes "L" and "P" of the Abejos' petition) and prohibiting
respondent Judge from further proceeding in Civil Case No. 48746 filed in his Court other than to dismiss
the same for lack or jurisdiction over the subject-matter;
(b) Dismissing the petition in G.R. Nos. 68450-51 and lifting the temporary restraining order issued on
September 24, 1984, effective immediately upon promulgation hereof;
(c) Directing the SEC through its Hearing Committee to proceed immediately with hearing and resolving
the pending mandamus petition for recording in the corporate books the transfer to Telectronics and its
nominees of the majority (56%) shares of stock of the corporation Pocket Bell pertaining to the Abejos
and Virginia Braga and all related issues, taking into consideration, without need of resubmittal to it, the
pleadings, annexes and exhibits filed by the contending parties in the cases at bar; and
(d) Likewise directing the SEC through its Hearing Committee to proceed immediately with the
implementation of its receivership or management committee Order of April 15, 1983 in SEC Case No.
2379 and for the purpose, the contending parties are ordered to submit to said Hearing Committee the
name of their designated representatives in the receivership/management committee within three (3)
days from receipt of this decision, on pain of forfeiture of such right in case of failure to comply herewith,
as provided in the said Order; and ordering the Bragas to perform only caretaker acts in the corporation
pending the organization of such receivership/management committee and assumption of its functions.
This decision shall be immediately executory upon its promulgation.
SO ORDERED.
||| (Spouses Abejo v. De la Cruz, G.R. No. L-63558, L-68450-51, [May 19, 1987], 233 PHIL 668-690)

[G.R. No. 87135. May 22, 1992.]

ALMA MAGALAD, petitioner, vs. PREMIERE FINANCING CORP., respondents.

SYLLABUS
1. COMMERCIAL LAW; SECURITIES AND EXCHANGE COMMISSION; HAS JURISDICTION OVER CASES INVOLVING
FRAUD COMMITTED BY CORPORATE OFFICIAL DETRIMENTAL TO THE INTEREST OF THE PUBLIC; CASE AT BAR.
— Considering that Magalad's complaint sufficiently alleges acts amounting to fraud and misrepresentation committed by
Premiere, the SEC must be held to retain its original and exclusive jurisdiction over the case, despite the fact that the suit
involves collection of sums of money paid to said corporation, the recovery of which would originally fall within the
jurisdiction of regular courts. The fraud committed is detrimental to the interest of the public and, therefore, encompasses a
category of relationship within the SEC jurisdiction. In this case, the recitals of the complaint sufficiently allege that devices
or schemes amounting to fraud and misrepresentation detrimental to the interest of the public have been resorted to by
Premiere Corporation. It can not but be conceded, therefore, that the SEC may exercise its adjudicative powers pursuant to
Sec. 5(a) of Pres. Decree No. 902-A (Supra).
2. ID.; ID.; CONTROVERSIES WITHIN THE EXCLUSIVE JURISDICTION THEREOF; RULE. — In order that the SEC can
take cognizance of a case, the controversy must pertain to any of the following relationships: (a) between the corporation,
partnership or association and the public; (b) between the corporation, partnership or association and its stockholders,
partners, members or officers; (c) between the corporation, partnership or association and the state so far as its franchise,
permit or license to operate is concerned; and (d) among the stockholders, partners or associates themselves (Union Glass
& Container Corp. v. SEC, 126 SCRA 31; 38; 1983; Abejo v. De la Cruz, 149 SCRA 654, 1987).
3. ID.; ID.; HAS JURISDICTION OVER CORPORATION UNDER THE MANAGEMENT OF A REHABILITATION
RECEIVER. — Bolstering the jurisdiction of the SEC in this case is the fact that said agency had already appointed a
Rehabilitation Receiver for Premiere and has directed all proceedings or claims against it be suspended. This, pursuant to
Sec. 6(c) of pres. Decree No. 902-A providing that "upon appointment of . . . rehabilitation receiver . . . all actions for claims
against corporations . . . under receivership pending before any court, tribunal, board or body shall be suspended
accordingly." By doing so, SEC has exercised its original and exclusive jurisdiction to hear and decide cases involving: "a)
petitions of corporations, partnerships or associations to be declared in the state of suspension of payments in cases where
the corporation, partnership or association possesses sufficient property to cover all its debts but foresees the impossibility
of meeting them when they respectively fall due or in cases where the corporation, partnership or association has no
sufficient assets to cover its liabilities but is under the management of a Rehabilitation Receiver or Management of a
Rehabilitation Receiver or Management Committee created pursuant to this Decree." (Section 5(d) of Pres. Decree No.
902-A as added by Pres. Decree 1758).

DECISION

PARAS, J p:

This is an appeal originally filed with the Court of Appeals but certified to this court for disposition since it involves purely
questions of law from the decision of the Regional Trial Court (RTC), Branch LXXXV, Quezon City, dated May 22, 1984, in
Civil Case No. Q-40392, ordering the defendant-appellant Premiere Financing Corporation (Premiere for short) to pay to the
plaintiff-appellee Alma Magalad (Magalad for short) the sum of: (a) P50,000.00, the principal obligation, plus interest at the
legal rate from September 12, 1983, until the full amount is paid; (b) P10,000.00, both for moral and exemplary damages;
(c) P5,000.00, for and as attorney's fees and (d) the costs of suit.
The antecedent facts of the case are as follows:
Premiere is a financing company engaged in soliciting and accepting money market placements or deposits (Original
Record, p. 29). LexLib
On September 12, 1983 with expired permit to issue commercial papers (Ibid., p. 8) and with intention not to pay or defraud
its creditors, Premiere induced and misled Magalad into making a money market placement of P50,000.00 at 22% interest
per annum for which it issued a receipt (Ibid., Exh. "B", p. 8). Aside from the receipt, Premier likewise issued two (2) post-
dated checks in the total sum of P51,079.00 (Ibid., Exh. "C". p. 9) and assigned to Magalad its receivable from a certain
David Saman for the same amount (Ibid., Exh. "C", p. 10).
When the said checks were presented for payment on their due dates, the drawee bank dishonored the checks for lack of
sufficient funds to cover the amount (Ibid., Exhs. "D-1", "E-1", pp. 11-12). Despite demands by Magalad for the replacement
of said checks with cash, Premiere, for no valid reason, failed and refused to honor such demands and due to fraudulent
acts of Premiere, Magalad suffered sleepless nights, mental anguish, fright, serious anxiety, considering the fact that the
money she invested is blood money and is the only source of support for her family (Ibid., p. 4).
Magalad in order to seek redress and retrieve her blood money, availed of the service of counsel for which she agreed to
pay twenty percent (20%) of the amount due as and for attorney's fees (Ibid.).
On January 10, 1984, Magalad filed a complaint for damages with prayer for writ of preliminary attachment with the RTC,
Branch LXXXV, Quezon City, docketed as Civil Case No. Q-40392 against herein Premiere (Ibid., pp. 3-6).
Premiere having failed to file an answer and acting on Magalad's motion, the lower court declared Premiere in default by
virtue of an order dated April 5, 1984 allowing Magalad to present evidence ex-parte (Ibid., pp. 21; 22). Cdpr
On May 22, 1984 the lower court rendered a default judgment against Premiere, the dispositive portion of which reads:
"From the foregoing evidence, the court finds that plaintiff has fully established her claim that defendant
had indeed acted fraudulently in incurring the obligation and considering that no evidence has been
adduced by the defendant to contradict the same, judgment is hereby rendered ordering the defendant to
pay plaintiff as follows:
"(a) P50,000.00, the principal obligation, plus interest at the legal rate from September
12, 1983 until the full amount is paid;"
"(b) P10,000.00 both for moral and exemplary damages;"
"(c) P5,000.00 for and as attorney's fees; and"
"(d) the costs of suit.
"SO ORDERED." (Ibid., p. 30)
Premiere filed a motion for reconsideration of the foregoing decision, based principally on a question of law alleging that the
Securities and Exchange Commission (SEC) has exclusive and original jurisdiction over a corporation under a state of
suspension of payments (Ibid., pp. 32-41).
Magalad filed an opposition to the motion for reconsideration on January 8, 1985 alleging among others that the regular
court has jurisdiction over the case to the exclusion of the SEC (Ibid., pp. 51-53).
On May 28, 1986 the lower court issued an order denying the motion for reconsideration (Ibid., p. 61).
On June 11, 1986 Premiere filed his notice of appeal which led to the issuance of the order of the lower court dated July 29,
1986 elevating the case to the Court of Appeals (CA) (Ibid., pp. 62; 63).
The Court of Appeals in its resolution dated September 8, 1987 dismissed the case for failure of Premiere to file its brief
despite the ninety-day extension granted to it, which expired on June 10, 1987 (Rollo, p. 16).
An omnibus motion for reconsideration and admission of late filing of Premiere's brief was filed on September 22, 1987
(Rollo, pp. 17-19; 32).
On September 30, 1987 the Court of Appeals issued a resolution which reconsidered its previous resolution dated
September 5, 1987 and admitted the Premiere's brief (Rollo, p. 26). prcd
On January 31, 1989 the Court of Appeals issued a resolution certifying the instant case to this Court on the ground that the
case involves a question of law, the dispositive part of which stating:
"ACCORDINGLY, pursuant to Rule 50, Sec. 3, in relation to the Judiciary Act of 1948, Sec. 17, par. 4(3)
(4), the appeal in this case is hereby certified to the Supreme Court on the ground that the only issue
raised concerns the jurisdiction of the trial court and only a question of law." (Rollo, p. 33)
Hence, this appeal.
The pivotal issue in this case is whether or not the court a quo had jurisdiction to try the instant case.
At the very core of this appeal assailing the aforesaid pronouncement of the lower court, and around which revolve the
arguments of the parties, is the applicability of Presidential Decree No. 902-A (Reorganization of the SEC with Additional
Powers), as amended by Presidential Decrees Nos. 1653, 1758 and 1799. Magalad submits that the legal suit which she
has brought against Premiere is an ordinary action for damages with the preliminary attachment cognizable solely by the
RTC. Premiere, on the other hand, espouses the original and exclusive jurisdiction of the Securities and Exchange
Commission.
Presidential Decree No. 902-A, Section 3, provides:
"SEC. 3. The Commission shall have absolute jurisdiction, supervision and control over all corporations,
partnerships or associations, who are the grantees of primary franchises and/or a license or permit
issued by the government to operate in the Philippines; and in the exercise of its authority, it shall have
the power to enlist the aid and support of and to deputize any and all enforcement agencies of the
government, civil or military as well as any private institution, corporation, firm, association or person."
(As amended by Presidential Decree No. 1758)
Sec. 3 of Pres. Decree No. 902-A should also be read in conjunction with Sec. 5 of the same law, providing:
"SEC. 5. In addition to the regulatory and adjudicative functions of the Securities and Exchange
Commission over corporations, partnerships and other forms of associations registered with it as
expressly granted under the existing laws and decrees, it shall have original and exclusive jurisdiction to
hear and decide cases involving:

'a) Devises or schemes employed by or any acts of the Board of Directors, business associates,
its officers or partners, amounting to fraud and misrepresentation which may be detrimental to
the public and/or to the stockholders, partners, members of associations or organizations
registered with the Commission.'"(Emphasis supplied)
Considering that Magalad's complaint sufficiently alleges acts amounting to fraud and misrepresentation committed by
Premiere, the SEC must be held to retain its original and exclusive jurisdiction over the case, despite the fact that the suit
involves collection of sums of money paid to said corporation, the recovery of which would originally fall within the
jurisdiction of regular courts. The fraud committed is detrimental to the interest of the public and, therefore, encompasses a
category of relationship within the SEC jurisdiction.
Otherwise stated, in order that the SEC can take cognizance of a case, the controversy must pertain to any of the following
relationships: (a) between the corporation, partnership or association and the public; (b) between the corporation,
partnership or association and its stockholders, partners, members or officers; (c) between the corporation, partnership or
association and the state so far as its franchise, permit or license to operate is concerned; and (d) among the stockholders,
partners or associates themselves (Union Glass & Container Corp. v. SEC, 126 SCRA 31; 38; 1983; Abejo v. De la Cruz,
149 SCRA 654, 1987). prLL
In this case, the recitals of the complaint sufficiently allege that devices or schemes amounting to fraud and
misrepresentation detrimental to the interest of the public have been resorted to by Premiere Corporation. It can not but be
conceded therefore, that the SEC may exercise its adjudicative powers pursuant to Sec. 5 (a) of Pres. Decree No. 902-A
(Supra).
The fact that Premiere's authority to engage in financing already expired will not have the effect of divesting the SEC of its
original and exclusive jurisdiction. The expanded jurisdiction of the SEC was conceived primarily to protect the interest of
the investing public. That Magalad's money placements were in the nature of investments in Premiere can not be gainsaid.
Magalad had reasonably expected to receive returns from moneys she had paid to Premiere. Unfortunately, however, she
was the victim of alleged fraud and misrepresentation.
Reliance by Magalad on the cases of DMRC v. Este del Sol, (132 SCRA 293) and Union Glass & Container Corp. v. SEC
(126 SCRA 31), where the jurisdiction of the ordinary Courts was upheld, is misplaced for, as explicitly stated in those
cases, nowhere in the complaints therein is found any averment of fraud of misrepresentation committed by the respective
corporations involved. The causes of action, therefore, were nothing more than simple money claims.
Further bolstering the jurisdiction of the SEC in this case is the fact that said agency already appointed a Rehabilitation
Receiver for Premiere and has directed all proceedings or claims against it be suspended. This, pursuant to Sec. 6(c) of
Pres. Decree No. 902-A providing that "upon appointment of a . . . rehabilitation receiver . . . all actions for claims against
corporations . . . under receivership pending before any court, tribunal, board or body shall be suspended accordingly."
By so doing, SEC has exercised its original and exclusive jurisdiction to hear and decide cases involving:
"a) Petitions of corporations, partnerships or associations to be declared in the state of suspension of
payments in cases where the corporation, partnership of association possesses sufficient property to
cover all its debts but foresees the impossibility of meeting them when they respectively fall due or in
cases where the corporation, partnership or association has no sufficient assets to cover its liabilities but
is under the management of a Rehabilitation Receiver or Management of a Rehabilitation Receiver or
Management Committee created pursuant to this Decree." (Section 5(d) of Pres. Decree No. 902-A as
added by Pres. Decree 1758).
In fine, the adjudicative powers of the SEC being clearly defined by law, its jurisdiction over this case has to be upheld.
PREMISES CONSIDERED, the instant appeal is GRANTED, and the order of the Presiding Judge of the Regional Trial
court, Quezon City, Branch LXXXV dated May 22, 1984, in Civil Case No. Q-40392 is REVERSED and SET ASIDE, without
prejudice to the filing by Alma Magalad of the appropriate complaint against Premiere Financing Corporation with the
Securities and Exchange Commission.
SO ORDERED.
||| (Magalad v. Premiere Financing Corp., G.R. No. 87135, [May 22, 1992], 284-A PHIL 728-735)

THE COLLECTOR OF INTERNAL REVENUE, petitioner, vs. THE CLUB FILIPINO, INC. DE CEBU,
respondent.
SYLLABUS

1. TAXATION; PERCENTAGE TAX; BAR AND RESTAURANT; WHEN OPERATOR NOT ENGAGED IN
BUSINESS. — The liability for fixed and percentage taxes as provided by Section 182, 183 and 191 of the Tax Code
does not ipso facto attach by mere reason of the operation of a bar and restaurant. For the liability to attach, the
operator thereof must be engaged in the business as a barkeeper and restauranteur.
2. ID.; WORDS AND PHRASES; "BUSINESS" MEANING OF. — The plain and ordinary meaning of a business
is restricted to activities or affairs where profit is the purpose or livelihood is the motive, and the term business when
used without qualification, should be construed in its plain and ordinary meaning, restricted to activities for profit or
livelihood.
3. ID.; CLUB FILIPINO INC. DE CEBU; NOT ENGAGED IN BAR AND RESTAURANT. — The Club Filipino Inc.
de Cebu was organized to develop and cultivate sports of all class and denomination, for the healthful recreation and
entertainment of its stockholders and members; that upon its dissolution, its remaining assets after paying debts shall
be donated to a charitable Philippine Institution in Cebu; that it is operated mainly with funds derived from membership
fees and dues; that the Club's bar and restaurant catered only to its members and their guests; that there was in fact no
cash dividend distribution to its stockholders and that whatever was derived on retail from its bar and restaurant was
used to defray its overall overhead expenses and to improve its golf course (cost-plus-expenses-basis), it stands to
reason that the Club is not engaged in the business of an operator of bar and restaurant.

DECISION

PAREDES, J p:

This is a petition to review the decision of the Court of Tax Appeals, reversing the decision of the Collector of
Internal Revenue, assessing against and demanding from the "Club Filipino, Inc. de Cebu," the sum of P12,068.84 as
fixed and percentage taxes, surcharge and compromise penalty, allegedly due from it as a keeper of bar and restaurant.
As found by the Court of Tax Appeals, the "Club Filipino, Inc. de Cebu," (Club, for short), is a civic corporation
organized under the laws of the Philippines, with an original authorized capital stock of P22,000.00, which was
subsequently increased to P200,000.00, among others, to "proporcionar, operar, y mantener un campo de golf, tenis,
gimnesio (gymnasiums), juego de bolos (bowling alleys), mesas de billar y pool, y toda clase de juegos no prohibidos
por leyes generales y ordenanzas generales; y desarollar y cultivar deportes de toda clase y denominacion cualquiera
para el recreo y entrenamiento saludable de sus miembros y accionistas" (sec. 2, Escritura de Incorporacion del Club
Filipino, Inc. Exh. A). Neither in the articles or by-laws is there a provision relative to dividends and their distribution,
although it is covenanted that upon its dissolution, the Club's remaining assets, after paying debts, shall be donated to a
charitable Philippine Institution in Cebu (Art. 27, Estatutos del Club, Exh. A-a).
The Club owns and operates a club house, a bowling alley, a golf course (on a lot leased from the government),
and a bar-restaurant where it sells wines and liquors, soft drinks, meals and short orders to its members and their
guests. The bar-restaurant was a necessary incident to the operation of the club and its golf-course. The club is
operated mainly with funds derived from membership fees and dues. Whatever profits it had, were used to defray its
overhead expenses and to improve its golf-course. In 1951, as a result of a capital surplus, arising from the re-valuation
of its real properties, the value or price of which increased, the Club declared stock dividends; but no actual cash
dividends were distributed to the stockholders. In 1952, a BIR agent discovered that the Club has never paid
percentage tax on the gross receipts of its bar and restaurant, although it secured B-4, B-9 (a) and B-7 licenses. In a
letter dated December 22, 1952, the Collector of Internal Revenue assessed against and demanded from the Club, the
following sums:—

As percentage tax on its gross receipts during the

taxyears 1946 to 1951 P9,599.07


Surcharge therein 2,399.77

As fixed tax for the years 1946 to 1952 70.00

Compromise penalty 500.00

The Club wrote the Collector, requesting for the cancellation of the assessment. The request having been denied, the
Club filed the instant petition for review.
The dominant issues involved in this case are twofold:
1. Whether the respondent Club is liable for the payment of the sum of P12,068.84, as fixed and
percentage taxes and surcharges prescribed in sections 182, 183 and 191 of the Tax
Code, under which the assessment was made, in connection with the operation of its bar
and restaurant, during the periods mentioned above; and
2. Whether it is liable for the payment of the sum of P500.00 as compromise penalty.
Section 182, of the Tax Code states "Unless otherwise provided, every person engaging in a business on which
the percentage tax is imposed shall pay in full a fixed annual tax of ten pesos for each calendar year or fraction thereof
in which such person shall engage in said business." Section 183 provides in general that "the percentage taxes on
business shall be payable at the end of each calendar quarter in the amount lawfully due on the business transacted
during each quarter; etc." And section 191, same Tax Code, provides "Percentage tax . . . Keepers of restaurants,
refreshment parlors and other eating places shall pay a tax three per centum, and keepers of bars and cafes where
wines or liquors are served, five per centum of their gross receipts . . ." It has been held that the liability for fixed and
percentage taxes, as provided by these sections, does not ipso facto attach by mere reason of the operation of a bar
and restaurant. For the liability to attach, the operator thereof must be engaged in the business as a barkeeper and
restauranteur. The plain and ordinary meaning of business is restricted to activities or affairs where profit is the purpose
or livelihood is the motive, and the term business when used without qualification, should be construed in its plain and
ordinary meaning, restricted to activities for profit or livelihood (The Coll. of Int. Rev. vs. Manila Lodge No. 761 of the
BPOE [Manila Elks Club] & Court of Tax Appeals, G.R. No. L-11176, June 29, 1959, giving full definitions of the word
"business"; Coll. of Int. Rev. vs. Sweeney, et al. [International Club of Iloilo, Inc.], G.R. No. L-12178, Aug. 21, 1959, the
facts of which are similar to ones at bar; Manila Polo Club v. B.L. Meer, etc., No. L-10854, Jan. 27, 1960).
Having found as a fact that the Club was organized to develop and cultivate sports of all class and
denomination, for the healthful recreation and entertainment of its stockholders and members; that upon its dissolution,
its remaining assets, after paying debts, shall be donated to a charitable Philippine Institution in Cebu; that it is operated
mainly with funds derived from membership fees and dues; that the Club's bar and restaurant catered only to its
members and their guests; that there was in fact no cash dividend distribution to its stockholders and that whatever was
derived on retail from its bar and restaurant was used to defray its overall overhead expenses and to improve its golf-
course (cost-plus-expenses-basis), it stands to reason that the Club is not engaged in the business of an operator of bar
and restaurant (same authorities, cited above).
It is conceded that the Club derived profit from the operation of its bar and restaurant, but such fact does not
necessarily convert it into a profit-making enterprise. The bar and restaurant are necessary adjuncts' of the Club to
foster its purposes and the profits derived therefrom are necessarily incidental to the primary object of developing and
cultivating sports for the healthful recreation and entertainment of the stockholders and members. That a Club makes
some profit, does not make it a profit-making club. As has been remarked, a club should always strive, whenever
possible, to have a surplus (Jesus Sacred Heart College vs. Collector of Int. Revenue, G.R. No. L-6807, May 24, 1954;
Collector of Int. Rev. v. Sinco Educational Corp., G.R. No. L-9276, Oct. 23 1956).
It is claimed that unlike the two cases just cited (supra), which are non-stock, the appellee Club is a stock
corporation. This is unmeritorious. The facts that the capital stock of the respondent Club is divided into shares, does
not detract from the finding of the trial court that it is not engaged in the business of operator of bar and restaurant.
What is determinative of whether or not the Club is engaged in such business is its object or purpose, as stated in its
articles and by-laws. It is a familiar rule that the actual purpose is not controlled by the corporate form or by the
commercial aspect of the business prosecuted, but may be shown by extrinsic evidence, including the by-laws and the
method of operation. From the extrinsic evidence adduced, the Tax Court concluded that the Club is not engaged in the
business as a barkeeper and restaurateur.
Moreover, for a stock corporation to exist, two requisites must be complied with, to wit: (1) a capital stock
divided into shares and (2) an authority to distribute to the holders of such shares, dividends or allotments of the surplus
profits on the basis of the shares held (sec. 3, Act No. 1459). In the case at bar, while the respondent Club's, capital
stock is divided into shares, nowhere in its articles of incorporation or by-laws could be found an authority for the
distribution of its dividends or surplus profits. Strictly speaking, it cannot, therefore, be considered a stock corporation,
within the contemplation of the corporation law.
"A tax is a burden, and, as such, it should not be deemed imposed upon fraternal, civic, non-profit, non-stock
organizations, unless the intent to the contrary is manifest and patent" (Collector vs. BPOE Elks Club, et al., supra),
which is not the case in the present appeal.
Having arrived at the conclusion that respondent Club is not engaged in the business as an operator of a bar
and restaurant, and therefore, not liable for fixed and percentage taxes, it follows that it is not liable for any penalty,
much less of a compromise penalty.
WHEREFORE, the decision appealed from, is affirmed, without costs.
||| (Collector of Internal Revenue v. Club Filipino, Inc., de Cebu, G.R. No. L-12719, [May 31, 1962], 115 PHIL 310-315)

[G.R. No. 91889. August 27, 1993.]

MANUEL R. DULAY ENTERPRISES, INC., VIRGILIO E. DULAY AND NEPOMUCENO REDOVAN,


petitioners, vs. THE HONORABLE COURT OF APPEALS, EDGARDO D. PABALAN, MANUEL A. TORRES,
JR., MARIA THERESA V. VELOSO and CASTRENSE C. VELOSO, respondents.

Virgilio E. Dulay for petitioners.


Torres, Tobias, Azura & Jocson for private respondents.

SYLLABUS

1. COMMERCIAL LAW; CORPORATION; CLOSE CORPORATION; BOARD MEETING WITHOUT PROPER NOTICE;
CORPORATE ACTION; DEEMED RATIFIED BY ABSENT DIRECTOR UNLESS PROMPTLY OBJECTED. — Petitioner
corporation is classified as a close corporation and consequently a board resolution authorizing the sale or mortgage of the
subject property is not necessary to bind the corporation for the action of its president. At any rate, a corporate action taken
at a board meeting without proper call or notice in a close corporation is deemed ratified by the absent director unless the
latter promptly files his written objection with the secretary of the corporation after having knowledge of the meeting which,
in this case, petitioner Virgilio Dulay failed to do. Petitioners' claim that the sale of the subject property by its president,
Manuel Dulay, to private respondents spouses Veloso is null and void as the alleged Board Resolution No. 18 was passed
without the knowledge and consent of the other members of the board of directors cannot be sustained. The sale of the
subject property to private respondents by Manuel Dulay is valid and binding.
2. ID.; ID.; PIERCING THE VEIL OF CORPORATE ENTITY; WHEN RESORTED TO. — Although a corporation is an entity
which has a personality distinct and separate from its individual stockholders or members, the veil of corporate fiction may
be pierced when it is used to defeat public convenience, justify wrong, protect fraud or defend crime. The privilege of being
treated as an entity distinct and separate from its stockholders or members is therefore confined to its legitimate uses and is
subject to certain limitations to prevent the commission of fraud or other illegal or unfair act. When the corporation is used
merely as an alter ego or business conduit of a person, the law will regard the corporation as the act of that person. The
Supreme Court had repeatedly disregarded the separate personality of the corporation where the corporate entity was used
to annul a valid contract executed by one of its members.
3. REMEDIAL LAW; EVIDENCE; FINDINGS OF TRIAL COURT, RESPECTED. — The appellate courts will not disturb the
findings of the trial judge unless he has plainly overlooked certain facts of substance and value that, if considered, might
affect the result of the case, which is not present in the instant case.
4. CIVIL LAW; SPECIAL CONTRACTS; SALES; OBLIGATIONS OF THE VENDOR; DELIVERY OF THE THING SOLD;
WHEN SALE EXECUTED IN PUBLIC INSTRUMENT. — Paragraph 1, Article 1498 of the New Civil Code provides: "When
the sale is made through a public instrument, the execution thereof shall be equivalent to the delivery of the thing which is
the object of the contract, if from the deed the contrary does not appear or cannot clearly be inferred." Under the
aforementioned article, the mere execution of the deed of sale in a public document is equivalent to the delivery of the
property. Likewise, this Court had held that: "It is settled that the buyer in a foreclosure sale becomes the absolute owner of
the property purchased if it is not redeemed during the period of one year after the registration of the sale. As such, he is
entitled to the possession of the said property and can demand it at any time following the consolidation of ownership in his
name and the issuance to him of a new transfer certificate of title. The buyer can in fact demand possession of the land
even during the redemption period except that he has to post a bond in accordance with Section 7 of Act No. 3133 as
amended. No such bond is required after the redemption period if the property is not redeemed. Possession of the land then
becomes an absolute right of the purchaser as confirmed owner." Therefore, prior physical delivery or possession is not
legally required since the execution of the Deed of Sale is deemed equivalent to delivery.
5. REMEDIAL LAW; CIVIL PROCEDURE; MOTION FOR RECONSIDERATION; DENIAL DESPITE FAILURE TO SUBMIT
COMMENT THEREOF, PROPER. — The respondent appellate court did not err in denying petitioner's motion for
reconsideration despite the fact that private respondents failed to submit their comment to said motion as required by the
respondent appellate court. There is nothing in the Revised Rules of Court which prohibits the respondent appellate court
from resolving petitioners' motion for reconsideration without the comment of the private respondent which was required
merely to aid the court in the disposition of the motion. The courts are as much interested as the parties in the early
disposition of cases before them. To require otherwise would unnecessarily clog the courts' dockets.

DECISION

NOCON, J p:

This is a petition for review on certiorari to annul and set aside the decision 1 of the Court of Appeals affirming the decision
2 of the Regional Trial Court of Pasay, Branch 114 in Civil Cases Nos. 8198-P, 8278-P and 2880-P, the dispositive portion
of which reads, as follows:
"WHEREFORE, in view of all the foregoing considerations, this Court hereby renders judgment, as
follows:
"In Civil Case No. 2880-P, the petition filed by Manuel R. Dulay Enterprises, Inc. and Virgilio E. Dulay for
annulment or declaration of nullity of the decision of the Metropolitan Trial Court, Branch 46, Pasay City,
in its Civil Case No. 38-81 entitled `Edgardo D. Pabalan, et al., vs. Spouses Florentino Manalastas, et al.,
' is dismissed for lack of merit;
"In Civil Case No. 8278-P, the complaint filed by Manuel R. Dulay Enterprises, Inc. for cancellation of title
of Manuel A. Torres, Jr. (TCT No. 24799 of the Register of Deeds of Pasay City) and reconveyance, is
dismissed for lack of merit; and,
"In Civil Case No. 8198-P, defendants Manuel R. Dulay Enterprises, Inc. and Virgilio E. Dulay are
ordered to surrender and deliver possession of the parcel of land, together with all the improvements
thereon, described in Transfer Certificate of Title No. 24799 of the Register of Deeds of Pasay City, in
favor of therein plaintiffs Manuel A. Torres, Jr. as owner and Edgardo D. Pabalan as real estate
administrator of said Manuel A. Torres, Jr.; to account for and return to said plaintiffs the rentals from
dwelling unit No. 8-A of the apartment building (Dulay Apartment) from June 1980 up to the present; to
indemnify plaintiffs, jointly and severally, expenses of litigation in the amount of P4,000.00 and attorney's
fees in the sum of P6,000.00, for all the three (3) cases. Co-defendant Nepomuceno Redovan is ordered
to pay the current and subsequent rentals on the premises leased by him to plaintiffs.
"The counterclaim of defendants Virgilio E. Dulay and Manuel R. Dulay Enterprises, Inc. and N. Redovan,
is dismissed for lack of merit. With costs against the three (3) aforenamed defendants." 3
The facts as found by the trial court are as follows:
Petitioner Manuel R. Dulay Enterprises, Inc., a domestic corporation with the following as members of its Board of Directors:
Manuel R. Dulay with 19,960 shares and designated as president, treasurer and general manager; Atty. Virgilio E. Dulay
with 10 shares and designated as vice-president; Linda E. Dulay with 10 shares; Celia Dulay-Mendoza with 10 shares; and
Atty. Plaridel C. Jose with 10 shares and designated as secretary, owned a property covered by TCT No. 17880 4 and
known as Dulay Apartment consisting of sixteen (16) apartment units on a six hundred eighty-nine (689) square meter lot,
more or less, located at Seventh Street (now Buendia Extension) and F.B. Harrison Street, Pasay City. LLpr
Petitioner corporation through its president, Manuel Dulay, obtained various loans for the construction of its hotel project,
Dulay Continental Hotel (now Frederick Hotel). It even had to borrow money from petitioner Virgilio Dulay to be able to
continue the hotel project. As a result of said loan, petitioner Virgilio Dulay occupied one of the unit apartments of the
subject property since 1973 while at the same time managing the Dulay Apartment as his shareholdings in the corporation
was subsequently increased by his father. 5
On December 23, 1976, Manuel Dulay by virtue of Board Resolution No. 18 6 of petitioner corporation sold the subject
property to private respondents spouses Maria Theresa and Castrense Veloso in the amount of P300,000.00 as evidenced
by the Deed of Absolute Sale. 7 Thereafter, TCT No. 17880 was cancelled and TCT No. 23225 was issued to private
respondent Maria Theresa Veloso. 8 Subsequently, Manuel Dulay and private respondents spouses Veloso executed a
Memorandum to the Deed of Absolute Sale of December 23, 1976 9 dated December 9, 1977 giving Manuel Dulay within
two (2) years or until December 9, 1979 to repurchase the subject property for P200,000.00 which was, however, not
annotated either in TCT No. 17880 or TCT No. 23225.
On December 24, 1976, private respondent Maria Veloso, without the knowledge of Manuel Dulay, mortgaged the subject
property to private respondent Manuel A. Torres for a loan of P250,000.00 which was duly annotated as Entry No. 68139 in
TCT No. 23225. 10
Upon the failure of private respondent Maria Veloso to pay private respondent Torres, the subject property was sold on April
5, 1978 to private respondent Torres as the highest bidder in an extrajudicial foreclosure sale as evidenced by the
Certificate of Sheriff's Sale 11 issued on April 20, 1978.
On July 20, 1978, private respondent Maria Veloso executed a Deed of Absolute Assignment of the Right to Redeem 12 in
favor of Manuel Dulay assigning her right to repurchase the subject property from private respondent Torres as a result of
the extrajudicial sale held on April 25, 1978.

As neither private respondent Maria Veloso nor her assignee Manuel Dulay was able to redeem the subject property within
the one year statutory period for redemption, private respondent Torres filed an Affidavit of Consolidation of Ownership 13
with the Registry of Deeds of Pasay City and TCT No. 24799 14 was subsequently issued to private respondent Manuel
Torres on April 23, 1979.
On October 1, 1979, private respondent Torres filed a petition for the issuance of a writ of possession against private
respondents spouses Veloso and Manuel Dulay in LRC Case No. 1742-P. However, when petitioner Virgilio Dulay
appeared in court to intervene in said case alleging that Manuel Dulay was never authorized by the petitioner corporation to
sell or mortgage the subject property, the trial court ordered private respondent Torres to implead petitioner corporation as
an indispensable party but the latter moved for the dismissal of his petition which was granted in an Order dated April 8,
1980. cdphil
On June 20, 1980, private respondent Torres and Edgardo Pabalan, real estate administrator of Torres, filed an action
against petitioner corporation, Virgilio Dulay and Nepomuceno Redovan, a tenant of Dulay Apartment Unit No. 8-A for the
recovery of possession, sum of money and damages with preliminary injunction in Civil Case No. 8198-P with the then
Court of First Instance of Rizal.
On July 21, 1980, petitioner corporation filed an action against private respondents spouses Veloso and Torres for the
cancellation of the Certificate of Sheriff's Sale and TCT No. 24799 in Civil Case No. 8278-P with the then Court of First
Instance of Rizal.
On January 29, 1981, private respondents Pabalan and Torres filed an action against spouses Florentino and Elvira
Manalastas, a tenant of Dulay Apartment Unit No. 7-B, with petitioner corporation as intervenor for ejectment in Civil Case
No. 38-81 with the Metropolitan Trial Court of Pasay City which rendered a decision on April 25, 1985, the dispositive
portion of which reads, as follows:
"WHEREFORE, judgment is hereby rendered in favor of the plaintiff (herein private
respondents) and against the defendants:
"1. Ordering the defendants and all persons claiming possession under them to vacate the
premises;
"2. Ordering the defendants to pay the rents in the sum of P500.00 a month from May,
1979 until they shall have vacated the premises with interest at the legal rate;
"3. Ordering the defendants to pay attorney's fees in the sum of P2,000.00 and P1,000.00
as other expenses of litigation and for them to pay the costs of the suit." 15
Thereafter or on May 17, 1985, petitioner corporation and Virgilio Dulay filed an action against the presiding judge of the
Metropolitan Trial Court of Pasay City, private respondents Pabalan and Torres for the annulment of said decision with the
Regional Trial Court of Pasay in Civil Case No. 2880-P.
Thereafter, the three (3) cases were jointly tried and the trial court rendered a decision in favor of private respondents.
Not satisfied with said decision, petitioners appealed to the Court of Appeals which rendered a decision on October 23,
1989, the dispositive portion of which reads, as follows:
"PREMISES CONSIDERED, the decision being appealed should be as it is hereby
AFFIRMED in full." 16
On November 8, 1989, petitioners filed a Motion for Reconsideration which was denied on January 26, 1990.
Hence, this petition.
During the pendency of this petition, private respondent Torres died on April 3, 1991 as shown in his death certificate 17
and named Torres-Pabalan Realty & Development Corporation as his heir in his holographic will 18 dated October 31,
1986.
Petitioners contend that the respondent court had acted with grave abuse of discretion when it applied the doctrine of
piercing the veil of corporate entity in the instant case considering that the sale of the subject property between private
respondents spouses Veloso and Manuel Dulay has no binding effect on petitioner corporation as Board Resolution No. 18
which authorized the sale of the subject property was resolved without the approval of all the members of the board of
directors and said Board Resolution was prepared by a person not designated by the corporation to be its secretary. LLjur
We do not agree.
Section 101 of the Corporation Code of the Philippines provides:
"Sec. 101. When board meeting is unnecessary or improperly held. Unless the by-laws
provide otherwise, any action by the directors of a close corporation without a meeting shall
nevertheless be deemed valid if:
"1. Before or after such action is taken, written consent thereto is signed by all the
directors; or
"2. All the stockholders have actual or implied knowledge of the action and make no
prompt objection thereto in writing; or
"3. The directors are accustomed to take informal action with the express or implied
acquiesce of all the stockholders; or
"4. All the directors have express or implied knowledge of the action in question and none
of them makes prompt objection thereto in writing.
"If a directors' meeting is held without proper call or notice, an action taken therein within
the corporate powers is deemed ratified by a director who failed to attend, unless he promptly files
his written objection with the secretary of the corporation after having knowledge thereof."
In the instant case, petitioner corporation is classified as a close corporation and consequently a board resolution
authorizing the sale or mortgage of the subject property is not necessary to bind the corporation for the action of its
president. At any rate, a corporate action taken at a board meeting without proper call or notice in a close corporation is
deemed ratified by the absent director unless the latter promptly files his written objection with the secretary of the
corporation after having knowledge of the meeting which, in this case, petitioner Virgilio Dulay failed to do.
It is relevant to note that although a corporation is an entity which has a personality distinct and separate from its individual
stockholders or members, 19 the veil of corporate fiction may be pierced when it is used to defeat public convenience,
justify wrong, protect fraud or defend crime. 20 The privilege of being treated as an entity distinct and separate from its
stockholders or members is therefore confined to its legitimate uses and is subject to certain limitations to prevent the
commission of fraud or other illegal or unfair act. When the corporation is used merely as an alter ego or business conduit of
a person, the law will regard the corporation as the act of that person. 21 The Supreme Court had repeatedly disregarded
the separate personality of the corporation where the corporate entity was used to annul a valid contract executed by one of
its members.
Petitioners' claim that the sale of the subject property by its president, Manuel Dulay, to private respondents spouses
Veloso is null and void as the alleged Board Resolution No. 18 was passed without the knowledge and consent of the other
members of the board of directors cannot be sustained. As correctly pointed out by the respondent Court of Appeals:
"Appellant Virgilio E. Dulay's protestations of complete innocence to the effect that he never participated
nor was even aware of any meeting or resolution authorizing the mortgage or sale of the subject
premises (see par. 8, affidavit of Virgilio E. Dulay, dated May 31, 1984, p. 14, Exh. "21") is difficult to
believe. On the contrary, he is very much privy to the transactions involved. To begin with, he is an
incorporator and one of the board of directors designated at the time of the organization of Manuel R.
Dulay Enterprises, Inc. In ordinary parlance, the said entity is loosely referred to as a 'family corporation'.
The nomenclature, if imprecise, however, fairly reflects the cohesiveness of a group and the parochial
instincts of the individual members of such an aggrupation of which Manuel R. Dulay Enterprises, Inc. is
typical: four-fifths of its incorporators being close relatives namely, three (3) children and their father
whose name identifies their corporation (Articles of Incorporation of Manuel R. Dulay Enterprises, Inc.,
Exh. "31-A")." 22
Besides, the fact that petitioner Virgilio Dulay on June 24, 1975 executed an affidavit 23 that he was a signatory witness to
the execution of the post-dated Deed of Absolute Sale of the subject property in favor of private respondent Torres indicates
that he was aware of the transaction executed between his father and private respondents and had, therefore, adequate
knowledge about the sale of the subject property to private respondents. LLpr
Consequently, petitioner corporation is liable for the act of Manuel Dulay and the sale of the subject property to private
respondents by Manuel Dulay is valid and binding. As stated by the trial court:
". . . the sale between Manuel R. Dulay Enterprises, Inc. and the spouses Maria Theresa
V. Veloso and Castrense C. Veloso, was a corporate act of the former and not a personal
transaction of Manuel R. Dulay. This is so because Manuel R. Dulay was not only president and
treasurer but also the general manager of the corporation. The corporation was a closed family
corporation and the only non-relative in the board of directors was Atty. Plaridel C. Jose who
appeared on paper as the secretary. There is no denying the fact, however, that Maria Socorro R.
Dulay at times acted as secretary. . . . , the Court can not lose sight of the fact that the Manuel R.
Dulay Enterprises, Inc. is a closed family corporation where the incorporators and directors belong
to one single family. It cannot be concealed that Manuel R. Dulay as president, treasurer and
general manager almost had absolute control over the business and affairs of the corporation." 24
Moreover, the appellate courts will not disturb the findings of the trial judge unless he has plainly overlooked certain facts of
substance and value that, if considered, might affect the result of the case, 25 which is not present in the instant case.

Petitioners' contention that private respondent Torres never acquired ownership over the subject property since the latter
was never in actual possession of the subject property nor was the property ever delivered to him is also without merit.
Paragraph 1, Article 1498 of the New Civil Code provides:
"When the sale is made through a public instrument, the execution thereof shall be
equivalent to the delivery of the thing which is the object of the contract, if from the deed the
contrary does not appear or cannot clearly be inferred."
Under the aforementioned article, the mere execution of the deed of sale in a public document is equivalent to the delivery
of the property. Likewise, this Court had held that:
"It is settled that the buyer in a foreclosure sale becomes the absolute owner of the
property purchased if it is not redeemed during the period of one year after the registration of the
sale. As such, he is entitled to the possession of the said property and can demand it at any time
following the consolidation of ownership in his name and the issuance to him of a new transfer
certificate of title. The buyer can in fact demand possession of the land even during the redemption
period except that he has to post a bond in accordance with Section 7 of Act No. 3133 as
amended. No such bond is required after the redemption period if the property is not redeemed.
Possession of the land then becomes an absolute right of the purchaser as confirmed owner." 26
Therefore, prior physical delivery or possession is not legally required since the execution of the Deed of Sale is deemed
equivalent to delivery. LLpr
Finally, we hold that the respondent appellate court did not err in denying petitioner's motion for reconsideration despite the
fact that private respondents failed to submit their comment to said motion as required by the respondent appellate court.
There is nothing in the Revised Rules of Court which prohibits the respondent appellate court from resolving petitioners'
motion for reconsideration without the comment of the private respondent which was required merely to aid the court in the
disposition of the motion. The courts are as much interested as the parties in the early disposition of cases before them. To
require otherwise would unnecessarily clog the courts' dockets.
WHEREFORE, the petition is DENIED and the decision appealed from is hereby AFFIRMED. SO ORDERED.
||| (Manuel R. Dulay Enterprises, Inc. v. Court of Appeals, G.R. No. 91889, [August 27, 1993])

[G.R. Nos. 84132-33. December 10, 1990.]


NATIONAL DEVELOPMENT COMPANY AND NEW AGRIX, INC., petitioners, vs. PHILIPPINE
VETERANS BANK, THE EX-OFFICIO SHERIFF and GODOFREDO QUILING, in his capacity as
Deputy Sheriff of Calamba, Laguna, respondents.

Vicente Pascual, Jr. and Lope E. Feble for Philippine Veterans Bank.

DECISION

CRUZ, J p:

This case involves the constitutionality of a presidential decree which, like all other issuances of President Marcos during his
regime, was at that time regarded as sacrosanct. It is only now, in a freer atmosphere, that his acts are being tested by the
touchstone of the fundamental law that even then was supposed to limit presidential action. cdrep
The particular enactment in question is Pres. Decree No. 1717, which ordered the rehabilitation of the Agrix Group of
Companies to be administered mainly by the National Development Company. The law outlined the procedure for filing
claims against the Agrix companies and created a Claims Committee to process these claims. Especially relevant to this
case, and noted at the outset, is Sec. 4(1) thereof providing that "all mortgages and other liens presently attaching to any of
the assets of the dissolved corporations are hereby extinguished."
Earlier, the Agrix Marketing, Inc. (AGRIX) had executed in favor of private respondent Philippine Veterans Bank a real
estate mortgage dated July 7, 1978, over three (3) parcels of land situated in Los Baños, Laguna. During the existence of
the mortgage, AGRIX went bankrupt. It was for the expressed purpose of salvaging this and the other Agrix companies that
the aforementioned decree was issued by President Marcos.
Pursuant thereto, the private respondent filed a claim with the AGRIX Claims Committee for the payment of its loan credit.
In the meantime, the New Agrix, Inc. and the National Development Company, petitioners herein, invoking Sec. 4 (1) of the
decree, filed a petition with the Regional Trial Court of Calamba, Laguna, for the cancellation of the mortgage lien in favor of
the private respondent. For its part, the private respondent took steps to extrajudicially foreclose the mortgage, prompting
the petitioners to file a second case with the same court to stop the foreclosure. The two cases were consolidated.
After the submission by the parties of their respective pleadings, the trial court rendered the impugned decision. Judge
Francisco Ma. Guerrero annulled not only the challenged provision, viz., Sec. 4 (1), but the entire Pres. Decree No. 1717 on
the grounds that: (1) the presidential exercise of legislative power was a violation of the principle of separation of powers;
(2) the law impaired the obligation of contracts; and (3) the decree violated the equal protection clause. The motion for
reconsideration of this decision having been denied, the present petition was filed. cdrep
The petition was originally assigned to the Third Division of this Court but because of the constitutional questions involved it
was transferred to the Court en banc. On August 30, 1988, the Court granted the petitioner's prayer for a temporary
restraining order and instructed the respondents to cease and desist from conducting a public auction sale of the lands in
question. After the Solicitor General and the private respondent had filed their comments and the petitioners their reply, the
Court gave due course to the petition and ordered the parties to file simultaneous memoranda. Upon compliance by the
parties, the case was deemed submitted.
The petitioners contend that the private respondent is now estopped from contesting the validity of the decree. In support of
this contention, it cites the recent case of Mendoza v. Agrix Marketing, Inc., 1 where the constitutionality of Pres. Decree No.
1717 was also raised but not resolved. The Court, after noting that the petitioners had already filed their claims with the
AGRIX Claims Committee created by the decree, had simply dismissed the petition on the ground of estoppel.
The petitioners stress that in the case at bar the private respondent also invoked the provisions of Pres. Decree No. 1717 by
filing a claim with the AGRIX Claims Committee. Failing to get results, it sought to foreclose the real estate mortgage
executed by AGRIX in its favor, which had been extinguished by the decree. It was only when the petitioners challenged the
foreclosure on the basis of Sec. 4 (1) of the decree, that the private respondent attacked the validity of the provision. At that
stage, however, consistent with Mendoza, the private respondent was already estopped from questioning the
constitutionality of the decree.
The Court does not agree that the principle of estoppel is applicable.
It is not denied that the private respondent did file a claim with the AGRIX Claims Committee pursuant to this decree. It must
be noted, however, that this was done in 1980, when President Marcos was the absolute ruler of this country and his
decrees were the absolute law. Any judicial challenge to them would have been futile, not to say foolhardy. The private
respondent, no less than the rest of the nation, was aware of that reality and knew it had no choice under the circumstances
but to conform. cdll
It is true that there were a few venturesome souls who dared to question the dictator's decisions before the courts of justice
then. The record will show, however, that not a single act or issuance of President Marcos was ever declared
unconstitutional, not even by the highest court, as long as he was in power. To rule now that the private respondent is
estopped for having abided with the decree instead of boldly assailing it is to close our eyes to a cynical fact of life during
that repressive time.
This case must be distinguished from Mendoza, where the petitioners, after filing their claims with the AGRIX Claims
Committee, received in settlement thereof shares of stock valued at P40,000.00 without protest or reservation. The herein
private respondent has not been paid a single centavo on its claim, which was kept pending for more than seven years for
alleged lack of supporting papers. Significantly, the validity of that claim was not questioned by the petitioner when it sought
to restrain the extrajudicial foreclosure of the mortgage by the private respondent. The petitioner limited itself to the
argument that the private respondent was estopped from questioning the decree because of its earlier compliance with its
provisions.
Independently of these observations, there is the consideration that an affront to the Constitution cannot be allowed to
continue existing simply because of procedural inhibitions that exalt form over substance.
The Court is especially disturbed by Section 4(1) of the decree, quoted above, extinguishing all mortgages and other liens
attaching to the assets of AGRIX. It also notes, with equal concern, the restriction in Subsection (ii) thereof that all
"unsecured obligations shall not bear interest" and in Subsection (iii) that "all accrued interests, penalties or charges as of
date hereof pertaining to the obligations, whether secured or unsecured, shall not be recognized."
These provisions must be read with the Bill of Rights, where it is clearly provided in Section 1 that "no person shall be
deprived of life, liberty or property without due course of law nor shall any person be denied the equal protection of the law"
and in Section 10 that "no law impairing the obligation of contracts shall be passed."
In defending the decree, the petitioners argue that property rights, like all rights, are subject to regulation under the police
power for the promotion of the common welfare. The contention is that this inherent power of the state may be exercised at
any time for this purpose so long as the taking of the property right, even if based on contract, is done with due process of
law.
This argument is an over-simplification of the problem before us. The police power is not a panacea for all constitutional
maladies. Neither does its mere invocation conjure an instant and automatic justification for every act of the government
depriving a person of his life, liberty or property.
A legislative act based on the police power requires the concurrence of a lawful subject and a lawful method. In more
familiar words, a) the interests of the public generally, as distinguished from those of a particular class, should justify the
interference of the state; and b) the means employed are reasonably necessary for the accomplishment of the purpose and
not unduly oppressive upon individuals. 2
Applying these criteria to the case at bar, the Court finds first of all that the interests of the public are not sufficiently involved
to warrant the interference of the government with the private contracts of AGRIX. The decree speaks vaguely of the
"public, particularly the small investors," who would be prejudiced if the corporation were not to be assisted. However, the
record does not state how many there are of such investors, and who they are, and why they are being preferred to the
private respondent and other creditors of AGRIX with vested property rights. Cdpr
The public interest supposedly involved is not identified or explained. It has not been shown that by the creation of the New
Agrix, Inc. and the extinction of the property rights of the creditors of AGRIX, the interests of the public as a whole, as
distinguished from those of a particular class, would be promoted or protected. The indispensable link to the welfare of the
greater number has not been established. On the contrary, it would appear that the decree was issued only to favor a
special group of investors who, for reasons not given, have been preferred to the legitimate creditors of AGRIX.
Assuming there is a valid public interest involved, the Court still finds that the means employed to rehabilitate AGRIX fall far
short of the requirement that they shall not be unduly oppressive. The oppressiveness is patent on the face of the decree.
The right to property in all mortgages, liens, interests, penalties and charges owing to the creditors of AGRIX is arbitrarily
destroyed. No consideration is paid for the extinction of the mortgage rights. The accrued interests and other charges are
simply rejected by the decree. The right to property is dissolved by legislative fiat without regard to the private interest
violated and, worse, in favor of another private interest.

A mortgage lien is a property right derived from contract and so comes under the protection of the Bill of Rights. So do
interests on loans, as well as penalties and charges, which are also vested rights once they accrue. Private property cannot
simply be taken by law from one person and given to another without compensation and any known public purpose. This is
plain arbitrariness and is not permitted under the Constitution.
And not only is there arbitrary taking, there is discrimination as well. In extinguishing the mortgage and other liens, the
decree lumps the secured creditors with the unsecured creditors and places them on the same level in the prosecution of
their respective claims. In this respect, all of them are considered unsecured creditors. The only concession given to the
secured creditors is that their loans are allowed to earn interest from the date of the decree, but that still does not justify the
cancellation of the interests earned before that date. Such interests, whether due to the secured or the unsecured creditors,
are all extinguished by the decree. Even assuming such cancellation to be valid, we still cannot see why all kinds of
creditors, regardless of security, are treated alike.
Under the equal protection clause, all persons or things similarly situated must be treated alike, both in the privileges
conferred and the obligations imposed. Conversely, all persons or things differently situated should be treated differently. In
the case at bar, persons differently situated are similarly treated, in disregard of the principle that there should be equality
only among equals. llcd
One may also well wonder why AGRIX was singled out for government help, among other corporations where the
stockholders or investors were also swindled. It is not clear why other companies entitled to similar concern were not
similarly treated. And surely, the stockholders of the private respondent, whose mortgage lien had been cancelled and
legitimate claims to accrued interests rejected, were no less deserving of protection, which they did not get. The decree
operated, to use the words of a celebrated case, 3 "with an evil eye and an uneven hand."
On top of all this, New Agrix, Inc. was created by special decree notwithstanding the provision of Article XIV, Section 4 of
the 1973 Constitution, then in force, that:
SEC. 4. The Batasang Pambansa shall not, except by general law, provide for the formation,
organization, or regulation of private corporations, unless such corporations are owned or controlled by
the Government or any subdivision or instrumentality thereof. 4
The new corporation is neither owned nor controlled by the government. The National Development Corporation was merely
required to extend a loan of not more than P10,000,000.00 to New Agrix, Inc. Pending payment thereof, NDC would
undertake the management of the corporation, but with the obligation of making periodic reports to the Agrix board of
directors. After payment of the loan, the said board can then appoint its own management. The stocks of the new
corporation are to be issued to the old investors and stockholders of AGRIX upon proof of their claims against the abolished
corporation. They shall then be the owners of the new corporation. New Agrix, Inc. is entirely private and so should have
been organized under the Corporation Law in accordance with the above-cited constitutional provision.
The Court also feels that the decree impairs the obligation of the contract between AGRIX and the private respondent
without justification. While it is true that the police power is superior to the impairment clause, the principle will apply only
where the contract is so related to the public welfare that it will be considered congenitally susceptible to change by the
legislature in the interest of the greater number. 5 Most present-day contracts are of that nature. But as already observed,
the contracts of loan and mortgage executed by AGRIX are purely private transactions and have not been shown to be
affected with public interest. There was therefore no warrant to amend their provisions and deprive the private respondent of
its vested property rights.
It is worth noting that only recently in the case of the Development Bank of the Philippines v. NLRC, 6 we sustained the
preference in payment of a mortgage creditor as against the argument that the claims of laborers should take precedence
over all other claims, including those of the government. In arriving at this ruling, the Court recognized the mortgage lien as
a property right protected by the due process and contract clauses notwithstanding the argument that the amendment in
Section 110 of the Labor Code was a proper exercise of the police power. prcd
The Court reaffirms and applies that ruling in the case at bar.
Our finding, in sum, is that Pres. Decree No. 1717 is an invalid exercise of the police power, not being in conformity with the
traditional requirements of a lawful subject and a lawful method. The extinction of the mortgage and other liens and of the
interest and other charges pertaining to the legitimate creditors of AGRIX constitutes taking without due process of law, and
this is compounded by the reduction of the secured creditors to the category of unsecured creditors in violation of the equal
protection clause. Moreover, the new corporation, being neither owned nor controlled by the Government, should have been
created only by general and not special law. And insofar as the decree also interferes with purely private agreements
without any demonstrated connection with the public interest, there is likewise an impairment of the obligation of the
contract.
With the above pronouncements, we feel there is no more need to rule on the authority of President Marcos to promulgate
Pres. Decree No. 1717 under Amendment No. 6 of the 1973 Constitution. Even if he had such authority, the decree must
fall just the same because of its violation of the Bill of Rights.
WHEREFORE, the petition is DISMISSED. Pres. Decree No. 1717 is declared UNCONSTITUTIONAL. The temporary
restraining order dated August 30, 1988, is LIFTED. Costs against the petitioners. llcdSO ORDERED.
||| (National Development Co. v. Philippine Veterans Bank, G.R. Nos. 84132-33, [December 10, 1990], 270 PHIL 349-360)

[G.R. No. 84197. July 28, 1989.]

PIONEER INSURANCE & SURETY CORPORATION, petitioner, vs. THE HON. COURT OF APPEALS,
BORDER MACHINERY & HEAVY EQUIPMENT, INC., (BORMAHECO), CONSTANCIO M. MAGLANA
and JACOB S. LIM, respondents.

[G.R. No. 84157. July 28, 1989.]

JACOB S. LIM, petitioner, vs. COURT OF APPEALS, PIONEER INSURANCE AND SURETY
CORPORATION, BORDER MACHINERY and HEAVY EQUIPMENT CO., INC., FRANCISCO and
MODESTO CERVANTES and CONSTANCIO MAGLANA, respondents.

Eriberto D. Ignacio for Pioneer Insurance & Surety Corporation.


Sycip, Salazar, Hernandez & Gatmaitan for Jacob S. Lim.
Renato J. Robles for BORMAHECO, Inc. and Cervanteses.
Leonardo B. Lucena for Constancio Maglana.

SYLLABUS

1. CIVIL LAW; DAMAGES; INSURANCE; AN INSURER IS SURROGATED TO THE RIGHTS OF THE


INSURED AGAINST THE WRONGDOER UPON RECEIPT OF THE INDEMNITY. — The petitioner's argument that the
respondents had no interest in the reinsurance contract as this is strictly between the petitioner as insured and the
reinsuring company pursuant to Section 91 (should be Section 98) of the Insurance Code has no basis. Under the
provisions of Article 2207 of the Civil Code if a property is insured and the owner receives the indemnity from the
insurer, the insurer is deemed subrogated to the rights of the insured against the wrongdoer and if the amount paid by
the insurer does not fully cover the loss, then the aggrieved party is the one entitled to recover the deficiency. Evidently,
under this legal provision, the real party in interest with regard to the portion of the indemnity paid is the insurer and not
the insured. (PAL v. Heald Lumber Co., 101 Phil. 1031; Manila Mahogany Manufacturing Corporation v. Court of
Appeals, 154 SCRA 650 [1987]
2. REMEDIAL LAW; ACTIONS; PARTIES; ONLY THE REISURER OF THE INSURER ACTING AS AN
ATTORNEY-IN-FACT OF THE REINSURER CAN COLLECT AGAINST THE INDEMNITY AGREEMENT. — The
appellate court did not commit a reversible error in dismissing the petitioner's complaint as against the respondents for
the reason that the petitioner was not the real party in interest in the complaint and, therefore, has no cause of action
against the respondents.
3. ID.; EVIDENCE; FINDINGS OF FACT OF THE TRIAL COURT UPHELD ON APPEAL. — We find the trial
court's findings on the matter replete with evidence to substantiate its finding that the counter-indemnitors are not liable
to the petitioner. Pioneer, having foreclosed the chattel mortgage on the planes and spare parts, no longer has any
further action against the defendants as indemnitors to recover any unpaid balance of the price. The indemnity
agreement was ipso jure extinguished upon the foreclosure of the chattel mortgage. These defendants, as indemnitors,
would be entitled to be subrogated to the right of Pioneer should they make payments to the latter. (Articles 2067 and
2080, New Civil Code)
4. CIVIL LAW; CONTRACTS; A DE FACTO PARTNERSHIP IS CREATED WHERE PERSONS ASSOCIATE
THEMSELVES BUT FAILED TO FORM A CORPORATION. — Where persons associate themselves together under
articles to purchase property to carry on a business, and their organization is so defective as to come short of creating a
corporation within the statute, they become in legal effect partners inter se, and their rights as members of the company
to the property acquired by the company will be recognized (Smith v. Schoodoc Pond Packing Co., 84 A 268, 109 Me.
555; Whipple v. Parker, 29 Mich. 369).
5. ID.; ID.; ID.; DOCTRINE NOT APPLICABLE WHERE THERE WAS REALLY NO INVENTION TO FORM A
CORPORATION; PARTIES NEED NOT SHARE IN LOSSES; CASE AT BAR. — The petitioner never had the intention
to form a corporation with the respondents despite his representations to them. This gives credence to the cross-claims
of the respondents to the effect that they were induced and lured by the petitioner to make contributions to a proposed
corporation which was never formed because the petitioner reneged on their agreement. Applying the principles of law
earlier cited to the facts of the case, necessarily, no de facto partnership was created among the parties which would
entitle the petitioner to a reimbursement of the supposed losses of the proposed corporation. The record shows that the
petitioner was acting on his own and not in behalf of his other would-be incorporators in transacting the sale of the
airplanes and spare parts.

DECISION

GUTIERREZ, JR., J p:
The subject matter of these consolidated petitions is the decision of the Court of Appeals in CA-G.R. CV No.
66195 which modified the decision of the then Court of First Instance of Manila in Civil Case No. 66135. The plaintiff's
complaint (petitioner in G.R. No. 84197) against all defendants (respondents in G.R. No. 84197) was dismissed but in
all other respects the trial court's decision was affirmed. LLpr
The dispositive portion of the trial court's decision reads as follows:
"WHEREFORE, judgment is rendered against defendant Jacob S. Lim requiring him to pay
plaintiff the amount of P311,056.02, with interest at the rate of 12% per annum compounded monthly;
plus 15% of the amount awarded to plaintiff as attorney's fees from July 2, 1966, until full payment is
made; plus P70,000.00 moral and exemplary damages.
"It is found in the records that the cross party plaintiffs incurred additional miscellaneous
expenses aside from P151,000.00, making a total of P184,878.74. Defendant Jacob S. Lim is further
required to pay cross party plaintiff, Bormaheco, the Cervanteses one-half and Maglana the other half,
the amount of P184,878.74 with interest from the filing of the cross-complaints until the amount is fully
paid; plus moral and exemplary damages in the amount of P184,878.84 with interest from the filing of
the cross-complaints until the amount is fully paid; plus moral and exemplary damages in the amount of
P50,000.00 for each of the two Cervanteses.
"Furthermore, he is required to pay P20,000.00 to Bormaheco and the Cervanteses, and
another P20,000.00 to Constancio B. Maglana as attorney's fees.
xxx xxx xxx
"WHEREFORE, in view of all above, the complaint of plaintiff Pioneer against defendants
Bormaheco, the Cervanteses and Constancio B. Maglana, is dismissed. Instead, plaintiff is required to
indemnify the defendants Bormaheco and the Cervanteses the amount of P20,000.00 as attorney's
fees and the amount of P4,379.21, per year from 1966 with legal rate of interest up to the time it is
paid.
"Furthermore, the plaintiff is required to pay Constancio B. Maglana the amount of P20,000.00
as attorney's fees and costs.
"No moral or exemplary damages is awarded against plaintiff for this action was filed in good
faith. The fact that the properties of the Bormaheco and the Cervanteses were attached and that they
were required to file a counterbond in order to dissolve the attachment, is not an act of bad faith. When
a man tries to protect his rights, he should not be saddled with moral or exemplary damages.
Furthermore, the rights exercised were provided for in the Rules of Court, and it was the court that
ordered it, in the exercise of its discretion.
"No damage is decided against Malayan Insurance Company, Inc., the third-party defendant,
for it only secured the attachment prayed for by the plaintiff Pioneer. If an insurance company would be
liable for damages in performing an act which is clearly within its power and which is the reason for its
being, then nobody would engage in the insurance business. No further claim or counter-claim for or
against anybody is declared by this Court." (Rollo — G.R. No. 24197, pp. 15-16)
In 1965, Jacob S. Lim (petitioner in G.R. No. 84157) was engaged in the airline business as owner-operator of
Southern Air Lines (SAL) a single proprietorship.
On May 17, 1965, at Tokyo, Japan, Japan Domestic Airlines (JDA) and Lim entered into and executed a sales
contract (Exhibit A) for the sale and purchase of two (2) DC-3A Type aircrafts and one (1) set of necessary spare parts
for the total agreed price of US $109,000.00 to be paid in installments. One DC-3 Aircraft with Registry No. PIC-718,
arrived in Manila on June 7, 1965 while the other aircraft, arrived in Manila on July 18, 1965.
On May 22, 1965, Pioneer Insurance and Surety Corporation (Pioneer, petitioner in G.R. No. 84197) as surety
executed and issued its Surety Bond No. 6639 (Exhibit C) in favor of JDA, in behalf of its principal, Lim, for the balance
price of the aircrafts and spare parts.
It appears that Border Machinery and Heavy Equipment Company, Inc. (Bormaheco), Francisco and Modesto
Cervantes (Cervanteses) and Constancio Maglana (respondents in both petitions) contributed some funds used in the
purchase of the above aircrafts and spare parts. The funds were supposed to be their contributions to a new corporation
proposed by Lim to expand his airline business. They executed two (2) separate indemnity agreements (Exhibits D-1
and D-2) in favor of Pioneer, one signed by Maglana and the other jointly signed by Lim for SAL, Bormaheco and the
Cervanteses. The indemnity agreements stipulated that the indemnitors principally agree and bind themselves jointly
and severally to indemnify and hold and save harmless Pioneer from and against any/all damages, losses, costs,
damages, taxes, penalties, charges and expenses of whatever kind and nature which Pioneer may incur in
consequence of having become surety upon the bond/note and to pay, reimburse and make good to Pioneer, its
successors and assigns, all sums and amounts of money which it or its representatives should or may pay or cause to
be paid or become liable to pay on them of whatever kind and nature.
On June 10, 1965, Lim doing business under the name and style of SAL executed in favor of Pioneer as deed
of chattel mortgage as security for the latter's suretyship in favor of the former. It was stipulated therein that Lim transfer
and convey to the surety the two aircrafts. The deed (Exhibit D) was duly registered with the Office of the Register of
Deeds of the City of Manila and with the Civil Aeronautics Administration pursuant to the Chattel Mortgage Law and the
Civil Aeronautics Law (Republic Act No. 776), respectively.
Lim defaulted on his subsequent installment payments prompting JDA to request payments from the surety.
Pioneer paid a total sum of P298,626.12.
Pioneer then filed a petition for the extrajudicial foreclosure of the said chattel mortgage before the Sheriff of
Davao City. The Cervanteses and Maglana, however, filed a third party claim alleging that they are co-owners of the
aircrafts.
On July 19, 1966, Pioneer filed an action for judicial foreclosure with an application for a writ of preliminary
attachment against Lim and respondents, the Cervanteses, Bormaheco and Maglana. cdll
In their Answers, Maglana, Bormaheco and the Cervanteses filed cross-claims against Lim alleging that they
were not privies to the contracts signed by Lim and, by way of counterclaim, sought for damages for being exposed to
litigation and for recovery of the sums of money they advanced to Lim for the purchase of the aircrafts in question.
After trial on the merits, a decision was rendered holding Lim liable to pay Pioneer but dismissed Pioneer's
complaint against all other defendants.
As stated earlier, the appellate court modified the trial court's decision in that the plaintiffs complaint against all
the defendants was dismissed. In all other respects the trial court's decision was affirmed.
We first resolve G.R. No. 84197.
Petitioner Pioneer Insurance and Surety Corporation avers that:
RESPONDENT COURT OF APPEALS GRIEVOUSLY ERRED WHEN IT DISMISSED THE
APPEAL OF PETITIONER ON THE SOLE GROUND THAT PETITIONER HAD ALREADY
COLLECTED THE PROCEEDS OF THE REINSURANCE ON ITS BOND IN FAVOR OF THE JDA
AND THAT IT CANNOT REPRESENT A REINSURER TO RECOVER THE AMOUNT FROM HEREIN
PRIVATE RESPONDENTS AS DEFENDANTS IN THE TRIAL COURT. (Rollo — G.R. No. 84197, p.
10)
The petitioner questions the following findings of the appellate court:
"We find no merit in plaintiffs appeal. It is undisputed that plaintiff Pioneer had reinsured its risk
of liability under the surety bond in favor of JDA and subsequently collected the proceeds of such
reinsurance in the sum of P295,000.00. Defendants' alleged obligation to Pioneer amounts to
P295,000.00, hence, plaintiff's instant action for the recovery of the amount of P298,666.28 from
defendants will no longer prosper. Plaintiff Pioneer is not the real party in interest to institute the instant
action as it does not stand to be benefited or injured by the judgment.
"Plaintiff Pioneer's contention that it is representing the reinsurer to recover the amount from
defendants, hence, it instituted the action is utterly devoid of merit. Plaintiff did not even present any
evidence that it is the attorney-in-fact of the reinsurance company, authorized to institute an action for
and in behalf of the latter. To qualify a person to be a real party in interest in whose name an action
must be prosecuted, he must appear to be the present real owner of the right sought to be enforced
(Moran, Vol. I, Comments on the Rules of Court, 1979 ed., p. 155.). It has been held that the real party
in interest is the party who would be benefited or injured by the judgment or the party entitled to the
avails of the suit (Salonga v. Warner Barnes & Co., Ltd., 88 Phil. 125, 131). By real party in interest is
meant a present substantial interest as distinguished from a mere expectancy or a future, contingent,
subordinate or consequential interest (Garcia v. David, 67 Phil. 27; Oglleaby v. Springfield Marine
Bank, 52 N.E. 2d 1600, 385 III, 414; Flowers v. Germana, 1 NW 2d 424; Weber v. City of Cheye, 97 P.
2d 667, 669, quoting 47 C.V. 35).
"Based on the foregoing premises, plaintiff Pioneer cannot be considered as the real party in
interest as it has already been paid by the reinsurer the sum of P295,000.00 — the bulk of defendants'
alleged obligation to Pioneer.
"In addition to the said proceeds of the reinsurance received by plaintiff Pioneer from its
reinsurer, the former was able to foreclose extra-judicially one of the subject airplanes and its spare
engine, realizing the total amount of P37,050.00 from the sale of the mortgaged chattels. Adding the
sum of P37,050.00, to the proceeds of the reinsurance amounting to P295,000.00, it is patent that
plaintiff has been overpaid in the amount of P33,383.72 considering that the total amount it had paid to
JDA totals to only P298,666.28. To allow plaintiff Pioneer to recover from defendants the amount in
excess of P298,666.28 would be tantamount to unjust enrichment as it has already been paid by the
reinsurance company of the amount plaintiff has paid to JDA as surety of defendant Lim vis-a-vis
defendant Lim's liability to JDA. Well settled is the rule that no person should unjustly enrich himself at
the expense of another (Article 22, New Civil Code)." (Rollo-84197, pp. 24-25).
The petitioner contends that — (1) it is at a loss where respondent court based its finding that petitioner was
paid by its reinsurer in the aforesaid amount, as this matter has never been raised by any of the parties herein both in
their answers in the court below and in their respective briefs with respondent court; (Rollo, p. 11) (2) even assuming
hypothetically that it was paid by its reinsurer, still none of the respondents had any interest in the matter since the
reinsurance is strictly between the petitioner and the re-insurer pursuant to section 91 of the Insurance Code; (3)
pursuant to the indemnity agreements, the petitioner is entitled to recover from respondents Bormaheco and Maglana;
and (4) the principle of unjust enrichment is not applicable considering that whatever amount he would recover from the
co-indemnitor will be paid to the reinsurer.
The records belie the petitioner's contention that the issue on the reinsurance money was never raised by the
parties.
A cursory reading of the trial court's lengthy decision shows that two of the issues threshed out were:
xxx xxx xxx
"1. Has Pioneer a cause of action against defendants with respect to so much of its obligations
to JDA as has been paid with reinsurance money?
2. If the answer to the preceding question is in the negative, has Pioneer still any claim against
defendants, considering the amount it has realized from the sale of the mortgaged properties? (Record
on Appeal, p. 359, Annex B of G.R. No. 84157).
In resolving these issues, the trial court made the following findings:
"It appearing that Pioneer reinsured its risk of liability under the surety bond it had executed in
favor of JDA, collected the proceeds of such reinsurance in the sum of P295,000, and paid with the
said amount the bulk of its alleged liability to JDA under the said surety bond, it is plain that on this
score it no longer has any right to collect to the extent of the said amount.
On the question of why it is Pioneer, instead of the reinsurance (sic), that is suing defendants
for the amount paid to it by the reinsurers, notwithstanding that the cause of action pertains to the
latter, Pioneer says: 'The reinsurers opted instead that the Pioneer Insurance & Surety Corporation
shall pursue alone the case.' '. . . . Pioneer Insurance & Surety Corporation is representing the
reinsurers to recover the amount.' In other words, insofar as the amount paid to it by the reinsurers
Pioneer is suing defendants as their attorney-in-fact.
But in the first place, there is not the slightest indication in the complaint that Pioneer is suing
as attorney-in-fact of the reinsurers for any amount. Lastly, and most important of all, Pioneer has no
right to institute and maintain in its own name an action for the benefit of the reinsurers. It is well-settled
that an action brought by an attorney-in-fact in his own name instead of that of the principal will not
prosper, and this is so even where the name of the principal is disclosed in the complaint.
"'Section 2 of Rule 3 of the Old Rules of Court provides that 'Every action must be
prosecuted in the name of the real party in interest.' This provision is mandatory. The real party
in interest is the party who would be benefited or injured by the judgment or is the party entitled
to the avails of the suit.
"'This Court has held in various cases that an attorney-in-fact is not a real party in
interest, that there is no law permitting an action to be brought by an attorney-in-fact. Arroyo v.
Granada and Gentero, 18 Phil. Rep. 484; Luchauco v. Limjuco and Gonzalo, 19 Phil. Rep. 12;
Filipinas Industrial Corporation v. San Diego G.R. No. L-22347, 1968, 23 SCRA 706, 710-714.'"
"The total amount paid by Pioneer to JDA is P299,666.29. Since Pioneer has collected
P295,000.00 from the reinsurers, the uninsured portion of what it paid to JDA is the difference between
the two amounts, or P3,666.28. This is the amount for which Pioneer may sue defendants, assuming
that the indemnity agreement is still valid and effective. But since the amount realized from the sale of
the mortgaged chattels are P35,000.00 for one of the airplanes and P2,050.00 for a spare engine, or a
total of P37,050.00, Pioneer is still overpaid by P33,383.72. Therefore, Pioneer has no more claim
against defendants."' (Record on Appeal, pp. 360-363).
The payment to the petitioner made by the reinsurers was not disputed in the appellate court. Considering this
admitted payment, the only issue that cropped up was the effect of payment made by the reinsurers to the petitioner.
Therefore, the petitioner's argument that the respondents had no interest in the reinsurance contract as this is strictly
between the petitioner as insured and the reinsuring company pursuant to Section 91 (should be Section 98) of the
Insurance Code has no basis.
"In general a reinsurer, on payment of a loss acquires the same rights by subrogation as are
acquired in similar cases where the original insurer pays a loss (Universal Ins. Co. v. Old Time
Molasses Co. C.C.A. La., 46 F 2nd 925).
"The rules of practice in actions on original insurance policies are in general applicable to
actions or contracts of reinsurance. (Delaware, Ins. Co. v. Pennsylvania Fire Ins. Co., 55 S.E. 330, 126
GA. 380, 7 Ann. Con. 1134)".
Hence the applicable law is Article 2207 of the new Civil Code, to wit:
"Art. 2207. If the plaintiffs property has been insured, and he has received indemnity from the
insurance company for the injury or loss arising out of the wrong or breach of contract complained of,
the insurance company shall be subrogated to the rights of the insured against the wrongdoer or the
person who has violated the contract. If the amount paid by the insurance company does not fully cover
the injury or loss, the aggrieved party shall be entitled to recover the deficiency from the person
causing the loss or injury."
Interpreting the aforesaid provision, we ruled in the case of Phil. Air Lines, Inc. v. Heald Lumber Co. (101 Phil.
1031 [1957]) which we subsequently applied in Manila Mahogany Manufacturing Corporation v. Court of Appeals (154
SCRA 650 [1987]):.
"Note that if a property is insured and the owner receives the indemnity from the insurer, it is
provided in said article that the insurer is deemed subrogated to the rights of the insured against the
wrongdoer and if the amount paid by the insurer does not fully cover the loss, then the aggrieved party
is the one entitled to recover the deficiency. Evidently, under this legal provision, the real party in
interest with regard to the portion of the indemnity paid is the insurer and not the insured." (Emphasis
supplied).
It is clear from the records that Pioneer sued in its own name and not as an attorney-in-fact of the reinsurer.
Accordingly, the appellate court did not commit a reversible error in dismissing the petitioner's complaint as
against the respondents for the reason that the petitioner was not the real party in interest in the complaint and,
therefore, has no cause of action against the respondents.
Nevertheless, the petitioner argues that the appeal as regards the counter indemnitors should not have been
dismissed on the premise that the evidence on record shows that it is entitled to recover from the counter indemnitors. It
does not, however, cite any grounds except its allegation that respondent "Maglana's defense and evidence are
certainly incredible" (p. 12, Rollo) to back up its contention.
On the other hand, we find the trial court's findings on the matter replete with evidence to substantiate its
finding that the counter-indemnitors are not liable to the petitioner. The trial court stated:
"Apart from the foregoing proposition, the indemnity agreement ceased to be valid and
effective after the execution of the chattel mortgage.
"Testimonies of defendants Francisco Cervantes and Modesto Cervantes.
"Pioneer Insurance, knowing the value of the aircrafts and the spare parts involved, agreed to
issue the bond provided that the same would be mortgaged to it, but this was not possible because the
planes were still in Japan and could not be mortgaged here in the Philippines. As soon as the aircrafts
were brought to the Philippines, they would be mortgaged to Pioneer Insurance to cover the bond, and
this indemnity agreement would be cancelled.
"The following is averred under oath by Pioneer in the original complaint:
"'The various conflicting claims over the mortgaged properties have impaired and
rendered insufficient the security under the chattel mortgage and there is thus no other sufficient
security for the claim sought to be enforced by this action.'"
"This is judicial admission and aside from the chattel mortgage there is no other security for the
claim sought to be enforced by this action, which necessarily means that the indemnity agreement had
ceased to have any force and effect at the time this action was instituted. Sec 2, Rule 129, Revised
Rules of Court.
"Prescinding from the foregoing, Pioneer, having foreclosed the chattel mortgage on the planes
and spare parts, no longer has any further action against the defendants as indemnitors to recover any
unpaid balance of the price. The indemnity agreement was ipso jure extinguished upon the foreclosure
of the chattel mortgage. These defendants, as indemnitors, would be entitled to be subrogated to the
right of Pioneer should they make payments to the latter. Articles 2067 and 2080 of the New Civil Code
of the Philippines.
Independently of the preceding proposition Pioneer's election of the remedy of foreclosure
precludes any further action to recover any unpaid balance of the price.
SAL or Lim, having failed to pay the second to the eight and last installments to JDA and
Pioneer as surety having made of the payments to JDA, the alternative remedies open to Pioneer were
as provided in Article 1484 of the New Civil Code, known as the Recto Law.
Pioneer exercised the remedy of foreclosure of the chattel mortgage both by extrajudicial
foreclosure and the instant suit. Such being the case, as provided by the aforementioned provisions,
Pioneer 'shall have no further action against the purchaser to recover any unpaid balance and any
agreement to the contrary is void.' Cruz, et al. v. Filipinas Investment & Finance Corp. No. L-24772,
May 27, 1968, 23 SCRA 791, 795-6.
The operation of the foregoing provision cannot be escaped from through the contention that
Pioneer is not the vendor but JDA. The reason is that Pioneer is actually exercising the rights of JDA as
vendor, having subrogated it in such rights. Nor may the application of the provision be validly opposed
on the ground that these defendants and defendant Maglana are not the vendee but indemnitors.
Pascual, et al. v. Universal Motors Corporation, G.R. No. L-27862, Nov. 20, 1974, 61 SCRA 124.
The restructuring of the obligations of SAL or Lim, thru the change of their maturity dates
discharged these defendants from any liability as alleged indemnitors. The change of the maturity
dates of the obligations of Lim, or SAL, extinguished the original obligations thru novations, thus
discharging the indemnitors.
"'The principal hereof shall be paid in eight equal successive three months interval
installments, the first of which shall be due and payable 25 August 1965, the remainder of
which . . . shall be due and payable on the 26th day . . . of each succeeding three months and
the last of which shall be due and payable 26th May 1967.'"
"However, at the trial of this case, Pioneer produced a memorandum executed by SAL, or Lim
and JDA, modifying the maturity dates of the obligations, as follows:
"'The principal hereof shall be paid in eight equal successive three month interval
installments the first of which shall be due and payable 4 September 1965, the remainder of
which . . . shall be due and payable on the 4th day . . . of each succeeding months and the last of
which shall be due and payable 4th June 1967.'"
"Not only that, Pioneer also produced eight purported promissory notes bearing maturity dates
different from that fixed in the aforesaid memorandum; the due date of the first installment appears as
October 15, 1965, and those of the rest of the installments, the 15th of each succeeding three months,
that of the last installment being July 15, 1967.
"These restructuring of the obligations with regard to their maturity dates, effected twice, were
done without the knowledge, much less, would have it believed that these defendants Maglana (sic).
Pioneer's official Numeriano Carbonel, would have it believed that these defendants and defendant
Maglana knew of and consented to the modification of the obligations. But if that were so, there would
have been the corresponding documents in the form of a written notice to as well as written conformity
of these defendants, and there are no such document. The consequence of this was the
extinguishment of the obligations and of the surety bond secured by the indemnity agreement which
was thereby also extinguished. Applicable by analogy are the rulings of the Supreme Court in the case
of Kabankalan Sugar Co. v. Pacheco, 55 Phil. 553, 563, and the case of Asiatic Petroleum Co. v. Hizon
David, 45 Phil. 532, 538.
"'Art. 2079. An extension granted to the debtor by the creditor without the consent of the
guarantor extinguishes the guaranty. The mere failure on the part of the creditor to demand
payment after the debt has become due does not of itself constitute any extension of time
referred to herein, (New Civil Code).'"
"Manresa, 4th ed., Vol. 12, pp. 316-317, Vol. VI, pp. 562-563, M.F. Stevenson & Co., Ltd., v.
Climacom et al. (C.A.) 36 O.G. 1571.
"Pioneer's liability as surety to JDA had already prescribed when Pioneer paid the same.
Consequently, Pioneer has no more cause of action to recover from these defendants, as supposed
indemnitors what it has paid to JDA. By virtue of an express stipulation in the surety bond, the failure of
JDA to present its claim to Pioneer within ten days from default of Lim or SAL on every installment,
released Pioneer from liability from the claim.
"Therefore, Pioneer is not entitled to exact reimbursement from these defendants thru the
indemnity.
"'Art. 1318. Payment by a solidary debtor shall not entitle him to reimbursement from his
co-debtors if such payment is made after the obligation has prescribed or became illegal.'"
"These defendants are entitled to recover damages and attorney's fees from Pioneer and its
surety by reason of the filing of the instant case against them and the attachment and garnishment of
their properties. The instant action is clearly unfounded insofar as plaintiff drags these defendants and
defendant Maglana." (Record on Appeal, pp. 363-369, Rollo of G.R. No. 84157).
We find no cogent reason to reverse or modify these findings.
Hence, it is our conclusion that the petition in G.R. No. 84197 is not meritorious.
We now discuss the merits of G.R. No. 84157.
Petitioner Jacob S. Lim poses the following issues:
"1. What legal rules govern the relationship among co-investors whose agreement was to do
business through the corporate vehicle but who failed to incorporate the entity in which they had
chosen to invest? How are the losses to be treated in situations where their contributions to the
intended 'corporation' were invested not through the corporate form? This Petition presents these
fundamental questions which we believe were resolved erroneously by the Court of Appeals ('CA')."
(Rollo, p. 6).
These questions are premised on the petitioner's theory that as a result of the failure of respondents
Bormaheco, Spouses Cervantes, Constancio Maglana and petitioner Lim to incorporate, a de facto partnership among
them was created, and that as a consequence of such relationship all must share in the losses and/or gains of the
venture in proportion to their contribution. The petitioner, therefore, questions the appellate court's findings ordering him
to reimburse certain amounts given by the respondents to the petitioner as their contributions to the intended
corporation, to wit:
"However, defendant Lim should be held liable to pay his co-defendants' cross-claims in the
total amount of P184,878.74 as correctly found by the trial court, with the interest from the filing of the
cross-claims until the amount is fully paid. Defendants Lim should pay one-half of the said amount to
Bormaheco and the Cervanteses and the other one-half to defendant Maglana. It is established in the
records that defendant Lim had duly received the amount of P151,000.00 from defendants Bormaheco
and Maglana representing the latter's participation in the ownership of the subject airplanes and spare
parts (Exhibit 58). In addition, the cross-party plaintiffs incurred additional expenses, hence, the total
sum of P184,878.74."
We first state the principles.
"While it has been held that as between themselves the rights of the stockholders in a
defectively incorporated association should be governed by the supposed charter and the laws of the
state relating thereto and not by the rules governing partners (Cannon v. Brush Electric Co., 54 A. 121,
96 Md. 446, 94 Am. S.R. 584), it is ordinarily held that persons who attempt, but fail, to form a
corporation and who carry on business under the corporate name occupy a position of partners inter se
(Lynch v. Perryman, 119 P. 229, 29 Okl. 615, Ann. Cas. 1913A 1065). Thus, where persons associate
themselves together under articles to purchase property to carry on a business, and their organization
is so defective as to come short of creating a corporation within the statute, they become in legal effect
partners inter se, and their rights as members of the company to the property acquired by the company
will be recognized (Smith v. Schoodoc Pond Packing Co., 84 A 268, 109 Me. 555; Whipple v. Parker,
29 Mich. 369). So, where certain persons associated themselves as a corporation for the development
of land for irrigation purposes, and each conveyed land to the corporation, and two of them contracted
to pay a third the difference in the proportionate value of the land conveyed by him, and no stock was
ever issued in the corporation, it was treated as a trustee for the associates in an action between them
for an accounting, and its capital stock was treated as partnership assets, sold, and the proceeds
distributed among them in proportion to the value of the property contributed by each (Shorb v.
Beaudry, 56 Cal. 446). However, such a relation does not necessarily exist, for ordinarily persons
cannot be made to assume the relation of partners, as between themselves, when their purpose is that
no partnership shall exist (London Assur. Corp. v. Drennen, Minn., 6 S. Ct. 442, 116 U.S. 461, 472, 29
L.Ed. 688), and it should be implied only when necessary to do justice between the parties; thus, one
who takes no part except to subscribe for stock in a proposed corporation which is never legally formed
does not become a partner with other subscribers who engage in business under the name of the
pretended corporation, so as to be liable as such in an action for settlement of the alleged partnership
and contribution (Ward v. Brigham, 127 Mass. 24). A partnership relation between certain stockholders
and other stockholders, who were also directors, will not be implied in the absence of an agreement, so
as to make the former liable to contribute for payment of debts illegally contracted by the latter (Heald
v. Owen, 44 N.W. 210, 79 Iowa 23). (Corpus Juris Secundum, Vol. 68, p. 464). (Emphasis supplied).
In the instant case, it is to be noted that the petitioner was declared non-suited for his failure to appear during
the pre-trial despite notification. In his answer, the petitioner denied having received any amount from respondents
Bormaheco, the Cervanteses and Maglana. The trial court and the appellate court, however, found through Exhibit 58,
that the petitioner received the amount of P151,000.00 representing the participation of Bormaheco and Atty.
Constancio B. Maglana in the ownership of the subject airplanes and spare parts. The record shows that defendant
Maglana gave P75,000.00 to petitioner Jacob Lim thru the Cervanteses. LexLib
It is therefore clear that the petitioner never had the intention to form a corporation with the respondents despite
his representations to them. This gives credence to the cross-claims of the respondents to the effect that they were
induced and lured by the petitioner to make contributions to a proposed corporation which was never formed because
the petitioner reneged on their agreement. Maglana alleged in his cross-claim:
". . . that sometime in early 1965, Jacob Lim proposed to Francisco Cervantes and Maglana to
expand his airline business. Lim was to procure two DC-3's from Japan and secure the necessary
certificates of public convenience and necessity as well as the required permits for the operation
thereof. Maglana sometime in May 1965, gave Cervantes his share of P75,000.00 for delivery to Lim
which Cervantes did and Lim acknowledged receipt thereof Cervantes, likewise, delivered his share of
the undertaking. Lim in an undertaking sometime on or about August 9, 1965, promised to incorporate
his airline in accordance with their agreement and proceeded to acquire the planes on his own account.
Since then up to the filing of this answer, Lim has refused, failed and still refuses to set up the
corporation or return the money of Maglana."
(Record on Appeal, pp. 337-338).
while respondents Bormaheco and the Cervanteses alleged in their answer, counterclaim, cross-claim and third party
complaint:
"Sometime in April 1965, defendant Lim lured and induced the answering defendants to
purchase two airplanes and spare parts from Japan which the latter considered as their lawful
contribution and participation in the proposed corporation to be known as SAL. Arrangements and
negotiations were undertaken by defendant Lim. Down payments were advanced by defendants
Bormaheco and the Cervanteses and Constancio Maglana (Exh. E-1). Contrary to the agreement
among the defendants, defendant Lim in connivance with the plaintiff, signed and executed the alleged
chattel mortgage and surety bond agreement in his personal capacity as the alleged proprietor of the
SAL. The answering defendants learned for the first time of this trickery and misrepresentation of the
other, Jacob Lim, when the herein plaintiff chattel mortgage (sic) allegedly executed by defendant Lim,
thereby forcing them to file an adverse claim in the form of third party claim. Notwithstanding repeated
oral demands made by defendants Bormaheco and Cervanteses, to defendant Lim, to surrender the
possession of the two planes and their accessories and or return the amount advanced by the former
amounting to an aggregate sum of P178,997.14 as evidenced by a statement of accounts, the latter
ignored, omitted and refused to comply with them." (Record on Appeal, pp. 341-342).
Applying therefore the principles of law earlier cited to the facts of the case, necessarily, no de facto partnership
was created among the parties which would entitle the petitioner to a reimbursement of the supposed losses of the
proposed corporation. The record shows that the petitioner was acting on his own and not in behalf of his other would-
be incorporators in transacting the sale of the airplanes and spare parts. LLjur
WHEREFORE, the instant petitions are DISMISSED. The questioned decision of the Court of Appeals is
AFFIRMED. SO ORDERED.
||| (Pioneer Insurance & Surety Corp. v. Court of Appeals, G.R. Nos. 84197 & 84157, [July 28, 1989], 256 PHIL 1061-1078)

FORMATION AND ORGANIZATION OF CORPORATION

[G.R. No. 96161. February 21, 1992.]

PHILIPS EXPORT B.V., PHILIPS ELECTRICAL LAMPS, INC. and PHILIPS INDUSTRIAL
DEVELOPMENT, INC., petitioners, vs. COURT OF APPEALS, SECURITIES & EXCHANGE
COMMISSION and STANDARD PHILIPS CORPORATION, respondents.

Emeterio V. Soliven & Associates for petitioners.


Narciso A. Manantan for private respondent.

SYLLABUS

1. COMMERCIAL LAW; CORPORATION CODE; SECTION 18 THEREOF APPLICABLE ONLY WHEN CORPORATE
NAMES ARE IDENTICAL. — Section 18 of the Corporation Code is applicable only when the corporate names in question
are identical. In the instant case, there is no confusing similarity between Petitioners' and Private Respondent's corporate
names as those of the Petitioners contain at least two words different from that of the Respondent.
2. ID.; CORPORATION; RIGHT TO USE ITS CORPORATE AND TRADE NAME, A PROPERTY RIGHT. — As early as
Western Equipment and Supply Co. v. Reyes, 51 Phil. 115 (1927), the Court declared that a corporation's right to use its
corporate and trade name is a property right, a right in rem, which it may assert and protect against the world in the same
manner as it may protect its tangible property, real or personal, against trespass or conversion. It is regarded, to a certain
extent, as a property right and one which cannot be impaired or defeated by subsequent appropriation by another
corporation in the same field (Red Line Transportation Co. vs. Rural Transit co., September 6, 1934, 60 Phil. 549).
3. ID.; ID.; IMPORTANCE OF CORPORATE NAME. — A name is peculiarly important as necessary to the very existence of
a corporation. Its name is one of its attributes, an element of its existence, and essential to its identity (6 Fletcher [Perm Ed],
pp. 3-4). The general rule as to corporations is that each corporation must have a name by which it is to sue and be sued
and do all legal acts. The name of a corporation in this respect designates the corporation in the same manner as the name
of an individual designates the person (Cincinnati Cooperage Co. vs. Bate, 96 Ky 356, 26 SW 538; Newport Mechanics Mfg.
Co. vs. Starbird, 10 NH 123); and the right to use its corporate name is as much a part of the corporate franchise as any
other privilege granted.
4. ID.; ID.; CORPORATE NAME DISTINGUISHED FROM INDIVIDUAL'S NAME. — A corporation acquires its name by
choice and need not select a name identical with or similar to one already appropriated by a senior corporation while an
individual's name is thrust upon him (See Standard Oil Co. of New Mexico, Inc. v. Standard Oil Co. of California, 56 F 2d
973, 977). A corporation can no more use a corporate name in violation of the rights of others that an individual can use his
name legally acquired so as to mislead the public and injure another (Armington vs. Palmer, 21 RI 109, 42 A 308).
5. ID.; ID.; ID.; STATUTORY PROHIBITION PROVIDED IN SEC. 18 OF CORPORATION CODE; REQUISITES. — Our
own Corporation Code, in its Section 18, expressly provides that: "No corporate name may be allowed by the Securities and
Exchange Commission if the proposed name is identical or deceptively or confusingly similar to that of any existing
corporation or to any other name already protected by law or is patently deceptive, confusing or contrary to existing law.
Where a change in the corporate name is approved, the commission shall issue an amended certificate of incorporation
under the amended name." (Emphasis supplied) The statutory prohibition cannot be any clearer. To come within its scope,
two requisites must be proven, namely: (1) that the complainant corporation acquired a prior right over the use of such
corporate name; and (2) the proposed name is either: (a) identical or (b) deceptively or confusingly similar to that of any
existing corporation or to any other name already protected by law; or (c) patently deceptive, confusing or contrary to
existing law.
6. ID.; ID.; ID.; RIGHT TO EXCLUSIVE USE OF CORPORATE NAME DETERMINED BY PRIORITY OF ADOPTION;
APPLIED IN CASE AT BAR. — The right to the exclusive use of a corporate name with freedom from infringement by
similarity is determined by priority of adoption (1 Thompson, p.80 citing Munn v. Americana Co., 82 N., Eq. 63 88 Atl. 30;
San Francisco Oyster House v. Mihich, 75 Wash. 274, 134 Pac. 921). In this regard, there is no doubt with respect to
Petitioners' prior adoption of the name "PHILIPS" as part of its corporate name. Petitioners Philips Electrical and Philips
Industrial were incorporated on 29 August 1956 and 25 May 1956, respectively, while Respondent Standard Philips was
issued a Certificate of Registration on 19 April 1982, twenty-six (26) years later (Rollo, p.16). Petitioner PEBV has also used
the trademark "PHILIPS" on electrical lamps of all types and their accessories since 30 September 1922, as evidenced by
Certificate of Registration No. 1651.
7. ID.; ID.; ID.; TEST IN DETERMINING EXISTENCE OF CONFUSING SIMILARITY; PROOF OF ACTUAL CONFUSION
NOT NECESSARY. — In determining the existence of confusing similarity in corporate names, the test is whether the
similarity is such as to mislead a person using ordinary care and discrimination. In so doing, the Court must look to the
record as well as the names themselves (Ohio Nat. Life Ins. Co. v. Ohio Life Ins. Co., 210 NE 2d 298). It is settled, however,
that proof of actual confusion need not be shown. It suffices that confusion is probably or likely to occur (6 Fletcher [Perm
Ed], pp. 107-108, enumerating a long line of cases).
8. ID.; ID.; ID.; INTENT OF SUBSEQUENT APPROPRIATOR OF NAME. — Petitioners pointed out that "[p]rivate
respondent's choice of 'PHILIPS' as part of its corporate name [STANDARD PHILIPS CORPORATION] . . . tends to show
said respondent's intention to ride on the popularity and established goodwill of said petitioner's business throughout the
world." The subsequent appropriator of the name or one confusingly similar thereto usually seeks an unfair advantage, a
free ride on another's goodwill (American Gold Star Mothers, Inc. v. National Gold Star Mothers, Inc., et al, 89 App DC 269,
191 F 2d 488).
9. ID.; ID.; ID.; RULE ON PROPOSED NAME. — True, under the Guidelines in the Approval of Corporate and Partnership
Names formulated by the SEC, the proposed name "should not be similar to one already used by another corporation or
partnership. If the proposed name contains a word already used as part of the firm name or style of a registered company,
the proposed name must contain two other words different from the company already registered." It is then pointed out that
Petitioners Philips Electrical and Philips Industrial have two words different from that of Private Respondent's name.
10. ID.; ID.; ID.; CORPORATION HAS EXCLUSIVE RIGHT TO THE USE OF ITS NAME WHICH MAY BE PROTECTED
BY INJUNCTION; BASIS FOR SUCH PRINCIPLE. — A corporation has an exclusive right to the use of its name, which
may be protected by injunction upon a principle similar to that upon which persons are protected in the use of trademarks
and tradenames (18 C.J.S. 574). Such principle proceeds upon the theory that it is a fraud on the corporation which has
acquired a right to that name and perhaps carried on its business thereunder, that another should attempt to use the same
name, or the same name with a slight variation in such a way as to induce persons to deal with it in the belief that they are
dealing with the corporation which has given a reputation to the name (6 Fletcher [Perm Ed.], pp. 39-40, citing Borden Ice
Cream Co. v. Borden's Condensed Milk Co., 210 F 510).

DECISION

MELENCIO-HERRERA, J p:

Petitioners challenge the Decision of the Court of Appeals, dated 31 July 1990, in CA-GR Sp. No. 20067, upholding the
Order of the Securities and Exchange Commission, dated 2 January 1990, in SEC-AC No. 202, dismissing petitioners'
prayer for the cancellation or removal of the word "PHILIPS" for private respondent's corporate name.
Petitioner Philips Export B.V. (PEBV), a foreign corporation organized under the laws of the Netherlands, although not
engaged in business here, is the registered owner of the trademarks PHILIPS and PHILIPS SHIELD EMBLEM under
Certificate of Registration Nos. R-1641 and R-1674, respectively issued by the Philippine Patent Office (presently known as
the Bureau of Patents, Trademarks and Technology Transfer). Petitioners Philips Electrical Lamps, Inc. (Philips Electrical,
for brevity) and Philips Industrial Development, Inc. (Philips Industrial, for short), authorized users of the trademarks
PHILIPS and PHILIPS SHIELD EMBLEM, were incorporated on 29 August 1956 and 25 may 1956, respectively. All
petitioner corporations belong to the PHILIPS Group of Companies.
Respondent Standard Philips Corporation (Standard Philips), on the other hand, was issued a Certificate of Registration by
respondent Commission on 19 May 1982.
On 24 September 1984, Petitioners filed a letter complaint with the Securities & Exchange Commission (SEC) asking for the
cancellation of the word "PHILIPS" from Private Respondent's corporate name in view of the prior registration with the
Bureau of Patents of the trademark "PHILIPS" and the logo "PHILIPS SHIELD EMBLEM" in the name of Petitioner PEBV,
and the previous registration of Petitioners Philips Electrical and Philips Industrial with the SEC.
As a result of Private Respondent's refusal to amend its Articles of Incorporation, Petitioners filed with the SEC, on 6
February 1985, a Petition (SEC Case No. 2743), praying for the issuance of a Writ of Preliminary Injunction, alleging,
among others, that Private Respondent's use of the word PHILIPS amounts to an infringement and clear violation of
Petitioner's exclusive right to use the same considering that both parties engage in the same business.
In its Answer, dated 7 March 1985, Private Respondent countered that Petitioner PEBV has no legal capacity to sue; that its
use of its corporate name is not at all similar to Petitioners' trademark PHILIPS when considered in its entirety; and that its
products consisting of chain rollers, belts, bearings and cutting saw are grossly different from Petitioners' electrical products.

After conducting hearings with respect to the prayer for Injunction, the SEC Hearing Officer, on 27 September 1985, ruled
against the issuance of such Writ.
On 30 January 1987, the same Hearing Officer dismissed the Petition for lack of merit. In so ruling, the latter declared that
inasmuch as the SEC found no sufficient ground for the granting of injunctive relief on the basis of the testimonial and
documentary evidence presented, it cannot order the removal or cancellation of the word "PHILIPS" from Private
Respondent's corporate name on the basis of the same evidence adopted in toto during trial on the merits. Besides, Section
18 of the Corporation Code (infra) is applicable only when the corporate names in question are identical. Here, there is no
confusing similarity between Petitioners' and Private Respondent's corporate names as those of the Petitioners contain at
least two words different from that of the Respondent. Petitioners' Motion for Reconsideration was likewise denied on 17
June 1987. LibLex
On appeal, the SEC en banc affirmed the dismissal declaring that the corporate names of Petitioners and Private
Respondent hardly breed confusion inasmuch as each contains at least two different words and, therefore, rules out any
possibility of confusing one for the other.
On 30 January 1990, Petitioners sought an extension of time to file a Petition for Review on Certiorari before this Court,
which Petition was later referred to the Court of Appeals in a Resolution dated 12 February 1990.
In deciding to dismiss the petition on 31 July 1990, the Court of Appeals 1 swept aside Petitioners' claim that following the
ruling in Converse Rubber Corporation v. Universal Converse Rubber Products, Inc., et al, (G.R. No. L-27906, January 8,
1987, 147 SCRA 154), the word PHILIPS cannot be used as part of Private Respondent's corporate name as the same
constitutes a dominant part of Petitioners' corporate names. In so holding, the Appellate Court observed that the Converse
case is not four-square with the present case inasmuch as the contending parties in Converse are engaged in a similar
business, that is, the manufacture of rubber shoes. Upholding the SEC, the Appellate Court concluded that "private
respondent's products consisting of chain rollers, belts, bearings and cutting saw are unrelated and non-competing with
petitioners' products i.e. electrical lamps such that consumers would not in any probability mistake one as the source or
origin of the product of the other."
The Appellate Court denied Petitioners' Motion for Reconsideration on 20 November 1990, hence, this Petition which was
given due course on 22 April 1991, after which the parties were required to submit their memoranda, the latest of which was
received on 2 July 1991. In December 1991, the SEC was also required to elevate its records for the perusal of this Court,
the same not having been apparently before respondent Court of Appeals.
We find basis for petitioners' plea.
As early as Western Equipment and Supply Co. v. Reyes, 51 Phil. 115 (1927), the Court declared that a corporation's right
to use its corporate and trade name is a property right, a right in rem, which it may assert and protect against the world in
the same manner as it may protect its tangible property, real or personal, against trespass or conversion. It is regarded, to a
certain extent, as a property right and one which cannot be impaired or defeated by subsequent appropriation by another
corporation in the same field (Red Line Transportation Co. vs. Rural Transit Co., September 6, 1934, 60 Phil 549).
A name is peculiarly important as necessary to the very existence of a corporation (American Steel Foundries vs.
Robertson, 269 US 372, 70 L ed 317, 46 S Ct 160; Lauman vs. Lebanon Valley R. Co., 30 Pa 42; First National Bank vs.
Huntington Distilling Co, 40 W Va 530, 23 SE 792). Its name is one of its attributes, an element of its existence, and
essential to its identity (6 Fletcher [Perm Ed], pp. 3-4). The general rule as to corporations is that each corporation must
have name by which it is to sue and be sued and do all legal acts. The name of a corporation in this respect designates the
corporation in the same manner as the name of an individual designates the person (Cincinnati Cooperage Co. vs. Bate, 96
Ky 356, 26 SW 538; Newport Mechanics Mfg. Co. vs. Starbird, 10 NH 123); and the right to use its corporate name is as
much a part of the corporate franchise as any other privilege granted (Federal Secur. Co. vs. Federal Secur. Corp., 129 Or
375, 276 P 1100, 66 ALR 934; Paulino vs. Portuguese Beneficial Association, 18 RI 165, 26 A 36). Cdpr
A corporation acquires its name by choice and need not select a name identical with or similar to one already appropriated
by a senior corporation while an individual's name is thrust upon him (See Standard Oil Co. of New Mexico, Inc. v. Standard
Oil Co. of California, 56 F 2d 973, 977). A corporation can no more use a corporate name in violation of the rights of others
than an individual can use his name legally acquired so as to mislead the public and injure another (Armington vs. Palmer,
21 RI 109, 42 A 308).
Our own Corporation Code, in its Section 18, expressly provides that:
"No corporate name may be allowed by the Securities and Exchange Commission if the proposed name
is identical or deceptively or confusingly similar to that of any existing corporation or to any other name
already protected by law or is patently deceptive, confusing or contrary to existing law. Where a change
in the corporate name is approved, the commission shall issue an amended certificate of incorporation
under the amended name." (Emphasis supplied).
The statutory prohibition cannot be any clearer. To come within its scope, two requisites must be proven, namely:
(1) that the complainant corporation acquired a prior right over the use of such corporate name; and
(2) the proposed name is either:
(a) identical or
(b) deceptively or confusingly similar
to that of any existing corporation or to any other name already protected by law; or
(c) patently deceptive, confusing or contrary to existing law.
The right to the exclusive use of a corporate name with freedom from infringement by similarity is determined by priority of
adoption (1 Thomson, p.80 citing Munn v. Americana Co., 82 N., Eq. 63, 88 Atl. 30; San Francisco Oyster House v. Mihich,
75 Wash, 274, 134 Pac. 921). In this regard, there is no doubt with respect to Petitioners' prior adoption of the name
"PHILIPS" as part of its corporate name. Petitioners Philips Electrical and Philips Industrial were incorporated on 29 August
1956 and 25 May 1956, respectively, while Respondent Standard Philips was issued a Certificate of Registration on 19 April
1982, twenty-six (26) years later (Rollo, p.16). Petitioner PEBV has also used the trademark "PHILIPS" on electrical lamps
of all types and their accessories since 30 September 1922, as evidenced by Certificate of Registration No. 1651.
The second requisite no less exists in this case. In determining the existence of confusing similarity in corporate names, the
test is whether the similarity is such as to mislead a person using ordinary care and discrimination. In so doing, the Court
must look to the record as well as the names themselves (Ohio Nat. Life Ins. Co. v. Ohio Life Ins. Co., 210 NE 2d 298).
While the corporate names of Petitioners and Private Respondent are not identical, a reading of Petitioner's corporate
names, to wit: PHILIPS EXPORT B.V., PHILIPS ELECTRICAL LAMPS, INC. and PHILIPS INDUSTRIAL DEVELOPMENT,
INC., inevitably leads one to conclude that "PHILIPS" is, indeed, the dominant word in that all the companies affiliated or
associated with the principal corporation, PEBV, are known in the Philippines and abroad as the PHILIPS Group of
Companies. cdll
Respondents maintain, however, that Petitioners did not present an iota of proof of actual confusion or deception of the
public much less a single purchaser or their product who has been deceived or confused or showed any likelihood of
confusion. It is settled, however, that proof of actual confusion need not be shown. It suffices that confusion is probably or
likely to occur (6 Fletcher [Perm Ed], pp. 107-108, enumerating a long line of cases).
It may be that Private Respondent's products also consist of chain rollers, belts, bearing and the like while petitioners deal
principally with electrical products. It is significant to note, however, that even the Director of Patents had denied Private
Respondent's application for registration of the trademarks "Standard Philips & Device" for chains, rollers, belts, bearings
and cutting saw. That office held that PEBV "had shipped to its subsidiaries in the Philippines equipment, machines and
their parts which fall under international class where chains, rollers, belts, bearings and cutting saw, the goods in connection
with which Respondent is seeking to register "STANDARD PHILIPS . . . also belong" (Inter Partes Case No. 2010, June 17,
1988, SEC Rollo).
Furthermore, the records show that among Private Respondent's primary purposes in its Articles of Incorporation (Annex D,
Petition; p. 37, Rollo) are the following:
"To buy, sell, barter, trade, manufacture, import, export or otherwise acquire, dispose of, and deal in and
deal with any kind of goods, wares, and merchandise such as but not limited to plastics, carbon products,
office stationery and supplies, hardware parts, electrical wiring devices, electrical component parts and/or
complement of industrial, agricultural or commercial machineries, constructive supplies, electrical
supplies and other merchandise which are or may become articles of commerce except food, drugs, and
cosmetics and to carry on such business as manufacturer, distributor, dealer, indentor, factor,
manufacturer's representative capacity for domestic or foreign companies." (emphasis ours).
For its part, Philips Electrical also includes, among its primary purposes, the following:

"To develop, manufacture and deal in electrical products, including electronic, mechanical and other
similar products . . . ." (p. 30, Record of SEC Case No. 2743)
Given Private Respondent's aforesaid underlined primary purpose, nothing could prevent it from dealing in the same line of
business of electrical devices, products or supplies which fall under its primary purposes. Besides, there is showing that
Private Respondent not only manufactured and sold ballasts for fluorescent lamps with their corporate name printed thereon
but also advertised the same as, among others, Standard Philips (TSN, before the SEC, pp. 14, 17, 25, 26, 37-42, June 14,
1985; pp. 16-19, July 25, 1985). As aptly pointed out by Petitioners, "[p]rivate respondent's choice of 'PHILIPS' as part of its
corporate name [STANDARD PHILIPS CORPORATION] . . . tends to show said respondent's intention to ride on the
popularity and established goodwill of said petitioner's business throughout the world" (Rollo, p. 137). The subsequent
appropriator of the name or one confusingly similar thereto usually seeks an unfair advantage, a free ride on another's
goodwill (American Gold Star Mothers, Inc. v. National Gold Star Mothers, Inc., et al, 89 App DC 269, 191 F 2d 488). prLL
In allowing Private Respondent the continued use of its corporate name, the SEC maintains that the corporate names of
Petitioners PHILIPS ELECTRICAL LAMPS, INC. and PHILIPS INDUSTRIAL DEVELOPMENT, INC. contain at least two
words different from that of the corporate name of respondent STANDARD PHILIPS CORPORATION, which words will
readily identify Private Respondent from Petitioners and vice-versa.
True, under the Guidelines in the Approval of Corporate and Partnership Names formulated by the SEC, the proposed
name "should not be similar to one already used by another corporation or partnership. If the proposed name contains a
word already used as part of the firm name or style of a registered company, the proposed name must contain two other
words different from the company already registered" (Emphasis ours). It is then pointed out that Petitioners Philips
Electrical and Philips Industrial have two words different from that of Private Respondent's name.
What is lost sight of, however, is that PHILIPS is a trademark or trade name which was registered as far back as 1922.
Petitioners, therefore, have the exclusive right to its use which must be free from any infringement by similarity. A
corporation has an exclusive right to the use of its name, which may be protected by injunction upon a principle similar to
that upon which persons are protected in the use of trademarks and tradenames (18 C.J.S. 574). Such principle proceeds
upon the theory that it is a fraud on the corporation which has acquired a right to that name and perhaps carried on its
business thereunder, that another should attempt to use the same name, or the same name with a slight variation in such a
way as to induce persons to deal with it in the belief that they are dealing with the corporation which has given a reputation
to the name (6 Fletcher [Perm Ed], pp. 39-40, citing Borden Ice Cream Co. v. Borden's Condensed Milk Co., 210 F 510).
Notably, too, Private Respondents' name actually contains only a single word, that is, "STANDARD", different from that of
Petitioners inasmuch as the inclusion of the term "Corporation" or "Corp." merely serves the purpose of distinguishing the
corporation from partnerships and other business organizations.
The fact that there are other companies engaged in other lines of business using the word "PHILIPS" as part of their
corporate names is no defense and does not warrant the use by Private Respondent of such word which constitutes an
essential feature of Petitioners' corporate name previously adopted and registered and having acquired the status of a well-
known mark in the Philippines and internationally, as well (Bureau of Patents Decision No. 88-35 [TM], June 17, 1988, SEC
Records).
In support of its application for the registration of its Articles of Incorporation with the SEC, Private Respondent had
submitted an undertaking "manifesting its willingness to change its corporate name in the event another person, firm or
entity has acquired a prior right to the use of the said firm name or one deceptively or confusingly similar to it." Private
Respondent must now be held to its undertaking. cdll
"As a general rule, parties organizing a corporation must choose a name at their peril; and the use of a
name similar to one adopted by another corporation, whether a business or a nonbusiness or nonprofit
organization if misleading and likely to injure it in the exercise of its corporate functions, regardless of
intent, may be prevented by the corporation having the prior right, by a suit for injunction against the new
corporation to prevent the use of the name (American Gold Star Mothers, Inc. v. National Gold Star
Mothers, Inc. 89 App DC 269, 191 F 2d 488, 27 ALR 2d 948)."
WHEREFORE, the Decision of the Court of Appeals dated 31 July 1990, and its Resolution dated 20 November 1990, are
SET ASIDE and a new one entered ENJOINING private respondent from using "PHILIPS" as a feature of its corporate
name, and ORDERING the Securities and Exchange Commission to amend private respondent's Articles of Incorporation
by deleting the word PHILIPS from the corporate name of private respondent.
No costs. SO ORDERED.
||| (Philips Export B.V. v. Court of Appeals, G.R. No. 96161, [February 21, 1992], 283 PHIL 371-383)

[G.R. No. 101897. March 5, 1993.]


LYCEUM OF THE PHILIPPINES, INC., petitioner, vs. COURT OF APPEALS, LYCEUM OF APARRI,
LYCEUM OF CABAGAN, LYCEUM OF CAMALANIUGAN, INC., LYCEUM OF LALLO, INC., LYCEUM
OF TUAO, INC., BUHI LYCEUM, CENTRAL LYCEUM OF CATANDUANES, LYCEUM OF SOUTHERN
PHILIPPINES, LYCEUM OF EASTERN MINDANAO, INC. and WESTERN PANGASINAN LYCEUM,
INC., respondents.

Quisumbing, Torres & Evangelista Law Offices and Ambrosio Padilla for petitioner.
Antonio M. Nuyles and Purungan, Chato, Chato, Tarriela & Tan Law Offices for respondents.
Froilan Siobal for Western Pangasinan Lyceum.

SYLLABUS

1. CORPORATION LAW; CORPORATE NAMES; REGISTRATION OF PROPOSED NAME WHICH IS IDENTICAL OR


CONFUSINGLY SIMILAR TO THAT OF ANY EXISTING CORPORATION, PROHIBITED; CONFUSION AND DECEPTION
EFFECTIVELY PRECLUDED BY THE APPENDING OF GEOGRAPHIC NAMES TO THE WORD "LYCEUM". — The
Articles of Incorporation of a corporation must, among other things, set out the name of the corporation. Section 18 of the
Corporation Code establishes a restrictive rule insofar as corporate names are concerned: "Section 18. Corporate name. —
No corporate name may be allowed by the Securities an Exchange Commission if the proposed name is identical or
deceptively or confusingly similar to that of any existing corporation or to any other name already protected by law or is
patently deceptive, confusing or contrary to existing laws. When a change in the corporate name is approved, the
Commission shall issue an amended certificate of incorporation under the amended name." The policy underlying the
prohibition in Section 18 against the registration of a corporate name which is "identical or deceptively or confusingly similar"
to that of any existing corporation or which is "patently deceptive" or "patently confusing" or "contrary to existing laws," is the
avoidance of fraud upon the public which would have occasion to deal with the entity concerned, the evasion of legal
obligations and duties, and the reduction of difficulties of administration and supervision over corporations. We do not
consider that the corporate names of private respondent institutions are "identical with, or deceptively or confusingly similar"
to that of the petitioner institution. True enough, the corporate names of private respondent entities all carry the word
"Lyceum" but confusion and deception are effectively precluded by the appending of geographic names to the word
"Lyceum." Thus, we do not believe that the "Lyceum of Aparri" can be mistaken by the general public for the Lyceum of the
Philippines, or that the "Lyceum of Camalaniugan" would be confused with the Lyceum of the Philippines.
2. ID.; ID.; DOCTRINE OF SECONDARY MEANING; USE OF WORD "LYCEUM," NOT ATTENDED WITH EXCLUSIVITY.
— It is claimed, however, by petitioner that the word "Lyceum" has acquired a secondary meaning in relation to petitioner
with the result that word, although originally a generic, has become appropriable by petitioner to the exclusion of other
institutions like private respondents herein. The doctrine of secondary meaning originated in the field of trademark law. Its
application has, however, been extended to corporate names sine the right to use a corporate name to the exclusion of
others is based upon the same principle which underlies the right to use a particular trademark or tradename. In Philippine
Nut Industry, Inc. v. Standard Brands, Inc., the doctrine of secondary meaning was elaborated in the following terms: " . . . a
word or phrase originally incapable of exclusive appropriation with reference to an article on the market, because
geographically or otherwise descriptive, might nevertheless have been used so long and so exclusively by one producer
with reference to his article that, in that trade and to that branch of the purchasing public, the word or phrase has come to
mean that the article was his product." The question which arises, therefore, is whether or not the use by petitioner of
"Lyceum" in its corporate name has been for such length of time and with such exclusivity as to have become associated or
identified with the petitioner institution in the mind of the general public (or at least that portion of the general public which
has to do with schools). The Court of Appeals recognized this issue and answered it in the negative: "Under the doctrine of
secondary meaning, a word or phrase originally incapable of exclusive appropriation with reference to an article in the
market, because geographical or otherwise descriptive might nevertheless have been used so long and so exclusively by
one producer with reference to this article that, in that trade and to that group of the purchasing public, the word or phrase
has come to mean that the article was his produce (Ana Ang vs. Toribio Teodoro, 74 Phil. 56). This circumstance has been
referred to as the distinctiveness into which the name or phrase has evolved through the substantial and exclusive use of
the same for a considerable period of time. . . . No evidence was ever presented in the hearing before the Commission
which sufficiently proved that the word 'Lyceum' has indeed acquired secondary meaning in favor of the appellant. If there
was any of this kind, the same tend to prove only that the appellant had been using the disputed word for a long period of
time. . . . In other words, while the appellant may have proved that it had been using the word 'Lyceum' for a long period of
time, this fact alone did not amount to mean that the said word had acquired secondary meaning in its favor because the
appellant failed to prove that it had been using the same word all by itself to the exclusion of others. More so, there was no
evidence presented to prove that confusion will surely arise if the same word were to be used by other educational
institutions. Consequently, the allegations of the appellant in its first two assigned errors must necessarily fail." We agree
with the Court of Appeals. The number alone of the private respondents in the case at bar suggests strongly that petitioner's
use of the word "Lyceum" has not been attended with the exclusivity essential for applicability of the doctrine of secondary
meaning. Petitioner's use of the word "Lyceum" was not exclusive but was in truth shared with the Western Pangasinan
Lyceum and a little later with other private respondent institutions which registered with the SEC using "Lyceum" as part of
their corporation names. There may well be other schools using Lyceum or Liceo in their names, but not registered with the
SEC because they have not adopted the corporate form of organization.
3. ID.; ID.; MUST BE EVALUATED IN THEIR ENTIRETY TO DETERMINE WHETHER THEY ARE CONFUSINGLY OR
DECEPTIVELY SIMILAR TO ANOTHER CORPORATE ENTITY'S NAME. — petitioner institution is not entitled to a legally
enforceable exclusive right to use the word "Lyceum" in its corporate name and that other institutions may use "Lyceum" as
part of their corporate names. To determine whether a given corporate name is "identical" or "confusingly or deceptively
similar" with another entity's corporate name, it is not enough to ascertain the presence of "Lyceum" or "Liceo" in both
names. One must evaluate corporate names in their entirety and when the name of petitioner is juxtaposed with the names
of private respondents, they are not reasonably regarded as "identical" or "confusingly or deceptively similar" with each
other.

DECISION

FELICIANO, J p:

Petitioner is an educational institution duly registered with the Securities and Exchange Commission ("SEC").
When it first registered with the SEC on 21 September 1950, it used the corporate name Lyceum of the Philippines, Inc.
and has used that name ever since.
On 24 February 1984, petitioner instituted proceedings before the SEC to compel the private respondents,
which are also educational institutions, to delete the word "Lyceum" from their corporate names and permanently to
enjoin them from using "Lyceum" as part of their respective names. prLL
Some of the private respondents actively participated in the proceedings before the SEC. These are the
following, the dates of their original SEC registration being set out below opposite their respective names:
Western Pangasinan Lyceum — 27 October 1950
Lyceum of Cabagan — 31 October 1962
Lyceum of Lallo, Inc. — 26 March 1972
Lyceum of Aparri — 28 March 1972
Lyceum of Tuao, Inc. — 28 March 1972
Lyceum of Camalaniugan — 28 March 1972
The following private respondents were declared in default for failure to file an answer despite service of summons:
Buhi Lyceum;
Central Lyceum of Catanduanes;
Lyceum of Eastern Mindanao, Inc.; and
Lyceum of Southern Philippines
Petitioner's original complaint before the SEC had included three (3) other entities:
1. The Lyceum of Malacanay;
2. The Lyceum of Marbel; and
3. The Lyceum of Araullo
The complaint was later withdrawn insofar as concerned the Lyceum of Malacanay and the Lyceum of Marbel, for
failure to serve summons upon these two (2) entities. The case against the Liceum of Araullo was dismissed when that
school motu proprio change its corporate name to "Pamantasan ng Araullo."
The background of the case at bar needs some recounting. Petitioner had sometime before commenced in the
SEC a proceeding (SEC-Case No. 1241) against the Lyceum of Baguio, Inc. to require it to change its corporate name
and to adopt another name not "similar [to] or identical" with that of petitioner. In an Order dated 20 April 1977,
Associate Commissioner Julio Sulit held that the corporate name of petitioner and that of the Lyceum of Baguio, Inc.
were substantially identical because of the presence of a "dominant" word, i.e., "Lyceum," the name of the geographical
location of the campus being the only word which distinguished one from the other corporate name. The SEC also
noted that petitioner had registered as a corporation ahead of the Lyceum of Baguio, Inc. in point of time, 1 and ordered
the latter to change its name to another name "not similar or identical [with]" the names of previously registered entities.
cdrep

The Lyceum of Baguio, Inc. assailed the Order of the SEC before the Supreme Court in a case docketed as
G.R. No. L-46595. In a Minute Resolution dated 14 September 1977, the Court denied the Petition for Review for lack of
merit. Entry of judgment in that case was made on 21 October 1977. 2
Armed with the Resolution of this Court in G.R. No. L-46595, petitioner then wrote all the educational
institutions it could find using the word "Lyceum" as part of their corporate name, and advised them to discontinue such
use of "Lyceum." When, with the passage of time, it became clear that this recourse had failed, petitioner instituted
before the SEC SEC-Case No. 2579 to enforce what petitioner claims as its proprietary right to the word "Lyceum." The
SEC hearing officer rendered a decision sustaining petitioner's claim to an exclusive right to use the word "Lyceum."
The hearing officer relied upon the SEC ruling in the Lyceum of Baguio, Inc. case (SEC-Case No. 1241) and held that
the word "Lyceum" was capable of appropriation and that petitioner had acquired an enforceable exclusive right to the
use of that word.
On appeal, however, by private respondents to the SEC En Banc, the decision of the hearing officer was
reversed and set aside. The SEC En Banc did not consider the word "Lyceum" to have become so identified with
petitioner as to render use thereof by other institutions as productive of confusion about the identity of the schools
concerned in the mind of the general public. Unlike its hearing officer, the SEC En Banc held that the attaching of
geographical names to the word "Lyceum" served sufficiently to distinguish the schools from one another, especially in
view of the fact that the campuses of petitioner and those of the private respondents were physically quite remote from
each other. 3
Petitioner then went on appeal to the Court of Appeals. In its Decision dated 28 June 1991, however, the Court
of Appeals affirmed the questioned Orders of the SEC En Banc. 4 Petitioner filed a motion for reconsideration, without
success.
Before this Court, petitioner asserts that the Court of Appeals committed the following errors:
1. The Court of Appeals erred in holding that the Resolution of the Supreme Court in G.R. No. L-
46595 did not constitute stare decisis as to apply to this case and in not holding that said Resolution
bound subsequent determinations on the right to exclusive use of the word Lyceum.
2. The Court of Appeals erred in holding that respondent Western Pangasinan Lyceum, Inc. was
incorporated earlier than petitioner.
3. The Court of Appeals erred in holding that the word Lyceum has not acquired a secondary
meaning in favor of petitioner.
4. The Court of Appeals erred in holding that Lyceum as a generic word cannot be appropriated
by the petitioner to the exclusion of others. 5
We will consider all the foregoing ascribed errors, though not necessarily seriatim. We begin by noting that the
Resolution of the Court in G.R. No. L-46595 does not, of course, constitute res adjudicata in respect of the case at bar,
since there is no identity of parties. Neither is stare decisis pertinent, if only because the SEC En Banc itself has re-
examined Associate Commissioner Sulit's ruling in the Lyceum of Baguio case. The Minute Resolution of the Court in
G.R. No. L-46595 was not a reasoned adoption of the Sulit ruling.
The Articles of Incorporation of a corporation must, among other things, set out the name of the corporation. 6
Section 18 of the Corporation Code establishes a restrictive rule insofar as corporate names are concerned:
"SECTION 18. Corporate name. — No corporate name may be allowed by the Securities an
Exchange Commission if the proposed name is identical or deceptively or confusingly similar to that of
any existing corporation or to any other name already protected by law or is patently deceptive, confusing
or contrary to existing laws. When a change in the corporate name is approved, the Commission shall
issue an amended certificate of incorporation under the amended name." (Emphasis supplied)
The policy underlying the prohibition in Section 18 against the registration of a corporate name which is "identical or
deceptively or confusingly similar" to that of any existing corporation or which is "patently deceptive" or "patently
confusing" or "contrary to existing laws," is the avoidance of fraud upon the public which would have occasion to deal
with the entity concerned, the evasion of legal obligations and duties, and the reduction of difficulties of administration
and supervision over corporations. 7
We do not consider that the corporate names of private respondent institutions are "identical with, or
deceptively or confusingly similar" to that of the petitioner institution. True enough, the corporate names of private
respondent entities all carry the word "Lyceum" but confusion and deception are effectively precluded by the appending
of geographic names to the word "Lyceum." Thus, we do not believe that the "Lyceum of Aparri" can be mistaken by the
general public for the Lyceum of the Philippines, or that the "Lyceum of Camalaniugan" would be confused with the
Lyceum of the Philippines. LLphil
Etymologically, the word "Lyceum" is the Latin word for the Greek lykeion which in turn referred to a locality on
the river Ilissius in ancient Athens "comprising an enclosure dedicated to Apollo and adorned with fountains and
buildings erected by Pisistratus, Pericles and Lycurgus frequented by the youth for exercise and by the philosopher
Aristotle and his followers for teaching." 8 In time, the word "Lyceum" became associated with schools and other
institutions providing public lectures and concerts and public discussions. Thus today, the word "Lyceum" generally
refers to a school or an institution of learning. While the Latin word "lyceum" has been incorporated into the English
language, the word is also found in Spanish (liceo) and in French (lycee). As the Court of Appeals noted in its Decision,
Roman Catholic schools frequently use the term; e.g., "Liceo de Manila," "Liceo de Baleno" (in Baleno, Masbate), "Liceo
de Masbate," "Liceo de Albay." 9 "Lyceum" is in fact as generic in character as the word "university." In the name of the
petitioner, "Lyceum" appears to be a substitute for "university;" in other places, however, "Lyceum," or "Liceo" or
"Lycee" frequently denotes a secondary school or a college. It may be (though this is a question of fact which we need
not resolve) that the use of the word "Lyceum" may not yet be as widespread as the use of "university," but it is clear
that a not inconsiderable number of educational institutions have adopted "Lyceum" or "Liceo" as part of their corporate
names. Since "Lyceum" or "Liceo" denotes a school or institution of learning, it is not unnatural to use this word to
designate an entity which is organized and operating as an educational institution.
It is claimed, however, by petitioner that the word "Lyceum" has acquired a secondary meaning in relation to
petitioner with the result that that word, although originally a generic, has become appropriable by petitioner to the
exclusion of other institutions like private respondents herein.
The doctrine of secondary meaning originated in the field of trademark law. Its application has, however, been
extended to corporate names sine the right to use a corporate name to the exclusion of others is based upon the same
principle which underlies the right to use a particular trademark or tradename. 10 In Philippine Nut Industry, Inc. v.
Standard Brands, Inc., 11 the doctrine of secondary meaning was elaborated in the following terms:
" . . . a word or phrase originally incapable of exclusive appropriation with reference to an article
on the market, because geographically or otherwise descriptive, might nevertheless have been used so
long and so exclusively by one producer with reference to his article that, in that trade and to that branch
of the purchasing public, the word or phrase has come to mean that the article was his product." 12
The question which arises, therefore, is whether or not the use by petitioner of "Lyceum" in its corporate name
has been for such length of time and with such exclusivity as to have become associated or identified with the petitioner
institution in the mind of the general public (or at least that portion of the general public which has to do with schools).
The Court of Appeals recognized this issue and answered it in the negative:
"Under the doctrine of secondary meaning, a word or phrase originally incapable of exclusive
appropriation with reference to an article in the market, because geographical or otherwise descriptive
might nevertheless have been used so long and so exclusively by one producer with reference to this
article that, in that trade and to that group of the purchasing public, the word or phrase has come to mean
that the article was his produce (Ana Ang vs. Toribio Teodoro, 74 Phil. 56). This circumstance has been
referred to as the distinctiveness into which the name or phrase has evolved through the substantial and
exclusive use of the same for a considerable period of time. Consequently, the same doctrine or principle
cannot be made to apply where the evidence did not prove that the business (of the plaintiff) has
continued for so long a time that it has become of consequence and acquired a good will of considerable
value such that its articles and produce have acquired a well-known reputation, and confusion will result
by the use of the disputed name (by the defendant) (Ang Si Heng vs. Wellington Department Store, Inc.,
92 Phil. 448). llcd
With the foregoing as a yardstick, [we] believe the appellant failed to satisfy the aforementioned
requisites. No evidence was ever presented in the hearing before the Commission which sufficiently
proved that the word 'Lyceum' has indeed acquired secondary meaning in favor of the appellant. If there
was any of this kind, the same tend to prove only that the appellant had been using the disputed word for
a long period of time. Nevertheless, its (appellant) exclusive use of the word (Lyceum) was never
established or proven as in fact the evidence tend to convey that the cross-claimant was already using
the word 'Lyceum' seventeen (17) years prior to the date the appellant started using the same word in its
corporate name. Furthermore, educational institutions of the Roman Catholic Church had been using the
same or similar word like 'Liceo de Manila,' 'Liceo de Baleno' (in Baleno, Masbate), 'Liceo de Masbate,'
'Liceo de Albay' long before appellant started using the word 'Lyceum'. The appellant also failed to prove
that the word 'Lyceum' has become so identified with its educational institution that confusion will surely
arise in the minds of the public if the same word were to be used by other educational institutions.

In other words, while the appellant may have proved that it had been using the word 'Lyceum' for
a long period of time, this fact alone did not amount to mean that the said word had acquired secondary
meaning in its favor because the appellant failed to prove that it had been using the same word all by
itself to the exclusion of others. More so, there was no evidence presented to prove that confusion will
surely arise if the same word were to be used by other educational institutions. Consequently, the
allegations of the appellant in its first two assigned errors must necessarily fail." 13 (Emphasis partly in
the original and partly supplied)
We agree with the Court of Appeals. The number alone of the private respondents in the case at bar suggests
strongly that petitioner's use of the word "Lyceum" has not been attended with the exclusivity essential for applicability
of the doctrine of secondary meaning. It may be noted also that at least one of the private respondents, i.e., the
Western Pangasinan Lyceum, Inc., used the term "Lyceum" seventeen (17) years before the petitioner registered its
own corporate name with the SEC and began using the word "Lyceum." It follows that if any institution had acquired an
exclusive right to the word "Lyceum," that institution would have been the Western Pangasinan Lyceum, Inc. rather than
the petitioner institution. cdphil
In this connection, petitioner argues that because the Western Pangasinan Lyceum, Inc. failed to reconstruct its
records before the SEC in accordance with the provisions of R.A. No. 62, which records had been destroyed during
World War II, Western Pangasinan Lyceum should be deemed to have lost all rights it may have acquired by virtue of its
past registration. It might be noted that the Western Pangasinan Lyceum, Inc. registered with the SEC soon after
petitioner had filed its own registration on 21 September 1950. Whether or not Western Pangasinan Lyceum, Inc. must
be deemed to have lost its rights under its original 1933 registration, appears to us to be quite secondary in importance;
we refer to this earlier registration simply to underscore the fact that petitioner's use of the word "Lyceum" was neither
the first use of that term in the Philippines nor an exclusive use thereof. Petitioner's use of the word "Lyceum" was not
exclusive but was in truth shared with the Western Pangasinan Lyceum and a little later with other private respondent
institutions which registered with the SEC using "Lyceum" as part of their corporation names. There may well be other
schools using Lyceum or Liceo in their names, but not registered with the SEC because they have not adopted the
corporate form of organization.
We conclude and so hold that petitioner institution is not entitled to a legally enforceable exclusive right to use
the word "Lyceum" in its corporate name and that other institutions may use "Lyceum" as part of their corporate names.
To determine whether a given corporate name is "identical" or "confusingly or deceptively similar" with another entity's
corporate name, it is not enough to ascertain the presence of "Lyceum" or "Liceo" in both names. One must evaluate
corporate names in their entirety and when the name of petitioner is juxtaposed with the names of private respondents,
they are not reasonably regarded as "identical" or "confusingly or deceptively similar" with each other.
WHEREFORE, the petitioner having failed to show any reversible error on the part of the public respondent
Court of Appeals, the Petition for Review is DENIED for lack of merit, and the Decision of the Court of Appeals dated 28
June 1991 is hereby AFFIRMED. No pronouncement as to costs. LexLibSO ORDERED.
||| (Lyceum of the Philippines, Inc. v. Court of Appeals, G.R. No. 101897, [March 5, 1993])

[G.R. No. 129552. June 29, 2005.]

P.C. JAVIER & SONS, INC., SPS. PABLO C. JAVIER, SR. and ROSALINA F. JAVIER, petitioners, vs.
HON. COURT OF APPEALS, PAIC SAVINGS & MORTGAGE BANK, INC., SHERIFFS GRACE
BELVIS, SOFRONIO VILLARIN, PIO MARTINEZ and NICANOR BLANCO, respondents.

DECISION

CHICO-NAZARIO, J p:

Before Us is an appeal by certiorari under Rule 45 of the Rules of Court which seeks to set aside the decision 1 of
the Court of Appeals dated 31 January 1997 which affirmed in toto the decision of Branch 62 of the Regional Trial Court
(RTC) of Makati City, dismissing the complaint for Annulment of Mortgage and Foreclosure with Preliminary Injunction,
Prohibition and Damages filed by petitioners, and its Resolution 2 dated 20 June 1997 denying petitioners' motion for
reconsideration.
A complaint 3 for Annulment of Mortgage and Foreclosure with Preliminary Injunction, Prohibition and Damages
was filed by petitioners P.C. Javier & Sons, Inc. and spouses Pablo C. Javier, Sr. and Rosalina F. Javier against PAIC
Savings & Mortgage Bank, Inc., Grace S. Belvis, Acting Ex Officio Regional Sheriff of Pasig, Metro Manila and Sofronio M.
Villarin, Deputy Sheriff-in-Charge, before Branch 62 of the RTC of Makati City, on 07 May 1984. The case was docketed as
Civil Case No. 7184.
On 10 May 1984, a Supplemental Complaint 4 was filed to include additional defendants, namely: Pio Martinez,
Acting Ex Officio Regional Sheriff of Antipolo, Rizal, and Nicanor D. Blanco, Deputy Sheriff-in-Charge.
The facts that gave rise to the aforesaid complaint, as found by Branch 62 of the RTC of Makati City, and adopted
by the respondent court, are as follows:
In February, 1981, Plaintiff P.C. Javier and Sons Services, Inc., Plaintiff Corporation, for short,
applied with First Summa Savings and Mortgage Bank, later on renamed as PAIC Savings and Mortgage
Bank, Defendant Bank, for short, for a loan accommodation under the Industrial Guarantee Loan Fund
(IGLF) for P1.5 Million. On March 21, 1981, Plaintiff Corporation through Plaintiff Pablo C. Javier, Plaintiff
Javier for short, was advised that its loan application was approved and that the same shall be forwarded
to the Central Bank (CB) for processing and release (Exhibit A also Exhibit 8). THAICD
The CB released the loan to Defendant Bank in two (2) tranches of P750,000 each. The first
tranche was released to the Plaintiff Corporation on May 18, 1981 in the amount of P750,000.00 and the
second tranche was released to Plaintiff Corporation on November 21, 1981 in the amount of
P750,000.00. From the second tranche release, the amount of P250,000.00 was deducted and deposited
in the name of Plaintiff Corporation under a time deposit.
Plaintiffs claim that the loan releases were delayed; that the amount of P250,000.00 was
deducted from the IGLF loan of P1.5 Million and placed under time deposit; that Plaintiffs were never
allowed to withdraw the proceeds of the time deposit because Defendant Bank intended this time deposit
as automatic payments on the accrued principal and interest due on the loan. Defendant Bank, however,
claims that only the final proceeds of the loan in the amount of P750,000.00 was delayed the same
having been released to Plaintiff Corporation only on November 20, 1981, but this was because of the
shortfall in the collateral cover of Plaintiff's loan; that this second tranche of the loan was precisely
released after a firm commitment was made by Plaintiff Corporation to cover the collateral deficiency
through the opening of a time deposit using a portion of the loan proceeds in the amount of P250,000.00
for the purpose; that in compliance with their commitment to submit additional security and open time
deposit, Plaintiff Javier in fact opened a time deposit for P250,000.00 and on February 15, 1983,
executed a chattel mortgage over some machineries in favor of Defendant Bank; that thereafter, Plaintiff
Corporation defaulted in the payment of its IGLF loan with Defendant Bank hence Defendant Bank sent a
demand letter dated November 22, 1983, reminding Plaintiff Javier to make payments because their
accounts have been long overdue; that on May 2, 1984, Defendant Bank sent another demand letter to
Plaintiff spouses informing them that since they have defaulted in paying their obligation, their mortgage
will now be foreclosed; that when Plaintiffs still failed to pay, Defendant Bank initiated extrajudicial
foreclosure of the real estate mortgage executed by Plaintiff spouses and accordingly the auction sale of
the property covered by TCT No. 473216 was scheduled by the Ex-Officio Sheriff on May 9, 1984. 5
The instant complaint was filed to forestall the extrajudicial foreclosure sale of a piece of land covered by Transfer
Certificate of Title (TCT) No. 473216 6 mortgaged by petitioner corporation in favor of First Summa Savings and Mortgage
Bank which bank was later renamed as PAIC Savings and Mortgage Bank, Inc. 7 It likewise asked for the nullification of the
Real Estate Mortgages it entered into with First Summa Savings and Mortgage Bank. The supplemental complaint added
several defendants who scheduled for public auction other real estate properties contained in the same real estate
mortgages and covered by TCTs No. N-5510, No. 426872, No. 506346 and Original Certificate of Title No. 10146. 8
Several extrajudicial foreclosures of the mortgaged properties were scheduled but were temporarily restrained by
the RTC notwithstanding the denial 9 of petitioners' prayer for a writ of preliminary injunction. In an Order 10 dated 10
December 1990, the RTC ordered respondents-sheriffs to maintain the status quo and to desist from further proceeding with
the extrajudicial foreclosure of the mortgaged properties. IDAESH
Among the issues raised by petitioners at the RTC are whether or not First Summa Savings and Mortgage Bank
and PAIC Savings and Mortgage Bank, Inc. are one and the same entity, and whether or not their obligation is already due
and demandable at the time respondent bank commenced to extrajudicially foreclose petitioners' properties in April 1984.
The RTC declared that First Summa Savings and Mortgage Bank and PAIC Savings and Mortgage Bank, Inc. are
one and the same entity and that petitioner corporation is liable to respondent bank for the unpaid balance of its Industrial
Guarantee Loan Fund (IGLF) loans. The RTC further ruled that respondent bank was justified in extrajudicially foreclosing
the real estate mortgages executed by petitioner corporation in its favor because the loans were already due and
demandable when it commenced foreclosure proceedings in April 1984.
In its decision dated 06 July 1993, the RTC disposed of the case as follows:
Premises considered, judgment is hereby rendered dismissing the Complaint against Defendant
Bank and ordering Plaintiffs to pay Defendant Bank jointly and severally, the following:
1. The principal amount of P700,453.45 under P.N. No. 713 plus all the accrued interests,
liquidated damages and other fees due thereon from March 18, 1983 until fully paid as provided in said
PN;
2. The principal amount of P749,879.38 under P.N. No. 841 plus all the accrued interests,
liquidated damages and other fees due thereon from September 1, 1982 until fully paid as provided in
such PN;
3. The amount of P40,000.00 as actual damages;
4. The amount of P30,000.00 as exemplary damages;
5. The amount of P50,000.00 as attorney's fees; plus
6. Cost of suit. 11
Petitioners filed a Motion for Reconsideration 12 which was opposed 13 by respondent bank. The motion was
denied in an Order dated 11 May 1994.
Petitioners appealed the decision to the Court of Appeals. The latter affirmed in toto the decision of the lower court.
It also denied petitioners' motion for reconsideration. ISDHcT
Hence, this appeal by certiorari.
Petitioners assigned the following as errors:
a. PUBLIC RESPONDENT COURT GRAVELY ERRED WHEN IT SUSTAINED THE DISMISSAL
OF PETITIONERS' COMPLAINT AND IN AFFIRMING THE RIGHT OF THE RESPONDENT BANK TO
COLLECT THE IGLF LOANS IN LIEU OF FIRST SUMMA SAVINGS AND MORTGAGE BANK WHICH
ORIGINALLY GRANTED SAID LOANS.
COROLLARY TO THE ABOVE ARGUMENT, THE PUBLIC RESPONDENT COURT ALSO
GRAVELY ERRED WHEN IT RULED THAT THE PETITIONERS CANNOT WITHHOLD THEIR
PAYMENT TO THE RESPONDENT BANK NOTWITHSTANDING THE ADMITTED INABILITY OF THE
RESPONDENT BANK TO FURNISH THE PETITIONERS THE SAID REQUESTED DOCUMENTS.
b. PUBLIC RESPONDENT COURT GRAVELY ERRED WHEN IT SUSTAINED THE
COLLECTION OF THE ENTIRE PROCEEDS OF THE IGLF LOANS OF P1,500,000.00 DESPITE THE
FACT THAT THE P250,000.00 OF THIS LOAN WAS WITHHELD BY THE FIRST SUMMA SAVINGS
AND MORTGAGE BANK TO BECOME PART OF THE COLLATERALS TO THE SAID P1,500,000.00
LOAN.
c. PUBLIC RESPONDENT COURT GRAVELY ERRED WHEN IT SUSTAINED THE DAMAGES
AWARDED TO THE RESPONDENT BANK DESPITE THE ABSENCE OF MALICE OR BAD FAITH ON
THE PART OF THE PETITIONERS IN FILING THIS CASE AGAINST THE RESPONDENT BANK.
On the first assigned error, petitioners argue that they are legally justified to withhold their amortized payments to
the respondent bank until such time they would have been properly notified of the change in the corporate name of First
Summa Savings and Mortgage Bank. They claim that they have never received any formal notice of the alleged change of
corporate name of First Summa Savings and Mortgage Bank to PAIC Savings & Mortgage Bank, Inc. They further claim that
the only and first time they received formal evidence of a change in the corporate name of First Summa Savings and
Mortgage Bank surfaced when respondent bank presented its witness, Michael Caguioa, on 03 April 1990, where he
presented the Securities and Exchange Commission (SEC) Certificate of Filing of the Amended Articles of Incorporation of
First Summa Savings and Mortgage Bank, 14 the Central Bank (CB) Certificate of Authority 15 to change the name of First
Summa Savings and Mortgage Bank to PAIC Savings and Mortgage Bank, Inc., and the CB Circular Letter 16 dated 27
June 1983.

Their argument does not hold water. Their defense that they should first be formally notified of the change of
corporate name of First Summa Savings and Mortgage Bank to PAIC Savings and Mortgage Bank, Inc., before they will
continue paying their loan obligations to respondent bank presupposes that there exists a requirement under a law or
regulation ordering a bank that changes its corporate name to formally notify all its debtors. After going over the Corporation
Code and Banking Laws, as well as the regulations and circulars of both the SEC and the Bangko Sentral ng Pilipinas
(BSP), we find that there is no such requirement. This being the case, this Court cannot impose on a bank that changes its
corporate name to notify a debtor of such change absent any law, circular or regulation requiring it. Such act would be
judicial legislation. The formal notification is, therefore, discretionary on the bank. Unless there is a law, regulation or circular
from the SEC or BSP requiring the formal notification of all debtors of banks of any change in corporate name, such
notification remains to be a mere internal policy that banks may or may not adopt.
In the case at bar, though there was no evidence showing that petitioners were furnished copies of official
documents showing the First Summa Savings and Mortgage Bank's change of corporate name to PAIC Savings and
Mortgage Bank, Inc., evidence abound that they had notice or knowledge thereof. Several documents establish this fact.
First, letter 17 dated 16 July 1983 signed by Raymundo V. Blanco, Accountant of petitioner corporation, addressed to PAIC
Savings and Mortgage Bank, Inc. Part of said letter reads: "In connection with your inquiry as to the utilization of funds we
obtained from the former First Summa Savings and Mortgage Bank, . . ." Second, Board Resolution 18 of petitioner
corporation signed by Pablo C. Javier, Sr. on 24 August 1983 authorizing him to execute a Chattel Mortgage over certain
machinery in favor of PAIC Savings and Mortgage Bank, Inc. Third, Secretary's Certificate 19 signed by Fortunato E.
Gabriel, Corporate Secretary of petitioner corporation, on 01 September 1983, certifying that a board resolution was passed
authorizing Mr. Pablo C. Javier, Sr. to execute a chattel mortgage on the corporation's equipment that will serve as
collateral to cover the IGLF loan with PAIC Savings and Mortgage Bank, Inc. Fourth, undated letter 20 signed by Pablo C.
Javier, Sr. and addressed to PAIC Savings and Mortgage Bank, Inc., authorizing Mr. Victor F. Javier, General Manager of
petitioner corporation, to secure from PAIC Savings and Mortgage Bank, Inc. certain documents for his signature. DCcIaE
From the foregoing documents, it cannot be denied that petitioner corporation was aware of First Summa Savings
and Mortgage Bank's change of corporate name to PAIC Savings and Mortgage Bank, Inc. Knowing fully well of such
change, petitioner corporation has no valid reason not to pay because the IGLF loans were applied with and obtained from
First Summa Savings and Mortgage Bank. First Summa Savings and Mortgage Bank and PAIC Savings and Mortgage
Bank, Inc., are one and the same bank to which petitioner corporation is indebted. A change in the corporate name does not
make a new corporation, whether effected by a special act or under a general law. It has no effect on the identity of the
corporation, or on its property, rights, or liabilities. 21 The corporation, upon such change in its name, is in no sense a new
corporation, nor the successor of the original corporation. It is the same corporation with a different name, and its character
is in no respect changed. 22
Anent the second assigned error, this Court rules that respondent court did not err when it sustained the collection
of the entire proceeds of the IGLF loans amounting to P1,500,000.00 despite the withholding of P250,000.00 to become
part of the collaterals to the said P1,500,000.00 IGLF loan.
Petitioners contend that the collaterals they submitted were more than sufficient to cover the P1,500,000.00 IGLF
loan. Such contention is untenable. Petitioner corporation was required to place P250,000.00 in a time deposit with
respondent bank for the simple reason that the collateral it put up was insufficient to cover the IGLF loans it has received. It
admitted the shortfall of its collateral when it authorized petitioner Pablo C. Javier, Sr., via a board resolution, 23 to execute
a chattel mortgage over certain machinery in favor of PAIC Savings and Mortgage Bank, Inc. which was certified by its
corporate secretary. 24 If the collateral it put up was sufficient, why then did it execute another chattel mortgage?
In his order dated 07 September 1984, Hon. Rafael T. Mendoza found that the loanable value of the lands,
buildings, machinery and equipment amounted only to P934,000.00. The order reads in part:
The terms and conditions of the IGLF loan extended to plaintiff corporation are governed by the
loan and security documents evidencing said loan. Although the loan agreement was approved by the
defendant bank, the same has to be processed and be finally approved by the Central Bank of the
Philippines, in pursuance to the IGLF program, of which the defendant bank is an accredited participant.
The defendant had to await Central Bank's advise (sic) regarding the final approval of the loan before the
release of the proceeds thereof. The proceeds of the loan was released to the plaintiff on 6 April and
November 20, 1981, and the final proceeds was released only on November 20, 1981, on account of
short fall in the collateral covered by the lands and buildings as well as the machineries and equipment
then subject of the existing mortgages in favor of the defendant bank, having only a loanable value of
P934,000.00, and only after a firm commitment made by plaintiff corporation to the defendant bank to
correct the collateral deficiency thru the execution of a chattel mortgage on additional machineries,
equipment and tools and thru the opening of a time deposit with PAIC Bank using a portion of the loan
proceeds in the amount of P250,000.00 to answer for its obligation to the defendant bank under the IGLF
loan was the final proceeds of the loan released in favor of the plaintiffs. The delay in the release of the
final proceeds of the IGLF loan was due to the aforestated collateral deficiency. 25
As declared by the respondent court, the finding in said order was not disputed in the appeal before it. It said that
what was contained in petitioners' brief was that "their loans were 'overcollateralized,' and fail to specify why or in what
manner it was so." 26 Having failed to raise this issue before the respondent court, petitioners thus cannot raise this issue
before this Court. Moreover, since the issue of whether or not the collateral put up by petitioners is sufficient is factual, the
same is not proper for this Court's consideration. The basic rule is that factual questions are beyond the province of the
Supreme Court in a petition for review. 27
Petitioners maintain that to collect the P250,000.00 from them would be a clear case of unjust enrichment because
they have not availed or used said amount for the same was unlawfully withheld from them.
We do not agree. The fundamental doctrine of unjust enrichment is the transfer of value without just cause or
consideration. The elements of this doctrine are: enrichment on the part of the defendant; impoverishment on the part of the
plaintiff; and lack of cause. The main objective is to prevent one to enrich himself at the expense of another. 28 It is
commonly accepted that this doctrine simply means that a person shall not be allowed to profit or enrich himself inequitably
at another's expense. 29 In the instant case, there is no unjust enrichment to speak of. The amount of P225,905.79 was
applied as payment for petitioner corporation's loan which was taken from the P250,000.00, together with its accrued
interest, that was placed in time deposit with First Summa Savings and Mortgage Bank. The use of said amount as payment
was approved by petitioner Pablo C. Javier, Sr. on 17 March 1983. 30 As further found by the RTC in its decision, the
balance of the time deposit was withdrawn by petitioners. 31
Petitioner corporation faults respondent bank, then known as First Summa Savings and Mortgage Bank, for
requiring it to put up as additional collateral the amount of P250,000.00 inasmuch as the CB never required it to do so. It
added that respondent bank took advantage of its urgent and immediate need at the time for the proceeds of the IGLF loans
that it had no choice but to comply with respondent bank's requirement to put in time deposits the said amount as additional
collateral.
We agree with respondent court that the questioning of the propriety of the placing of the P250,000.00 in time
deposits 32 with respondent bank as additional collateral was belatedly made. As above-discussed, the requirement to give
additional collateral was warranted because the collateral petitioner corporation put up failed to cover its IGLF loans. If
petitioner corporation was really bent on questioning the reasonableness of putting up the aforementioned amount as
additional collateral, it should have done immediately after it made the time deposits on 26 November 1981. This, it did not
do. It questioned the placing of the time deposits only on 08 February 1984 33 or long after defendant bank had already
demanded full payment of the loans, then amounting to P2,045,401.79 as of 22 November 1983. It is too late in the day for
petitioner corporation to question the placing of the P250,000.00 in time deposits after it failed to pay its loan obligations as
scheduled, making them due and demandable, and after a demand for full payment has been made. We will not allow
petitioner corporation to have one's cake and eat it too.

As regards the payments made by petitioner corporation, respondent court has this to say:
The trial court held, based on plaintiffs' own exhibits, that plaintiff[s] made the following
payments:
On Promissory Note No. 713:
Date Actual Date of Amount

(Per PN Schedule) Payment

July 6, 1981 August 3, 1981 P28,125.00

October 6, 1981 October 28, 1981 28,836.13

January 6, 1982 January 22, 1982 29,227.38

March 17, 1983 225,905.79

––––––––––

TOTAL P312,094.30

And on Promissory Note No. 841:


Date Actual Date of Amount

(Per PN Schedule) Payment

February 20, 1982 April 13, 1982 P28,569.30

May 20, 1982 July 7, 1982 29,254.31

August 20, 1982 August 31, 1982 36,795.44

–––––––––

TOTAL P94,619.05

Plaintiff-appellant[s] does not dispute the finding, which is obvious from the foregoing summary,
that plaintiff[s] stopped payments on March 17, 1983 on Promissory Note No. 713, and on August 31,
1982 on Promissory Note No. 841. TIcEDC
By simply looking at the amortization schedule attached to the two promissory notes, it is clear
that plaintiff[s] already defaulted on its loan obligations when the defendant Bank gave notice of the
foreclosure proceedings on April 28, 1984. On amortization payments alone, plaintiff[s] should have paid
a total of P459,339 as of April 6, 1984 on Promissory [Note] No. 713, and a total of P328,173.00 as of
February 20, 1984 on Promissory Note [No.] 841. No extended computation is necessary to demonstrate
that, even without imputing the liquidated damages equivalent to 2% a month on the delayed payments
(see second paragraph of the promissory notes), the plaintiffs were grossly deficient in amortization
payments, and already in default when the foreclosure proceedings were commenced. Further, we note
that under the terms of the promissory note, "failure to pay an installment when due shall entitle the bank
or its assign to declare all the obligations as immediately due and payable" (second paragraph). 34
As to the third assigned error, petitioners argue that there being no malice or bad faith on their part when they filed
the instant case, no damages should have been awarded to respondent bank.
We cannot sustain such argument. The presence of malice or bad faith is very evident in the case before us. By the
documents it executed, petitioner corporation was well aware that First Summa Savings and Mortgage Bank changed its
corporate name to PAIC Savings and Mortgage Bank, Inc. Despite knowledge that First Summa Savings and Mortgage
Bank and PAIC Savings and Mortgage Bank, Inc., are one and the same entity, it pretended otherwise. It used this
purported ignorance as an excuse to renege on its obligation to pay its loans after they became due and after demands for
payment were made, claiming that it never obtained the loans from respondent bank.
No good faith was shown by petitioner corporation. If it were in good faith in complying with its loan obligations
since it believed that respondent bank had no right to the payment, it should have made a valid consignation in court. This, it
did not do. If petitioner corporation were at a loss as to who should receive the payment, it could have easily taken steps
and inquired from the SEC, CB of the Philippines or from the bank itself from which it received the loans and to where it
made previous payments. Further, the fact that it was respondent bank that was demanding payment for loans already due
and demandable and not First Summa Savings and Mortgage Bank is sufficient to make petitioner corporation wonder why
this is so. It never took any initiative to clear the matter. Instead, it paid no attention to the valid demands of respondent
bank.
The awarding of actual and compensatory damages, as well as attorney's fees, is justified under the circumstances.
We quote with approval the reasons given by the RTC for the grant of the same:
Considering that Defendant Bank had been prevented at least four (4) times from foreclosing the
mortgages (i.e., Temporary Restraining Orders of May 9 and 19 and October 22, 1984 and status quo
order of December 10, 1990 enjoining the extrajudicial foreclosure sales of May 9 and 16 and October
23, 1984 and December 20, 1990, respectively), it is proper that Defendant Bank be reimbursed its actual
expenses. The amount of P40,000.00 is reasonable reimbursement for the publication and other
expenses incurred in the four (4) extrajudicial foreclosures which were enjoined by the Court. Considering
the wanton and reckless filing of this clearly unfounded and baseless legal action and the fact that
Defendant Bank had to defend itself against such suit, attorney's fees in the amount of P50,000.00
should be paid by the Plaintiffs to the Defendant Bank. Defendant Bank failed to adduce indubitable proof
on the moral and exemplary damages that it seeks. Nevertheless, since such proof is not absolutely
necessary and primarily as an example for the public good to deter others from filing a similar clearly
unfounded legal action, Defendant Bank should be entitled to an award of exemplary damages. 35
This Court finds that petitioners failed to comply with what is incumbent upon them — to pay their loans when they
became due. The lame excuse they belatedly advanced for their non-payment cannot and should not prevent respondent
bank from exercising its right to foreclose the real estate mortgages executed in its favor.
WHEREFORE, premises considered, the Court of Appeals decision dated 31 January 1997 and its resolution dated
20 June 1997 are hereby AFFIRMED in toto. Costs against petitioners. SO ORDERED.
||| (P.C. Javier & Sons Inc. v. Court of Appeals, G.R. No. 129552, [June 29, 2005], 500 PHIL 419-437)

[G.R. No. L-26370. July 31, 1970.]

PHILIPPINE FIRST INSURANCE COMPANY, INC., plaintiff-appellant, vs. MARIA CARMEN


HARTIGAN, CGH, and O. ENGKEE, defendants-appellees.

Bausa, Ampil & Suarez for plaintiff-appellant.


Nicasio E. Martin for defendants-appellees.

DECISION

BARREDO, J p:
Appeal from the decision dated 6 October 1962 of the Court of First Instance of Manila — dismissing the action
in its Civil Case No. 48925 — brought by the herein plaintiff-appellant Philippine First Insurance Co., Inc. to the Court of
Appeals which could, upon finding that the said appeal raises purely questions of law, declared itself without jurisdiction
to entertain the same and, in its resolution dated 15 July 1966, certified the records thereof to this Court for proper
determination.
The antecedent facts are set forth in the pertinent portions of the resolution of the Court of Appeals referred to
as follows:
"According to the complaint, plaintiff was originally organized as an insurance corporation
under the name of 'The Yek Tong Lin Fire and Marine Insurance Co., Ltd.' The articles of
incorporation originally presented before the Securities and Exchange Commissioner and
acknowledged before Notary Public Mr. E. D. Ignacio on June 1, 1953 state that the name of the
corporation was 'The Yek Tong Lin Fire and Marine Insurance Co., Ltd.'.' On May 26, 1961 the
articles of incorporation were amended pursuant to a certificate of the Board of Directors dated
March 8, 1961 changing the name of the corporation to 'Philippine First Insurance Co., Inc.'
"The complaint alleges that the plaintiff Philippine First Insurance Co., Inc., doing business
under the name of 'The Yek Tong Lin Fire and Marine Insurance Co., Lt.' signed as co-maker
together with defendant Maria Carmen Hartigan, CGH, a promissory note for P5,000.00 in favor of
the China Banking Corporation payable within 30 days after the date of the promissory note with
the usual banking interest; that the plaintiff agreed to act as such co-maker of the promissory note
upon the application of the defendant Maria Carmen Hartigan, CGH, who together with Antonio F.
Chua and Chang Ka Fu, signed an indemnity agreement in favor of the plaintiff, undertaking jointly
and severally, to pay the plaintiff damages, losses or expenses of whatever kind or nature,
including attorney's fees and legal costs, which the plaintiff may sustain as a result of the execution
by the plaintiff as co-maker of Maria Carmen Hartigan, CGH, of the promissory note above referred
to; that as a result of the execution of the promissory note by the plaintiff and Maria Carmen
Hartigan, CGH, the China Banking Corporation delivered to the defendant Maria Carmen Hartigan,
CGH, the sum of P5,000.00 which said defendant failed to pay in full, such that on August 31,
1961 the same was renewed and as of November 27, 1961 there was due on account of the
promissory note the sum of P4,559.50 including interest. The complaint ends with a prayer for
judgment against the defendants, jointly and severally, for the sum of P4,559.50 with interest at the
rate of 12% per annum from November 23, 1961 plus P911.90 by way of attorney's fees and costs.
"Although O. Engkee was made as party defendant in the caption of the complaint, his
name is not mentioned in the body of said complaint. However, his name appears in the Annex A
attached to the complaint which is the counter indemnity agreement supposed to have been
signed according to the complaint by Maria Carmen Hartigan, CGH, Antonio F. Chua and Chang
Ka Fu.
"In their answer the defendants deny the allegation that the plaintiff formerly conducted
business under the name and style of 'The Yek Tong Lin Fire and Marine Insurance Co., Ltd.',
They admit the execution of the indemnity agreement but they claim that they signed said
agreement in favor of the 'Yek Tong Lin Fire and Marine Insurance Co., Ltd.' and not in favor of the
plaintiff. They likewise admit that they failed to pay the promissory note when it fell due but they
allege that since their obligation with the China Banking Corporation based on the promissory note
still subsists, the surety who co-signed the promissory note is not entitled to collect the value
thereof from the defendants otherwise they will be liable for double amount of their obligation, there
being no allegation that the surety has paid the obligation to the creditor.
"By way of special defense, defendants claim that there is no privity of contract between
the plaintiff and the defendants and consequently, the plaintiff has no cause of action against them,
considering that the complaint does not allege that the plaintiff and the 'Yek Tong Lin Fire and
Marine Insurance Co., Ltd.' are one and the same or that the plaintiff has acquired the rights of the
latter. The parties after the admission of Exhibit A which is the amended articles of incorporation
and Exhibit 1 which is a demand letter dated August 16, 1962 signed by the manager of the loans
and discount department of the China Banking Corporation showing that the promissory note up to
said date in the sum of P4,500.00 was still unpaid, submitted the case for decision based on the
pleadings."
Under date of 6 October 1962, the Court of First Instance of Manila rendered the decision appealed. It
dismissed the action with costs against the plaintiff Philippine First Insurance Co., Inc., reasoning as follows:
". . . With these undisputed facts in mind, the parties correctly concluded that the issues for
resolution by this Court are as follows:
"(a) Whether or not the plaintiff is the real party in interest that may validly sue on
the indemnity agreement signed by the defendants and the Yek Tong Lin Fire & Marine
Insurance Co., Ltd. (Annex A to plaintiff's complaint); and
"(b) Whether or not a suit for indemnity or reimbursement may under said
indemnity agreement prosper without plaintiff having yet paid the amount due under said
promissory note.
"In the first place, the change of name of the Yek Tong Lin Fire & Marine Insurance Co.,
Ltd. to the Philippine First Insurance Co., Inc. is of dubious validity. Such change of name in effect
dissolved the original corporation by a process of dissolution not authorized by our corporation law
(see Secs. 62 and 67, inclusive, of our Corporation Law). Moreover, said change of name,
amounting to a dissolution of the Yek Tong Lin Fire & Marine Insurance Co., Ltd., does not appear
to have been effected with the written note or assent of stockholders representing at least two-
thirds of the subscribed capital stock of the corporation, a voting proportion required not only for
the dissolution of a corporation but also for any amendment of its articles of incorporation (Secs.
18 and 62, Corporation Law). Furthermore, such change of corporate name appears to be against
public policy and may be effected only by express authority of law (Red Line Transportation Co. v.
Rural Transit Co., Ltd., 60 Phil. 549, 555; Cincinnati Cooperage Co., Ltd. vs. Vate, 26 SW 538,
539; Pilsen Brewing Co. vs. Wallace, 125 NE 714), but there is nothing in our corporation law
authorizing the change of corporate name in this jurisdiction.
"In the second place, assuming that the change of name of the Yek Tong Lin Fire &
Marine Insurance Co., Ltd., to Philippine First Insurance Co., Inc., as accomplished on March 8,
1961, is valid, that would mean that the original corporation; the Yek Tong Lin Fire & Marine
Insurance Co., Ltd., became dissolved and of no further existence since March 8, 1961, 80 that on
May 15, 1961, the date the indemnity agreement, Annex A, was executed, said original corporation
had no more power to enter into any agreement with the defendants, and the agreement entered
into by it was ineffective for lack of capacity of said dissolved corporation to enter into said
agreement. At any rate, even if we hold that said change of name is valid, the fact remains that
there is no evidence showing that the new city entity, the Philippine First Insurance Co., Inc. has,
with the consent of the original parties, assumed the obligations or was assigned the rights of
action in the original corporation, the Yek Tong Lin Fire & Marine Insurance Co., Ltd. In other
words, there is no evidence of conventional subrogation of the plaintiffs in the rights of the Yek
Tong Lin Fire & Marine Insurance Co., Ltd. under said indemnity agreement (Arts. 1300 1301, New
Civil Code). Without such subrogation, or assignment of rights, the herein plaintiff has no cause of
action against the defendants, and is, therefore, not the right party in interest as plaintiff.
"Last, but not least, assuming that the said change of name was legal and operated to
dissolve the original corporation, the dissolved corporation, must pursuant to Sec. 77 of our
corporation law, be deemed as continuing as a body corporate for three (3) years from March 8,
1961 for the purpose of prosecuting and defending suits. It is, therefore, the Yek Tong Lin Fire &
Marine Insurance Co., Ltd. that is the proper party to sue the defendants under said indemnity
agreement up to March 8, 1964.
"Having arrived at the foregoing conclusions, this Court need not squarely pass upon issue
(b) formulated above.
"WHEREFORE, plaintiff's action is hereby dismissed, with costs against the plaintiff."
In due time, the Philippine First Insurance Company, Inc. moved for reconsideration of the decision aforesaid,
but said motion was denied on December 3, 1962 in an order worded thus:
"The motion for reconsideration, dated November 8, 1962, raises no new issue that we
failed to consider in rendering our decision of October 6, 1962. However, it gives us an opportunity
to amplify our decision as regards the question of change of name of a corporation in this
jurisdiction.
"We find nothing in our Corporation Law authorizing a change of name of a corporation
organized pursuant to its provisions. Sec. 18 of the Corporation law authorizes, in our opinion,
amendment to the Articles of Incorporation of a corporation only as to matters other than its
corporate name. Once a corporation is organized in this jurisdiction by the execution and
registration of its Articles of Incorporation, it shall continue to exist under its corporate name for the
lifetime of its corporate existence fixed in its Articles of Incorporation, unless sooner legally
dissolved (Sec. 11, Corp. Law). Significantly, change of name is not one of the methods of
dissolution of corporations expressly authorized by our Corporation Law. Also significant is the fact
that the power to change its corporate name is not one of the general powers conferred on
corporations in this jurisdiction (Sec. 13, Corp. Law). The enumeration of corporate powers made
in our Corporation Law implies the exclusion of all others (Thomas v. West Jersey R. Co., 101 U.S.
71, 25 L. ed. 950). It is obvious, in this connection, that change of name is not one of the powers
necessary to the exercise of the powers conferred on Corporations by said Sec. 13 (see Sec. 14,
Corp. Law).
"To rule that Sec. 18 of our Corporation Law authorizes the change of name of a
corporation by amendment of its Articles of Incorporation is to indulge in judicial legislation. We
have examined the cases cited in Volume 13 of American Jurisprudence in support of the
proposition that the general power to alter or amend the charter of a corporation necessarily
includes the power to alter the name of a corporation, and find no justification for said conclusion
arrived at by the editors of American Jurisprudence. On the contrary, the annotations in favor of
plaintiff's view appear to have been based on decisions in cases where the statute itself expressly
authorizes change of corporate name by amendment of its Articles of Incorporation. The correct
rule in harmony with the provisions of our Corporation Law is well expressed in an English case as
follows:
'After a company has been completely registered without defect or omission, so as
to be incorporated by the name set forth in the deed of settlement, such incorporated
company has not the power to change its name . . . Although the King by his prerogative
might incorporate by a new name, and the newly named corporation might retain former
rights, and sometimes its former name also, . . . it never appears to be such an act as the
corporation could do by itself, but required the same power as created the corporation.
(Reg. v. Registrar of Joint Stock Cos. 10 Q.B. 839, 59 E.C.L. 839).'
The contrary view appears to represent the minority doctrine, judging from the annotations on
decided cases on the matter.
"The movant invokes as persuasive precedent the action of the Securities Commissioner
in tacitly approving the Amended Articles of Incorporation on May 26, 1961. We regret that we
cannot in good conscience lend approval to this action of the Securities and Exchange
Commissioner. We find no justification, legal, moral, or practical, for adhering to the view taken by
the Securities and Exchange Commissioner that the name of a corporation in the Philippines may
be changed by mere amendment of its Articles of Incorporation as to its corporate name. A change
of corporate name would serve no useful purpose, but on the contrary would most probably cause
confusion. Only a dubious purpose could inspire a change of a corporate name which, unlike a
natural person's name, was chosen by the incorporators themselves; and our Courts should not
lend their assistance to the accomplishment of dubious purposes.
"WHEREFORE, we hereby deny plaintiff's motion for reconsideration, dated November 8,
1962, for lack of merit."
In this appeal appellant contends that —
"I
"THE TRIAL COURT ERRED IN HOLDING THAT IN THIS JURISDICTION, THERE IS
NOTHING IN OUR CORPORATION LAW AUTHORIZING THE CHANGE OF CORPORATE
NAME;
"II
"THE TRIAL COURT ERRED IN DECLARING THAT A CHANGE OF CORPORATE
NAME APPEARS TO BE AGAINST PUBLIC POLICY;
"III
"THE TRIAL COURT ERRED IN HOLDING THAT A CHANGE OF CORPORATE NAME
HAS THE LEGAL EFFECT OF DISSOLVING THE ORIGINAL CORPORATION;
"IV
"THE TRIAL COURT ERRED IN HOLDING THAT THE CHANGE OF NAME OF THE YEK
TONG LIN FIRE & MARINE INSURANCE CO., LTD. IS OF DUBIOUS VALIDITY;
"V
"THE TRIAL COURT ERRED IN HOLDING THAT THE APPELLANT HEREIN IS NOT
THE RIGHT PARTY IN INTEREST TO SUE DEFENDANTS-APPELLEES;
"VI
"THE TRIAL COURT FINALLY ERRED IN DISMISSING THE COMPLAINT."
Appellant's position is correct; all the above assignments of error are well taken. The whole case, however,
revolves around only one question. May a Philippine corporation change its name and still retain its original personality
and individuality as such?
The answer is not difficult to find. True, under Section 6 of the Corporation Law, the first thing required to be
stated in the Articles of Incorporation of any corporation is its name, but it is only one among many matters equally if not
more important, that must be stated therein. Thus, it is also required, for example, to state the number and names of
and residences of the incorporators and the residence or location of the principal office of the corporation, its term of
existence, the amount of its capital stock and the number of shares into which it is divided, etc., etc.
On the other hand, Section 18 explicitly permits the articles of incorporation to be amended thus:
"Sec. 18. — Any corporation may for legitimate corporate purpose or purposes, amend its
articles of incorporation by a majority vote of its board of directors or trustees and the vote or
written assent of two-thirds of its members, if it be a nonstock corporation or, if it be a stock
corporation, by the vote or written assent of the stockholders representing at least two thirds of the
subscribed capital stock of the corporation: Provided, however, That if such amendment to the
articles of incorporation should consist in extending the corporate existence or in any change in the
rights of holders of shares of any class, or would authorize shares with preferences in any respect
superior to those of outstanding shares of any class, or would restrict the rights of any stockholder,
then any stockholder who did not vote for such corporate action may, within forty days after the
date upon which such action was authorized, object thereto in writing and demand payment for his
shares. If, after such a demand by a stockholder. the corporation and the stockholder cannot agree
upon the value of his share or shares at the time such corporate action was authorized, such value
shall be ascertained by three disinterested persons, one of whom shall be named by the
stockholder, another by the corporation, and the third by the two thus chosen. The findings of the
appraisers shall be final, and if their award is not paid by the corporation within thirty days after it is
made, it may be recovered in an action by the stockholder against the corporation. Upon payment
by the corporation to the stockholder of the agreed or awarded price of his share or shares, the
stockholder shall forthwith transfer and assign the share or shares held by him as directed by the
Corporation: Provided, however, That their own shares of stock purchased or otherwise acquired
by banks, trust companies, and insurance companies, should be disposed of within six months
after acquiring title thereto.
"Unless and until such amendment to the articles of incorporation shall have been
abandoned or the action rescinded, the stockholder making such demand in writing shall cease to
be a stockholder and shall have no rights with respect to such shares, except the right to receive
payment therefor as aforesaid.
"A stockholder shall not be entitled to payment for his shares under the provisions of this
section unless the value of the corporate assets which would remain after such payment would be
at least equal to the aggregate amount of its debts and liabilities and the aggregate par value
and/or issued value of the remaining subscribed capital stock.
"A copy of the articles of incorporation as amended, duly certified to be correct by the
president and the secretary of the corporation and a majority of the board of directors or trustees,
shall be filed with the Securities and Exchange Commissioner, who shall attach the same to the
original articles of incorporation, on file in his office. From the time of filing such copy of the
amended articles of incorporation, the corporation shall have the same powers and it and the
members and stockholders thereof shall thereafter be subject to the same liabilities as if such
amendment had been embraced in the original articles of incorporation: Provided, however, That
should the amendment consist in extending the corporate life, the extension shall not exceed 50
years in any one instance. Provided, further, That the original articles and amended articles
together shall contain all provisions required by law to be set out in the articles of incorporation:
And provided, further, That nothing in this section shall be construed to authorize any corporation
to increase or diminish its capital stock or so as to effect any rights or actions which accrued to
others between the time of filing the original articles of incorporation and the filing of the amended
articles.
"The Securities and Exchange Commissioner shall be entitled to collect and receive the
sum of ten pesos for filing said copy of the amended articles of incorporation. Provided, however,
That when the amendment consists in extending the term of corporate existence, the Securities
and Exchange Commissioner shall be entitled to collect and receive for the filing of its amended
articles of incorporation the same fees collectible under existing law for the filing of articles of
incorporation. The Securities & Exchange Commissioner shall not hereafter file any amendment to
the articles of incorporation of any bank, banking institution, or building and loan association unless
accompanied by a certificate of the Monetary Board (of the Central sank) to the effect that such
amendment is in accordance with law. (As further amended by Act No. 3610, Sec. 2 and Sec. 9.
R.A. No. 337 and R.A. No. 3531.)"
It can be gleaned at once that this section does not only authorize corporations to amend their charter; it also
lays down the procedure for such amendment; and, what is more relevant to the present discussion, it contains provisos
restricting the power to amend when it comes to the term of their existence and the increase or decrease of the capital
stock. There is no prohibition therein against the change of name. The inference is clear that such a change is allowed,
for if the legislature had intended to enjoin corporations from changing names, it would have expressly stated so in this
section or in any other provision of the law.
No doubt, "(the) name (of a corporation) is peculiarly important as necessary to the very existence of a
corporation. The general rule as to corporations is that each corporation shall have a name by which it is to sue and be
sued and do all legal acts. The name of a corporation in this respect designates the corporation in the same manner as
the name of an individual designates the person." 1 Since an individual has the right to change his name under certain
conditions, there is no Compelling reason why a corporation may not enjoy the same right. There is nothing sacrosanct
in a name when it comes to artificial beings, The sentimental considerations which individuals attach to their names are
not present in corporations and partnerships. Of course, as in the case of an individual, such change may not be made
exclusively by the corporation's own act. It has to follow the procedure prescribed by law for the purpose; and this is
what is important and indispensably prescribed — strict adherence to such procedure.
Local well known corporation law commentators are unanimous in the view that a corporation may change its
name by merely amending its charter in the manner prescribed by law. 2 American authorities which have persuasive
force here in this regard because our corporation law is of American origin, the same being a sort of codification of
American corporate law, 3 are of the same opinion.
"A general power to alter or amend the charter of a corporation necessarily includes the
power to alter the name of the corporation. Ft. Pitt Bldg., etc., Assoc. v. Model Plan Bldg., etc.,
Assoc., 159 Pa. St. 308, 28 Atl. 215; In re Fidelity Mut. Aid Assoc., 12 W.N.C. (Pa.) 271; Excelsior
Oil Co., 3 Pa. Co. Ct. 184; Wetherill Steel Casting Co., 5 Pa. Co. Ct. 337.
xxx xxx xxx
"Under the General Laws of Rhode Island, c 176, sec. 7, relating to an increase of the
capital stock of a corporation, it is provided that 'such agreement may be amended in and other
particular, excepting as provided in the following section', which relates to a decrease of the capital
stock. This section has been held to authorize a change in the name of a corporation. Armington v.
Palmer, 21 R.I. 109, 42 Atl. 308, 48 L.R.A. 95, 79 Am St. Rep. 786." (Vol. 19, American and
English Annotated Cases, p. 1239.)
Fletcher, a standard authority on American and English corporation law also says:
"Statutes are to be found in the various jurisdictions dealing with the matter of change in
corporate names. Such statutes have been subjected to judicial construction and have, in the
main, been upheld as constitutional. In direct terms or by necessary implication, they authorize
corporations to adopt new names and prescribe the mode of procedure for that purpose. The same
steps must be taken under some statutes to effect a change in a corporate name, as when any
other amendment of the corporate charter is sought . . . When the general law thus deals with the
subject, a corporation can change its name only in the manner provided." (6 Fletcher, Cyclopedia
of the Law of Private Corporations, 1968 Revised Volume, pp. 212213.) (Italic Ours)
The learned trial judge held that the above-quoted propositions are not supported by the weight of authority
because they are based on decisions in cases where the statutes expressly authorize change of corporate name by
amendment of the articles of incorporation. We have carefully examined these authorities and We are satisfied of their
relevance. Even Lord Denman who has been quoted by His Honor from In Reg. v. Registrar of Joint Stock Cos. 10,
Q.B., 59 E.C.L. maintains merely that the change of its name "never appears to be such an act as the corporation could
do for itself, but required the same power as created a corporation." What seems to have been overlooked, therefore, is
that the procedure prescribed by Section 18 of our Corporation Law for the amendment of corporate charters is
practically identical with that for the incorporation itself of a corporation.
In the appealed order of dismissal, the trial court made the observation that, according to this Court in Red Line
Transportation Co. v. Rural Transit Co., Ltd., 60 Phil. 549, 555, change of name of a corporation is against public policy.
We must clarify that such is not the import of Our said decision. What this Court held in that case is simply that:
"We know of no law that empowers the Public Service Commission or any court in this
jurisdiction to authorize one corporation to assume the name of another corporation as a trade
name. Both the Rural Transit Company, Ltd., and the Bachrach Motor Co., Inc., are Philippine
corporations and the very law of their creation and continued existence requires each to adopt and
certify a distinctive name. The incorporators 'constitute a body politic and corporate under the
name stated in the certificate.' (Section 11, Act No. 1459, as amended.) A corporation has
the ,power 'of succession by its corporate name.' (Section 13, ibid.) The name of a corporation is
therefore essential to its existence. It cannot change its name except in the manner provided by
the statute. By that name alone is it authorized to transact business. The law gives a corporation
no express or implied authority to assume another name that is unappropriated; still less that of
another corporation, which is expressly set apart for it and protected by the law. If any corporation
could assume at pleasure as an unregistered trade name the name of another corporation, this
practice would result in confusion and open the door to frauds and evasions and difficulties of
administration and supervision. The policy of the law as expressed in our corporation statute and
the Code of Commerce is clearly against such a practice. (Cf. Scarsdale Pub. Co. — Colonial
Press vs. Carter, 116 New York Supplement, 731; Svenska Nat. F. i. C. vs. Swedish Nat. Assn.,
205 Illinois [Appellate Courts], 428, 434.)"
In other words, what We have held to be contrary to public policy is the use by one corporation of the name of
another corporation as its trade name. We are certain no one will disagree that such an act can only "result in confusion
and open the door to frauds and evasions and difficulties of administration and supervision." Surely, the Red Line case
was not one of change of name.
Neither can We share the posture of His Honor that the change of name of a corporation results in its
dissolution. There is unanimity of authorities to the contrary.
"An authorized change in the name of a corporation has no more effect upon its identity as
a corporation than a change of name of a natural person has upon his identity. It does not affect
the rights of the corporation or lessen or add to its obligations. After a corporation has effected a
change in its name it should sue and be sued in its new name . . ." (13 Am. Jur. 276-277, citing
cases.)
"A mere change in the name of a corporation, either by the legislature or by the
corporators or stockholders under legislative authority, does not, generally speaking, affect the
identity of the corporation, nor in any way affect the rights, privileges, or obligations previously
acquired or incurred by it. Indeed, it has been said that a change of name by a corporation has no
more effect upon the identity of the corporation than a change of name by a natural person has
upon the identity of such person. The corporation, Upon such change in its name, is in no sense a
new corporation, nor the successor of the original one, but remains and continues to be the original
corporation. It is the same corporation with a different name, and its character is in no respect
changed . . ."(6 Fletcher, Cyclopedia of the Law of Private Corporations, 224-225, citing cases.)
"The change in the name of a corporation has no more effect upon its identity as a
corporation than a change of name of a natural person has upon his identity. It does not affect the
rights of the corporation, or lessen or add to its obligations.
"England. — Doe v. Norton, 11 M. & W. 913, 7 Jur. 751, 12 L.J. Exch. 418.
"United States. — Metropolitan Nat. Bank v. Claggett, 141 U.S. 520, 12 S. Ct. 60, 35 U.S.
(L. ed.) 841.
"Alabama. — Lomb v. Pioneer Sav., etc., Co., 106 Ala. 591, 17 So. 670; North
Birmingham Lumber Co. v. Sims, 157 Ala. 595, 48 So. 84.
"Connecticut. — Trinity Church v. Hall, 22 Com. 125.
"Illinois. — Mt. Palatine Academy v. Kleinschnitz, 28 Ill. 133; St. Louis, etc. R. Co. v. Miller,
43 Ill. 199; Reading v. Wedder, 66 Ill. 80.
"Indiana. — Rosenthal v. Madison, etc., Plank Road Co., 10 Ind. 358.
"Kentucky. — Cahill v. Bigger, 8 B. Mon. 211; Wilhite v. Convent of Good Shepherd, 177
Ky. 251, 78 S. W. 138.
Maryland. — Phinney v. Sheppard & Enoch Pratt Hospital, 88 Md. 633, 42 Atl. 58, writ of
error dismissed, 177 U.S. 170 20 S. Ct. 573, 44 U.S. (L. ed.) 720.
"Missouri. — Dean v. La Motte Lead Co., 59 Mo. 523.
"Nebraska. — Carlon v. City Sav. Bank, 82 Neb. 582, 188 N. W. 334.
"New York. — First Soc. of M.E. Church v. Brownell, 5 Hun 464.
"Pennsylvania. — Com. v. Pittsburgh, 41 Pa. St. 278.
"South Carolina. — South Carolina Mut. Ins. Co. v. Price 67 S.C. 207, 45 S.E. 173.
"Virginia. — Wilaon v. Chesapeake, etc., R. Co., 21: Gratt, 654; Wright-Caesar Tobacco
Co. v. Hoen, 105 Va. 327, 54 S.E. 309.
"Washington. — King v. Ilwaco R. etc., Co., 1 Wash. 127, 23 Pac. 924.
"Wisconsin. — Racine County Bank v. Ayers, 12 Wis. 512.
"The fact that the corporation by its old name makes a formal transfer of its property to the
corporation by its new name does not of itself show that the change in name has affected a
change in the identity of the corporation. Palfrey v. Association for Relief, etc., 110 La. 452, 34 So.
600. The fact that a corporation organized as a state bank afterwards becomes a national bank by
complying with the provisions of the National Banking Act, and changes its name accordingly, has
no effect on its right to sue upon obligations or liabilities incurred to it by its former name. Michigan
Ins. Bank v. Eldred, 143 U.S. 293, 12 S. Ct. 450, 36 U.S. (L. ed.) 162.
"A deed of land to a church by a particular name has been held not to be affected by the
fact that the church afterwards took a different name. Cahill v. Bigger, 8 B. Mon. (ky) 211.
"A change in the name of a corporation is not a divestiture of title or such a change as
requires a regular transfer of title to property, whether real or personal, from the corporation under
one name to the same corporation under another name. McCloskey v. Doherty, 97 Ky. 300, 30 S.
W. 649." (19 American and English Annotated Cases 1242-1243.)
As was very aptly said in Pacific Bank v. De Ro, 37 Cal. 538, "The changing of the name of a corporation is no
more the creation of a corporation than the changing of the name of a natural person is the begetting of a natural
person. The act, in both cases, would seem to be what the language which we use to designate it imports — a change
of name, and not a change of being."
Having arrived at the above conclusion, We have to agree with appellant's pose that the lower court also erred
in holding that it is not the right party in interest to sue defendants-appellees. 4 As correctly pointed out by appellant, the
approval by the stockholders of the amendment of its articles of incorporation changing the name "The Yek Tong Lin
Fire & Marine Insurance Co., Ltd." to "Philippine First Insurance Co., Inc." on March 8, 1961, did not automatically
change the name of said corporation on that date. To be effective, Section 18 of the Corporation Law, earlier quoted,
requires that "a copy of the articles of incorporation as amended, duly certified to be correct by the president and the
secretary of the corporation and a majority of the board of directors or trustees, shall be filed with the Securities &
Exchange Commissioner", and it is only from the time of such filing, that "the corporation shall have the same powers
and it and the members and stockholders thereof shall thereafter be subject to the same liabilities, as if such
amendment had been embraced in the original articles of incorporation." It goes without saying then that appellant
rightly acted in its old name when on May 15, 1961, it entered into the indemnity agreement, Annex A, with the
defendants-appellees; for only after the filing of the amended articles of incorporation with the Securities & Exchange
Commission on May 26, 1961, did appellant legally acquire its new name; and it was perfectly right for it to file the
present case in that new name on December 6, 1961. Such is, but the logical effect of the change of name of the
corporation upon its actions.
"Actions brought by a corporation after it has changed its name should be brought under
the new name although for the enforcement of rights existing at the time the change was made.
Lomb v. Pioneer Sav., etc., Co., 106 Ala. 591,17 So. 670; Newlan v. Lombard University, 62 Ill.
195; Thomas v. Visitors of Frederick County School, 7 Gill & J (M.d.) 388; Delaware, etc., R. Co. v.
Irick, 23 N. J. L. 321; Northumberland County Bank v. Eyer, 60 Pa. St. 436; Wilson v. Chesapeake,
etc., R. Co., 21 Gratt. (Va.) 654.
"The change in the name of the corporation does not affect its right to bring an action on a
note given to the corporation under its former name. Cumberland College v. Ish. 22 Cal 641;
Northwestern College v. Schwagler, 37 Ia. 577." (19 American and English Annotated Cases
1243.)
In consequence, We hold that the lower court erred in dismissing appellant's complaint. We take this
opportunity, however, to express the Court's feeling that it is apparent that appellee's position is more technical than
otherwise. Nowhere in the record is it seriously pretended that the indebtedness sued upon has already been paid. If
appellees entertained any fear that they might against be made liable to Yek Tong Lin Fire & Marine Insurance Co. Ltd.,
or to someone else in its behalf, a cursory examination of the records of the Securities & Exchange Commission would
have sufficed to clear up the fact that Yek Tong Lin had just changed its name but it had not ceased to be their creditor.
Everyone should realize that when the time of the courts is utilized for cases which do not involve substantial questions
and the claim of one of the parties therein is based on pure technicality that can at most delay only the ultimate outcome
necessarily adverse to such party because it has no real cause on the merits, grave injustice is committed to
numberless litigants whose meritorious cases cannot be given all the needed time by the courts. We address this
appeal once more to all members of the bar, in particular, since it is their bounden duty to the profession and to our
country and people at large to help ease as fast as possible the clogged dockets of the courts. Let us not wait until the
people resort to other means to secure speedy, just and inexpensive determination of their cases.
WHEREFORE, judgment of the lower court is reversed, and this case is remanded to the trial court for further
proceedings consistent herewith. With costs against appellees.
||| (Philippine First Insurance Co., Inc. v. Hartigan, G.R. No. L-26370, [July 31, 1970], 145 PHIL 310-329)

[G.R. No. 157900. July 22, 2013.]

ZUELLIG FREIGHT AND CARGO SYSTEMS, petitioner, vs. NATIONAL LABOR RELATIONS
COMMISSION AND RONALDO V. SAN MIGUEL, respondents.

DECISION

BERSAMIN, J p:

The mere change in the corporate name is not considered under the law as the creation of a new corporation;
hence, the renamed corporation remains liable for the illegal dismissal of its employee separated under that guise.
The Case
Petitioner employer appeals the decision promulgated on November 6, 2002, 1 whereby the Court of Appeals (CA)
dismissed its petition for certiorari and upheld the adverse decision of the National Labor Relations Commission (NLRC)
finding respondent Ronaldo V. San Miguel to have been illegally dismissed.
Antecedents
San Miguel brought a complaint for unfair labor practice, illegal dismissal, non-payment of salaries and moral
damages against petitioner, formerly known as Zeta Brokerage Corporation (Zeta). 2 He alleged that he had been a
checker/customs representative of Zeta since December 16, 1985; that in January 1994, he and other employees of Zeta
were informed that Zeta would cease operations, and that all affected employees, including him, would be separated; that
by letter dated February 28, 1994, Zeta informed him of his termination effective March 31, 1994; that he reluctantly
accepted his separation pay subject to the standing offer to be hired to his former position by petitioner; and that on April 15,
1994, he was summarily terminated, without any valid cause and due process.
San Miguel contended that the amendments of the articles of incorporation of Zeta were for the purpose of
changing the corporate name, broadening the primary functions, and increasing the capital stock; and that such
amendments could not mean that Zeta had been thereby dissolved. 3
On its part, petitioner countered that San Miguel's termination from Zeta had been for a cause authorized by the
Labor Code; that its non-acceptance of him had not been by any means irregular or discriminatory; that its predecessor-in-
interest had complied with the requirements for termination due to the cessation of business operations; that it had no
obligation to employ San Miguel in the exercise of its valid management prerogative; that all employees had been given
sufficient time to make their decision whether to accept its offer of employment or not, but he had not responded to its offer
within the time set; that because of his failure to meet the deadline, the offer had expired; that he had nonetheless been
hired on a temporary basis; and that when it decided to hire another employee instead of San Miguel, such decision was not
arbitrary because of seniority considerations. 4 EcHTCD
Decision of the Labor Arbiter
On November 15, 1999, Labor Arbiter Francisco A. Robles rendered a decision holding that San Miguel had been
illegally dismissed, 5 to wit:
Contrary to respondents' claim that Zeta ceased operations and closed its business, we believe
that there was merely a change of business name and primary purpose and upgrading of stocks of the
corporation. Zuellig and Zeta are therefore legally the same person and entity and this was admitted by
Zuellig's counsel in its letter to the VAT Department of the Bureau of Internal Revenue on 08 June 1994
(Reply, Annex "A"). As such, the termination of complainant's services allegedly due to cessation of
business operations of Zeta is deemed illegal. Notwithstanding his receipt of separation benefits from
respondents, complainant is not estopped from questioning the legality of his dismissal. 6
xxx xxx xxx
WHEREFORE, in view of the foregoing, complainant is found to have been illegally dismissed.
Respondent Zuellig Freight and Cargo Systems, Inc. is hereby ordered to pay complainant his
backwages from April 1, 1994 up to November 15, 1999, in the amount of THREE HUNDRED TWENTY
FOUR THOUSAND SIX HUNDRED FIFTEEN PESOS (P324,615.00).
The same respondent is ordered to pay the complainant Ronaldo San Miguel attorney's fees
equivalent to ten percent (10%) of the total award.
All other claims are dismissed.
SO ORDERED. 7
Decision of the NLRC
Petitioner appealed, but the NLRC issued a resolution on April 4, 2001, 8 affirming the decision of the Labor Arbiter.
The NLRC later on denied petitioner's motion for reconsideration via its resolution dated June 15, 2001. 9
Decision of the CA
Petitioner then filed a petition for certiorari in the CA, imputing to the NLRC grave abuse of discretion amounting to
lack or excess of jurisdiction, as follows:
1. In failing to consider the circumstances attendant to the cessation of business of Zeta;
2. In failing to consider that San Miguel failed to meet the deadline Zeta fixed for its employees to
accept the offer of petitioner for re-employment;
3. In failing to consider that San Miguel's employment with petitioner from April 1 to 15, 1994
could in no way be interpreted as a continuation of employment with Zeta;
4. In admitting in evidence the letter dated January 21, 1994 of petitioner's counsel to the Bureau
of Internal Revenue; and
5. In awarding attorney's fees to San Miguel based on Article 2208 of the Civil Code and Article
111 of the Labor Code.
On November 6, 2002, the CA promulgated its assailed decision dismissing the petition for certiorari, 10 viz.:
A careful perusal of the records shows that the closure of business operation was not validly
made. Consider the Certificate of Filing of the Amended Articles of Incorporation which clearly shows that
petitioner Zuellig is actually the former Zeta as per amendment dated January 21, 1994. The same
observation can be deduced with respect to the Certificate of Filing of Amended By-Laws dated May 10,
1994. As aptly pointed out by private respondent San Miguel, the amendment of the articles of
incorporation merely changed its corporate name, broadened its primary purpose and increased its
authorized capital stocks. The requirements contemplated in Article 283 were not satisfied in this case.
Good faith was not established by mere registration with the Securities and Exchange Commission (SEC)
of the Amended Articles of Incorporation and By-Laws. The factual milieu of the case, considered in its
totality, shows that there was no closure to speak of. The termination of services allegedly due to
cessation of business operations of Zeta was illegal. Notwithstanding private respondent San Miguel's
receipt of separation benefits from petitioner Zuellig, the former is not estopped from questioning the
legality of his dismissal. HAIDcE
Petitioner Zuellig's allegation that the five employees who refused to receive the termination
letters were verbally informed that they had until 6:00 p.m. of March 1, 1994 to receive the termination
letters and sign the employment contracts, otherwise the former would be constrained to withdraw its
offer of employment and seek for replacements in order to ensure the smooth operations of the new
company from its opening date, is of no moment in view of the foregoing circumstances. There being no
valid closure of business operations, the dismissal of private respondent San Miguel on alleged
authorized cause of cessation of business pursuant to Article 283 of the Labor Code,was utterly illegal.
Despite verbal notice that the employees had until 6:00 p.m. of March 1, 1994 to receive the termination
letters and sign the employment contracts, the dismissal was still illegal for the said condition is null and
void. In point of facts and law, private respondent San Miguel remained an employee of petitioner Zuellig.
If at all, the alleged closure of business operations merely operates to suspend employment relation
since it is not permanent in character.
Where there is no showing of a clear, valid, and legal cause for the termination of employment,
the law considers the matter a case of illegal dismissal and the burden is on the employer to prove that
the termination was for a valid or authorized cause.
Findings of facts of the NLRC, particularly when both the NLRC and Labor Arbiter are in
agreement, are deemed binding and conclusive upon the Supreme Court.
As regards the second and last argument advanced by petitioner Zuellig that private respondent
San Miguel is not entitled to attorney's fees, this Court finds no reason to disturb the ruling of the public
respondent NLRC. Petitioner Zuellig maintains that the factual backdraft (sic) of this petition does not call
for the application of Article 2208 of the Civil Code and Article 111 of the Labor Code as private
respondent's wages were not withheld. On the other hand, public respondent NLRC argues that
paragraphs 2 and 3, Article 2208 of the Civil Code and paragraph (a), Article 111 of the Labor Code
justify the award of attorney's fees. NLRC was saying to the effect that by petitioner Zuellig's act of
illegally dismissing private respondent San Miguel, the latter was compelled to litigate and thus incurred
expenses to protect his interest. In the same passion, private respondent San Miguel contends that
petitioner Zuellig acted in gross and evident bad faith in refusing to satisfy his plainly valid, just and
demandable claim.
After careful and judicious evaluation of the arguments advanced to support the propriety or
impropriety of the award of attorney's fees to private respondent San Miguel, this Court finds the
resolutions of public respondent NLRC supported by laws and jurisprudence. It does not need much
imagination to see that by reason of petitioner Zuellig's feigned closure of business operations, private
respondent San Miguel incurred expenses to protect his rights and interests. Therefore, the award of
attorney's fees is in order.
WHEREFORE, in view of the foregoing, the resolutions dated April 4, 2001 and June 15, 2001 of
the National Labor Relations Commission affirming the November 15, 1999 decision of the Labor Arbiter
in NLRC NCR 05-03639-94 (CA No. 022861-00) are hereby AFFIRMED and the instant petition for
certiorari is hereby DENIED and ordered DISMISSED.
SO ORDERED.
Hence, petitioner appeals.
Issues
Petitioner asserts that the CA erred in holding that the NLRC did not act with grave abuse of discretion in ruling that
the closure of the business operation of Zeta had not been bona fide, thereby resulting in the illegal dismissal of San Miguel;
and in holding that the NLRC did not act with grave abuse of discretion in ordering it to pay San Miguel attorney's fees. 11
In his comment, 12 San Miguel counters that the CA correctly found no grave abuse of discretion on the part of the
NLRC because the ample evidence on record showed that he had been illegally terminated; that such finding accorded with
applicable laws and jurisprudence; and that he was entitled to back wages and attorney's fees. HIACac
In its reply, 13 petitioner reiterates that the cessation of Zeta's business, which resulted in the severance of San
Miguel from his employment, was valid; that the CA erred in upholding the NLRC's finding that San Miguel had been illegally
terminated; that his acknowledgment of the validity of his separation from Zeta by signing a quitclaim and waiver estopped
him from claiming that it had subsequently employed him; and that the award of attorney's fees had no basis in fact and in
law.
Ruling
The petition for review on certiorari is denied for its lack of merit.
First of all, the outcome reached by the CA that the NLRC did not commit any grave abuse of discretion was borne
out by the records of the case. We cannot undo such finding without petitioner making a clear demonstration to the Court
now that the CA gravely erred in passing upon the petition for certiorari of petitioner.
Indeed, in a special civil action for certiorari brought against a court or quasi-judicial body with jurisdiction over a
case, petitioner carries the burden of proving that the court or quasi-judicial body committed not a merely reversible error
but a grave abuse of discretion amounting to lack or excess of jurisdiction in issuing the impugned order. 14 Showing mere
abuse of discretion is not enough, for it is necessary to demonstrate that the abuse of discretion was grave. Grave abuse of
discretion means either that the judicial or quasi-judicial power was exercised in an arbitrary or despotic manner by reason
of passion or personal hostility, or that the respondent judge, tribunal or board evaded a positive duty, or virtually refused to
perform the duty enjoined or to act in contemplation of law, such as when such judge, tribunal or board exercising judicial or
quasi-judicial powers acted in a capricious or whimsical manner as to be equivalent to lack of jurisdiction. 15 Under the
circumstances, the CA committed no abuse of discretion, least of all grave, because its justifications were supported by the
records and by the applicable laws and jurisprudence.
Secondly, it is worthy to point out that the Labor Arbiter, the NLRC, and the CA were united in concluding that the
cessation of business by Zeta was not a bona fide closure to be regarded as a valid ground for the termination of
employment of San Miguel within the ambit of Article 283 of the Labor Code. The provision pertinently reads:
Article 283. Closure of establishment and reduction of personnel. — The employer may also
terminate the employment of any employee due to the installation of labor-saving devices, redundancy,
retrenchment to prevent losses or the closing or cessation of operation of the establishment or
undertaking unless the closing is for the purpose of circumventing the provisions of this Title, by
serving a written notice on the workers and the Department of Labor and Employment at least one
(1) month before the intended date thereof. . . . .
The unanimous conclusions of the CA, the NLRC and the Labor Arbiter, being in accord with law, were not tainted
with any abuse of discretion, least of all grave, on the part of the NLRC. Verily, the amendments of the articles of
incorporation of Zeta to change the corporate name to Zuellig Freight and Cargo Systems, Inc. did not produce the
dissolution of the former as a corporation. For sure, the Corporation Code defined and delineated the different modes of
dissolving a corporation, and amendment of the articles of incorporation was not one of such modes. The effect of the
change of name was not a change of the corporate being, for, as well stated in Philippine First Insurance Co., Inc. v.
Hartigan: 16 "The changing of the name of a corporation is no more the creation of a corporation than the changing of the
name of a natural person is begetting of a natural person. The act, in both cases, would seem to be what the language
which we use to designate it imports — a change of name, and not a change of being."
The consequences, legal and otherwise, of the change of name were similarly dealt with in P.C. Javier & Sons, Inc.
v. Court of Appeals, 17 with the Court holding thusly: EaSCAH
From the foregoing documents, it cannot be denied that petitioner corporation was aware of First
Summa Savings and Mortgage Bank's change of corporate name to PAIC Savings and Mortgage Bank,
Inc. Knowing fully well of such change, petitioner corporation has no valid reason not to pay because the
IGLF loans were applied with and obtained from First Summa Savings and Mortgage Bank. First Summa
Savings and Mortgage Bank and PAIC Savings and Mortgage Bank, Inc., are one and the same bank to
which petitioner corporation is indebted. A change in the corporate name does not make a new
corporation, whether effected by a special act or under a general law. It has no effect on the
identity of the corporation, or on its property, rights, or liabilities. The corporation, upon such
change in its name, is in no sense a new corporation, nor the successor of the original
corporation. It is the same corporation with a different name, and its character is in no respect
changed. (Bold underscoring supplied for emphasis)
In short, Zeta and petitioner remained one and the same corporation. The change of name did not give petitioner
the license to terminate employees of Zeta like San Miguel without just or authorized cause. The situation was not similar to
that of an enterprise buying the business of another company where the purchasing company had no obligation to rehire
terminated employees of the latter. 18 Petitioner, despite its new name, was the mere continuation of Zeta's corporate
being, and still held the obligation to honor all of Zeta's obligations, one of which was to respect San Miguel's security of
tenure. The dismissal of San Miguel from employment on the pretext that petitioner, being a different corporation, had no
obligation to accept him as its employee, was illegal and ineffectual.
And, lastly, the CA rightfully upheld the NLRC's affirmance of the grant of attorney's fees to San Miguel. Thereby,
the NLRC did not commit any grave abuse of its discretion, considering that San Miguel had been compelled to litigate and
to incur expenses to protect his rights and interest. In Producers Bank of the Philippines v. Court of Appeals, 19 the Court
ruled that attorney's fees could be awarded to a party whom an unjustified act of the other party compelled to litigate or to
incur expenses to protect his interest. It was plain that petitioner's refusal to reinstate San Miguel with backwages and other
benefits to which he had been legally entitled was unjustified, thereby entitling him to recover attorney's fees.
WHEREFORE, the Court AFFIRMS the decision of the Court of Appeals promulgated on November 6, 2002; and
ORDERS petitioner to pay the costs of suit. SO ORDERED.
||| (Zuellig Freight and Cargo Systems v. National Labor Relations Commission, G.R. No. 157900, [July 22, 2013], 714 PHIL
401-413)

[G.R. No. L-28113. March 28, 1969.]

THE MUNICIPALITY OF MALABANG, LANAO DEL SUR and AMER MACAORAO BALINDONG,
petitioners, vs. PANGANDAPUN BENITO, HADJI NORODIN MACAPUNUNG, HADJI HASAN
MACARAMPAD, FREDERICK V. DUJERTE, MONDACO ONTAL, MARONSONG ANDOY,
MACALABA INDAR LAO, respondents.

L. Amores and R. Gonzales for petitioners.


Jose W . Diokno for respondents.

SYLLABUS

1. ADMINISTRATIVE LAW; MUNICIPAL CORPORATIONS; RIGHT OF INDIVIDUAL TO ATTACK


CORPORATION COLLATERALLY. — It is indeed true that, generally, an inquiry into the legal existence of a
municipality is reserved to the State in a proceeding for quo warranto or other direct proceeding, and that only in a few
exceptions may a private person exercise this function of government. But the rule disallowing collateral attacks applies
only where the municipal corporation is at least a de facto corporation. For where it is neither a corporation de jure nor
de facto, but a nullity, the rule is that its existence may be questioned collaterally or directly in any action or proceeding
by any one whose rights or interests are affected thereby, including the citizens of the territory incorporated unless they
are estopped by their conduct from doing so.
2. ID.; ID.; MUNICIPALITY IN QUESTION IS NOT A DE FACTO CORPORATION. — In the cases where a de
facto municipal corporation was recognized as such despite the fact that the statute creating it was later invalidated, the
decisions could fairly be made to rest on the consideration that there was some other valid law giving corporate validity
to the organization. Hence, in the case at bar, the mere fact that Balabagan was organized at a time when the statute
had not been invalidated cannot conceivably make it a de facto corporation, as, independently of Section 68 of the
Administrative Code, there is no other valid statute to give color of authority to its creation.
3. ID.; ID.; EFFECT OF NULLITY OF EXECUTIVE ORDER CREATING MUNICIPALITY UPON ACTS
THEREOF BEFORE DECLARATION OF NULLITY. — Executive Order 386 creating the municipality in question is a
nullity pursuant to the ruling in Pelaez vs. Auditor General and Municipality of San Joaquin vs. Siva. The executive order
therefore "created no office." This is not to say, however, that the acts done by the municipality of Balabagan in the
exercise of its corporate powers are a nullity because the executive order "is, in legal contemplation, as inoperative as
though it had never been passed." For the existence of Executive Order 386 is "an operative fact which cannot justly be
ignored." There is then no basis for the respondents' apprehension that the invalidation of the executive order creating
Balabagan would have the effect of unsettling many an act done in reliance upon the validity of the creation of that
municipality.

DECISION

CASTRO, J p:

The petitioner Amer Macaorao Balindong is the mayor of Malabang, Lanao del Sur, while the respondent
Pangandapun Benito is the mayor, and the rest of the respondents are the councilors, of the municipality of Balabagan
of the same province. Balabagan was formerly a part of the municipality of Malabang, having been created on March
15, 1960, by Executive Order 386 of the then President Carlos P. Garcia, out of barrios and sitios 1 of the latter
municipality.
The petitioners brought this action for prohibition to nullify Executive Order 386 and to restrain the respondent
municipal officials from performing the functions of their respective offices, relying on the ruling of this Court in Pelaez v.
Auditor General 2 and Municipality of San Joaquin v. Siva. 3
In Pelaez this Court, through Mr. Justice (now Chief Justice) Concepcion, ruled: (1) that Section 23 of Republic
Act 2370 [Barrio Charter Act, approved January 1, 1960], by vesting the power to create barrios in the provincial board,
is a "statutory denial of the presidential authority to create a new barrio [and] implies a negation of the bigger power to
create municipalities," and (2) that Section 68 of the Administrative Code, insofar as it gives the President the power to
create municipalities, is unconstitutional (a) because it constitutes an undue delegation of legislative power and (b)
because it offends against Section 10 (1) of Article VII of the Constitution, which limits the President's power over local
governments to mere supervision. As this Court summed up its discussion: "In short, even if it did not entail an undue
delegation of legislative powers, as it certainly does, said Section 68, as part of the Revised Administrative Code,
approved on March 10, 1917, must be deemed repealed by the subsequent adoption of the Constitution, in 1935, which
is utterly incompatible and inconsistent with said statutory enactment."
On the other hand, the respondents, while admitting the facts alleged in the petition, nevertheless argue that
the rule announced in Pelaez can have no application in this case because unlike the municipalities involved in Pelaez,
the municipality of Balabagan is at least a de facto corporation, having been organized under color of a statute before
this was declared unconstitutional, its officers having been either elected or appointed, and the municipality itself having
discharged its corporate functions for the past five years preceding the institution of this action. It is contended that as a
de facto corporation, its existence cannot be collaterally attacked, although it may be inquired into directly in an action
for quo warranto at the instance of the State and not of an individual like the petitioner Balindong.
It is indeed true that, generally, an inquiry into the legal existence of a municipality is reserved to the State in a
proceeding for quo warranto or other direct proceeding, and that only in a few exceptions may a private person exercise
this function of government. 4 But the rule disallowing collateral attacks applies only where the municipal corporation is
at least a de facto corporation. 5 For where it is neither a corporation de jure nor de facto, but a nullity, the rule is that its
existence may be questioned collaterally or directly in any action or proceeding by any one whose rights or interests are
affected thereby, including the citizens of the territory incorporated unless they are estopped by their conduct from doing
so. 6
And so the threshold question is whether the municipality of Balabagan is a de facto corporation. As earlier
stated, the claim that it is rests on the fact that it was organized before the promulgation of this Court's decision in
Pelaez. 7
Accordingly, we address ourselves to the question whether a statute can lend color of validity to an attempted
organization of a municipality despite the fact that such statute is subsequently declared unconstitutional.
This has been a litigiously prolific question, sharply dividing courts in the United States. Thus, some hold that a
de facto corporation cannot exist where the statute or charter creating it is unconstitutional because there can be no de
facto corporation where there can be no de jure one, 8 while others hold otherwise on the theory that a statute is
binding until it is condemned as unconstitutional. 9
An early article in the Yale Law Journal offers the following analysis:
"It appears that the true basis for denying to the corporation a de facto status lay in the absence
of any legislative act to give vitality to its creation. An examination of the cases holding, some of them
unreservedly, that a de facto office or municipal corporation can exist under color of an unconstitutional
statute will reveal that in no instance did the invalid act give life to the corporation, but that either in other
valid acts or in the constitution itself the office or the corporation was potentially created . . .
"The principle that color of title under an unconstitutional statute can exist only where there is
some other valid law under which the organization may be effected, or at least an authority in potencia by
the state constitution, has its counterpart in the negative propositions that there can be no color of
authority in an unconstitutional statute that plainly so appears on its face or that attempts to authorize the
ousting of a de jure or de facto municipal corporation upon the same territory, in the one case the fact
would imply the imputation of had faith, in the other the new organization must be regarded as a mere
usurper . . .
"As a result of this analysis of the cases the following principles may be deduced which seem to
reconcile the apparently conflicting decisions:
"I. The color of authority requisite to the organization of a de facto municipal corporation may be:
"1. A valid law enacted by the legislature.
"2. An unconstitutional law, valid on its face, which has either (a) been upheld for a time by the
courts or (b) not yet been declared void; provided that a warrant for its creation can be found in some
other valid law or in the recognition of its potential existence by the general laws or constitution of the
state.
"II. There can be no de facto municipal corporation unless either directly or potentially, such a de
jure corporation is authorized by some legislative fiat.
"III. There can be no color of authority in an unconstitutional statute alone, the invalidity of which
is apparent on its face.
"IV. There can be no de facto corporation created to take place of an existing de jure corporation,
as such organization would clearly be an usurper." 10
In the cases where a de facto municipal corporation was recognized as such despite the fact that the statute
creating it was later invalidated, the decisions could fairly be made to rest on the consideration that there was some
other valid law giving corporate vitality to the organization. Hence, in the case at bar, the mere fact that Balabagan was
organized at a time when the statute had not been invalidated cannot conceivably make it a de facto corporation, as,
independently of the Administrative Code provision in question, there is no other valid statute to give color of authority to
its creation. Indeed, in Municipality of San Joaquin v. Siva, 11 this Court granted a similar petition for prohibition and
nullified an executive order creating the municipality of Lawigan in Iloilo on the basis of the Pelaez ruling, despite the
fact that the municipality was created in 1961, before Section 68 of the Administrative Code, under which the President
had acted, was invalidated. Of course the issue of de facto municipal corporation did not arise in that case.
In Norton v. Shelby County, 12 Mr. Justice Field said: "An unconstitutional act is not a law; it confers no rights; it
imposes no duties; it affords no protection; it creates no office; it is, in legal contemplation, as inoperative as though it
had never been passed." Accordingly, he held that bonds issued by a board of commissioners created under an invalid
statute were unenforceable.
Executive Order 386 "created no office." This is not to say, however, that the acts done by the municipality of
Balabagan in the exercise of its corporate powers are a nullity because the executive order "is, in legal contemplation,
as inoperative as though it had never been passed." For the existence of Executive Order 386 is "an operative fact
which cannot justly be ignored." As Chief Justice Hughes explained in Chicot County Drainage District v. Baxter State
Bank: 13
"The courts below have proceeded on the theory that the Act of Congress, having been found to
be unconstitutional, was not a law; that it was inoperative, conferring no rights and imposing no duties,
and hence affording no basis for the challenged decree. Norton v. Shelby County, 118 U.S. 425, 442;
Chicago, I. & L. Ry. Co. v. Hackett, 228 U.S. 559, 566. It is quite clear, however, that such broad
statements as to the effect of a determination of unconstitutionality must be taken with qualifications. The
actual existence of a statute, prior to such a determination, in an operative fact and may have
consequences which cannot justly be ignored. The past cannot always be erased by a new judicial
declaration. The effect of the subsequent ruling as to invalidity may have to be considered in various
aspects — with respect to particular relations, individual and corporate, and particular conduct, private
and official. Questions of rights claimed to have become vested, of status, of prior determinations
deemed to have finality and acted upon accordingly, of public policy in the light of the nature both of the
statute and of its previous application, demand examination. These questions are among the most
difficult of those which have engaged the attention of courts, state and federal, and it is manifest from
numerous decisions that an all- inclusive statement of a principle of absolute retroactive invalidity cannot
be justified."
There is then no basis for the respondents' apprehension that the invalidation of the executive order creating
Balabagan would have the effect of unsettling many an act done in reliance upon the validity of the creation of that
municipality. 14
ACCORDINGLY, the petition is granted, Executive Order 386 is declared void, and the respondents are hereby
permanently restrained from performing the duties and functions of their respective offices. No pronouncement as to
costs.
||| (Municipality of Malabang v. Benito, G.R. No. L-28113, [March 28, 1969], 137 PHIL 358-370)

HARRILL v. DAVIS et al.

No. 2,805

Circuit Court of Appeals, Eighth Circuit

168 F. 187; 1909 U.S. App. LEXIS 4434

March 2, 1909

PRIOR HISTORY: [**1] In Error to the United States Court of Appeals in the Indian Territory.

CORE TERMS: stock, de facto, cotton, color, articles of incorporation, incorporation, milling, individual liability, charter,
actively, supposed, promoters, estopped, ginning, buying, commencement, stockholder, franchise, gin, cotton gin, selling,
general manager, good faith, pretended, assurance, exempt, clerk, incur, user, estoppel

COUNSEL: R. C. Allen (J. C. Pinson, on the brief), for plaintiff in error.

Geo. A. Murphey (S. M. Rutherford, W. T. Hutchings, and W. P. Z. German, on the brief), for defendants in error.

OPINION BY: SANBORN

OPINION

[*190] Before SANBORN and VAN DEVANTER, Circuit Judges, and W. H. MUNGER, District Judge.
SANBORN, Circuit Judge. The patent and indisputable facts in this case are that the four defendants associated themselves
together, and from June, 1902, until December 22, 1902, actively engaged in purchasing lumber, material, and labor of the
plaintiff, and in constructing a cotton gin under the name "The Coweta Gin Company," and in conducting the business of
buying, selling, and ginning cotton for profit under the name "The Coweta Cotton & Milling Company," and that during this
time they incurred more than $4,700 of the indebtedness of $5,145.48 for which this action was brought. On December 22,
1902, they made their first real attempt to incorporate, and for the first time took on the color or appearance of a corporation.
On that day they filed articles [*191] of incorporation with the clerk of the Court [**2] of Appeals, but they never filed any
duplicate of them with the clerk of the judicial district in which their place of business was located, as required by the
statutes in order to constitute them a legal corporation and to authorize them to do business as such. Act Feb. 18, 1901, c.
379, 31 Stat. 794; Mansfield's Dig. Laws Ark. §§ 960, 968, 979.
The general rule is that parties who associate themselves together and actively engage in business for profit under any
name are liable as partners for the debts they incur under that name. It is an exception to this rule that such associates may
escape individual liability for such debts by a compliance with incorporation laws or by a real attempt to comply with them
which gives the color of a legal corporation, and by the user of the franchise of such a corporation in the honest belief that it
is duly incorporated. When the fact appears, as it does in the case at bar, by indisputable evidence that parties associated
and knowingly incurred liabilities under a given name, the legal presumption is that they are governed by the general rule,
and the burden is upon them to prove that they fall under some exception to it. Owen v. Shepard, [**3] 59 Fed. 746, 8
C.C.A. 244; Wechselberg v. Flour City National Bank, 64 Fed. 90, 94, 12 C.C.A. 56, 60, 61, 26 L.R.A. 470; Clark v. Jones,
87 Ala. 474, 6 South. 362.
Counsel for the defendants argue with much force and persuasiveness that they escape liability because they became a
corporation de facto, although they concede that they never became a corporation de jure, and in support of this position
they cite, among other cases: Wells Company v. Gastonia Cotton Mfg. Co., 198 U.S. 177, 25 Sup. Ct. 640, 49 L. Ed. 1003;
Andes v. Ely, 158 U.S. 312, 322, 15 Sup. Ct. 954, 39 L. Ed. 996; New Orleans Debenture Redemption Co. v. Louisiana, 180
U.S. 320, 327, 21 Sup. Ct. 378, 45 L. Ed. 550; Gartside Coal Co. v. Maxwell (C.C.) 22 Fed. 197; Johnson v. Okerstrom, 70
Minn. 303, 73 N.W. 147; Tennessee Automatic Lighting Company v. Massey (Tenn. Ch. App.) 56 S.W. 35; Finnegan v.
Noerenberg, 52 Minn. 239, 53 N.W. 1150, 18 L.R.A. 778, 38 Am. St. Rep. 552; Doty v. Patterson, 155 Ind. 60, 56 N.E. 668;
Merchants' National Bank v. Stone, 38 Mich. 779; Gow v. Collin Lumber Company, 109 Mich. 45, 66 N.W. 676, 678; Eaton
v. Aspinwall, 19 N.Y. 119; Leonardsville Bank v. Willard, 25 N.Y. 574; Cahall v. Citizens' [**4] Mutual Bldg. Ass'n, 61 Ala.
232; Fay v. Noble, 7 Cush. (Mass.) 188, 192, 193; Snider Sons' Company v. Troy, 91 Ala. 224, 8 South. 658, 11 L.R.A. 515,
24 Am. St. Rep. 887; Cochran v. Arnold, 58 Pa. 399, 404; Laflin & Rand Powder Co. v. Sinsheimer, 46 Md. 315, 321, 24
Am. Rep. 522; Rutherford v. Hill, 22 Or. 218, 29 Pac. 546, 17 L.R.A. 549, 29 Am. St Rep. 596. But in every one of these
authorities articles of incorporation had been filed under a general enabling act, or a charter had been issued and there had
been a user of the franchise of the supposed corporation which had been colorably created by the filing of the articles or the
issue of the charter before the indebtedness in question was created, while nothing of this nature had been done before the
debt for the $4,700 which we are now considering was incurred. The authorities which [*192] have been recited rest upon
the proposition that where parties procure a charter or file articles of association under a general law, thereby secure the
color of a legal incorporation, believe that they are a corporation, and use the supposed franchise of the corporation in good
faith, and third parties deal with them as a corporation, [**5] they become a corporation de facto and exempt from
individual liability to such third parties, although there are unknown defects in the proceedings for their incorporation. The
statement of Morawetz on Corporations, at section 748, upon which counsel seem to rely, that:
"If an association assumes to enter into a contract in a corporate capacity, and the party dealing with the association
contracts with it as if it were a corporation, the individual members of the association cannot be charged as parties to the
contract, either severally or jointly, or as partners. This is equally true whether the association was in fact a corporation or
not, and whether the contract with the association in its corporate capacity was authorized by the Legislature or prohibited
by law, or illegal"

is too broad to be sound. Parties who actively engage in business for profit under the name and pretense of a corporation
which they know neither exists nor has any color of existence may not escape individual liability because strangers are led
by their pretense to contract with their pretended entity as a corporation. In such cases they act as the agents of a principal
that they know does not [**6] exist, and they are liable under a familiar rule, because there is no responsible principal. 2
Kent's Commentaries (14th Ed.) 630; Queen City Furniture & Carpet Co. v. Crawford, 127 Mo. 356, 364, 30 S.W. 163. The
burden is not on the strangers who deal with them as a corporation, but on themselves who act under the name of a
pretended corporation, to see that it is so organized that it exempts them from individual liability, and if they fail in this they
must pay the liabilities they incur, even in the absence of fraud or bad faith, upon the salutary principle that where one of
two parties must suffer he must bear the loss whose breach of duty caused it.
There are cases in which stockholders who took no active part in the business of a pretended corporation which was acting
without any charter or filed articles, who supposed that the corporation was duly organized, have been held exempt from
individual liability for the debts it incurred; but if they had been actively conducting its business with knowledge of its lack of
incorporation, those decisions must have been otherwise. Seacord v. Pendleton, 55 Hun, 579, 9 N.Y. Supp. 46; Fuller v.
Rowe, 57 N.Y. 23, 26.
Neither the hope, [**7] the belief, nor the statement by parties that they are incorporated, nor the signing of articles of
incorporation which are not filed, where filing is requisite to create the corporation, nor the user of the pretended franchise of
such a nonexistent corporation, will constitute such a corporation de facto as will exempt those who actively and knowingly
use its name to incur obligations from their individual liability to pay them. Color of legal organization as a corporation under
some charter or law and user of the supposed corporate franchise in good faith are indispensable to such exemption.

[*193] Under the general law of Arkansas in force in the Indian Territory, the filing of articles of incorporation with the clerk
of the Court of Appeals was a sine qua non of any color of a legal corporation. Without that there was not, and there could
not be, an apparent corporation or the color of a corporation. Agreements to form one, statements that there was one,
signed articles of association to make one, acts as one, created no color of incorporation, because there could be no
incorporation or color of it under the law until the articles were filed. Johnson v. Corser, 34 Minn. [**8] 355, 25 N.W. 799;
Finnegan v. Noerenberg, 52 Minn. 239, 243, 244, 53 N.W. 1150, 1151, 18 L.R.A. 778, 38 Am. St. Rep. 552; Taylor on
Private Corporations, p. 145, Roberts Mfg. Co. v. Schlick, 62 Minn. 332, 64 N.W. 826. In Finnegan v. Noerenberg, supra,
Chief Justice Gilfillan well said:

"To give to a body of men assuming to act as a corporation, where there has been no attempt to comply with the provisions
of any law authorizing them to become such, the status of a de facto corporation, might open the door to frauds upon the
public. It would certainly be impolitic to permit a number of men to have the status of a corporation to any extent merely
because there is a law under which they might have become incorporated, and they have agreed among themselves to act,
and they have acted, as a corporation. That was the condition in Johnson v. Corser, 34 Minn. 355, 25 N.W. 799, in which it
was held that what had been done was ineffectual to limit the individual liability of the associates. They had not gone far
enough to become a de facto corporation. They had merely signed articles, but had not attempted to give them publicity by
filing for record, which the statute required."

The [**9] defendants cannot escape individual liability for the $4,700 on the ground that the Coweta Cotton & Milling
Company was a corporation de facto when that portion of the plaintiff's claim was incurred, because it then had no color of
incorporation, and they knew it and yet actively used its name to incur the obligation. Owen v. Shepard, 8 C.C.A. 244, 59
Fed. 746; Wechselberg v. Flour City National Bank, 64 Fed. 90, 94, 12 C.C.A. 56, 60, 61, 26 L.R.A. 470; Abbott v. Omaha
Smelting & Refining Co., 4 Neb. 416, 423, 424; Garnett v. Richardson, 35 Ark. 144; Johnson v. Corser, 34 Minn. 355, 357,
25 N.W. 799; Queen City Furniture & Carpet Co. v. Crawford, 127 Mo. 356, 364, 30 S.W. 163; Bigelow v. Gregory, 73 Ill.
197, 202; Parsons on Partnership, p. 544; Hill v. Beach, 12 N.J. Eq. 31; Kaiser v. Lawrence Savings Bank, 56 Iowa, 104, 8
N.W. 772, 41 Am. St. Rep. 85; Pettis v. Atkins, 60 Ill. 454; Coleman v. Coleman, 78 Ind. 344; Lawler v. Murphy, 58 Conn.
313, 20 Atl. 457, 8 L.R.A. 113; Hurt v. Salisbury, 55 Mo. 310, 314; Beach on Private Corporations, § 16, p. 25; Martin v.
Fewell, 79 Mo. 401, 411; Smith v. Warden, 86 Mo. 382, 399; McVicker v. Cone, 21 Or. 353, 28 Pac. 77.

Another contention [**10] is that the defendants are released from liability because the materials and labor for which the
$4,700 became due were furnished to them while they were promoting the organization of the corporation for the future
corporation, and that the latter has received the benefit of them and ratified their purchase; and in support of this position
they cite Whitney v. Wyman, 101 U.S. 396, 25 L. Ed. 1050; Little Rock & Ft. Smith R.R. Co. v. Perry, 37 Ark. 164; Paxton
Co. v. First National Bank, 21 Neb. 621, 33 N.W. 271, [*194] 59 Am. St. Rep. 852; Stanton v. New York R.R. Co., 59 Conn.
272, 22 Atl. 300, 21 Am. St. Rep. 110; Davis v. Montgomery, 101 Ala. 127, 8 South. 496; Reichwald v. Commercial Hotel
Co., 106 Ill. 439; Wall v. Niagara Co., 20 Utah, 474, 59 Pac. 399; Lancaster Co. v. Murray Co., 19 Tex. Civ. App. 110, 47
S.W. 387; Kaeppler v. Redfield Co., 12 S.D. 483, 81 N.W. 907; Chase v. Redfield, 12 S.D. 529, 81 N.W. 951. In Whitney v.
Wyman, after the articles of incorporation were signed, but before they were filed, three promoters of the incorporation wrote
to the plaintiff that the company was so far organized that by direction of its officers they ordered seven lathes and the [**11]
necessary fixtures for clasping. These lathes were necessary to enable the corporation to commence its contemplated
business, were received and used by it, and the Supreme Court held that the promoters were not individually liable for their
purchase price.

Little Rock & Ft. Smith R.R. Co. v. Perry was an action against the corporation, and the liability of the promoters was not in
issue. The court declared that the rule here invoked grew out of decisions in equity that contracts necessarily made by
promoters on behalf of a future corporation in order to obtain its charter or to complete its organization would be specifically
enforced against it, as in Stanley v. Birkenhead Railway Co., 9 Simons, 264, 16 Eng. Ch. Rep. 264, where the projectors of
a railroad seeking a charter agreed with a landed proprietor, on behalf of the proposed company, in consideration that he
would withdraw his opposition to their bill, to pay him £ 20,000 for the portion of his estate required by the road, and the
court enforced the specific performance of this obligation against the corporation when the charter had been granted, and,
as in Edwards v. Grand Junction Railway Company, 1 Mylne & C. 650, Preston [**12] v. Liverpool, Manchester, etc.,
Railway Co., 7 Eng. L. & Eq. 124, Webb v. Direct London & Portsmouth Railway Company, 9 Hare, 129, Low v. Ct. &
Passumpric Railway, 45 N.H. 375, which are there cited, and the Arkansas court held that in order to recover of such a
corporation the plaintiff must show "either an express promise of the new company, or that the contract was made with
persons then engaged in its formation and taking preliminary steps thereto, and that the contract was made on behalf of the
new company, in the expectation on the part of the plaintiff and with the assurance on the part of the projectors that it would
become a corporate debt, and that the company afterwards entered upon and enjoyed the benefit of the contract, and by no
other title than that derived through it." But there is no evidence that the materials and labor furnished to the defendants
prior to December 22, 1902, were sold by the plaintiff with the assurance on their part, or with the expectation on its part,
that their price would not be paid by them, but would become the debt or obligation of a corporation to be organized in the
future. On the other hand, Davis, who sold these articles for the plaintiff, [**13] and Knight, who bought them for the
defendants, both testified that in the purchase and the sale of all of them they treated themselves as a corporation before,
as completely as after, the filing of their articles. The rule of law here invoked applies to contracts preliminary and incidental
to the organization or to the commencement of the business of a contemplated [*195] corporation, and this debt for $4,700
was not the result of any such contract. It is part of the balance of an account of many tens of thousands of dollars which
arose out of the conduct of a business preliminary, not to its commencement, but to its close. The business of the
defendants was the buying and ginning of cotton. They commenced to construct their buildings in June, to buy cotton in
September, to operate their gin in the first days of October, they filed their articles on December 22d, and ceased to operate
their gin in the following January. They cannot escape liability for debts incurred in this business prior to December 22d on
the ground that their construction of buildings and their dealing in and ginning cotton for two months and a half were
necessary preliminaries to the organization of [**14] their corporation or to the commencement of their business, nor on the
ground that the claim of the plaintiff was incurred on their assurance that it was for and should become the debt of a
corporation to be formed, because these grounds are not sustained by the evidence.

Counsel insist that the defendants are not liable here because one who deals with a corporation de facto is estopped from
denying its existence as a corporation; but the true meaning and legal effect of this rule is that such a dealer is estopped
from denying its existence on the ground that it was not legally incorporated. One who deals with parties who masquerade
under a name which represents no corporation de facto is no more estopped from denying that it is a corporation than he
would be from denying that they constituted or acted for the Union Pacific Railroad Company, or any other well-known
corporation, when they did not. The fact that the plaintiff dealt with and treated the Coweta Cotton & Milling Company as a
corporation did not estop it from denying that it was such before the defendants filed their articles of incorporation, because
it was not a corporation de facto before that time and because the [**15] indispensable elements of an estoppel in pais,
ignorance of the truth and absence of equal means of knowledge of it by the party who claims the estoppel, and action by
the latter induced by the misrepresentation of the party against whom the estoppel is invoked, do not exist in the case at
bar. Bigelow on Estoppel (4th Ed.) p. 679. The plaintiffs did not, and the defendants did, represent that the milling company
was a corporation when it was not.The defendants had better means of knowledge of the fact than the plaintiff, and they
knew it was not a corporation, and they were not induced to act on any representation of the plaintiff that it was such, or by
its treatment of it as such.
Nor was the plaintiff estopped by the fact that its general manager stated under oath in its claim for a lien in May, 1903, that
the milling company was a corporation, first, because the defendants were not induced to take any action by this statement
from which they can suffer any injury by the proof of the truth, and, second, because one is not estopped from pursuing his
true legal remedy by a mistaken attempt to pursue a supposed remedy that does not exist. Standard Oil Company v.
Hawkins, 20 C.C.A. [**16] 468, 472, 74 Fed. 395, 398, 399, 33 [*196] L.R.A. 739; Barnsdall v. Waltemeyer, 73 C.C.A.
515, 520, 142 Fed. 415, 420; Bunch v. Grave, 111 Ind. 351, 12 N.E. 514, 517.

It is said that the plaintiff is estopped from denying the existence of the defendant's supposed corporation because it was
one of its promoters and stockholders, but the evidence fails to convince us that it was ever either. F. M. Davis was the
general manager of the plaintiff. He testified that in June, 1902, he agreed with the other defendants to take a $2,000 share
for the plaintiff in a corporation to be organized with a capital of $10,000 for the purpose of ginning and dealing in cotton,
that Mann agreed to take a $4,000 share, R. S. Davis and James G. Knight a share of $2,000 each, that in September he
signed the articles of incorporation and subscribed for this stock, that the other defendants also subscribed, that these
subscribers paid the first assessment of $3,750 on $10,000 of the stock in the fall of 1902, that the second assessment of
$2,000 was made in January, 1903, that he never reported this stock to the plaintiff until January, 1903, but that in the
summer and fall of 1902 he talked with Edwards [**17] and Wallace, two of the directors, who had 80 shares of stock each
in the plaintiff, about this stock which he was to take and which he had taken, that the plaintiff and they acquiesced in his
action and told him to do the best he could with it, but that they did not direct or instruct him to take the stock or agree that
he should take it, and that he did not talk with the president, who was the owner of the majority of the stock of the plaintiff,
although another witness testified that some time in the fall of 1902 he told Naylor that Davis had taken stock in the milling
company for the plaintiff. Davis, however, subscribed for the stock in his own name, and the plaintiff did not. He testified that
he paid the first assessment in the fall of 1902, but he never charged the plaintiff and credited himself with that payment;
but, on the contrary, on February 23, 1903, after the milling company had ceased to operate its gin, he caused an entry to
be made on the books of the investment company charging it and crediting the milling company with $1,150, the amount of
the two assessments on his stock, an entry which the plaintiff subsequently repudiated. There are two reasons why, under
[**18] the evidence in this record, the plaintiff never became a holder, either in law or in equity, of any share in the
defendant's enterprise or company, either as a stockholder or otherwise. In the first place, the construction and operation of
a cotton gin was beyond the powers of the plaintiff corporation, the nature of whose business was declared and limited by
its articles to "buying, selling, leasing and dealing in lands, securities, bonds, notes, stocks and other negotiable paper, and
also buying and selling general merchandise." In the second place, if by any conceivable interpretation the construction and
operation of a cotton gin and the formation of the corporation, and the taking of stock therein to accomplish that purpose,
could be deemed to be within the powers of this corporation, they are so far beyond the scope of its ordinary business that a
general manager could be authorized to commit his corporation to them only by the express authority of its board of
directors, or of its principal officers, after a full disclosure to them of all the facts relating to the proposed enterprise, and the
desultory [*197] talks which Davis had with the two directors fall far short [**19] of any evidence of such authority.

Much is made in argument of the testimony of Davis and Knight that they acted in good faith, that the defendants never
received any benefit from the materials and labor for the purchase price of which the plaintiff sues, but good faith and the
use of a name which they know represents no corporation as the name of a corporation under which they do business
creates a partnership, and neither a corporation de jure nor de facto. And the defendants had all the benefit there was from
the materials and labor furnished by the plaintiff, for the milling company never issued any stock, and these defendants
owned their respective shares in its property, and whatever it had they had, and, as far as they have not disposed of it, they
still have. The fact is that during this entire transaction while Davis was the general manager of the plaintiff he was the
partner of the defendants, and, in all transactions between the plaintiff and the defendants, was pecuniarily interested
adversely to his principal.

The sum of the whole matter is that the defendants agreed in April or June, 1902, to take certain shares in a $10,000
enterprise for the purpose of building [**20] a cotton gin, and buying, ginning, and selling cotton, and to organize a
corporation to carry on this business they bought between June and December 22, 1902, materials and labor with which
they built the cotton gin, and between September 15th and December 22d operated their cotton gin and carried on the
business of buying, ginning, and selling cotton with the plaintiff to the amount of several tens of thousands of dollars, and
there remains a balance of about $4,700 due the plaintiff on this account. They never issued any stock, but in September,
November, and December they signed articles of incorporation which they filed with the clerk of the Court of Appeals on
December 22, 1902. During this time they treated themselves and the plaintiff dealt with them as a corporation. They
represented themselves to be a corporation when they knew they were not; under the name of a corporation which did not
exist they purchased these goods and services.
And our conclusion is that the defendants never became a corporation de facto prior to December 22, 1902, that they never
became a corporation de jure, that the indebtedness here in question was not incurred under any promise or assurance
[**21] of the defendants as promoters that it should become the obligation of a corporation to be formed, that a large part of
it was incurred in the conduct of a general commercial business, and not to prepare for the commencement of such a
business or for the organization of a corporation, and that the trial court below should have instructed the jury that the
defendants were individually liable for that portion of the plaintiff's claim which was incurred prior to December 22, 1902. Its
failure to do so was a fatal error which necessitates a reversal of the judgments below.
In view of the conclusion which has now been reached, it is unnecessary to discuss at length or to determine other
questions which are presented in this record. It is sufficient to say regarding the portion of the plaintiff's claim incurred
subsequent to December 22, 1902, that while there is a conflict of authority upon the question whether [*198] or not
incorporators or stockholders remain personally liable after the filing of articles in one office only where the statute requires
them to be filed in two offices as a condition of incorporation or of the commencement of business ( Mokelumne Hill Canal &
Mining [**22] Co. v. Woodbury, 14 Cal. 265, 267), the statute under which this case arose was brought into the Indian
Territory from the state of Arkansas, and the Supreme Court of that state had held, before it was adopted in the Indian
Territory, that such corporators or stockholders remain individually liable under this statute unless and until their articles of
incorporation are filed in both offices. Garnett v. Richardson, 35 Ark. 144. This conclusion is sustained by eminent authority
(Wechselberg v. Flour City National Bank, 12 C.C.A. 56, 60, 61, 64 Fed. 90, 94, 26 L.R.A. 470, and authorities there cited),
and it is an established rule of statutory construction that the adoption of a statute previously in force in some other
jurisdiction is presumed to be the adoption of the interpretation thereof which had been theretofore placed upon it by the
judicial tribunal whose duty it was to construe it. Black, Interpretation of Laws, p. 159, § 70; McDonald v. Hovey, 110 U.S.
619, 628, 4 Sup. Ct. 142, 28 L. Ed. 269; Sanger v. Flow, 1 C.C.A. 56, 58, 48 Fed. 152, 154; Blaylock v. Incorporated Town
of Muskogee, 54 C.C.A. 639, 117 Fed. 125.

The judgments of the courts below must be reversed, and [**23] the case must be remanded to the proper court for a new
trial; and it is so ordered.

[G.R. No. L-2598. June 29, 1950.]

C. ARNOLD HALL and BRADLEY P. HALL, petitioners, vs. EDMUNDO S. PICCIO, Judge of the
Court of First Instance of Leyte, FRED BROWN, EMMA BROWN, HIPOLITA CAPUCIONG, in his
capacity as receiver of the Far Eastern Lumber and Commercial Co., Inc., respondent.

Claro M. Recto for petitioners.


Ramon Diokno and Jose W. Diokno for respondents.

SYLLABUS

1. CORPORATION "DE FACTO"; DISSOLUTION BY SUIT OF STOCKHOLDERS; JURISDICTION OF


COURT. — An entity whose certificate of incorporation had not been obtained may be terminated in a private suit for its
dissolution between stockholders, without the intervention of the state. The question as to the right of minority
stockholders to sue for dissolution does not affect the court's jurisdiction, and is a matter for decision by the judge,
subject to review on appeal by the aggrieved party at the proper time.
2. ID.; RIGHTS OF. — Persons acting as corporation may not claim rights of "de facto" corporation if they have
not obtained certificate of incorporation.

DECISION

BENGZON, J p:

This is a petition to set aside all the proceedings had in civil case No. 381 of the Court of First Instance of Leyte
and to enjoin the respondent judge from further acting upon the same.

Facts: (1) On May 28, 1947, the petitioners C. Arnold Hall and Bradley P. Hall, and the respondents Fred
Brown, Emma Brown, Hipolita D. Chapman and Ceferino S. Abella, signed and acknowledged in Leyte, the articles of
incorporation of the Far Eastern Lumber and Commercial Co., Inc., organized to engage in a general lumber business
to carry on as general contractors, operators and managers, etc. Attached to the articles was an affidavit of the
treasurer stating that 23,428 shares of stock had been subscribed and fully paid with certain properties transferred to
the corporation described in a list appended thereto. .
(2) Immediately after the execution of said articles of incorporation, the corporation proceeded to do business
with the adoption of by-laws and the election of its officers. (3) On December 2, 1947, the said articles of incorporation
were filed in the office of the Securities and Exchange Commissioner, for the issuance of the corresponding certificate
of incorporation. (4) On March 22, 1948, pending action on the articles of incorporation by the aforesaid governmental
office, the respondents Fred Brown, Emma Brown, Hipolita D. Chapman and Ceferino S. Abella filed before the Court of
First Instance of Leyte the civil case numbered 381, entitled "Fred Brown et al. vs. Arnold C. Hall et al.", alleging among
other things that the Far Eastern Lumber and Commercial Co. was an unregistered partnership; that they wished to
have it dissolved because of bitter dissension among the members, mismanagement and fraud by the managers and
heavy financial losses. (5) The defendants in the suit, namely, C. Arnold Hall and Bradley P. Hall, filed a motion to
dismiss, contesting the court's jurisdiction and the sufficiency of the cause of action. (6) After hearing the parties, the
Hon. Edmundo S. Piccio ordered the dissolution of the company; and at the request of plaintiffs, appointed the
respondent Pedro A. Capuciong as receiver of the properties thereof, upon the filing of a P20,000 bond. (7) The
defendants therein (petitioners herein) offered to file a counter-bond for the discharge of the receiver, but the
respondent judge refused to accept the offer and to discharge the receiver. Whereupon the present special civil action
was instituted in this court. It is based upon two main propositions, to wit: .
(a) The court had no jurisdiction in civil case No. 381 to decree the dissolution of the company, because it being
a de facto corporation, dissolution thereof may only be ordered in a quo warranto proceeding instituted in accordance
with section 19 of the Corporation Law. .
(b) Inasmuch as respondents Fred Brown and Emma Brown had signed the articles of incorporation, they are
estopped from claiming that it is not a corporation but only a partnership. .
Discussion: The second proposition may at once be dismissed. All the parties are informed that the Securities
and Exchange Commission has not, so far, issued the corresponding certificate of incorporation. All of them know, or
ought to know, that the personality of a corporation begins to exist only from the moment such certificate is issued - not
before (sec. 11, Corporation Law). The complaining associates have not represented to the others that they were
incorporated any more than the latter had made similar representations to them. And as nobody was led to believe
anything to his prejudice and damage, the principle of estoppel does not apply. Obviously this is not an instance
requiring the enforcement of contracts with the corporation through the rule of estoppel. .
The first proposition above stated is premised on the theory that, inasmuch as the Far Eastern Lumber and
Commercial Co., is a de facto corporation, section 19 of the Corporation Law applies, and therefore the court had no
jurisdiction to take cognizance of said civil case number 381. Section 19 reads in part as follows: .
"*** The due incorporation of any corporations claiming in good faith to be a corporation under this Act and its
right to exercise corporate powers shall not be inquired into collaterally in any private suit to which the corporation may
be a party, but such inquiry may be had at the suit of the Insular Government on information of the Attorney-General." .
There are at least two reasons why this section does not govern the situation. Not having obtained the
certificate of incorporation, the Far Eastern Lumber and Commercial Co. - even its stockholders - may not probably
claim "in good faith" to be a corporation. .
"Under our statute it is to be noted (Corporation Law, sec. 11) that it is the issuance of a certificate of
incorporation by the Director of the Bureau of Commerce and Industry which calls a corporation into being. The
immunity of collateral attack is granted to corporations 'claiming in good faith to be a corporation under this act.' Such a
claim is compatible with the existence of errors and irregularities; but not with a total or substantial disregard of the law.
Unless there has been an evident attempt to comply with the law the claim to be a corporation 'under this act' could not
be made 'in good faith.' " (Fisher on the Philippine Law of Stock Corporations, p. 75. See also Humphreys vs. Drew, 59
Fla., 295; 52 So., 362.) .
Second, this is not a suit in which the corporation is a party. This is a litigation between stockholders of the
alleged corporation, for the purpose of obtaining its dissolution. Even the existence of a de jure corporation may be
terminated in a private suit for its dissolution between stockholders, without the intervention of the state. .
There might be room for argument on the right of minority stockholders to sue for dissolution;1 but that question
does not affect the court's jurisdiction, and is a matter for decision by the judge, subject to review on appeal. Which
brings us to one principal reason why this petition may not prosper, namely: the petitioners have their remedy by
appealing the order of dissolution at the proper time. .
There is a secondary issue in connection with the appointment of a receiver. But it must be admitted that
receivership is proper in proceedings for dissolution of a company or corporation, and it was no error to reject the
counter-bond, the court having decreed the dissolution. As to the amount of the bond to be demanded of the receiver,
much depends upon the discretion of the trial court, which in this instance we do not believe has been clearly abused. .
Judgment: The petition will, therefore, be dismissed, with costs. The preliminary injunction heretofore issued will
be dissolved. .
||| (Hall v. Piccio, G.R. No. L-2598, [June 29, 1950], 86 PHIL 603-607)

[G.R. No. 22106. September 11, 1924.]

ASIA BANKING CORPORATION, plaintiff-appellee, vs. STANDARD PRODUCTS CO., INC., defendant-
appellant.

Charles C. De Selms for appellant.


Gibbs & McDonough and Roman Ozaeta for appellee.

SYLLABUS

1. CORPORATION; CORPORATE EXISTENCE, ESTOPPEL FROM DENYING. — In the absence of fraud, a


person who has contracted or dealt with an association in such a way as to recognize and in effect admit its legal
existence as a corporate body is thereby estopped to deny its corporate existence in an action leading out of or
involving such contract or dealing, unless the existence is attacked for causes which have arisen since making the
contract or other dealing relied on as an estoppel.
2. ID.; ID.; EVIDENCE. — The defendant having recognized the or the corporate existence of the plaintiff by
making a promissory note in its favor and making payments on the same, and the defendant having held itself out as a
corporate and being therefore estopped from denying its own corporate existence it is necessary for the plaintiff to
present other evidence of the corporate existence of either of the parties.

DECISION

OSTRAND, J p:

This action is brought to recover the sum of P24,736.47, the balance due on the following promissory note:
"P37,757.22

"MANILA, P. I., Nov. 28, 1921.


"On demand, after date we promise to pay to the Asia Banking Corporation, or order, the sum of
thirty-seven and 22/100 pesos at their office in Manila, for value received, together with interest at the
rate of ten per cent per annum.
"No._______Due _________
"THE STANDARD PRODUCTS CO., INC.
"By (Sgd.) GEORGE H. SEAVER
"President"
The court below rendered judgment in favor of the plaintiff for the sum demanded in the complaint, with interest
on the sum of P24,147.34 from November 1, 1923, at the rate of 10 per cent per annum, and the costs. From this
judgment the defendant appeals to this court.
At the trial of the case the plaintiff failed to prove affirmatively the corporate existence of the parties and the
appellant insists that under these circumstances the court erred in finding that the parties were corporations with
juridical personality and assigns same as reversible error.
There is no merit whatever i the appellant's contention. The general rule is that in the absence of fraud a person
who has contracted or otherwise dealt with a association in such a way as to recognized and in effect admit its legal
existence as a corporate body is thereby estopped to deny its corporate existence in any action leading out of or
involving such contract or dealing, unless its existence is attacked for causes which have arisen since making the
contract or other dealing relied on as an estoppel and this applies to foreign as well as to domestic corporations. (14 C.
J., 227; Chinese Chamber of Commerce vs. Pua Te Ching, 14 Phil., 222)
The defendant having recognized the corporate existence of the plaintiff by making a promissory note in its
favor and making partial payments on the same is therefore estopped to deny said plaintiff's corporate existence. It is,
of course, also estopped from denying its own corporate existence. Under these circumstances it was unnecessary for
the plaintiff to present other evidence of the corporate existence of either of the parties. It may be noted that there is no
evidence showing circumstances taking the case out of the rules stated.
The judgment appealed from is affirmed, with the costs against the appellant. So ordered.
||| (Asia Banking Corp. v. Standard Products Co., Inc., G.R. No. 22106, [September 11, 1924], 46 PHIL 144-146)

CRANSON v. INTERNATIONAL BUSINESS MACHINES CORPORATION

Court of Appeals of Maryland.

Decided April 30, 1964.

Attorney(s) appearing for the Case

William J. Brannan, Jr., with whom were Kardy, Brannan & Neumann on the brief, for the appellant.

Henry J. Noyes for the appellee.


The cause was argued before HENDERSON, HAMMOND, HORNEY, MARBURY and SYBERT, JJ.

HORNEY, J., delivered the opinion of the Court.


On the theory that the Real Estate Service Bureau was neither a de jure nor a de facto corporation and that Albion C.
Cranson, Jr., was a partner in the business conducted by the Bureau and as such was personally liable for its debts, the
International Business Machines Corporation brought this action against Cranson for the balance due on electric typewriters
purchased by the Bureau. At the same time it moved for summary judgment and supported the motion by affidavit. In due
course, Cranson filed a general issue plea and an affidavit in opposition to summary judgment in which he asserted in effect
that the Bureau was a de facto corporation and that he was not personally liable for its debts.
The agreed statement of facts shows that in April 1961, Cranson was asked to invest in a new business corporation which
was about to be created. Towards this purpose he met with other interested individuals and an attorney and agreed to
purchase stock and become an officer and director. Thereafter, upon being advised by the attorney that the corporation had
been formed under the laws of Maryland, he paid for and received a stock certificate evidencing ownership of shares in the
corporation, and was shown the corporate seal and minute book. The business of the new venture was conducted as if it
were a corporation, through corporate bank accounts, with auditors maintaining corporate books and records, and under a
lease
[234 Md. 480]
entered into by the corporation for the office from which it operated its business. Cranson was elected president and all
transactions conducted by him for the corporation, including the dealings with I.B.M., were made as an officer of the
corporation. At no time did he assume any personal obligation or pledge his individual credit to I.B.M. Due to an oversight
on the part of the attorney, of which Cranson was not aware, the certificate of incorporation, which had been signed and
acknowledged prior to May 1, 1961, was not filed until November 24, 1961. Between May 17 and November 8, the Bureau
purchased eight typewriters from I.B.M., on account of which partial payments were made, leaving a balance due of
$4,333.40, for which this suit was brought.
Although a question is raised as to the propriety of making use of a motion for summary judgment as the means of
determining the issues presented by the pleadings, we think the motion was appropriate. Since there was no genuine
dispute as to the material facts, the only question was whether I.B.M. was entitled to judgment as a matter of law. The trial
court found that it was, but we disagree.
The fundamental question presented by the appeal is whether an officer1 of a defectively incorporated association may be
subjected to personal liability under the circumstances of this case. We think not.
Traditionally, two doctrines have been used by the courts to clothe an officer of a defectively incorporated association with
the corporate attribute of limited liability. The first, often referred to as the doctrine of de facto corporations, has been
applied in those cases where there are elements showing: (1) the existence of law authorizing incorporation: (2) an effort in
good faith to incorporate under the existing law; and (3) actual user or exercise of corporate powers. Ballantine, Private
Corporations, § 23; 8 Fletcher, Cyclopedia of the Law of Private
[234 Md. 481]
Corporations, § 3777; 13 Am. Jur., Corporations, §§ 49-56; 18 C.J.S., Corporations, § 99. The second, the doctrine of
estoppel to deny the corporate existence, is generally employed where the person seeking to hold the officer personally
liable has contracted or otherwise dealt with the association in such a manner as to recognize and in effect admit its
existence as a corporate body. Ballantine, op.cit., § 29; Machen, Modern Law of Corporations, §§ 278-282; 18 C.J.S.,
op.cit., § 109.
It is not at all clear what Maryland has done with respect to the two doctrines. There have been no recent cases in this State
on the subject and some of the seemingly irreconcilable earlier cases offer little to clarify the problem. 2
In one line of cases, the Court, in determining the rights and liabilities of a defectively organized corporation, or a member or
stockholder thereof, seems to have drawn a distinction between those acts or requirements which are a condition precedent
to corporate existence and those acts prescribed by law to be done after incorporation. In so doing, it has been generally
held that where there had been a failure to comply with a requirement which the law declared to be a condition precedent to
the existence of the corporation, the corporation was not a legal entity and was therefore precluded from suing or being
sued as such. Boyce v. M.E. Church, 46 Md. 359 (1877); Regester v. Medcalf, 71 Md. 528, 18 Atl. 966 (1889); Bonaparte v.
Lake Roland R.R. Co., 75 Md. 340, 23 Atl. 784 (1892); Jones v. Linden Building Asso., 79 Md. 73, 29 Atl. 76 (1894);
Maryland Tube Works v. West End Imp. Co., 87 Md. 207, 39 Atl. 620 (1898); Cleaveland v. Mullin, 96 Md. 598,
[234 Md. 482]
54 Atl. 665 (1903); National Shutter Bar Co. v. Zimmerman, 110 Md. 313, 73 Atl. 19 (1909). These cases appear to stand
for the proposition that substantial compliance with those formalities of the corporation law, which are made a condition
precedent to corporate existence, was not only necessary for the creation of a corporation de jure, but was also a
prerequisite to the existence of a de facto corporation or a corporation by estoppel.
In the Boyce case, an action in assumpsit against a defectively incorporated religious society, the Court (at p. 373 and p.
374), in holding that the society was not estopped to deny its corporate existence, said:
We think it would be extending the doctrine of estoppel to an extent, not justified by the principles of public policy, to allow it
to operate through the conduct of the parties concerned, to create substantially a de facto corporation, with just such powers
as the parties may by their acts give to it. * * * The statute law of the State, expressly requiring certain prescribed acts to be
done to constitute a corporation, to permit parties indirectly, or upon the principle of estoppel, virtually to create a
corporation for any purpose, or to have acts so construed, would be in manifest opposition to the statute law, and clearly
against its policy, and justified upon no sound principle in the administration of justice.
In the Maryland Tube case, an action by a corporation for specific performance of a contract to convey land which it had
entered into prior to its becoming a legal entity, the Court, having cited (at p. 217) the statements in Jones v. Aspen
Hardware Co., 40 Pac. 457 (Colo. 1895),3 with approval for the
[234 Md. 483]
proposition that "`the doctrine of estoppel cannot be successfully invoked, unless the corporation has at least a de facto
existence,'" that "`a de facto corporation can never be recognized in violation of a positive law'" and that "`there is a broad
distinction between those acts made necessary by the statute as a prerequisite to the exercise of corporate powers, and
those acts required of individuals seeking incorporation but not made prerequisite to the exercise of such powers,'" went on
to say (at p. 218) that "these principles were clearly recognized and applied" in the Boyce case.
In the National Shutter Bar case, an action by a corporation for an alleged libel which had occurred before the performance
of a condition precedent necessary for legal incorporation, it was held — citing the Maryland Tube case for the proposition
that statutory conditions precedent must have been complied with to give existence to corporations formed under general
laws — that the corporation had no legal existence at the time of the alleged libel. In referring to the Boyce case, it was said
(at p. 320) that "it has been held by our predecessors that a corporation cannot be actually or virtually created by estoppel in
Maryland." And, on the basis of the statements in Jones v. Aspen Hardware Co., supra (also relied on in the Maryland Tube
case), it was concluded that the corporation could not maintain the action.
On the other hand, where the corporation has obtained legal existence but has failed to comply with a condition subsequent
to corporate existence, this Court has held that such nonperformance afforded the State the right to institute proceedings for
the forfeiture of the charter, but that such neglect or omission could never be set up by the corporation itself, or by its
members and stockholders, as a defense to an action to enforce their liabilities. C. & O. Canal Co. v. B. & O. Railroad Co., 4
G. & J. 1 (1832); Hammond v. Straus, 53 Md. 1 (1880); Murphy v. Wheatley, 102 Md. 501, 63 Atl. 62 (1906).
[234 Md. 484]
In the Hammond case, an action by a creditor against a stockholder of a state bank on his statutory liability, the Court, after
stating that a corporation or a stockholder could not defeat an action by showing noncompliance with the requirements of
the corporation law unless the acts required are conditions precedent to corporate existence, said (at p. 15): By holding
otherwise, parties might avail themselves of the powers and privileges of a corporation, without in any manner subjecting
themselves to its duties and obligations, and might set up their own neglect of duty, of wilful omission to comply with the
requirements of the statute, as means of discharge from all their just obligations under the law. This is forbidden by every
principle of law and justice, and hence such a defense could never be tolerated.
It seems clear therefore that when a defect in the incorporation process resulted from a failure to comply with a condition
subsequent, the doctrine of estoppel may be applied for the benefit of a creditor to estop the corporation, or the members or
stockholders thereof, from denying its corporate existence. See Brune (Herbert M., Jr.), Maryland Corporation Law and
Practice (rev. ed.), § 339.
In another line of Maryland cases which determined the rights and liabilities of a defectively organized corporation, or a
member or stockholder thereof, the Court, apparently disregarding the distinction made between those requirements which
are conditions precedent and those which are conditions subsequent to corporate existence, has generally precluded, on
the grounds of estoppel or collateral attack, inquiry into the question of corporate existence. Maltby v. Northwestern Va.
R.R. Co., 16 Md. 422 (1860); Franz v. Teutonia Building Asso., 24 Md. 259 (1866); Grape Sugar & Vinegar Mfg. Co. v.
Small, 40 Md. 395 (1874); Laflin & Rand Powder Co. v. Sinsheimer, 46 Md. 315 (1877); Keene v. Van Reuth, 48 Md. 184
(1878); Bartlett v. Wilbur, 53 Md. 485 (1880); Pott & Co. v. Schmucker, 84 Md. 535, 36 Atl. 592 (1897). In the Grape Sugar
case, an action against a defectively organized corporation to
[234 Md. 485]
recover the balance due for work done and materials furnished, the Court said (at p. 400): The second prayer proceeds
upon the assumption that the [corporation] is not liable, provided the work was done prior to the recording of the certificate
of incorporation. It is true, that under the general incorporation law of this State, the recording of the certificate was
necessary to constitute the [corporation] a body politic. If, however, the contract was made with the [creditor] through * * *
[the] President of the [corporation], after the certificate had been signed by the members of the proposed corporation, but
before it was recorded, and the company, after its incorporation was complete, accepted the work done under the contract,
it will be estopped, both in law and equity, from denying its liability, on account of the same.
Cf. Hammond v. Straus, supra. And see to the contrary Boyce v. M.E. Church, supra, which might be distinguishable in that
it involved an effort to impose liability on a religious society and not a business corporation.
In the Laflin & Rand case, decided in the same year (1877) as the Boyce case, the Court, in an action against certain
members of a corporation to make them individually liable for goods sold and delivered to the corporation, said (at p. 321):
[The company] has been clothed with all the forms of a corporation by the laws of a neighboring State, and was in the
exercise and use of the franchises conferred upon it. It was a corporation de facto at the time the goods were sold and
delivered to it * * * and its existence as a corporation cannot be collaterally drawn into question. To permit a recovery
against the defendants, and thereby to say that they are to be regarded in law as a voluntary unincorporated association,
would be a departure from all the cases. The debt was not created with them individually, but with a company acting under a
formal incorporation, and in the exercise of its corporate powers. This [creditor] dealt with it and gave it credit as a
corporation. If its assets are not ample to pay, it is the misfortune of the creditor.4
See also the Franz case at p. 270 (of 24 Md.) and the Bartlett case at p. 498 (of 53 Md.) for similar statements of the law.
From these cases it appears that where the parties have assumed corporate existence and dealt with each other on that
basis, the Court will apply the estoppel doctrine on the theory that the parties by recognizing the organization as a
corporation were thereafter prevented from raising a question as to its corporate existence.
When summarized, the law in Maryland pertaining to the de facto and estoppel doctrines reveals that the cases seem to fall
into one or the other of two categories. In one line of cases, the Court, choosing to disregard the nature of the dealings
between the parties, refused to recognize both doctrines where there had been a failure to comply with a condition
precedent to corporate existence, but, whenever such noncompliance concerned a condition subsequent to incorporation,
the Court often applied the estoppel doctrine. In the other line of cases, the Court, choosing to make no distinction between
defects which
[234 Md. 487]
were conditions precedent and those which were conditions subsequent, emphasized the course of conduct between the
parties and applied the estoppel doctrine when there had been substantial dealings between them on a corporate basis.
Whether or not the decisions in the Boyce and Maryland Tube cases had the effect of repudiating the de facto doctrine in
this state, as some of the text writers seem to think, is a question we do not reach in this case and therefore need not
consider at this time. On the other hand, since it is clear that the Maryland Tube and National Shutter Bar cases are
inconsistent with other Maryland cases insofar as they held (in relying on the statements in Jones v. Aspen Hardware Co.,
supra) that the doctrine of estoppel cannot be invoked unless a corporation has at least a de facto existence, both cases —
Maryland Tube and National Shutter Bar — should be, and are hereby, overruled to the extent of the inconsistency. There
is, as we see it, a wide difference between creating a corporation by means of the de facto doctrine and estopping a party,
due to his conduct in a particular case, from setting up the claim of no incorporation. Although some cases tend to
assimilate the doctrines of incorporation de facto and by estoppel, each is a distinct theory and they are not dependent on
one another in their application. See 8 Fletcher, op.cit., § 3763; France on Corporations (2nd ed.), § 29; 18 C.J.S., op.cit., §
111h. Where there is a concurrence of the three elements necessary for the application of the de facto corporation doctrine,
there exists an entity which is a corporation de jure against all persons but the state. On the other hand, the estoppel theory
is applied only to the facts of each particular case and may be invoked even where there is no corporation de facto.
Accordingly, even though one or more of the requisites of a de facto corporation are absent, we think that this factor does
not preclude the application of the estoppel doctrine5 in a proper case, such as the one at bar.
[234 Md. 488]
I.B.M. contends that the failure of the Bureau to file its certificate of incorporation debarred all corporate existence. But, in
spite of the fact that the omission might have prevented the Bureau from being either a corporation de jure or de facto,6
Jones v. Linden Building Asso., supra, we think that I.B.M. having dealt with the Bureau as if it were a corporation and relied
on its credit rather than that of Cranson, is estopped to assert that the Bureau was not incorporated at the time the
typewriters were purchased. Laflin & Rand Powder Co. v. Sinsheimer, supra. See also Tulane Improvement Co. v. S.A.
Chapman & Co., 56 So. 509 (La. 1911). In 1 Clark and Marshall, Private Corporations, § 89, it is stated: The doctrine in
relation to estoppel is based upon the ground that it would generally be inequitable to permit the corporate existence of an
association to be denied by persons who have represented it to be a corporation, or held it out as a corporation, or by any
persons who have recognized it as a corporation by dealing with it as such; and by the overwhelming weight of authority,
therefore, a person may be estopped to deny the legal incorporation of an association which is not even a corporation de
facto.
In cases similar to the one at bar, involving a failure to file articles of incorporation, the courts of other jurisdictions have held
that where one has recognized the corporate existence of an association, he is estopped to assert the contrary with respect
to a claim arising out of such dealings. See, for example, Tarbell v. Page, 24 Ill. 46 (1860); Magnolia Shingle Co. v. J.
Zimmern's Co., 58 So. 90 (Ala. 1912); Lockwood v. Wynkoop, 144 N.W. 846 (Mich. 1914); John Lucas Co. v. Bernhardt's
Estate, 100 So. 399 (La. 1924).
Since I.B.M. is estopped to deny the corporate existence of the Bureau, we hold that Cranson was not liable for the balance
due on account of the typewriters.

Judgment reversed; the appellee to pay the costs.

[G.R. No. L-11442. May 23, 1958.]


MANUELA T. VDA. DE SALVATIERRA, petitioner, vs. HON. LORENZO C. GARLITOS, in his capacity
as Judge of the Court of First Instance of Leyte, Branch II, and SEGUNDINO REFUERZO,
respondents.

Jimenez, Tantuico, Jr. & Tolete for petitioner.


Francisco Astilla for respondent Segundino Refuerzo.

SYLLABUS

1. PLEADING AND PRACTICE; PETITION FOR RELIEF; WHEN TO FILE PETITION. — Rule 38, Section 3, of
the Rules of Court treats of 2 periods within which a petition for relief may be filed. The petition must be filed within 60
days after the petitioner learns of the judgment and not more than 6 months after the judgment or order was rendered,
both of which must be satisfied.
2. CORPORATION LAW; LIABILITY OF PERSON DEALING WITH ASSOCIATION AS A CORPORATE
BODY; WHEN ESTOPPEL MAY NOT BE INVOKED. — While as a general rule, a person who deals with an
association in such a way to recognize its existence as a corporate body is estopped from denying the same in an
action arising out of such transaction, yet this doctrine may not be held to be applicable where fraud takes a part in the
said transaction. In the instant case, on plaintiff's charge that she was unaware of the fact that the defendant
corporation had no juridical personality, its president gave no confirmation or denial of the same and the circumstance
surrounding the execution of the contract lead to the inescapable conclusion that plaintiff was really made to believe
that such corporation was duly organized in accordance with law.
3. ID.; LIABILITY OF MEMBERS WHO ACT AS AGENTS OF AN UNINCORPORATED ASSOCIATION. — A
corporation when registered has a juridical personality separate and distinct from its component members or
stockholders and officers, such that a corporation cannot be held liable for the personal in indebtedness of a
stockholder even if he should be its president (Walter A. Smith Co. vs. Ford, SC-G. R. No. 42420) and conversely, a
stockholder cannot be held personally liable for any financial obligation by the corporation in excess of his unpaid
subscription. But this rule is understood to refer merely to registered corporations and cannot be made applicable to the
liability of members of an unincorporated association. The reason behind this doctrine is obvious - an unincorporated
association has no personality and would be incompetent to act and appropriate for itself the power and attributes of a
corporation as provided by law, it cannot create agents or confer authority on another to act in its behalf; thus, those
who act or purport to act as its representatives or agents do so without authority and at their own risk. And as it is an
elementary principle of law that a person who acts as an agent without authority or without a principal is himself
regarded as the principal, possessed of all the right and subject to all the liabilities of a principal, a person acting or
purporting to act on behalf of a corporation which has no valid existence assumes such privileges and obligations and
becomes personally liable for contracts entered into or for other acts performed as such agent (Fay vs. Noble, 7
Cushing [Mass.] 188. Cited in II Tolentino's Commercial Laws of the Philippines, Fifth Ed., p. 689-690).

DECISION

FELIX, J p:

This is a petition for certiorari filed by Manuela T. Vda. de Salvatierra seeking to nullify the order of the Court of
First Instance of Leyte in Civil Case No. 1912, dated March 21, 1956, relieving Segundino Refuerzo of liability for the
contract entered into between the former and the Philippine Fibers Producers Co., Inc., of which Refuerzo is the
president. The facts of the case are as follows:
Manuela T. Vda. de Salvatierra appeared to be the owner of a parcel of land located at Maghobas, Población,
Burauen, Leyte. On March 7, 1954, said landholder entered into a contract of lease with the Philippine Fibers Producers
Co., Inc., allegedly a corporation "duly organized and existing under the laws of the Philippines, domiciled at Burauen,
Leyte, Philippines, and with business address therein, represented in this instance by Mr. Segundino Q. Refuerzo, the
President". It was provided in said contract, among other things, that the lifetime of the lease would be for a period of 10
years; that the land would be planted to kenaf, ramie or other crops suitable to the soil; that the lessor would be entitled
to 30 per cent of the net income accruing from the harvest of any crop without being responsible for the cost of
production thereof; and that after every harvest, the lessee was bound to declare at the earliest possible time the
income derived therefrom and to deliver the corresponding share due the lessor.
Apparently, the aforementioned obligations imposed on the alleged corporation were not complied with
because on April 5, 1955, Manuela T. Vda. de Salvatierra filed with the Court of First Instance of Leyte a complaint
against the Philippine Fibers Producers Co., Inc., and Segundino Q. Refuerzo, for accounting, rescission and damages
(Civil Case No. 1912). She averred that sometime in April, 1954, defendants planted kenaf on 3 hectares of the leased
property which crop was, at the time of the commencement of the action, already harvested, processed and sold by
defendants; that notwithstanding that fact, defendants refused to render an accounting of the income derived therefrom
and to deliver the lessor's share; that the estimated gross income was P4,500, and the deductible expenses a mounted
to P1,000; that as defendants' refusal to undertake such task was in violation of the terms of the covenant entered into
between the plaintiff and defendant corporation, a rescission was but proper.
As defendants apparently failed to file their answer to the complaint, of which they were allegedly notified, the
Court declared them in default and proceeded to receive plaintiff's evidence. On June 8, 1955, the lower Court rendered
judgment granting plaintiff's prayer, and required defendants to render a complete accounting of the harvest of the land
subject of the proceeding within 15 days from receipt of the decision and to deliver 30 per cent of the net income
realized from the last harvest to plaintiff, with legal interest from the date defendants received payment for said crop. It
was further provided that upon defendants' failure to abide by the said requirement, the gross income would be fixed at
P4,200 or a net income of P3,200 after deducting the expenses for productions, 30 per cent of which or P960 was held
to be due the plaintiff pursuant to the aforementioned contract of lease, which was declared rescinded.
No appeal therefrom having been perfected within the reglementary period, the Court, upon motion of plaintiff,
issued a writ of execution, in virtue of which the Provincial Sheriff of Leyte caused the attachment of 3 parcels of land
registered in the name of Segundino Refuerzo. No property of the Philippine Fibers Producers Co., Inc., was found
available for attachment.
On January 31, 1956, defendant Segundino Refuerzo filed a motion claiming that the decision rendered in said
Civil Case No. 1912 was null and void with respect to him, there being no allegation in the complaint pointing to his
personal liability and thus prayed that an order be issued limiting such liability to defendant corporation. Over plaintiff's
opposition, the Court a quo granted the same and ordered the Provincial Sheriff of Leyte to release all properties
belonging to the movant that might have already been attached, after finding that the evidence on record made no
mention or referred to any fact which might hold movant personally liable therein. As plaintiff's petition for relief from
said order was denied, Manuela T. Vda. de Salvatierra instituted the instant action asserting that the trial Judge in
issuing the order complained of, acted with grave abuse of discretion and prayed that same be declared a nullity.
From the foregoing narration of facts, it is clear that the order sought to be nullified was issued by the
respondent Judge upon motion of defendant Refuerzo, obviously pursuant to Rule 38 of the Rules of Court. Section 3 of
said Rule, however, in providing for the period within which such a motion may be filed, prescribes that:
SEC. 3. WHEN PETITION FILED; CONTENTS AND VERIFICATION. — A petition provided for
in either of the preceding sections of this rule must be verified, filed within sixty days after the petitioner
learns of the judgment, order, or other proceeding to be set aside, and not more than six months after
such judgment or order was entered, or such proceeding was taken; and must be accompanied with
affidavit showing the fraud, accident, mistake, or excusable negligence relied upon, and the facts
constituting the petitioner's good and substantial cause of action or defense, as the case may be, which
he may prove if his petition be granted". (Rule 33)
The aforequoted provision treats of 2 periods, i.e., 60 days after petitioner learns of the judgment, and not more
than 6 months after the judgment or order was rendered, both of which must be satisfied. As the decision in the case at
bar was under date of June 8, 1955, whereas the motion filed by respondent Refuerzo was dated January 31, 1956, or
after the lapse of 7 months and 23 days, the filing of the aforementioned motion was clearly made beyond the
prescriptive period provided for by the rules. The remedy allowed by Rule 38 to a party adversely affected by a decision
or order is certainly an act of grace or benevolence intended to afford said litigant a penultimate opportunity to protect
his interest. Considering the nature of such relief and the purpose behind it, the periods fixed by said rule are non-
extendible and never interrupted; nor could it be subjected to any condition or contingency because it is of itself devised
to meet a condition or contingency (Palomares vs. Jimenez, * G. R. No. L-4513, January 31, 1952). On this score alone,
therefore, the petition for a writ of certiorari filed herein may be granted. However, taking note of the question presented
by the motion for relief involved herein, We deem it wise to delve in and pass upon the merit of the same.

Refuerzo, in praying for his exoneration from any liability resulting from the non-fulfillment of the obligation
imposed on defendant Philippine Fibers Producers Co., Inc, interposed the defense that the complaint filed with the
lower court contained no allegation which would hold him liable personally, for while it was stated therein that he was a
signatory to the lease contract, he did so in his capacity as president of the corporation. And this allegation was found
by the Court a quo to be supported by the records. Plaintiff on the other hand tried to refute this averment by contending
that her failure to specify defendant's personal liability was due to the fact that all the time she was under the impression
that the Philippine Fibers Producers Co., Inc., represented by Refuerzo was a duly registered corporation as appearing
in the contract, but a subsequent inquiry from the Securities & Exchange Commission yielded otherwise. While as a
general rule a person who has contracted or dealt with an association in such a way as to recognize its existence as a
corporate body is estopped from denying the same in an action arising out of such transaction or dealing, (Asia Banking
Corporation vs. Standard Products Co., 46 Phil., 144; Compañia Agrícola de Ultramar vs. Reyes, 4 Phil., 1; Ohta
Development Co. vs. Steamship Pompey, 49 Phil., 117), yet this doctrine may not be held to be applicable where fraud
takes a part in the said transaction. In the instant case, on plaintiff's charge that she was unaware of the fact that the
Philippine Fibers Producers Co., Inc., had no juridical personality, defendant Refuerzo gave no confirmation or denial
and the circumstances surrounding the execution of the contract lead to the inescapable conclusion that plaintiff
Manuela T. Vda. de Salvatierra was really made to believe that such corporation was duly organized in accordance with
law.
There can be no question that a corporation when registered has a juridical personality separate and distinct
from its component members or stockholders and officers such that a corporation cannot be held liable for the personal
indebtedness of a stockholder even if he should be its president (Walter A. Smith Co. vs. Ford, SC-G. R. No. 42420)
and conversely, a stockholder or member cannot be held personally liable for any financial obligation by the corporation
in excess of his unpaid subscription. But this rule is understood to refer merely to registered corporations and cannot be
made applicable to the liability of members of an unincorporated association. The reason behind this doctrine is obvious
— since an organization which before the law is non-existent has no personality and would be incompetent to act and
appropriate for itself the powers and attribute of a corporation as provided by law; it cannot create agents or confer
authority on another to act in its behalf; thus, those who act or purport to act as its representatives or agents do so
without authority and at their own risk. And as it is an elementary principle of law that a person who acts as an agent
without authority or without a principal is himself regarded as the principal, possessed of all the rights and subject to all
the liabilities of a principal, a person acting or purporting to act on behalf of a corporation which has no valid existence
assumes such privileges and obligations and becomes personally liable for contracts entered into or for other acts
performed as such agent (Fay vs. Noble, 7 Cushing [Mass.] 188. Cited in II Tolentino's Commercial Laws of the
Philippines, Fifth Ed., p. 689-690). Considering that defendant Refuerzo, as president of the unregistered corporation
Philippine Fibers Producers Co., Inc., was the moving spirit behind the consummation of the lease agreement by acting
as its representative, his liability cannot be limited or restricted to that imposed upon corporate shareholders. In acting
on behalf of a corporation which he knew to be unregistered, he assumed the risk of reaping the consequential
damages or resultant rights, if any, arising out of such transaction.
Wherefore, the order of the lower Court of March 21, 1956, amending its previous decision on this matter and
ordering the Provincial Sheriff of Leyte to release any and all properties of movant therein which might have been
attached in the execution of such judgment, is hereby set aside and nullified as if it had never been issued. With costs
against respondent Segundino Refuerzo. It is so ordered.
||| (Vda. de Salvatierra v. Garlitos, G.R. No. L-11442, [May 23, 1958], 103 PHIL 757-764)

[G.R. No. 58028. April 18, 1989.]

CHIANG KAI SHEK SCHOOL, petitioner, vs. COURT OF APPEALS and FAUSTINA FRANCO OH,
respondents.

SYLLABUS
1. REMEDIAL LAW; PARTIES IN A CIVIL ACTION; FAILURE OF SCHOOL TO INCORPORATE DOES NOT
EXEMPT IT FROM SUIT AS A JURIDICAL ENTITY. — It is true that Rule 3, Section 1, of the Rules of Court clearly
provides that "only natural or juridical persons may be parties in a civil action." It is also not denied that the school has
not been incorporated. However, this omission should not prejudice the private respondent in the assertion of her claims
against the school. As a school, the petitioner was governed by Act No. 2706 as amended by C.A. No. 180, which
provided as follows: Unless exempted for special reasons by the Secretary of Public Instruction, any private school or
college recognized by the government shall be incorporated under the provisions of Act No. 1459 known as the
Corporation Law, within 90 days after the date of recognition, and shall file with the Secretary of Public Instruction a
copy of its incorporation papers and by-laws. Having been recognized by the government, it was under obligation to
incorporate under the Corporation Law within 90 days from such recognition. It appears that it had not done so at the
time the complaint was filed notwithstanding that it had been in existence even earlier than 1932. The petitioner cannot
now invoke its own non-compliance with the law to immunize it from the private respondent's complaint.
2. ID.; ID.; SCHOOL HAVING REPRESENTED ITSELF AS POSSESSED OF JURIDICAL PERSONALITY
ESTOPPED FROM DENYING THE SAME. — There is no question that having contracted with the private respondent
every year for thirty two years and thus represented itself as possessed of juridical personality to do so, the petitioner
Chiang Kai Shek School is now estopped from denying such personality to defeat her claim against it. According to
Article 1431 of the Civil Code, "through estoppel an admission or representation is rendered conclusive upon the person
making it and cannot be denied or disproved as against the person relying on it."
3. LABOR LAW; CHARITABLE INSTITUTIONS COVERED THEREIN. — It is clear now that a charitable
institution is covered by the labor laws although the question was still unsettled when this case arose in 1968. At any
rate, there was no law even then exempting such institutions from the operation of the labor laws (although they were
exempted by the Constitution from ad valorem taxes).Hence, even assuming that the petitioner was a charitable
institution as it claims, the private respondent was nonetheless still entitled to the protection of the Termination Pay
Law, which was then in force.
4. ID.;TERMINATION OF EMPLOYMENT; DISMISSAL OF PERMANENT EMPLOYEE WITHOUT CAUSE
AND DUE NOTICE NOT PROPER. — The Court holds, after considering the particular circumstance of Oh's
employment, that she had become a permanent employee of the school and entitled to security of tenure at the time of
her dismissal. Since no cause was shown and established at an appropriate hearing, and the notice then required by
law had not been given, such dismissal was invalid. The private respondent's position is no different from that of the
rank-and-file employees involved in Gregorio Araneta University Foundation v. NLRC, of whom the Court had the
following to say: Undoubtedly, the private respondents' positions as deans and department heads of the petitioner
university are necessary in its usual business. Moreover, all the private respondents have been serving the university
from 18 to 28 years. All of them rose from the ranks starting as instructors until they became deans and department
heads of the university. A person who has served the University for 28 years and who occupies a high administrative
position in addition to teaching duties could not possibly be a temporary employee or a casual.
5. ID.;ID.;ILLEGAL DISMISSAL DONE IN WANTON AND OPPRESSIVE MANNER, AWARD OF MORAL AND
EXEMPLARY DAMAGES PROPER. — We find that the private respondent was arbitrarily treated by the petitioner,
which has shown no cause for her removal nor had it given her the notice required by the Termination Pay Law. As the
respondent court said, the contention that she did not report one week before the start of classes is a flimsy justification
for replacing her. She had been in its employ for all of thirty-two years. Her record was apparently unblemished. There
is no showing of any previous strained relations between her and the petitioner. Oh had every reason to assume, as
she had done in previous years, that she would continue teaching as usual. It is easy to imagine the astonishment and
hurt she felt when she was flatly and without warning told she was dismissed. There was not even the amenity of a
formal notice of her replacement, with perhaps a graceful expression of thanks for her past services. She was simply
informed she was no longer in the teaching staff. To put it bluntly, she was fired. For the wrongful act of the petitioner,
the private respondent is entitled to moral damages. As a proximate result of her illegal dismissal, she suffered mental
anguish, serious anxiety, wounded feelings and even besmirched reputation as an experienced teacher for more than
three decades. We also find that the respondent court did not err in awarding her exemplary damages because the
petitioner acted in a wanton and oppressive manner when it dismissed her.

DECISION
CRUZ,J p:

An unpleasant surprise awaited Fausta F. Oh when she reported for work at the Chiang Kai Shek School in
Sorsogon on the first week of July, 1968. She was told she had no assignment for the next semester. Oh was shocked.
She had been teaching in the school since 1932 for a continuous period of almost 33 years. And now, out of the blue,
and for no apparent or given reason, this abrupt dismissal.
Oh sued. She demanded separation pay, social security benefits, salary differentials, maternity benefits and
moral and exemplary damages. 1 The original defendant was the Chiang Kai Shek School but when it filed a motion to
dismiss on the ground that it could not be sued, the complaint was amended. 2 Certain officials of the school were also
impleaded to make them solidarily liable with the school.
The Court of First Instance of Sorsogon dismissed the complaint. 3 On appeal, its decision was set aside by the
respondent court, which held the school suable and liable while absolving the other defendants. 4 The motion for
reconsideration having been denied, 5 the school then came to this Court in this petition for review on certiorari.
The issues raised in the petition are:
1. Whether or not a school that has not been incorporated may be sued by reason alone of its long continued
existence and recognition by the government.
2. Whether or not a complaint filed against persons associated under a common name will justify a judgment
against the association itself and not its individual members.
3. Whether or not the collection of tuition fees and book rentals will make a school profit-making and not
charitable.
4. Whether or not the Termination Pay Law then in force was available to the private respondent who was
employed on a year-to-year basis.
5. Whether or not the awards made by the respondent court were warranted.
We hold against the petitioner on the first question. It is true that Rule 3, Section 1, of the Rules of Court clearly
provides that "only natural or juridical persons may be parties in a civil action." It is also not denied that the school has
not been incorporated. However, this omission should not prejudice the private respondent in the assertion of her claims
against the school. LLphil
As a school, the petitioner was governed by Act No. 2706 as amended by C.A. No. 180, which provided as
follows:
Unless exempted for special reasons by the Secretary of Public Instruction, any private school or college
recognized by the government shall be incorporated under the provisions of Act No. 1459 known as the
Corporation Law, within 90 days after the date of recognition, and shall file with the Secretary of Public
Instruction a copy of its incorporation papers and by-laws.
Having been recognized by the government, it was under obligation to incorporate under the Corporation Law
within 90 days from such recognition. It appears that it had not done so at the time the complaint was filed
notwithstanding that it had been in existence even earlier than 1932. The petitioner cannot now invoke its own non-
compliance with the law to immunize it from the private respondent's complaint.
There should also be no question that having contracted with the private respondent every year for thirty two
years and thus represented itself as possessed of juridical personality to do so, the petitioner is now estopped from
denying such personality to defeat her claim against it. According to Article 1431 of the Civil Code, "through estoppel an
admission representation is rendered conclusive upon the person making it and cannot be denied or disproved as
against the person relying on it."
As the school itself may be sued in its own name, there is no need to apply Rule 3, Section 15, under which the
persons joined in an association without any juridical personality may be sued with such association. Besides, it has
been shown that the individual members of the board of trustees are not liable, having been appointed only after the
private respondent's dismissal. 6
It is clear now that a charitable institution is covered by the labor laws 7 although the question was still
unsettled when this case arose in 1968. At any rate, there was no law even then exempting such institutions from the
operation of the labor laws (although they were exempted by the Constitution from ad valorem taxes).Hence, even
assuming that the petitioner was a charitable institution as it claims, the private respondent was nonetheless still entitled
to the protection of the Termination Pay Law, which was then in force.
While it may be that the petitioner was engaged in charitable works, it would not necessarily follow that those in
its employ were as generously motivated. Obviously, most of them would not have the means for such charity. The
private respondent herself was only a humble school teacher receiving a meager salary of P180.00 per month.
At that, it has not been established that the petitioner is a charitable institution, considering especially that it
charges tuition fees and collects book rentals from its students. 8 While this alone may not indicate that it is profit-
making, it does weaken its claim that it is a non-profit entity. llcd
The petitioner says the private respondent had not been illegally dismissed because her teaching contract was
on a yearly basis and the school was not required to rehire her in 1968. The argument is that her services were
terminable at the end of each year at the discretion of the school. Significantly explanation was given by the petitioner,
and no advance notice either, of her relief. After teaching year in and year out for all of thirty-two years, the private
respondent was simply told she could not teach any more.
The Court holds, after considering the particular circumstance of Oh's employment, that she had become a
permanent employee of the school and entitled to security of tenure at the time of her dismissal. Since no cause was
shown and established at an appropriate hearing, and the notice then required by law had not been given, such
dismissal was invalid.
The private respondent's position is no different from that of the rank-and-file employees involved in Gregorio
Araneta University Foundation v. NLRC, 9 of whom the Court had the following to say:
Undoubtedly, the private respondents' positions as deans and department heads of the petitioner
university are necessary in its usual business. Moreover, all the private respondents have been serving
the university from 18 to 28 years. All of them rose from the ranks starting as instructors until they
became deans and department heads of the university. A person who has served the University for 28
years and who occupies a high administrative position in addition to teaching duties could not possibly be
a temporary employee or a casual.
The applicable law is the Termination Pay Law, which provided:
SECTION 1. In cases of employment, without a definite period, in a commercial, industrial, or agricultural
establishment or enterprise, the employer or the employee may terminate at any time the employment
with just cause; or without just cause in the case of an employee by serving written notice on the
employer at least one month in advance, or in the case of an employer, by serving such notice to the
employee at least one month in advance or one-half month for every year of service of the employee,
whichever, is longer, a fraction of at least six months being considered as one whole year.
The employer, upon whom no such notice was served in case of termination of employment without just
cause may hold the employee liable for damages.
The employee, upon whom no such notice was served in case of termination of employment without just
cause shall be entitled to compensation from the date of termination of his employment in an amount
equivalent to his salaries or wages corresponding to the required period of notice. ...
The respondent court erred, however, in awarding her one month pay instead of only one-half month salary for
every year of service. The law is quite clear on this matter. Accordingly, the separation pay should be computed at
P90.00 times 32 months, for a total of P2,880.00. Cdpr
Parenthetically, R.A. No. 4670, otherwise known as the Magna Carta for Public School Teachers, confers
security of tenure on the teacher upon appointment as long as he possesses the required qualification. 10 And under
the present policy of the Department of Education, Culture and Sports, a teacher becomes permanent and automatically
acquires security of tenure upon completion of three years in the service. 11
While admittedly not applicable to the case at bar, these rules nevertheless reflect the attitude of the
government on the protection of the worker's security of tenure, which is now guaranteed by no less than the
Constitution itself. 12
We find that the private respondent was arbitrarily treated by the petitioner, which has shown no cause for her
removal nor had it given her the notice required by the Termination Pay Law. As the respondent court said, the
contention that she did not report one week before the start of classes is a flimsy justification for replacing her. 13 She
had been in its employ for all of thirty-two years. Her record was apparently unblemished. There is no showing of any
previous strained relations between her and the petitioner. Oh had every reason to assume, as she had done in
previous years, that she would continue teaching as usual.
It is easy to imagine the astonishment and hurt she felt when she was flatly and without warning told she was
dismissed. There was not even the amenity of a formal notice of her replacement, with perhaps a graceful expression of
thanks for her past services. She was simply informed she was no longer in the teaching staff. To put it bluntly, she was
fired.
For the wrongful act of the petitioner, the private respondent is entitled to moral damages. 14 As a proximate
result of her illegal dismissal, she suffered mental anguish, serious anxiety, wounded feelings and even besmirched
reputation as an experienced teacher for more than three decades. We also find that the respondent court did not err in
awarding her exemplary damages because the petitioner acted in a wanton and oppressive manner when it dismissed
her. 15
The Court takes this opportunity to pay a sincere tribute to the grade school teachers, who are always at the
forefront in the battle against illiteracy and ignorance. If only because it is they who open the minds of their pupils to an
unexplored world awash with the magic of letters and numbers, which is an extraordinary feat indeed, these humble
mentors deserve all our respect and appreciation.
WHEREFORE, the petition is DENIED. The appealed decision is AFFIRMED except for the award of
separation pay, which is reduced to P2,880.00. All the other awards are approved. Costs against the petitioner.
This decision is immediately executory. SO ORDERED.
||| (Chiang Kai Shek School v. Court of Appeals, G.R. No. 58028, [April 18, 1989], 254 PHIL 394-403)

[G.R. No. 125221. June 19, 1997.]

REYNALDO M. LOZANO, petitioner, vs. HON. ELIEZER R. DE LOS SANTOS, Presiding Judge, RTC,
Br. 58, Angeles City; and ANTONIO ANDA, respondents.

Willie B. Rivera for petitioner.


Yabut Law Office for respondents.

SYLLABUS

1. COMMERCIAL LAW; SECURITIES AND EXCHANGE COMMISSION; JURISDICTION; DETERMINATION


THEREOF. — The grant of jurisdiction to the SEC must be viewed in the light of its nature and function under the law. This
jurisdiction is determined by a concurrence of two elements: (1) the status or relationship of the parties; and (2) the nature of
the question that is the subject of their controversy. The first element requires that the controversy must arise out of
intracorporate or partnership relations between and among stockholders, members, or associates; between any or all of
them and the corporation, partnership or association of which they are stockholders, members or associates, respectively;
and between such corporation, partnership or association and the State in so far as it concerns their individual franchises.
The second element requires that the dispute among the parties be intrinsically connected with the regulation of the
corporation, partnership or association or deal with the internal affairs of the corporation, partnership or association. After
all, the principal function of the SEC is the supervision and control of corporations, partnerships and associations with the
end in view that investments in these entities may be encouraged and protected, and their activities pursued for the
promotion of economic development. DaScAI
2. ID.; ID.; ID.; DISPUTE BETWEEN MEMBERS OF TWO SEPARATE AND DISTINCT CORPORATIONS WHO
HAVE NO INTRACORPORATE RELATION, DOES NOT FALL WITHIN THE JURISDICTION OF SECURITIES AND
EXCHANGE COMMISSION; CASE AT BAR. — The KAMAJDA and SAMAJODA to which petitioner and private respondent
belong are duly registered with the SEC, but these associations are two separate entities. The dispute between petitioner
and private respondent is not within the KAMAJDA nor the SAMAJODA. It is between members of separate and distinct
associations. Petitioner and private respondent have no intracorporate relation much less do they have an intracorporate
dispute. The SEC therefore has no jurisdiction over the complaint.
3. ID.; ID.; ID.; DOCTRINE OF CORPORATION BY ESTOPPEL CANNOT OVERRIDE JURISDICTIONAL
REQUIREMENTS. — The doctrine of corporation by estoppel advanced by private respondent cannot override jurisdictional
requirements. Jurisdiction is fixed by law and is not subject to the agreement of the parties. It cannot be acquired through or
waived, enlarged or diminished by, any act or omission of the parties, neither can it be conferred by the acquiescence of the
court.
4. ID.; ID.; ID.; WHERE THERE IS NO THIRD PERSON INVOLVED AND THE CONFLICT ARISES ONLY AMONG
THOSE ASSUMING THE FORM OF A CORPORATION, THERE IS NO CORPORATION BY ESTOPPEL. — Corporation
by estoppel is founded on principles of equity and is designed to prevent injustice and unfairness. It applies when persons
assume to form a corporation and exercise corporate functions and enter into business relations with third persons. Where
there is no third person involved and the conflict arises only among those assuming the form of a corporation, who therefore
know that it has not been registered, there is no corporation by estoppel. EATcHD

DECISION

PUNO, J p:

This petition for certiorari seeks to annul and set aside the decision of the Regional Trial Court, Branch 58, Angeles
City which ordered the Municipal Circuit Trial Court, Mabalacat and Magalang, Pampanga to dismiss Civil Case No. 1214
for lack of jurisdiction.
The facts are undisputed. On December 19, 1995, petitioner Reynaldo M. Lozano filed Civil Case No. 1214 for
damages against respondent Antonio Anda before the Municipal Circuit Trial Court (MCTC), Mabalacat and Magalang,
Pampanga. Petitioner alleged that he was the president of the Kapatirang Mabalacat-Angeles Jeepney Drivers' Association,
Inc. (KAMAJDA) while respondent Anda was the president of the Samahang Angeles-Mabalacat Jeepney Operators' and
Drivers' Association, Inc. (SAMAJODA); in August 1995, upon the request of the Sangguniang Bayan of Mabalacat,
Pampanga, petitioner and private respondent agreed to consolidate their respective associations and form the Unified
Mabalacat-Angeles Jeepney Operators' and Drivers' Association, Inc. (UMAJODA); petitioner and private respondent also
agreed to elect one set of officers who shall be given the sole authority to collect the daily dues from the members of the
consolidated association; elections were held on October 29, 1995 and both petitioner and private respondent ran for
president; petitioner won; private respondent protested and, alleging fraud, refused to recognize the results of the election;
private respondent also refused to abide by their agreement and continued collecting the dues from the members of his
association despite several demands to desist. Petitioner was thus constrained to file the complaint to restrain private
respondent from collecting the dues and to order him to pay damages in the amount of P25,000.00 and attorney's fees of
P500.00. 1
Private respondent moved to dismiss the complaint for lack of jurisdiction, claiming that jurisdiction was lodged with
the Securities and Exchange Commission (SEC). The MCTC denied the motion on February 9, 1996. 2 It denied
reconsideration on March 8, 1996. 3
Private respondent filed a petition for certiorari before the Regional Trial Court, Branch 58, Angeles City. 4 The trial
court found the dispute to be intracorporate, hence, subject to the jurisdiction of the SEC, and ordered the MCTC to dismiss
Civil Case No. 1214 accordingly. 5 It denied reconsideration on May 31, 1996. 6
Hence this petition. Petitioner claims that:
"THE RESPONDENT JUDGE ACTED WITH GRAVE ABUSE OF DISCRETION AMOUNTING TO LACK
OR EXCESS OF JURISDICTION AND SERIOUS ERROR OF LAW IN CONCLUDING THAT THE
SECURITIES AND EXCHANGE COMMISSION HAS JURISDICTION OVER A CASE OF DAMAGES
BETWEEN HEADS/PRESIDENTS OF TWO (2) ASSOCIATIONS WHO INTENDED TO
CONSOLIDATE/MERGE THEIR ASSOCIATIONS BUT NOT YET [SIC] APPROVED AND REGISTERED
WITH THE SECURITIES AND EXCHANGE COMMISSION." 7
The jurisdiction of the Securities and Exchange Commission (SEC) is set forth in Section 5 of Presidential Decree
No. 902-A.. Section 5 reads as follows:
"Section 5. . . . [T]he Securities and Exchange Commission [has] original and exclusive
jurisdiction to hear and decide cases involving:
(a) Devices or schemes employed by or any acts of the board of directors, business associates,
its officers or partners, amounting to fraud and misrepresentation which may be detrimental to the interest
of the public and/or of the stockholders, partners, members of associations or organizations registered
with the Commission. cdtai
(b) Controversies arising out of intracorporate or partnership relations, between and among
stockholders, members or associates; between any or all of them and the corporation, partnership or
association of which they are stockholders, members, or associates, respectively; and between such
corporation, partnership or association and the state insofar as it concerns their individual franchise or
right to exist as such entity.
(c) Controversies in the election or appointment of directors, trustees, officers or managers of
such corporations, partnerships or associations.
(d) Petitions of corporations, partnerships or associations to be declared in the state of
suspension of payments in cases where the corporation, partnership or association possesses sufficient
property to cover all its debts but foresees the impossibility of meeting them when they respectively fall
due or in cases where the corporation, partnership or association has no sufficient assets to cover its
liabilities, but is under the management of a Rehabilitation Receiver or Management Committee created
pursuant to this Decree."
The grant of jurisdiction to the SEC must be viewed in the light of its nature and function under the law. 8 This
jurisdiction is determined by a concurrence of two elements: (1) the status or relationship of the parties; and (2) the nature of
the question that is the subject of their controversy. 9
The first element requires that the controversy must arise out of intracorporate or partnership relations between and
among stockholders, members, or associates; between any or all of them and the corporation, partnership or association of
which they are stockholders, members or associates, respectively; and between such corporation, partnership or
association and the State in so far as it concerns their individual franchises. 10 The second element requires that the
dispute among the parties be intrinsically connected with the regulation of the corporation, partnership or association or deal
with the internal affairs of the corporation, partnership or association. 11 After all, the principal function of the SEC is the
supervision and control of corporations, partnerships and associations with the end in view that investments in these entities
may be encouraged and protected, and their activities pursued for the promotion of economic development. 12
There is no intracorporate nor partnership relation between petitioner and private respondent. The controversy
between them arose out of their plan to consolidate their respective jeepney drivers' and operators' associations into a
single common association. This unified association was, however, still a proposal. It had not been approved by the SEC,
neither had its officers and members submitted their articles of consolidation in accordance with Sections 78 and 79 of the
Corporation Code. Consolidation becomes effective not upon mere agreement of the members but only upon issuance of
the certificate of consolidation by the SEC. 13 When the SEC, upon processing and examining the articles of consolidation,
is satisfied that the consolidation of the corporations is not inconsistent with the provisions of the Corporation Code and
existing laws, it issues a certificate of consolidation which makes the reorganization official. 14 The new consolidated
corporation comes into existence and the constituent corporations dissolve and cease to exist. 15
The KAMAJDA and SAMAJODA to which petitioner and private respondent belong are duly registered with the
SEC, but these associations are two separate entities. The dispute between petitioner and private respondent is not within
the KAMAJDA nor the SAMAJODA. It is between members of separate and distinct associations. Petitioner and private
respondent have no intracorporate relation much less do they have an intracorporate dispute. The SEC therefore has no
jurisdiction over the complaint.
The doctrine of corporation by estoppel 16 advanced by private respondent cannot override jurisdictional
requirements. Jurisdiction is fixed by law and is not subject to the agreement of the parties. 17 It cannot be acquired through
or waived, enlarged or diminished by, any act or omission of the parties, neither can it be conferred by the acquiescence of
the court. 18
Corporation by estoppel is founded on principles of equity and is designed to prevent injustice and unfairness. 19 It
applies when persons assume to form a corporation and exercise corporate functions and enter into business relations with
third persons. Where there is no third person involved and the conflict arises only among those assuming the form of a
corporation, who therefore know that it has not been registered there is no corporation by estoppel. 20
IN VIEW WHEREOF, the petition is granted and the decision dated April 18, 1996 and the order dated May 31,
1996 of the Regional Trial Court, Branch 58, Angeles City are set aside. The Municipal Circuit Trial Court of Mabalacat and
Magalang, Pampanga is ordered to proceed with dispatch in resolving Civil Case No. 1214. No costs. SO ORDERED.
||| (Lozano v. De los Santos, G.R. No. 125221, [June 19, 1997], 340 PHIL 563-570)
LIM TONG LIM, petitioner, vs. PHILIPPINE FISHING GEAR INDUSTRIES, INC., respondent.

Roberto A. Abad for petitioner.


Benjamin S. Benito & Associates for private respondent.

SYNOPSIS

Antonio Chua and Peter Yao entered into a contract in behalf of Ocean Quest Fishing Corporation for the purchase
of fishing nets from respondent Philippine Fishing Gear Industries, Inc. Chua and Yao claimed that they were engaged in
business venture with petitioner Lim Tong Lim, who, however, was not a signatory to the contract. The buyers failed to pay
the fishing nets. Respondent filed a collection against Chua, Yao and petitioner Lim in their capacities as general partners
because it turned out that Ocean Quest Fishing Corporation is a non-existent corporation. The trial court issued a Writ of
Preliminary Attachment, which the sheriff enforced by attaching the fishing nets. The trial court rendered its decision ruling
that respondent was entitled to the Writ of Attachment and that Chua, Yao and Lim, as general partners, were jointly liable
to pay respondent. Lim appealed to the Court of Appeals, but the appellate court affirmed the decision of the trial court that
petitioner Lim is a partner and may thus be held liable as such. Hence, the present petition. Petitioner claimed that since his
name did not appear on any of the contracts and since he never directly transacted with the respondent corporation, ergo,
he cannot be held liable. cIaCTS
The Supreme Court denied the petition. The Court ruled that having reaped the benefits of the contract entered into
by Chua and Yao, with whom he had an existing relationship, petitioner Lim is deemed a part of said association and is
covered by the doctrine of corporation by estoppel. The Court also ruled that under the principle of estoppel, those acting on
behalf of a corporation and those benefited by it, knowing it to be without valid existence, are held liable as general partners.

SYLLABUS

1. CIVIL LAW; PARTNERSHIP; AGREEMENT THAT ANY LOSS OR PROFIT FROM THE SALE AND
OPERATION OF THE BOATS WOULD BE DIVIDED EQUALLY AMONG THEM SHOWS THAT THE PARTIES HAD
INDEED FORMED A PARTNERSHIP. — From the factual findings of both lower courts, it is clear that Chua, Yao and Lim
had decided to engage in a fishing business, which they started by buying boats worth P3.35 million, financed by a loan
secured from Jesus Lim who was petitioner's brother. In their Compromise Agreement, they subsequently revealed their
intention to pay the loan with the proceeds of the sale of the boats, and to divide equally among them the excess or loss.
These boats, the purchase and the repair of which were financed with borrowed money, fell under the term "common fund"
under Article 1767. The contribution to such fund need not be cash or fixed assets; it could be an intangible like credit or
industry. That the parties agreed that any loss or profit from the sale and operation of the boats would be divided equally
among them also shows that they had indeed formed a partnership. Moreover, it is clear that the partnership extended not
only to the purchase of the boat, but also to that of the nets and the floats. The fishing nets and the floats, both essential to
fishing, were obviously acquired in furtherance of their business. It would have been inconceivable for Lim to involve himself
so much in buying the boat but not in the acquisition of the aforesaid equipment, without which the business could not have
proceeded. Given the preceding facts, it is clear that there was, among petitioner, Chua and Yao, a partnership engaged in
the fishing business. They purchased the boats, which constituted the main assets of the partnership, and they agreed that
the proceeds from the sales and operations thereof would be divided among them.
2. ID.; ID.; COMPROMISE AGREEMENT OF THE PARTIES NOT THE SOLE BASIS OF PARTNERSHIP. —
Petitioner argues that the appellate court's sole basis for assuming the existence of a partnership was the Compromise
Agreement. He also claims that the settlement was entered into only to end the dispute among them, but not to adjudicate
their preexisting rights and obligations. His arguments are baseless. The Agreement was but an embodiment of the
relationship extant among the parties prior to its execution. A proper adjudication of claimants' rights mandates that courts
must review and thoroughly appraise all relevant facts. Both lower courts have done so and have found, correctly, a
preexisting partnership among the parties. In implying that the lower courts have decided on the basis of one piece of
document alone, petitioner fails to appreciate that the CA and the RTC delved into the history of the document and explored
all the possible consequential combinations in harmony with law, logic and fairness. Verily, the two lower courts' factual
findings mentioned above nullified petitioner's argument that the existence of a partnership was based only on the
Compromise Agreement.
3. ID.; ID.; PETITIONER WAS A PARTNER, NOT A LESSOR. — Verily, as found by the lower courts, petitioner
entered into a business agreement with Chua and Yao, in which debts were undertaken in order to finance the acquisition
and the upgrading of the vessels which would be used in their fishing business. The sale of the boats, as well as the division
among the three of the balance remaining after the payment of their loans, proves beyond cavil that F/B Lourdes, though
registered in his name, was not his own property but an asset of the partnership. It is not uncommon to register the
properties acquired from a loan in the name of the person the lender trusts, who in this case is the petitioner himself. After
all, he is the brother of the creditor, Jesus Lim. We stress that it is unreasonable — indeed, it is absurd — for petitioner to
sell his property to pay a debt he did not incur, if the relationship among the three of them was merely that of lessor-lessee,
instead of partners.
4. MERCANTILE LAW; PRIVATE CORPORATIONS; HAVING REAPED THE BENEFITS OF THE CONTRACT
ENTERED INTO BY PERSONS WITH WHOM HE PREVIOUSLY HAD AN EXISTING RELATIONSHIP, PETITIONER IS
DEEMED TO BE PART OF SAID ASSOCIATION AND IS COVERED BY THE DOCTRINE OF CORPORATION BY
ESTOPPEL. — There is no dispute that the respondent, Philippine Fishing Gear Industries, is entitled to be paid for the nets
it sold. The only question here is whether petitioner should be held jointly liable with Chua and Yao. Petitioner contests such
liability, insisting that only those who dealt in the name of the ostensible corporation should be held liable. Since his name
does not appear on any of the contracts and since he never directly transacted with the respondent corporation, ergo, he
cannot be held liable. Unquestionably, petitioner benefited from the use of the nets found inside F/B Lourdes, the boat
which has earlier been proven to be an asset of the partnership. He in fact questions the attachment of the nets, because
the Writ has effectively stopped his use of the fishing vessel. It is difficult to disagree with the RTC and the CA that Lim,
Chua and Yao decided to form a corporation. Although it was never legally formed for unknown reasons, this fact alone
does not preclude the liabilities of the three as contracting parties in representation of it. Clearly, under the law on estoppel,
those acting on behalf of a corporation and those benefited by it, knowing it to be without valid existence, are held liable as
general partners. Technically, it is true that petitioner did not directly act on behalf of the corporation. However, having
reaped the benefits of the contract entered into by persons with whom he previously had an existing relationship, he is
deemed to be part of said association and is covered by the scope of the doctrine of corporation by estoppel.
5. REMEDIAL LAW; PROVISIONAL REMEDIES; ATTACHMENT; ISSUE OF VALIDITY THEREOF, MOOT AND
ACADEMIC. — Petitioner claims that the Writ of Attachment was improperly issued against the nets. We agree with the
Court of Appeals that this issue is now moot and academic. As previously discussed, F/B Lourdes was an asset of the
partnership and that it was placed in the name of petitioner, only to assure payment of the debt he and his partners owed.
The nets and the floats were specifically manufactured and tailor-made according to their own design, and were bought and
used in the fishing venture they agreed upon. Hence, the issuance of the Writ to assure the payment of the price stipulated
in the invoices is proper. Besides, by specific agreement, ownership of the nets remained with Respondent Philippine
Fishing Gear, until full payment thereof.
VITUG, J., concurring:
1. CIVIL LAW; PARTNERSHIP; EXTENT OF LIABILITY OF PARTNERS IN A GENERAL PARTNERSHIP. —
When a person by his act or deed represents himself. as a partner in an existing partnership or with one or more persons
not actual partners, he is deemed an agent of such persons consenting to such representation and in the same manner, if
he were a partner, with respect to persons who rely upon the representation. The association formed by Chua, Yao and
Lim, should be, as it has been deemed, a de facto partnership with all the consequent obligations for the purpose of
enforcing the rights of third persons. The liability of general partners (in a general partnership as so opposed to a limited
partnership) is laid down in Article 1816 which posits that all partners shall be liable pro rata beyond the partnership assets
for all the contracts which may have been entered into in its name, under its signature, and by a person authorized to act for
the partnership.
2. ID.; ID.; ID.; INSTANCES WHEN THE PARTNERS CAN BE HELD SOLIDARILY LIABLE WITH THE
PARTNERSHIP. — This rule is to be construed along with other provisions of the Civil Code which postulate that the
partners can be held solidarily liable with the partnership specifically in these instances. — (1) where, by any wrongful act or
omission of any partner acting in the ordinary course of the business of the partnership or with the authority of his co-
partners, loss or injury is caused to any person, not being a partner in the partnership, or any penalty is incurred, the
partnership is liable therefor to the same extent as the partner so acting or omitting to act; (2) where one partner acting
within the scope of his apparent authority receives money or property of a third person and misapplies it; and (3) where the
partnership in the course of its business receives money or property of a third person and the money or property so
received is misapplied by any partner while it is in the custody of the partnership — consistently with the rules on the nature
of civil liability in delicts and quasi-delicts.
DECISION

PANGANIBAN, J p:

A partnership may be deemed to exist among parties who agree to borrow money to pursue a business and to
divide the profits or losses that may arise therefrom, even if it is shown that they have not contributed any capital of their
own to a "common fund." Their contribution may be in the form of credit or industry, not necessarily cash or fixed assets.
Being partners, they are all liable for debts incurred by or on behalf of the partnership. The liability for a contract entered into
on behalf of an unincorporated association or ostensible corporation may lie in a person who may not have directly
transacted on its behalf, but reaped benefits from that contract. cda
The Case
In the Petition for Review on Certiorari before us, Lim Tong Lim assails the November 26, 1998 Decision of the
Court of Appeals in CA-GR CV 41477, 1 which disposed as follows:
"WHEREFORE, [there being] no reversible error in the appealed decision, the same is hereby
affirmed." 2
The decretal portion of the Quezon City Regional Trial Court (RTC) ruling, which was affirmed by the CA, reads as
follows:
"WHEREFORE, the Court rules:
1. That plaintiff is entitled to the writ of preliminary attachment issued by this Court on September
20, 1990; cdphil
2. That defendants are jointly liable to plaintiff for the following amounts, subject to the
modifications as hereinafter made by reason of the special and unique facts and circumstances and the
proceedings that transpired during the trial of this case;
a. P532,045.00 representing [the] unpaid purchase price of the fishing nets covered by the
Agreement plus P68,000.00 representing the unpaid price of the floats not covered by said Agreement;
b. 12% interest per annum counted from date of plaintiff's invoices and computed on their
respective amounts as follows:
i. Accrued interest of P73,221.00 on Invoice No. 14407 for P385,377.80 dated February 9, 1990;
ii. Accrued interest of P27,904.02 on Invoice No. 14413 for P146,868.00 dated February 13,
1990;
iii. Accrued interest of P12,920.00 on Invoice No. 14426 for P68,000.00 dated February 19,
1990;
c. P50,000.00 as and for attorney's fees, plus P8,500.00 representing P500.00 per appearance
in court;
d. P65,000.00 representing P5,000.00 monthly rental for storage charges on the nets counted
from September 20, 1990 (date of attachment) to September 12, 1991 (date of auction sale); cdasia
e. Cost of suit.
"With respect to the joint liability of defendants for the principal obligation or for the unpaid price
of nets and floats in the amount of P532,045.00 and P68,000.00, respectively, or for the total amount of
P600,045.00, this Court noted that these items were attached to guarantee any judgment that may be
rendered in favor of the plaintiff but, upon agreement of the parties, and, to avoid further deterioration of
the nets during the pendency of this case, it was ordered sold at public auction for not less than
P900,000.00 for which the plaintiff was the sole and winning bidder. The proceeds of the sale paid for by
plaintiff was deposited in court. In effect, the amount of P900,000.00 replaced the attached property as a
guaranty for any judgment that plaintiff may be able to secure in this case with the ownership and
possession of the nets and floats awarded and delivered by the sheriff to plaintiff as the highest bidder in
the public auction sale. It has also been noted that ownership of the nets [was] retained by the plaintiff
until full payment [was] made as stipulated in the invoices; hence, in effect, the plaintiff attached its own
properties. It [was] for this reason also that this Court earlier ordered the attachment bond filed by plaintiff
to guaranty damages to defendants to be cancelled and for the P900,000.00 cash bidded and paid for by
plaintiff to serve as its bond in favor of defendants.
"From the foregoing, it would appear therefore that whatever judgment the plaintiff may be
entitled to in this case will have to be satisfied from the amount of P900,000.00 as this amount replaced
the attached nets and floats. Considering, however, that the total judgment obligation as computed above
would amount to only P840,216.92, it would be inequitable, unfair and unjust to award the excess to the
defendants who are not entitled to damages and who did not put up a single centavo to raise the amount
of P900,000.00 aside from the fact that they are not the owners of the nets and floats. For this reason,
the defendants are hereby relieved from any and all liabilities arising from the monetary judgment
obligation enumerated above and for plaintiff to retain possession and ownership of the nets and floats
and for the reimbursement of the P900,000.00 deposited by it with the Clerk of Court.
SO ORDERED." 3 cdasia
The Facts
On behalf of "Ocean Quest Fishing Corporation," Antonio Chua and Peter Yao entered into a Contract dated
February 7, 1990, for the purchase of fishing nets of various sizes from the Philippine Fishing Gear Industries, Inc. (herein
respondent). They claimed that they were engaged in a business venture with Petitioner Lim Tong Lim, who however was
not a signatory to the agreement. The total price of the nets amounted to P532,045. Four hundred pieces of floats worth
P68,000 were also sold to the Corporation. 4
The buyers, however, failed to pay for the fishing nets and the floats; hence, private respondent filed a collection
suit against Chua, Yao and Petitioner Lim Tong Lim with a prayer for a writ of preliminary attachment. The suit was brought
against the three in their capacities as general partners, on the allegation that "Ocean Quest Fishing Corporation" was a
nonexistent corporation as shown by a Certification from the Securities and Exchange Commission. 5 On September 20,
1990, the lower court issued a Writ of Preliminary Attachment, which the sheriff enforced by attaching the fishing nets on
board F/B Lourdes which was then docked at the Fisheries Port, Navotas, Metro Manila. LLpr
Instead of answering the Complaint, Chua filed a Manifestation admitting his liability and requesting a reasonable
time within which to pay. He also turned over to respondent some of the nets which were in his possession. Peter Yao filed
an Answer, after which he was deemed to have waived his right to cross-examine witnesses and to present evidence on his
behalf, because of his failure to appear in subsequent hearings. Lim Tong Lim, on the other hand, filed an Answer with
Counterclaim and Crossclaim and moved for the lifting of the Writ of Attachment. 6 The trial court maintained the Writ, and
upon motion of private respondent, ordered the sale of the fishing nets at a public auction. Philippine Fishing Gear
Industries won the bidding and deposited with the said court the sales proceeds of P900,000. 7
On November 18, 1992, the trial court rendered its Decision, ruling that Philippine Fishing Gear Industries was
entitled to the Writ of Attachment and that Chua, Yao and Lim, as general partners, were jointly liable to pay respondent. 8
The trial court ruled that a partnership among Lim, Chua and Yao existed based (1) on the testimonies of the
witnesses presented and (2) on a Compromise Agreement executed by the three 9 in Civil Case No. 1492-MN which Chua
and Yao had brought against Lim in the RTC of Malabon, Branch 72, for (a) a declaration of nullity of commercial
documents; (b) a reformation of contracts; (c) a declaration of ownership of fishing boats; (d) an injunction and (e) damages.
10 The Compromise Agreement provided: cdll
"a) That the parties plaintiffs & Lim Tong Lim agree to have the four (4) vessels sold in the
amount of P5,750,000.00 including the fishing net. This P5,750,000.00 shall be applied
as full payment for P3,250,000.00 in favor of JL Holdings Corporation and/or Lim Tong
Lim;
"b) If the four (4) vessel[s] and the fishing net will be sold at a higher price than P5,750,000.00
whatever will be the excess will be divided into 3: 1/3 Lim Tong Lim; 1/3 Antonio Chua;
1/3 Peter Yao;
"c) If the proceeds of the sale the vessels will be less than P5,750,000.00 whatever the
deficiency shall be shouldered and paid to JL Holding Corporation by 1/3 Lim Tong Lim;
1/3 Antonio Chua; 1/3 Peter Yao." 11
The trial court noted that the Compromise Agreement was silent as to the nature of their obligations, but that joint
liability could be presumed from the equal distribution of the profit and loss. 12
Lim appealed to the Court of Appeals (CA) which, as already stated, affirmed the RTC.
Ruling of the Court of Appeals
In affirming the trial court, the CA held that petitioner was a partner of Chua and Yao in a fishing business and may
thus be held liable as such for the fishing nets and floats purchased by and for the use of the partnership. The appellate
court ruled:
"The evidence establishes that all the defendants including herein appellant Lim Tong Lim
undertook a partnership for a specific undertaking, that is for commercial fishing . . . . Obviously, the
ultimate undertaking of the defendants was to divide the profits among themselves which is what a
partnership essentially is . . . . By a contract of partnership, two or more persons bind themselves to
contribute money, property or industry to a common fund with the intention of dividing the profits among
themselves (Article 1767, New Civil Code)." 13 cdtai
Hence, petitioner brought this recourse before this Court. 14
The Issues
In his Petition and Memorandum, Lim asks this Court to reverse the assailed Decision on the following grounds:
"I THE COURT OF APPEALS ERRED IN HOLDING, BASED ON A COMPROMISE
AGREEMENT THAT CHUA, YAO AND PETITIONER LIM ENTERED INTO IN A
SEPARATE CASE, THAT A PARTNERSHIP AGREEMENT EXISTED AMONG THEM.
"II SINCE IT WAS ONLY CHUA WHO REPRESENTED THAT HE WAS ACTING FOR OCEAN
QUEST FISHING CORPORATION WHEN HE BOUGHT THE NETS FROM PHILIPPINE
FISHING, THE COURT OF APPEALS WAS UNJUSTIFIED IN IMPUTING LIABILITY TO
PETITIONER LIM AS WELL.
"III THE TRIAL COURT IMPROPERLY ORDERED THE SEIZURE AND ATTACHMENT OF
PETITIONER LIM'S GOODS."
In determining whether petitioner may be held liable for the fishing nets and floats purchased from respondent, the
Court must resolve this key issue: whether by their acts, Lim, Chua and Yao could be deemed to have entered into a
partnership. cdasia
This Court's Ruling
The Petition is devoid of merit.
First and Second Issues:
Existence of a Partnership
and Petitioner's Liability
In arguing that he should not be held liable for the equipment purchased from respondent, petitioner controverts the
CA finding that a partnership existed between him, Peter Yao and Antonio Chua. He asserts that the CA based its finding on
the Compromise Agreement alone. Furthermore, he disclaims any direct participation in the purchase of the nets, alleging
that the negotiations were conducted by Chua and Yao only, and that he has not even met the representatives of the
respondent company. Petitioner further argues that he was a lessor, not a partner, of Chua and Yao, for the "Contract of
Lease" dated February 1, 1990, showed that he had merely leased to the two the main asset of the purported partnership —
the fishing boat F/B Lourdes. The lease was for six months, with a monthly rental of P37,500 plus 25 percent of the gross
catch of the boat.
We are not persuaded by the arguments of petitioner. The facts as found by the two lower courts clearly showed
that there existed a partnership among Chua, Yao and him, pursuant to Article 1767 of the Civil Code which provides:
"ARTICLE 1767. By the contract of partnership, two or more persons bind themselves to
contribute money, property, or industry to a common fund, with the intention of dividing the profits among
themselves." llcd
Specifically, both lower courts ruled that a partnership among the three existed based on the following factual
findings: 15
(1) That Petitioner Lim Tong Lim requested Peter Yao who was engaged in commercial fishing to
join him, while Antonio Chua was already Yao's partner;
(2) That after convening for a few times, Lim Chua, and Yao verbally agreed to acquire two
fishing boats, the FB Lourdes and the FB Nelson for the sum of P3.35 million;
(3) That they borrowed P3.25 million from Jesus Lim, brother of Petitioner Lim Tong Lim, to
finance the venture.
(4) That they bought the boats from CMF Fishing Corporation, which executed a Deed of Sale
over these two (2) boats in favor of Petitioner Lim Tong Lim only to serve as security for the loan
extended by Jesus Lim;
(5) That Lim, Chua and Yao agreed that the refurbishing, re-equipping, repairing, dry docking
and other expenses for the boats would be shouldered by Chua and Yao;
(6) That because of the "unavailability of funds," Jesus Lim again extended a loan to the
partnership in the amount of P1 million secured by a check, because of which, Yao and Chua entrusted
the ownership papers of two other boats, Chua's FB Lady Anne Mel and Yao's FB Tracy to Lim Tong
Lim. cdtai
(7) That in pursuance of the business agreement, Peter Yao and Antonio Chua bought nets from
Respondent Philippine Fishing Gear, in behalf of "Ocean Quest Fishing Corporation," their purported
business name.
(8) That subsequently, Civil Case No. 1492-MN was filed in the Malabon RTC, Branch 72 by
Antonio Chua and Peter Yao against Lim Tong Lim for (a) declaration of nullity of commercial documents;
(b) reformation of contracts; (c) declaration of ownership of fishing boats; (4) injunction; and (e) damages.
(9) That the case was amicably settled through a Compromise Agreement executed between the
parties-litigants the terms of which are already enumerated above.
From the factual findings of both lower courts, it is clear that Chua, Yao and Lim had decided to engage in a fishing
business, which they started by buying boats worth P3.35 million, financed by a loan secured from Jesus Lim who was
petitioner's brother. In their Compromise Agreement, they subsequently revealed their intention to pay the loan with the
proceeds of the sale of the boats, and to divide equally among them the excess or loss. These boats, the purchase and the
repair of which were financed with borrowed money, fell under the term "common fund" under Article 1767. The contribution
to such fund need not be cash or fixed assets; it could be an intangible like credit or industry. That the parties agreed that
any loss or profit from the sale and operation of the boats would be divided equally among them also shows that they had
indeed formed a partnership.
Moreover, it is clear that the partnership extended not only to the purchase of the boat, but also to that of the nets
and the floats. The fishing nets and the floats, both essential to fishing, were obviously acquired in furtherance of their
business. It would have been inconceivable for Lim to involve himself so much in buying the boat but not in the acquisition
of the aforesaid equipment, without which the business could not have proceeded. cdtai
Given the preceding facts, it is clear that there was, among petitioner, Chua and Yao, a partnership engaged in the
fishing business. They purchased the boats, which constituted the main assets of the partnership, and they agreed that the
proceeds from the sales and operations thereof would be divided among them.
We stress that under Rule 45, a petition for review like the present case should involve only questions of law. Thus,
the foregoing factual findings of the RTC and the CA are binding on this Court, absent any cogent proof that the present
action is embraced by one of the exceptions to the rule. 16 In assailing the factual findings of the two lower courts, petitioner
effectively goes beyond the bounds of a petition for review under Rule 45.
Compromise Agreement
Not the Sole Basis of Partnership
Petitioner argues that the appellate court's sole basis for assuming the existence of a partnership was the
Compromise Agreement. He also claims that the settlement was entered into only to end the dispute among them, but not
to adjudicate their preexisting rights and obligations. His arguments are baseless. The Agreement was but an embodiment
of the relationship extant among the parties prior to its execution.
A proper adjudication of claimants' rights mandates that courts must review and thoroughly appraise all relevant
facts. Both lower courts have done so and have found, correctly, a preexisting partnership among the parties. In implying
that the lower courts have decided on the basis of one piece of document alone, petitioner fails to appreciate that the CA
and the RTC delved into the history of the document and explored all the possible consequential combinations in harmony
with law, logic and fairness. Verily, the two lower courts' factual findings mentioned above nullified petitioner's argument that
the existence of a partnership was based only on the Compromise Agreement. LLphil
Petitioner Was a Partner,
Not a Lessor
We are not convinced by petitioner's argument that he was merely the lessor of the boats to Chua and Yao, not a
partner in the fishing venture. His argument allegedly finds support in the Contract of Lease and the registration papers
showing that he was the owner of the boats, including F/B Lourdes where the nets were found.
His allegation defies logic. In effect, he would like this Court to believe that he consented to the sale of his own
boats to pay a debt of Chua and Yao, with the excess of the proceeds to be divided among the three of them. No lessor
would do what petitioner did. Indeed, his consent to the sale proved that there was a preexisting partnership among all
three.
Verily, as found by the lower courts, petitioner entered into a business agreement with Chua and Yao, in which
debts were undertaken in order to finance the acquisition and the upgrading of the vessels which would be used in their
fishing business. The sale of the boats, as well as the division among the three of the balance remaining after the payment
of their loans, proves beyond cavil that F/B Lourdes, though registered in his name, was not his own property but an asset
of the partnership. It is not uncommon to register the properties acquired from a loan in the name of the person the lender
trusts, who in this case is the petitioner himself. After all, he is the brother of the creditor, Jesus Lim. prLL
We stress that it is unreasonable — indeed, it is absurd — for petitioner to sell his property to pay a debt he did not
incur, if the relationship among the three of them was merely that of lessor-lessee, instead of partners.
Corporation by Estoppel
Petitioner argues that under the doctrine of corporation by estoppel, liability can be imputed only to Chua and Yao,
and not to him. Again, we disagree.
Section 21 of the Corporation Code of the Philippines provides:
"Sec. 21. Corporation by estoppel. — All persons who assume to act as a corporation knowing it
to be without authority to do so shall be liable as general partners for all debts, liabilities and damages
incurred or arising as a result thereof: Provided however, That when any such ostensible corporation is
sued on any transaction entered by it as a corporation or on any tort committed by it as such, it shall not
be allowed to use as a defense its lack of corporate personality.
"One who assumes an obligation to an ostensible corporation as such, cannot resist
performance thereof on the ground that there was in fact no corporation." LibLex
Thus, even if the ostensible corporate entity is proven to be legally nonexistent, a party may be estopped from
denying its corporate existence. "The reason behind this doctrine is obvious — an unincorporated association has no
personality and would be incompetent to act and appropriate for itself the power and attributes of a corporation as provided
by law; it cannot create agents or confer authority on another to act in its behalf; thus, those who act or purport to act as its
representatives or agents do so without authority and at their own risk. And as it is an elementary principle of law that a
person who acts as an agent without authority or without a principal is himself regarded as the principal, possessed of all
the right and subject to all the liabilities of a principal, a person acting or purporting to act on behalf of a corporation which
has no valid existence assumes such privileges and obligations and becomes personally liable for contracts entered into or
for other acts performed as such agent." 17
The doctrine of corporation by estoppel may apply to the alleged corporation and to a third party. In the first
instance, an unincorporated association, which represented itself to be a corporation, will be estopped from denying its
corporate capacity in a suit against it by a third person who relied in good faith on such representation. It cannot allege lack
of personality to be sued to evade its responsibility for a contract it entered into and by virtue of which it received
advantages and benefits.
On the other hand, a third party who, knowing an association to be unincorporated, nonetheless treated it as a
corporation and received benefits from it, may be barred from denying its corporate existence in a suit brought against the
alleged corporation. In such case, all those who benefited from the transaction made by the ostensible corporation, despite
knowledge of its legal defects, may be held liable for contracts they impliedly assented to or took advantage of. cdrep
There is no dispute that the respondent, Philippine Fishing Gear Industries, is entitled to be paid for the nets it sold.
The only question here is whether petitioner should be held jointly 18 liable with Chua and Yao. Petitioner contests such
liability, insisting that only those who dealt in the name of the ostensible corporation should be held liable. Since his name
does not appear on any of the contracts and since he never directly transacted with the respondent corporation, ergo, he
cannot be held liable.
Unquestionably, petitioner benefited from the use of the nets found inside F/B Lourdes, the boat which has earlier
been proven to be an asset of the partnership. He in fact questions the attachment of the nets, because the Writ has
effectively stopped his use of the fishing vessel.
It is difficult to disagree with the RTC and the CA that Lim, Chua and Yao decided to form a corporation. Although it
was never legally formed for unknown reasons, this fact alone does not preclude the liabilities of the three as contracting
parties in representation of it. Clearly, under the law on estoppel, those acting on behalf of a corporation and those
benefited by it, knowing it to be without valid existence, are held liable as general partners.
Technically, it is true that petitioner did not directly act on behalf of the corporation. However, having reaped the
benefits of the contract entered into by persons with whom he previously had an existing relationship, he is deemed to be
part of said association and is covered by the scope of the doctrine of corporation by estoppel. We reiterate the ruling of the
Court in Alonso v. Villamor: 19 prLL
"A litigation is not a game of technicalities in which one, more deeply schooled and skilled in the
subtle art of movement and position, entraps and destroys the other. It is, rather, a contest in which each
contending party fully and fairly lays before the court the facts in issue and then, brushing aside as wholly
trivial and indecisive all imperfections of form and technicalities of procedure, asks that justice be done
upon the merits. Lawsuits, unlike duels, are not to be won by a rapier's thrust. Technicality, when it
deserts its proper office as an aid to justice and becomes its great hindrance and chief enemy, deserves
scant consideration from courts. There should be no vested rights in technicalities."
Third Issue:
Validity of Attachment
Finally, petitioner claims that the Writ of Attachment was improperly issued against the nets. We agree with the
Court of Appeals that this issue is now moot and academic. As previously discussed, F/B Lourdes was an asset of the
partnership and that it was placed in the name of petitioner, only to assure payment of the debt he and his partners owed.
The nets and the floats were specifically manufactured and tailor-made according to their own design, and were bought and
used in the fishing venture they agreed upon. Hence, the issuance of the Writ to assure the payment of the price stipulated
in the invoices is proper. Besides, by specific agreement, ownership of the nets remained with Respondent Philippine
Fishing Gear, until full payment thereof.
WHEREFORE, the Petition is DENIED and the assailed Decision AFFIRMED. Costs against petitioner. CdprSO
ORDERED.
|||
(Lim Tong Lim v. Philippine Fishing Gear Industries, Inc., G.R. No. 136448, [November 3, 1999], 376 PHIL 76-95)

[G.R. No. 119020. October 19, 2000.]

INTERNATIONAL EXPRESS TRAVEL & TOUR SERVICES, INC., petitioner, vs. HON. COURT OF
APPEALS, HENRI KAHN, PHILIPPINES FOOTBALL FEDERATION, respondents.

DECISION

KAPUNAN, J p:

On June 30 1989, petitioner International Express Travel and Tour Services, Inc., through its managing director,
wrote a letter to the Philippine Football Federation (Federation), through its president private respondent Henri Kahn,
wherein the former offered its services as a travel agency to the latter. 1 The offer was accepted. AaCEDS
Petitioner secured the airline tickets for the trips of the athletes and officials of the Federation to the South East
Asian Games in Kuala Lumpur as well as various other trips to the People's Republic of China and Brisbane. The total cost
of the tickets amounted to P449,654.83. For the tickets received, the Federation made two partial payments, both in
September of 1989, in the total amount of P176,467.50. 2
On 4 October 1989, petitioner wrote the Federation, through the private respondent a demand letter requesting for
the amount of P265,894.33. 3 On 30 October 1989, the Federation, through the Project Gintong Alay, paid the amount of
P31,603.00. 4
On 27 December 1989, Henri Kahn issued a personal check in the amount of P50,000 as partial payment for the
outstanding balance of the Federation. 5 Thereafter, no further payments were made despite repeated demands.
This prompted petitioner to file a civil case before the Regional Trial Court of Manila. Petitioner sued Henri Kahn in
his personal capacity and as President of the Federation and impleaded the Federation as an alternative defendant.
Petitioner sought to hold Henri Kahn liable for the unpaid balance for the tickets purchased by the Federation on the ground
that Henri Kahn allegedly guaranteed the said obligation. 6
Henri Kahn filed his answer with counterclaim. While not denying the allegation that the Federation owed the
amount P207,524.20, representing the unpaid balance for the plane tickets, he averred that the petitioner has no cause of
action against him either in his personal capacity or in his official capacity as president of the Federation. He maintained that
he did not guarantee payment but merely acted as an agent of the Federation which has a separate and distinct juridical
personality. 7
On the other hand, the Federation failed to file its answer, hence, was declared in default by the trial court. 8
In due course, the trial court rendered judgment and ruled in favor of the petitioner and declared Henri Kahn
personally liable for the unpaid obligation of the Federation. In arriving at the said ruling, the trial court rationalized:
Defendant Henri Kahn would have been correct in his contentions had it been duly established
that defendant Federation is a corporation. The trouble, however, is that neither the plaintiff nor the
defendant Henri Kahn has adduced any evidence proving the corporate existence of the defendant
Federation. In paragraph 2 of its complaint, plaintiff asserted that "defendant Philippine Football
Federation is a sports association . . . ." This has not been denied by defendant Henri Kahn in his
Answer. Being the President of defendant Federation, its corporate existence is within the personal
knowledge of defendant Henri Kahn. He could have easily denied specifically the assertion of the plaintiff
that it is a mere sports association if it were a domestic corporation. But he did not.
xxx xxx xxx
A voluntary unincorporated association, like defendant Federation has no power to enter into, or
to ratify, a contract. The contract entered into by its officers or agents on behalf of such association is not
binding on, or enforceable against it. The officers or agents are themselves personally liable.
xxx xxx xxx 9
The dispositive portion of the trial court's decision reads:
WHEREFORE, judgment is rendered ordering defendant Henri Kahn to pay the plaintiff the
principal sum of P207,524.20, plus the interest thereon at the legal rate computed from July 5, 1990, the
date the complaint was filed, until the principal obligation is fully liquidated; and another sum of
P15,000.00 for attorney's fees. SEHDIC
The complaint of the plaintiff against the Philippine Football Federation and the counterclaims of
the defendant Henri Kahn are hereby dismissed.
With the costs against defendant Henri Kahn. 10
Only Henri Kahn elevated the above decision to the Court of Appeals. On 21 December 1994, the respondent court
rendered a decision reversing the trial court, the decretal portion of said decision reads:
WHEREFORE, premises considered, the judgment appealed from is hereby REVERSED and
SET ASIDE and another one is rendered dismissing the complaint against defendant Henri S. Kahn. 11
In finding for Henri Kahn, the Court of Appeals recognized the juridical existence of the Federation. It rationalized
that since petitioner failed to prove that Henri Kahn guaranteed the obligation of the Federation, he should not be held liable
for the same as said entity has a separate and distinct personality from its officers.
Petitioner filed a motion for reconsideration and as an alternative prayer pleaded that the Federation be held liable
for the unpaid obligation. The same was denied by the appellate court in its resolution of 8 February 1995, where it stated
that:
As to the alternative prayer for the Modification of the Decision by expressly declaring in the
dispositive portion thereof the Philippine Football Federation (PFF) as liable for the unpaid obligation, it
should be remembered that the trial court dismissed the complaint against the Philippine Football
Federation, and the plaintiff did not appeal from this decision. Hence, the Philippine Football Federation is
not a party to this appeal and consequently, no judgment may be pronounced by this Court against the
PFF without violating the due process clause, let alone the fact that the judgment dismissing the
complaint against it, had already become final by virtue of the plaintiff's failure to appeal therefrom. The
alternative prayer is therefore similarly DENIED. 12
Petitioner now seeks recourse to this Court and alleges that the respondent court committed the following assigned
errors: 13
A. THE, HONORABLE COURT OF APPEALS ERRED IN HOLDING THAT PETITIONER HAD
DEALT WITH THE PHILIPPINE FOOTBALL FEDERATION (PFF) AS A CORPORATE
ENTITY AND IN NOT HOLDING THAT PRIVATE RESPONDENT HENRI KAHN WAS
THE ONE, WHO REPRESENTED THE PFF AS HAVING CORPORATE
PERSONALITY.
B. THE HONORABLE COURT OF APPEALS ERRED IN NOT HOLDING PRIVATE
RESPONDENT HENRI KAHN PERSONALLY LIABLE FOR THE OBLIGATION OF THE
UNINCORPORATED PFF, HAVING NEGOTIATED WITH PETITIONER AND
CONTRACTED THE OBLIGATION IN BEHALF OF THE PFF, MADE A PARTIAL
PAYMENT AN ASSURED PETITIONER OF FULLY SETTLING THE OBLIGATION.
C. ASSUMING ARGUENDO THAT PRIVATE RESPONDENT KAHN IS NOT PERSONALLY
LIABLE, THE HONORABLE COURT OF APPEALS ERRED IN NOT EXPRESSLY
DECLARING IN ITS DECISION THAT THE PFF IS SOLELY LIABLE FOR THE
OBLIGATION.
The resolution of the case at bar hinges on the determination of the existence of the Philippine Football Federation
as a juridical person. In the assailed decision, the appellate court recognized the existence of the Federation. In support of
this, the CA cited Republic Act 3135, otherwise known as the Revised Charter of the Philippine Amateur Athletic Federation,
and Presidential Decree No. 604 as the laws from which said Federation derives its existence.
As correctly observed by the appellate court, both R.A. 3135 and P.D. No. 604 recognized the juridical existence of
national sports associations. This may be gleaned from the powers and functions granted to these associations. Section 14
of R.A. 3135 provides:
SEC. 14. Functions, powers and duties of Associations. — The National Sports' Association shall
have the following functions, powers and duties:
1. To adopt a constitution and by-laws for their internal organization and government.
2. To raise funds by donations benefits, and other means for their purposes.
3. To purchase, sell, lease or otherwise encumber property both real and personal, for
the accomplishment of their purpose;
4. To affiliate with international or regional sports' Associations after due consultation
with the executive committee;
xxx xxx xxx
13. To perform such other acts as may be necessary for the proper accomplishment of
their purposes and not inconsistent with this Act.
Section 8 of P.D. 604, grants similar functions to these sports associations:
SEC. 8. Functions, Powers, and Duties of National Sports Association. — The National sports
associations shall have the following functions, powers, and duties:
1. Adopt a Constitution and By-Laws for their internal organization and government
which shall be submitted to the Department and any amendment hereto shall take effect upon
approval by the Department: Provided, however, That no team, school, club, organization or
entity shall be admitted as a voting member of an association unless 60 per cent of the athletes
composing said team, school, club, organization or entity are Filipino citizens.
2. Raise funds by donations, benefits, and other means for their purpose subject to the
approval of the Department;
3. Purchase, sell, lease, or otherwise encumber property, both real and personal, for the
accomplishment of their purpose;
4. Conduct local, interport, and international competitions, other than the Olympic and
Asian Games, for the promotion of their sport;
5. Affiliate with international or regional sports associations after due consultation with
the Department;
xxx xxx xxx
13. Perform such other functions as may be provided by law.
The above powers and functions granted to national sports associations clearly indicate that these entities may
acquire a juridical personality. The power to purchase, sell, lease and encumber property are acts which may only be done
by persons, whether natural or artificial, with juridical capacity. However, while we agree with the appellate court that
national sports associations may be accorded corporate status, such does not automatically take place by the mere
passage of these laws.
It is a basic postulate that before a corporation may acquire juridical personality, the State must give its consent
either in the form of a special law or a general enabling act. We cannot agree with the view of the appellate court and the
private respondent that the Philippine Football Federation came into existence upon the passage of these laws. Nowhere
can it be found in R.A. 3135 or P.D. 604 any provision creating the Philippine Football Federation. These laws merely
recognized the existence of national sports associations and provided the manner by which these entities may acquire
juridical personality. Section 11 of R.A. 3135 provides:
SEC. 11. National Sports' Association; organization and recognition. — A National Association
shall be organized for each individual sports in the Philippines in the manner hereinafter provided to
constitute the Philippine Amateur Athletic Federation. Applications for recognition as a National Sports'
Association shall be filed with the executive committee together with, among others, a copy of the
constitution and by-laws and a list of the members of the proposed association, and a filing fee of ten
pesos.
The Executive Committee shall give the recognition applied for if it is satisfied that said
association will promote the purposes of this Act and particularly section three thereof. No application
shall be held pending for more than three months after the filing thereof without any action having been
taken thereon by the executive committee. Should the application be rejected, the reasons for such
rejection shall be clearly stated in a written communication to the applicant. Failure to specify the reasons
for the rejection shall not affect the application which shall be considered as unacted upon: Provided
however, That until the executive committee herein provided shall have been formed, applications for
recognition shall be passed upon by the duly elected members of the present executive committee of the
Philippine Amateur Athletic Federation. The said executive committee shall be dissolved upon the
organization of the executive committee herein provided: Provided, further, That the functioning executive
committee is charged with the responsibility of seeing to it that the National Sports' Associations are
formed and organized within six months from and after the passage of this Act.
Section 7 of P.D. 604, similarly provides:
SEC. 7. National Sports Associations: — Application for accreditation or recognition as a
national sports association for each individual sport in the Philippines shall be filed with the Department
together with, among others, a copy of the Constitution and By-Laws and a list of the members of the
proposed association.
The Department shall give the recognition applied for if it is satisfied that the national sports
association to be organized will promote the objectives of this Decree and has substantially complied with
the rules and regulations of the Department: Provided, That the Department may withdraw accreditation
or recognition for violation of this Decree and such rules and regulations formulated by it.
The Department shall supervise the national sports association: Provided, That the latter shall
have exclusive technical control over the development and promotion of the particular sport for which
they are organized.
Clearly the above cited provisions require that before an entity may be considered as a national sports association,
such entity must be recognized by the accrediting organization, the Philippine, Amateur Athletic Federation under R.A.
3135, and the Department of Youth and Sports Development under P.D. 604.
This fact of recognition, however, Henri Kahn failed to substantiate. In attempting to prove the juridical existence of
the Federation, Henri Kahn attached to his motion for reconsideration before the trial court a copy of the constitution and by-
laws of the Philippine Football Federation. Unfortunately, the same does not prove that said Federation has indeed been
recognized and accredited by either the Philippine Amateur Athletic Federation or the Department of Youth and Sports
Development. Accordingly, we rule that the Philippine Football Federation is not a national sports association within the
purview of the aforementioned laws and does not have corporate existence of its own. caCTHI
Thus being said, it follows that private respondent Henry Kahn should be held liable for the unpaid obligations of the
unincorporated Philippine Football Federation. It is a settled principle in corporation law that any person acting or purporting
to act on behalf of a corporation which has no valid existence assumes such privileges and becomes personally liable for
contract entered into or for other acts performed as such agent. 14 As president of the Federation, Henri Kahn is presumed
to have known about the corporate existence or non-existence of the Federation. We cannot subscribe to the position taken
by the appellate court that even assuming that the Federation was defectively incorporated, the petitioner cannot deny the
corporate existence of the Federation because it had contracted and dealt with the Federation in such a manner as to
recognize and in effect admit its existence. 15 The doctrine of corporation by estoppel is mistakenly applied by the
respondent court to the petitioner. The application of the doctrine applies to a third party only when he tries to escape
liabilities on a contract from which he has benefited on the irrelevant ground of defective incorporation. 16 In the case at bar,
the petitioner is not trying to escape liability from the contract but rather is the one claiming from the contract.
WHEREFORE, the decision appealed from is REVERSED and SET ASIDE. The decision of the Regional Trial
Court of Manila, Branch 35, in Civil Case No. 90-53595 is hereby REINSTATED. SO ORDERED.
||| (International Express Travel & Tour Services, Inc. v. Court of Appeals, G.R. No. 119020, [October 19, 2000], 397 PHIL
751-762)
[G.R. No. 117188. August 7, 1997.]
LOYOLA GRAND VILLAS HOMEOWNERS (SOUTH) ASSOCIATION, INC., petitioner, vs. HON.
COURT OF APPEALS, HOME INSURANCE AND GUARANTY CORPORATION, EMDEN
ENCARNACION and HORATIO AYCARDO, respondents.

SYLLABUS

1. STATUTORY CONSTRUCTION; STATUTE; INTERPRETATION; THE WORD "MUST" IS NOT ALWAYS


IMPERATIVE. — Ordinarily, the word "must" connotes an imperative act or operates to impose a duty which may be
enforced. It is synonymous with "ought" which connotes compulsion or mandatoriness. However, the word "must" in a
statute, like, "shall", is not always imperative. It may be consistent with an exercise of discretion. In this jurisdiction, the
tendency has been to interpret "shall" as the context or a reasonable construction of the statute in which it is used demands
or requires. This is equally true as regards the word "must". Thus, if the language of a statute considered as a whole and
with due regard to its nature and object reveals that the legislature intended to use the words "shall" and "must" to be
directory, they should be given that meaning. cdt
2. COMMERCIAL LAW; CORPORATION CODE;SEC. 46 (ADOPTION OF BY-LAWS); BY-LAWS; REQUIREMENT
FOR THE ADOPTION THEREOF WITHIN THE PERIOD PROVIDED; NOT MANDATORY. — Taken as a whole and under
the principle that the best interpreter of a statute is the statute itself (optima statuli interpretatix est ipsum statutum). Section
46 of the Corporation Code reveals the legislative intent to attach a directory, and not mandatory, meaning for the word
"must" in the first sentence thereof. Note should be taken of the second paragraph of the law which allows the filing of the
by-laws even prior to incorporation. This provision in the same section of the Code rules out mandatory compliance with the
requirement of filing the by-laws "within one (1) month after receipt of official notice of the issuance of its certificate of
incorporation by the Securities and Exchange Commission". It necessarily follows that failure to file the by-laws within any
period does not imply the "demise" of the corporation. By-laws may be necessary for the "government" of the corporation
but these are subordinate to the articles of incorporation as well as to the Corporation Code and related statutes. There are
in fact cases where by-laws are unnecessary to corporate existence or to the valid exercise of corporate powers, thus: "In
the absence of charter or statutory provisions to the contrary, by-laws are not necessary either to the existence of a
corporation or to the valid exercise of the powers conferred upon it, certainly in all cases where the charter sufficiently
provides for the government of the body; and even where the governing statute in express terms confers upon the
corporation the power to adopt by-laws, the failure to exercise the power will be ascribed to mere nonaction which will not
render void any acts of the corporation which would otherwise be valid." As the "rules and regulations or private laws
enacted by the corporation to regulate, govern and control its own actions, affairs and concerns and its stockholders or
members and directors and officers with relation thereto and among themselves in their relation to it," by-laws are
indispensable to corporations in this jurisdiction. These may not be essential to corporate birth but certainly, these are
required by law for an orderly governance and management of corporations. Nonetheless, failure to file them within the
period required by law by no means tolls the automatic dissolution of a corporation.
3. ID.; ID.; ID.; EFFECT OF FAILURE TO FILE. — Although the Corporation Code requires the filing of by-laws, it
does not expressly provide for the consequences of the non-filing of the same within the period provided for in Section 46.
However, such omission has been rectified by Presidential Decree No. 902-A, the pertinent provisions on the jurisdiction of
the SEC of which state: "SEC. 6. In order to effectively exercise such jurisdiction, the Commission shall possess the
following powers: . . . (1) to suspend, or revoke, after proper notice and hearing, the franchise or certificate of registration of
corporations, partnerships or associations, upon any of the grounds provided by law, including the following: . . . Failure to
file by-laws within the required period; . . . In the exercise of the foregoing authority and jurisdiction of the Commissions or
by a Commissioner or by such there bodies, boards committees and/or any officer as may be created or designated by the
Commission for the purpose. The decision, ruling or order of any such Commissioner, bodies, boards, committees and/or
officer may be appealed to the Commission sitting en banc within thirty (30) days after receipt by the appellant of notice of
such decision, ruling or order. The Commission shall promulgate rules of procedures to govern the proceedings, hearings
and appeals of cases falling within its jurisdiction. The aggrieved party may appeal the order, decision or ruling of the
Commission sitting en banc to the Supreme Court by petition for review in accordance with the pertinent provisions of the
Rules of Court." Even under the foregoing express grant of power and authority, there can be no automatic corporate
dissolution simply because the incorporators failed to abide by the required filing of by-laws embodied in Section 46 of the
Corporation Code. There is no outright "demise" private of corporate existence. Proper notice and hearing are cardinal
components of due process in any democratic institution, agency or society. In other words, the incorporators must be given
the chance to explain their neglect or omission and remedy the same. That the failure to file by-laws is not provided for by
the Corporation Code but in another law is of no moment. P.D. No. 902-A, which took effect immediately after its
promulgation on March 11, 1976, is very much apposite to the Code. Accordingly, the provisions above-quoted supply the
law governing the situation in the case at bar, inasmuch as the Corporation Code and P.D. No. 902-A are statutes in pari
materia. Interpretare et concordare legibus est optimus interpretandi. Every statute must be so construed and harmonized
with other statutes as to form a uniform system of jurisprudence. cdasia

DECISION

ROMERO, J p:

May the failure of a corporation to file its by-laws within one month from the date of its incorporation, as mandated
by Section 46 of the Corporation Code, result in its automatic dissolution?
This is the issue raised in this petition for review on certiorari of the Decision 1 of the Court of Appeals affirming the
decision of the Home Insurance and Guaranty Corporation (HIGC). This quasi-judicial body recognized Loyola Grand Villas
Homeowners Association (LGVHA) as the sole homeowners' association in Loyola Grand Villas, a duly registered
subdivision in Quezon City and Marikina City that was owned and developed by Solid Homes, Inc. It revoked the certificates
of registration issued to Loyola Grand Villas Homeowners (North) Association Incorporated (the North Association for
brevity) and Loyola Grand Villas Homeowners (South) Association Incorporated (the South Association). aisadc
LGVHAI was organized on February 8, 1983 as the association of homeowners and residents of the Loyola Grand
Villas. It was registered with the Home Financing Corporation, the predecessor of herein respondent HIGC, as the sole
homeowners' organization in the said subdivision under Certificate of Registration No. 04-197. It was organized by the
developer of the subdivision and its first president was Victorio V. Soliven, himself the owner of the developer. For unknown
reasons, however, LGVHAI did not file its corporate by-laws.
Sometime in 1988, the officers of the LGVHAI tried to register its by-laws. They failed to do so. 2 'To the officers'
consternation, they discovered that there were two other organizations within the subdivision — the North Association and
the South Association. According to private respondents, a non-resident and Soliven himself, respectively headed these
associations. They also discovered that these associations had five (5) registered homeowners each who were also the
incorporators, directors and officers thereof. None of the members of the LGVHAI was listed as member of the North
Association while three (3) members of LGVHAI were listed as members of the South Association. 3 The North Association
was registered with the HIGC on February 13, 1989 under Certificate of Registration No. 04-1160 covering Phases West II,
East III, West III and East IV. It submitted its by-laws on December 20, 1988.
In July, 1989, when Soliven inquired about the status of LGVHAI, Atty. Joaquin A. Bautista, the head of the legal
department of the HIGC, informed him that LGVHAI had been automatically dissolved for two reasons. First, it did not
submit its by-laws within the period required by the Corporation Code and, second, there was non-user of corporate charter
because HIGC had not received any report on the association's activities. Apparently, this information resulted in the
registration of the South Association with the HIGC on July 27, 1989 covering Phases West I, East I and East II. It filed its
by-laws on July 26, 1989.
These developments prompted the officers of the LGVHAI to lodge a complaint with the HIGC. They questioned the
revocation of LGVHAI's certificate of registration without due notice and hearing and concomitantly prayed for the
cancellation of the certificates of registration of the North and South Associations by reason of the earlier issuance of a
certificate of registration in favor of LGVHAI.
On January 26, 1993, after due notice and hearing, private respondents obtained a favorable ruling from HIGC
Hearing Officer Danilo C. Javier who disposed of HIGC Case No. RRM-5-89 as follows:
"WHEREFORE, judgment is hereby rendered recognizing the Loyola Grand Villas Homeowners
Association, Inc., under Certificate of Registration No. 04-197 as the duly registered and existing
homeowners association for Loyola Grand Villas homeowners, and declaring the Certificates of
Registration of Loyola Grand Villas Homeowners (North) Association, Inc. and Loyola Grand Villas
Homeowners (South) Association, Inc. as hereby revoked or cancelled; that the receivership be
terminated and the Receiver is hereby ordered to render an accounting and turn-over to Loyola Grand
Villas Homeowners Association, Inc., all assets and records of the Association now under his custody
and possession."
The South Association appealed to the Appeals Board of the HIGC. In its Resolution of September 8, 1993, the
Board 4 dismissed the appeal for lack of merit.
Rebuffed, the South Association in turn appealed to the Court of Appeals, raising two issues. First, whether or not
LGVHAI's failure to file its by-laws within the period prescribed by Section 46 of the Corporation Code resulted in the
automatic dissolution of LGVHAI. Second, whether or not two homeowners' associations may be authorized by the HIGC in
one "sprawling subdivision." However, in the Decision of August 23, 1994 being assailed here, the Court of Appeals
affirmed the Resolution of the HIGC Appeals Board.
In resolving the first issue, the Court of Appeals held that under the Corporation Code, a private corporation
commences to have corporate existence and juridical personality from the date the Securities and Exchange Commission
(SEC) issues a certificate of incorporation under its official seal. The requirement for the filing of by-laws under Section 46 of
the Corporation Code within one month from official notice of the issuance of the certificate of incorporation presupposes
that it is already incorporated, although it may file its by-laws with its articles of incorporation. Elucidating on the effect of a
delayed filing of by-laws, the Court of Appeals said:
"We also find nothing in the provisions cited by the petitioner, i.e., Sections 46 and 22,
Corporation Code,or in any other provision of the Code and other laws which provide or at least imply
that failure to file the by-laws results in an automatic dissolution of the corporation. While Section 46, in
prescribing that by-laws must be adopted within the period prescribed therein, may be interpreted as a
mandatory provision, particularly because of the use of the word 'must,' its meaning cannot be stretched
to support the argument that automatic dissolution results from non-compliance.
We realize that Section 46 or other provisions of the Corporation Code are silent on the result of
the failure to adopt and file the by-laws within the required period. Thus, Section 46 and other related
provisions of the Corporation Code are to be construed with Section 6 (1) of P.D. 902-A. This section
empowers the SEC to suspend or revoke certificates of registration on the grounds listed therein. Among
the grounds stated is the failure to file by-laws (see also II Campos: The Corporation Code, 1990 ed., pp.
124-125). Such suspension or revocation, the same section provides, should be made upon proper
notice and hearing. Although P.D. 902-A refers to the SEC, the same principles and procedures apply to
the public respondent HIGC as it exercises its power to revoke or suspend the certificates of registration
or homeowners associations. (Section 2 [a], E.O. 535, series 1979, transferred the powers and
authorities of the SEC over homeowners associations to the HIGC.)
We also do not agree with the petitioner's interpretation that Section 46, Corporation Code
prevails over Section 6, P.D. 902-A and that the latter is invalid because it contravenes the former. There
is no basis for such interpretation considering that these two provisions are not inconsistent with each
other. They are, in fact, complementary to each other so that one cannot be considered as invalidating
the other."
The Court of Appeals added that, as there was no showing that the registration of LGVHAI had been validly
revoked, it continued to be the duly registered homeowners' association in the Loyola Grand Villas. More importantly, the
South Association did not dispute the fact that LGVHAI had been organized and that, thereafter, it transacted business
within the period prescribed by law.
On the second issue, the Court of Appeals reiterated its previous ruling 5 that the HIGC has the authority to order
the holding of a referendum to determine which of two contending associations should represent the entire community,
village or subdivision.
Undaunted, the South Association filed the instant petition for review on certiorari. It elevates as sole issue for
resolution the first issue it had raised before the Court of Appeals, i.e., whether or not the LGVHAI's failure to file its by-laws
within the period prescribed by Section 46 of the Corporation Code had the effect of automatically dissolving the said
corporation.
Petitioner contends that, since Section 46 uses the word "must" with respect to the filing of by-laws, noncompliance
therewith would result in "self-extinction" either due to non-occurrence of a suspensive condition or the occurrence of a
resolutory condition ''under the hypothesis that (by) the issuance of the certificate of registration alone the corporate
personality is deemed already formed." It asserts that the Corporation Code provides for a "gradation of violations of
requirements." Hence, Section 22 mandates that the corporation must be formally organized and should commence
transactions within two years from date of incorporation. Otherwise, the corporation would be deemed dissolved. On the
other hand, if the corporation commences operations but becomes continuously inoperative for five years, then it may be
suspended or its corporate franchise revoked.
Petitioner concedes that Section 46 and the other provisions of the Corporation Code do not provide for sanctions
for non-filing of the by-laws. However, it insists that no sanction need be provided "because the mandatory nature of the
provision is so clear that there can be no doubt about its being an essential attribute of corporate birth." To petitioner, its
submission is buttressed by the facts that the period for compliance is "spelled out distinctly," that the certification of the
SEC/HIGC must show that the by-laws are not inconsistent with the Code, and that a copy of the by-laws "has to be
attached to the articles of incorporation." Moreover, no sanction is provided for because "in the first place, no corporate
identity has been completed." Petitioner asserts that "non-provision for remedy or sanction is itself the tacit proclamation
that non-compliance is fatal and no corporate existence had yet evolved," and therefore, there was "no need to proclaim its
demise." 6 In a bid to convince the Court of its arguments, petitioner stresses that:
". . . the word MUST is used in Sec. 46 in its universal literal meaning and corollary human
implication — its compulsion is integrated in its very essence — MUST is always enforceable by the
inevitable consequence — that is, 'OR ELSE'. The use of the word MUST in Sec. 46 is no exception — it
means file the by-laws within one month after notice of issuance of certificate of registration OR ELSE.
The OR ELSE, though not specified, is inextricably a part of MUST. Do this or if you do not you are
'Kaput'. The importance of the by-laws to corporate existence compels such meaning for as decreed the
by-laws is 'the government' of the corporation. Indeed, how can the corporation do any lawful act as such
without by-laws. Surely, no law is intended to create chaos." 7
Petitioner asserts that P.D. No. 902-A cannot exceed the scope and power of the Corporation Code which itself
does not provide sanctions for non-filing of by-laws. For the petitioner, it is "not proper to assess the true meaning of Sec.
46 . . . on an unauthorized provision on such matter contained in the said decree."
In their comment on the petition, private respondents counter that the requirement of adoption of by-laws is not
mandatory. They point to P.D. No. 902-A as having resolved the issue of whether said requirement is mandatory or merely
directory. Citing Chung Ka Bio v. Intermediate Appellate Court, 8 private respondents contend that Section 6(I) of that
decree provides that non-filing of by-laws is only a ground for suspension or revocation of the certificate of registration of
corporations and, therefore, it may not result in automatic dissolution of the corporation. Moreover, the adoption and filing of
by-laws is a condition subsequent which does not affect the corporate personality of a corporation like the LGVHAI. This is
so because Section 9 of the Corporation Code provides that the corporate existence and juridical personality of a
corporation begins from the date the SEC issues a certificate of incorporation under its official seal. Consequently, even if
the by-laws have not yet been filed, a corporation may be considered a de facto corporation. To emphasize the fact the
LGVHAI was registered as the sole homeowners' association in the Loyola Grand Villas, private respondents point out that
membership in the LGVHAI was an "unconditional restriction in the deeds of sale signed by lot buyers." cdtai
In its reply to private respondents' comment on the petition, petitioner reiterates its argument that the word "must" in
Section 46 of the Corporation Code is mandatory. It adds that, before the ruling in Chung Ka Bio v. Intermediate Appellate
Court could be applied to this case, this Court must first resolve the issue of whether or not the provisions of P.D. No. 902-A
prescribing the rules and regulations to implement the Corporation Code can "rise above and change" the substantive
provisions of the Code.
The pertinent provision of the Corporation Code that is the focal point of controversy in this case states:
"Sec. 46. Adoption of by-laws. — Every corporation formed under this Code, must within one (1)
month after receipt of official notice of the issuance of its certificate of incorporation by the Securities and
Exchange Commission, adopt a code of by-laws for its government not inconsistent with this Code. For
the adoption of by-laws by the corporation, the affirmative vote of the stockholders representing at least a
majority of the outstanding capital stock, or of at least a majority of the members, in the case of non-stock
corporations, shall be necessary. The by-laws shall be signed by the stockholders or members voting for
them and shall be kept in the principal office of the corporation, subject to inspection of the stockholders
or members during office hours; and a copy thereof, shall be filed with the Securities and Exchange
Commission which shall be attached to the original articles of incorporation.
Notwithstanding the provisions of the preceding paragraph, by-laws may be adopted and filed
prior to incorporation; in such case, such by-laws shall be approved and signed by all the incorporators
and submitted to the Securities and Exchange Commission, together with the articles of incorporation.
In all cases, by-laws shall be effective only upon the issuance by the Securities and Exchange
Commission of a certification that the by-laws are not inconsistent with this Code.
The Securities and Exchange Commission shall not accept for filing the by-laws or any
amendment thereto of any bank, banking institution, building and loan association, trust company,
insurance company, public utility, educational institution or other special corporations governed by special
laws, unless accompanied by a certificate of the appropriate government agency to the effect that such
by-laws or amendments are in accordance with law."
As correctly postulated by the petitioner, interpretation of this provision of law begins with the determination of the
meaning and import of the word "must" in this section. Ordinarily, the word "must" connotes an imperative act or operates to
impose a duty which may be enforced. 9 It is synonymous with "ought" which connotes compulsion or mandatoriness. 10
However, the word "must" in a statute, like "shall," is not always imperative. It may be consistent with an exercise of
discretion. In this jurisdiction, the tendency has been to interpret "shall" as the context or a reasonable construction of the
statute in which it is used demands or requires. 11 This is equally true as regards the word "must." Thus, if the language of
a statute considered as a whole and with due regard to its nature and object reveals that the legislature intended to use the
words "shall" and "must" to be directory, they should be given that meaning. 12
In this respect, the following portions of the deliberations of the Batasang Pambansa No. 68 are illuminating:
"MR. FUENTEBELLA. Thank you, Mr. Speaker.
On page 34, referring to the adoption of by-laws, are we made to understand here, Mr. Speaker,
that by-laws must immediately be filed within one month after the issuance? In other words, would this be
mandatory or directory in character?
MR. MENDOZA. This is mandatory.
MR. FUENTEBELLA. It being mandatory, Mr. Speaker, what would be the effect of the failure of
the corporation to file these by- laws within one month?
MR. MENDOZA. There is a provision in the latter part of the Code which identifies and describes
the consequences of violations of any provision of this Code. One such consequence is the dissolution of
the corporation for its inability, or perhaps, incurring certain penalties.
MR. FUENTEBELLA. But it will not automatically amount to a dissolution of the corporation by
merely failing to file the by-laws within one month. Supposing the corporation was late, say, five days,
what would be the mandatory penalty?
MR. MENDOZA. I do not think it will necessarily result in the automatic or ipso facto dissolution
of the corporation. Perhaps, as in the case, as you suggested, in the case of El Hogar Filipino where a
quo warranto action is brought, one takes to account the gravity of the violation committed. If the by-laws
were late — the filing of the by-laws were late by, perhaps, a day or two, I would suppose that might be a
tolerable delay, but if they are delayed over a period of months — as is happening now — because of the
absence of a clear requirement that by-laws must be completed within a specified period of time, the
corporation must suffer certain consequences." 13
This exchange of views demonstrates clearly that automatic corporate dissolution for failure to file the by-laws on
time was never the intention of the legislature. Moreover, even without resorting to the records of deliberations of the
Batasang Pambansa, the law itself provides the answer to the issue propounded by petitioner.
Taken as a whole and under the principle that the best interpreter of a statute is the statute itself (optima statuli
interpretatix est ipsum statutum), 14 Section 46 aforequoted reveals the legislative intent to attach a directory, and not
mandatory, meaning for the word ''must" in the first sentence thereof. Note should be taken of the second paragraph of the
law which allows the filing of the by-laws even prior to incorporation. This provision in the same section of the Code rules
out mandatory compliance with the requirement of filing the by-laws "within one (1) month after receipt of official notice of
the issuance of its certificate of incorporation by the Securities and Exchange Commission." It necessarily follows that
failure to file the by-laws within that period does not imply the "demise" of the corporation. By-laws may be necessary for the
"government" of the corporation but these are subordinate to the articles of incorporation as well as to the Corporation Code
and related statutes. 15 There are in fact cases where by-laws are unnecessary to corporate existence or to the valid
exercise of corporate powers, thus:
"In the absence of charter or statutory provisions to the contrary, by-laws are not necessary
either to the existence of a corporation or to the valid exercise of the powers conferred upon it, certainly
in all cases where the charter sufficiently provides for the government of the body; and even where the
governing statute in express terms confers upon the corporation the power to adopt by-laws, the failure to
exercise the power will be ascribed to mere nonaction which will not render void any acts of the
corporation which would otherwise be valid." 16 (Emphasis supplied.)
As Fletcher aptly puts it:
"It has been said that the by-laws of a corporation are the rule of its life, and that until by-laws
have been adopted the corporation may not be able to act for the purposes of its creation, and that the
first and most important duty of the members is to adopt them. This would seem to follow as a matter of
principle from the office and functions of by-laws. Viewed in this light, the adoption of by-laws is a matter
of practical, if not one of legal, necessity. Moreover, the peculiar circumstances attending the formation of
a corporation may impose the obligation to adopt certain by-laws, as in the case of a close corporation
organized for specific purposes. And the statute or general laws from which the corporation derives its
corporate existence may expressly require it to make and adopt by-laws and specify to some extent what
they shall contain and the manner of their adoption. The mere fact, however, of the existence of power in
the corporation to adopt by-laws does not ordinarily and of necessity make the exercise of such power
essential to its corporate life, or to the validity of any of its acts." 17
Although the Corporation Code requires the filing of by-laws, it does not expressly provide for the consequences of
the non-filing of the same within the period provided for in Section 46. However, such omission has been rectified by
Presidential Decree No. 902-A, the pertinent provisions on the jurisdiction of the SEC of which state:
"SEC. 6. In order to effectively exercise such jurisdiction, the Commission shall possess the
following powers:
xxx xxx xxx
(I) To suspend, or revoke, after proper notice and hearing, the franchise or certificate of
registration of corporations, partnerships or associations, upon any of the grounds provided by law,
including the following:
xxx xxx xxx
5. Failure to file by-laws within the required period;
xxx xxx xxx
In the exercise of the foregoing authority and jurisdiction of the Commission, hearings shall be
conducted by the Commission or by a Commissioner or by such other bodies, boards, committees and/or
any officer as may be created or designated by the Commission for the purpose. The decision, ruling or
order of any such Commissioner, bodies, boards, committees and/or officer may be appealed to the
Commission sitting en banc within thirty (30) days after receipt by the appellant of notice of such decision,
ruling or order. The Commission shall promulgate rules of procedures to govern the proceedings,
hearings and appeals of cases falling within its jurisdiction. cdpr
The aggrieved party may appeal the order, decision or ruling of the Commission sitting en banc
to the Supreme Court by petition for review in accordance with the pertinent provisions of the Rules of
Court."
Even under the foregoing express grant of power and authority, there can be no automatic corporate dissolution
simply because the incorporators failed to abide by the required filing of by-laws embodied in Section 46 of the Corporation
Code. There is no outright "demise" of corporate existence. Proper notice and hearing are cardinal components of due
process in any democratic institution, agency or society. In other words, the incorporators must be given the chance to
explain their neglect or omission and remedy the same.
That the failure to file by-laws is not provided for by the Corporation Code but in another law is of no moment. P.D.
No. 902-A, which took effect immediately after its promulgation on March 11, 1976, is very much apposite to the Code.
Accordingly, the provisions abovequoted supply the law governing the situation in the case at bar, inasmuch as the
Corporation Code and P.D. No. 902-A are statutes in pari materia. Interpretare et concordare legibus est optimus
interpretandi. Every statute must be so construed and harmonized with other statutes as to form a uniform system of
jurisprudence. 18
As the "rules and regulations or private laws enacted by the corporation to regulate, govern and control its own
actions, affairs and concerns and its stockholders or members and directors and officers with relation thereto and among
themselves in their relation to it," 19 by-laws are indispensable to corporations in this jurisdiction. These may not be
essential to corporate birth but certainly, these are required by law for an orderly governance and management of
corporations. Nonetheless, failure to file them within the period required by law by no means tolls the automatic dissolution
of a corporation.
In this regard, private respondents are correct in relying on the pronouncements of this Court in Chung Ka Bio v.
Intermediate Appellate Court, 20 as follows:
". . . Moreover, failure to file the by-laws does not automatically operate to dissolve a corporation
but is now considered only a ground for such dissolution.
Section 19 of the Corporation Law, part of which is now Section 22 of the Corporation Code,
provided that the powers of the corporation would cease if it did not formally organize and commence the
transaction of its business or the continuation of its works within two years from date of its incorporation.
Section 20, which has been reproduced with some modifications in Section 46 of the Corporation Code,
expressly declared that 'every corporation formed under this Act, must within one month after the filing of
the articles of incorporation with the Securities and Exchange Commission, adopt a code of by-laws.'
Whether this provision should be given mandatory or only directory effect remained a controversial
question until it became academic with the adoption of PD 902-A. Under this decree, it is now clear that
the failure to file by-laws within the required period is only a ground for suspension or revocation of the
certificate of registration of corporations.
Non-filing of the by-laws will not result in automatic dissolution of the corporation. Under Section
6(I) of PD 902-A, the SEC is empowered to 'suspend or revoke, after proper notice and hearing, the
franchise or certificate of registration of a corporation' on the ground inter alia of 'failure to file by-laws
within the required period.' It is clear from this provision that there must first of all be a hearing to
determine the existence of the ground, and secondly, assuming such finding, the penalty is not
necessarily revocation but may be only suspension of the charter. In fact, under the rules and regulations
of the SEC, failure to file the by-laws on time may be penalized merely with the imposition of an
administrative fine without affecting the corporate existence of the erring firm.
It should be stressed in this connection that substantial compliance with conditions subsequent
will suffice to perfect corporate personality. Organization and commencement of transaction of corporate
business are but conditions subsequent and not prerequisites for acquisition of corporate personality.
The adoption and filing of by-laws is also a condition subsequent. Under Section 19 of the Corporation
Code, a corporation commences its corporate existence and juridical personality and is deemed
incorporated from the date the Securities and Exchange Commission issues certificate of incorporation
under its official seal. This may be done even before the filing of the by-laws, which under Section 46 of
the Corporation Code, must be adopted 'within one month after receipt of official notice of the issuance of
its certificate of incorporation.'" 21
That the corporation involved herein is under the supervision of the HIGC does not alter the result of this case. The
HIGC has taken over the specialized functions of the former Home Financing Corporation by virtue of Executive Order No.
90 dated December 17, 1986. 22 With respect to homeowners associations, the HIGC shall "exercise all the powers,
authorities and responsibilities that are vested on the Securities and Exchange Commission . . ., the provision of Act 1459,
as amended by P.D. 902-A, to the contrary notwithstanding." 23
WHEREFORE, the instant petition for review on certiorari is hereby DENIED and the questioned Decision of the
Court of Appeals AFFIRMED. This Decision is immediately executory. Costs against petitioner. cdaSO ORDERED.
||| (Loyola Grand Villas Homeowners (South) Association, Inc. v. Court of Appeals, G.R. No. 117188, [August 7, 1997], 342
PHIL 651-669)

[G.R. No. 23241. March 14, 1925.]

HENRY FLEISCHER, plaintiff-appellee, vs. BOTICA NOLASCO CO., INC., defendant-appellant.

Antonio Gonzalez for appellant.


Emilio M. Javier for appellee.

SYLLABUS

1. CORPORATIONS; CORPORATE STOCK; RIGHT OF CORPORATIONS TO IMPOSE A LIMITATION ON


TRANSFERS OF STOCK. — A stock corporation in adopting by-laws governing the transfer of shares of stock should
take into consideration the specific provisions of the Corporation Law. The by-laws of corporations should be made to
harmonize with the provisions of the Corporation Law. By-laws must not be inconsistent with the provisions of the
Corporation Law. By-laws of a corporation are valid if they are reasonable and calculated to carry into effect the objects
of the corporations provided they are not contradictory to the general policy of the laws of the land. Under a statute
authorizing by-laws for the transfer of stock of a corporation, it can do more than prescribe a general mode of transfer
on the corporate books and cannot justify an unreasonable restriction upon the right to sell. The shares of stock of a
corporation are personal property and the holder thereof may transfer the same without unreasonable restrictions.
2. ID.; TRANSFER OF SHARES OF STOCK. — The power to enact by-laws restraining the sale and transfer of
stock must be found in the governing statute or charter. Restrictions upon the traffic in stock must have their source in
legislative enactments, as the corporation itself cannot create such impediments. By-laws of a corporations are intended
merely for the protection of the corporation, and prescribe regulations and not restrictions; they are always subject to
the charter of the corporation. The corporation, in the absence of such a power, cannot ordinarily inquire into or pass
upon the legality of the transaction by which its stock passes from one person to another, nor can it question the
consideration upon which a sale is based. A by-law of a corporation cannot take away or abridge the substantial rights
of stockholders. Courts will carefully scrutinize any attempt in the on a part of a corporation to impose restrictions or
limitations upon the right of stockholders to sell and assign their stock. Restrictions cannot be imposed upon a
stockholder by a by-law without statutory or charter authority. The owner of a corporate stock has the same
uncontrollable right to sell or alienate, which attaches to the ownership of any other species of property.

DECISION

JOHNSON, J p:

This action was commenced in the Court of First Instance of the Province of Oriental Negros on the 14th day of
August, 1923, against the board of directors of the Botica Nolasco, Inc., a corporation duly organized and existing under
the laws of the Philippine Islands. the plaintiff prayed that said board of directors be ordered to register in the books of
the corporation five shares of its stock in the name of Henry Fleischer, the plaintiff, and to pay him the sum of P500 for
damages sustained by him resulting from the refusal of said body to register the share of stock in question. the
defendant filed the demurrer on the ground that the facts alleged in the complaint did not constitute sufficient cause of
action, and that the action was not brought against the proper party, which was the Botica Nolasco, Inc. the demurrer
was sustained, and the plaintiff was granted five days to amend his complaint.
On November 15, 1923, the plaintiff filed an amended complaint against the Botica Nolasco, Inc., alleging that
he became the owner of five shares of stock of said corporation, by purchase from their original owner, one Manuel
Gonzalez; that the said shares were fully paid; and that the defendant refused to register said shares in his name in the
books of the corporation in spite of repeated demands to that effect made by him upon said corporation, which refusal
caused him damages amounting to P500. Plaintiff prayed for a judgment ordering the Botica Nolasco, Inc. to register in
his name in the books of the corporation the five shares of stock recorded in said books in the name of Manuel
Gonzales, and to indemnity him in the sum of P500 as damages, and to pay the costs. The defendant again filed a
demurrer in the ground that the amended complaint did not state facts sufficient to constitute a cause of action, and that
said amended complaint was ambiguous, unintelligence, uncertain, which demurrer was overruled by the court.
The defendant answered the amended complaint denying generally and specifically each and every one of the
material allegations thereof, and, as a special defense, alleged that the defendant, pursuant to article 12 of its by-laws,
had preferential right to buy from the plaintiff said shares at the par value of P100 a share, plus P90 as dividends
corresponding to the year 1922, and that said offer was refused by the plaintiff. The defendant prayed for a judgment
absolving it from all liability under the complaint and directing the plaintiff to deliver to the defendant the five shares of
stock in question, and to pay damages in the sum of P500, and the costs.
Upon the issued presented by the pleadings above stated, the cause was brought in for trial, at the conclusion
of which, and on August 21, 1924, the Honorable N. Capistrano, judge, held that, in his opinion, article 12 of the by-laws
of the corporation which gives it preferential right to buy its shares from retiring stockholders, is in conflict with Act No.
1459 (Corporation Law), especially with section 34 thereof; and rendered a judgment ordering the defendant
corporation, through its board of directors, to register in the books of said corporation the said five shares of stock in the
name of the plaintiff, Henry Fleischer, as the shareholder or owner thereof instead of the original owner, Manuel
Gonzalez, with costs against the defendant.
The defendant appealed from said judgment, and now makes several assignments of error, all of which, in
substance, raise the question whether or not article 12 of the by-laws of the corporation is in conflict with the provisions
of the Corporation Law (Act No. 1459).
There is no controversy as to the facts of the present case. They are simple and may be stated as follows:
That Manuel Gonzalez was the original owner of the five shares of stock in question, No. 16, 17, 18, 19 and 20
of the Botica Nolasco, Inc.; that on March 11, 1923, he assigned and delivered said five shares to the plaintiff, Henry
Fleischer, by accomplishing the form of endorsement provided in the back thereof, together with other credits, in
consideration of a large sum of money owed by Gonzalez to Fleischer (Exhibit A, B, B-1, B-2, B-3, B-4); that on March
13, 1923, Dr. Eduardo Miciano, who was the secretary-treasurer of said corporation, offered to buy from Henry
Fleischer, on behalf of the corporation, said shares of stock, at their par value of P100 a share, for P500; that by virtue
of article 12 of the by-laws of Botica Nolasco, Inc., said corporation had the preferential right to buy from Manuel
Gonzalez said shares (Exhibit 2); that the plaintiff refused to sell them to the defendant; that the plaintiff requested
Doctor Miciano to register said shares in his name; that Doctor Miciano refused to do so, saying that it would be in
contravention of the by-laws of the corporation.
It also appears from the record that on the 13th day of March, 1923, two days after the assignment of th shares
to the plaintiff, Manuel Gonzalez made a written statement to the Botica Nolasco, Inc., requesting that the five shares of
stock sold by him to Henry Fleischer be not transferred to Fleischer's name. He also acknowledged in said written
statement the preferential right of the corporation to buy said five shares (Exhibit 3). On June 14, 1923, Gonzalez wrote
a letter to the Botica Nolasco, withdrawing and cancelling his written statement On March 14, 1923 (Exhibit C), to which
letter the Botica Nolasco , in June 15, 1923, replied, declaring that his written statement was in conformity with the by-
laws of the corporation that his letter of June 14th was of no effect, and that the shares in question had been registered
in the name of the Botica Nolasco, Inc., (Exhibit X).
As indicated above, the important question raised in this appeal is whether or not article of the by-laws of the
Botica Nolasco, Inc., is in conflict with the provisions of the Corporation Law (Act No. 1459). Appellant invoked said
article as its ground for denying the request of th plaintiff that the shares in question be registered in his(plaintiff's)
name, and for claiming that it (Botica Nolasco, Inc.) had the preferential right to buy said shares from Gonzalez.
Appellant now contends that article 12 of the said by-laws is in conformity with the provisions of Act No. 1459. Said
article is as follows:
"ART. 12. Las acciones de la Corporacion peden ser transferidas a otra person, pero para que
estas transferencias tengan validez legal, deben constar en los registros de la Corporacion con el debido
endoso del accionista a cuyo nombre se ha expedido la accion o acciones que se tranfieran, o un
documento de transferencia. Entendiendose que, ningun accionista transferira accion alguna a otra rero.
En igualdad de condiciones, la sociedad tendra el derecho de adquirir par si la accion o acciones que se
traten de transferir." (Exhibit 2.)
The above-quoted article constitutes a by-law or regulation adopted by the Botica Nolasco, Inc., governing the
transfer of shares of stock of said corporation. The latter part of said article creates in favor of the Botica Nolasco, a
preferential right to buy, under the same conditions, the share or shares of stock of a retiring shareholder. Has said
corporation any power, under the Corporation Law (Act No. 1459), to adopt such by-laws?
The particular provisions of the Corporation Law referring to transfer of shares of stock are as follows:

"Sec 13. Every corporation has the power:


xxx xxx xxx
"(7) To make by-laws, not inconsistent with any existing law, for the fixing or changing of the
number of its officers and directors within the limits prescribed by law, of its corporate affairs, etc.
xxx xxx xxx
"Sec 35. The capital stock corporations shall be divided into shares for which certificate signed
by the president or the vice-president, countersigned by the secretary or clerk and sealed of the
corporation, shall be issued in accordance with the by-laws. Share of stock so issued are personal
property and may be transferred by delivery of the certificate indorsed by the owner or his attorney in fact
or other person legally authorized to make transfer. No transfer, however, shall be valid, except as
between the parties, until the transfer is entered and noted upon the books of the corporation so as to
show the names of the parties to the transaction, the date of the transfer, the number of the certificate,
and the number of shares transferred.
"No share of stock against which the corporation holds any unpaid claim shall be transferable on
the books of the corporation."
Section 13, paragraph 7, above-quoted, empowers a corporation to make by-laws, not
inconsistent with any existing law, for the transferring of its stock. It follows from said provision, that a by-
law adopted by a corporation relating to transfer of stock should be in harmony with the law in the subject
of transfer of stock. The law on this subject is found in section 35 of Act No. 1459 above quoted. Said
section 35 specifically provides that the shares of stock "are personal property and may be transferred by
delivery of the certificate indorsed by the owner, etc. Said section 35 defines the nature, character and
transferability of shares of stock. Under said section they are personal property and may be transferred
as therein provided. Said section contemplates no restriction as to whom they may be transferred or sold.
It does not suggest that any discrimination may be created by the corporation in favor or against a certain
purchaser. The holder of shares, as owner of personal property, is at liberty, under said section, to
dispose of them in favor of whomsoever he pleases, without any other limitation in this respect, than the
general provisions of law. Therefore, a stock corporation in adopting a by-law governing transfer of
shares of stock should take into consideration the specific provisions of section 35 of Act No. 1459, and
said by-law should be made to harmonize with said provisions. It should not be inconsistent therewith.
The by-law now in question was adopted under the power conferred upon the corporation by section 13,
paragraph 7, above quoted; but in adopting said by-law the corporation has transcended the limits fixed by law in the
same section, and has not taken into consideration the provisions of section 35 of Act No. 1459.
As general rule, the ly-laws of a corporation are valid if they are reasonable and calculated to carry into effect
the objects of the corporation, and are not contradictory to the general policy of the laws of the land. (Supreme
Commandery of the Knights of the Golden Rule vs. Ainswoth, 71 Ala., 436; 46 Am. Rep., 332.)
On the other hand, it is equally well settled that by-laws of a corporation must be reasonable and for a
corporate purpose, and always within the charter limits. They must always be strictly subordinate to the constitution and
the general laws of the land. They must infringe the policy of the state, nor be hostile to public welfare. (46 Am. Rep.,
332.) They must not disturb vested rights or impair the obligation of a contract, take away or abridge the substantial
rights of stockholder or member, affect rights of property or create obligations unknown to the law. (People's Home
Savings Bank vs. Superior Court, 104 Cal., Co., 649; 43 Am. St. Rep., 147; Ireland vs. Globe Milling Co., 79 Am. St.
Rep., 769.)
The validity of the by-law of a corporation is purely a question of law. (South Florida Railroad Co. vs. Rhodes,
25 Fla., 40.)
"The power to enact by-laws restraining the sale and transfer of stock must be found in the
governing statute or the charter. Restrictions upon the traffic in stock must have their source in legislative
enactment, as the corporation itself cannot create such impediments. By-laws are intended merely for the
protection of the corporation, and prescribed regulation and not restriction; they are always subject to the
charter of the corporation. The corporation, in the absence of such a power, cannot ordinarily inquire into
or pass upon the legality of the transaction by which its stock passes from one person to another, nor can
it question the consideration upon which a sale is based. A by-law cannot take away or abridge the
substantial rights of stockholder. Under a statute authorizing by-laws for the transfer of stock, a
corporation can do no more than prescribe a general mode of transfer on the corporate books and cannot
justify an unreasonable restriction upon the right of sale." (4 Thompson on Corporations, Sec. 4137, p.
674.)
"The right of unrestrained transfer of shares inheres in the very nature of a corporation, and
courts will carefully scrutinize any attempt to impose restrictions or limitations upon the right of
stockholders to sell and assign their stock. The right to impose any restraint in this respect must be
conferred upon the corporation either by the governing statute or by the articles of the corporation. It
cannot be done by a by-law without statutory or charter authority." (4 Thompson on Corporations, sec.
4334, pp. 818, 819.)
"The jus disponendi, being an incident of the ownership of property, the general rule (subject to
exceptions hereafter pointed out and discussed) is that every owner of corporate shares has the same
uncontrollable right to alien them which attaches to the ownership of any other species of property. A
shareholder is under no obligation to refrain from selling his shares at the sacrifice of his personal
interest, in order to secure the welfare of the corporation, or to enable another shareholder to make gains
and profits." (10 Cyc., p. 577.)
"It follows from the foregoing that a corporation has no power to prevent or to restrain transfers of
its shares, unless such power is expressly conferred in its charter or governing statute. This conclusion
follows from the further consideration that by-laws or other regulations restraining such transfers, unless
derived from authority expressly granted by the legislature, would be regarded as impositions in restraint
of trade." (10 Cyc., p. 578.)
The foregoing authorities go farther than the stand we are taking on this question. They hold the power of a
corporation to enact by-laws restraining the sale and transfer of shares, should not only be in harmony with the law or
charter of the corporation, but such power should be expressly granted in said law or charter.
The only restraint imposed by the Corporation Law upon transfer of shares is found in section 35 of Act No.
1459, quoted above, as follows: "No transfer, however, shall be valid, except as between the parties, until the transfer is
entered and noted upon the books of the corporation, the to show the names of the parties to the transaction, the date
of transfer, the number of the certificate, and the number of shares transferred." This restriction is necessary in order
that the officers of the corporation may know who are the stockholders, which is essential in conducting elections of
officers, in calling meetings of stockholders, and for other purposes. But any restriction of the nature of that imposed in
the by-law now in question, is ultra vires, violative of the property rights of shareholders, and in restraint of trade.
And moreover, the by-law now in question cannot have any effect on the appellee. He had no knowledge of
such by-law when the shares were assigned to him. He obtained them in good faith and for a valuable consideration.
He was not a privy to the contract created by said by-law between the shareholder Manuel Gonzalez and the Botica
Nolasco, Inc. Said by-law cannot operate to defeat his rights as a purchaser.
"An unauthorized by-law forbidding a shareholder to sell his shares without first offering them to
the corporation for a period of thirty days is not binding upon an assignee of the stock as a personal
contract, although his assignor knew of the by-law and took part in its adoption." (10 Cyc., 579; Ireland
vs. Globe Milling Co., 21 R. I., 9.)
"When no restriction is placed by public law on the transfer of corporate stock, a purchaser is not
affected by any contractual restriction of which he had no notice." (Brinkerhoff-Farris Trust & Savings Co.
vs. Home Lumber Co., 118 Mo., 447.)
"The assignment of shares of stock in a corporation by one who has assented to an unauthorized
by-law has only the effect of a contract by, and enforceable against, that assignor; the assignee is not
bound by such by-law by virtue of the assignment alone." (Ireland vs. Globe Milling Co., 21 R.I., 9.)
"A by-law of a corporation which provides that transfers of stock shall not be valid unless
approved by the board of directors, while it may be enforced as a reasonable regulation for the protection
of the corporation against worthless stockholders, cannot be made available to defeat the rights of third
persons." (Farmers' & Merchants' Bank of Lineville vs. Wasson, 48 Iowa, 336.)
Counsel for defendant incidentally argues in his brief, that the plaintiff does not have any right of action against
the defendant corporation, but against the president and secretary thereof, inasmuch as the signing and registration of
shares is incumbent upon said officers pursuant to section 35 of the Corporation Law. This contention cannot be
sustained now. The question should have been raised in the lower court. It is too late to raise it now in this appeal.
Besides, as stated above, the corporation was made defendant in this action upon the demurrer of the attorney of the
original defendant in the lower court, who contended that the Botica Nolasco, Inc., should be made the party defendant
in this action. Accordingly, upon order of the court, the complaint was amended and the said corporation was made the
party defendant .

Whenever the corporation refuses to transfer and register stock in case like the present, mandamus will lie to
compel the officers of the corporation to transfer said stock upon the books of the corporation. (26 Cyc., 347; Hager vs.
Bryan, 19 Phil., 138.)
In view of all the foregoing, we are of the opinion, and so hold, that the decision of the lower court is in
accordance with law and should be and is hereby affirmed, with costs. So ordered.
||| (Fleischer v. Botica Nolasco Co., Inc., G.R. No. 23241, [March 14, 1925], 47 PHIL 583-594)

[G.R. No. 26649. July 13, 1927.]


THE GOVERNMENT OF THE PHILIPPINE ISLANDS (on relation of the Attorney-General), plaintiff,
vs. EL HOGAR FILIPINO, defendant.

Attorney-General Jaranilla and Solicitor-General Reyes for plaintiff.


Fisher, DeWitt, Perkins & Brady; Camus, Delgado & Recto and Antonio Sanz for defendant.
Wm. J. Rohde as amicus curiae.

SYLLABUS

1. CORPORATIONS; HOLDING OF REAL PROPERTY FOR PERIOD IN EXCESS OF THAT ALLOWED BY


LAW; FORFEITURE OF FRANCHISE. — The extreme penalty of the forfeiture of its franchise will not be visited upon a
corporation for holding a piece of real property for a period slightly in excess of the time allowed by law, where the
conduct of the corporation does not appear to have been characterized by obduracy or pertinacity in contempt of law.
2. ID.; ID.; DEDUCTION OF PERIOD DURING WHICH CORPORATION IS UNDER CONTRACT TO SELL. —
In estimating the period during which a corporation may be allowed to hold property purchased at its own foreclosure
sale, deduction should be made of any period during which the corporation was under obligation to sell the land to a
particular person by reason of the acceptance by the corporation of his offer to buy, the sale having been made
nugatory by virtue of the failure of the purchaser to carry out the contract.
3. ID.; ID.; FORFEITURE OF FRANCHISE; DISCRETION OF COURT. — In an action of quo warranto the
courts have a discretion with respect to the infliction of, capital punishment upon corporations, and there are certain
misdemeanors and misusers of franchises which are insufficient to justify dissolution.
4. ID.; ID.; ID.; ID.; EFFECT OF SECTION 3 OF ACT NO. 2792. — Section 3 of Act NO. 2792 has not
abrogated the discretion of the courts with respect to the application of the remedy of quo warranto to corporations
which are alleged to have violated the provisions of the Corporation Law (Act No. 1459).
5. CONSTITUTIONAL LAW; TITLE OF ACT NOT EXPRESSING SUBJECT OF BILL.— The title to Act No.
2792 is defective for failure to express the subject-matter of section 3 of said Act, with the result that said section 3 is
invalid for repugnance to constitutional requirement.
6. CORPORATIONS; BUILDING AND LOAN ASSOCIATION; POWER TO ACQUIRE AND HOLD REAL.
PROPERTY; OFFICE BUILDING. — A building and loan association may acquire and hold a lot in the financial district
of the city where it has its principal place of business and may erect thereon a suitable building as the site of its offices.
7. ID.; ID.; ID.; ID.; LEASING OF EXCESS OFFICE SPACE TO PUBLIC. — The circumstance that the building
so erected by the association has office accommodations in excess of its own needs and that such offices are rented to
the public by the association for its benefit and profit does not make the ownership and holding of such office building
an ultra vires act. Having acquired the property under lawful authority, the corporation is entitled to the full beneficial use
thereof.
8. ID.; ID.; POWER OF ASSOCIATION TO ADMINISTER MORTGAGED PROPERTY FOR PURPOSE OF
SATISFYING OBLIGATIONS OF DELINQUENT SHAREHOLDERS. — When the shareholders of a building and loan
association become delinquent in the performance of their obligations, the association may take over the management
of the mortgaged property and administer it for the purpose of applying the income to the obligations of the debtor party,
provided authority so to do is conferred in the contract of mortgage.
9. ID.; I D.; ASSOCIATION WITHOUT POWER TO UNDERTAKE MANAGEMENT OF PROPERTY IN
GENERAL. — A building and loan association has no authority to conduct the business of a real estate agent, as by
managing and administering property not mortgaged to it; and the fact that the owner of such property may have
become a shareholder of the association for the purpose of supposedly qualifying himself to receive such service from
the association does not change the ease.
10. ID.; ID.; INVALID BY-LAW; FORFEITURE OF FRANCHISE. — The circumstance that one of the provisions
contained in the by-laws of a building and loan association is invalid as conflicting with the express provision of statute
is not a misdemeanor on the part of the corporation for which the association can be penalized by the forfeiture of its
charter.
11. ID.; ID.; FAILURE OF SHAREHOLDERS TO ATTEND ANNUAL MEETING. — The circumstance that the
shareholders of a building and loan association do not attend the annual meetings in sufficient number to constitute a
quorum does not render the corporation subject to dissolution.
12. ID.; ID.; FILLING OF VACANCIES IN DIRECTORATE; TERM OF OFFICE OF DIRECTORS. — The
directors of a building and loan association may lawfully fill vacancies occurring in the board of directors in conformity
with a by-law to this effect. Such officials, as well as the original directors, hold until qualification of their successors.
13. ID.; ID.; COMPENSATION OF DIRECTORS. — The power to fix the compensation of the directors of a
building and loan association pertains to the corporation, to be determined in its by-laws; and where the amount of the
compensation to be paid is thus fixed, the court will not concern itself with the question of the propriety and wisdom of
the measure of compensation adopted.
14. ID.; ID.; CONTRACT FOR COMPENSATION OF MANAGER. — Where a building and loan association
makes a contract with its promoter and manager — which contract is expressly ratified in the by-laws of the association,
— by which the association concedes to him, in consideration of valuable services rendered and to be rendered, a right
to receive 5 per centum of the net earnings of the association, this court will not, in a quo warranto proceeding where
there is no allegation that the contract was ultra vires or vitiated by fraud, order the dissolution of the corporation for
entering into such contract, on the mere ground that the compensation granted is excessive; nor will the court enjoin the
association from performing the same.
15. ID.; ID.; BY LAW DEFINING QUALIFICATIONS OF DIRECTORS; BY LAW DISABLING DIRECTORS
FROM RECEIVING LOANS. — The shareholders of a corporation may in the by-laws define the qualifications of
directors and require that shares of a specified value shall be put up as security for their action. A provision in the by-
laws disabling the directors from receiving loans from the association is also valid.
16. ID.; ID.; VALIDITY OF SPECIAL SHARES. — Severino vs. El Hogar Filipino, G. R. NO. 24926, 1 and
related cases followed with respect to validity of special shares issued by respondent association.
17. ID.; ID.; ID.; STATUTORY AUTHORITY FOR PREPAYMENT OF DUES. — Under a statutory provision
authorizing a building and loan association to receive payment of dues in advance, the association is authorized to
issue the two kinds of special shares described in the opinion.
18. ID.; ID.; AUTHORITY OF DIRECTORATE TO ALLOW FOR DEPRECIATION. — The directorate of a
building and loan association has a discretion, in determining the results of the operations of the association for any
year, to write off from the assets a reasonable amount for depreciation, with a view to the determination of the real
profits.
19. ID.; ID.; AUTHORITY OF DIRECTORATE TO MAINTAIN RESERVES. — Under the by-laws of the
respondent building and loan association, the directorate has the power to maintain a general reserve and a special
reserve, whenever in their judgment it is advisable to do so, conformably with the by-laws.
20. ID.; ID.; PURPOSE OF LOAN; HOMEBUILDING. — While the creation of building and loan associations
was intended to serve the beneficent purpose of enabling people to procure homes of their own, and such associations
have been fostered with this end in view, nevertheless the lawmaker in this jurisdiction has not limited the activities of
building and loan associations to the exclusive function of making loans for the building of homes. Home building is only
one of several purposes proposed in the creation of such associations; and a building and loan association cannot be
dissolved in a quo warranto proceeding, on the ground that it has made loans without reference to the purpose for
which the money was intended to be used.
21. ID.; ID.; DISCRETION OF BOARD AS TO SIZE OF LOAN. — The law sets no limit upon the amount of the
loans which may be made to particular persons or entities; and a building and loan association cannot be dissolved on
the ground that some of its loans have been made in large amounts. The matter of the size of the loan is confided to the
discretion of the board of directors.
22. ID., ID., FINAL DISTRIBUTION OF ASSETS. — A by-law of a building and loan association declaring that,
upon the final liquidation of the association, the funds shall be applied to the repayment of shares and the balance, if
any, distributed in the manner established for the distribution of annual profits, is valid.
23. ID., ID.; LOANS TO ARTIFICIAL ENTITIES VALID. — Where the statute says that "any person" may
become a stockholder in a building and loan association, a loan made to an artificial entity, such as a corporation or
partnership, cannot be declared invalid; nor is the admission of such entity to the status of stockholder an ultra vires act,
especially in the absence of any allegation that the particular entity so admitted is prohibited by the law of its own
organization from entering into such contracts.
24. ID., ID.;. SALE OF REAL PROPERTY BY ASSOCIATION. — In making sales of land which has been
bought in by the association at its own foreclosure sales, the association may lawfully sell to a purchaser who obligates
himself to pay in installments. The law does not require such sales to be made for cash; nor does the purchaser have to
be a shareholder of the association.

DECISION

STREET, J p:

This is a quo warranto proceeding instituted originally in this court by the Government of the Philippine Islands
on the relation of the Attorney-General against the building and loan association known as El Hogar Filipino, for the
purpose of depriving it of its corporate franchise, excluding it from all corporate rights and privileges, and effecting a
final dissolution of said corporation. The complaint enumerates seventeen distinct causes of action, to all of which the
defendant has answered upon the merits, first admitting the averments of the first paragraph in the statement of the first
cause of action, wherein it is alleged that the defendant was organized in the year 1911 as a building and loan
association under the laws of the Philippine Islands, and that, since its organization, the corporation has been doing
business in the Philippine Islands, with its principal office in the City of Manila. Other facts alleged in the various causes
of action in the complaint are either denied in the answer or controverted in legal effect by other facts.
After issue had been thus joined upon the merits, the attorneys entered into an elaborate agreement as to the
facts, thereby removing from the field of dispute such matters of fact as are necessary to the solution of the controversy.
It follows that we are here confronted only with the legal questions arising upon the agreed statement.
On March 1, 1906, the Philippine Commission enacted what is known as the Corporation Law (Act No. 1459)
effective upon April 1 of the same year. Sections 171 to 190, inclusive, of this Act are devoted to the subject of building
and loan associations, defining their objects and king various provisions governing their organization and administration,
and providing for the supervision to be exercised over them. These provisions appear to be adopted from American
statutes governing building and loan associations and they of course reflect the ideals and principles found in American
law relative to such associations. The respondent, El Hogar Filipino, was apparently the first corporation organized in
the Philippine Islands under the provisions cited, and the association has been favored with extraordinary success. The
articles of incorporation bear the date of December 28, 1910, at which time capital stock in the association had been
subscribed to the amount of P150,000, of which the sum of P10,620 had been paid in. Under the law as it then stood,
the capital of the association was not permitted to exceed P3,000,000, but by Act No. 2092, passed December 23,
1911, the statute was so amended as to permit the capitalization of building and loan associations to the amount of ten
millions. Soon thereafter the association took advantage of this enactment by amending its articles so as to provide that
the capital should be in an amount not exceeding the then lawful limit. From the time of its first organization the number
of shareholders has constantly increased, with the result that on December 31, 1925, the association had 5,826
shareholders holding 125,750 shares, with a total paid-up value of P8,703,602.25. During the period of its existence
prior to the date last above-mentioned the association paid to withdrawing stockholders the amount of P7,618,257.72;
and in the same period it distributed in the form of dividends among its stockholders the sum of P7,621,565.81.
First cause of action.— The first cause of action is based upon the alleged illegal holding by the respondent of
the title to real property for a period in excess of five years after the property had been bought in by the respondent at
one of its own foreclosure sales. The provision of law relevant to the matter is found in section 75 of Act of Congress of
July 1, 1902 (repeated in subsection 5 of section 13 of the Corporation Law). In both of these provisions it is in
substance declared that while corporations may loan funds upon real estate security and purchase real estate when
necessary for the collection of loans, they shall dispose of real estate so obtained within five years after receiving the
title.
In this connection it appears that in the year 1920 El Hogar Filipino was the holder of a recorded mortgage
upon a tract of land in the municipality of San Clemente, Province of Tarlac, as security for a loan of P24,000 to the
shareholders of El Hogar Filipino who were the owners of said property. The borrowers having defaulted in their
payments, El Hogar Filipino foreclosed the mortgage and purchased the land at the foreclosure sale for the net mount
of the indebtedness, namely, the sum of P23,744.18. The auction sale of the mortgaged property took place November
18, 1920, and the deed conveying the property to El Hogar Filipino was executed and delivered December 22, 1920. On
December 27, 1920, the deed conveying the property to El Hogar Filipino was sent to the register of deeds of the
Province of Tarlac, with the request that the certificate of title then standing in the name of the former owners be
cancelled and that a new certificate of title be issued in the name of El Hogar Filipino. Said deed was received in the
office of the register of deeds of Tarlac on December 28, 1920, together with the old certificate of title and thereupon the
register made upon the said deed the following annotation:
"The foregoing document was received in this office at 4.10 p. m., December 28, 1920, according
to entry 1898, page 50 of Book One of the Day Book and registered on the back of certificate of title No.
2211 and its duplicate, folio 193 of Book A-10 of the register of original certificate. Tarlac, Tarlac, January
12, 1921. (Sgd.) SILVINO LOPEZ DE JESUS, Register of Deeds."
For months no reply was received by El Hogar Filipino from the register of deeds of Tarlac, and letters were
written to him by El Hogar Filipino on the subject in March and April, 1921, requesting action. No answer having been
received to these letters, a complaint was made by El Hogar Filipino to the Chief of the General Land Registration
Office; and on May 7, 1921, the certificate of title to the San Clemente land was received by El Hogar Filipino from the
register of deeds of Tarlac.
On March 10, 1921, the board of directors of El Hogar Filipino adopted a resolution authorizing Vicente
Bengzon, an agent of the corporation, to endeavor to find a buyer for the San Clemente land. On July 27, 1921, El
Hogar Filipino authorized one Jose Laguardia to endeavor to find a purchaser for the San Clemente land for the sum of
P23,000, undertaking to pay the said Laguardia a commission of 5 per centum of the selling price for his services, but
no offers to purchase were obtained through this agent or through the agent Bengzon. In July, 1923, plans of the San
Clemente land were sent to Mr. Luis Gomez, Mr. J. Gonzalez and Mr. Alfonso de Castelvi, as prospective purchasers,
but no offers were received from them. In January, 1926, the agents not having succeeded in finding a buyer, the San
Clemente land was advertised for sale by El Hogar Filipino in El Debate, La Vanguardia and Taliba, three newspapers
of general circulation in the Philippine Islands published in the City of Manila. On March 16, 1926, the first offer for the
purchase of the San Clemente land was received by El Hogar Filipino. This offer was made to it in writing by one
Alcantara, who offered to buy it for the sum of P4,000, Philippine currency, payable P500 in cash, and the remainder
within thirty days. Alcantara's offer having been reported by the manager of El Hogar Filipino to its board of directors, it
was decided, by a resolution adopted at a meeting of the board held on March 25, 1926, to accept the offer, and this
acceptance was communicated to the prospective buyer. Alcantara was given successive extensions of the time, the
last of which expired April 30, 1926, within which to make the payment agreed upon; and upon his failure to do so El
Hogar Filipino treated the contract with him as rescinded, and efforts were made at once to find another buyer. Finally
the land was sold to Dona Felipa Alberto for P6,000 by a public instrument executed before a notary public at Manila, P.
I., on July 30, 1926.
Upon consideration of the facts above set forth it is evident that the strict letter of the law was violated by the
respondent; but it is equally obvious that its conduct has not been characterized by obduracy or pertinacity in contempt
of the law. Moreover, several facts connected with the incident tend to mitigate the offense. The Attorney-General points
out that the respondent acquired title on December 22, 1920, when the deed was executed and delivered, by which the
property was conveyed to it as purchaser at its foreclosure sale, and this title remained in it until July 30, 1926, when
the property was finally sold to Felipa Alberto. The interval between these two conveyances is thus more than five
years; and it is contended that, as a consequence, the respondent has become amenable to dissolution. For the
respondent it is contended that the five-year period did not begin to run against the respondent until May 7, 1921, when
the register of deeds of Tarlac delivered the new certificate of title to the respondent pursuant to the deed by which the
property was acquired. As an equitable consideration affecting the case this contention, though not decisive, is in our
opinion more than respectable. It has been held by this court that a purchaser of land registered under the Torrens
system cannot acquire the status of an innocent purchaser for value unless his vendor is able to place in his hands an
owner's duplicate showing the title of such land to be in the vendor (Director of Lands vs. Addison, 49 Phil., 19;
Rodriguez vs. Llorente, G. R. No. 26615 ). It results that prior to May 7, 1921, El Hogar Filipino was not really in a
position to pass an indefeasible title to any purchaser. In this connection it will be noted that section 75 of the Act of
Congress of July 1, 1902, and the similar provision in section 13 of the Corporation Law, allow the corporation "five
years after receiving the title," within which to dispose of the property. A fair interpretation of these provisions would
seem to indicate that the date of the receiving of the title in this case was the date when the respondent received the
owner's certificate, or May 7, 1921, for it was only after that date that the respondent had an unequivocal and
unquestionable power to pass a complete title. The failure of the respondent to receive the certificate sooner was not
due in any wise to its fault, but to unexplained delay on the part of the register of deeds. For this delay the respondent
cannot be held accountable.
Again, it is urged for the respondent that the period between March 25, 1926, and April 30, 1926, should not be
counted as part of the five-year period. This was the period during which the respondent was under obligation to sell the
property to Alcantara, prior to the rescission of the contract by reason of Alcantara's failure to make the stipulated first
payment. Upon this point the contention of the respondent is, in our opinion, well founded. The acceptance by it of
Alcantara's offer obligated the respondent to Alcantara; and if it had not been for the default of Alcantara, the effective
sale of the property would have resulted. The respondent was not at all chargeable with the collapse of these
negotiations; and hence in any equitable application of the law this period should be deducted from the five-year period
within which the respondent ought to have made the sale. Another circumstance explanatory of the respondent's delay
in selling the property is found in the fact that it purchased the property for the full amount of the indebtedness due to it
from the former owner, which was nearly P24,000. It was subsequently found that the property was not salable for
anything like that amount and in the end it had to be sold for P6,000, notwithstanding energetic efforts on the part of the
respondent to find a purchaser upon better terms.
The question then arises whether the failure of the respondent to get rid of the San Clemente property within
five years after it first acquired the deed thereto, even supposing the five-year period to be properly counted from that
date, is such a violation of law as should work a forfeiture of its franchise and require a judgment to be entered for its
dissolution in this action of quo warranto. Upon this point we do not hesitate to say that in our opinion the corporation
has not been shown to have offended against the law in a manner that should entail a forfeiture of its charter. Certainly
no court with any discretion to use in the matter would visit upon the respondent and its thousands of shareholders the
extreme penalty of the law as a consequence of the delinquency here shown to have been committed.
The law applicable to the case is in our opinion found in section 212 of the Code of Civil Procedure, as applied
by this court in Government of the Philippine Islands vs. Philippine Sugar Estates Development Co. (38 Phil., 15). This
section (212), in prescribing the judgment to be rendered against a corporation in an action of quo warranto, among
other things says:
" . . . When it is found and adjudged that a corporation has offended in any matter or manner
which does not by law work as a surrender or forfeiture, or has misused a franchise or exercised a power
not conferred by law, but not of such a character as to work a surrender or forfeiture of its franchise,
judgment shall be rendered that it be ousted from the continuance of such offense or the exercise of such
power."
This provision clearly shows that the court has a discretion with respect to the infliction of capital punishment
upon corporations and that there are certain misdemeanors and misusers of franchises which should not be recognized
as requiring their dissolution. In Government of the Philippine Islands vs. Philippine Sugar Estates Development Co. (38
Phil., 15), it was found that the offending corporation had been largely (though indirectly) engaged in the buying and
holding of real property for speculative purposes in contravention of its charter and contrary to the express provisions of
law. Moreover, in that case the offending corporation was found to be still interested in the properties so purchased for
speculative purposes at the time the action was brought. Nevertheless, instead! of making an absolute and
unconditional order for the dissolution of the corporation, the judgment of ouster was made conditional upon the failure
of the corporation to discontinue its unlawful conduct within six months after final decision. In the case before us the
respondent appears to have rid itself of the San Clemente property many months prior to the institution of this action. It
is evident from this that the dissolution of the respondent would not be an appropriate remedy in this case. We do not of
course undertake to say that a corporation might not be dissolved for offenses of this nature perpetrated in the past,
especially if its conduct had exhibited a willful obduracy and contempt of law. We content ourselves with holding that
upon the facts here before us the penalty of dissolution would be excessively severe and fraught with consequences
altogether disproportionate to the offense committed.
The evident purpose behind the law restricting the rights of corporations with respect to the tenure of land was
to prevent the revival of the entail (mayorazgo) or other similar institution by which land could be fettered and its
alienation hampered over long periods of time. In the case before us the respondent corporation has in good faith
disposed of the piece of property which appears to have been in its hands at the expiration of the period fixed by law,
and a fair explanation is given of its failure to dispose of it sooner. Under these circumstances the destruction of the
corporation would bring irreparable loss upon the thousands of innocent shareholders of the corporation without any
corresponding benefit to the public. The discretion permitted to this court in the application of the remedy of quo
warranto forbids so radical a use of the remedy.
But the case for the plaintiff supposes that the discretion of this court in matters like that now before us has
been expressly taken away by the third section of Act No. 2792, and that the dissolution of the corporation is obligatory
upon the court upon a mere finding that the respondent has violated the provisions of the Corporation Law in any
respect. This makes it necessary to examine the Act last above-mentioned with some care. Upon referring thereto, we
find that it consists of three sections under the following style:
"No. 2792. — An Act to amend certain sections of the Corporation Law, Act Numbered Fourteen
hundred and fifty-nine, providing for the publication of the assets and liabilities of corporations registering
in the Bureau of Commerce and Industry, determining the liability of the officers of corporations with
regard to the issuance of stock or bonds, establishing penalties for certain things, and for other
purposes."
The first two sections contain amendments to the Corporation Law with respect to matters with which we are
not here concerned. The third section contains a new enactment to be inserted as section 190(A) in the corporation Law
immediately following section 190. This new section reads as follows:
"SEC. 190. (A). Penalties.— The violation of any of the provisions of this Act and its amendments
not otherwise penalized therein, shall be punished by a fine of not more than one thousand pesos, or by
imprisonment for not more than five years, or both, in the discretion of the court. If the violation is
committed by a corporation, the same shall, upon such violation being proved, be dissolved by quo
warranto proceedings instituted by the Attorney-General or by any provincial fiscal, by order of said
Attorney-General: Provided, That nothing in this section provided shall be construed to repeal the other
causes for the dissolution of corporations prescribed by existing law, and the remedy provided for in this
section shall be considered as additional to the remedies already existing."
The contention for the plaintiff is to the effect that the second sentence in this enactment has entirely abrogated
the discretion of this court with respect to the application of the remedy of quo warranto, as expressed in section 212 of
the Code of Civil Procedure, and that it is now mandatory upon us to dissolve any corporation whenever we find that it
has committed any violation of the Corporation Law, however trivial. In our opinion this radical view of the meaning of
the enactment is untenable. When the statute says, "If the violation is committed by a corporation, the same shall, upon
such violation being proved, be dissolved by quo warranto proceedings . . . ," the intention was to indicate that the
remedy against the corporation shall be by action of quo warranto. There as no intention to define the principles
governing said remedy, and it must be understood that in applying the remedy the court is still controlled by the
principles established in immemorial jurisprudence. The interpretation placed upon this language in the brief of the
Attorney- General would be dangerous in the extreme, since it would actually place the life of all corporate investments
in the country within the absolute power of a single Government official. No corporate enterprise of any moment can be
conducted perpetually without some trivial misdemeanor against corporate law being committed by some one or other
of its numerous employees. As illustrations of the preposterous effects of the provision, in the sense con- tended for by
the Attorney-General, the attorneys for the respondent have called attention to the fact that under section 52 of the
Corporation Law, a business corporation is required to keep a stock book and a transfer book in which the names of
stockholders shall be kept in alphabetical order. Again, under section 94, railroad corporations are required to cause all
employees working on passenger trains or at a station for passengers to wear a badge on his cap or hat which will
indicate his office. Can it be supposed that the Legislature intended to penalize the violation of such provisions as these
by dissolution of the corporation involved? Evidently such could not have been the intention; and the only way to avoid
the consequence suggested is to hold, as we now hold, that the provision now under consideration has not impaired the
discretion of this court in applying the writ of quo warranto
Another way to put the same conclusion is to say that the expression "shall be dissolved by quo warranto
proceedings" means in effect, "may be dissolved by quo warranto proceedings in the discretion of the court." The
proposition that the word "shall" may be construed as "may," when addressed by the Legislature to the courts, is well
supported in jurisprudence. In the case of Becker vs. Lebanon and M. St. Ry. Co., (188 Pa., 484), the Supreme Court of
Pennsylvania had under consideration a statute providing as follows:
"It shall be the duty of the court . . . to examine, inquire and ascertain whether such corporation
does in fact possess the right or franchise to do the act from which such alleged injury to private rights or
to the rights and franchises of other corporations results; and if such rights or franchises have not been
conferred upon such corporations, such courts, if exercising equitable power, shall, by injunction, at suit
of the private parties or other corporations, restrain such injurious acts."
In an action based on this statute the plaintiff claimed injunctive relief as a matter of right. But this was denied,
the court saying:
"Notwithstanding, therefore, the use of the imperative 'shall,' the injunction is not to be granted
unless a proper case for injunction be made out, in accordance with the principles and practice of equity.
The word 'shall' when used by the legislature to a court, is usually a grant of authority and means 'may,'
and even if it be intended to be mandatory it must be subject to the necessary limitation that a proper
case has been made out for the exercise of the power."
Other authorities amply sustain this view ( People vs. Nusebaum, 66 N. Y. Supp., 129, 133; West Wisconsin R.
Co. vs. Foley, 94 U. S., 100, 103; 24 Law. Ed., 71; Clancy vs. McElroy, 30 Wash., 567; 70 Pac., 1095; State vs. West, 3
Ohio State, 509, 511; In re Lent, 40 N. Y. Supp., 570, 572; 16 Misc. Rep., 606; Ludlow vs. Ludlow's Executors, 4 N. J.
Law [1 Southard], 387, 394; Whipple vs. Eddy, 161 Ill., 114; 43 N. E., 789, 790; Borkheim vs. Fireman's Fund Ins. Co.,
38 Cal., 505, 506; Beasley vs. People, 89 Ill., 571, 575; Donnelly vs. Smith, 128 Iowa, 257; 103 N. W., 776).
But section 3 of Act No. 2792 is challenged by the respondent on the ground that the subject-matter of this
section is not expressed in the title of the Act, with the result that the section is invalid. This criticism is in our opinion
well founded. Section 3 of our organic law (Jones Bill) declares, among other things, that "No bill which may be enacted
into law shall embrace more than one subject, and that subject shall be expressed in the title of the bill." Any law or part
of a law passed by the Philippine Legislature since this provision went into effect and offending against its requirement
is necessarily void.
Upon examining the entire Act (No. 2792), we find that it is directed to three ends which are successively dealt
with in the first three sections of the Act. But it will be noted that these three matters all relate to the Corporation Law;
and it is at once apparent that they might properly have been embodied in a single Act if a title of sufficient unity and
generality had been prefixed thereto. Furthermore, it is obvious, even upon casual inspection, that the subject-matter of
each of the first two sections is expressed and defined with sufficient precision in the title. With respect to the subject-
matter of section 3 the only words in the title which can be taken to refer to the subject- matter of said section are these,
"An Act . . . establishing penalties for certain things, and for other purposes." These words undoubtedly have sufficient
generality to cover the subject-matter of section 3 of the Act. But this is not enough. The Jones Law requires that the
subject-matter of the bill "shall be expressed in the title of the bill."
When reference is had to the expression "establishing penalties for certain things," it is obvious that these
words express nothing. The constitutional provision was undoubtedly adopted in order that the public might be informed
as to what the Legislature is about while bills are in process of passage. The expression "establishing penalties for
certain things" would give no definite information to anybody as to the project of legislation intended under this
expression. An examination of the decided cases shows that courts have always been indulgent of the practices of the
Legislature with respect to the form and generality of title, for if extreme refinements were indulged by the courts, the
work of legislation would be unnecessarily hampered. But, as has been observed by the California court, there must be
some reasonable limit to the generality of titles that will be allowed. The measure of legality is whether the title is
sufficient to give notice of the general subject of the proposed legislation to the persons and interests likely to be
affected.
In Lewis vs. Dunne (134 Cal., 291), the court had before it a statute entitled "An Act to revise the Code of Civil
Procedure of the State of California, by amending certain sections, repealing others, and adding certain new sections."
This title was held to embrace more than one subject, which were not sufficiently expressed in the title. In discussing the
question the court said:
". . . It is apparent that the language of the title f the act in question, in and of itself, expresses no
subject whatever. No one could tell from the title alone what subject of legislation was dealt with in the
body of the act; such subject, so far as the title of the act informs us, might have been entirely different
from anything to be found in the act itself. . . .
"We cannot agree with the contention of some of respondent's counsel — apparently to some
extent countenanced by a few authorities — that the provision of the constitution in question can be
entirely avoided by the simple device of putting into the title of an act words which denote a subject
'broad' enough to cover everything. Under that view, the title, 'An act concerning the laws of the state,'
would be good, and the convention and people who framed and adopted the constitution would be
convicted of the folly of elaborately constructing a grave constitutional limitation of legislative power upon
a most important subject, which the legislature could at once circumvent by a mere verbal trick. The word
'subject' is used in the constitution in its ordinary sense; and when it says that an act shall embrace but
'one subject,' it necessarily implies what everybody knows — that there are numerous subjects of
legislation, and declares that only one of these subjects shall be embraced in any one act. All subjects
cannot be conjured into one subject by the mere magic of a word in a title. . . ."
In Rader vs. Township of Union (39 N. J. L., 509, 515), the Supreme Court of New Jersey made the following
observation:
". . . It is true, that it may be difficult to indicate, by a formula, how specialized the title of a statute
must be; but it is not difficult to conclude that it must mean something in the way of being a notice of what
is doing. Unless it does this, it can answer no useful end. It is not enough that it embraces the legislative
purpose — it must express it; and where the language is too general, it will accomplish the former, but
not the latter. Thus, a law entitled 'An act for a certain purpose,' would embrace any subject, but would
express none, and, consequently, it would not stand the constitutional test."
The doctrine properly applicable in matters of this kind is, we think, fairly summed up in a current repository of
jurisprudence in the following language:
". . . While it may be difficult to formulate a rule by which to determine the extent to which the title
of a bill must specialize its object, it may be safely assumed that the title must not only embrace the
subject of proposed legislation, but also express it clearly and fully enough to give notice of the legislative
purpose." (25 R. C. L., p. 853.)
In dealing with the problem now before us the words "and for other purposes" found at the end of the caption of
Act No. 2792, must be land completely out of consideration. They express nothing, and amount to nothing as a
compliance with the constitutional requirement to which attention has been directed. This expression ("for other
purposes") is frequently found in the title of acts adopted by the Philippine Legislature; and its presence in our laws is
due to the adoption by our Legislature of the style used in Congressional legislation. But it must be remembered that the
legislation of Congress is subject to no constitutional restriction with respect to the title of bills. Consequently, in
Congressional legislation the words "and for other purposes" at least serve the purpose of admonishing the public that
the bill whose heading contains these words contains legislation upon other subjects than that expressed in the title.
Now, so long as the Philippine Legislature was subject to no restriction with respect to the title of bills intended for
enactment into general laws, the expression "for other purposes" could be appropriately used in titles, not precisely for
the purpose of conveying information as to the matter legislated upon, but for the purpose of admonishing the public
that any bill containing such words in the title might contain other subjects than that expressed in the definitive part of
the title. But, when Congress adopted the Jones Law, the restriction with which we are now dealing became effective
here and the words "for other purposes" could no longer be appropriately used in the title of legislative bills.
Nevertheless, the custom of using these words has still been followed, although they can no longer serve to cover
matter not germane to the bill in the title of which they are used. But the futility of adding these words to the style of any
act is now obvious (Cooley, Const. Lims., 8th ed., p. 302).
In the brief for the plaintiff it is intimated that the constitutional restriction which we have been discussing is
more or less of a dead letter in this jurisdiction; and it seems to be taken for granted that no court would ever presume
to hold a legislative act or part of a legislative act invalid for non-compliance with the requirement. This is a mistake; and
no utterance of this court can be cited as giving currency to any such notion. On the contrary the discussion contained
in Central Capiz vs. Ramirez (40 Phil., 883), shows that when a case arises where a violation of the restriction is
apparent, the court has no alternative but to declare the legislation affected thereby to be invalid.
Second cause of action. — The second cause of action is based upon a charge that the respondent is owning
and holding a business lot, with the structure thereon, in the financial district of the City of Manila in excess of its
reasonable requirements and in contravention of subsection 5 of section 13 of the Corporation Law. The facts on which
this charge is based appear to be these:
On August 28, 1913, the respondent purchased 1,413 square meters of land at the corner of Juan Luna Street
and the Muelle de la Industria, in the City of Manila, immediately adjacent to the building then occupied by the
Hongkong and Shanghai Banking Corporation. At the time the respondent acquired this lot there stood upon it a
building, then nearly fifty years old, which was occupied in part by the offices of an importing firm and in part by
warehouses of the same firm. The material used in the construction was Guadalupe stone and hewn timber, and the
building contained none of the facilities usually found in a modern office building.
In pursuance of a design which had been formed prior to the purchase of the property, the directors of the El
Hogar Filipino caused the old building to be demolished; and they erected thereon a modern reinforced concrete office
building. As at first constructed the new building was three stories high in the main, but in 1920, in order to obtain
greater advantage from the use of the land, an additional story was added to the building, making a structure of four
stories except in one corner where an additional story was placed, making it five stories high over an area of 117.52
square meters. It is admitted in the plaintiff's brief that this "noble and imposing structure" — to use the words of the
Attorney-General — "has greatly improved the aspect of the banking and commercial district of Manila and has greatly
contributed to the movement and campaign for the Manila Beautiful." It is also admitted that the completed building is
reasonably proportionate in value and revenue producing capacity to the value of the land upon which it stands. The
total outlay of the respondent for the land and the improvements thereon was P690,000 and at this valuation the
property is carried on, the books of the company, while the assessed valuation of the land and improvements is at
P786,478.
Since the new building was completed the respondent has used about 324 square meters of floor space for its
own offices and has rented the remainder of the office space in said building, consisting of about 3,175 square meters,
to other persons and entities. In the second cause of action of the complaint it is supposed that the acquisition of this
lot, the construction of the new office building thereon, and the subsequent renting of the same in great part to third
persons, are ultra vires acts on the part of the corporation, and that the proper penalty to be enforced against it in this
action is that of dissolution.
With this contention we are unable to agree. Under subsection 5 of section 13 of the Corporation Law, every
corporation has the power to purchase, hold and lease such real property as the transaction of the lawful business of
the corporation may reasonably and necessarily require. When this property was acquired in 1916, the business of El
Hogar Filipino had developed to such an extent, and its prospects for the future were such as to justify its directors in
acquiring a lot in the financial district of the City of Manila and in constructing thereon a suitable building as the site of its
offices; and it cannot be fairly said that the area of the lot — 1,413 square meters — was in excess of its reasonable
requirements. The law expressly declares that corporations may acquire such real estate as is reasonably necessary to
enable them to carry out the purposes for which they were created; and we are of the opinion that the owning of a
business lot upon which to construct and maintain its offices is reasonably necessary to a building and loan association
such as the respondent was at the time this property was acquired. A different ruling on this point would compel
important enterprises to conduct their business exclusively in leased offices — a result which could serve no useful end
but would retard industrial growth and be inimical to the best interests of society.
We are furthermore of the opinion that, inasmuch as the lot referred to was lawfully acquired by the respondent,
it is entitled to the full beneficial use thereof. No legitimate principle can be discovered which would deny to one owner
the right to enjoy his (or its) property to the same extent that is conceded to any other owner; and an intention to
discriminate between owners in this respect is not lightly to be imputed to the Legislature. The point here involved has
been the subject of consideration in many decisions of American courts under statutes even more restrictive than that
which prevails in this jurisdiction; and the conclusion has uniformly been that a corporation whose business may
properly be conducted in a populous center may acquire an appropriate lot and construct thereon an edifice with
facilities in excess of its own immediate requirements.
Thus in People vs. Pullman's Palace-Car Co. (175 Ill., 125; 64 L. R. A., 366), it appeared that the respondent
corporation owned and controlled a large ten-story business block in the City of Chicago, worth $2,000,000, and that it
occupied only about one-fourth thereof for its own purposes, leasing the remainder to others at heavy rentals. The
corporate charter merely permitted the holding of such real estate by the respondent as might be necessary for the
successful prosecution of its business. An attempt was made to obtain the dissolution of the corporation in a quo
warranto proceeding similar to that now before us, but the remedy was denied.
In Rector vs. Hartford Deposit Co. (190 Ill., 380; 60 N. E., 528), a question was raised as to the power of the
Deposit Company to erect and own a fourteen-story building — containing eight storerooms, one hundred suites of
offices, and one safety deposit vault, under a statute authorizing the corporation to possess so much real estate "as
shall be necessary for the transaction of their business." The court said:
"That the appellee company possessed ample power to acquire real property and construct a
building thereon for the purpose of transacting therein the legitimate business of the corporation is
beyond the range of debate. Nor is the contrary contented, but the insistence is that, under the guise of
erecting a building for corporate purposes, the appellee company purposely constructed a much larger
building than its business required, containing many rooms intended to be rented to others for offices and
business purposes, — among them, the basement rooms contracted to be leased to the appellant, —
and that in so doing it designedly exceeded its corporate powers. The position of appellant, therefore, is
that the appellee corporation has flagrantly abused its general power to acquire real estate and construct
a building thereon . . . It was within the general scope of the express powers of the appellee corporation
to own and possess a building necessary for its proper corporate purposes. In planning and constructing
such a building, as was said in People vs. Pullman's Palace Car Co., supra, the corporation should not
necessarily be restricted to a building containing the precise number of rooms its then business might
require, and no more, but that the future probable growth and volume of its business might be considered
and anticipated, and a larger building, and one containing more rooms than the present volume of
business required be erected, and the rooms not needed might be rented by the corporation, — provided,
of course, such course should be taken in good faith, and not as a mere evasion of the public law and the
policy of the state relative to the ownership of real estate by corporations. In such state of case the
question is whether the corporation has abused or excessively and unjustifiably used the power and
authority granted it by the state to construct buildings and own real estate necessary for its corporate
purposes."
In Home Savings Building Association vs. Driver (129 Ky., 754), one of the questions before the court was
precisely the same as that now before us. Upon this point the Supreme Court of Kentucky said:
"The third question is, has the association the right to erect, remodel, or own a building of more
than sufficient capacity to accommodate its own business and to rent out the excess? There is nothing in
the Constitution, charter of the association, or statutes placing any limitation upon the character of a
building which a corporation may erect as a home in which to conduct its business. A corporation
conducting a business of the character of that in which appellant is engaged naturally expects its
business to grow and expand from time to time, and, in building a home, it would be exercising but a
short-sighted judgment if it did not make provision for the future by building a home large enough to take
care of its expanding business, and hence, even if it should build a house larger and roomier than its
present needs or interests require, it would be acting clearly within the exercise of its corporate right and
power. The limitation which the statute imposes is that it shall not own more real estate than is necessary
for the proper conduct of its business, but it does not attempt to place any restriction or limitation upon
the right of the corporation or association as to the character of building it shall erect on said real estate;
and, while the Constitution and the statutes provide that no corporation shall engage in any business
other than that expressly authorized by its charter, we are of opinion that, in renting out the unoccupied
and unused portions of the building so erected, the association could not be said to be engaged in any
other business than that authorized by its charter. The renting of the unused portions of the building is a
mere incident in the conduct of its real business. We would not say that a building association might
embark in the business of building houses and renting or leasing them, but there is quite a difference in
building or renting a house in which to conduct its own business and leasing the unused portion thereof
for the time being, or until such time as they may be needed by the association, and in building houses
for the purpose of renting or leasing them. The one might properly be said to be the proper exercise of a
power incident to the conduct of its legitimate business, whereas the other would be a clear violation of
that provision of the statute which denies to any corporation the right to conduct any business other than
that authorized by its charter. To hold otherwise would be to charge most of the banking institutions, trust
companies and other corporations, such as title guaranty companies, etc., doing business in the state,
and especially in the large cities, with violating the law; for it is well known that there are few of such
institutions that do not, at times, rent out or lease the unneeded portions of the building occupied by them
as homes. We do not think that in so doing they are violating any provisions of the law, but that the
renting out of the unused or unoccupied portions of their buildings is but an incident in the conduct of
their business."
In Wingert vs. First National Bank of Hagerstown, Md. (175 Fed., 739, 741), a stockholder sought to enjoin the
bank from building a six-story building on a lot then owned by the bank in the commercial district of Hagerstown of
which only the first story was to be used by the bank, the remaining stories to be rented out for offices and places of
business, on the theory that such action was ultra vires and in violation of the provisions of the national banking act
confining such corporations to the holding, only, of such real estate "as shall be necessary for its immediate
accommodation in the transaction of its business.
The injunction was denied, the court adopting the opinion of the lower court in which the following was said:
" 'The other ground urged by the complainant is that the proposed action is violative of the
restriction which permits a national bank to hold only such real estate as shall be necessary for its
immediate accommodation in the transaction of its business, and that, therefore, the erection of a
building which will contain offices not necessary for the business of the bank is not permitted by the law,
although that method of improving the lot may be the most beneficial use that can be made of it. It is
matter of common knowledge that the actual practice of national banks is to the contrary. Where ground
is valuable, it may probably be truly said that the majority of national bank buildings are built with
accommodations in excess of the needs of the bank for the purpose of lessening the bank's expense by
renting out the unused portion. If that were not allowable, many smaller banks in cities would be driven to
become tenants as the great cost of the lot would be prohibitive of using it exclusively for the banking
accommodation of a single bank. As indicative of the interpretation of the how commonly received and
acted upon, reference may be made to the reply of the Comptroller of the Currency to the inquiry by the
bank in this case asking whether the law forbids the bank constructing such a building as was
contemplated.
" 'The reply was as follows: "Your letter of the 9th instant received, stating that the directors
contemplate making improvements in the bank building and inquiring if there is anything in the national
banking laws prohibiting the construction of a building which will contain floors for offices to be rented out
by the bank as well as the banking room. Your attention is called to the case of Brown vs. Schleier, 118
Fed., 981 [55 C. C. A., 475], in which the court held that: 'If the land which a national bank purchases or
leases for the accommodation of its business is very valuable it may exercise the same rights that belong
to other landowners of improving it in a way that will yield the largest income, lessen its own rent, and
render that part of its funds which are invested in realty most productive.' " This seems to be the common
sense interpretation of the act of Congress and is the one which prevails.' "
It would. seem to be unnecessary to extend the opinion by lengthy citations upon the point under consideration,
but Brown vs. Schleier (118 Fed., 981), may be cited as being in harmony with the foregoing authorities. In ealing with
the powers of a national bank the court, in this case, said:
"When an occasion arises for an investment in real property for either of the purposes specified
in the statute, the national bank act permits banking associations to act as any prudent person would act
in making an investment in real estate, and to exercise the same measure of judgment and discretion.
The act ought not to be construed in such a way as to compel a national bank, when it acquires real
property for a legitimate purpose, to deal with it otherwise than a prudent landowner would ordinarily deal
with such property."
In the brief of the Attorney-General reliance is placed almost entirely upon two Illinois cases, namely, Africani
Home Purchase and Loan Association vs. Carroll (267 Ill., 380), and First Methodist Episcopal Church of Chicago vs.
Dixon (178 Ill., 260). In our opinion these cases are either distinguishable from that now before us, or they reflect a view
of the law which is incorrect. At any rate the weight of judicial opinion is so overwhelmingly in favor of sustaining the
validity of the acts alleged in the second cause of action to have been done by the respondent in excess of its powers
that we refrain from commenting at any length upon said cases. The ground stated in the second cause of action is in
our opinion without merit.
Third cause of action. — Under the third cause of action the respondent is charged with engaging in activities
foreign to the purposes for which the corporation was created and not reasonably necessary to its legitimate ends. The
specifications under this cause of action relate to three different sorts of activities. The first consists of the administration
of the offices in the El Hogar building not sed by the respondent itself and the renting of such offices to the public. As
stated in the discussion connected with the second cause of action, the respondent uses only about ten per cent of the
office space in the El Hogar building for its own purposes, and it leases the remainder to strangers. In the years 1924
and 1925 the respondent received as rent for the leased portions of the building the sums of P75,395.06 and
P58,259.27, respectively. The activities here criticised clearly fall within the legitimate powers of the respondent, as
shown in what we have said above relative to the second cause of action. This matter will therefore no longer detain us.
If the respondent had the power to acquire the lot, construct the edifice and hold it beneficially, as there decided, the
beneficial administration by it of such parts of the building as are let to others must necessarily be lawful.
The second specification under the third cause of action has reference to the administration and management
of properties belonging to delinquent shareholders of the association. In this connection it appears that in case of
delinquency on the part of its shareholders in the payment of interest, premiums, and dues, the association has been
accustomed (pursuant to clause 8 of its standard mortgage) to take over and manage the mortgaged property for the
purpose of applying the income to the obligations of the debtor party. For these services the respondent charges a
commission at the rate of 2 1/2 per centum on sums collected. The case for the Government supposes that the only
remedy which the respondent has in case of default on the part of its shareholders is to proceed to enforce collection of
the whole loan in the manner contemplated in section 185 of the Corporation Law. It will be noted, however, that,
according to said section, the association may treat the whole indebtedness as due, "at the option of the board of
directors," and this remedy is not made exclusive. We see no reason to doubt the validity of the clause giving the
association the right to take over the property which constitutes the security for the delinquent debt and to manage it
with a view to the satisfaction of the obligations due to the association. Such course is certainly more favorable to the
debtor than the immediate enforcement of the entire obligation, and the validity of the clause allowing this course to be
taken appears to us to be not open to doubt. The second specification under this cause of action is therefore without
merit, as was true of the first.
The third specification under this cause of action relates to certain activities which are described in the following
paragraphs contained in the agreed statement of facts:
"El Hogar Filipino has undertaken the management of some parcels of improved real estate
situated in Manila not under mortgage to it, but owned by shareholders, and has held itself out by
advertisement as prepared to do so. The number of properties so managed during the years 1921 to
1925, inclusive, was as follows:

1921 eight properties

1922 six properties

1923 ten properties

1924 fourteen properties

1925 fourteen properties

"This service is limited to shareholders; but some of the persons whose properties are so
managed for them became shareholders only to enable them to take advantage thereof.
"The services rendered in the management of such improved real estate by El Hogar Filipino
consist in the renting of the same, the payment of real estate taxes and insurance for the account of the
owner, causing the necessary repairs for upkeep to be made, and collecting rents due from tenants. For
the services so rendered in the management of such properties El Hogar Filipino receives compensation
in the form of commissions upon the gross receipts from such properties at rates varying from two and
one-half per centum to five per centum of the sums so collected, according to the location of the property
and the effort involved in its management.
"The work of managing real estate belonging to non-borrowing shareholders administered by El
Hogar Filipino is carried on by the same members of the staff who attend to the details of the
management of properties administered by the manager of El Hogar Filipino under the provisions of
paragraph 8 of the standard mortgage form, and of properties bought in on foreclosure of mortgage."
The practice described in the passage above quoted from the agreed facts is in our opinion unauthorized by
law. Such was the view taken by the bank examiner of the Treasury Bureau in his report to the Insular Treasurer on
December 21, 1925, wherein the practice in question was criticised. The administration of property in the manner
described is more befitting to the business of a real estate agent or trust company than to the business of a building and
loan association. The practice to which this criticism is directed relates of course solely to the management and
administration of properties which are not mortgaged to the association. The circumstance that the owner of the
property may have been required to subscribe to one or more shares of the association with a view to qualifying him to
receive this service is of no significance. It is a general rule of law that corporations possess only such express powers
as are actually conferred and such implied powers as are reasonably necessary to the exercise of the express powers.
The management and administration of the property of the shareholders of the corporation is not expressly authorized
by law, and we are unable to see that, upon any fair construction of the law, these activities are necessary to the
exercise of any of the granted powers. The corporation, upon the point now under criticism, has clearly extended itself
beyond the legitimate range of its powers. But it does not result that the dissolution of the corporation is in order, and it
will merely be enjoined from further activities of this sort.
Fourth cause of action.— It appears that among the by-laws of the association there is an article (No. 10) which
reads as follows:
"The board of directors of the association, by the vote of an absolute majority of its members, is
empowered to cancel shares and to return to the owner thereof the balance resulting from the liquidation
thereof whenever, by reason of their conduct, or for any other motive, the continuation as members of the
owners of such shares is not desirable."
This by-law is of course a patent nullity, since it is in direct conflict with the latter part of section 187 of the
Corporation Law, which expressly declares that the board of directors shall not have the power to force the surrender
and withdrawal of unmatured stock except in case of liquidation of the corporation or of forfeiture of the stock for
delinquency. It is agreed that this provision of the by-laws has never been enforced, and in fact no attempt has ever
been made by the board of directors to make use of the power therein conferred. In November, 1923, the Acting Insular
Treasurer addressed a letter to El Hogar Filipino, calling attention to article 10 of its by-laws and expressing the view
that said article was invalid. It was therefore suggested that the article in question should be eliminated from the by-
laws. At the next meeting of the board of directors the matter was called to their attention and it was resolved to
recommend to the shareholders that in their next annual meeting the article in question be abrogated. It appears,
however, that no annual meeting of the shareholders called since that date has been attended by a sufficient number of
shareholders to constitute a quorum, with the result that the provision referred to has not been eliminated from the by-
laws, and it still stands among the bylaws of the association, notwithstanding its patent conflict with the law.
It is supposed, in the fourth cause of action, that the existence of this article among the bylaws of the
association is a misdemeanor on the part of the respondent which justifies its dissolution. In this view we are unable to
concur. The obnoxious by-law, as it stands, is a mere nullity, and could not be enforced even if the directors were to
attempt to do so. There is no provision of law making it a misdemeanor to incorporate an invalid provision in the by-laws
of a corporation; and if there were such, the hazards incident to corporate effort would certainly be largely increased.
There is no merit in this cause of action.
Fifth cause of action.— In section 31 of the Corporation Law it is declared that, "at all elections of directors
there must be present, either in person or by representative authorized to act by written proxy, the owners of the
majority of the subscribed capital stock entitled to vote, . . .." Conformably with this requirement it is declared in article
61 of the by-laws of El Hogar Filipino that, "the attendance in person or by proxy of shareholders owning one-half plus
one of the shareholders shall be necessary to constitute a quorum for the election of directors. At the general annual
meetings of the El Hogar Filipino held in the years 1911 and 1912, there was a quorum of shares present or
represented at the meetings and directors were duly elected accordingly. As the corporation has grown, however, it has
been found increasingly difficult to get together a quorum of the shareholders, or their proxies, at the annual meetings;
and with the exception of the annual meeting held in 1917, when a new directorate was elected, the meetings have
failed for lack of quorum. It has been foreseen by the officials in charge of the respondent that this condition of affairs
would lead to embarrassment, and a special effort was made by the management to induce a sufficient number of
shareholders to attend the annual meeting for February, 1923. In addition to the publication of notices in the
newspapers, as required by the by-laws, a letter of notification was sent to every shareholder at his last known address,
together with a blank form of proxy to be used in the event the shareholder could not personally attend the meeting.
Notwithstanding these special efforts the meeting was attended only by shareholders, in person and by proxy,
representing 3,889 shares, out of a total of 106,491 shares then outstanding and entitled to vote.
Owing to the failure of a quorum at most of the general meetings since the respondent has been in existence, it
has been the practice of the directors to fill vacancies in the directorate by choosing suitable persons from among the
stockholders. This custom finds its sanction in article 71 of the by-laws, which reads as follows:
"ART. 71. The directors shall elect from among the share-holders members to fill the vacancies
that may occur in the board of directors until the election at the general meeting."
The persons thus chosen to fill vacancies in the directorate have, it is admitted, uniformly been experienced
and successful business and professional men of means, enjoying earned incomes of from P12,000 to P50,000 per
annum, with an annual average of P30,000 in addition to such income as they derive from their properties. Moreover, it
appears that several of the individuals constituting the original directorate and persons chosen to supply vacancies
therein belong to prominent Filipino families, and that they are more or less related to each other by blood or marriage.
In addition to this it appears that it has been the policy of the directorate to keep thereon some member or another of a
single prominent American law firm in the City.
It is supposed in the statement of the fifth cause of action in the complaint that the failure of the corporation to
hold annual meetings and the filling of vacancies in the directorate in the manner described constitute misdemeanors on
the part of the respondent which justify the resumption of the franchise by the Government and dissolution of the
corporation; and in this connection it is charged that the board of directors of the respondent has become a permanent
and self perpetuating body composed of wealthy men instead of wage earners and persons of moderate means. We
are unable to see the slightest merit in the charge. No fault can be imputed to the corporation on account of the failure
of the shareholders to attend the annual meetings; and their non-attendance at such meetings is doubtless to be
interpreted in part as expressing their satisfaction at the way in which things have been conducted. Upon failure of a
quorum at any annual meeting the directorate naturally holds over and continues to function until another directorate is
chosen and qualified. Unless the law or the charter of a corporation expressly provides that an office shall become
vacant at the expiration of the term of office for which the officer was elected, the general rule is to allow the officer to
hold over until his successor is duly qualified. Mere failure of a corporation to elect officers does not terminate the terms
of existing officers nor dissolve the corporation (Quitman Oil Company vs. Peacock, 14 Ga. App., 550; Jenkins vs.
Baxter, 160 Pa. State, 199; New York B. & E. Ry. Co. vs. Motil, 81 Conn., 466; Hatch vs. Lucky Bill Mining Company,
71 Pac., 865; Youree vs. Home Town Mutual Ins. Company, 180 Missouri, 153; Cassell vs. Lexington, H. & P. Turnpike
Road Co., 10 Ky. L. R., 486). The doctrine above stated finds expression in article 66 of the by-laws of the respondent
which declares in so many words that directors shall hold office "for the term of one year or until their successors shall
have been elected and taken possession of their offices."
It results that the practice of the directorate of filling vacancies by the action of the directors themselves is valid.
Nor can any exception be taken to the personality of the individuals chosen by the directors to fill vacancies in the body.
Certainly it is no fair criticism to say that they have chosen competent businessmen of financial responsibility instead of
electing poor persons to so responsible a position. The possession of means does not disqualify a man for filling
positions of responsibility in corporate affairs.
Sixth cause of action. — Under the sixth cause of action it is alleged that the directors of El Hogar Filipino,
instead of serving without pay, or receiving nominal pay or a fixed salary, — as the complaint supposes would be
proper, — have been receiving large compensation, varying in amount from time to time, out of the profits of the
respondent. The facts relating to this cause of action are in substance these:
Under section 92 of the by-laws of El Hogar Filipino 5 per centum of the net profit shown by the annual balance
sheet is distributed to the directors in proportion to their attendance at meetings of the board. The compensation paid to
the directors from time to time since the organization was organized in 1910 to the end of the year 1925, together with
the number of meetings of the board held each year, is exhibited in the following table:

Year Compensation Number of

paid directors meetings Rate per meet-

as a whole held ing as a whole


1911 P4,167.96 25 P166.71

1912 10,511.87 29 362.47

1913 15,479.29 27 573.30

1914 19,164.72 27 709.80

1915 24 032.85 25 961.31

1916 27 539.50 28 983.55

1917 31,327.00 26 1,204.88

1918 32,858.35 20 1,642.91

1919 36,318.78 21 1,729.46

1920 63,517.01 28 2,268.46

1921 36,815.33 25 1,472.61

1922 43,133.73 25 1,725.34

1923 39,773.61 27 1,473.09

1924 38,651.92 26 1,486.61

1925 35.719.27 26 1.373.81

It will be noted that the compensation above indicated as accruing to the directorate as a whole has been
divided among the members actually present at the different meetings. As a result of this practice, and the liberal
measure of compensation adopted, we find that the attendance of the membership at the board meetings has been
extraordinarily good. Thus, during the years 1920 to 1925, inclusive, when the board was composed of nine members,
the attendance has regularly been eight at each meeting with the exception of two years when the average attendance
was seven. It is insisted in the brief for the Attorney-General that the payment of the compensation indicated is
excessive and prejudicial to the interests of the shareholders at large. For the respondent, attention is directed to the
fact that the liberal policy adopted by the association with respect to the compensation of the directors has had highly
beneficial results, not only in securing a constant attendance on the part of the membership, but in obtaining their
intelligent attention to the affairs of the association. Certainly, in this connection, the following words from the report of
the Government examiners for 1918 to the Insular Treasurer contain matter worthy of consideration:
"The management of the association is entrusted to men of recognized ability in financial affairs
and it is believed that they have long foreseen all possible future contingencies and that under such men
the interests of the stockholders are duly protected. The steps taken by the directorate to curtail the influx
of unnecessary capital into the Association's coffers, as mentioned above, reveals how the men at the
helm of the Association are always on the lookout to grasp the situation and to apply the necessary
remedy as the circumstances may require. The accounts and documents were found in the same
excellent condition as in the previous examination."
In so far as this court is concerned the question here before us is not one concerning the propriety and wisdom
of the measure of compensation adopted by the respondent but rather the question of the validity of the measure. Upon
this point there can, it seems to us, be no difference of intelligent opinion. The Corporation Law does not undertake to
prescribe the rate of compensation for the directors of corporations. The power to fixed the compensation they shall
receive, if any, is left to the corporation, to be determined in its by-laws (Act No. 1459, sec. 21). Pursuant to this
authority the compensation for the directors of El Hogar Filipino has been fixed in section 92 of its by-laws, as already
stated. The justice and propriety of this provision was a proper matter for the shareholders when the by-laws were
framed; and the circumstance that, with the growth of the corporation, the amount paid as compensation to the directors
has increased beyond what would probably be necessary to secure adequate service from them is a matter that cannot
be corrected in this action; nor can it properly be made a basis for depriving the respondent of its franchise, or even for
enjoining it from compliance with the provisions of its own by-laws. If a mistake has been made, or the rule adopted in
the by-laws has been found to work harmful results, the remedy is in the hands of the stockholders who have power at
any lawful meeting to change the rule. The remedy, if any, seems to lie rather in publicity and competition, rather than in
a court proceeding. The sixth cause of action is in our opinion without merit.
Seventh cause of action.— It appears that the promoter and organizer of El Hogar Filipino was Mr. Antonio
Melian, and in the early stages of the organization of the association the board of directors authorized the association to
make a contract with him with regard to the services to be rendered by him and the compensation to be paid to him
therefor. Pursuant to this authority the president of the corporation, on January 11, 1911, entered into a written
agreement with Mr. Melian, which is reproduced in the agreed statement of facts and of which the important clauses are
these:
"1. The corporation 'El Hogar Filipino Sociedad Mutua de Construccion y Prestamos,' and on its
behalf its president, Don Antonio R. Roxas, hereby confers on Don Antonio Melian the office of manager
of said association for the period of one year from the date of this contract.
"2. Don Antonio Melian accepts said office and undertakes to render the services thereto
corresponding for the period of one year, as prescribed by the by-laws of the corporation, without salary.
"3. Don Antonio Melian furthermore undertakes to pay, for his own account, all the expenses
incurred in the organization of the corporation.
"4. Don Antonio Melian further undertakes to lend to the corporation, without interest, the sum of
six thousand pesos (P6,000), Philippine currency, for the purpose of meeting the expense of rent, office
supplies, etcetera, until such time as the association has sufficient funds of its own with which to return
this loan: Provided, nevertheless, That the maximum period thereof shall not exceed three (3) years.
"5. Don Antonio Melian undertakes that the capital of the association shall amount to the sum of
four hundred thousand pesos (P400,000), Philippine currency, par value, during the first year of its
duration.
"6. In compensation of the studies made and services rendered by Don Antonio Melian for its
organization, the expenses incurred by him to that end, and in further consideration of the said loan of six
thousand pesos (P6,000), and of the services to be rendered by him as manager, and of the obligation
assumed by him that the nominal value of the capital of the association shall reach the sum of four
hundred thousand pesos (P400,000) during the first year of its duration, the corporation 'El Hogar Filipino
Sociedad Mutua de Construccion y Prestamos' hereby grants him five per centum (5%) of the net profits
to be earned by it in each year during the period fixed for the duration of the association by its articles of
incorporation; Provided, That this participation in the profits shall be transmitted to the heirs of Senor
Melian in the event of his death; And provided further, That the performance of all the obligations
assumed by Senor Melian in favor of the association, in accordance with this contract, shall and does
constitute a condition precedent to the acquisition by Senor Melian of the right to the said participation in
the profits of the association, unless the non-performance of such obligations shall be due to a fortuitous
event or force majeure."
In conformity with this agreement there was inserted in section 92 of the by-laws of the association a provision
recognizing the rights of Mr. Melian, as founder, to 5 per centum of the net profits shown by the annual balance sheet,
payment of the same to be made to him or his heirs during the life of the association. It is declared in said article that
this portion of the earnings of the association is conceded to him in compensation for the studies, work and
contributions made by him for the organization of El Hogar Filipino, and the performance on his part of the contract of
January 11, 1911, above quoted. During the whole life of the association, thus far, it has complied with the obligations
assumed by it in the contract above-mentioned; and during the years 1911 to 1925, inclusive, it paid to him as founder's
royalty the sum of P459,011.19, in addition to compensation received from the association by him in remuneration of
services to the association in various official capacities.
As a seventh cause of action it is alleged in the complaint that this royalty of the founder is "unconscionable,
excessive and out of all proportion to the services rendered, besides being contrary to and incompatible with the spirit
and purpose of building and loan associations." It is not alleged that the making of this contract was beyond the powers
of the association (ultra vires); nor is it alleged that it is vitiated by fraud of any kind in its procurement. Nevertheless, it
is pretended that in making and observing said contract the respondent committed an offense requiring its dissolution,
or, as is otherwise suggested, that the association should be enjoined from preforming the agreement.
It is our opinion that this contention is entirely without merit. Stated in its true simplicity, the primary question
here is whether the making of a (possibly) indiscreet contract is a capital offense in a corporation,— a question which
answers itself. No possible doubt exists as to the power of a corporation to contract for services rendered and to be
rendered by a promoter in connection with organizing and maintaining the corporation. It is true that contracts with
promoters must be characterized by good faith; but could it be said with certainty, in the light of facts existing at the time
this contract was made, that the compensation therein provided was excessive? If the amount of the compensation now
appears to be a subject of legitimate criticism, this must be due to the extraordinary development of the association in
recent years.
If the Melian contract had been clearly ultra vires— which is not charged and is certainly untrue— its continued
performance might conceivably be enjoined in such a proceeding as this; but if the defect from which it suffers is mere
matter for an action of nullity? an injunction can- not be obtained in this action because Melian is not a party. It is
rudimentary in law that an action to annul a contract cannot be maintained without joining both the contracting parties as
defendants. Moreover, the proper party to bring such an action is either the corporation itself, or some shareholder who
has an interest to protect.
The mere fact that the compensation paid under this contract is in excess of what, in the full light of history, may
be considered appropriate is not a proper consideration for this court, and supplies no ground for interfering with its
performance. In the case of El Hogar Filipino vs. Rafferty (37 Phil., 995), which was before this court nearly ten years
ago, this court held that the El Hogar Filipino is a mutual benefit society and that the existence of this contract with Mr.
Melian did not affect the association's legal character. The inference is that the contract under consideration was then
considered binding, and it occurred to no one that it was invalid. It would be a radical step indeed for a court to attempt
to substitute its judgment for the judgment of the contracting parties and to hold, as we are invited to hold under this
cause of action, that the making of such a contract as this removes the respondent association from the pale of the law.
The majority of the court is of the opinion that our traditional respect for the sanctity of the contract obligation should
prevail over the radical. and innovating tendencies which find acceptance with some and which, if given full rein, would
go far to sink legitimate enterprise in the Islands into the pit of populism and bolshevism. The seventh count is not
sustainable.
Eighth cause of action. — Under the fourth cause of ac- tion we had a case where the alleged ground for the
revocation of the respondent's charter was based upon the presence in the by-laws of article 10 that was found to be
inconsistent with the express provisions of law. Under the eighth cause of action the alleged ground for putting an end
to the corporate life of the respondent is found in the presence of other articles in the by-laws, namely, articles 70 and
76, which are alleged to be unlawful but which, as will presently be seen, are entirely valid. Article 70 of the by-laws in
effect requires that persons elected to the board of directors must be holders of shares of the paid up value of P5,000,
which shall be held as security for their action; but it is added that said security may be put up in the behalf of any
director by some other holder of shares in the amount stated. Article 76 of the by-laws declares that the directors waive
their right as shareholders to receive loans from the association.
It is asserted, under the eight cause of action, that article 70 is objectionable in that, under the requirement for
security, a poor member, or wage-earner, cannot serve as director, irrespective of other qualifications, and that as a
matter of fact only men of means actually sit on the board. Article 76 is criticized on the ground that the provision
requiring directors to renounce their right to loans unreasonably limits their rights and privileges as members. There is
nothing of value in either of these suggestions. Section 21 of the Corporation Law expressly gives the power to the
corporation to provide in its by-laws for the qualifications of directors; and the requirement of security from them for the
proper discharge of the duties of their office, in manner prescribed in article 70, is highly prudent and in conformity with
good practice. Article 76, prohibiting directors from making loans to themselves, is of course designed to prevent the
possibility of the looting of the corporation by unscrupulous directors. A more discreet provision to insert in the by-laws
of a building and loan association would be hard to imagine. Clearly, the eighth cause of action cannot be sustained.
Ninth cause of action.— The specification under this head is in effect that the respondent has abused its
franchise in issuing "special" shares. The issuance of these shares is alleged to be illegal and inconsistent with the plan
and purposes of building and loan associations; and in particular, it is alleged that they are, in the main, held by well-to-
do people purely for purposes of investment and not by wage-earners for accumulating their modest savings for the
building of homes.
In the articles of incorporation we find the special shares described as follows:
" 'Special' shares shall be issued upon the payment of 80 per cent of their par value in cash, or in
monthly dues of P10. The 20 per cent remaining of the par value of such shares shall be completed by
the accumulation thereto of their proportionate part of the profits of the corporation. At the end of each
quarter the holders of special shares shall be entitled to receive in cash such part of the net profits of the
corporation corresponding to the amount on such date paid in by the holders of special shares, on
account thereof, as shall be determined by the directors, and at the end of each year the full amount of
the net profits available for distribution corresponding to the special shares. The directors shall apply
such part as they deem advisable to the amortization of the subscription to capital with respect to shares
not fully paid up, and the remainder of the profits, if any, corresponding to such shares, shall be delivered
to the holders thereof in accordance with the provisions of the by-laws."
The ground for supposing the issuance of the "special" shares to be unlawful is that special shares are not
mentioned in the Corporation Law as one of the forms of security which may be issued by the association. In the agreed
statement of facts it is said that special shares are issued upon two plans. By the first, the subscriber pays to the
association, upon subscribing, P160 in cash, on account of each share. By the second, the shareholder, upon
subscribing, pays in cash P10 for each share taken, and undertakes to pay P10 a month, as dues, until the total so paid
in amounts to P160 per share. On December 31, 1925, there were outstanding 20,844 special shares of a total paid
value (including accumulations) of P3,680,162.51. The practice of El Hogar Filipino, since 1915, has been to
accumulate to each special share, at the end of the year, one-tenth of the dividend declared and to pay the remainder of
the dividend in cash to the holders of shares. Since the same year dividends have been declared on the special and
common shares at the rate of 10 per centum per annum. When the amount paid in upon any special share plus the
accumulated dividends accruing to it, amounts to the par value of the share (P200), such share matures and ceases to
participate further in the earnings. The amount of the par value of the share (P200) is then returned to the shareholder
and the share cancelled. Holders of special and ordinary shares participate ratably in the dividends declared and
distributed, the part pertaining to each share being computed on the basis of the capital paid in, plus the accumulated
dividends pertaining to each share at the end of the year. The total number of shares of El Hogar Filipino outstanding on
December 31, 1925, was 125,760, owned by 5,826 shareholders, and divided into classes as follows:

Preferred shares 1,503

Special shares 20,884

Ordinary shares 103,363

The matter of the propriety of the issuance of special shares by El Hogar Filipino has been before this court in
two earlier cases, in both of which the question has received the fullest consideration from this court. In El Hogar
Filipino vs. Rafferty (37 Phil., 995), it was insisted that the issuance of such shares constituted a departure on the part
of the association from the principle of mutuality; and it was claimed by the Collector of Internal Revenue that this
rendered the association liable for the income tax to which other corporate entities are subject. It was held that this
contention was untenable and that El Hogar Filipino was a legitimate building and loan association notwithstanding the
issuance of said shares. In Severino vs. El Hogar Filipino (G. R. No. 24926), 1 and the related cases of Gervasio
Miraflores and Gil Lopez against the same entity, it was asserted by the plaintiffs that the emission of special shares
deprived the herein respondent of the privileges and immunities of a building and loan association and that as a
consequence the loans that had been made to the plaintiffs in those cases were usurious. Upon an elaborate review of
the authorities, the court, though divided, adhered to the principle announced in the earlier case and held that the
issuance of the special shares did not affect the respondent's character as a building and loan association nor make its
loans usurious. In view of the lengthy discussion contained in the decisions above-mentioned, it would appear to be an
act of supererogation on our part to go over the same ground again. The discussion will therefore not be repeated, and
what is now to be said should be considered supplemental thereto.
Upon examination of the nature of the special shares in the light of American usage, it will be found that said
shares are precisely the same kind of shares that, in some American jurisdictions, are generally known as advance-
payment shares; and if close attention be paid to the language used in the last sentence of section 178 of the
Corporation Law, it will be found that special shares were evidently created for the purpose of meeting the conditions
caused by the prepayment of dues that is there permitted. The language of this provision is as follows: "Payments of
dues or interest may be made in advance, but the corporation shall not allow interest on such advance payments at a
greater rate than six per centum per annum nor for a longer period than one year." In one sort of special shares the
dues are prepaid to the extent of P160 per share; in the other sort prepayment is made in the amount of P10 per share,
and the subscribers assume the obligation to pay P10 monthly until P160 shall have been paid.
It will not escape notice that the provision quoted says that interest shall not be allowed on advance payments
at a greater rate than 6 per centum per annum nor for a longer period than one year. The word "interest" as there used
must be taken in its true sense of compensation for the use of money loaned, and it must not be confused with the dues
upon which it is contemplated that the interest may be paid. Now, in the absence of any showing to the contrary, we
infer that no interest is ever paid by the association in any amount for the advance payments made on these shares;
and the reason is to be found in the fact that the participation of the special shares in the earnings of the corporation, in
accordance with section 188 of the Corporation Law, sufficiently compensates the shareholder for the advance
payments made by him; and no other incentive is necessary to induce investors to purchase the stock.
It will be observed that the final 20 per centum of the par value of each special share is not paid for by the
shareholder with funds out of the pocket. The amount is satisfied by applying a portion of the shareholder's participation
in the annual earnings. But as the right of every shareholder to such participation in the earnings is undeniable, the
portion thus annually applied is as much the property of the shareholder as if it were in fact taken out of his pocket. It
follows that the emission of the special shares does not involve any violation of the principle that the shares must be
sold at par.
From what has been said it will be seen that there is express authority, even in the very letter of the law, for the
emission of advance-payment or "special" shares, and the argument that these shares are invalid is seen to be
baseless. In addition to this it is satisfactorily demonstrated in Severino vs. El Hogar Filipino, supra, that even assuming
that the statute has not expressly authorized such shares, yet the association has implied authority to issue them. The
complaint consequently fails also as regards the ground stated in the ninth cause of action.
Tenth cause of action. — Under this head of the complaint it is alleged that the defendant is pursuing a policy of
depreciating, at the rate of 10 per centum per annum, the value of the real properties acquired by it at its sales; and it is
alleged that this rate is excessive. From the agreed statement it appears that since its organization in 1910 El Hogar
Filipino, prior to the end of the year 1925, had made 1,373 loans to its shareholders secured by first mortgages on real
estate as well as by the pledge of the shares of the borrowers. In the same period the association has purchased at
foreclosure sales the real estate constituting the security for 54 of the aforesaid loans. In making these purchases the
association has always bid the full amount due to it from the debtor, after deducting the withdrawal value of the shares
pledged as collateral, with the result that in no case has the shareholder been called upon to pay a deficiency judgment
on foreclosure.
El Hogar Filipino places real estate so purchased in its inventory at actual cost, as determined by the amount
bid on foreclosure sale; and thereafter until sold the book value of such real estate is depreciated at the rate fixed by the
directors in accordance with their judgment as to each parcel, the annual average depreciation having varied from
nothing to a maximum of 14.138 per cent. The book value of such real estate is not followed in making sales thereof,
but sales are made for the best prices obtainable, whether greater or less than the book value.
It is alleged in the complaint that depreciation is charged by the association at the rate of 10 per centum per
annum. The agreed statement of facts on this point shows that the annual average varies from nothing to a maximum of
something over 14 per centum. We are thus left in the dark as to the precise depreciation allowed from year to year. It is
not claimed for the Government that the association is without power to allow some depreciation; and it is quite clear
that the board of directors possesses a discretion in this matter. There is no positive provision of law prohibiting the
association from writing off a reasonable amount for depreciation on its assets for the purpose of determining its real
profits; and article 74 of its by-laws expressly authorizes the board of directors to determine each year the amount to be
written down upon the expenses of installation and the property of the corporation. There can be no question that the
power to adopt such a by-law is embraced within the power to make by-laws for the administration of the corporate
affairs of the association and for the management of its business, as well as the care, control and disposition of its
property (Act No. 1459, sec. 13 [7]). But the Attorney-General questions the exercise of the discretion confided to the
board; and it is insisted that the excessive depreciation of the property of the association is objectionable in several
respects, but mainly because it tends to increase unduly the reserves of the association, thereby frustrating the right of
the shareholders to participate annually and equally in the earnings of the association.
This count of the complaint proceeds, in our opinion, upon an erroneous notion as to what a court may do in
determining the internal policy of a business corporation. If the criticism contained in the brief of the Attorney-General
upon the practice of the respondent association with respect to depreciation be well founded, the Legislature should
supply the remedy by defining the extent to which depreciation may be allowed by building and loan associations.
Certainly this court cannot undertake to control the discretion of the board of directors of the association about an
administrative matter as to which they have legitimate power of action. The tenth cause of action is therefore not well
founded.
Eleventh and twelfth causes of action. — The same comment is appropriate with respect to the eleventh and
twelfth causes of action, which are treated together in the briefs, and will be here combined. The specification in the
eleventh cause of action is that the respondent maintains excessive reserve funds, and in the twelfth cause of action
that the board of directors has settled upon the unlawful policy of paying a straight annual dividend of 10 per centum,
regardless of losses suffered and profits made by the corporation and in contravention of the requirements of section
188 of the Corporation Law. The facts relating to these two counts in the complaint, as set forth in the stipulation, are
these:
In article 92 of the by-laws of El Hogar Filipino it is provided that 5 per centum of the net profits earned each
year, as shown by the annual balance sheet shall be carried to a reserve fund. The fund so created is called the-
General Reserve. Article 93 of the by-laws authorizes the directors to carry funds to a Special Reserve, whenever in
their judgment it is advisable to do so, provided that the annual dividend in the year in which funds are carried to special
reserve exceeds 8 per centum. It appears to have been the policy of the board of directors for several years past to
place in the special reserve any balance in the profit and loss account after the satisfaction of preferential charges and
the payment of a dividend of 10 per centum to all special and ordinary shares (with accumulated dividends). As things
stood in 1926 the General Reserve contained an amount equivalent to about 5 per centum of the paid-in value of
shares. This fund has never been drawn upon for the purpose of maintaining the regular annual dividend; but recourse
has been had to the Special Reserve on three different occasions to make good the amount necessary to pay
dividends. It appears that in the last five years the reserves have declined from something over 9 per cent to something
over 7.
It is insisted in the brief of the Attorney-General that the maintenance of reserve funds is unnecessary in the
case of building and loan associations, and at any rate the keeping of reserves is inconsistent with section 188 of the
Corporation Law. Moreover, it is said that the practice of the association in declaring regularly a 10 per cent dividend is
in effect a guaranty by the association of a fixed dividend which is contrary to the intention of the statute.
Upon careful consideration of the questions involved we find no reason to doubt the right of the respondent to
maintain these reserves. It is true that the Corporation Law does not expressly grant this power, but we think it is to be
implied. It is a fact of common observation that all commercial enterprises encounter periods when earnings fall below
the average, and the prudent manager makes provision for such contingencies. To regard all surplus as profit is to
neglect one of the primary canons of good business practice. Building and loan associations, though among the most
solid of financial institutions, are nevertheless subject to vicissitudes. Fluctuations in the dividend rate are highly
detrimental to any fiscal institutions, while uniformity in the payments of dividends, continued over long periods, supplies
the surest foundation of public confidence.
The question now under consideration is not new in jurisprudence, for the American courts have been called
upon more than once to consider the legality of the maintenance of reserves by institutions of this or similar character.
In Greeff vs. Equitable Life Assurance Society (160 N. Y., 19; 73 Am. St. Rep., 659), the court had under
consideration a charter provision of a life insurance company, organized on the mutual plan, in its relation to the power
of the company to provide reserves. There the statute provided that "the officers of the company, within sixty days from
the expiration of the first five years, from December 31, 1859, and within the first sixty days of every subsequent period
of five years, shall cause a balance to be struck of the affairs of the company, which shall exhibit its assets and
liabilities, both present and contingent, and also the net surplus, after deducting a sufficient amount to cover all
outstanding risks and other obligations. Each policy holder shall be credited with an equitable share of the said surplus."
The court said:
"No prudent person would be inclined to take a policy in a company which had so improvidently
conducted its affairs that it only retained a fund barely sufficient to pay its present liabilities, and,
therefore, was in a condition where any change by the reduction of interest upon, or depreciation in, the
value of its securities, or any increase of mortality, would render it insolvent and subject to be placed in
the hands of a receiver. The evident purpose of the provisions of the defendant's charter and policy
relating to this subject was to vest in the directors of the corporation a discretion to determine the
proportion of its surplus which should be divided each year."
In a friendly suit tried in a circuit court of Wisconsin in 1916, entitled Bohemian Bldg. and Loan Association V.5.
Knolt, the court, in commenting on the nature of these reserves, said:
"The apparent function of this fund is to insure the stockholders against losses. Its purpose is not
unlike that of the various forms of insurance now in such common use. . . . This contribution is as
legitimate an item of expense as are the premiums paid on any insurance policy." (See Clarks and
Chase, Building and Loan Associations, footnote, page 344.)
In commenting on the necessity of such funds, Sundheim says:
"It is optional with the association whether to maintain such a fund or not, but justice and good
business policy seem to require it. The retiring stockholder must be paid the value of his stock in cash
and leave for those remaining a large number of securities and perhaps some real estate purchased to
protect the association's interest. How much will be realized on these securities, or real estate, no human
foresight can tell. Further, the realizing on these securities may entail considerable litigation and expense.
There are many other contingencies which might cause a shrinkage in the association's assets, such as
defective titles, undisclosed defalcations on the part of an officer, a miscalculation of assets and liabilities,
and many other errors and omissions which must always be reckoned with in the conduct of human
affairs.
"The contingent fund is merely insurance against possible loss. That losses may occur from time
to time seems almost inevitable and it is, therefore, inequitable that the remaining stockholders should be
compelled to accept all securities at par, so, to say the least, the maintenance of this fund is justified. The
association teaches the duty of providing for the proverbial rainy day. Why should it not provide for the
hour of adversity? The reserve fund has protected the maturing or withdrawing member during the period
of his membership. In case of loss it has or would have, reimbursed him and, at all times, it has protected
him and given strength and standing to the association. Losses may occur, after his membership ceases,
that arose from some mistake or mismanagement committed during the period of his membership, and in
fairness and equity the remaining members should have some protection against this." (Sundheim, Law
of Building and Loan Associations, sec. 53.)
The Government insists, we think, upon an interpretation of section 188 of the Corporation Law that is
altogether too strict and literal. From the fact that the statute provides that profits and losses shall be annually
apportioned among the shareholders it is argued that all earnings should be distributed without carrying anything to the
reserve. But it will be noted that it is provided in the same section that the profits and losses shall be determined by the
board of directors; and this means that they shall exercise the usual discretion of good businessmen in allocating a
portion of the annual profits to purposes needful to the welfare of the association. The law contemplates the distribution
of earnings and losses after other legitimate obligations have been met.
Our conclusion is that the respondent has the power to maintain the reserves criticized in the eleventh and
twelfth counts of the complaint; and at any rate, if it be supposed that the reserves referred to have become excessive,
the remedy is in the hands of the Legislature. It is no proper function of the court to arrogate to itself the control of
administrative matters which have been confided to the discretion of the board of directors. The causes of action under
discussion must be pronounced to be without merit.
Thirteenth cause of action. — The specification under this head is, in effect, that the respondent association
has made loans which, to the knowledge of the association's officers, were intended to be used by the borrowers for
other purposes than the building of homes. In this connection it appears that, though loans have been made by the
association exclusively to its shareholders, no attempt has been made by it to control the borrowers with respect to the
use made of the borrowed funds, the association being content to see that the security given for the loan in each case is
sufficient. On December 31, 1925, the respondent had five hundred forty-four loans outstanding secured by mortgages
upon real estate and by the pledge of the borrowers' shares in an amount sufficient at maturity to amortize the loans.
With respect to the nature of the real estate upon which these loans were made it appears that three hundred fifty-one
loans were secured by mortgages upon city residences, seven by mortgages upon commercial building in cities, and
three by mortgages upon unimproved city lots. At the same time one hundred eighty-three of the loans were secured by
mortgages upon improved agricultural property consisting of coconut groves, sugar land, and rice land, with a total area
of about 7,558 hectares. From information gathered by the association from voluntary statements of borrowers given at
the time of application with respect to the use intended to be made of the borrowed funds, it appears that the amount of
P693,200 was borrowed to redeem real property from existing mortgages or pactos de retro, P280,800 to buy real
estate, P449,100 to erect buildings, P24,000 to improve and repair buildings, P1,480,900 for agricultural purposes,
while the amount of P5,763,700 was borrowed for purposes not disclosed.
Upon these facts an elaborate argument has been constructed in behalf of the plaintiff to the effect that in
making loans for other purposes than the building of residential houses the association has illegally departed from its
charter and made itself amenable to the penalty of dissolution. Aside from being directly opposed to the decision of this
court in Lopez and Javelona vs. El Hogar Filipino and Registrar of Deeds of Occidental Negros (47 Phil., 249), this
contention finds no substantial support in the prevailing decisions made in American courts; and our attention has not
been directed to a single case wherein the dissolution of a building and loan association has been decreed in a quo
warranto proceeding because the association allowed its borrowers to use the loans for other purposes than the
acquisition of homes.
The case principally relied upon for the Government appears to be Pfeister vs. Wheeling Building Association
(19 W. Va., 676, 716), which involved the question whether a building and loan association could recover the full
amount of a note given to it by a member and secured by a mortgage from a stranger. At the time the case arose there
was a statute in force in the State of West Virginia expressly forbidding building and loan associations to use or direct
their funds for or to any other object or purpose than the buying of lots or houses or in building and repairing houses,
and it was declared that in case the funds should be improperly directed to other objects, the offending association
should forfeit all rights and privileges as a corporation. Under the statute so worded the court held that the plaintiff could
only recover the amount actually advanced by it with lawful interest and fines, without premium; and judgment was
given accordingly. The suggestion in that case that the result would have been the same even in the absence of statute
was mere dictum and is not supported by respectable authority.
Reliance is also placed in the plaintiff's brief upon Mc-Cauley vs. Building & Saving Association (97 Tenn., 421).
The statute in force in the State of Tennessee at the time this action arose provided that all loans should be made to the
members of the association at open stated meetings and that the money should be lent to the highest bidder.
Inconsistently with this provision, there was inserted in the by-laws of the association a provision to the effect that no
loan should be made at a greater premium than 30 per cent, nor at a less premium than 29 7/8 per cent. It was held that
this by-law made free and open competition impossible and that it in effect established a fixed premium. It was
accordingly held, in the case cited, that an association could not recover such part of the loan as had been applied by it
to the satisfaction of a premium of 30 per centum.
We have no criticism to make upon the result reached in either of the two decisions cited, but it is apparent that
much of the discussion contained in the opinions in those cases does not reflect the doctrine now prevailing in the
United States; and much less are those decisions applicable in this jurisdiction. There is no statute here expressly
declaring that loans may be made by these associations solely for the purpose of building homes. On the contrary, the
building of homes is mentioned in section 171 of the Corporation Law as only one among several ends which building
and loan associations are designed to promote. Furthermore, section 181 of the Corporation Law expressly authorizes
the board of directors of the association from time to time to fix the premium to be charged.
In the brief of the plaintiff a number of excerpts from textbooks and decisions have been collated in which the
idea is developed that the primary design of building and loan associations should be to help poor people to procure
homes of their own. This beneficent end is undoubtedly served by these associations, and it is not to be denied that
they have been generally fostered with this end in view. But in this jurisdiction at least the lawmaker has taken care not
to limit the activities of building and loan associations in an exclusive manner, and the exercise of the broader powers
must in the end approve itself to the business community. Judging from the past history of these institutions it can be
truly said that they have done more to encourage thrift, economy and having among the people at large than any other
institution of modern times, not excepting even the savings banks. In this connection Mr. Sundheim, in a late treatise
upon the subject of the law of building and loan associations, makes the following comment:
"They have grown to such an extent in recent years that they no longer restrict their money to the
home buyer, but loan their money to the mere investor or dealer in real estate. They are the holders of
large mortgages se- cured upon farms, factories and other business properties and rows of stores and
dwellings. This is not an abuse of their powers or a departure from their main purposes, but only a natural
and proper expansion along healthy and legitimate lines." (Sundheim, Building and Loan Associations,
sec. 7.)
Speaking of the purposes for which loans may be made, the same author adds:
"Loans are made for the purpose of purchasing a homestead, or other real estate, or for any
lawful purpose or business, but there is no duty or obligation of the association to inquire for what
purpose the loan is obtained, or to require any stipulation from the borrower as to what use he will make
of the money, or in any manner to supervise or control its disbursement." (Sundheim, Building and Loan
Associations, sec. 111.)
In Lopez and Javelona vs. El Hogar Filipino and Registrar of Deeds of Occidental Negros (47 Phil., 249), this
court had before it the question whether a loan made by the respondent association upon the security of a mortgage
upon agricultural land, — where the loan was doubtless used for agricultural purposes, — was usurious or not; and the
case turned upon the point whether, in making such loans, the association had violated the law and departed from its
fundamental purposes. The conclusion of the court was that the loan was valid and could be lawfully enforced by a
nonjudicial foreclosure in conformity with the terms of the contract between the association and the borrowing member.
We now find no reason to depart from the conclusion reached in that case, and it is unnecessary to repeat what was
then said. The thirteenth cause of action must therefore be pronounced unfounded.
Fourteenth cause of action. — The specification under this head is that the loans made by the defendant for
purposes other than building or acquiring homes have been extended in extremely large amounts and to wealthy per-
sons and large companies. In this connection attention is directed to eight loans made at different times in the last
several years to different persons or entities, ranging in amounts from P120,000 to P390,000 and to two large loans
made to the Roxas Estate and to the Pacific Warehouse Company in the amounts of P1,122,000 and P2,320,000,
respectively. In connection with the larger of the two loans just mentioned it is shown that for about ten months after this
loan was made the available funds of El Hogar Filipino were reduced to the point that the association was compelled to
take advantage of certain provisions of its by-laws authorizing the postponement of the payment of claims resulting from
withdrawals, whereas previously the association had always settled these claims promptly from current funds. At no
time was there apparently any delay in the payment of matured shares; but in four or five cases there was as much as
ten months delay in the payment of withdrawal applications.
There is little that can be said upon the legal aspects of this cause of action. In so far as it relates to the
purposes for which these loans were made, the matter is covered by what was said above with reference to the
thirteenth cause of action; and in so far as it relates to the personality of the borrowers, the question belongs more
directly to the discussion under the sixteenth cause of action, which will be found below. The point, then, which remains
for consideration here is whether it is a suicidal act on the part of a building and loan association to make loans in large
amounts. If the loans which are here the subject of criticism had been made upon inadequate security especially in case
of the largest two, the consequences certainly would have been disastrous to the association in the extreme; but no
such fact is alleged; and it is to be assumed that none of the ten borrowers have defaulted in their contracts.
Now, it must be admitted that two of these loans at least are of a very large size, considering the average range
of financial transactions in this country; and the making of the largest loan was followed, as we have already seen, with
unpleasant consequences to the association in dealing with current claims. Nevertheless the agreed statement of facts
shows that all of the loans referred to are only ten out of a total of five hundred forty-four outstanding on December 31,
1925; and the average of all the loans taken together is modest enough. It appears that the chief examiner of banks and
corporations of the Philippine Treasury, after his examination of El Hogar Filipino at the end of the year 1925, made a
report concerning this association as of January 31, 1926, in which he criticized the Pacific Warehouse Company loan
as being so large that it temporarily crippled the lending power of the association for some time. This criticism was
apparently justified as proper comment on the activities of the association; but the question for us here to decide is
whether the making of this and the other large loans constitutes such a misuser of the franchise as would justify us in
depriving the association of its corporate life. This question appears to us to be so simple as almost to answer itself.
The law states no limit with respect to the size of the loans to be made by the association. That matter is confided to the
discretion of the board of directors; and this court cannot arrogate to itself a control over the discretion of the chosen
officials of the company. If it should be thought wise in the future to put a limit upon the amount of loans to be made to a
single person or entity, resort should be had to the Legislature; it is not a matter amenable to judicial control. The
fourteenth cause of action is therefore obviously without merit.
Fifteenth cause of action.— The criticism here comes back to the supposed misdemeanor of the respondent in
maintaining its reserve funds, — a matter already discussed under the eleventh and twelfth causes of action. Under the
fifteenth cause of action it is claimed that upon the expiration of the franchise of the association through the effluxion of
time, or earlier liquidation of its business, the accumulated reserves and other properties will accrue to the founder, or
his heirs, and the then directors of the corporation and to those persons who may at that time to be holders of the
ordinary and special shares of the corporation. In this connection we note that article 95 of the by-laws reads as follows:
"ART. 95. The funds obtained by the liquidation of the association shall be applied in the first
place to the repayment of shares and the balance, if any, shall be distributed in accordance with the
system established for the distribution of annual profits."
It will be noted that the cause of action with which we are now concerned is not directed to any positive
misdemeanor supposed to have been committed by the association. It has exclusive relation to what may happen some
thirty-five years hence when the franchise expires, supposing of course that the corporation should not be reorganized
and continued after that date. There is nothing in article 95 of the by-laws which is, in our opinion, subject to criticism.
The real point of criticism is that upon the final liquidation of the corporation years hence there may be in existence a
reserve fund out of all proportion to the requirements that may then fall upon it in the liquidation of the company. It
seems to us that this is a matter that may be left to the prevision of the directors or to legislative action if it should be
deemed expedient to require the gradual suppression of the reserve funds as the time for dissolution approaches. It is
no matter for judicial interference, and much less could the resumption of the franchise on this ground be justified.
There is no merit in the fifteenth cause of action.
Sixteenth cause of action. — This part of the complaint assigns as cause of action that various loans now
outstanding have been made by the respondent to corporations and partnerships, and that these entities have in some
instances subscribed to shares in the respondent for the sole purpose of obtaining such loans. In this connection it
appears from the stipulation of facts that of the 5,826 shareholders of El Hogar Filipino, which composed its
membership on December 31, 1925, twenty-eight are juridical entities, comprising sixteen corporations and fourteen
partnerships; while of the five hundred forty-four loans of the association outstanding on the same date, nine had been
made to corporations and five to partnerships. It is also admitted that some of these juridical entities became
shareholders merely for the purpose of qualifying themselves to take loans from the association, and the same is said
with respect to many natural persons who have taken shares in the association. Nothing is said in the agreed statement
of facts on the point whether the corporations and partnerships that have taken loans from the respondent are qualified
by law governing their own organization to enter into these contracts with the respondent.
In section 173 of the Corporation Law it is declared that "any person" may become a stockholder in building and
loan associations. The word "person" appears to be here used in its general sense, and there is nothing in the context
to indicate that the expression is used in the restricted sense of "natural person." It should therefore be taken to include
both natural and artificial persons, as indicated in section 2 of the Administrative Code. We would not say that the word
"person," or "persons," is to be taken in this broad sense in every part of the Corporation Law. For instance, it would
seem reasonable to say that the incorporators of a corporation ought to be natural persons, although in section 6 it is
said that five or more "persons," not exceeding fifteen, may form a private corporation. But the context there, as well as
the common sense of the situation, suggests that natural persons are meant. When it is said, however, in section 173,
that "any person" may become a stockholder in a building and loan association, no reason is seen why the phrase may
not be taken in its proper broad sense of either a natural or artificial person. At any rate the question whether these
loans and the attendant subscriptions were properly made involves a consideration of the power of the subscribing
corporations and partnerships to own the stock and take the loans; and it is not alleged in the complaint that they were
without power in the premises. Of course the mere motive with which subscriptions are made, whether to qualify the
stockholders to take a loan or for some other reason, is of no moment in determining whether the subscribers were
competent to make the contracts. The result is that we find nothing in the allegations of the sixteenth cause of action, or
in the facts developed in connection therewith, that would justify us in granting the relief.
Seventeenth cause of action. — Under the seventeenth cause of action, it is charged that in disposing of real
estates purchased by it in the collection of its loans, the defendant has on various occasions sold some of the said real
estate on credit, transferring the title thereto to the purchaser; that the properties sold are then mortgaged to the
defendant to secure the payment of the purchase price, said amount being considered as a loan, and carried as such in
the books of the defendant, and that several such obligations are still outstanding. It is further charged that the persons
and entities to which said properties are sold under the condition charged are not members or shareholders nor are
they made members or shareholders of the defendant.
This part of the complaint is based upon a mere technicality of bookkeeping. The central idea involved in the
discussion is the provision of the Corporation Law requiring loans to be made to stockholders only and on the security
of real estate and shares in the corporation, or of shares alone. It seems to be supposed that, when the respondent
sells property acquired at its own foreclosure sales and takes a mortgage to secure the deferred payments, the
obligation of the purchaser is a true loan, and hence prohibited. But in requiring the respondent to sell real estate which
it acquires in connection with the collection of its loans within five years after receiving title to the same, the law does
not prescribe that the property must be sold for cash or that the purchaser shall be a shareholder in the corporation.
Such sales can of course be made upon terms and conditions approved by the parties; and when the association takes
a mortgage to secure the deferred payments, the obligation of the purchaser cannot be fairly described as arising out of
a loan. Nor does the fact that it is carried as a loan on the books of the respondent make it a loan in law. The contention
of the Government under this head is untenable.
In conclusion, the respondent is enjoined in the future from administering real property not owned by itself,
except as may be permitted to it by contract when a borrowing shareholder defaults in his obligation. In all other
respects the complaint is dismissed, without costs. So ordered.
||| (Government of the Philippine Islands v. El Hogar Filipino, G.R. No. 26649, [July 13, 1927], 50 PHIL 399-477)

[G.R. No. L-18216. October 30, 1962.]

STOCKHOLDERS OF F. GUANZON AND SONS, INC., petitioners-appellants, vs. REGISTER OF


DEEDS OF MANILA, respondent-appellee.

Ramon C. Fernandez for petitioner-appellants.


Solicitor General for respondent-appellee.

SYLLABUS

1. CORPORATIONS; LIQUIDATION AND DISTRIBUTION OF ASSETS FOR TRANSFER TO


STOCKHOLDERS; CERTIFICATE OF LIQUIDATION IN THE NATURE OF TRANSFER OR CONVEYANCE. — Where
the purpose of the liquidation, as well as the distribution of the assets of the corporation, is to transfer their title from the
corporation to the stockholders in proportion to their shareholdings, that transfer cannot be affected without the
corresponding deed of conveyance from the corporation to the stockholders, and the certificate should be considered as
one in the nature of a transfer or conveyance.

DECISION

BAUTISTA ANGELO, J p:
On September 19, 1960, the five stockholders of the F. Guanzon and Sons, Inc. executed a certificate of
liquidation of the assets of the corporation reciting, among other things, that by virtue of a resolution of the stockholders
adopted on September 17, 1960, dissolving the corporation, they have distributed among themselves in proportion to
their shareholdings, as liquidating dividends, the assets of said corporation, including real properties located in Manila.
The certificate of liquidation, when presented to the Register of Deeds of Manila, was denied registration on
seven grounds, of which the following were disputed by the stockholders:
"3. The number of parcels not certified to in the acknowledgment;
"5. P430.50 Reg. fees need be paid;
"6. P940.45 documentary stamps need be attached to the document;
"7. The judgment of the Court approving the dissolution and directing the disposition of the assets of the
corporation need be presented (Rules of Court, Rule 104, Sec. 3)."
Deciding the consulta elevated by the stockholders, the Commissioner of Land Registration overruled ground
No. 7 and sustained requirements Nos. 3, 5 and 6.
The stockholders interposed the present appeal.
As correctly stated by the Commissioner of Land Registration, the propriety or impropriety of the three grounds
on which the denial of the registration of the certificate of liquidation was predicated hinges on whether or not that
certificate merely involves a distribution of the corporation assets or should be considered a transfer or conveyance.
Appellants contend that the certificate of liquidation is not a conveyance or transfer but merely a distribution of
the assets of the corporation which has ceased to exist for having been dissolved. This is apparent in the minutes of
dissolution attached to the document. Not being a conveyance the certificate need not contain a statement of the
numbers of parcels of land involved in the distribution in the acknowledgment appearing therein. Hence the amount of
documentary stamps to be affixed thereon should only be P0.30 and not P940.45, as required by the register of deeds.
Neither is it correct to require appellants to pay the amount of P430.50 as registration fee.
The Commissioner of Land Registration, however, entertained a different opinion. He concurred in the view
expressed by the register of deeds to the effect that the certificate of liquidation in question, though it involves a
distribution of the corporation's assets, in the last analysis represents a transfer of said assets from the corporation to
the stockholders. Hence, in substance it is a transfer or conveyance.
We agree with the opinion of these two officials. A corporation is a juridical person distinct from the members
composing it. Properties registered in the name of the corporation are owned by it as an entity separate and distinct
from its members. While shares of stock constitute personal property, they do not represent property of the corporation.
The corporation has property of its own which consists chiefly of real estate (Nelson vs. Owen, 113 Ala., 372, 21 So. 75;
Morrow vs. Gould, 145 Iowa 1, 123 N. W. 743). A share of stock only typifies an aliquot part of the corporation's
property, or the right to share in its proceeds to that extent when distributed according to law and equity (Hall & Faley
vs. Alabama Terminal, 173 Ala., 398, 56 So., 235), but its holder is not the owner of any part of the capital of the
corporation (Bradley vs. Bauder, 36 Ohio St., 28). Nor is he entitled to the possession of any definite portion of its
property or assets (Gottfried vs. Miller, 104 U.S., 521; Jones vs. Davis, 35 Ohio St., 474). The stockholder is not a co-
owner or tenant in common of the corporate property (Harton vs. Johnston, 166 Ala., 317, 51 So., 992).
On the basis of the foregoing authorities, it is clear that the act of liquidation made by the stockholders of the F.
Guanzon and Sons, Inc. of the latter's assets is not and cannot be considered a partition of community property, but
rather a transfer or conveyance of the title of its assets to the individual stockholders. Indeed, since the purpose of the
liquidation, as well as the distribution of the assets of the corporation, is to transfer their title from the corporation to the
stockholders in proportion to their shareholdings, — and this is in effect the purpose which they seek to obtain from the
Register of Deeds of Manila, — that transfer cannot be effected without the corresponding deed of conveyance from the
corporation to the stockholders. It is, therefore, fair and logical to consider the certificate of liquidation as one in the
nature of a transfer or conveyance.
WHEREFORE, we affirm the resolution appealed from, with costs against appellants.
||| (Stockholders of Guanzon v. Register of Deeds of Manila, G.R. No. L-18216, [October 30, 1962], 116 PHIL 689-692)

[G.R. No. L-48627. June 30, 1987.]


FERMIN Z. CARAM, JR. and ROSA O. DE CARAM, petitioner, vs. THE HONORABLE COURT OF
APPEALS and ALBERTO V. ARELLANO, respondents.

DECISION

CRUZ, J p:

We gave limited due course to this petition on the question of the solidary liability of the petitioners with their co-defendants
in the lower court 1 because of the challenge to the following paragraph in the dispositive portion of the decision of the
respondent court: **
"1. Defendants are hereby ordered to jointly and severally pay the plaintiff the amount of P50,000.00 for
the preparation of the project study and his technical services that led to the organization of the
defendant corporation, plus P10,000.00 attorney's fees;" 2
The petitioners claim that this order has no support in fact and law because they had no contract whatsoever with the
private respondent regarding the above-mentioned services. Their position is that as mere subsequent investors in the
corporation that was later created, they should not be held solidarily liable with the Filipinas Orient Airways, a separate
juridical entity, and with Barretto and Garcia, their co-defendants in the lower court, *** who were the ones who requested
the said services from the private respondent. 3
We are not concerned here with the petitioners' co-defendants, who have not appealed the decision of the respondent court
and may, for this reason, be presumed to have accepted the same. For purposes of resolving this case before us, it is not
necessary to determine whether it is the promoters of the proposed corporation, or the corporation itself after its
organization, that shall be responsible for the expenses incurred in connection with such organization.
The only question we have to decide now is whether or not the petitioners themselves are also and personally liable for
such expenses and, if so, to what extent.
The reasons for the said order are given by the respondent court in its decision in this wise:
"As to the 4th assigned error we hold that as to the remuneration due the plaintiff for the preparation of
the project study and the pre-organizational services in the amount of P50,000.00, not only the defendant
corporation but the other defendants including defendants Caram should he jointly and severally liable for
this amount. As we above related it was upon the request of defendants Barretto and Garcia that plaintiff
handled the preparation of the project study which project study was presented to defendant Caram so
the latter was convinced to invest in the proposed airlines. The project study was revised for purposes of
presentation to financiers and the banks. It was on the basis of this study that defendant corporation was
actually organized and rendered operational. Defendants Garcia and Caram, and Barretto became
members of the Board and/or officers of defendant corporation. Thus, not only the defendant corporation
but all the other defendants who were involved in the preparatory stages of the incorporation, who
caused the preparation and/or benefited from the project study and the technical services of plaintiff must
be liable." 4
It would appear from the above justification that the petitioners were not really involved in the initial steps that finally led to
the incorporation of the Filipinas Orient Airways. Elsewhere in the decision, Barretto was described as "the moving spirit."
The finding of the respondent court is that the project study was undertaken by the private respondent at the request of
Barretto and Garcia who, upon its completion, presented it to the petitioners to induce them to invest in the proposed airline.
The study could have been presented to other prospective investors. At any rate, the airline was eventually organized on
the basis of the project study with the petitioners as major stockholders and, together with Barretto and Garcia, as principal
officers.
The following portion of the decision in question is also worth considering:
". . .. Since defendant Barretto was the moving spirit in the pre-organization work of defendant
corporation based on his experience and expertise, hence he was logically compensated in the amount
of P200,000.00 shares of stock not as industrial partner but more for is technical services that brought to
fruition the defendant corporation. By the same token, We find no reason why the plaintiff should not be
similarly compensated not only for having actively participated in the preparation of the project study for
several months and its subsequent revision but also in his having been involved in the pre-organization of
the defendant corporation, in the preparation of the franchise, in inviting the interest of the financiers and
in the training and screening of personnel. We agree that for these special services of the plaintiff the
amount of P50,000.00 as compensation is reasonable." 5
The above finding bolsters the conclusion that the petitioners were not involved in the initial stages of the organization of the
airline, which were being directed by Barretto as the main promoter. It was he who was putting all the pieces together, so to
speak. The petitioners were merely among the financiers whose interest was to be invited and who were in fact persuaded,
on the strength of the project study, to invest in the proposed airline.
Significantly, there was no showing that the Filipinas Orient Airways was a fictitious corporation and did not have a separate
juridical personality, to justify making the petitioners, as principal stockholders thereof, responsible for its obligations. As a
bona fide corporation, the Filipinas Orient Airways should alone be liable for its corporate acts as duly authorized by its
officers and directors.
In the light of these circumstances, we hold that the petitioners cannot be held personally liable for the compensation
claimed by the private respondent for the services performed by him in the organization of the corporation. To repeat, the
petitioners did not contract such services. It was only the results of such services that Barretto and Garcia presented to
them and which persuaded them to invest in the proposed airline. The most that can be said is that they benefited from such
services, but that surely is no justification to hold them personally liable therefor. Otherwise, all the other stockholders of the
corporation, including those who came in later, and regardless of the amount of their shareholdings, would be equally and
personally liable also with the petitioners for the claims of the private respondent.
The petition is rather hazy and seems to be flawed by an ambiguous ambivalence. Our impression is that it is opposed to
the imposition of solidary responsibility upon the Carams but seems to be willing, in a vague, unexpressed offer of
compromise, to accept joint liability. While it is true that it does here and there disclaim total liability, the thrust of the petition
seems to be against the imposition of solidary liability only rather than against any liability at all, which is what it should have
categorically argued.
Categorically, the Court holds that the petitioners are not liable at all, jointly or jointly and severally, under the first paragraph
of the dispositive portion of the challenged decision. So holding, we find it unnecessary to examine at this time the rules on
solidary obligations, which the parties — needlessly, as it turns out — have belabored unto death.
WHEREFORE, the petition is granted. The petitioners are declared not liable under the challenged decision, which is
hereby modified accordingly. It is so ordered.
||| (Caram, Jr. v. Court of Appeals, G.R. No. L-48627, [June 30, 1987], 235 PHIL 369-374)

[G.R. No. 124293. January 31, 2005.]

J.G. SUMMIT HOLDINGS, INC., petitioner,vs.COURT OF APPEALS; COMMITTEE ON


PRIVATIZATION, its Chairman and Members; ASSET PRIVATIZATION TRUST; and PHILYARDS
HOLDINGS, INC., respondents.

RESOLUTION

PUNO, J p:

For resolution before this Court are two motions filed by the petitioner, J.G. Summit Holdings, Inc. for
reconsideration of our Resolution dated September 24, 2003 and to elevate this case to the Court En Banc.The petitioner
questions the Resolution which reversed our Decision of November 20, 2000, which in turn reversed and set aside a
Decision of the Court of Appeals promulgated on July 18, 1995.
I. Facts
The undisputed facts of the case, as set forth in our Resolution of September 24, 2003, are as follows:
On January 27, 1997, the National Investment and Development Corporation (NIDC),a
government corporation, entered into a Joint Venture Agreement (JVA) with Kawasaki Heavy Industries,
Ltd. of Kobe, Japan (KAWASAKI) for the construction, operation and management of the Subic National
Shipyard, Inc. (SNS) which subsequently became the Philippine Shipyard and Engineering Corporation
(PHILSECO).Under the JVA, the NIDC and KAWASAKI will contribute P330 million for the capitalization
of PHILSECO in the proportion of 60%-40% respectively. One of its salient features is the grant to the
parties of the right of first refusal should either of them decide to sell, assign or transfer its interest in the
joint venture, viz:
1.4 Neither party shall sell, transfer or assign all or any part of its interest in SNS
[PHILSECO] to any third party without giving the other under the same terms the right of
first refusal. This provision shall not apply if the transferee is a corporation owned or
controlled by the GOVERNMENT or by a KAWASAKI affiliate.
On November 25, 1986, NIDC transferred all its rights, title and interest in PHILSECO to the
Philippine National Bank (PNB). Such interests were subsequently transferred to the National
Government pursuant to Administrative Order No. 14. On December 8, 1986, President Corazon C.
Aquino issued Proclamation No. 50 establishing the Committee on Privatization (COP) and the Asset
Privatization Trust (APT) to take title to, and possession of, conserve, manage and dispose of non-
performing assets of the National Government. Thereafter, on February 27, 1987, a trust agreement was
entered into between the National Government and the APT wherein the latter was named the trustee of
the National Government's share in PHILSECO. In 1989, as a result of a quasi-reorganization of
PHILSECO to settle its huge obligations to PNB, the National Government's shareholdings in PHILSECO
increased to 97.41% thereby reducing KAWASAKI's shareholdings to 2.59%. CAacTH
In the interest of the national economy and the government, the COP and the APT deemed it
best to sell the National Government's share in PHILSECO to private entities. After a series of
negotiations between the APT and KAWASAKI, they agreed that the latter's right of first refusal under the
JVA be "exchanged" for the right to top by five percent (5%) the highest bid for the said shares. They
further agreed that KAWASAKI would be entitled to name a company in which it was a stockholder, which
could exercise the right to top. On September 7, 1990, KAWASAKI informed APT that Philyards
Holdings, Inc. (PHI) 1 would exercise its right to top.
At the pre-bidding conference held on September 18, 1993, interested bidders were given copies
of the JVA between NIDC and KAWASAKI, and of the Asset Specific Bidding Rules (ASBR) drafted for
the National Government's 87.6% equity share in PHILSECO. The provisions of the ASBR were
explained to the interested bidders who were notified that the bidding would be held on December 2,
1993. A portion of the ASBR reads:
1.0 The subject of this Asset Privatization Trust (APT) sale through public
bidding is the National Government's equity in PHILSECO consisting of 896,869,942
shares of stock (representing 87.67% of PHILSECO's outstanding capital stock),which
will be sold as a whole block in accordance with the rules herein enumerated.
xxx xxx xxx
2.0 The highest bid, as well as the buyer, shall be subject to the final approval of
both the APT Board of Trustees and the Committee on Privatization (COP).
2.1 APT reserves the right in its sole discretion, to reject any or all bids.
3.0 This public bidding shall be on an Indicative Price Bidding basis. The
Indicative price set for the National Government's 87.67% equity in PHILSECO is
PESOS: ONE BILLION THREE HUNDRED MILLION (P1,300,000,000.00).
xxx xxx xxx
6.0 The highest qualified bid will be submitted to the APT Board of Trustees at
its regular meeting following the bidding, for the purpose of determining whether or not it
should be endorsed by the APT Board of Trustees to the COP, and the latter approves
the same. The APT shall advise Kawasaki Heavy Industries, Inc. and/or its nominee,
[PHILYARDS] Holdings, Inc.,that the highest bid is acceptable to the National
Government. Kawasaki Heavy Industries, Inc. and/or [PHILYARDS] Holdings, Inc. shall
then have a period of thirty (30) calendar days from the date of receipt of such advice
from APT within which to exercise their "Option to Top the Highest Bid" by offering a bid
equivalent to the highest bid plus five (5%) percent thereof.
6.1 Should Kawasaki Heavy Industries, Inc. and/or [PHILYARDS] Holdings, Inc.
exercise their "Option to Top the Highest Bid," they shall so notify the APT about such
exercise of their option and deposit with APT the amount equivalent to ten percent (10%)
of the highest bid plus five percent (5%) thereof within the thirty (30)-day period
mentioned in paragraph 6.0 above. APT will then serve notice upon Kawasaki Heavy
Industries, Inc. and/or [PHILYARDS] Holdings, Inc. declaring them as the preferred
bidder and they shall have a period of ninety (90) days from the receipt of the APT's
notice within which to pay the balance of their bid price. TIHDAa
6.2 Should Kawasaki Heavy Industries, Inc. and/or [PHILYARDS] Holdings, Inc.
fail to exercise their "Option to Top the Highest Bid" within the thirty (30)-day period, APT
will declare the highest bidder as the winning bidder.
xxx xxx xxx
12.0 The bidder shall be solely responsible for examining with appropriate care
these rules, the official bid forms, including any addenda or amendments thereto issued
during the bidding period. The bidder shall likewise be responsible for informing itself
with respect to any and all conditions concerning the PHILSECO Shares which may, in
any manner, affect the bidder's proposal. Failure on the part of the bidder to so examine
and inform itself shall be its sole risk and no relief for error or omission will be given by
APT or COP. ...
At the public bidding on the said date, petitioner J.G. Summit Holdings, Inc. 2 submitted a bid of
Two Billion and Thirty Million Pesos (P2,030,000,000.00) with an acknowledgment of
KAWASAKI/[PHILYARDS'] right to top, viz:
4. I/We understand that the Committee on Privatization (COP) has up to thirty
(30) days to act on APT's recommendation based on the result of this
bidding. Should the COP approve the highest bid, APT shall advise
Kawasaki Heavy Industries, Inc. and/or its nominee, [PHILYARDS]
Holdings, Inc. that the highest bid is acceptable to the National
Government. Kawasaki Heavy Industries, Inc. and/or [PHILYARDS]
Holdings, Inc. shall then have a period of thirty (30) calendar days from
the date of receipt of such advice from APT within which to exercise
their "Option to Top the Highest Bid" by offering a bid equivalent to the
highest bid plus five (5%) percent thereof.
As petitioner was declared the highest bidder, the COP approved the sale on December 3, 1993
"subject to the right of Kawasaki Heavy Industries, Inc./[PHILYARDS] Holdings, Inc. to top JGSMI's bid
by 5% as specified in the bidding rules."
On December 29, 1993, petitioner informed APT that it was protesting the offer of PHI to top its
bid on the grounds that: (a) the KAWASAKI/PHI consortium composed of KAWASAKI,
[PHILYARDS],Mitsui, Keppel, SM Group, ICTSI and Insular Life violated the ASBR because the last four
(4) companies were the losing bidders thereby circumventing the law and prejudicing the weak winning
bidder; (b) only KAWASAKI could exercise the right to top; (c) giving the same option to top to PHI
constituted unwarranted benefit to a third party; (d) no right of first refusal can be exercised in a public
bidding or auction sale; and (e) the JG Summit consortium was not estopped from questioning the
proceedings.
On February 2, 1994, petitioner was notified that PHI had fully paid the balance of the purchase
price of the subject bidding. On February 7, 1994, the APT notified petitioner that PHI had exercised its
option to top the highest bid and that the COP had approved the same on January 6, 1994. On February
24, 1994, the APT and PHI executed a Stock Purchase Agreement. Consequently, petitioner filed with
this Court a Petition for Mandamus under G.R. No. 114057. On May 11, 1994, said petition was referred
to the Court of Appeals. On July 18, 1995, the Court of Appeals denied the same for lack of merit. It ruled
that the petition for mandamus was not the proper remedy to question the constitutionality or legality of
the right of first refusal and the right to top that was exercised by KAWASAKI/PHI, and that the matter
must be brought "by the proper party in the proper forum at the proper time and threshed out in a full
blown trial." The Court of Appeals further ruled that the right of first refusal and the right to top are prima
facie legal and that the petitioner, "by participating in the public bidding, with full knowledge of the right to
top granted to KAWASAKI/[PHILYARDS] is ...estopped from questioning the validity of the award given to
[PHILYARDS] after the latter exercised the right to top and had paid in full the purchase price of the
subject shares, pursuant to the ASBR." Petitioner filed a Motion for Reconsideration of said Decision
which was denied on March 15, 1996. Petitioner thus filed a Petition for Certiorari with this Court alleging
grave abuse of discretion on the part of the appellate court. HITEaS

On November 20, 2000, this Court rendered ...[a] Decision ruling among others that the Court of
Appeals erred when it dismissed the petition on the sole ground of the impropriety of the special civil
action of mandamus because the petition was also one of certiorari.It further ruled that a shipyard like
PHILSECO is a public utility whose capitalization must be sixty percent (60%) Filipino-owned.
Consequently, the right to top granted to KAWASAKI under the Asset Specific Bidding Rules (ASBR)
drafted for the sale of the 87.67% equity of the National Government in PHILSECO is illegal — not only
because it violates the rules on competitive bidding — but more so, because it allows foreign
corporations to own more than 40% equity in the shipyard. It also held that "although the petitioner had
the opportunity to examine the ASBR before it participated in the bidding, it cannot be estopped from
questioning the unconstitutional, illegal and inequitable provisions thereof." Thus, this Court voided the
transfer of the national government's 87.67% share in PHILSECO to Philyard[s] Holdings, Inc.,and
upheld the right of JG Summit, as the highest bidder, to take title to the said shares, viz:
WHEREFORE, the instant petition for review on certiorari is GRANTED. The
assailed Decision and Resolution of the Court of Appeals are REVERSED and SET
ASIDE. Petitioner is ordered to pay to APT its bid price of Two Billion Thirty Million
Pesos (P2,030,000,000.00),less its bid deposit plus interests upon the finality of this
Decision. In turn, APT is ordered to:
(a) accept the said amount of P2,030,000,000.00 less bid deposit and interests
from petitioner;
(b) execute a Stock Purchase Agreement with petitioner;
(c) cause the issuance in favor of petitioner of the certificates of stocks
representing 87.6% of PHILSECO's total capitalization;
(d) return to private respondent PHGI the amount of Two Billion One Hundred
Thirty-One Million Five Hundred Thousand Pesos
(P2,131,500,000.00);and
(e) cause the cancellation of the stock certificates issued to PHI.
SO ORDERED.
In separate Motions for Reconsideration, respondents submit[ted] three basic issues
for ...resolution: (1) Whether PHILSECO is a public utility; (2) Whether under the 1977 JVA, KAWASAKI
can exercise its right of first refusal only up to 40% of the total capitalization of PHILSECO; and (3)
Whether the right to top granted to KAWASAKI violates the principles of competitive bidding. 3 (citations
omitted)
In a Resolution dated September 24, 2003, this Court ruled in favor of the respondents. On the first issue, we held
that Philippine Shipyard and Engineering Corporation (PHILSECO) is not a public utility, as by nature, a shipyard is not a
public utility 4 and that no law declares a shipyard to be a public utility. 5 On the second issue, we found nothing in the 1977
Joint Venture Agreement (JVA) which prevents Kawasaki Heavy Industries, Ltd. of Kobe, Japan (KAWASAKI) from
acquiring more than 40% of PHILSECO's total capitalization. 6 On the final issue, we held that the right to top granted to
KAWASAKI in exchange for its right of first refusal did not violate the principles of competitive bidding. 7
On October 20, 2003, the petitioner filed a Motion for Reconsideration 8 and a Motion to Elevate This Case to the
Court En Banc. 9 Public respondents Committee on Privatization (COP) and Asset Privatization Trust (APT), and private
respondent Philyards Holdings, Inc. (PHILYARDS) filed their Comments on J.G. Summit Holdings, Inc.'s (JG Summit's)
Motion for Reconsideration and Motion to Elevate This Case to the Court En Banc on January 29, 2004 and February 3,
2004, respectively. DCcHIS
II. Issues
Based on the foregoing, the relevant issues to resolve to end this litigation are the following:
1. Whether there are sufficient bases to elevate the case at bar to the Court en banc.
2. Whether the motion for reconsideration raises any new matter or cogent reason to warrant a reconsideration of
this Court's Resolution of September 24, 2003.
Motion to Elevate this Case to the
Court En Banc
The petitioner prays for the elevation of the case to the Court en banc on the following grounds:
1. The main issue of the propriety of the bidding process involved in the present case has been confused with the
policy issue of the supposed fate of the shipping industry which has never been an issue that is determinative of this case.
10
2. The present case may be considered under the Supreme Court Resolution dated February 23, 1984 which
included among en banc cases those involving a novel question of law and those where a doctrine or principle laid down by
the Court en banc or in division may be modified or reversed. 11
3. There was clear executive interference in the judicial functions of the Court when the Honorable Justice Jose
Isidro Camacho, Secretary of Finance, forwarded to Chief Justice Davide, a memorandum dated November 5, 2001,
attaching a copy of the Foreign Chambers Report dated October 17, 2001, which matter was placed in the agenda of the
Court and noted by it in a formal resolution dated November 28, 2001. 12
Opposing J.G. Summit's motion to elevate the case en banc,PHILYARDS points out the petitioner's inconsistency in
previously opposing PHILYARDS' Motion to Refer the Case to the Court En Banc.PHILYARDS contends that J.G. Summit
should now be estopped from asking that the case be referred to the Court en banc.PHILYARDS further contends that the
Supreme Court en banc is not an appellate court to which decisions or resolutions of its divisions may be appealed citing
Supreme Court Circular No. 2-89 dated February 7, 1989. 13 PHILYARDS also alleges that there is no novel question of
law involved in the present case as the assailed Resolution was based on well-settled jurisprudence. Likewise, PHILYARDS
stresses that the Resolution was merely an outcome of the motions for reconsideration filed by it and the COP and APT and
is "consistent with the inherent power of courts to 'amend and control its process and orders so as to make them
conformable to law and justice.' (Rule 135, sec. 5)" 14 Private respondent belittles the petitioner's allegations regarding the
change in ponente and the alleged executive interference as shown by former Secretary of Finance Jose Isidro Camacho's
memorandum dated November 5, 2001 arguing that these do not justify a referral of the present case to the Court en banc.
CDcaSA
In insisting that its Motion to Elevate This Case to the Court En Banc should be granted, J.G. Summit further argued
that: its Opposition to the Office of the Solicitor General's Motion to Refer is different from its own Motion to Elevate;
different grounds are invoked by the two motions; there was unwarranted "executive interference";and the change in
ponente is merely noted in asserting that this case should be decided by the Court en banc. 15
We find no merit in petitioner's contention that the propriety of the bidding process involved in the present case has
been confused with the policy issue of the fate of the shipping industry which, petitioner maintains, has never been an issue
that is determinative of this case. The Court's Resolution of September 24, 2003 reveals a clear and definitive ruling on the
propriety of the bidding process. In discussing whether the right to top granted to KAWASAKI in exchange for its right of first
refusal violates the principles of competitive bidding, we made an exhaustive discourse on the rules and principles of public
bidding and whether they were complied with in the case at bar. 16 This Court categorically ruled on the petitioner's
argument that PHILSECO, as a shipyard, is a public utility which should maintain a 60%-40% Filipino-foreign equity ratio, as
it was a pivotal issue. In doing so, we recognized the impact of our ruling on the shipbuilding industry which was beyond
avoidance. 17
We reject petitioner's argument that the present case may be considered under the Supreme Court Resolution
dated February 23, 1984 which included among en banc cases those involving a novel question of law and those where a
doctrine or principle laid down by the court en banc or in division may be modified or reversed. The case was resolved
based on basic principles of the right of first refusal in commercial law and estoppel in civil law. Contractual obligations
arising from rights of first refusal are not new in this jurisdiction and have been recognized in numerous cases. 18 Estoppel
is too known a civil law concept to require an elongated discussion. Fundamental principles on public bidding were likewise
used to resolve the issues raised by the petitioner. To be sure, petitioner leans on the right to top in a public bidding in
arguing that the case at bar involves a novel issue. We are not swayed. The right to top was merely a condition or a
reservation made in the bidding rules which was fully disclosed to all bidding parties. In Bureau Veritas, represented by
Theodor H. Hunermann v. Office of the President, et al., 19 we dealt with this conditionality, viz:
...It must be stressed, as held in the case of A.C. Esguerra & Sons v. Aytona, et al.,(L-18751, 28
April 1962, 4 SCRA 1245),that in an "invitation to bid, there is a condition imposed upon the bidders to
the effect that the bidding shall be subject to the right of the government to reject any and all bids subject
to its discretion. In the case at bar, the government has made its choice and unless an unfairness or
injustice is shown, the losing bidders have no cause to complain nor right to dispute that choice. This is a
well-settled doctrine in this jurisdiction and elsewhere."
The discretion to accept or reject a bid and award contracts is vested in the Government
agencies entrusted with that function. The discretion given to the authorities on this matter is of such wide
latitude that the Courts will not interfere therewith, unless it is apparent that it is used as a shield to a
fraudulent award (Jalandoni v. NARRA,108 Phil. 486 [1960])....The exercise of this discretion is a policy
decision that necessitates prior inquiry, investigation, comparison, evaluation, and deliberation. This task
can best be discharged by the Government agencies concerned, not by the Courts. The role of the
Courts is to ascertain whether a branch or instrumentality of the Government has transgressed its
constitutional boundaries. But the Courts will not interfere with executive or legislative discretion
exercised within those boundaries. Otherwise, it strays into the realm of policy decision-making.

It is only upon a clear showing of grave abuse of discretion that the Courts will set aside the
award of a contract made by a government entity. Grave abuse of discretion implies a capricious,
arbitrary and whimsical exercise of power (Filinvest Credit Corp. v. Intermediate Appellate Court,No.
65935, 30 September 1988, 166 SCRA 155).The abuse of discretion must be so patent and gross as to
amount to an evasion of positive duty or to a virtual refusal to perform a duty enjoined by law, as to act at
all in contemplation of law, where the power is exercised in an arbitrary and despotic manner by reason
of passion or hostility (Litton Mills, Inc. v. Galleon Trader, Inc.,et al[.],L-40867, 26 July 1988, 163 SCRA
489).
The facts in this case do not indicate any such grave abuse of discretion on the part of public
respondents when they awarded the CISS contract to Respondent SGS. In the "Invitation to Prequalify
and Bid" (Annex "C," supra),the CISS Committee made an express reservation of the right of the
Government to "reject any or all bids or any part thereof or waive any defects contained thereon and
accept an offer most advantageous to the Government." It is a well-settled rule that where such
reservation is made in an Invitation to Bid, the highest or lowest bidder, as the case may be, is not
entitled to an award as a matter of right (C & C Commercial Corp. v. Menor,L-28360, 27 January 1983,
120 SCRA 112).Even the lowest Bid or any Bid may be rejected or, in the exercise of sound discretion,
the award may be made to another than the lowest bidder (A.C. Esguerra & Sons v. Aytona, supra,citing
43 Am. Jur.,788).(emphases supplied) STaCIA
Like the condition in the Bureau Veritas case, the right to top was a condition imposed by the government in the bidding
rules which was made known to all parties. It was a condition imposed on all bidders equally, based on the APT's
exercise of its discretion in deciding on how best to privatize the government's shares in PHILSECO.It was not a
whimsical or arbitrary condition plucked from the ether and inserted in the bidding rules but a condition which the APT
approved as the best way the government could comply with its contractual obligations to KAWASAKI under the JVA
and its mandate of getting the most advantageous deal for the government. The right to top had its history in the mutual
right of first refusal in the JVA and was reached by agreement of the government and KAWASAKI.
Further, there is no "executive interference" in the functions of this Court by the mere filing of a memorandum by
Secretary of Finance Jose Isidro Camacho. The memorandum was merely "noted" to acknowledge its filing. It had no
further legal significance. Notably too, the assailed Resolution dated September 24, 2003 was decided unanimously by the
Special First Division in favor of the respondents.
Again, we emphasize that a decision or resolution of a Division is that of the Supreme Court 20 and the Court en
banc is not an appellate court to which decisions or resolutions of a Division may be appealed. 21
For all the foregoing reasons, we find no basis to elevate this case to the Court en banc.
Motion for Reconsideration
Three principal arguments were raised in the petitioner's Motion for Reconsideration. First, that a fair resolution of
the case should be based on contract law, not on policy considerations; the contracts do not authorize the right to top to be
derived from the right of first refusal. 22 Second, that neither the right of first refusal nor the right to top can be legally
exercised by the consortium which is not the proper party granted such right under either the JVA or the Asset Specific
Bidding Rules (ASBR). 23 Third, that the maintenance of the 60%-40% relationship between the National Investment and
Development Corporation (NIDC) and KAWASAKI arises from contract and from the Constitution because PHILSECO is a
landholding corporation and need not be a public utility to be bound by the 60%-40% constitutional limitation. 24
On the other hand, private respondent PHILYARDS asserts that J.G. Summit has not been able to show compelling
reasons to warrant a reconsideration of the Decision of the Court. 25 PHILYARDS denies that the Decision is based mainly
on policy considerations and points out that it is premised on principles governing obligations and contracts and corporate
law such as the rule requiring respect for contractual stipulations, upholding rights of first refusal, and recognizing the
assignable nature of contracts rights. 26 Also, the ruling that shipyards are not public utilities relies on established case law
and fundamental rules of statutory construction. PHILYARDS stresses that KAWASAKI's right of first refusal or even the
right to top is not limited to the 40% equity of the latter. 27 On the landholding issue raised by J.G. Summit, PHILYARDS
emphasizes that this is a non-issue and even involves a question of fact. Even assuming that this Court can take
cognizance of such question of fact even without the benefit of a trial, PHILYARDS opines that landholding by PHILSECO at
the time of the bidding is irrelevant because what is essential is that ultimately a qualified entity would eventually hold
PHILSECO's real estate properties. 28 Further, given the assignable nature of the right of first refusal, any applicable
nationality restrictions, including landholding limitations, would not affect the right of first refusal itself, but only the manner of
its exercise. 29 Also, PHILYARDS argues that if this Court takes cognizance of J.G. Summit's allegations of fact regarding
PHILSECO's landholding, it must also recognize PHILYARDS' assertions that PHILSECO's landholdings were sold to
another corporation. 30 As regards the right of first refusal, private respondent explains that KAWASAKI's reduced
shareholdings (from 40% to 2.59%) did not translate to a deprivation or loss of its contractually granted right of first refusal.
31 Also, the bidding was valid because PHILYARDS exercised the right to top and it was of no moment that losing bidders
later joined PHILYARDS in raising the purchase price. 32
In cadence with the private respondent PHILYARDS, public respondents COP and APT contend:
1. The conversion of the right of first refusal into a right to top by 5% does not violate any provision in the JVA
between NIDC and KAWASAKI. ADcHES
2. PHILSECO is not a public utility and therefore not governed by the constitutional restriction on foreign ownership.
3. The petitioner is legally estopped from assailing the validity of the proceedings of the public bidding as it
voluntarily submitted itself to the terms of the ASBR which included the provision on the right to top.
4. The right to top was exercised by PHILYARDS as the nominee of KAWASAKI and the fact that PHILYARDS
formed a consortium to raise the required amount to exercise the right to top the highest bid by 5% does not violate the JVA
or the ASBR.
5. The 60%-40% Filipino-foreign constitutional requirement for the acquisition of lands does not apply to PHILSECO
because as admitted by petitioner itself, PHILSECO no longer owns real property.
6. Petitioner's motion to elevate the case to the Court en banc is baseless and would only delay the termination of
this case. 33
In a Consolidated Comment dated March 8, 2004, J.G. Summit countered the arguments of the public and private
respondents in this wise:
1. The award by the APT of 87.67% shares of PHILSECO to PHILYARDS with losing bidders
through the exercise of a right to top, which is contrary to law and the constitution is null
and void for being violative of substantive due process and the abuse of right provision in
the Civil Code.
a. The bidders['] right to top was actually exercised by losing bidders.
b. The right to top or the right of first refusal cannot co-exist with a genuine competitive
bidding.
c. The benefits derived from the right to top were unwarranted.
2. The landholding issue has been a legitimate issue since the start of this case but is
shamelessly ignored by the respondents.
a. The landholding issue is not a non-issue.
b. The landholding issue does not pose questions of fact.
c. That PHILSECO owned land at the time that the right of first refusal was agreed upon
and at the time of the bidding are most relevant.
d. Whether a shipyard is a public utility is not the core issue in this case.
3. Fraud and bad faith attend the alleged conversion of an inexistent right of first refusal to the
right to top.
a. The history behind the birth of the right to top shows fraud and bad faith.
b. The right of first refusal was, indeed, "effectively useless."
4. Petitioner is not legally estopped to challenge the right to top in this case.
a. Estoppel is unavailing as it would stamp validity to an act that is prohibited by law or
against public policy.
b. Deception was patent; the right to top was an attractive nuisance.
c. The 10% bid deposit was placed in escrow.
J.G. Summit's insistence that the right to top cannot be sourced from the right of first refusal is not new and we
have already ruled on the issue in our Resolution of September 24, 2003. We upheld the mutual right of first refusal in the
JVA. 34 We also ruled that nothing in the JVA prevents KAWASAKI from acquiring more than 40% of PHILSECO's total
capitalization. 35 Likewise, nothing in the JVA or ASBR bars the conversion of the right of first refusal to the right to top. In
sum, nothing new and of significance in the petitioner's pleading warrants a reconsideration of our ruling. DAEaTS
Likewise, we already disposed of the argument that neither the right of first refusal nor the right to top can legally be
exercised by the consortium which is not the proper party granted such right under either the JVA or the ASBR. Thus, we
held:
The fact that the losing bidder, Keppel Consortium (composed of Keppel, SM Group, Insular Life
Assurance, Mitsui and ICTSI),has joined PHILYARDS in the latter's effort to raise P2.131 billion
necessary in exercising the right to top is not contrary to law, public policy or public morals. There is
nothing in the ASBR that bars the losing bidders from joining either the winning bidder (should the right to
top is not exercised) or KAWASAKI/PHI (should it exercise its right to top as it did),to raise the purchase
price. The petitioner did not allege, nor was it shown by competent evidence, that the participation of the
losing bidders in the public bidding was done with fraudulent intent. Absent any proof of fraud, the
formation by [PHILYARDS] of a consortium is legitimate in a free enterprise system. The appellate court
is thus correct in holding the petitioner estopped from questioning the validity of the transfer of the
National Government's shares in PHILSECO to respondent. 36

Further, we see no inherent illegality on PHILYARDS' act in seeking funding from parties who were losing bidders.
This is a purely commercial decision over which the State should not interfere absent any legal infirmity. It is emphasized
that the case at bar involves the disposition of shares in a corporation which the government sought to privatize. As such,
the persons with whom PHILYARDS desired to enter into business with in order to raise funds to purchase the shares are
basically its business. This is in contrast to a case involving a contract for the operation of or construction of a government
infrastructure where the identity of the buyer/bidder or financier constitutes an important consideration. In such cases, the
government would have to take utmost precaution to protect public interest by ensuring that the parties with which it is
contracting have the ability to satisfactorily construct or operate the infrastructure.
On the landholding issue, J.G. Summit submits that since PHILSECO is a landholding company, KAWASAKI could
exercise its right of first refusal only up to 40% of the shares of PHILSECO due to the constitutional prohibition on
landholding by corporations with more than 40% foreign-owned equity. It further argues that since KAWASAKI already held
at least 40% equity in PHILSECO, the right of first refusal was inutile and as such, could not subsequently be converted into
the right to top. 37 Petitioner also asserts that, at present, PHILSECO continues to violate the constitutional provision on
landholdings as its shares are more than 40% foreign-owned. 38 PHILYARDS admits that it may have previously held land
but had already divested such landholdings. 39 It contends, however, that even if PHILSECO owned land, this would not
affect the right of first refusal but only the exercise thereof. If the land is retained, the right of first refusal, being a property
right, could be assigned to a qualified party. In the alternative, the land could be divested before the exercise of the right of
first refusal. In the case at bar, respondents assert that since the right of first refusal was validly converted into a right to top,
which was exercised not by KAWASAKI, but by PHILYARDS which is a Filipino corporation (i.e., 60% of its shares are
owned by Filipinos), then there is no violation of the Constitution. 40 At first, it would seem that questions of fact beyond
cognizance by this Court were involved in the issue. However, the records show that PHILYARDS admits it had owned land
up until the time of the bidding. 41 Hence, the only issue is whether KAWASAKI had a valid right of first refusal over
PHILSECO shares under the JVA considering that PHILSECO owned land until the time of the bidding and KAWASAKI
already held 40% of PHILSECO's equity.
We uphold the validity of the mutual rights of first refusal under the JVA between KAWASAKI and NIDC. First of all,
the right of first refusal is a property right of PHILSECO shareholders, KAWASAKI and NIDC, under the terms of their JVA.
This right allows them to purchase the shares of their co-shareholder before they are offered to a third party. The agreement
of co-shareholders to mutually grant this right to each other, by itself, does not constitute a violation of the provisions of the
Constitution limiting land ownership to Filipinos and Filipino corporations.As PHILYARDS correctly puts it, if PHILSECO still
owns land, the right of first refusal can be validly assigned to a qualified Filipino entity in order to maintain the 60%-40%
ratio. This transfer, by itself, does not amount to a violation of the Anti-Dummy Laws, absent proof of any fraudulent intent.
The transfer could be made either to a nominee or such other party which the holder of the right of first refusal feels it can
comfortably do business with. Alternatively, PHILSECO may divest of its landholdings, in which case KAWASAKI, in
exercising its right of first refusal, can exceed 40% of PHILSECO's equity. In fact, it can even be said that if the foreign
shareholdings of a landholding corporation exceeds 40%,it is not the foreign stockholders' ownership of the shares which is
adversely affected but the capacity of the corporation to own land — that is, the corporation becomes disqualified to own
land.This finds support under the basic corporate law principle that the corporation and its stockholders are separate
juridical entities. In this vein, the right of first refusal over shares pertains to the shareholders whereas the capacity to own
land pertains to the corporation. Hence, the fact that PHILSECO owns land cannot deprive stockholders of their right of first
refusal. No law disqualifies a person from purchasing shares in a landholding corporation even if the latter will exceed the
allowed foreign equity, what the law disqualifies is the corporation from owning land. This is the clear import of the following
provisions in the Constitution:
Section 2. All lands of the public domain, waters, minerals, coal, petroleum, and other mineral
oils, all forces of potential energy, fisheries, forests or timber, wildlife, flora and fauna, and other natural
resources are owned by the State. With the exception of agricultural lands, all other natural resources
shall not be alienated. The exploration, development, and utilization of natural resources shall be under
the full control and supervision of the State. The State may directly undertake such activities, or it may
enter into co-production, joint venture, or production-sharing agreements with Filipino citizens, or
corporations or associations at least sixty per centum of whose capital is owned by such citizens.Such
agreements may be for a period not exceeding twenty-five years, renewable for not more than twenty-five
years, and under such terms and conditions as may be provided by law. In cases of water rights for
irrigation, water supply, fisheries, or industrial uses other than the development of water power, beneficial
use may be the measure and limit of the grant. IcAaEH
xxx xxx xxx
Section 7. Save in cases of hereditary succession, no private lands shall be transferred or
conveyed except to individuals, corporations, or associations qualified to acquire or hold lands of the
public domain. 42 (emphases supplied)
The petitioner further argues that "an option to buy land is void in itself (Philippine Banking Corporation v. Lui She,
21 SCRA 52 [1967]). The right of first refusal granted to KAWASAKI, a Japanese corporation, is similarly void. Hence, the
right to top, sourced from the right of first refusal, is also void." 43 Contrary to the contention of petitioner, the case of Lui
She did not that say "an option to buy land is void in itself ," for we ruled as follows:
...To be sure, a lease to an alien for a reasonable period is valid. So is an option giving an alien
the right to buy real property on condition that he is granted Philippine citizenship.As this Court said in
Krivenko vs. Register of Deeds:
[A]liens are not completely excluded by the Constitution from the use of lands for
residential purposes. Since their residence in the Philippines is temporary, they may be
granted temporary rights such as a lease contract which is not forbidden by the
Constitution. Should they desire to remain here forever and share our fortunes and
misfortunes, Filipino citizenship is not impossible to acquire.
But if an alien is given not only a lease of, but also an option to buy, a piece of land, by virtue of
which the Filipino owner cannot sell or otherwise dispose of his property, this to last for 50 years, then it
becomes clear that the arrangement is a virtual transfer of ownership whereby the owner divests himself
in stages not only of the right to enjoy the land (jus possidendi, jus utendi, jus fruendi and jus abutendi)
but also of the right to dispose of it (jus disponendi) — rights the sum total of which make up ownership.It
is just as if today the possession is transferred, tomorrow, the use, the next day, the disposition, and so
on, until ultimately all the rights of which ownership is made up are consolidated in an alien. And yet this
is just exactly what the parties in this case did within this pace of one year, with the result that Justina
Santos'[s] ownership of her property was reduced to a hollow concept. If this can be done, then the
Constitutional ban against alien landholding in the Philippines, as announced in Krivenko vs. Register of
Deeds,is indeed in grave peril. 44 (emphases supplied; Citations omitted)
In Lui She,the option to buy was invalidated because it amounted to a virtual transfer of ownership as the owner could not
sell or dispose of his properties. The contract in Lui She prohibited the owner of the land from selling, donating, mortgaging,
or encumbering the property during the 50-year period of the option to buy. This is not so in the case at bar where the
mutual right of first refusal in favor of NIDC and KAWASAKI does not amount to a virtual transfer of land to a non-Filipino. In
fact, the case at bar involves a right of first refusal over shares of stock while the Lui She case involves an option to buy the
land itself. As discussed earlier, there is a distinction between the shareholder's ownership of shares and the corporation's
ownership of land arising from the separate juridical personalities of the corporation and its shareholders.
We note that in its Motion for Reconsideration, J.G. Summit alleges that PHILSECO continues to violate the
Constitution as its foreign equity is above 40% and yet owns long-term leasehold rights which are real rights. 45 It cites
Article 415 of the Civil Code which includes in the definition of immovable property, "contracts for public works, and
servitudes and other real rights over immovable property." 46 Any existing landholding, however, is denied by PHILYARDS
citing its recent financial statements. 47 First, these are questions of fact, the veracity of which would require introduction of
evidence. The Court needs to validate these factual allegations based on competent and reliable evidence. As such, the
Court cannot resolve the questions they pose. Second, J.G. Summit misreads the provisions of the Constitution cited in its
own pleadings, to wit:

29.2 Petitioner has consistently pointed out in the past that private respondent is not a 60%-40%
corporation, and this violates the Constitution . . . The violation continues to this day because under the
law, it continues to own real property...
xxx xxx xxx
32. To review the constitutional provisions involved, Section 14, Article XIV of the 1973
Constitution (the JVA was signed in 1977), provided:
"Save in cases of hereditary succession, no private lands shall be transferred or
conveyed except to individuals, corporations, or associations qualified to acquire or hold
lands of the public domain."
32.1 This provision is the same as Section 7, Article XII of the 1987 Constitution.
32.2 Under the Public Land Act, corporations qualified to acquire or hold lands of the
public domain are corporations at least 60% of which is owned by Filipino citizens (Sec.
22, Commonwealth Act 141, as amended). (emphases supplied) HSCAIT
As correctly observed by the public respondents, the prohibition in the Constitution applies only to ownership of land. 48
It does not extend to immovable or real property as defined under Article 415 of the Civil Code. Otherwise, we would
have a strange situation where the ownership of immovable property such as trees, plants and growing fruit attached to
the land 49 would be limited to Filipinos and Filipino corporations only.
III.
WHEREFORE, in view of the foregoing, the petitioner's Motion for Reconsideration is DENIED WITH FINALITY and
the decision appealed from is AFFIRMED. The Motion to Elevate This Case to the Court En Banc is likewise DENIED for
lack of merit. SO ORDERED.
||| (J.G. Summit Holdings Inc. v. Court of Appeals, G.R. No. 124293 (Resolution), [January 31, 2005], 490 PHIL 579-606)

[G.R. No. 111008. November 7, 1994.]

TRAMAT MERCANTILE, INC. AND DAVID ONG, petitioners, vs. HON. COURT OF APPEALS AND
MELCHOR DE LA CUESTA, respondents.

DECISION

VITUG, J p:

This petition for review on certiorari challenges the 04th March 1993 of the Court of Appeals and its resolution
of 01 July 1993 denying the motion for reconsideration. prcd
On 09 April 1984, Melchor de la Cuesta, doing business under the name and style of "Farmers Machineries,"
sold to Tramat Mercantile, Inc. (Tramat), one (1) unit HINOMOTO TRACTOR Model MB 1100D powered by a 13 H.P.
diesel engine. In payment, David Ong, Tramat's president and manager, issued a check for P33,500.00 (apparently
replacing an earlier postdated check for P33,080.00). Tramat, in turn, sold the tractor, together with an attached lawn
mower fabricated by it, to the Metropolitan Waterworks and Sewerage System ("NAWASA") for P67,000.00. David Ong
caused a stop payment of the check when NAWASA refused to pay the tractor and lawn mower after discovering that,
aside from some stated defects of the attached lawn mower, the engine (sold by de la Cuesta) was a reconditioned unit.
On 28 May 1985, de la Cuesta filed an action for the recovery of P33,500.00, as well as attorney's fees of
P10,000.00, and the costs of suit. Ong, in his answer, averred, among other things, that de la Cuesta had no cause of
action; that the questioned transaction was between plaintiff and Tramat Mercantile, Inc., and not with Ong in his
personal capacity; and that the payment of the check was stopped because the subject tractor had been priced as a
brand new, not as a reconditioned unit.
On 02 November 1989, after the reception of evidence, the trial court rendered a decision, the dispositive
portions of which read:
"WHEREFORE, in view of the foregoing consideration, judgment is hereby rendered:
"1. Ordering the defendants, jointly and severally, to pay the plaintiff the sum of P33,500.00 with
legal interest thereon at the rate of 12% per annum from July 7, 1984 until fully paid; and
"2. Ordering the defendants, jointly and severally, to pay the plaintiff the sum of P10,000.00 as
attorney's fees, and the costs of this suit.
SO ORDERED. 1
An appeal was timely interposed by the defendants. On 04 March 1993, the Court of Appeals affirmed in toto
the decision of the trial court. Defendant-appellants' motion for reconsideration was denied.
Hence, the instant petition.
We could find no reason to reverse the factual findings of both the trial court and the appellate court, particularly
in holding that the contract between de la Cuesta and TRAMAT was one of absolute, not conditional, sale of the tractor
and that de la Cuesta did not violate any warranty on the sale of the tractor to TRAMAT. The appellate court, in its
decision, adequately explained:
"If the perfection of the sale was dependent upon acceptance by the MWSS of the subject tractor
why did the appellants issue a check in payment of the item to the appellee? And long after MWSS had
complained about the defective tractor engine, and after the appellee had failed to remedy the defect,
why did the appellants still draw and deliver a replacement check to the appeal for the increased amount
of P33,500.00?
"These payments argue against the claim now made by the defendants that the sale was
conditional.
"According to the appellee, the additional amount covered the cost of replacing the oil gasket of
the tractor engine when it was repaired in Soledad Cac's gasoline station in Quezon City. The appellants,
on the other hand, claims the amount represented the freight charges for transporting the tractor from
Cauayan, Isabela to Metro Manila. cdrep
"The appellants should have explained why they failed to include the freight charges in the first
check. The tractor was transported from Isabela to Metro Manila as early as April 1984, and the first
check was drawn at about the same time. The freight charges cannot be said to have been incurred
when the tractor engine was delivered back to the supplier for repairs. The appellants admitted that the
engine was not brought back to Isabela. The repairs were done at Soledad Cac's gasoline station in
Quezon City.
"Anent the first assigned error, We sustain the trial court's finding that at the time of the
purchase, the appellants did not reveal to the appellee the true purpose for which the tractor would be
used. Granting that the appellants informed the appellee that they would be reselling the unit to the
MWSS, an entity admittedly not engaged in farming, and that they ordered the tractor without the power
tiller, an indispensable accessory if the tractor would be used in farming, these in themselves would not
constitute the required implied notice to the appellee as seller.
"xxx xxx xxx
"In regard to the second assigned error, We do not agree that the appellee should have been
held liable for the tractor's alleged hidden defects. . . .
"It has to be noted in this regard that, to satisfy the requirements of the MWSS, the appellants
borrowed a lawn mower from the MWSS so they could fabricate one such mower. The appellants'
witness stated that the kind of mid-mounted lawn mower was being manufactured by their competitor,
Alpha Machinery, which had by then stopped supplying the same (tsn, Nov. 29, 1988, pp. 73-74). There
is no showing that the appellants had had any previous experience in the fabrication of this lawn mower.
In fact, as aforesaid, they had to borrow one from the MWSS which they could copy. But although they
made a copy with the same specifications and design, there was no assurance that the copy would
function as well as with the model.
"xxx xxx xxx
"Although the trial court discussed it in a different light, We view the matter in the same way the
trial court did — that the lawn mower as fabricated by the appellants was the root of the parties'
problems.
"Having had no previous experience in the manufacture of lawn mowers of the same type as that
in litigation, and in a possibly patent-infringing effort to undercut their competition, the appellants
gathered enough daring to do the fabrication themselves. But the product might have proved too much
for the subject tractor to power, and the tractor's engine was strained beyond its limits, causing it to
overheat and damage its gaskets.
"No wonder, then, it was a gasket Soledad Cac had to replace, at a cost chargeable to the
appellants. No wonder, furthermore, the appellants' witness declared that even after the replacement of
that one gasket, the engine still leaked oil after being torture-tested. The integrity of the other engine
gaskets might have been impaired, too. Such was the burden placed on the engine. The engine
malfunctioned not necessarily because the engine, as alleged by the appellants, had been a
reconditioned, and not a brand new, one. It malfunctioned because it was made to do what it simply
could not. 2
It was, nevertheless, an error to hold David Ong jointly and severally liable with TRAMAT to de la Cuesta under
the questioned transaction. Ong had there so acted, not in his personal capacity, but as an officer of a corporation,
TRAMAT, with a distinct and separate personality. As such, it should only be the corporation, not the person acting for
and on its behalf, that properly could be made liable thereon. 3
Personal liability of a corporate director, trustee or officer along (although not necessarily) with the corporation
may so validly attach, as a rule, only when —
1. He assents (a) to a patently unlawful act of the corporation, or (b) for bad faith, or (c) for conflict of interest,
resulting in damages to the corporation, its stockholders or other persons; 4
2. He consents to the issuance of watered stocks or who, having knowledge thereof, does not forthwith file with
the corporate secretary his written objection thereto; 5
3. He agrees to hold himself personally and solidarily liable with the corporation; 6 or
4. He is made, by a specific provision of law, to personally answer for his corporate action. 7
In the case at bench, there is no indication that petitioner David Ong could be held personally accountable
under any of the abovementioned cases.
WHEREFORE, the petition is given DUE COURSE and the decision of the trial court, affirmed by the appellate
court, is MODIFIED insofar as it holds petitioner David Ong jointly and severally liable with Tramat Mercantile, Inc.,
which portion of the questioned judgment is SET ASIDE. In all other respects, the decision appealed from is
AFFIRMED. No costs. SO ORDERED.
||| (Tramat Mercantile, Inc. v. Court of Appeals, G.R. No. 111008, [November 7, 1994], 308 PHIL 13-18)

[G.R. No. L-5081. February 24, 1954.]

MARVEL BUILDING CORPORATION, ET AL., plaintiffs-appellees, vs. SATURNINO DAVID, in his


capacity as Collector, Bureau of Internal Revenue, defendant-appellant.

SYLLABUS

CORPORATIONS; CIRCUMSTANTIAL EVIDENCE SHOWING ONE-MAN CORPORATION. — the existence


of endorsed certificates discovered by internal revenue agents between 1948 and 1949 in the possession of the
Secretary-Treasurer of a supposed corporation; the fact that twenty-five certificates were signed by its president for no
justifiable reason; the fact that its principal stockholder had made enormous profits and, therefore, had a motive to hide
them to evade the payment of taxes; the fact that the other subscribers had no incomes of sufficient magnitude to justify
their big subscriptions; the fact that the treasurer in the name of the alleged corporation but were kept by the principal
stockholder herself; the fact that the stockholders or the directors never appeared to have ever met to discuss the
business of the corporation; the fact that she advanced big sums of money to the corporation without any previous
arrangement or accounting; and the fact that the books of accounts were kept as if they belonged to her alone — are
circumstantial evidence which are not only convincing but conclusive that she is the sole and exclusive owner of all the
shares of stock of the corporation and that the other partners are her dummies.

DECISION

LABRADOR, J p:

This action was brought by plaintiffs as stockholders of the Marvel Building Corporation to enjoin the defendant
Collector of Internal Revenue from selling at public auction various properties described in the complaint, including three
parcels of land, with the buildings situated thereon, known as the Aguinaldo Building, the Wise Building, and the Dewey
Boulevard-Padre Faura Mansion, all registered in the name of said corporation. Said properties were seized and
distrained by defendant to collect war profits taxes assessed against plaintiff Maria B. Castro (Exhibit B). Plaintiffs allege
that the said three properties (lands and buildings) belong to the Marvel Building corporation and not to Maria B. Castro,
while the defendant claims that Maria B. Castro is the true and sole owner of all the subscribed stock of the Marvel
Building Corporation, including those appearing to have been subscribed and paid for by the other members, and
consequently said Maria B. Castro is also the true and exclusive owner of the properties seized. The trial court held that
the evidence, which is mostly circumstantial, fails to show to its satisfaction that Maria B. Castro is the true owner of all
the stock certificates of the corporation, because the evidence is susceptible of two interpretations and an interpretation
may not be made which would deprive one of property without due process of law.
It appears that on September 15, 1950, the Secretary of Finance, upon consideration of the report of a special
committee assigned to study the war profits tax case of Mrs. Maria B. Castro, recommended the collection of
P3,593,950.78 as war profits taxes for the latter, and on September 22, 1953 the President instructed the Collector that
steps be taken to collect the same (Exhibits 114, 114-A to 114-D). Pursuant thereto various properties, including the
three above mentioned, were seized by the Collector of Internal Revenue on October 31, 1950. On November 13, 1950,
the original complaint in this case was filed. After trial, the Court of First Instance of Manila rendered judgment ordering
the release of the properties mentioned, and enjoined the Collector of Internal Revenue from selling the same. The
Collector of Internal Revenue has appealed to this Court against the judgment.
The following facts are not disputed, or are satisfactorily proved by the evidence:
The Articles of Incorporation of the Marvel Building Corporation is dated February 12, 1947 and according to it
the capital stock is P2,000,000, of which P1,025,000 was (at the time of incorporation) subscribed and paid by the
following incorporators:
Maria B. Castro -------- 250shares ------P250,000.00
Amado A. Yatco ------- 100" ------ 100,000.00
Santiago Tan ----------- 100" ------ 100,000.00
Jose T. Lopez ---------- 90" ------ 90,000.00
Benita Lamagna --------- 90" ------ 90,000.00
C.S. Gonzales ----------- 80" ------ 80,000.00
Maria Cristobal --------- 70" ------ 70,000.00
Segundo Esguerra, Sr. -- 75" ------ 75,000.00
Ramon Sangalang -------- 70" ------ 70,000.00
Maximo Cristobal ------- 55" ------ 55,000.00
Antonio Cristobal ------ 45" ------ 45,000.00
____________
P1,025,000.00.
Maria B. Castro was elected President and Maximo Cristobal, Secretary-Treasurer (Exhibit A).
The Wise Building was purchased on September 4, 1946, the purchase being made in the name of Dolores
Trinidad, wife of Amado A. Yatco (Exhibit V), and the Aguinaldo Building, on January 17, 1947, in the name of Segundo
Esguerra, Sr. (Exhibit M). Both building were purchased for P1,800,00, but as the corporation had only P1,025,000, the
balance of the purchase price was obtained as loans from the Insular Life Assurance Co., Ltd. and the Philippine
Guaranty Co., Inc. (Exhibit C).
Of the incorporators of the Marvel Building Corporation, Maximo Cristobal and Antonio Cristobal are half-
brothers of Maria B. Castro, Manila Cristobal is a half-sister, and Segundo Esguerra, Sr. a brother-in-law, husband of
Maria Cristobal, Maria B. Castro's half-sister. Maximo B. Cristobal did not file any income tax returns before the year
1946, except for the years 1939 and 1940, but in these years he was exempted from the tax. He has not filed any war
profits tax return (Exhibit 54). Antonio Cristobal, Segundo Esguerra, Sr. and Jose T. Lopez did not file any income tax
returns for the years prior to 1946, and neither did they file any war profits tax returns (Exhibit 52). Maria Cristobal filed
income tax returns for the years 1929 to 1942, but they were exempt from the tax (Exhibit 53). Benita A. Lamagna did
not file any income tax returns prior to 1945, except for 1942 which was exempt. Since did not file any war profits tax
(Exhibit 55). Ramon M. Sangalang did not file income tax returns up to 1945 except for the years 1936, 1937, 1938,
1939 and 1940. He has not filed any war profits tax return (Exhibit 56). Santiago Tan did not file any income tax returns
prior to 1945, except for the years 1938, 1939, 1940 and 1942, but all of these were exempt. He did not file any war
profits tax return (Exhibit 57). Amado A. Yatco did not file income tax returns prior to 1945, except for the years 1937,
1938, 1939, 1941 and 1942, but these were exempt. He did not file any war profits tax return (Exhibit 58).
Antonio Cristobal's income in 1946 is P15,630, and in 1947, P4,550 (Exhibits 59-60); Maximo B. Cristobal's
income in 1946 is P19,759.10, in 1947, P9,773.47 (Exhibits 61-62); Segundo Esguerra's income in 1946 is P5,500, in
1947, P7,754.32 (Exhibits 63-64); Jose T. Lopez's income in 1946 is P20,785, in 1947, P14,302.77 (Exhibits 69- 70);
Benita A. Lamagnas income in 1945 is P1,559, in 1946, P6,463.36, in 1947, P6,189.79 and her husband's income in
1947 is P10,825.53 (Exhibits 65-68); Ramon M. Sangalang's income in 1945 is P5,500, in 1946, P18,300.00 (Exhibits
71-72); Santiago Tan's income in 1945 is P456, in 1947 is P9,167.95, and in 1947, P7,620.11 (Exhibits 73-75); and
Amado Yatco's income in 1945 is P12,600 in 1946, P23,960, and in 1947, P11,160 (Exhibits 76-78).
In October, 1945 Maria B. Castro, Nicasio Yatco, Maxima Cristobal de Esguerra, Maria Cristobal Lopez and
Maximo Cristobal organized the Maria B. Castro, Inc. with a capital stock of P100,00, of which Maria B. Castro
subscribed for P99,600 and all the others for P100 each. This was increased in 1950 to P500,000 and Maria B. Castro
subscribed P76,000 and the others P1,000 each (Exhibit 126).
It does not appear that the stockholders or the board of directors of the Marvel Building Corporation have ever
held a business meeting, for no books thereof or minutes of meeting were ever mentioned by the officers thereof or
presented by them at the trial. The by-laws of the corporation, if any had ever been approved, has not been presented.
Neither does it appear that any report of the affairs of the corporation has been made, either of its transactions or
accounts.
From the book of accounts of the corporation, advances to the Marvel Building Corporation of P125,000 were
made by Maria B. Castro in 1947, P102,916.05 in 1948, and P160,910.96 in 1949 (Exhibit 118).
The main issue involved in these proceedings is: Is Maria B. Castro the owner of all the shares of stock of the
Marvel Building Corporation and the other stockholders mere dummies of hers?
The most important evidence presented by the Collector of Internal Revenue to prove his claim that Maria B.
Castro is the sole and exclusive owner of the shares of stock of the Marvel Building Corporation is the supposed
endorsement in blank of the shares of stock issued in the name of the other incorporators, and the possession thereof
by Maria B. Castro. The existence of said endorsed certificates was testified to by witnesses Felipe Aquino, internal
revenue examiner, Antonio Mariano, examiner, and Crispin Llamado, Under-Secretary of Finance, who declared as
follows: Towards the end of the year of 1948 and about the beginning of the year 1949, while Aquino and Mariano were
examining the books and papers of the Marvel Building Corporation at its place of business, which books and papers
were furnished by its Secretary, Maximo Cristobal, they came across an envelope containing eleven stock certificates,
bound together by an Acco fastener, which (certificates) corresponded in number and in amount on their face to the
subscriptions of the stockholders that all the certificates, except that in the name of Maria B. Castro, were endorsed in
blank by the subscribers; that as two revenue agents could not agree what to do with the certificates, Aquino brought
them to Under-Secretary of Finance Llamado, who thereupon suggested that photostatic copies thereof be taken; that
this was done, and the photostatic copies placed by him in his office safe; that Aquino returned the certificates that
same day after the photostatic copies had been taken; that the photostatic copies taken are Exhibits 4, 5, 6, 7, 8, 9, 10,
11, 12, and 13; and in that July, 1950, copy-cat copies of the above photostats were taken, and said copy-cat copies
are Exhibits 40-49.
Julio Llamado, bookkeeper of the Marvel Building Corporation from 1947 to May, 1948, also testified that he
was the one who had prepared the original certificates, putting therein the number of shares in words in handprint; that
the originals were given to him by Maria B. Castro for comparison with the articles of incorporation; that they were not
yet signed by the President and by the Secretary-Treasurer when he had the certificates; and that after the checking he
returned all of them to Mrs. Castro. He recognized the photostats, Exhibits 4 to 13 as photostats of the said originals.
He also declared that he also prepared a set of stock certificates, similar to the certificates which were copied in the
photostats, filling the blanks for the name of the stockholder, the number of shares, and the date of issue, and that the
certificates he had prepared are Exhibits H, H-1 to H-7 and J (Exhibits 30-38). This set of certificates was made him first
and the set of which photostats were taken, a few days later.
The plaintiffs offered a half-hearted denial of the existence of the endorsed blank certificates, Maximo Cristobal,
secretary of the corporation, saying that no investigation was ever made by Aquino and Mariano in which said
certificates were discovered by the latter. The, however, vigorously attack the credibility of the witnesses for the
defendant, imputing to the Llamados, enmity against Maria B. Castro, and to Aquino and Mariano, a very doubtful
conduct in not divulging the existence of the certificates either to Lobrin, Chief Income Tax Examiner, or to the Collector
of Internal Revenue, both their immediate chiefs. Reliance is also placed on a certificate, Exhibit W, wherein Aquino and
others, declare that the certificates (Exhibits not endorsed when the same were examined. In connection with this
certificate, Exhibit W, we note that it states that the certificates examined were Exhibits 30 to 38, the existence or
character of which are not disputed. But the statement contains nothing to the effect that the above certificates were the
only ones in existence, according to their knowledge. Again the certificate was issued for an examination on September
1949, not by Aquino and Mariano at the end of 1948 or the beginning of 1949. The certificate, therefore, neither denies
the existence of the endorsed certificates, nor that Aquino and Mariano had made an examination of the papers of the
corporation at the end of the year 1948. It can not, therefore, discredit the testimonies of the defendant's witnesses.
As to the supposed enmity of the Llamados towards the plaintiff Maria B. Castro, we note that, supposing that
there really was such enmity, it does not appear that it was of such magnitude or force as could have induced the
Llamados to lie or fabricate evidence against her. It seems that the Llamados and Maria B. Castro were close friends
way back in 1947 and up to 1949; but that at the time of the trial the friendship had been marred by misunderstandings.
We believe that in 1948 and 1949 the Llamados were trusted friends of Maria B. Castro, and this explains why they had
knowledge of her secret transactions. The younger Llamado even made advances for the hand of Maria B. Castro's
daughter, and this at the time when as a bookkeeper he was entrusted with checking up the certificates of stock. When
the older Llamado kept secret the existence of the endorsed certificates, the friendship between the two families was
yet intact; hence, the existence of the endorsed certificates must have been kept to himself by the older Llamado. All
the above circumstances reinforce our belief that the Llamados had personal knowledge of the facts they testified to,
and the existence of this knowledge in turn renders improbable plaintiffs' claim that their testimonies were biased.
Attempt was also made by the plaintiffs to show by expert evidence that the endorsement could have been
super-imposed, i.e., that the signatures made on other papers and these were pasted and thereafter the documents
photographed. Judicial experience is to the effect that expert witnesses can always be obtained for both sides of an
issue, most likely because expert witnesses are no longer impermeable to the influence of fees (II Wigmore, Sec. 563
(2), p. 646). and if parties are capable of paying fees, expert opinion should be received with caution. In the case at bar,
the opinion on the supposed superimposition was merely a possibility, and we note various circumstances which prove
that the signatures were not superimposed and corroborate defendant's claim that they were genuine. In the first place,
the printed endorsement contains a very heavy line at the bottom for the signature of the endorsee. This line in almost
all of the endorsements is as clear as the printed letters above it, and at the points where the letters of the signature
extend down and traverse it (the line), there is no indication that the line is covered by a superimposed paper. Again in
these places both the signatures and the lines are clear and distinct where the cross one another. Had there been
superimpositions the above features could not have been possible. In the second place, Maria B. Castro admitted
having singed 25 stock certificates, but only eleven were issued (t.s.n., p. 662). No explanation is given by her why she
had to sign as many as 25 forms when there were only eleven subscribers and eleven forms to be filled. This
circumstances corroborate the young Llamado's declaration that two sets of certificates had been prepared. The
nineteen issued must be Exhibits H, H-1 to H-7 and J, or Nos. 30 to 38, and the stock certificates endorsed whose
photostatic copies are Exhibits 4 to 13. It is to be remembered also, that it is a common practice among unscrupulous
merchants to carry two sets of books, one set for themselves and another to be shown to tax collectors. This practice
could not have been unknown to Maria B. Castro, who apparently had been able to evade the payment of her war
profits taxes. These circumstances, coupled with the testimony of Julio Llamado that two sets of certificates were given
to him for checking, show to an impartial mind the existence of the set of certificates endorsed in blank, thus confirming
the testimonies of the defendant's witnesses, Aquino, Mariano and Crispin Llamado, and thus discrediting the obviously
partial testimony of the expert presented by plaintiffs. The genuineness of the signatures on the endorsement is not
disputed. How could the defendant had secured these genuine signatures? Plaintiffs offer no explanation for this,
although they do not question them. It follows that the genuine signatures must have been made on the stock
certificates themselves.
Next in importance among the evidence submitted by the defendant collector to prove his contention that Maria
B. Castro is the sole owner of the shares of stock of the Marvel Building Corporation, is the fact that the other
stockholders did not have incomes in such amounts, during the time of the organization of the corporation in 1947, or
immediately thereto, as to enable them to pay in full for their supposed subscriptions. This fact is proved by their income
tax returns, or the absence thereof. Let us take Amado A. Yatco as an example. Before 1945 his returns were exempt
from the tax, in 1945 he had P12,600 and in 1946, P23,000. He has four children. How could he have paid P100,000 in
1945 and 1946? Santiago Tan who also contributed P100,000 had no appreciable income before 1946, and in this year
an income of only P9,167.95. Jose T. Lopez also did not file any income tax returns before 1940 and in 1946 he had an
income of only P20,785, whereas he is supposed to have subscribed P90,000 worth of stock early in 1947. Benita
Lamagna had no returns either up to 1945, except in 1942, which was exempt, and in 1945 she had an income of
P1,550 and in 1946, P6,463.36. In the same situation are all the others, and besides, brothers and sisters and brother-
in- law of Maria B. Castro. On the other hand, Maria B. Castro had been found to have made enormous gains or profits
in her business such that the taxes thereon were assessed at around P3,000,000. There was, therefore, a prima facie
case made out by the defendant collector that Maria B. Castro had furnished all the money that the Marvel Building
Corporation had.
In order to meet the above evidence only three of the plaintiffs testified, namely, Maximo Cristobal, the
corporation's secretary, who made the general assertion on the witness stand that the other stockholders paid for their
shares in full, Maria B. Castro, who stated that payments of the subscriptions were made to her, and C. S. Gonzales,
who admitted that Maria B. Castro paid for his subscription. After a careful study of the above testimonies, however, we
find them subject to various objections. Maximo Cristobal declared that he issued provincial receipts for the
subscriptions supposedly paid to him in 1946; but none of the supposed receipts was presented. If the subscriptions
were really received by him, big as the amounts were, he would have been able to tell specifically, by dates and in fixed
amounts, when and how the payments were made. The general assertion of alleged payments, without the concrete
days and amounts of payments, are, according to our experience, positive indications of untruthfulness, for when a
witness testified to a fact that actually occurs, the act is concretely stated and no generalization is made.
With respect to Maria B. Castro's testimony, we find it to be as untruthful as that of Cristobal. She declared that
payments of the subscriptions took place between July and December, 1946, and that said payments were first
deposited by her in the National City Bank of New York. A study of her account in said bank (Exhibit 82), however, fails
to show the alleged deposit of the subscriptions during the year 1946 (See Exhibits 83-112). This fact completely belies
her assertion. As to the testimony of C. S. Gonzales that Maria B. Castro advanced his subscription, there is nothing in
the evidence to corroborate it, and the circumstances show otherwise. If he had really been a stockholder and Maria B.
Castro advanced his subscription, the agreement between him and Castro should have been put in writing, the amount
advanced being quite considerable (P80,000), and it appearing further that Gonzales is no close relative or confidant of
Castro.

Lastly, it is significant that the plaintiffs, the supposed subscribers, who should have come to court to assert that
they actually paid for their subscriptions, and are not mere dummies, did not do so. They could not have afforded such a
costly indifference, valued at from P70,000 to P100,000 each, if they were not actual dummies. This failure on their part
to take the witness stand to deny or refute the charge that they were mere dummies is to us of utmost significance.
What could have been easier to disprove the charge that they were dummies, than for them to come to court and show
their receipts and testify on the payments they have made on their subscriptions? This they, however, refused to do.
They had it in their power to rebut the charges, but they chose to keep silent. The non-production of evidence that
would naturally have been produced by an honest and therefore fearless claimant permits the inference that its tenor is
unfavorable to the party's cause (II Wigmore, Sec. 285, p. 162). A party's silence to adverse testimony is equivalent to
an admission of its truth (Ibid, Sec. 289, p. 175).
Our consideration of the evidence submitted on both sides, leads us to a conclusion exactly opposite that
arrived at by the trial court. In general the evidence offered by the plaintiffs is testimonial and direct evidence, easy of
fabrication; that offered by defendant, documentary and circumstantial, not only difficult of fabrication but in most cases
found in the possession of plaintiffs. There is very little room for choice as between the two. The circumstantial evidence
is not only convincing; it is conclusive. The existence of endorsed certificates, discovered by the internal revenue agents
between 1943 and 1949 in the possession of the Secretary-Treasurer, the fact that twenty-five certificates were signed
by the president of the corporation, for no justifiable reason, the fact that two sets of certificates were issued, the
undisputed fact that Maria B. Castro had made enormous profits and, therefore, had a motive to hide them to evade the
payment of taxes, the fact that the other subscribers had no incomes of sufficient magnitude to justify their big
subscriptions, the fact that the subscriptions were not receipted for and deposited by the treasurer in the name of the
corporation but were kept by Maria B. Castro herself, the fact that the stockholders or the directors never appeared to
have ever met to discuss the business of the corporation, the fact that Maria B. Castro advanced big sums of money to
the corporation without any previous arrangement or accounting, and the fact that the books of accounts were kept as if
they belonged to Maria B. Castro alone — these facts are of patent and potent significance. What are their necessary
implications? Maria B. Castro would not have asked them to endorse their stock certificates, or be keeping these in her
possession, if they were really the owners. They never would have consented that Maria B. Castro keep the funds
without receipts or accounting, nor that she manages the business without their knowledge or concurrence, were they
owners of the stocks in their own rights. Each and every one of the facts all set forth above, in the same manner, is
inconsistent with the claim that the stockholders, other than Maria B. Castro, owned their shares in their own right. On
the other hand, each and every one of them, and all of them, can point to no other conclusion than that Maria B. Castro
was the sole and exclusive owner of the shares and that they were only her dummies.
In our opinion, the facts and circumstances duly set forth above, all of which have been proved to our
satisfaction, prove conclusively and beyond reasonable doubt (section 89, Rule 123 of the Rules of Court and section
42 of the Provisional law for the application of the Penal Code) that Maria B. Castro is the sole and exclusive owner of
all the shares of stock of the Marvel Building Corporation and that the other partners are her dummies.
Wherefore, the judgment appealed from should be, as it hereby is, reversed and the action filed by plaintiffs-
appellees, dismissed, with costs against plaintiffs-appellees. So ordered.
||| (Marvel Building Corp. v. David, G.R. No. L-5081, [February 24, 1954], 94 PHIL 376-389)

[G.R. No. L-15121. August 31, 1962.]

GREGORIO PALACIO, in his own behalf and in behalf of his minor child MARIO PALACIO,
plaintiffs-appellants, vs. FELY TRANSPORTATION COMPANY, defendant-appellee.

Antonio A. Saba for plaintiff-appellants.


Mercado, Ver & Reyes for defendant-appellee.

SYLLABUS

1. CORPORATIONS; SUBSIDIARY CIVIL LIABILITY FOR DAMAGES; FICTION OF CORPORATE ENTITY


NOT TO USED TO EVADE LIABILITY. — Where the main purpose in forming the corporation was to evade one's
subsidiary civil liability for damages in a criminal case, the corporation may not be heard to say that it has a personality
separate and distinct from its members, because to allow it to do so would be a shield to further an end subversive of
justice. (La Campana Coffee Factory, et al., vs. Kaisahan ng mga Manggagawa, etc., et al., 93 Phil., 160) The Supreme
Court can even substitute the real party in interest in place of the defendant corporation in order to avoid multiplicity of
suits and thereby save the parties unnecessary expenses and delay. (Alonzo vs. Villamor, 16 Phil., 315)

DECISION

REGALA, J p:
This is an appeal by the plaintiffs from the decision of the Court of First Instance of Manila which dismissed
their complaint.
Originally taken to the Court of Appeals, this appeal was certified to this Court on the ground that it raises
purely questions of law.
The parties in this case adopt the following findings of fact of the lower court:
"In their complaint filed with this Court on May 15, 1954, plaintiffs allege, among other things,
'that about December, 1952, the defendant company hired Alfredo Carillo as driver of AC-787 (687) (a
registration for 1952) owned and operated by the said defendant company; that on December 24, 1952,
at about 11:30 a.m., while the driver Alfonso (Alfredo) Carillo was driving AC-687 at Halcon Street,
Quezon City, wilfully, unlawfully and feloniously and in a negligent, reckless and imprudent manner, run
over a child Mario Palacio of the herein plaintiff Gregorio Palacio; that on account of the aforesaid injuries
Mario Palacio suffered a simple fracture of the right temor (sic), complete third, thereby hospitalizing him
at the Philippine Orthopedic Hospital from December 24, 1952, up to January 8, 1953, and continued to
be treated for a period of five months thereafter; that the plaintiff Gregorio Palacio herein is a welder by
occupation and owner of a small welding shop and because of the injuries of his child he has abandoned
his shop where he derives income of P10.00 a day for the support of his big family; that during the period
that the plaintiff's (Gregorio Palacio's) child was in the hospital and when said child was under treatment
for five months in order to meet the needs of his big family, he was forced to sell one air compressor
(heavy duty) and one heavy duty electric drill, for a sacrifice sale of P150.00 which could easily sell at
P350.00; that as a consequence of the negligent and reckless act of the driver Alfredo Carillo of the
herein defendant company, the herein plaintiffs were forced to litigate this case in Court for an agreed
amount of P300.00 for attorney's fee; that the herein plaintiffs have now incurred the amount of P500.00
for actual expenses for transportation, representation and similar expenses for gathering evidence and
witnesses; and that because of the nature of the injuries of plaintiff Mario Palacio, and the fear that the
child might become a useless invalid, the herein plaintiff Gregorio Palacio has suffered moral damages
which could be conservatively estimated at P1,200.00.'
"On May 23, 1956, defendant Fely Transportation Co. filed a Motion to Dismiss on the grounds
(1) that there is no cause of action against the defendant company, and (2) that the cause of action is
barred by prior judgment.
"In its Order, dated June 8, 1956, this Court deferred the determination of the grounds alleged in
the Motion to Dismiss until the trial of this case.
"On June 20, 1956, defendant filed its answer. By way of affirmative defenses, it alleges (1) that
complaint states no cause of action against defendant, and (2) that the sale and transfer of the jeep AC-
687 by Isabelo Calingasan to the Fely Transportation was made on December 24, 1955, long after the
driver Alfredo Carillo of said jeep had been convicted and had served his sentence in Criminal Case No.
Q-1084 of the Court of First Instance of Quezon City, in which both the civil and criminal cases were
simultaneously tried by agreement of the parties in said case. In the Counterclaim of the Answer,
defendant alleges that in view of the filing of this complaint which is a clearly unfounded civil action
merely to harass the defendant, it was compelled to engage the services of a lawyer for an agreed
amount of P500.00.
"During the trial, plaintiffs presented the transcript of the stenographic notes of the trial of the
case of 'People of the Philippines vs. Alfredo Carillo, Criminal Case No. Q-1084,' in the Court of First
Instance of Rizal, Quezon City (Branch IV), as Exhibit 'A'.
"It appears from Exhibit 'A' that Gregorio Palacio, one of the herein plaintiffs, testified that Mario
Palacio, the other plaintiff, is his son; that as a result of the reckless driving of accused Alfredo Carillo, his
child Mario was injured and hospitalized from December 24, 1952, to January 8, 1953; that during all the
time that his child was in the hospital, he watched him during the night and his wife during the day; that
during that period of time he could not work as he slept during the day; that before his child was injured,
he used to earn P10.00 a day on ordinary days and on Sundays from P20 to P50 a Sunday; that to meet
his expenses he had to sell his compressor and electric drill for P150 only; and that they could have been
sold for P300 at the lowest price.
"During the trial of the criminal case against the driver of the jeep in the Court of First Instance of
Quezon City (Criminal Case No. Q-1084), an attempt was unsuccessfully made by the prosecution to
prove moral damages allegedly suffered by herein plaintiff Gregorio Palacio. Likewise an attempt was
made in vain by the private prosecutor in that case to prove the agreed attorney's fees between him and
plaintiff Gregorio Palacio and the expenses allegedly incurred by the herein plaintiffs in connection with
that case. During the trial of this case, plaintiff Gregorio Palacio testified substantially to the same facts.
"The Court of First Instance of Quezon City in its decision in Criminal Case No. 1084 (Exhibit '2')
determined and thoroughly discussed the civil liability of the accused in that case. The dispositive part
thereof reads as follows:
"IN VIEW OF THE FOREGOING, the Court finds the accused Alfredo Carillo y Damaso
guilty beyond reasonable doubt of the crime charged in the information and he is hereby
sentenced to suffer imprisonment for a period of Two Months & One Day of Arresto Mayor; to
indemnify the offended party, by way of consequential damages, in the sum of P500.00 which
the Court, deems reasonable; with subsidiary imprisonment in case of insolvency but not to
exceed 1/3 of the principal penalty imposed; and to pay the costs.'"
On the basis of these facts, the lower court held that the action is barred by the judgment in the criminal case
and, that under Article 103 of the Revised Penal Code, the person subsidiarily liable to pay damages is Isabelo
Calingasan, the employer, and not the defendant corporation.
Against that decision, the plaintiffs appealed, contending that:
"THE LOWER COURT ERRED IN NOT SUSTAINING THAT THE DEFENDANT-APPELLEE
FOR DAMAGES AS A RESULT OF CRIMINAL CASE NO. Q-1084 OF THE COURT OF FIRST
INSTANCE OF QUEZON CITY FOR THE REASON THAT THE INCORPORATORS OF THE FELY
TRANSPORTATION COMPANY, THE DEFENDANT-APPELLEE HEREIN, ARE ISABELO
CALINGASAN HIMSELF, HIS SON AND DAUGHTERS;
"THE LOWER COURT ERRED IN NOT CONSIDERING THAT THE INTENTION OF ISABELO
CALINGASAN IN INCORPORATING THE FELY TRANSPORTATION COMPANY, THE DEFENDANT-
APPELLEE HEREIN, WAS TO EVADE HIS CIVIL LIABILITY AS A RESULT OF THE CONVICTION OF
HIS DRIVER OF VEHICLE AC-687 THEN OWNED BY HIM:
"THE LOWER COURT ERRED IN HOLDING THAT THE CAUSE OF ACTION OF THE
PLAINTIFFS-APPELLANTS IS BARRED BY PRIOR JUDGMENT."
With respect to the first and second assignments of errors, plaintiffs contend that the defendant corporation
should be made subsidiarily liable for damages in the criminal case because the sale to it of the jeep in question, after
the conviction of Alfredo Carillo in Criminal Case No. Q-1084 of the Court of First Instance of Quezon City, was merely
an attempt on the part of Isabelo Calingasan, its president and general manager, to evade his subsidiary civil liability.
The Court agrees with this contention of the plaintiffs. Isabelo Calingasan and defendant Fely Transportation
may be regarded as one and the same person. It is evident that Isabelo Calingasan's main purpose in forming the
corporation was to evade his subsidiary civil liability 1 resulting from the conviction of his driver, Alfredo Carillo. This
conclusion is borne out by the fact that the incorporators of the Fely Transportation are Isabelo Calingasan, his wife, his
son, Dr. Calingasan, and his two daughters. We believe that this is one case where the defendant corporation should
not be heard to say that it has a personality separate and distinct from its members when to allow it to do so would be to
sanction the use of the fiction of corporate entity as a shield to further an end subversive of justice. (La Campana Coffee
Factory, et al. vs. Kaisahan ng mga Manggagawa, etc., et al., G.R. No. L-5677, May 25, 1953) Furthermore, the failure
of the defendant corporation to prove that it has other property than the jeep (AC-687) strengthens the conviction that its
formation was for the purpose above indicated.
And while it is true that Isabelo Calingasan is not a party in this case, yet, as held in the case of Alonso vs.
Villamor, 16 Phil. 315, this Court can even substitute him in place of the defendant corporation as to the real party in
interest. This is so in order to avoid multiplicity of suits and thereby save the parties unnecessary expenses and delay.
(Sec. 2, Rule 17, Rules of Court; Cuyugan vs. Dizon, 79 Phil., 80; Quison vs. Salud, 12 Phil., 109)
Accordingly, defendants Fely Transportation and Isabelo Calingasan should be held subsidiarily liable for
P500.00 which Alfredo Carillo was ordered to pay in the criminal case and which amount he could not pay on account of
insolvency.
We also sustain plaintiffs' third assignment of error and hold that the present action is not barred by the
judgment of the Court of First Instance of Quezon City in the criminal case. While there seems to be some confusion on
the part of the plaintiffs as to the theory on which the case is based — whether ex-delicto or quasi ex-delicto (culpa
aquiliana) — We are convinced, from the discussion and prayer in the brief on appeal, that they are insisting on the
subsidiary civil liability of the defendant. As a matter of fact, the record shows that plaintiffs merely presented the
transcript of the stenographic notes (Exhibit "A" ) taken at the hearing of the criminal case, which Gregorio Palacio
corroborated, in support of their claim for damages. This rules out the defense of res judicata, because such liability
proceeds precisely from the judgment in the criminal action, where the accused was found guilty and ordered to pay an
indemnity in the sum of P500.00.
WHEREFORE, the decision of the lower court is hereby reversed and defendants Fely Transportation and
Isabelo Calingasan are ordered to pay, jointly and severally, the plaintiffs the amount of P500.00 and the costs.
||| (Palacio v. Fely Transportation Co., G.R. No. L-15121, [August 31, 1962], 116 PHIL 154-160)

[G.R. No. L-20886. April 27, 1967.]

NATIONAL MARKETING CORPORATION (NAMARCO), plaintiff-appellant, vs. ASSOCIATED


FINANCE COMPANY, INC. and FRANCISCO SYCIP, defendant, FRANCISCO SYClP, defendant-
appellee.

Tomas P. Matic, Jr. for plaintiff and appellant.


Francisco Sycip on his behalf as defendant and appellee.

SYLLABUS

1. CORPORATION; STOCKHOLDERS; LIABILITY FOR CORPORATE OBLIGATIONS. — A stockholder is


guilty of fraud where, through false representations, he succeeded in inducing another corporation to enter into an
exchange agreement with the corporation he represented and over whose business he had absolute control knowing
that the letter was in no position to comply with the obligation it had assumed. Consequently, said stockholder cannot
now seek refuge behind the general principle that a corporation has a personality distinct and separate from that of its
stockholders and that the latter are not personally liable for the corporate obligations. Upon the facts proven, the court is
justified in "piercing the veil of corporate fiction" and in holding said stockholder personally liable, jointly and severally
with the corporation, for the sums of money adjudged in favor of the aggrieved party.
2. ID.; WHEN MAY CORPORATE FICTION BE DISREGARDED. — It is settled law in this and other
jurisdictions that when the corporation is the mere alter ego of a person, the corporate fiction may be disregarded; the
same being true when the corporation is controlled, and its affairs are so conducted as to make it merely an
instrumentality, agency or conduit of another (Koppel Phils. etc. vs. Yatco etc., 43 Off. Gaz., No. 11, November, 1947;
Yutivo Sons etc. vs. Court of Tax Appeals etc., 110 Phil., 751; promulgated on January 28, 1961.)

DECISION

DIZON, J p:

Appeal taken by the National Marketing Corporation — hereinafter referred to as NAMARCO, from the decision
of the Court of First Instance of Manila in Civil Case No. 45770 ordering the Associated Finance Company, Inc. —
hereinafter referred to as the ASSOCIATED — to pay the NAMARCO the sum of P403,514.28, with legal interest
thereon from the date of filing of the action until fully paid, P80,702.26 as liquidated damages, P5,000.00 as attorney's
fees, plus costs, but dismissing the complaint insofar as defendant Francisco Sycip was concerned, as well as the
latter's counterclaim. The appear is only from that portion of the decision dismissing the case as against Francisco
Sycip.
On March 25, 1958, ASSOCIATED, a domestic corporation, through its President, appellee Francisco Sycip,
entered into an agreement to exchange sugar with NAMARCO, represented by its then General Manager, Benjamin
Estrella, whereby the former would deliver to the latter 22,516 bags (each weighing 100 pounds) of "Victorias" and/or
"National" refined sugar in exchange for 7,732.71 bags of "Busilak" and 17,285.08 piculs of "Pasumil" raw sugar
belonging to NAMARCO, both agreeing to pay liquidated damages equivalent to 20% of the contractual value of the
sugar should either party fail to comply with the terms and conditions stipulated (Exhibit A). Pursuant thereto, on May
19, 1958, NAMARCO delivered to ASSOCIATED 7,732.71 bags of "Busilak" and 17,285.08 piculs of "Pasumil"
domestic raw sugar. As ASSOCIATED failed to deliver to NAMARCO the 22,516 bags of "Victorias" and/or "National"
refined sugar agreed upon, latter, an January 12, 1959, demanded in writing from the ASSOCIATED either (a)
immediate delivery thereof before January 20, or (b) payment of its equivalent cash value amounting to P372,639.80.
On January 19, 1959, ASSOCIATED, through Sycip, offered to pay NAMARCO the value of 22,516 bags of
refined sugar at the rate of P15.30 per bag, but the latter rejected the offer. Instead, on January 21 of the same year, it
demanded payment of the 7,732.71 bags of "Busilak" raw sugar at P15.30 per bag, amounting to P118,310.40, and of
the 17,285.08 piculs of "Pasumil" raw sugar at P16.50 per picul, amounting to P285,203.82, or a total price of
P403,514.28 for both kinds of sugar, based on the sugar quotations (Exhibit H) as of March 20, 1958 — the date when
the exchange agreement was entered into.
As ASSOCIATED refused to deliver the raw sugar or pay for the refined sugar delivered to it, in spite of
repeated demands therefore, NAMARCO instituted the present action in the lower court to recover the sum of
P403,514.28 in payment of the raw sugar received by defendants from it; P80,702.86; as liquidated damages;
P10,000.00 as attorney's fees, expenses of litigation and exemplary damages, with legal interest thereon from the filing
of the complaint until fully paid.
In their amended answer defendants, by way of affirmative defenses, alleged that the correct value of the sugar
delivered by NAMARCO to them was P259,451.09 or P13.30 per bag of 100 lbs. weight (quedan basis) and not
P403,514.58 as claimed by NAMARCO. As counterclaim they prayed for the award of P500,000.00 as moral damages,
P100,000.00 as exemplary damages and P10,000.00 as attorney's fees.
After trial the court rendered the appealed judgment. The appeal was taken to the Court of Appeals, but on
January 15, 1963 the latter certified the case to Us for final adjudication pursuant to sections 17 and 31 of the Judiciary
Act of 1948, as amended, the amount involved being more than P200,000.00, exclusive of interests and costs.
The only issue to be resolved is whether, upon the facts found by the trial court, — which, in our opinion, are
fully supported by the evidence — Francisco Sycip may be held liable, jointly and severally with his co-defendant, for
the sums of money adjudged in favor of NAMARCO.
The evidence of record shows that, of the capital stock of ASSOCIATED, Sycip owned P60,000.00 worth of
shares, while his wife — the second biggest stockholder — owned P20,000.00 worth of shares; that the par value of the
subscribed capital stock of ASSOCIATED was only P105,000.00; that negotiations that lead to the execution of the
exchange agreement in question were conducted exclusively by Sycip on behalf of ASSOCIATED; that, as a matter of
fact, in the course of his testimony, Sycip referred to himself as the one who contracted or transacted the business in
his personal capacity, and asserted that the exchange agreement was his personal contract; that it was Sycip who
made personal representations and gave assurances that ASSOCIATED was in actual possession of the 22,516 bags
of "Victorias" and/or "National" refined sugar which the latter had agreed to deliver to NAMARCO, and that the same
was ready for delivery; that, as a matter of fact, ASSOCIATED was at that time already insolvent; that when NAMARCO
made written demands upon ASSOCIATED to deliver the 22,516 bags of refined sugar it was under obligation to deliver
to the former, ASSOCIATED and Sycip, instead of making delivery of the sugar, offered to pay its value at the rate of
P15.30 per bag — a clear indication that they did not have the sugar contracted for.
The foregoing facts, fully established by the evidence, can lead to no other conclusion than that Sycip was
guilty of fraud because through false representations he succeeded in inducing NAMARCO to enter into the aforesaid
exchange agreement, with full knowledge, on his part of the fact that ASSOCIATED whom he represented and over
whose business and affairs he had absolute control, was in no position to comply with the obligation it had assumed.
Consequently, he can not now seek refuge behind the general principle that a corporation has a personality distinct and
separate from that of its stockholders and that the latter are not personally liable for the corporate obligations. To the
contrary, upon the proven facts, We feel perfectly justified in "piercing the veil of corporate fiction" and in holding Sycip
personally liable, jointly and severally with his co-defendant, for the sums of money adjudged in favor of appellant. It is
settled law in this and other jurisdictions that when the corporation is the mere alter ego of a person, the corporate
fiction may be disregarded; the same being true when the corporation is controlled, and its affairs are so conducted as
to make it merely an instrumentality, agency or conduit of another (Koppel Phils. etc. vs. Yatco etc., 43 Off. Gaz. No. 11,
November, 1947; Yutivo Sons etc. vs. Court of Tax Appeals etc., G.R. No. L-13203 promulgated on January 28, 1961.)
Wherefore, the decision appealed from is modified by sentencing defendant-appellee Francisco Sycip to pay,
jointly and severally with the Associated Finance Company, Inc., the sums of money which the trial court sentenced the
latter to pay to the National Marketing Corporation, as follows: the sum of FOUR HUNDRED THREE THOUSAND FIVE
HUNDRED FOURTEEN P E S O S AND TWENTY-EIGHT CENTAVOS (P403,514.28), with interest at the legal rate
from the date of the filing of the action until fully paid plus an additional amount of EIGHTY THOUSAND SEVEN
HUNDRED TWO PESOS AND EIGHTY-SIX CENTAVOS (80,702.86) as liquidated damages and P5,000.00 as
attorney's fees and further to pay the costs. With costs.
||| (National Marketing Corp. v. Associated Finance Co., Inc., G.R. No. L-20886, [April 27, 1967], 126 PHIL 196-201)

[G.R. No. L-41337. June 30, 1988.]

TAN BOON BEE & CO., INC., petitioner, vs. THE HONORABLE HILARION U. JARENCIO, PRESIDING
JUDGE OF BRANCH XXIII of the Court of First Instance of Manila, GRAPHIC PUBLISHING, INC., and
PHILIPPINE AMERICAN DRUG COMPANY, respondents.

De Santos, Balgos & Perez Law Office for petitioner.


Araneta, Mendoza & Papa Law Office for respondent Phil. American Drug Company.

DECISION

PARAS, J p:

This is a petition for certiorari, with prayer for preliminary injunction, to annul and set aside the March 26, 1975 Order of the
then Court of First Instance of Manila, Branch XXIII, setting aside the sale of "Heidelberg" cylinder press executed by the
sheriff in favor of the herein petitioner, as well as the levy on the said property, and ordering the sheriff to return the said
machinery to its owner, herein private respondent Philippine American Drug Company.
Petitioner herein, doing business under the name and style of Anchor Supply Co., sold on credit to herein private
respondent Graphic Publishing, Inc. (GRAPHIC for short) paper products amounting to P55,214.73. On December 20,
1972, GRAPHIC made partial payment by check to petitioner in the total amount of P24,848.74; and on December 21,
1972, a promissory note was executed to cover the balance of P30,365.99. In the said promissory note, it was stipulated
that the amount will be paid on monthly installments and that failure to pay any installment would make the amount
immediately demandable with an interest of 12% per annum. On September 6, 1973, for failure of GRAPHIC to pay any
installment, petitioner filed with the then Court of First Instance of Manila, Branch XXIII, presided over by herein respondent
judge, Civil Case No. 91857 for a Sum of Money (Rollo, pp. 36-38). Respondent judge declared GRAPHIC in default for
failure to file its answer within the reglementary period and plaintiff (petitioner herein) was allowed to present its evidence ex
parte. In a Decision dated January 18, 1974 (Ibid., pp. 39-40), the trial court ordered GRAPHIC to pay the petitioner the sum
of P30,365.99 with 12% interest from March 30, 1973 until fully paid, plus the costs of suit. On motion of petitioner, a writ of
execution was issued by respondent judge; but the aforestated writ having expired without the sheriff finding any property of
GRAPHIC, an alias writ of execution was issued on July 2, 1974.
Pursuant to the said issued alias writ of execution, the executing sheriff levied upon one (1) unit printing machine identified
as "Original Heidelberg Cylinder Press" Type H 222, NR 78048, found in the premises of GRAPHIC. In a Notice of Sale of
Execution of Personal Property dated July 29, 1974, said printing machine was scheduled for auction sale on July 26, 1974
at 10:00 o'clock at 14th St., Cor. Atlanta St., Port Area, Manila (Ibid., p. 45); but in a letter dated July 19, 1974, herein
private respondent, Philippine American Drug Company (PADCO for short) had informed the sheriff that the printing
machine is its property and not that of GRAPHIC, and accordingly, advised the sheriff to cease and desist from carrying out
the scheduled auction sale on July 26, 1974. Notwithstanding the said letter, the sheriff proceeded with the scheduled
auction sale, sold the property to the petitioner, it being the highest bidder, and issued a Certificate of Sale in favor of
petitioner (Rollo, p. 48). More than five (5) hours after the auction sale and the issuance of the certificate of sale, PADCO
filed an "Affidavit of Third Party Claim" with the Office of the City Sheriff (ibid., p. 47). Thereafter, on July 30, 1974, PADCO
filed with the Court of First Instance of Manila, Branch XXIII, a Motion to Nullify Sale on Execution (With Injunction) (Ibid.,
pp. 49-55), which was opposed by the petitioner (Ibid., pp. 56-68). Respondent judge, in an Order dated March 26, 1975
(Ibid., pp. 64-69), ruled in favor of PADCO. The decretal portion of the said order, reads: cdrep
"WHEREFORE, the sale of the 'Heidelberg' cylinder press executed by the Sheriff in favor of the plaintiff
as well as the levy on the said property is hereby set aside and declared to be without any force and
effect. The Sheriff is ordered to return the said machinery to its owner, the Philippine American Drug Co."
Petitioner filed a Motion For Reconsideration (Ibid., pp. 70-93) and an Addendum to Motion for Reconsideration (Ibid., pp.
94-108), but in an Order dated August 13, 1975, the same was denied for lack of merit (Ibid., p. 109). Hence, the instant
petition.
In a Resolution dated September 12, 1975, the Second Division of this Court resolved to require the respondents to
comment, and to issue a temporary restraining order (Rollo, p. 111). After submission of the parties' Memoranda, the case
was submitted for decision in the Resolution of November 28, 1975 (Ibid., p. 275).
Petitioner, to support its stand, raised two (2) issues, to wit:
I
THE RESPONDENT JUDGE GRAVELY EXCEEDED, IF NOT ACTED WITHOUT JURISDICTION
WHEN HE ACTED UPON THE MOTION OF PADCO, NOT ONLY BECAUSE SECTION 17, RULE 39
OF THE RULES OF COURT WAS NOT COMPLIED WITH, BUT ALSO BECAUSE THE CLAIMS OF
PADCO WHICH WAS NOT A PARTY TO THE CASE COULD NOT BE VENTILATED IN THE CASE
BEFORE HIM BUT IN INDEPENDENT PROCEEDING.
II
THE RESPONDENT JUDGE GRAVELY ABUSED HIS DISCRETION WHEN HE REFUSED TO PIERCE
THE PADCO'S (IDENTITY) AND DESPITE THE ABUNDANCE OF EVIDENCE CLEARLY SHOWING
THAT PADCO WAS CONVENIENTLY SHIELDING UNDER THE THEORY OF CORPORATE FICTION.
Petitioner contends that respondent judge gravely exceeded, if not, acted without jurisdiction, in nullifying the sheriff's sale
not only because Section 17, Rule 39 of the Rules of Court was not complied with, but more importantly because PADCO
could not have litigated its claim in the same case, but in an independent civil proceeding.
This contention is well-taken.
In the case of Bayer Philippines, Inc. vs. Agana (63 SCRA 355, 366-367 [1975]), this Court categorically ruled as follows:
"In other words, construing Section 17 of Rule 39 of the Revised Rules of Court, the rights of third-party
claimants over certain properties levied upon by the sheriff to satisfy the judgment should not be decided
in the action where the third-party claims have been presented, but in the separate action instituted by
the claimants.
". . . Otherwise stated. the court issuing a writ of execution is supposed to enforce the authority only over
properties of the judgment debtor, and should a third party appear to claim the property levied upon by
the sheriff, the procedure laid down by the Rules is that such claim should be the subject of a separate
and independent action.
xxx xxx xxx
". . . This rule is dictated by reasons of convenience as 'intervention is more likely to inject confusion into
the issues between the parties in the case . . . with which the third-party claimant has nothing to do and
thereby retard instead of facilitate the prompt dispatch of the controversy which is the underlying
objective of the rules of pleading and practice.' Besides, intervention may not be permitted after trial has
been concluded and a final judgment rendered in the case."
However, the fact that petitioner questioned the jurisdiction of the court during the initial hearing of the case but
nevertheless actively participated in the trial, bars it from questioning now the court's jurisdiction. A party who voluntarily
participated in the trial, like the herein petitioner, cannot later on raise the issue of the court's lack of jurisdiction
(Philippine National Bank vs. Intermediate Appellate Court, 143 SCRA [1986]). cdll
As to the second issue (the non-piercing of PADCO's corporate identity) the decision of respondent judge is as follows:
"The plaintiff, however, contends that the controlling stockholders of the Philippine American Drug Co.
are also the same controlling stockholders of the Graphic Publishing, Inc. and, therefore, the levy upon
the said machinery which was found in the premises occupied by the Graphic Publishing, Inc. should be
upheld. This contention cannot be sustained because the two corporations were duly incorporated under
the Corporation Law and each of them has a juridical personality distinct and separate from the other and
the properties of one cannot be levied upon to satisfy the obligation of the other. This legal preposition is
elementary and fundamental."
It is true that a corporation, upon coming into being, is invested by law with a personality separate and distinct from that of
the persons composing it as well as from any other legal entity to which it may be related (Yutivo & Sons Hardware
Company vs. Court of Tax Appeals, 1 SCRA 160 [1961]; and Emilio Cano Enterprises, Inc. vs. CIR, 13 SCRA 290 [1965]).
As a matter of fact, the doctrine that a corporation is a legal entity distinct and separate from the members and stockholders
who compose it is recognized and respected in all cases which are within reason and the law (Villa Rey Transit, Inc. vs.
Ferrer, 25 SCRA 845 [1968]). However, this separate and distinct personality is merely a fiction created by law for
convenience and to promote justice (Laguna Transportation Company vs. SSS, 107 Phil. 833 [1960]). Accordingly, this
separate personality of the corporation may be disregarded, or the veil of corporate fiction pierced, in cases where it is used
as a cloak or cover for fraud or illegality, or to work an injustice, or where necessary to achieve equity or when necessary for
the protection of creditors (Sulo ng Bayan, Inc. vs. Araneta, Inc., 72 SCRA 347 [1976]). Corporations are composed of
natural persons and the legal fiction of a separate corporate personality is not a shield for the commission of injustice and
inequity (Chenplex, Philippines, Inc., et al. vs. Hon. Pamatian, et al., 57 SCRA 408 [1974]). Likewise, this is true when the
corporation is merely an adjunct, business conduit or alter ego of another corporation. In such case, the fiction of separate
and distinct corporation entities should be disregarded (Commissioner of Internal Revenue vs. Norton & Harrison, 11 SCRA
714 [1964]).

In the instant case, petitioner's evidence established that PADCO was never engaged in the printing business; that the
board of directors and the officers of GRAPHIC and PADCO were the same; and that PADCO holds 50% share of stock of
GRAPHIC. Petitioner likewise stressed that PADCO's own evidence shows that the printing machine in question had been
in the premises of GRAPHIC since May, 1965, long before PADCO even acquired its alleged title on July 11, 1966 from
Capitol Publishing. That the said machine was allegedly leased by PADCO to GRAPHIC on January 24, 1966, even before
PADCO purchased it from Capital Publishing on July 11, 1966, only serves to show that PADCO's claim of ownership over
the printing machine is not only farce and sham but also unbelievable.
Considering the aforestated principles and the circumstances established in this case, respondent judge should have
pierced PADCO's veil of corporate identity.
Respondent PADCO argues that if respondent judge erred in not piercing the veil of its corporate fiction, the error is merely
an error of judgment and not an error of jurisdiction correctable by appeal and not by certiorari.
To this argument of respondent, suffice it to say that the same is a mere technicality. In the case of Rubio vs. Mariano (52
SCRA 338, 343 [1973]), this Court ruled:
"While We recognize the fact that these movants — the MBTC, the Phillips spouses, the Phillips
corporation and the Hacienda Benito, Inc. — did raise in their respective answers the issue as to the
propriety of the instant petition for certiorari on the ground that the remedy should have been appeal
within the reglementary period, We considered such issue as a mere technicality which would have
accomplished nothing substantial except to deny to the petitioner the right to litigate the matters he
raised . . ."
Litigations should, as much as possible, be decided on their merits and not on technicality (De las Alas vs. Court of
Appeals, 83 SCRA 200, 216 [1978]). Every party-litigant must be afforded the amplest opportunity for the proper and
just determination of his cause, free from the unacceptable plea of technicalities (Heirs of Ceferino Morales vs. Court of
Appeals, 67 SCRA 304, 310 [1975]). LexLib
PREMISES CONSIDERED, the March 26, 1975 Order of the then Court of First Instance of Manila, is ANNULLED and SET
ASIDE, and the Temporary Restraining Order issued is hereby made permanent. SO ORDERED.
||| (Tan Boon Bee & Co., Inc. v. Jarencio, G.R. No. L-41337, [June 30, 1988], 246 PHIL 211-219)

[G.R. No. 58168. December 19, 1989.]

CONCEPCION MAGSAYSAY-LABRADOR, SOLEDAD MAGSAYSAY-CABRERA, LUISA MAGSAYSAY-


CORPUZ, assisted by her husband, Dr. Jose Corpuz, FELICIDAD P. MAGSAYSAY, and MERCEDES
MAGSAYSAY-DIAZ, petitioner, vs. THE COURT OF APPEALS and ADELAIDA RODRIGUEZ-MAGSAYSAY,
Special Administratrix of the state of the late Genaro F. Magsaysay, respondents.
SYLLABUS

1. REMEDIAL LAW; CIVIL PROCEDURE; INTERVENTION; WHEN ALLOWED. — Viewed in the light of Section 2, Rule 12
of the Revised Rules of Court, this Court affirms the respondent court's holding that petitioners herein have no legal interest
in the subject matter in litigation so as to entitle them to intervene in the proceedings below. In the case of Batama Farmers'
Cooperative Marketing Association, Inc. v. Rosal, we held: "As clearly stated in Section 2 of Rule 12 of the Rules of Court,
to be permitted to intervene in a pending action, the party must have a legal interest in the matter in litigation, or in the
success of either of the parties or an interest against both, or he must be so situated as to be adversely affected by a
distribution or other disposition of the property in the custody of the court or an officer thereof." To allow intervention, [a] it
must be shown that the movant has legal interest in the matter in litigation, or otherwise qualified; and [b] consideration must
be given as to whether the adjudication of the rights of the original parties may be delayed or prejudiced, or whether the
intervenor's rights may be protected in a separate proceeding or not. Both requirements must concur as the first is not more
important than the second.
2. ID.; ID.; ID.; ID.; "INTEREST IN THE SUBJECT MATTER", EXPLAINED. — The interest which entitles a person to
intervene in a suit between other parties must be in the matter in litigation and of such direct and immediate character that
the intervenor will either gain or lose by the direct legal operation and effect of the judgment. Otherwise, if persons not
parties of the action could be allowed to intervene, proceedings will become unnecessarily complicated, expensive and
interminable. And this is not the policy of the law. The words "an interest in the subject" mean a direct interest in the cause
of action as pleaded, and which would put the intervenor in a legal position to litigate a fact alleged in the complaint, without
the establishment of which plaintiff could not recover.
3. COMMERCIAL LAW; CORPORATION; SHARES OF STOCK; DOES NOT VEST LEGAL RIGHT OR TITLE TO ANY OF
THE PROPERTY OF THE CORPORATION. — While a share of stock represents a proportionate or aliquot interest in the
property of the corporation, it does not vest the owner thereof with any legal right or title to any of the property, his interest in
the corporate property being equitable or beneficial in nature. Shareholders are in no legal sense the owners of corporate
property, which is owned by the corporation as a distinct legal person.
4. ID.; ID.; ID.; REQUISITES OF A VALID TRANSFER. — The petitioners cannot claim the right to intervene on the strength
of the transfer of shares allegedly executed by the late Senator. The corporation did not keep books and records. Perforce,
no transfer was ever recorded, much less affected as to prejudice third parties. The transfer must be registered in the books
of the corporation to affect third persons. The law on corporations is explicit, Section 63 of the Corporation Code provides,
thus: "No transfer, however, shall be valid, except as between the parties, until the transfer is recorded in the books of the
corporation showing the names of the parties to the transaction, the date of the transfer, the number of the certificate or
certificates and the number of shares transferred."

DECISION

FERNAN, C.J p:

In this petition for review on certiorari, petitioners seek to reverse and set aside [1] the decision of the Court of Appeals
dated July 13, 1981, 1 affirming that of the Court of First Instance of Zambales and Olongapo City which denied petitioners'
motion to intervene in an annulment suit filed by herein private respondent, and [2] its resolution dated September 7, 1981,
denying their motion for reconsideration. LLphil
Petitioners are raising a purely legal question; whether or not respondent Court of Appeals correctly denied their motion for
intervention.
The facts are not controverted.
On February 9, 1979, Adelaida Rodriguez-Magsaysay, widow and special Administratrix of the estate of the late Senator
Genaro Magsaysay, brought before the then Court of First Instance of Olongapo an action against Artemio Panganiban,
Subic Land Corporation (SUBIC), Filipinas Manufacturer's Bank (FILMANBANK) and the Register of Deeds of Zambales. In
her complaint, she alleged that in 1958, she and her husband acquired, thru conjugal funds, a parcel of land with
improvements, known as "Pequeña Island", covered by TCT No. 3258; that after the death of her husband, she discovered
[a] an annotation at the back of TCT No. 3258 that "the land was acquired by her husband from his separate capital;" [b] the
registration of a Deed of Assignment dated June 25, 1976 purportedly executed by the late Senator in favor of SUBIC, as a
result of which TCT No. 3258 was cancelled and TCT No. 22431 issued in the name of SUBIC; and [c] the registration of
Deed of Mortgage dated April 28, 1977 in the amount of P2,700,000.00 executed by SUBIC in favor of FILMANBANK; that
the foregoing acts were void and done in an attempt to defraud the conjugal partnership considering that the land is
conjugal, her marital consent to the annotation on TCT No. 3258 was not obtained, the change made by the Register of
Deeds of the title holders was effected without the approval of the Commissioner of Land Registration and that the late
Senator did not execute the purported Deed of Assignment or his consent thereto, if obtained, was secured by mistake,
violence and intimidation. She further alleged that the assignment in favor of SUBIC was without consideration and
consequently null and void. She prayed that the Deed of Assignment and the Deed of Mortgage be annulled and that the
Register of Deeds be ordered to cancel TCT No. 22431 and to issue a new title in her favor. LexLib
On March 7, 1979, herein petitioners, sisters of the late senator, filed a motion for intervention on the ground that on June
20, 1978, their brother conveyed to them one-half (1/2) of his shareholdings in SUBIC or a total of 416,566.6 shares and as
assignees of around 41% of the total outstanding shares of such stocks of SUBIC, they have a substantial and legal interest
in the subject matter of litigation and that they have a legal interest in the success of the suit with respect to SUBIC.
On July 26, 1979, the court denied the motion for intervention, and ruled that petitioners have no legal interest whatsoever
in the matter in litigation and their being alleged assignees or transferees of certain shares in SUBIC cannot legally entitle
them to intervene because SUBIC has a personality separate and distinct from its stockholders.
On appeal, respondent Court of Appeals found no factual or legal justification to disturb the findings of the lower court. The
appellate court further stated that whatever claims the petitioners have against the late Senator or against SUBIC for that
matter can be ventilated in a separate proceeding, such that with the denial of the motion for intervention, they are not left
without any remedy or judicial relief under existing law.
Petitioners' motion for reconsideration was denied. Hence, the instant recourse.
Petitioners anchor their right to intervene on the purported assignment made by the late Senator of a certain portion of his
shareholdings to them as evidenced by a Deed of Sale dated June 20, 1978. 2 Such transfer, petitioners posit, clothes them
with an interest, protected by law, in the matter of litigation. LLphil
Invoking the principle enunciated in the case of PNB v. Phil. Veg. Oil Co., 49 Phil. 857, 862 & 853 (1927), 3 petitioners
strongly argue that their ownership of 41.66% of the entire outstanding capital stock of SUBIC entitles them to a significant
vote in the corporate affairs; that they are affected by the action of the widow of their late brother for it concerns the only
tangible asset of the corporation and that it appears that they are more vitally interested in the outcome of the case than
SUBIC.
Viewed in the light of Section 2, Rule 12 of the Revised Rules of Court, this Court affirms the respondent court's holding that
petitioners herein have no legal interest in the subject matter in litigation so as to entitle them to intervene in the
proceedings below. In the case of Batama Farmers' Cooperative Marketing Association, Inc. v. Rosal, 4 we held: "As clearly
stated in Section 2 of Rule 12 of the Rules of Court, to be permitted to intervene in a pending action, the party must have a
legal interest in the matter in litigation, or in the success of either of the parties or an interest against both, or he must be so
situated as to be adversely affected by a distribution or other disposition of the property in the custody of the court or an
officer thereof." LLphil
To allow intervention, [a] it must be shown that the movant has legal interest in the matter in litigation, or otherwise qualified;
and [b] consideration must be given as to whether the adjudication of the rights of the original parties may be delayed or
prejudiced, or whether the intervenor's rights may be protected in a separate proceeding or not. Both requirements must
concur as the first is not more important than the second. 5
The interest which entitles a person to intervene in a suit between other parties must be in the matter in litigation and of
such direct and immediate character that the intervenor will either gain or lose by the direct legal operation and effect of the
judgment. Otherwise, if persons not parties of the action could be allowed to intervene, proceedings will become
unnecessarily complicated, expensive and interminable. And this is not the policy of the law. 6

The words "an interest in the subject" mean a direct interest in the cause of action as pleaded, and which would put the
intervenor in a legal position to litigate a fact alleged in the complaint, without the establishment of which plaintiff could not
recover. 7
Here, the interest, if it exists at all, of petitioners-movants is indirect, contingent, remote, conjectural, consequential and
collateral. At the very least, their interest is purely inchoate, or in sheer expectancy of a right in the management of the
corporation and to share in the profits thereof and in the properties and assets thereof on dissolution, after payment of the
corporate debts and obligations. LLphil
While a share of stock represents a proportionate or aliquot interest in the property of the corporation, it does not vest the
owner thereof with any legal right or title to any of the property, his interest in the corporate property being equitable or
beneficial in nature. Shareholders are in no legal sense the owners of corporate property, which is owned by the corporation
as a distinct legal person. 8
Petitioners further contend that the availability of other remedies, as declared by the Court of Appeals, is totally immaterial
to the availability of the remedy of intervention.
We cannot give credit to such averment. As earlier stated, that the movant's interest may be protected in a separate
proceeding is a factor to be considered in allowing or disallowing a motion for intervention. It is significant to note at this
juncture that as per records, there are four pending cases involving the parties herein, enumerated as follows: [1] Special
Proceedings No. 122122 before the CFI of Manila, Branch XXII, entitled "Concepcion Magsaysay-Labrador, et al. v. Subic
Land Corp., et al.", involving the validity of the transfer by the late Genaro Magsaysay of one-half of his shareholdings in
Subic Land Corporation; [2] Civil Case No. 2577-0 before the CFI of Zambales, Branch III, "Adelaida Rodriguez-Magsaysay
v. Panganiban, etc.; Concepcion Labrador, et al. Intervenors", seeking to annul the purported Deed of Assignment in favor
of SUBIC and its annotation at the back of TCT No. 3258 in the name of respondent's deceased husband; [3] SEC Case
No. 001770, filed by respondent praying, among other things that she be declared in her capacity as the surviving spouse
and administratrix of the estate of Genaro Magsaysay as the sole subscriber and stockholder of SUBIC. There, petitioners,
by motion, sought to intervene. Their motion to reconsider the denial of their motion to intervene was granted; [4] SP No. Q-
26739 before the CFI of Rizal, Branch IV, petitioners herein filing a contingent claim pursuant to Section 5, Rule 86, Revised
Rules of Court. 9 Petitioners' interests are no doubt amply protected in these cases. LLpr
Neither do we lend credence to petitioners' argument that they are more interested in the outcome of the case than the
corporation-assignee, owing to the fact that the latter is willing to compromise with widow-respondent and since a
compromise involves the giving of reciprocal concessions, the only conceivable concession the corporation may give is a
total or partial relinquishment of the corporate assets. 10
Such claim all the more bolsters the contingent nature of petitioners' interest in the subject of litigation.
The factual finding of the trial court are clear on this point. The petitioner cannot claim the right to intervene on the strength
of the transfer of shares allegedly executed by the late Senator. The corporation did not keep books and records. 11
Perforce, no transfer was ever recorded, much less affected as to prejudice third parties. The transfer must be registered in
the books of the corporation to affect third persons. The law on corporations is explicit, Section 63 of the Corporation Code
provides, thus: "No transfer, however, shall be valid, except as between the parties, until the transfer is recorded in the
books of the corporation showing the names of the parties to the transaction, the date of the transfer, the number of the
certificate or certificates and the number of shares transferred." LexLib
And even assuming arguendo that there was a valid transfer, petitioners are nonetheless barred from intervening inasmuch
as their right can be ventilated and amply protected in another proceeding.
WHEREFORE, the instant petition is hereby DENIED. Costs against petitioners.SO ORDERED.
||| (Magsaysay-Labrador v. Court of Appeals, G.R. No. 58168, [December 19, 1989], 259 PHIL 748-756)

[G.R. No. 96490. February 3, 1992.]

INDOPHIL TEXTILE MILL WORKERS UNION-PTGWO, petitioner, vs. VOLUNTARY ARBITRATOR


TEODORICO P. CALICA AND INDOPHIL TEXTILE MILLS, INC., respondents.

Romeo C. Lagman for petitioner.


Borreta, Gutierrez & Leogardo for respondent Indophil Textile Mills, Inc.

SYLLABUS

1. LABOR LAW; VOLUNTARY ARBITRATION; DECISIONS OF VOLUNTARY ARBITRATORS GIVEN


HIGHEST RESPECT AND FINALITY; EXCEPTIONS. — Time and again, We stress that the decisions of voluntary
arbitrators are to be given the highest respect and a certain measure of finality, but this is not a hard and fast rule, it
does not preclude judicial review thereof where want of jurisdiction, grave abuse of discretion, violation of due process,
denial of substantial justice, or erroneous interpretation of the law were brought to our attention. (see Ocampo, et al. v.
National Labor Relations Commission, G.R. No. 81677, 25 July 1990, First Division Minute Resolution citing Oceanic
Bic Division (FFW) v. Romero, G.R. No. L-43890, July 16, 1984, 130 SCRA 392).
2. COMMERCIAL LAW; CORPORATION LAW; DOCTRINE OF PIERCING THE VEIL OF CORPORATE
ENTITY; EXPLAINED. — Under the doctrine of piercing the veil of corporate entity, when valid grounds therefore exist,
the legal fiction that a corporation is an entity with a juridical personality separate and distinct from its members or
stockholders may be disregarded. In such cases, the corporation will be considered as a mere association of persons.
The members or stockholders or the corporation will be considered as the corporation, that is liability will attach directly
to the officers and stockholders. The doctrine applies when the corporate fiction is used to defeat public convenience,
justify wrong, protect fraud, or defend crime, or when it is made as a shield to confuse the legitimate issues, or where a
corporation is the mere alter ego or business conduit of a person, or where the corporation is so organized and
controlled and its affairs are so conducted as to make it merely an instrumentality, agency, conduit or adjunct of another
corporation. (Umali et al. v. Court of Appeals, G.R. No. 89561, September 13, 1990, 189 SCRA 529, 542)
3. ID.; ID.; ID.; NOT APPLICABLE IN CASE AT BAR. — In the case at bar, petitioner seeks to pierce the veil of
corporate entity of Acrylic, alleging that the creation of the corporation is a devise to evade the application of the CBA
between petitioner Union and private respondent Company. While we do not discount the possibility of the similarities of
the businesses of private respondent and Acrylic, neither are we inclined to apply the doctrine invoked by petitioner in
granting the relief sought. The fact that the businesses of private respondent and Acrylic are related, that some of the
employees of the private respondent are the same persons manning and providing for auxiliary services to the units of
Acrylic, and that the physical plants, offices and facilities are situated in the same compound, it is our considered
opinion that these facts are not sufficient to justify the piercing of the corporate veil of Acrylic. In the same case of
Umali, et al. v. Court of Appeals (189 SCRA 529), We already emphasized that "the legal corporate entity is
disregarded only if it is sought to hold the officers and stockholders directly liable for a corporate debt or obligation." In
the instant case, petitioner does not seek to impose a claim against the members of the Acrylic. Furthermore, We
already ruled in the case of Diatagon Labor Federation Local 110 of the ULGWP v. Ople (supra) that it is grave abuse of
discretion to treat two companies as a single bargaining unit when these companies are indubitably distinct entities with
separate juridical personalities. Hence, the Acrylic not being an extension or expansion of private respondent, the rank-
and-file employees working at Acrylic should not be recognized as part of, and/or within the scope of the petitioner, as
the bargaining representative of private respondent.

DECISION

MEDIALDEA, J p:

This is a petition for certiorari seeking the nullification of the award issued by the respondent Voluntary
Arbitrator Teodorico P. Calica dated December 8, 1990 finding that Section 1 (c), Article I of the Collective Bargaining
Agreement between Indophil Textile Mills, Inc. and Indophil Textile Mill Workers Union-PTGWO does not extend to the
employees of Indophil Acrylic Manufacturing Corporation as an extension or expansion of Indophil Textile Mills,
Incorporated.
The antecedent facts are as follows:
Petitioner Indophil Textile Mill Workers Union-PTGWO is a legitimate labor organization duly registered with the
Department of Labor and Employment and the exclusive bargaining agent of all the rank-and-file employees of Indophil
Textile Mills, Incorporated. Respondent Teodorico P. Calica is impleaded in his official capacity as the Voluntary
Arbitrator of the National Conciliation and Mediation Board of the Department of Labor and Employment, while private
respondent Indophil Textile Mills, Inc. is a corporation engaged in the manufacture, sale and export of yarns of various
counts and kinds and of materials of kindred character and has its plants at Barrio Lambakin, Marilao, Bulacan.
In April, 1987, petitioner Indophil Textile Mill Workers Union-PTGWO and private respondent Indophil Textile
Mills, Inc. executed a collective bargaining agreement effective from April 1, 1987 to March 31, 1990.
On November 3, 1987, Indophil Acrylic Manufacturing Corporation was formed and registered with the
Securities and Exchange Commission. Subsequently, Acrylic applied for registration with the Board of Investments for
incentives under the 1987 Omnibus Investments Code. The application was approved on a preferred non-pioneer
status.
In 1988, Acrylic became operational and hired workers according to its own criteria and standards. Sometime in
July, 1989, the workers of Acrylic unionized and a duly certified collective bargaining agreement was executed.
In 1990 or a year after the workers of Acrylic have been unionized and a CBA executed, the petitioner union
claimed that the plant facilities built and set up by Acrylic should be considered as an extension or expansion of the
facilities of private respondent Company pursuant to Section 1(c), Article I of the CBA, to wit.
"c) This Agreement shall apply to the Company's plant facilities and installations and to any extension
and expansion thereat." (Rollo, p. 4)
In other words, it is the petitioner's contention that Acrylic is part of the Indophil bargaining unit.
The petitioner's contention was opposed by private respondent which submits that it is a juridical entity
separate and distinct from Acrylic.
The existing impasse led the petitioner and private respondent to enter into a submission agreement on
September 6, 1990. The parties jointly requested the public respondent to act as voluntary arbitrator in the resolution of
the pending labor dispute pertaining to the proper interpretation of the CBA provision.
After the parties submitted their respective position papers and replies, the public respondent Voluntary
Arbitrator rendered its award on December 8, 1990, the dispositive portion of which provides as follows:
"PREMISES CONSIDERED, it would be a strained interpretation and application of the questioned CBA
provision if we would extend to the employees of Acrylic the coverage clause of Indophil Textile Mills
CBA. Wherefore, an award is made to the effect that the proper interpretation and application of Sec. 1,
(c), Art. I, of the 1987 CBA do (sic) not extend to the employees of Acrylic as an extension or expansion
of Indophil Textile Mills, Inc." (Rollo, p. 21)
Hence, this petition raising four (4) issues, to wit:
"1. WHETHER OR NOT THE RESPONDENT ARBITRATOR ERRED IN INTERPRETING
SECTION 1 (c), ART I OF THE CBA BETWEEN PETITIONER UNION AND
RESPONDENT COMPANY.
"2. WHETHER OR NOT INDOPHIL ACRYLIC IS A SEPARATE AND DISTINCT ENTITY FROM
RESPONDENT COMPANY FOR PURPOSES OF UNION REPRESENTATION.
"3. WHETHER OR NOT THE RESPONDENT ARBITRATOR GRAVELY ABUSED HIS
DISCRETION AMOUNTING TO LACK OR IN EXCESS OF HIS JURISDICTION.
"4. WHETHER OR NOT THE RESPONDENT ARBITRATOR VIOLATED PETITIONER UNION'S
CARDINAL PRIMARY RIGHT TO DUE PROCESS." (Rollo, pp. 6-7)
The central issue submitted for arbitration is whether or not the operations in Indophil Acrylic Corporation are an
extension or expansion of private respondent Company. Corollary to the aforementioned issue is the question of
whether or not the rank-and-file employees working at Indophil Acrylic should be recognized as part of, and/or within the
scope of the bargaining unit.
Petitioner maintains that public respondent Arbitrator gravely erred in interpreting Section 1(c), Article I of the
CBA in its literal meaning without taking cognizance of the facts adduced that the creation of the aforesaid Indophil
Acrylic is but a devise of respondent Company to evade the application of the CBA between petitioner Union and
respondent Company.
Petitioner stresses that the articles of incorporation of the two corporations establish that the two entities are
engaged in the same kind of business, which is the manufacture and sale of yarns of various counts and kinds and of
other materials of kindred character or nature.
Contrary to petitioner's assertion, the public respondent through the Solicitor General argues that the Indophil
Acrylic Manufacturing Corporation is not an alter ego or an adjunct or business conduit of private respondent because it
has a separate legitimate business purpose. In addition, the Solicitor General alleges that the primary purpose of private
respondent is to engage in the business of manufacturing yarns of various counts and kinds and textiles. On the other
hand, the primary purpose of Indophil Acrylic is to manufacture, buy, sell at wholesale basis, barter, import, export and
otherwise deal in yarns of various counts and kinds. Hence, unlike private respondent, Indophil Acrylic cannot
manufacture textiles while private respondent cannot buy or import yarns.
Furthermore, petitioner emphasizes that the two corporations have practically the same incorporators, directors
and officers. In fact, of the total stock subscription of Indophil Acrylic, P1,749,970.00 which represents seventy percent
(70%) of the total subscription of P2,500,000.00 was subscribed to by respondent Company.
On this point, private respondent cited the case of Diatagon Labor Federation v. Ople, G.R. No. L-44493-94,
December 3, 1980, 101 SCRA 534 which ruled that two corporations cannot be treated as a single bargaining unit even
if their businesses are related. It submits that the fact that there are as many bargaining units as there are companies in
a conglomeration of companies is a positive proof that a corporation is endowed with a legal personality distinctly its
own, independent and separate from other corporations. (see Rollo, pp. 160-161)
Petitioner notes that the foregoing evidence sufficiently establish that Acrylic is but an extension or expansion of
private respondent, to wit:
(a) the two corporations have their physical plants, offices and facilities situated in the same compound,
at Barrio Lambakin, Marilao, Bulacan;
(b) many of private respondent's own machineries, such as dyeing machines, reeling, boiler, Kamitsus
among others, were transferred to and are now installed and being used in the Acrylic plant;
(c) the services of a number of units, departments or sections of private respondent are provided to
Acrylic; and
(d) the employees of private respondent are the same persons manning and servicing the units of Acrylic.
(see Rollo, pp. 12-13)
Private respondent insists that the existence of a bonafide business relationship between Acrylic and private
respondent is not a proof of being a single corporate entity because the services which are supposedly provided by it to
Acrylic are auxiliary services or activities which are not really essential in the actual production of Acrylic. It also pointed
out that the essential services are discharged exclusively by Acrylic personnel under the control and supervision of
Acrylic managers and supervisors.
In sum, petitioner insists that the public respondent committed grave abuse of discretion amounting to lack or in
excess of jurisdiction in erroneously interpreting the CBA provision and in failing to disregard the corporate entity of
Acrylic.
We find the petition devoid of merit.
Time and again, We stress that the decisions of voluntary arbitrators are to be given the highest respect and a
certain measure of finality, but this is not a hard and fast rule, it does not preclude judicial review thereof where want of
jurisdiction, grave abuse of discretion, violation of due process, denial of substantial justice, or erroneous interpretation
of the law were brought to our attention. (see Ocampo, et al. v. National Labor Relations Commission, G.R. No. 81677,
25 July 1990, First Division Minute Resolution citing Oceanic Bic Division (FFW) v. Romero, G.R. No. L-43890, July 16,
1984, 130 SCRA 392).
It should be emphasized that in rendering the subject arbitral award, the voluntary arbitrator Teodorico Calica, a
professor of the U.P. Asian Labor Education Center, now the Institute for Industrial Relations, found that the existing law
and jurisprudence on the matter, supported the private respondent's contentions. Contrary to petitioner's assertion,
public respondent cited facts and the law upon which he based the award. Hence, public respondent did not abuse his
discretion.
Under the doctrine of piercing the veil of corporate entity, when valid grounds therefore exist, the legal fiction
that a corporation is an entity with a juridical personality separate and distinct from its members or stockholders may be
disregarded. In such cases, the corporation will be considered as a mere association of persons. The members or
stockholders of the corporation will be considered as the corporation, that is liability will attach directly to the officers and
stockholders. The doctrine applies when the corporate fiction is used to defeat public convenience, justify wrong, protect
fraud, or defend crime, or when it is made as a shield to confuse the legitimate issues, or where a corporation is the
mere alter ego or business conduit of a person, or where the corporation is so organized and controlled and its affairs
are so conducted as to make it merely an instrumentality, agency, conduit or adjunct of another corporation. (Umali et
al. v. Court of Appeals, G.R. No. 89561, September 13, 1990, 189 SCRA 529, 542)
In the case at bar, petitioner seeks to pierce the veil of corporate entity of Acrylic, alleging that the creation of
the corporation is a devise to evade the application of the CBA between petitioner Union and private respondent
Company. While we do not discount the possibility of the similarities of the businesses of private respondent and
Acrylic, neither are we inclined to apply the doctrine invoked by petitioner in granting the relief sought. The fact that the
businesses of private respondent and Acrylic are related, that some of the employees of the private respondent are the
same persons manning and providing for auxiliary services to the units of Acrylic, and that the physical plants, offices
and facilities are situated in the same compound, it is our considered opinion that these facts are not sufficient to justify
the piercing of the corporate veil of Acrylic.
In the same case of Umali, et al. v. Court of Appeals (supra), We already emphasized that "the legal corporate
entity is disregarded only if it is sought to hold the officers and stockholders directly liable for a corporate debt or
obligation." In the instant case, petitioner does not seek to impose a claim against the members of the Acrylic.
Furthermore, We already ruled in the case of Diatagon Labor Federation Local 110 of the ULGWP v. Ople
(supra) that it is grave abuse of discretion to treat two companies as a single bargaining unit when these companies are
indubitably distinct entities with separate juridical personalities.
Hence, the Acrylic not being an extension or expansion of private respondent, the rank-and-file employees
working at Acrylic should not be recognized as part of, and/or within the scope of the petitioner, as the bargaining
representative of private respondent.
All premises considered, the Court is convinced that the public respondent Voluntary Arbitrator did not commit
grave abuse of discretion in its interpretation of Section 1(c), Article I of the CBA that the Acrylic is not an extension or
expansion of private respondent.
ACCORDINGLY, the petition is DENIED and the award of the respondent Voluntary Arbitrator is hereby
AFFIRMED. SO ORDERED.
||| (Indophil Textile Mill Workers Union-PTGWO v. Calica, G.R. No. 96490, [February 3, 1992], 282 PHIL 725-734)

[G.R. No. 80043. June 6, 1991.]

ROBERTO A. JACINTO, petitioner, vs. HONORABLE COURT OF APPEALS AND METROPOLITAN


BANK AND TRUST COMPANY, respondents.

Romeo G. Carlos for petitioner.


Jorge, Perez & Associates for private respondents.

SYLLABUS

1. REMEDIAL LAW; CIVIL PROCEDURE; AMENDMENT OF PLEADING TO CONFORM TO OR AUTHORIZE


PRESENTATION OF EVIDENCE. — Petitioner, faults the courts below for piercing the veil of corporate fiction despite the
absence of any allegation in the complaint questioning the separate identity and existence of Inland Industries, Inc. This is
not accurate. While on the face of the complaint there is no specific allegation that the corporation is a mere alter ego of
petitioner, subsequent developments, from the stipulation of facts up to the presentation of evidence and the examination of
witnesses, unequivocably show that respondent Metropolitan Bank and Trust Company sought to prove that petitioner and
the corporation are one or that he is the corporation. No serious objection was heard from petitioner. Pursuant to Section 5
of Rule 10 of the Rules of Court "when evidence is presented by one party, with the express or implied consent of the
adverse party, as to issues not alleged in the pleadings, judgment may be rendered validly as regards those issues, which
shall be considered as if they have been raised in the pleadings. There is implied consent to the evidence thus presented
when the adverse party fails to object thereto."

DECISION

DAVIDE, JR., J p:

This is an appeal by certiorari to partially set aside the Decision of the Court of Appeals in C.A.-G.R. CV No. 08153 1
promulgated on 19 August 1987, which affirmed in toto the decision of the Regional Trial Court of Manila, Branch 11, in Civil
Case No. 133164 entitled "Metropolitan Bank and Trust Co. vs. Inland Industries Inc. and Roberto Jacinto," the dispositive
portion of which reads:
"WHEREFORE, judgment is hereby rendered ordering defendants to pay, jointly and severally, the
plaintiff, the principal obligation of P382,015.80 (Annex J-1 to J-3 of Stipulation), with interest charges
thereon at the rate of 16% per annum from January 1, 1979 up to the time the said amount is fully paid,
plus the sum of P20,000.00 as attorney's fees. Said defendants are further ordered to pay in solidum the
costs of this suit.
SO ORDERED." 2
Petitioner's co-defendant in the courts below, Inland Industries Inc., just as in the case of petitioner's motion to reconsider
the questioned decision, 3 chose not to join him in this appeal.
In Our resolution of 28 August 1988 We required the respondents to comment on the petition. Respondent Metropolitan
Bank and Trust Co. filed its comment 4 on 12 October 1988. We required the petitioner to file a reply thereto, 5 which he
complied with on 20 December 1988. 6
We gave due course to the petition on 8 May 1989 7 and required the parties to submit their respective memoranda.
Private respondent filed its memorandum on 29 June 1989 8 while petitioner asked leave to adopt his petition and reply as
his memorandum, 9 which We granted on 14 June 1989. 10
Petitioner submits the following issues:
"1. Whether or not the respondent Court of Appeals can validly pierce the fiction of corporate identity of
the defendant corporation Inland Industries, Inc. even if there is no allegation in the complaint regarding
the same, nor is there anything in the prayer demanding the piercing of the corporate veil of the
corporation Inland Industries, Inc.;
2. Whether or not the Court of Appeals can validly pierce the fiction of corporate identity of the defendant
Inland Industries, Inc. even if absolutely no proof was presented in court to serve as legal justification for
the same."
We find this petition to be bereft of merit. The issues are basically factual and a careful scrutiny of the decisions of both
courts below reveals that their findings and conclusions on the matter of piercing the veil of corporate fiction and on the
liability of herein petitioner are overwhelmingly supported by the evidence. cdrep
Insofar as material and relevant to the issues raised, the trial court found and held: 11
"As to [the] liability of [the] defendant Roberto A. Jacinto, it would appear that he is in factetum (sic), or, in
fact, the corporation itself known as Inland Industries, Inc. Aside from the fact that he is admittedly the
President and General Manager of the corporation and a substantial stockholders (sic) thereof, it was
defendant Roberto A. Jacinto who dealt entirely with the plaintiff in those transactions. In the Trust
Receipts that he signed supposedly in behalf of Inland Industries, Inc., it is not even mentioned that he
did so in this official capacity.
xxx xxx xxx
In this case, the Court is satisfied that Roberto A. Jacinto was practically the corporation itself, the Inland
Industries, Inc."
In a detailed fashion, the respondent Court of Appeals brushed aside the posturings of petitioner as follows:
"Defendant Roberto Jacinto, tried to escape liability and shift the entire blame under the trust receipts
solely and exclusively on defendant-appellant corporation. He asserted that he cannot be held solidarily
liable with the latter (defendant corporation) because he just signed said instruments in his official
capacity as president of Inland Industries, Inc. and the latter (defendant corporation) has a juridical
personality distinct and separate from its officers and stockholders. It is likewise asserted, citing an
American case, that the principle of piercing the fiction of corporate entity should be applied with great
caution and not precipitately, because a dual personality by a corporation and its stockholders would
defeat the principal purpose for which a corporation is formed. Upon the other hand, plaintiff-appellee
reiterated its allegation in the complaint that defendant corporation is just a mere alter ego of defendant
Roberto Jacinto who is its President and General Manager, while the wife of the latter owns a majority of
its shares of stock.
Defendants-appellants' assertion is plainly without legal basis. This is shown by the undisputed fact that
Roberto Jacinto even admitted that he and his wife own 52% of the stocks of defendant corporation
(TSN, April 22, 1985, p. 6). We cannot accept as true the assertion of defendant Jacinto that he only
acted in his official capacity as President and General Manager of Inland Industries, Inc. when he signed
the aforesaid trust receipts. To Our mind the same is just a clever ruse and a convenient ploy to thwart
his personal liability therefor by taking refuge under the protective mantle of the separate corporate
personality of defendant corporation. LibLex
As could be expected, Roberto Jacinto in his direct testimony presented a different corporate scenario
regarding Inland Industries, Inc. and vehemently declared that it is Bienvenida Catabas who is its
President, while Aurora Heresa is its Chairman of the Board. His assertion on this point, however, is not
convincing in new of his admission in the same breath, that his wife, Hedy U. Jacinto, own (sic) with him
52% of the shares of stock of said corporation. Indeed, this circumstance even if standing alone cannot
but engender in the most unprejudiced mind doubt and misgiving why Catabas and Heresa would be
defendant corporation's President and Chairman of the Board, respectively. Pertinent portion of his
testimony on this point is quoted hereunder:
'Atty. Carlos Do you know the defendant Inland Industries, Inc.?
A Yes, sir. Because I am the General Manager of this corporation.
Q Aside from being the General Manager of the defendant corporation are you in any other way
connected with the same?
A I am also a stockholder.
Q Does your corporation have a Board of Directors?
A Yes, sir.
Q By the way, who are the stockholders of this corporation?
A Bienvenida Catabas, Aurora Heresa, Paz Yulo, Hedy Y. Jacinto and myself.
Q Who is the President of the defendant corporation?
A Bienvenida Catabas.
Q Who is the Chairman of the Board?
A Aurora Heresa.
Q Do you have any relation with Hedy Y. Jacinto?
A She is my wife.
Q If you combine the stockholdings of your wife together with yours and percentage wise, how
much is your equity?
Atty. Dizon raised some objections. However, the Court allowed the same.
A About 52% (Ibid., pp. 3-6)
Furthermore, a cursory perusal of the stipulation of facts clearly shows that defendant Roberto Jacinto
acted in his capacity as President and General Manager of Inland Industries, Inc. when he signed said
trust receipts. Pertinent portion of his testimony are quoted below:
'(d) All the goods covered by the three (3) Letters of Credit (Annexes "A", "B" & "C") and
paid for under the Bills of Exchange (Annexes "D", "E" & F") were delivered to and received by
defendant Inland Industries, Inc. through its co-defendant Roberto A. Jacinto, its President and
General Manager, who signed for and in behalf of defendant Inland and agreed to the terms and
conditions of three (3) separate trust receipts covering the same and herein identified as
follows: . . . .' (p. 3 of Stipulations of Facts and Formulation of Issues [p. 95, Records]).
The conflicting statements by defendant Jacinto place in extreme doubt his credibility anent his alleged
participation in said transactions and We are thus persuaded to agree with the findings of the lower court
that the latter (Roberto Jacinto) was practically the corporation itself. Indeed, a painstaking examination
of the records show that there is no clear-cut delimitation between the personality of Roberto Jacinto as
an individual and the personality of Inland Industries, Inc. as a corporation. cdll
The circumstances aforestated lead Us to conclude that the corporate veil that en-shrouds defendant
Inland Industries, Inc. could be validly pierced, and a host of cases decided by our High Court is
supportive of this view. Thus it held that 'when the veil of corporate fiction is made as a shield to
perpetuate fraud and or confuse legitimate issues, the same should be pierced.' (Republic vs. Razon, 20
SCRA 234; A.D. Santos, Inc. vs. Vasquez, 22 SCRA 1156; Emilio Cano Enterprises, Inc. vs. Court of
Appeals, 13 SCRA 290). Almost in the same vein is the dictum enunciated by the same court in the case
of Commissioner of Internal Revenue vs. Norton & Harrison Co., (11 SCRA 714), that Where a
corporation is merely an adjunct, business conduit or alter ego, the fiction of separate and distinct
corporate entity should be disregarded.'"
In its resolution of 29 September 1987, the respondent Court of Appeals, on the contention again of petitioner that the
finding that defendant corporation is his mere alter ego is not supported by the evidence and has no legal justification, ruled
that:
"The contention . . . is nothing but an empty assertion. A cursory perusal of the decision would at once
readily show on pages 11-13 of the same that said factual findings of the court is well grounded as the
same in fact even include a portion of the very testimony of said defendant-appellant admitting that he
and his wife own 52% of the stocks of defendant corporation. The stipulation of facts also show (sic) that
appellant Roberto Jacinto acted in his capacity as President/General Manager of defendant corporation
and that 'all the goods covered by the three (3) Letters of Credit (Annexes "A", "B" & "C") and paid for
under the Bills of Exchange (Annexes "D", "E" & "F") were delivered to and received by defendant Inland
Industries, Inc. through its co-defendant Roberto A. Jacinto, its President and General Manager, who
signed for and in behalf of defendant Inland and agreed to the terms and conditions of three (3) separate
trust receipts covering the same.'"
Petitioner, however, faults the courts below for piercing the veil of corporate fiction despite the absence of any allegation in
the complaint questioning the separate identity and existence of Inland Industries, Inc. This is not accurate. While on the
face of the complaint there is no specific allegation that the corporation is a mere alter ego of petitioner, subsequent
developments, from the stipulation of facts up to the presentation of evidence and the examination of witnesses,
unequivocably show that respondent Metropolitan Bank and Trust Company sought to prove that petitioner and the
corporation are one or that he is the corporation. No serious objection was heard from petitioner.
Section 5 of Rule 10 of the Rules of Court provides:
"SEC. 5. Amendment to conform to or authorize presentation of evidence. — When issues not raised by
the pleadings are tried by express or implied consent of the parties, they shall be treated in all respects,
as if they had been raised in the pleadings. Such amendment of the pleadings as may be necessary to
cause them to conform to the evidence and to raise these issues may be made upon motion of any party
at any time, even after judgment; but failure so to amend does not affect the trial of these issues. If the
evidence is objected to at the time of trial on the ground that it is not within the issues made by the
pleadings, the court may allow the pleadings to be amended and shall do so freely when the presentation
of the merits of the action will be subserved thereby and the objecting party fails to satisfy the court that
the admission of such evidence would prejudice him in maintaining his action or defense upon the merits.
The court may grant continuance to enable the objecting party to meet such evidence."
Pursuant thereto, "when evidence is presented by one party, with the express or implied consent of the adverse party, as to
issues not alleged in the pleadings, judgment may be rendered validly as regards those issues, which shall be considered
as if they have been raised in the pleadings. There is implied consent to the evidence thus presented when the adverse
party fails to object thereto." 12
WHEREFORE, for lack of merit, the Petition is DISMISSED with costs against petitioner. SO ORDERED.
||| (Jacinto v. Court of Appeals, G.R. No. 80043, [June 6, 1991], 275 PHIL 235-242)

[G.R. No. 108734. May 29, 1996.]

CONCEPT BUILDERS, INC., petitioner, vs. THE NATIONAL LABOR RELATIONS, COMMISSION,
(First Division); and Norberto Marabe, Rodolfo Raquel, Cristobal Riego, Manuel Gillego, Palcronio
Giducos, Pedro Aboigar, Norberto Comendador, Rogelio Salut, Emilio Garcia, Jr., Mariano Rio,
Paulina Basea, Alfredo Albera, Paquito Salut, Domingo Guarino, Romeo Galve, Dominador
Sabina, Felipe Radiana, Gavino Sualibio, Moreno Escares, Ferdinand Torres, Felipe Basilan, and
Ruben Robalos, respondents.

The Law Firm of Araullo and Raymundo for petitioner.


Ciriaco S. Cruz for private respondents.

SYLLABUS

1. COMMERCIAL LAW; CORPORATION LAW; DOCTRINE OF PIERCING THE VEIL OF CORPORATE ENTITY;
WHEN APPLICABLE. — It is a fundamental principle of corporation law that a corporation is an entity separate and distinct
from its stockholders and from other corporations to which it may be connected. But, this separate and distinct personality of
a corporation is merely a fiction created by law for convenience and to promote justice. So when the notion of separate
juridical personality is used to defeat public convenience, justify wrong, protect fraud or defend crime, or is used as a device
to defeat the labor laws, this separate personality of the corporation may be disregarded or the veil of corporate fiction
pierced. This is true likewise when the corporation is merely an adjunct, a business conduit or an alter ego of another
corporation.
2. ID.; ID.; ID.; PROBATIVE FACTORS OF IDENTITY THAT WILL JUSTIFY THE APPLICATION THEREOF. —
The conditions under which the juridical entity may be disregarded vary according to the peculiar facts and circumstances of
each case. No hard and fast rule can be accurately laid down, but certainly, there are some probative factors of identity that
will justify the application of the doctrine of piercing the corporate veil, to wit: "1. Stock ownership by one or common
ownership of both corporations. 2. Identity of directors and officers. 3. The manner of keeping corporate books and records.
4. Methods of conducting the business."
3. ID.; ID.; ID.; TEST IN DETERMINING THE APPLICABILITY THEREOF. — The test in determining the
applicability of the doctrine of piercing the veil of corporation fiction is as follows: "1. Control, not mere majority or complete
stock control, but complete domination, not only of finances but of policy and business practice in respect to the transaction
attacked so that the corporate entity as to this transaction had at the time no separate mind, will or existence of its own; 2.
Such control must have been used by the defendant to commit fraud or wrong, to perpetuate the violation of a statutory or
other positive legal duty, or dishonest and unjust act in contravention of plaintiff's legal rights; and 3. The aforesaid control
and breach of duty must proximately cause the injury or unjust loss complained of. The absence of any one of these
elements prevent 'piercing the corporate veil.' In applying the 'instrumentality' or 'alter ego' doctrine, the courts are
concerned with reality and not form, with how the corporation operated and the individual defendant's relationship to that
operation."
4. ID.; ID.; ID.; APPLICABLE IN CASE AT BAR. — In this case, the NLRC noted that, while petitioner claimed that it
ceased its business operations on April 29, 1986, it filed an Information Sheet with the Securities and Exchange
Commission on May 15, 1987, stating that its office address is at 355 Maysan Road, Valenzuela, Metro Manila. On the
other hand, HPPI, the third-party claimant, submitted on the same day, a similar information sheet stating that its office
address is at 355 Maysan Road, Valenzuela, Metro Manila. Furthermore, the NLRC stated that: "Both information sheets
were filed by the same Virgilio O. Casiño as the corporate secretary of both corporations. It would also not be amiss to note
that both corporations had the same president, the same board of directors, the same corporate officers, and substantially
the same subscribers. From the foregoing, it appears that, among other things, the respondent (herein petitioner) and the
third-party claimant shared the same address and/or premises. Under this circumstances, (sic) it cannot be said that the
property levied upon by the sheriff were not of respondents." Clearly, petitioner ceased its business operations in order to
evade the payment to private respondents of backwages and to bar their reinstatement to their former positions. HPPI is
obviously a business conduit of petitioner corporation and its emergence was skillfully orchestrated to avoid the financial
liability that already attached to petitioner corporation.
5. ID.; NATIONAL LABOR RELATIONS COMMISSION MANUAL OF EXECUTION OF JUDGMENT; SECTION 3,
RULE VII THEREOF; PROPERLY OBSERVED IN CASE AT BAR. — In view of the failure of the sheriff, in the case at bar,
to effect a levy upon the property subject of the execution, private respondents had no other recourse but to apply for a
break-open order after the third-party claim of HPPI was dismissed for lack of merit by the NLRC. This is in consonance with
Section 3, Rule VII of the NLRC Manual of Execution of Judgment which provides that: "Should the losing party, his agent
or representative, refuse or prohibit the Sheriff or his representative entry to the place where the property subject of
execution is located or kept, the judgment creditor may apply to the Commissioner or Labor Arbiter concerned for a break-
open order."

DECISION

HERMOSISIMA, JR., J p:

The corporate mask may be lifted and the corporate veil may be pierced when a corporation is just but the alter ego
of a person or of another corporation. Where badges of fraud exist; where public convenience is defeated; where a wrong is
sought to be justified thereby, the corporate fiction or the notion of legal entity should come to naught. The law in these
instances will regard the corporation as a mere association of persons and, in case of two corporations, merge them into
one.
Thus, where a sister corporation is used as a shield to evade a corporation's subsidiary liability for damages, the
corporation may not be heard to say that it has a personality separate and distinct from the other corporation. The piercing
of the corporate veil comes into play.
This special civil action ostensibly raises the question of whether the National Labor Relations Commission
committed grave abuse of discretion when it issued a "break-open order" to the sheriff to be enforced against personal
property found in the premises of petitioner's sister company.
Petitioner Concept Builders, Inc., a domestic corporation, with principal office at 355 Maysan Road, Valenzuela,
Metro Manila, is engaged in the construction business. Private respondents were employed by said company as laborers,
carpenters and riggers.
On November, 1981, private respondents were served individual written notices of termination of employment
by petitioner, effective on November 30, 1981. It was stated in the individual notices that their contracts of employment
had expired and the project in which they were hired had been completed.
Public respondent found it to be, the fact, however, that at the time of the termination of private respondent's
employment, the project in which they were hired had not yet been finished and completed. Petitioner had to engage
the services of sub-contractors whose workers performed the functions of private respondents.
Aggrieved, private respondents filed a complaint for illegal dismissal, unfair labor practice and non-payment of
their legal holiday pay, overtime pay and thirteenth-month pay against petitioner.
On December 19, 1984, the Labor Arbiter rendered judgment 1 ordering petitioner to reinstate private
respondents and to pay them back wages equivalent to one year or three hundred working days.
On November 27, 1985, the National Labor Relations Commission (NLRC) dismissed the motion for
reconsideration filed by petitioner on the ground that the said decision had already become final and executory. 2
On October 16, 1986, the NLRC Research and Information Department made the finding that private
respondents' backwages amounted to P199,800.00. 3
On October 29, 1986, the Labor Arbiter issued a writ of execution directing the sheriff to execute the Decision,
dated December 19, 1984. The writ was partially satisfied through garnishment of sums from petitioner's debtor, the
Metropolitan Waterworks and Sewerage Authority, in the amount of P81,385.34. Said amount was turned over to the
cashier of the NLRC.
On February 1, 1989, an Alias Writ of Execution was issued by the Labor Arbiter directing the sheriff to collect
from herein petitioner the sum of P117,414.76, representing the balance of the judgment award, and to reinstate private
respondents to their former positions.
On July 13, 1989, the sheriff issued a report stating that he tried to serve the alias writ of execution on petitioner
through the security guard on duty but the service was refused on the ground that petitioner no longer occupied the
premises.
On September 26, 1986, upon motion of private respondents, the Labor Arbiter issued a second alias writ of
execution.
The said writ had not been enforced by the special sheriff because, as stated in his progress report, dated
November 2, 1989:
1. All the employees inside petitioner's premises at 355 Maysan Road, Valenzuela, Metro Manila, claimed that
they were employees of Hydro Pipes Philippines, Inc. (HPPI) and not by respondent;
2. Levy was made upon personal properties he found in the premises;
3. Security guards with high-powered guns prevented him from removing the properties he had levied upon. 4
The said special sheriff recommended that a "break-open order" be issued to enable him to enter petitioner's
premises so that he could proceed with the public auction sale of the aforesaid personal properties on November 7,
1989.
On November 6, 1989, a certain Dennis Cuyegkeng filed a third-party claim with the Labor Arbiter alleging that
the properties sought to be levied upon by the sheriff were owned by Hydro (Phils.), Inc. (HPPI) of which he is the Vice-
President.
On November 23, 1989, private respondents filed a "Motion for Issuance of a Break-Open Order," alleging that
HPPI and petitioner corporation were owned by the same incorporator/stockholders. They also alleged that petitioner
temporarily suspended its business operations in order to evade its legal obligations to them and that private
respondents were willing to post an indemnity bond to answer for any damages which petitioner and HPPI may suffer
because of the issuance of the break-open order.
In support of their claim against HPPI, private respondents presented duly certified copies of the General
Information Sheet, dated May 15, 1987, submitted by petitioner to the Securities Exchange Commission (SEC) and the
General Information Sheet, dated May 15, 1987, submitted by HPPI to the Securities and Exchange Commission.
The General Information Sheet submitted by the petitioner revealed the following:

"1. Breakdown of Subscribed Capital

Name of Stockholder Amount Subscribed

HPPI P6,999,500.00

Antonio W. Lim 2,900,000.00

Dennis S. Cuyegkeng 300.00

Elisa C. Lim 100,000.00

Teodulo R. Dino 100.00

Virgilio O. Casino 100.00

2. Board of Directors

Antonio W. Lim Chairman

Dennis S. Cuyegkeng Member

Elisa C. Lim Member

Teodulo R. Dino Member

Virgilio O. Casino Member

3. Corporate Officers

Antonio W. Lim President

Dennis S. Cuyegkeng Assistant to the President


Elisa O. Lim Treasurer

Virgilio O. Casino Corporate Secretary

4. Principal Office

355 Maysan Road

Valenzuela, Metro Manila." 5

On the other hand, the General Information Sheet of HPPI revealed the following:

"1. Breakdown of Subscribed Capital

Name of Stockholder Amount Subscribed

Antonio W. Lim P400,000.00

Elisa C. Lim 57,700.00

AWL Trading 455,000.00

Dennis S. Cuyegkeng 40,100.00

Teodulo R. Dino 100.00

Virgilio O. Casino 100 00

2. Board of Directors

Antonio W. Lim Chairman

Elisa C. Lim Member

Dennis S. Cuyegkeng Member

Virgilio O. Casino Member

Teodulo R. Dino Member

3. Corporate Officers
Antonio W. Lim President

Dennis S. Cuyegkeng Assistant to the President

Elisa C. Lim Treasurer

Virgilio O. Casino Corporate Secretary

4. Principal Office

355 Maysan Road, Valenzuela, Metro Manila." 6

On February 1, 1990, HPPI filed an Opposition to private respondents' motion for issuance of a break-open order,
contending that HPPI is a corporation which is separate and distinct from petitioner. HPPI also alleged that the two
corporations are engaged in two different kinds of businesses, i.e., HPPI is a manufacturing firm while petitioner was then
engaged in construction.
On March 2, 1990, the Labor Arbiter issued an Order which denied private respondents' motion for break-open
order.
Private respondents then appealed to the NLRC. On April 23, 1992, the NLRC set aside the order of the Labor
Arbiter, issued a break-open order and directed private respondents to file a bond. Thereafter, it directed the sheriff to
proceed with the auction sale of the properties already levied upon. It dismissed the third-party claim for lack of merit.
Petitioner moved for reconsideration but the motion was denied by the NLRC in a Resolution, dated December 3,
1992.
Hence, the resort to the present petition.
Petitioner alleges that the NLRC committed grave abuse of discretion when it ordered the execution of its
decision despite a third-party claim on the levied property. Petitioner further contends, that the doctrine of piercing the
corporate veil should not have been applied, in this case, in the absence of any showing that it created HPPI in order to
evade its liability to private respondents. It also contends that HPPI is engaged in the manufacture and sale of steel,
concrete and iron pipes, a business which is distinct and separate from petitioner's construction business. Hence, it is of
no consequence that petitioner and HPPI shared the same premises, the same President and the same set of officers
and subscribers. 7
We find petitioner's contention to be unmeritorious.
It is a fundamental principle of corporation law that a corporation is an entity separate and distinct from its
stockholders and from other corporations to which it may be connected. 8 But, this separate and distinct personality of a
corporation is merely a fiction created by law for convenience and to promote justice. 9 So, when the notion of separate
juridical personality is used to defeat public convenience, justify wrong, protect fraud or defend crime, or is used as a
device to defeat the labor laws, 10 this separate personality of the corporation may be disregarded or the veil of
corporate fiction pierced. 11 This is true likewise when the corporation is merely an adjunct, a business conduit or an
alter ego of another corporation. 12
The conditions under which the juridical entity may be disregarded vary according to the peculiar facts and
circumstances of each case. No hard and fast rule can be accurately laid down, but certainly, there are some probative
factors of identity that will justify the application of the doctrine of piercing the corporate veil, to wit:
"1. Stock ownership by one or common ownership of both corporations.
2. Identity of directors and officers.
3. The manner of keeping corporate books and records.
4. Methods of conducting the business." 13
The SEC en banc explained the "instrumentality rule" which the courts have applied in disregarding the
separate juridical personality of corporations as follows:
"Where one corporation is so organized and controlled and its affairs are conducted so that it is,
in fact, a mere instrumentality or adjunct of the other, the fiction of the corporate entity of the
'instrumentality' may be disregarded. The control necessary to invoke the rule is not majority or even
complete stock control but such domination of finances, policies and practices that the controlled
corporation has, so to speak, no separate mind, will or existence of its own, and is but a conduit for its
principal. It must be kept in mind that the control must be shown to have been exercised at the time the
acts complained of took place. Moreover, the control and breach of duty must proximately cause the
injury or unjust loss for which the complaint is made."
The test in determining the applicability of the doctrine of piercing the veil of corporate fiction is as follows:
"1. Control, not mere majority or complete stock control, but complete domination, not only of
finances but of policy and business practice in respect to the transaction attacked so that the corporate
entity as to this transaction had at the time no separate mind, will or existence of its own;
2. Such control must have been used by the defendant to commit fraud or wrong, to perpetuate
the violation of a statutory or other positive legal duty, or dishonest and unjust act in contravention of
plaintiff's legal rights; and
3. The aforesaid control and breach of duty must proximately cause the injury or unjust loss
complained of:
The absence of any one of these elements prevents 'piercing the corporate veil'. In applying the
'instrumentality' or 'alter ego' doctrine, the courts are concerned with reality and not form, with how the
corporation operated and the individual defendant's relationship to that operation." 14
Thus, the question of whether a corporation is a mere alter ego, a mere sheet or paper corporation, a sham or a
subterfuge is purely one of fact. 15
In this case, the NLRC noted that, while petitioner claimed that it ceased its business operations on April 29,
1986, it filed an Information Sheet with the Securities and Exchange Commission on May 15, 1987, stating that its office
address is at 355 Maysan Road, Valenzuela, Metro Manila. On the other hand, HPPI, the third-party claimant, submitted
on the same day, a similar information sheet stating that its office address is at 355 Maysan Road, Valenzuela, Metro
Manila.
Furthermore, the NLRC stated that:
"Both information sheets were filed by the same Virgilio O. Casiño as the corporate secretary of
both corporations. It would also not be amiss to note that both corporations had the same president, the
same board of directors, the same corporate officers, and substantially the same subscribers.
From the foregoing, it appears that, among other things, the respondent (herein petitioner) and
the third-party claimant shared the same address and/or premises. Under this circumstances, (sic) it
cannot be said that the property levied upon by the sheriff were not of respondents. 16
Clearly, petitioner ceased its business operations in order to evade the payment to private respondents of
backwages and to bar their reinstatement to their former positions. HPPI is obviously a business conduit of petitioner
corporation and its emergence was skillfully orchestrated to avoid the financial liability that already attached to petitioner
corporation.
The facts in this case are analogous to Claparols v. Court of Industrial Relations, 17 where we had the occasion
to rule:
"Respondent court's findings that indeed the Claparols Steel and Nail Plant, which ceased
operation of June 30, 1957, was SUCCEEDED by the Claparols Steel Corporation effective the next day,
July 1, 1957, up to December 7, 1962, when the latter finally ceased to operate, were not disputed by
petitioner. It is very clear that the latter corporation was a continuation and successor of the first entity . . .
Both predecessors and successor were owned and controlled by petitioner Eduardo Claparols and there
was no break in the succession and continuity of the same business. This 'avoiding-the-liability' scheme
is very patent, considering that 90% of the subscribed shares of stock of the Claparols Steel Corporation
(the second corporation) was owned by respondent . . . Claparols himself, and all the assets of the
dissolved Claparols Steel and Nail Plant were turned over to the emerging Claparols Steel Corporation.
It is very obvious that the second corporation seeks the protective shield of a corporate fiction
whose veil in the present case could, and should, be pierced as it was deliberately and maliciously
designed to evade its financial obligation to its employees."
In view of the failure of the sheriff, in the case at bar, to effect a levy upon the property subject of the execution,
private respondents had no other recourse but to apply for a break-open order after the third-party claim of HPPI was
dismissed for lack of merit by the NLRC. This is in consonance with Section 3, Rule VII of the NLRC Manual of
Execution of Judgment which provides that:
"Should the losing party, his agent or representative, refuse or prohibit the Sheriff or his
representative entry to the place where the property subject of execution is located or kept, the judgment
creditor may apply to the Commission or Labor Arbiter concerned for a break-open order."
Furthermore, our perusal of the records shows that the twin requirements of due notice and hearing were
complied with. Petitioner and the third-party claimant were given the opportunity to submit evidence in support of their
claim.
Hence, the NLRC did not commit any grave abuse of discretion when it affirmed the break-open order issued
by the Labor Arbiter.
Finally, we do not find any reason to disturb the rule that factual findings of quasi-judicial agencies supported by
substantial evidence are binding on this Court and are entitled to great respect, in the absence of showing of grave
abuse of discretion. 18
WHEREFORE, the petition is DISMISSED and the assailed resolutions of the NLRC, dated April 23, 1992 and
December 3, 1992, are AFFIRMED.
SO ORDERED.
||| (Concept Builders, Inc. v. National Labor Relations Commission, G.R. No. 108734, [May 29, 1996], 326 PHIL 955-969)

[G.R. No. L-30822. July 31, 1975.]

EDUARDO CLAPAROLS, ROMULO AGSAM and/or CLAPAROLS STEEL AND NAIL PLANT,
petitioners, vs. COURT OF INDUSTRIAL RELATIONS, ALLIED WORKERS' ASSOCIATION and/or
DEMETRIO GARLITOS, ALFREDO ONGSUCO, JORGE SEMILLANO, SALVADOR DOROTEO,
ROSENDO ESPINOSA, LUDOVICO BALOPENOS, ASER AMANCIO, MAXIMO QUIOYO,
GAUDENCIO QUIOYO, and IGNACIO QUIOYO, respondents.

SYNOPSIS

Found guilty of union busting and of illegally dismissing the respondent workers, petitioners were ordered to reinstate the
former with backwages from date of dismissal to reinstatement. Petitioners opposed the execution of the judgment as well
as the order directing the court examiner to compute the bonuses aside from backwages. They contended that the company
had ceased to operate and, therefore, pursuant to Sta. Cecilia Sawmills v. CIR (L-19273, Feb. 20, 1964), the workers,
assuming they are entitled to backwages, should only be limited to three-months' pay. Respondent workers, however,
contended that the company was succeeded by another company which is controlled by the same stockholders. The Court
of Industrial Relations denied the opposition, and on appeal, the Supreme Court sustained the industrial court.
Thereafter, the Court of Industrial Relations after the recomputation of the award again directed the petitioners to pay the
respective backwages and bonuses of the respondents. When petitioners' opposition was denied, they again appealed to
the Supreme Court.
The Supreme Court denied the appeal with treble costs against petitioner.

SYLLABUS

1. JUDGMENTS; LAW OF THE CASE; JUDGMENT IN A PRIOR CASE INVOLVING THE SAME ISSUES CONSTITUTES
THE LAW OF THE CASE IN A SUBSEQUENT CASE. — Where the same issues invoked in a subsequent case were
raised and decided in a prior case, the resolution of the Supreme Court in the prior case which had long become final
constitutes the law of the case in the subsequent case.
2. LABOR RELATIONS; UNFAIR LABOR PRACTICE; REINSTATEMENT; WHEN BACKWAGES INCLUDED BONUSES.
— A bonus is not a demandable and enforceable obligation, except when it is made part of the wage or salary
compensation. Whether or not bonus forms part of wages depends upon the condition or circumstances for its payment. If it
is an additional compensation which the employer promised and agreed to give without any condition imposed for its
payment then it is part of the wage.
3. ID.; ID.; ID.; ID.; WHERE BONUS IS EARMARKED AS A MATTER OF TRADITION IT FORMS PART OF
RECOVERABLE WAGES FROM COMPANY. — An employee is not entitled to bonus where there is no showing that it had
been granted by the employer to its employees periodically or regularly as to become part of their wages or salaries. The
clear implication is that bonus is recoverable as part of the wage or salary where the employer regularly or periodically gives
it to employees. Thus, where the bonus for a given year is earmarked as a matter of tradition for distribution to employees,
and the company distributes bonuses even if the company has suffered losses, it becomes part of the recoverable wages
from the company.
4. ID.; CORPORATIONS; PIECING THE VEIL OF CORPORATE EXISTENCE. — The ruling in Sta. Cecilia Sawmills to the
effect that the recoverable backwages shall be limited to only three (3) months where the company had ceased operations,
does not apply to a case where the company after ceasing it as operations is succeeded by another company, which
continued the operations of the first entity, and its emergence was skillfully timed to avoid the financial liability that already
attached to its predecessor, and where the "avoiding-the-liability" scheme is patently shown by the fact that 90% of the
subscribed shares of stock of the second company was owned by the same person and all the assets of the dissolved
company were turned over to the new company. The second company cannot seek the protective shield of a corporate
function whose veil could and should be pierced as it was deliberately and maliciously designed to evade its financial
obligation to its employees.
5. CORPORATIONS; NOTION OF LEGAL ENTITY CANNOT BE USED TO DEFEAT PUBLIC CONVENIENCE. — When
the notion of legal entity is used to defeat public convenience, justify wrong, protect fraud, or defend crime, the law will
regard the corporation as an association of persons, or in the case of two persons, will merge them into one. Thus, where a
corporation is a dummy and serves no business purpose and is intended only as a blind, the corporate fiction may be
ignored. And where a corporation is merely an adjunct, business conduct or alter ego of another corporation the fiction of
separate and distinct corporate entities should be disregarded.

DECISION

MAKASIAR, J p:

A petition for certiorari to set aside the order of respondent Court of Industrial Relations dated May 30, 1969 directing
petitioners to pay back wages and bonuses to private respondents as well as its resolution of July 5, 1969 denying the
motion for reconsideration of said order in Case No. 32-ULP-Iloilo entitled "Allied Workers' Association, et. al., versus
Eduardo Claparols, et. al. . .
It appears that on August 6, 1957, a complaint for unfair labor practice was filed by herein private respondent Allied
Workers' Association, respondent Demetrio Garlitos and ten (10) respondent workers against herein petitioners on account
of the dismissal of respondent workers from petitioner Claparols Steel and Nail Plant.
On September 16, 1963, respondent Court rendered its decision finding "Mr. Claparols guilty of union busting and" of having
"dismissed said complainants because of their union activities," and ordering respondents "(1) To cease and desist from
committing unfair labor practices against their employees and laborers; (2) To reinstate said complainants to their former or
equivalent jobs, as soon as possible, with back wages from the date of their dismissal up to their actual reinstatement" (p.
12, Decision; p. 27, rec.).
A motion to reconsider the above decision was filed by herein petitioners, which respondent Court, sitting en banc, denied in
a resolution dated January 27, 1964.
On March 30, 1964, counsel for herein respondent workers (complainants in the ULP case) filed a motion for execution of
respondent Court's September 16, 1963 decision.
On May 14, 1964, respondent Court, in its order of September 16, 1963, granted execution and directed herein petitioners.
"to reinstate the above complainants to their former or equivalent jobs within five (5) days after receipt of
a copy of this order. In order to implement the award of back wages, the Chief of the Examining Division
or any of his assistants is hereby directed to proceed to the office of the respondents at Matab-ang,
Talisay, Negros Occidental, and examine its payrolls and other pertinent records and compute the back
wages of the complainants in accordance with the decision dated September 16, 1963, and, upon
termination, to submit his report as soon as possible for further disposition" (p. 7, Brief for Respondents,
p. 113, rec.).
which was reiterated by respondent Court in a subsequent order dated November 10, 1964 (pp. 7-8, Brief for
Respondents, p. 113, rec.).
On December 14, 1964, respondent workers were accompanied by the Chief of Police of Talisay, Negros Occidental to the
compound of herein petitioner company to report for reinstatement per order of the court. Respondent workers were,
however, refused reinstatement by company accountant Francisco Cusi for he had no order from plant owner Eduardo
Claparols nor from his lawyer Atty. Plaridel Katalbas, to reinstate respondent workers.
Again, on December 15, 1964, respondent workers were accompanied by a police officer to the company compound, but
then, they were again refused reinstatement by Cusi on the same ground.
On January 15, 1965, the CIR Chief Examiner submitted his report containing three computations, to wit:
"The first computation covers the period February 1, 1957 to October 31, 1964. The second is up to and
including December 7, 1962, when the corporation stopped operations, while the third is only up to June
30, 1957 when the Claparols Steel and Nail Plant ceased to operate" (Annex B, Petition for Review on
Certiorari, p. 14, Brief for appellees, p. 113, rec.).
with the explanation that:
"6. Since the records of the Claparols Steel Corporation show that it was established on July 1, 1957
succeeding the Claparols Steel and Nail Plant which ceased operations on June 30, 1957, and that the
Claparols Steel Corporation stopped operations on December 7, 1962, three (3) computations are
presented herein for the consideration of this Honorable Court" (p. 2, Report of Examiner, p. 29, rec.)
On January 23, 1965, petitioners filed an opposition alleging that under the circumstances presently engulfing the company,
petitioner Claparols could not personally reinstate respondent workers; that assuming the workers are entitled to back
wages, the same should only be limited to three months pursuant to the court ruling in the case of Sta. Cecilia Sawmills vs.
CIR (L-19273-74, February 20, 1964); and that since Claparols Steel Corporation ceased to operate on December 7, 1962,
re-employment of respondent workers cannot go beyond December 7, 1962.
A reply to petitioner's opposition was filed by respondent workers, alleging among others, that Claparols Steel and Nail
Plant and Claparols Steel and Nail Corporation are one and the same corporation controlled by petitioner Claparols, with the
latter corporation succeeding the former.
On November 28, 1966, after conducting a series of hearings on the report of the examiner, respondent Court issued an
order, the dispositive portion of which reads:
WHEREFORE, the Report of the Examiner filed on January 15, 1965, is hereby approved subject to the
foregoing findings and dispositions. Consequently, the Corporation Auditing Examiner is directed to
recompute the back wages of complainants Demetrio Garlitos and Alfredo Ongsuco on the basis of
P200.00 and P270.00 a month, respectively; to compute those of complainant Ignacio Quioyo as
aforesaid; to compute the deductible earnings of complainants Ongsuco, Jorge Semillano and Garlitos,
as found in the body of this order; and to compute the bonuses of each and every complainant, except
Honorato Quioyo. Thereafter, as soon as possible, the Examiner should submit a report in compliance
herewith of the Court's further disposition" (p. 24, Brief for Respondents, p. 113, rec.)

On December 7, 1966, a motion for reconsideration was filed by petitioner, assailing respondent Court's ruling that (1) the
ruling in the case of Sta. Cecilia Sawmills Inc. CIR, et. al. does not apply in the case at bar; and (2) that bonus should be
included in the recoverable wages.
On December 14, 1966, a counter-opposition was filed by private respondents alleging that petitioners' motion for
reconsideration was pro forma, it not making express reference to the testimony or documentary evidence or to the
provision of law alleged to be contrary to such findings or conclusions of respondent Court.
On February 8, 1967, respondent Court of Industrial Relations dismissed petitioners' motion for reconsideration for being
pro forma.
Whereupon, petitioners filed a petition for certiorari with this COURT in G.R. No. L-27272 to set aside the November 28,
1966 order of respondent Court, as well as its February 8, 1967 resolution. Petitioners assigned therein as errors of law the
very same assignment of errors it raises in the present case, to wit:
"I
"THE RESPONDENT COURT ERRED AND/OR ACTED WITH GRAVE ABUSE OF
DISCRETION, AMOUNTING TO LACK OF JURISDICTION, IN HOLDING IN THE ORDER
UNDER REVIEW THAT BONUSES SHOULD BE PAID TO THE RESPONDENT WORKERS
DESPITE THE FACT THAT THE SAME WAS NOT ADJUDICATED IN ITS ORIGINAL DECISION.
"II
"THE RESPONDENT COURT ERRED AND/OR ACTED WITH GRAVE ABUSE OF
DISCRETION, AMOUNTING TO LACK OF JURISDICTION, IN NOT APPLYING THE DOCTRINE
LAID DOWN BY THIS HONORABLE TRIBUNAL IN THE CASE OF 'STA. CECILIA SAWMILLS,
INC. VS. C.I.R., ET. AL.,' G.R. No. L-19273-74, PROMULGATED ON FEBRUARY 29, 1964" (pp.
10-11, rec.)
On April 27, 1967, the Supreme Court denied petitioners' petition for certiorari (p. 77, rec. of L-27272), which was reiterated
on May 19, 1967 (p. 27, Respondent's Brief, p. 113, rec.; p. 81, rec. of L-27272).
On May 3, 1967, private respondents moved to have the workers' back wages properly recomputed. A motion to the same
end was reiterated by private respondents on June 14, 1967.
On July 13, 1967, respondent Court directed a recomputation of the back wages of respondent workers in accordance with
its order dated November 28, 1966. The said order in part reads:
"WHEREFORE, the Chief Auditing Examiner of the Court or any of his assistants, is hereby directed to
recompute the back wages of the workers involved in this case in accordance with the Order of
November 28, 1966, within 20 days from receipt of a copy of this Order" (p. 28, Brief for Respondents, p.
113, rec.)
Then on March 21, 1968, the Chief Examiner came out with his report, the disputed portion of which
(regarding bonuses) reads:
"xxx xxx xxx
"4. The yearly bonuses of the employees and laborers of respondent corporation are given on the
following basis:
"Basic Additional:
"a. For every dependent 1% of monthly salary
"b. For every dependent in elementary grade 2% of monthly salary
"c. For every dependent in high school 3% of monthly salary
"d. For every dependent in college 5% of monthly salary
xxx xxx xxx
"7. The computed . . . bonuses after deducting the earnings elsewhere of Messrs. Ongsuco, Garlitos and
Semillano, are as follows:
"Name . . . Bonuses . . .
1. Alfredo Ongsuco P1,620.00
2. Demetrio Garlitos 1,200.00
3. Ignacio Quioyo 455.23
4. Aser Abancio 461.00
5. Ludovico Belopeños 752.05
6. Salvador Doroteo 714.70
7. Rosendo Espinosa 1,075.40
8. Gaudencio Quioyo 1,167.92
9. Jorge Semillano 1,212.08
10. Maximo Quioyo 449.41
———
Total P9,107.79"
(Pp. 30-31, Respondent's Brief, p. 113, rec.).
On April 16, 1968, petitioners filed their opposition to the report of the Examiner dated March 21, 1968 on grounds already
rejected by respondent Court in its order dated November 28, 1966, and by the Supreme Court also in its ruling in G.R. No.
L-27272.
On May 4, 1968, a rejoinder to petitioners' opposition was filed by private respondents, alleging among others "that the
grounds of petitioners' opposition were the same grounds raised by them before and passed upon by respondent Court and
this Honorable Tribunal; that this order of November 28, 1966 which passed upon these issues became final and executory
on June 3, 1967 from the Honorable Supreme Court. (Order of respondent Court dated July 13, 1967)." [P. 32, Brief for
Respondents, p. 113, rec.].
On July 26, 1968, private respondents filed their motion for approval of the Report of the Examiner submitted on March 21,
1968, alleging, among others, that petitioners, in their opposition, did not actually dispute the data elicited by the Chief
Examiner but rather harped on grounds which, as already stated, had already been turned down by the Supreme Court.
On October 19, 1968, herein private respondents filed their "Constancia", submitting the case for resolution of respondent
Court of Industrial Relations.
On May 30, 1969, respondent Court issued an order, subject of the present appeal, the dispositive portion of which reads:
"WHEREFORE, there being no proof offered to substantiate respondent Eduardo Claparols' opposition,
the Examiner's Report should be, and it is hereby, APPROVED. Consequently, pursuant to the decision
dated September 16, 1963, respondent . . . (petitioners herein) are hereby directed to pay the respective
back wages and bonuses of the complainants (respondents herein) . . ." (p 35, Brief for Respondents; p.
113, rec.; emphasis supplied).
On June 7, 1969, petitioners filed a motion for reconsideration on practically the same grounds previously raised by them.
On June 30, 1969, respondents filed an opposition to petitioners' motion for reconsideration, with the following allegations:
"1. The issues raised, namely, whether bonuses should be included in the award for back wages had
already been resolved by respondent court in its orders dated November 28, 1966, and December 7,
1966, and in the Resolution of the Honorable Supreme Court in G.R. No. L-27272 dated April 26, 1967
and May 19, 1967, and the same is already a settled and final issue.
"2. Petitioners' motion for reconsideration is merely a rehash of previous arguments, effete and
unrejuvenated, pro forma, and intended merely to delay the proceedings.
As correctly contended by private respondents, the present petition is barred by Our resolutions of April 26, 1967 and May
19, 1967 in G.R. No. L-27272 (Eduardo Claparols, et. al. vs. CIR, et. al.) [pp. 77-83, rec. of L-27272], dismissing said case,
wherein said petitioners invoked the applicability of the doctrine in Sta. Cecilia Sawmills, Inc. vs. CIR, et. al. (L-19273-74,
Feb. 29, 1964, 10 SCRA 433) and impugned the illegality of the order of respondent Court dated November 28, 1966
directing the computation and payment of the bonuses, aside from back wages on the ground that these bonuses were not
included in the decision of September 16, 1963, which had long become final.
The aforesaid resolutions in G.R. No. L-27272 constitute the law of the instant case, wherein herein petitioners raised again
practically the same issues invoked in the above mentioned case. The denial of the petition in G.R. No. L-27272 suffices to
warrant the denial of the present petition; and We need not go any further.
However, without lending a sympathetic ear to the obvious desire of herein petitioners of this Court to re-examine — which
would be an exercise in futility — the final ruling in G.R. No. L-27272, which as above-stated is the law of the instant case,
but solely to remind herein petitioners, We reiterate the governing principles.
WE uniformly held that "a bonus is not a demandable and enforceable obligation, except when it is made part of the wage
or salary compensation" (Philippine Education Co. vs. CIR and the Union of Philippine Co. Employees [NLU], 92 Phil. 381;
Ansay, et. al. vs. National Development Co., et. al., 107 Phil. 998, 999; Emphasis supplied).
In Atok Big Wedge Mining Co. vs. Atok Big Wedge Mutual Benefit Association (92 Phil. 754), this Court, thru Justice
Labrador, held:
"Whether or not bonus forms part of wages depends upon the condition or circumstance for its payment.
If it is an additional compensation WHICH THE EMPLOYER PROMISED AND AGREED to give without
any condition imposed for its payment . . . then it is part of the wage." (Emphasis supplied).
In Altomonte vs. Philippine American Drug Co. (106 Phil. 137), the Supreme Court held that an employee is not entitled to
bonus where there is no showing that it had been granted by the employer to its employees periodically or regularly as to
become part of their wages or salaries. The clear implication is that bonus is recoverable as part of the wage or salary
where the employer regularly or periodically gives it to employees.
American jurisprudence equally regards bonuses as part of compensation or recoverable wages.
Thus, it was held that ". . . it follows that in determining the regular rate of pay, a bonus which in fact constitutes PART OF
AN EMPLOYEE'S compensation, rather than a true gift or gratuity, has to be taken into consideration." (48 Am. Jur. 2d,
Labor and Labor Relations, No. 1555, citing the cases of Triple "AAA" Co. vs. Wirtz and Haber vs. Americana Corporation;
Emphasis supplied). It was further held that ". . . the regular rate includes incentive bonuses paid to the employees in
addition to the guaranteed base rates regardless of any contract provision to the contrary and even though such bonuses
could not be determined or paid until such time after the payday" (48 Am. Jur. 2d, Labor and Labor Relations, No. 1555,
citing the case of Walling vs. Harnischfeger Corp., 325 US 427, 89 L Ed 1711, 65 S Ct. 1246; Emphasis supplied).
Petitioners in the present case do not dispute that as a matter of tradition, the company has been doling out bonuses to
employees. In fact, the company balance sheets for the years 1956 to 1962 contained bonus and pension computations
which were never repudiated or questioned by petitioners. As such, bonus for a given year earmarked as a matter of
tradition for distribution to employees has formed part of their recoverable wages from the company. Moreover, with greater
reason, should recovery of bonuses as part of back wages be observed in the present case since the company, in the light
of the very admission of company accountant Francisco Cusi, distributes bonuses to its employees even if the company has
suffered losses. Specifically, petitioner company has done this in 1962 (t.s.n., p. 149, Sept. 20, 1965).

Since bonuses are part of back wages of private respondents, the order of May 30, 1969, directing the payment of their
bonuses, did not amend the decision of September 16, 1963 of respondent Court directing payment of their wages, which
has long become final and executory, in the same way that the previous order of May 14, 1964 granting execution of said
decision of September 16, 1963 also directed the computation of the wages to be paid to private respondents as decreed by
the decision of September 16, 1963. All the orders of May 30, 1969, November 28, 1966 and May 14, 1964 merely
implement the already final and executory decision of September 16, 1963.
Petitioners insist that We adopt the ruling in the Sta. Cecilia Sawmills case wherein the recoverable back wages were
limited to only three (3) months: because as in the Sta. Cecilia Sawmills case, the Claparols Steel and Nail Plant ceased
operations due to enormous business reverses.
Respondent Court's findings that indeed the Claparols Steel and Nail Plant, which ceased operation of June 30, 1957, v. as
SUCCEEDED by the Claparols Steel Corporation effective the next day, July 1, 1957 up to December 7, 1962, when the
latter finally ceased to operate, were not disputed by petitioners. It is very clear that the latter corporation was a continuation
and successor of the first entity, and its emergence was skillfully timed to avoid the financial liability that already attached to
its predecessor, the Claparols Steel and Nail Plant. Both predecessors and successor were owned and controlled by
petitioner Eduardo Claparols and there was no break in the succession and continuity of the same business. This "avoiding-
the-liability" scheme is very patent, considering that 90% of the subscribed shares of stocks of the Claparols Steel
Corporation (the second corporation) was owned by respondent (herein petitioner) Claparols himself, and all the assets of
the dissolved Claparols Steel and Nail Plant were turned over to the emerging Claparols Steel Corporation.
It is very obvious that the second corporation seeks the protective shield of a corporate fiction whose veil in the present
case could, and should, be pierced as it was deliberately and maliciously designed to evade its financial obligation to its
employees.
It is well remembering that in Yutivo & Sons Hardware Company vs. Court of Tax Appeals (L-13203, Jan. 28, 1961, 1 SCRA
160), We held that when the notion of legal entity is used to defeat public convenience, justify wrong, protect fraud, or
defend crime, the law will regard the corporation as an association or persons, or, in the case of two corporations, will
merge them into one.
In Liddel & Company, Inc. vs. Collector of Internal Revenue (L-9687, June 30, 1961, 2 SCRA 632), this Court likewise held
that where a corporation is a dummy and serves no business purpose and is intended only as a blind, the corporate fiction
may be ignored.
In Commissioner of Internal Revenue vs. Norton and Harrison Company (L-17618, Aug. 31, 1964, 11 SCRA 714), We ruled
that where a corporation is merely an adjunct, business conduit or alter ego of another corporation, the fiction of separate
and distinct corporate entities should be disregarded.
To the same uniform effect are the decisions in the cases of Republic vs. Razon (L-17462, May 29, 1967, 20 SCRA 234)
and A.D. Santos, Inc. vs. Vasquez (L-23586, March 20, 1968, 22 SCRA 1156).
WE agree with respondent Court of Industrial Relations, therefore, that the amount of back wages recoverable by
respondent workers from petitioners should be the amount accruing up to December 7, 1962 when the Claparols Steel
Corporation ceased operations.
WHEREFORE, PETITION IS HEREBY DENIED WITH TREBLE COSTS AGAINST PETITIONERS TO BE PAID BY THEIR
COUNSEL.
||
(Claparols v. Court of Industrial Relations, G.R. No. L-30822, [July 31, 1975], 160 PHIL 624-637)

[G.R. No. L-23893. October 29, 1968.]

VILLA REY TRANSIT INC., plaintiff-appellant, vs. EUSEBIO E. FERRER, PANGASINAN


TRANSPORTATION CO., INC., and PUBLIC SERVICE COMMISSION, defendants, EUSEBIO E.
FERRER and PANGASINAN TRANSPORTATION CO., INC., defendants-appellants.

PANGASINAN TRANSPORTATION CO., INC., third-party plaintiff-appellant, vs. JOSE M. VILLARAMA,


third-party defendant-appellee.
Chuidian Law Office for plaintiff-appellant Villa Rey Transit, Inc.
Bengzon, Zarraga & Villegas for defendant-appellant Pangasinan Transportation Co., Inc.
Laurea & Pison for third-party defendant-appellee Jose Villarama.

SYLLABUS

1. COMMERCIAL LAW; CORPORATIONS; CORPORATION SEPARATE AND DISTINCT FROM MEMBERS


THEREOF; DOCTRINE OF PIERCING THE CORPORATE VEIL. — The doctrine that a corporation is a legal entity
distinct and separate from the members and stockholders who compose it is recognized and respected in all cases
which are within reason and the law. When the fiction is urged as a means of perpetrating a fraud or an illegal act or as
a vehicle for the evasion of an existing obligation, the circumvention of statutes, the achievement or perfection of a
monopoly or generally the perpetration of knavery or crime, the veil with which the law covers and isolates the
corporation from the members or stockholders who compose it will be lifted to allow for its consideration merely as an
aggregation of individuals.
2. ID.; ID.; ALTER EGO; CORPORATION BOUND BY CONTRACT WHEN SHOWN AS ALTER EGO OF
COVENANTOR. — The preponderance of evidence have shown that the Villa Rey Transit, Inc. is an alter ego of Jose
M. Villarama, and that the restrictive clause in the contract entered into by the latter and Pantranco is also enforceable
and binding against the said Corporation. For the rule is that a seller or promissor may not make use of a corporate
entity as a means of evading the obligation of his covenant. Where the corporation is substantially the alter ego of the
covenantor to the restrictive agreement, it can be enjoined from competing with the covenantee.
3. CIVIL LAW; CONTRACTS; VALIDITY OF STIPULATION IN RESTRAINT OF TRADE. — The 10-year
restrictive clause in the contract between Villarama and Pantranco, while in the nature of an agreement suppressing
competition, it is, however, merely ancillary or incidental to the main agreement which is that of sale. The suppression
or restraint is only partial or limited: first, in scope, it refers only to application for TPU by the seller in competition with
the lines sold to the buyer; second, in duration, it is only for ten (10) years; and third, with respect to situs or territory, the
restraint is only along the lines covered by the certificates sold. In view of these limitations, coupled with the
consideration of P350,000.00 for just two certificates of public convenience, and considering, furthermore, that the
disputed stipulation is only incidental to a main agreement, the same is reasonable and it is not harmful nor obnoxious
to public service. It does not appear that the ultimate result of the clause or stipulation would be to leave solely to
Pantranco the right to operate along the lines in question, thereby establishing a monopoly or predominance
approximating thereto. The main purpose of the restraint was to protect for a limited time the business of the buyer.
4. ID.; ID.; PURCHASER IN GOOD FAITH; RULE OF CAVEAT EMPTOR. — The 10 year prohibition upon
Villarama is not against his application for, or purchase of, certificates of public convenience, but merely the operation of
TPU along the lines covered by the certificates sold by him to Pantranco. Consequently, the sale between Fernando
and the Corporation is valid, such that the rightful ownership of the disputed certificates still belongs to the plaintiff being
the prior purchaser in good faith and for value thereof. In view of the ancient rule of caveat emptor prevailing in this
jurisdiction, what was acquired by Ferrer in the sheriff's sale was only the right which Fernando, judgment debtor, had in
the certificates of public convenience on the day of the sale.
5. ADMINISTRATIVE LAW; PUBLIC SERVICE LAW; CERTIFICATE OF PUBLIC CONVENIENCE; SALE
THEREOF; APPROVAL BY PUBLIC SERVICE COMMISSION NOT NECESSARY FOR THIS CONSUMMATION OF
THE SALE IN THE INSTANT CASE. — There is no merit in Pantranco and Ferrer's theory that the sale of the
certificates of public convenience in question, between the corporation and Fernando, was not consummated, it being
only a conditional sale subject to the suspensive condition of its approval by the Public Service Commission. It is clear,
that the requisite approval of the PSC is not a condition precedent for the validity and consummation of the sale.

DECISION

ANGELES, J p:
This is a tri-party appeal from the decision of the Court of First Instance of Manila, Civil Case No. 41845,
declaring null and void the sheriff's sale of two certificates of public convenience in favor of defendant Eusebio E. Ferrer
and the subsequent sale thereof by the latter to defendant Pangasinan Transportation Co., Inc.; declaring the plaintiff
Villa Rey Transit, Inc., to be the lawful owner of the said certificates of public convenience; and ordering the private
defendants, jointly and severally, to pay to the plaintiff, the sum of P5,000.00 as and for attorney's fees. The case
against the PSC was dismissed.
The rather ramified circumstances of the instant case can best be understood by a chronological narration of
the essential facts, to wit:
Prior to 1959, Jose M. Villarama was an operator of a bus transportation, under the business name of Villa Rey
Transit, pursuant to certificates of public convenience granted him by the Public Service Commission (PSC, for short) in
Cases Nos. 44213 and 104651, which authorized him to operate a total of thirty-two (32) units on various routes or lines
from Pangasinan to Manila, and vice-versa. On January 8, 1959, he sold the aforementioned two certificates of public
convenience to the Pangasinan Transportation Company, Inc. (otherwise known as Pantranco), for P350,000.00 with
the condition, among others, that the seller (Villarama) "shall not for a period of 10 years from the date of this sale,
apply for any TPU service identical or competing with the buyer."
Barely three months thereafter, or on March 6, 1959, a corporation called Villa Rey Transit, Inc. (which shall be
referred to hereafter as the Corporation) was organized with a capital stock of P500,000.00 divided into 5,000 shares of
the par value of P100.00 each; P200,000.00 was the subscribed stock; Natividad R. Villarama (wife of Jose M.
Villarama) was one of the incorporators, and she subscribed for P1,000.00; the balance of P199,000.00 was subscribed
by the brother and sister-in-law of Jose M. Villarama; of the subscribed capital stock, P105,000.00 was paid to the
treasurer of the corporation, who was Natividad R. Villarama.
In less than a month after its registration with the Securities and Exchange Commission (March 10, 1959), the
Corporation, on April 7, 1959, bought five certificates of public convenience, forty-nine buses, tools and equipment from
one Valentin Fernando, for the sum of P249,000.00, of which P100,000.00 was paid upon the signing of the contract;
P50,000.00 was payable upon the final approval of the sale by the PSC; P49,500.00 one year after the final approval of
the sale; and the balance of P50,000.00 "shall be paid by the BUYER to the different suppliers of the SELLER."
The very same day that the aforementioned contract of sale was executed, the parties thereto immediately
applied with the PSC for its approval, with a prayer for the issuance of a provisional authority in favor of the vendee
Corporation to operate the service therein involved. 1 On May 19, 1959, the PSC granted the provisional permit prayed
for, upon the condition that "it may be modified or revoked by the Commission at any time, shall be subject to whatever
action that may be taken on the basic application and shall be valid only during the pendency of said application."
Before the PSC could take final action on said application for approval of sale, however, the Sheriff of Manila, on July 7,
1959, levied on two of the five certificates of public convenience involved therein, namely those issued under PSC
cases Nos. 59494 and 63780, pursuant to a writ of execution issued by the Court of First Instance of Pangasinan in
Civil Case No. 13798, in favor of Eusebio Ferrer, plaintiff, judgment creditor, against Valentin Fernando, defendant,
judgment debtor. The Sheriff made and entered the levy in the records of the PSC. On July 16, 1959, a public sale was
conducted by the Sheriff of the said two certificates of public convenience. Ferrer was the highest bidder, and a
certificate of sale was issued in his name.
Thereafter, Ferrer sold the two certificates of public convenience to Pantranco, and jointly submitted for
approval their corresponding contract of sale to the PSC. 2 Pantranco therein prayed that it be authorized provisionally
to operate the service involved in the said two certificates.
The applications for approval of sale, filed before the PSC, by Fernando and the Corporation, Case No.
124057, and that of Ferrer and Pantranco, Case No. 126278, were scheduled for a joint hearing. In the meantime, to
wit, on July 22, 1959, the PSC issued an order disposing that during the pendency of the cases and before a final
resolution on the aforesaid applications, the Pantranco shall be the one to operate provisionally the service under the
two certificates embraced in the contract between Ferrer and Pantranco. The Corporation took issue with this particular
ruling of the PSC and elevated the matter to the Supreme Court, 3 which decreed, after deliberation, that until the issue
on the ownership of the disputed certificates shall have been finally settled by the proper court, the Corporation should
be the one to operate the lines provisionally.
On November 4, 1959, the Corporation filed in the Court of First Instance of Manila, a complaint for the
annulment of the sheriff's sale of the aforesaid two certificates of public convenience (PSC Cases Nos. 59494 and
63780) in favor of the defendant Ferrer, and the subsequent sale thereof by the latter to Pantranco, against Ferrer,
Pantranco and the PSC. The plaintiff Corporation prayed therein that all the orders of the PSC relative to the parties'
dispute over the said certificates be annulled.
In separate answers, the defendants Ferrer and Pantranco averred that the plaintiff Corporation had no valid
title to the certificates in question because the contract pursuant to which it acquired them from Fernando was subject to
a suspensive condition — the approval of the PSC — which has not yet been fulfilled, and, therefore, the Sheriff's levy
and the consequent sale at public auction of the certificates referred to, as well as the sale of the same by Ferrer to
Pantranco, were valid and regular, and vested unto Pantranco, a superior right thereto.
Pantranco, on its part, filed a third-party complaint against Jose M. Villarama, alleging that Villarama and the
Corporation, are one and the same; that Villarama and/or the Corporation was disqualified from operating the two
certificates in question by virtue of the aforementioned agreement between said Villarama and Pantranco, which
stipulated that Villarama "shall not for a period of 10 years from the date of this sale, apply for any TPU service identical
or competing with the buyer."
Upon the joinder of the issues in both the complaint and third- party complaint, the case was tried, and
thereafter decision was rendered in the terms as above stated.
As stated at the beginning, all the parties involved have appealed from the decision. They submitted a joint
record on appeal.
Pantranco disputes the correctness of the decision insofar as it holds that Villa Rey Transit, Inc. (Corporation) is
a distinct and separate entity from Jose M. Villarama, that the restriction clause in the contract of January 8, 1959
between Pantranco and Villarama is null and void, that the Sheriff's sale of July 16, 1959, is likewise null and void; and
the failure to award damages in its favor and against Villarama.
Ferrer, for his part, challenges the decision insofar as it holds that the sheriff's sale is null and void, and the sale
of the two certificates in question by Valentin Fernando to the Corporation, is valid. He also assails the award of
P5,000.00 as attorney's fees in favor of the Corporation, and the failure to award moral damages to him as prayed for in
his counterclaim.
The Corporation, on the other hand, prays for a review of that portion of the decision awarding only P5,000.00
as attorney's fees, and insisting that it is entitled to an award of P100,000.00 by way of exemplary damages.
After a careful study of the facts obtaining in the case, the vital issues to be resolved are: (1) Does the
stipulation between Villarama and Pantranco, as contained in the deed of sale, that the former "SHALL NOT FOR A
PERIOD OF 10 YEARS FROM THE DATE OF THIS SALE, APPLY FOR ANY TPU SERVICE IDENTICAL OR
COMPETING WITH THE BUYER," apply to new lines only or does it include existing lines?; (2) Assuming that said
stipulation covers all kinds of lines, is such stipulation valid and enforceable?; (3) In the affirmative, that said stipulation
is valid, did it bind the Corporation?
For convenience, We propose to discuss the foregoing issues by starting with the last proposition.
The evidence has disclosed that Villarama, albeit was not an incorporator or stockholder of the Corporation,
alleging that he did not become such, because he did not have sufficient funds to invest, his wife, however, was an
incorporator with the least subscribed number of shares, and was elected treasurer of the Corporation. The finances of
the Corporation which, under all concepts in the law, are supposed to be under the control and administration of the
treasurer keeping them as trust fund for the Corporation, were, nonetheless, manipulated and disbursed as if they were
the private funds of Villarama, in such a way and extent that Villarama appeared to be the actual owner-treasurer of the
business without regard to the rights of the stockholders. The following testimony of Villarama, 4 together with the other
evidence on record, attests to that effect:
"Q. Doctor, I want to go back again to the incorporation of the Villa Rey Transit, Inc. You heard
the testimony presented here by the bank regarding the initial opening deposit of ONE
HUNDRED FIVE THOUSAND PESOS, of which amount Eighty-Five Thousand Pesos
was a check drawn by yourself personally. In the direct examination you told the Court
that the reason you drew a check for Eighty-Five Thousand Pesos was because you and
your wife, or your wife, had spent the money of the stockholders given to her for
incorporation. Will you please tell the Honorable Court if you knew at the time your wife
was spending the money to pay debts, you personally know she was spending the
money of the incorporators?
"A. You know my money and my wife's money are one. We never talk about those things.
"Q. Doctor, your answer then is that since your money and your wife's money are one money
and you did not know when your wife was paying debts with the incorporator's money?
"A. Because sometimes she uses my money, and sometimes the money given to her she gives
to me and I deposit the money.
"Q. Actually, aside from your wife, you were also the custodian of some of the incorporators here,
in the beginning?
"A. Not necessarily, they give to my wife and when my wife hands to me I did not know it
belonged to the incorporators.
"Q. It supposes then your wife gives you some of the money received by her in her capacity as
treasurer of the corporation?
"A. Maybe.
"Q. What did you do with the money, deposit in a regular account?
"A. Deposit in my account.
"Q. Of all the money given to your wife, she did not receive any check?
"A. I do not remember.
"Q. Is it usual for you, Doctor, to be given Fifty Thousand Pesos without even asking what is
this?
xxx xxx xxx
JUDGE:Reform the question.
"Q. The subscription of your brother-in-law, Mr. Reyes, is Fifty Two Thousand Pesos, did your
wife give you Fifty-Two Thousand Pesos?
"A. I have testified before that sometimes my wife gives me money and I do not know exactly for
what."
The evidence further show that the initial cash capitalization of the corporation of P105,000.00 was mostly
financed by Villarama. Of the P105,000.00 deposited in the First National City Bank of New York, representing the initial
paid-up capital of the Corporation, P85,000.00 was covered by Villarama's personal check. The deposit slip for the said
amount of P105,000.00 was admitted in evidence as Exh. 23, which shows on its face that P20,000.00 was paid in cash
and P85,000.00 thereof was covered by Check No. F-50271 of the First National City Bank of New York. The
testimonies of Alfonso Sancho 5 and Joaquin Amansec, 6 both employees of said bank, have proved that the drawer of
the check was Jose Villarama himself.
Another witness, Celso Rivera, accountant of the Corporation, testified that while in the books of the corporation
there appears an entry that the treasurer received P95,000.00 as second installment of the paid-in subscriptions, and,
subsequently, also P100,000.00 as the first installment of the offer for second subscriptions worth P200,000.00 from the
original subscribers, yet Villarama directed him (Rivera) to make vouchers liquidating the sum. 7 Thus, it was made to
appear that the P95,000.00 was delivered to Villarama in payment for equipment purchased from him, and the
P100,000.00 was loaned as advances to the stockholders. The said accountant, however, testified that he was not
aware of any amount of money that had actually passed hands among the parties involved, 8 and actually the only
money of the corporation was the P105,000.00 covered by the deposit slip Exh. 23, of which, as mentioned above,
P85,000.00 was paid by Villarama's personal check.
Further, the evidence show that when the Corporation was in its initial months of operation, Villarama
purchased and paid with his personal checks Ford trucks for the Corporation. Exhibits 20 and 21 disclose that the said
purchases were paid by Philippine Bank of Commerce Checks Nos. 992618-B and 993621-B, respectively. These
checks have been sufficiently established by Fausto Abad, Assistant Accountant of Manila Trading & Supply Co., from
which the trucks were purchased 9 and Aristedes Solano, an employee of the Philippine Bank of Commerce, 10 as
having been drawn by Villarama.
Exhibits 6 to 19 and Exh. 22, which are photostatic copies of ledger entries and vouchers showing that
Villarama had co-mingled his personal funds and transactions with those made in the name of the Corporation, are very
illuminating evidence. Villarama has assailed the admissibility of these exhibits, contending that no evidentiary value
whatsoever should be given to them since "they were merely photostatic copies of the originals, the best evidence
being the originals themselves." According to him, at the time Pantranco offered the said exhibits, it was the most likely
possessor of the originals thereof because they were stolen from the files of the Corporation and only Pantranco was
able to produce the alleged photostat copies thereof.
Section 5 of Rule 130 of the Rules of Court provides for the requisites for the admissibility of secondary
evidence when the original is in the custody of the adverse party, thus: (1) opponent's possession of the original; (2)
reasonable notice to opponent to produce the original; (3) satisfactory proof of its existence; and (4) failure or refusal of
opponent to produce the original in court. 11 Villarama has practically admitted the second and fourth requisites. 12 As
to the third, he admitted their previous existence in the files of the Corporation and also that he had seen some of them.
13 Regarding the first element, Villarama's theory is that since even at the time of the issuance of the subpoena duces
tecum, the originals were already missing, therefore, the Corporation was no longer in possession of the same.
However, it is not necessary for a party seeking to introduce secondary evidence to show that the original is in the
actual possession of his adversary. It is enough that the circumstances are such as to indicate that the writing is in his
possession or under his control. Neither is it required that the party entitled to the custody of the instrument should, on
being notified to produce it, admit having it in his possession. 14 Hence, secondary evidence is admissible where he
denies having it in his possession. The party calling for such evidence may introduce a copy thereof as in the case of
loss. For, among the exception to the best evidence rule is "when the original has been lost, destroyed, or cannot be
produced in court. 15 The originals of the vouchers in question must be deemed to have been lost, as even the
Corporation admits such loss. Viewed upon this light, there can be no doubt as to the admissibility in evidence of
Exhibits 6 to 19 and 22.
Taking account of the foregoing evidence, together with Celso Rivera's testimony, 16 it would appear that:
Villarama supplied the organization expenses and the assets of the Corporation, such as trucks and equipments; 17
there was no actual payment by the original subscribers of the amounts of P95,000.00 and P100,000.00 as appearing
in the books; 18 Villarama made use of the money of the Corporation and deposited them to his private accounts; 19
and the Corporation paid his personal accounts. 20
Villarama himself admitted that he mingled the corporate funds with his own money. 21 He also admitted that
gasoline purchases of the Corporation were made in his name 22 because "he had existing account with Stanvac which
was properly secured and he wanted the Corporation to benefit from the rebates that he received." 23
The foregoing circumstances are strong persuasive evidence showing that Villarama has been too much
involved in the affairs of the Corporation to altogether negative the claim that he was only a part-time general manager.
They show beyond doubt that the Corporation is his alter ego.
It is significant that not a single one of the acts enumerated above as proof of Villarama's oneness with the
Corporation has been denied by him. On the contrary, he has admitted them with offered excuses.
Villarama has admitted, for instance, having paid P85,000.00 of the initial capital of the Corporation with the
lame excuse that "his wife had requested him to reimburse the amount entrusted to her by the incorporators and which
she had used to pay the obligations of Dr. Villarama (her husband) incurred while he was still the owner of Villa Rey
Transit, a single proprietorship. "But with his admission that he had received P350,000.00 from Pantranco for the sale of
the two certificates and one unit, 24 it becomes difficult to accept Villarama's explanation that he and his wife, after
consultation, 25 spent the money of their relatives (the stockholders) when they were supposed to have their own
money. Even if Pantranco paid the P50,000.00 in check to him, as claimed, it could have been easy for Villarama to
have deposited said check in his account and issued his own check to pay his obligations. And there is no evidence
adduced that the said amount of P350,000.00 was all spent or was insufficient to settle his prior obligations in his
business, and in the light of the stipulation in the deed of sale between Villarama and Pantranco that P350,000.00 of the
selling price was earmarked for the payments of accounts due to his creditors, the excuse appears unbelievable.
On his having paid for purchases by the Corporation of trucks from the Manila Trading & Supply Co. with his
personal checks, his reason was that he was only sharing with the Corporation his credit with some companies. And his
main reason for mingling his funds with that of the Corporation and for the latter's paying his private bills is that it would
be more convenient that he kept the money to be used in paying the registration fees on time, and since he had loaned
money to the Corporation, this would be set-off by the latter's paying his bills. Villarama admitted, however, that the
corporate funds in his possession were not only for registration fees but for other important obligations which were not
specified. 26
Indeed, while Villarama was not the Treasurer of the Corporation but was, allegedly, only a part-time Manager,
27 he admitted not only having held the corporate money but that he advanced and lent funds for the Corporation, and
yet there was no Board Resolution allowing it. 28
Villarama's explanation on the matter of his involvement with the corporate affairs of the Corporation only
renders more credible Pantranco's claim that his control over the corporation, especially in the management and
disposition of its funds, was so extensive and intimate that it is impossible to segregate and identify which money
belonged to whom. The interference of Villarama in the complex affairs of the corporation, and particularly its finances,
are much too inconsistent with the ends and purposes of the Corporation Law, which, precisely, seeks to separate
personal responsibilities from corporate undertakings. It is the very essence of incorporation that the acts and conduct
of the corporation be carried out in its own corporate name because it has its own personality.
The doctrine that a corporation is a legal entity distinct and separate from the members and stockholders who
compose it is recognized and respected in all cases which are within reason and the law. 29 When the fiction is urged
as a means of perpetrating a fraud or an illegal act or as a vehicle for the evasion of an existing obligation, the
circumvention of statutes, the achievement or perfection of a monopoly or generally the perpetration of knavery or
crime, 30 the veil with which the law covers and isolates the corporation from the members or stockholders who
compose it will be lifted to allow for its consideration merely as an aggregation of individuals.
Upon the foregoing considerations, We are of the opinion, and so hold, that the preponderance of evidence
have shown that the Villa Rey Transit, Inc. is an alter ego of Jose M. Villarama, and that the restrictive clause in the
contract entered into by the latter and Pantranco is also enforceable and binding against the said Corporation. For the
rule is that a seller or promissor may not make use of a corporate entity as a means of evading the obligation of his
covenant. 31 Where the Corporation is substantially the alter ego of the covenantor to the restrictive agreement, it can
be enjoined from competing with the covenantee. 32
The Corporation contends that even on the supposition that Villa Rey Transit, Inc. and Villarama are one and
the same, the restrictive clause in the contract between Villarama and Pantranco does not include the purchase of
existing lines but it only applies to application for new lines. The clause in dispute reads thus:
"(4) The SELLER shall not, for a period of ten (10) years from the date of this sale apply for any
TPU service identical or competing with the BUYER. " (Emphasis supplied)
As We read the disputed clause, it is evident from the context thereof that the intention of the parties was to
eliminate the seller as a competitor of the buyer for ten years along the lines of operation covered by the certificates of
public convenience subject of their transaction. The word "apply" as broadly used has for frame of reference, a service
by the seller on lines or routes that would compete with the buyer along the routes acquired by the latter. In this
jurisdiction, prior authorization is needed before anyone can operate a TPU service, 33 whether the service consists in
a new line or an old one acquired from a previous operator. The clear intention of the parties was to prevent the seller
from conducting any competitive line for 10 years since, anyway, he has bound himself not to apply for authorization to
operate along such lines for the duration of such period. 34
If the prohibition is to be applied only to the acquisition of new certificates of public convenience thru an
application with the Public Service Commission, this would, in effect, allow the seller just the same to compete with the
buyer as long as his authority to operate is only acquired thru transfer or sale from a previous operator, thus defeating
the intention of the parties. For what would prevent the seller, under the circumstances, from having a representative or
dummy apply in the latter's name and then later on transferring the same by sale to the seller? Since stipulations in a
contract is the law between the contracting parties,
"Every person must, in the exercise of his rights and in the performance of his duties, act with
justice, give everyone his due, and observe honesty and good faith." (Art. 19, New Civil Code.)
We are not impressed of Villarama's contention that the rewording of the two previous drafts of the contract of
sale between Villarama and Pantranco is significant in that as it now appears, the parties intended to effect the least
restriction. We are persuaded, after an examination of the supposed drafts, that the scope of the final stipulation, while
not as long and prolix as those in the drafts, is just as broad and comprehensive. At most, it can be said that the re-
wording was done merely for brevity and simplicity.
The evident intention behind the restriction was to eliminate the seller as a competitor, and this must be,
considering such factors as the good will 35 that the seller had already gained from the riding public and his adeptness
and proficiency in the trade. On this matter, Corbin, an authority on Contracts, has this to say: 36
"When one buys the business of another as a going concern, he usually wishes to keep it going;
he wishes to get the location, the building, the stock, in trade, and the customers. He wishes to step into
the seller's shoes and to enjoy the same business relations with other men. He is willing to pay much
more if he can get the `good will' of the business, meaning by this the good will of the customers, that
they may continue to tread the old footpath to his door and maintain with him the business relations
enjoyed by the seller.
" . . . In order to be well assured of this, he obtains and pays for the seller's promise not to
reopen business in competition with the business sold."
As to whether or not such a stipulation in restraint of trade is valid, our jurisprudence on the matter 37 says:
"The law concerning contracts which tend to restrain business or trade has gone through a long
series of changes from time to time with the changing condition of trade and commerce. With trifling
exceptions, said changes have been a continuous development of a general rule. The early cases show
plainly a disposition to avoid and annul all contract which prohibited or restrained any one from using a
lawful trade `at any time or at any place,' as being against the benefit of the state. Later, however, the
rule became well established that if the restraint was limited to `a certain time' and within `a certain place,'
such contracts were valid and not `against the benefit of the state.' Later cases, and we think the rule is
now well established, have held that a contract in restraint of trade is valid providing there is a limitation
upon either time or place. A contract, however, which restrains a man from entering into business or
trade without either a limitation as to time or place, will be held invalid.

"The public welfare of course must always be considered and if it be not involved and the
restraint upon one party is not greater than protection to the other requires, contracts like the one we are
discussing will be sustained. The general tendency, we believe, of modern authority, is to make the test
whether the restraint is reasonably necessary for the protection of the contracting parties. If the contract
is reasonable necessary to protect the interest of the parties, it will be upheld." (Emphasis supplied.)
Analyzing the characteristics of the questioned stipulation, We find that although it is in the nature of an
agreement suppressing competition, it is, however, merely ancillary or incidental to the main agreement which is that of
sale. The suppression or restraint is only partial or limited: first, in scope, it refers only to application for TPU by the
seller in competition with the lines sold to the buyer; second, in duration, it is only for ten (10) years; and third, with
respect to situs or territory, the restraint is only along the lines covered by the certificates sold. In view of these
limitations, coupled with the consideration of P350,000.00 for just two certificates of public convenience, and
considering, furthermore, that the disputed stipulation is only incidental to a main agreement, the same is reasonable
and it is not harmful nor obnoxious to public service. 38 It does not appear that the ultimate result of the clause or
stipulation would be to leave solely to Pantranco the right to operate along the lines in question, thereby establishing a
monopoly or predominance approximating thereto. We believe the main purpose of the restraint was to protect for a
limited time the business of the buyer.
Indeed, the evils of monopoly are farfetched here. There can be no danger of price controls or deterioration of
the service because of the close supervision of the Public Service Commission. 39 This Court had stated long ago 40
that "when one devotes his property to a use in which the public has an interest, he virtually grants to the public an
interest in that use and submits it to such public use under reasonable rules and regulations to be fixed by the Public
Utility Commission."
Regarding that aspect of the clause that it is merely ancillary or incidental to a lawful agreement, the underlying
reason sustaining its validity is well explained in 36 Am. Jur. 537-539, to it:
" . . . Numerous authorities hold that a covenant which is incidental to the sale and transfer of a
trade or business, and which purports to bind the seller not to engage in the same business in
competition with the purchaser, is lawful and enforceable. While such covenants are designed to prevent
competition on the part of the seller, it is ordinarily neither their purpose nor effect to stifle competition
generally in the locality, nor to prevent it at all in a way or to an extent injurious to the public. The
business in the hands of the purchaser is carried on just as it was in the hands of the seller; the former
merely takes the place of the latter; the commodities of the trade are as open to the public as they were
before; the same competition exists as existed before; there is the same employment furnished to others
after as before; the profits of the business go as they did before to swell the sum of public wealth; the
public has the same opportunities of purchasing, if it is a mercantile business; and production is not
lessened if it is a manufacturing plant."
The reliance by the lower court on the case of Red Line Transportation Co. v. Bachrach, 41 and finding that the
stipulation is illegal and void seems misplaced. In the said Red Line case, the agreement therein sought to be enforced
was virtually a division of territory between two operators, each company imposing upon itself an obligation not to
operate in any territory covered by the routes of the other. Restraints of this type, among common carriers, have always
been covered by the general rule invalidating agreements in restraint of trade. 42
Neither are the other cases relied upon by the plaintiff-appellee applicable to the instant case. In Pampanga
Bus Co. Inc. v. Enriquez, 43 the undertaking of the applicant therein not to apply for the lifting of restrictions imposed on
his certificates of public convenience was not an ancillary or incidental agreement. The restraint was the principal
objective. On the other hand, in Red Line Transportation Co. Inc. v. Gonzaga, 44 the restraint there in question not to
ask for extension of the line, or trips, or increase of equipment — was not an agreement between the parties but a
condition imposed in the certificate of public convenience itself.
Upon the foregoing considerations, Our conclusion is that the stipulation prohibiting Villarama for a period of 10
years to "apply" for TPU service along the lines covered by the certificates of public convenience sold by him to
Pantranco is valid and reasonable. Having arrived at this conclusion, and considering that the preponderance of the
evidence have shown that Villa Rey Transit, Inc. is itself the alter ego of Villarama, We hold, as prayed for in
Pantranco's third party complaint, that the said Corporation should, until the expiration of the 1-year period
abovementioned, be enjoined from operating the lines subject of the prohibition.
To avoid any misunderstanding, it is here to be emphasized that the 10-year prohibition upon Villarama is not
against his application for, or purchase of, certificates of public convenience, but merely the operation of TPU along the
lines covered by the certificates sold by him to Pantranco. Consequently, the sale between Fernando and the
Corporation is valid, such that the rightful ownership of the disputed certificates still belongs to the plaintiff being the
prior purchaser in good faith and for value thereof. In view of the ancient rule of caveat emptor prevailing in this
jurisdiction, what was acquired by Ferrer in the sheriff's sale was only the right which Fernando, judgment debtor, had in
the certificates of public convenience on the day of the sale. 45
Accordingly, by the "Notice of Levy Upon Personalty" the Commissioner of Public Service was notified that "by
virtue of an Order of Execution issued by the Court of First Instance of Pangasinan, the rights, interests, or participation
which the defendant, VALENTIN A. FERNANDO — in the above entitled case may have in the following
realty/personalty is attached or levied upon, to wit: The rights, interests and participation on the Certificates of Public
Convenience issued to Valentin A. Fernando, in Cases Nos. 59494, etc. . . . Lines — Manila to Lingayen, Dagupan, etc.
vice versa." Such notice of levy only shows that Ferrer, the vendee at auction of said certificates, merely stepped into
the shoes of the judgment debtor. Of the same principle is the provision of Article 1544 of the Civil Code, that "If the
same thing should have been sold to different vendees, the ownership shall be transferred to the person who may have
first taken possession thereof in good faith, if it should be movable property.
"There is no merit in Pantranco and Ferrer's theory that the sale of the certificates of public convenience in
question, between the Corporation and Fernando, was not consummated, it being only a conditional sale subject to the
suspensive condition of its approval by the Public Service Commission. "While section 20(g) of the Public Service Act
provides that "subject to established limitation and exceptions and saving provisions to the contrary, it shall be unlawful
for any public service or for the owner, lessee or operator thereof, without the approval and authorization of the
Commission previously had . . . to sell, alienate, mortgage, encumber or lease its property, franchise, certificates,
privileges, or rights or any part thereof, . . ., "the same section also provides:
" . . . Provided, however, that nothing herein contained shall be construed to prevent the
transaction from being negotiated or completed before its approval or to prevent the sale, alienation, or
lease by any public service of any of its property in the ordinary course of its business."
It is clear, therefore, that the requisite approval of the PSC is not a condition precedent for the validity and
consummation of the sale.
Anent the question of damages allegedly suffered by the parties, each of the appellants has its or his own
version to allege.
Villa Rey Transit, Inc. claims that by virtue of the "tortious acts" of defendants (Pantranco and Ferrer) in
acquiring the certificates of public convenience in question, despite constructive and actual knowledge on their part of a
prior sale executed by Fernando in favor of the said corporation, which necessitated the latter to file the action to annul
the sheriff's sale to Ferrer and the subsequent transfer to Pantranco, it is entitled to collect actual and compensatory
damages, and attorney's fees in the amount of P25,000.00. The evidence on record, however, does not clearly show
that said defendants acted in bad faith in their acquisition of the certificates in question. They believed that because the
bill of sale has yet to be approved by the Public Service Commission, the transaction was not a consummated sale,
and, therefore, the title to or ownership of the certificates was still with the seller. The award by the lower court of
attorney's fees of P5,000.00 in favor of Villa Rey Transit, Inc. is, therefore, without basis and should be set aside.
Eusebio Ferrer's charge that by reason of the filing of the action to annul the sheriff's sale, he had suffered and
should be awarded moral, exemplary damages and attorney's fees, cannot be entertained, in view of the conclusion
herein reached that the sale by Fernando to the Corporation was valid.
Pantranco, on the other hand, justifies its claim for damages with the allegation that when it purchased
Villarama's business for P350,000.00, it intended to build up the traffic along the lines covered by the certificates but it
was not afforded an opportunity to do so since barely three months had elapsed when the contract was violated by
Villarama operating along the same lines in the name of Villa Rey Transit, Inc. It is further claimed by Pantranco that the
underhanded manner in which Villarama violated the contract is pertinent in establishing punitive or moral damages. Its
contention as to the proper measure of damages is that it should be the purchase price of P350,000.00 that it paid to
Villarama. While We are fully in accord with Pantranco's claim of entitlement to damages it suffered as a result of
Villarama's breach of his contract with it, the record does not sufficiently supply the necessary evidentiary materials
upon which to base the award and there is need for further proceedings in the lower court to ascertain the proper
amount.
PREMISES CONSIDERED, the judgment appealed from is hereby modified as follows:
1. The sale of the two certificates of public convenience in question by Valentin Fernando to Villa Rey Transit,
Inc. is declared preferred over that made by the Sheriff at public auction of the aforesaid certificate of public
convenience in favor of Eusebio Ferrer;
2. Reversed, insofar as it dismisses the third-party complaint filed by Pangasinan Transportation Co. against
Jose M. Villarama, holding that Villa Rey Transit, Inc. is an entity distinct and separate from the personality of Jose M.
Villarama, and insofar as it awards the sum of P5,000.00 as attorney's fees in favor of Villa Rey Transit, Inc.;
3. The case is remanded to the trial court for the reception of evidence in consonance with the above findings
as regards the amount of damages suffered by Pantranco; and
4. On equitable considerations, without costs. So ordered.
||| (Villa Ray Transit Inc. v. Ferrer, G.R. No. L-23893, [October 29, 1968], 134 PHIL 796-818)

[G.R. No. 160039. June 29, 2004.]

RAYMUNDO ODANI SECOSA, EL BUENASENSO SY and DASSAD WAREHOUSING and PORT


SERVICES, INCORPORATED, petitioners, vs. HEIRS OF ERWIN SUAREZ FRANCISCO, respondents.

DECISION

YNARES-SANTIAGO, J p:

This is a petition for review under Rule 45 of the Rules of Court seeking the reversal of the decision 1 of the Court
of Appeals dated February 27, 2003 in CA-G.R. CV No. 61868, which affirmed in toto the June 19, 1998 decision 2 of
Branch 20 of the Regional Trial Court of Manila in Civil Case No. 96-79554.
The facts are as follows:
On June 27, 1996, at around 4:00 p.m., Erwin Suarez Francisco, an eighteen year old third year physical therapy
student of the Manila Central University, was riding a motorcycle along Radial 10 Avenue, near the Veteran Shipyard Gate
in the City of Manila. At the same time, petitioner, Raymundo Odani Secosa, was driving an Isuzu cargo truck with plate
number PCU-253 on the same road. The truck was owned by petitioner, Dassad Warehousing and Port Services, Inc.
Traveling behind the motorcycle driven by Francisco was a sand and gravel truck, which in turn was being tailed by
the Isuzu truck driven by Secosa. The three vehicles were traversing the southbound lane at a fairly high speed. When
Secosa overtook the sand and gravel truck, he bumped the motorcycle causing Francisco to fall. The rear wheels of the
Isuzu truck then ran over Francisco, which resulted in his instantaneous death. Fearing for his life, petitioner Secosa left his
truck and fled the scene of the collision. 3
Respondents, the parents of Erwin Francisco, thus filed an action for damages against Raymond Odani Secosa,
Dassad Warehousing and Port Services, Inc. and Dassad’s president, El Buenasucenso Sy. The complaint was docketed
as Civil Case No. 96-79554 of the RTC of Manila, Branch 20.
On June 19, 1998, after a full-blown trial, the court a quo rendered a decision in favor of herein respondents, the
dispositive portion of which states:
WHEREFORE, premised on the foregoing, judgment is hereby rendered in favor of the plaintiffs
ordering the defendants to pay plaintiffs jointly and severally:
1. The sum of P55,000.00 as actual and compensatory damages;
2. The sum of P20,000.00 for the repair of the motorcycle;
3. The sum of P100,000.00 for the loss of earning capacity;
4. The sum of P500,000.00 as moral damages;
5. The sum of P50,000.00 as exemplary damages;
6. The sum of P50,000.00 as attorney’s fees plus cost of suit.
SO ORDERED.
Petitioners appealed the decision to the Court of Appeals, which affirmed the appealed decision in toto. 4
Hence the present petition, based on the following arguments:
I.
THE COURT OF APPEALS SERIOUSLY ERRED WHEN IT AFFIRMED THE DECISION OF THE TRIAL
COURT THAT PETITIONER DASSAD DID NOT EXERCISE THE DILIGENCE OF A GOOD FATHER
OF A FAMILY IN THE SELECTION AND SUPERVISION OF ITS EMPLOYEES WHICH IS NOT IN
ACCORDANCE WITH ARTICLE 2180 OF THE NEW CIVIL CODE AND RELATED JURISPRUDENCE
ON THE MATTER.
II.
THE COURT OF APPEALS SERIOUSLY ERRED WHEN IT AFFIRMED THE DECISION OF THE TRIAL
COURT IN HOLDING PETITIONER EL BUENASENSO SY SOLIDARILY LIABLE WITH PETITIONERS
DASSAD AND SECOSA IN VIOLATION OF THE CORPORATION LAW AND RELATED
JURISPRUDENCE ON THE MATTER.
III.
THE JUDGMENT OF THE TRIAL COURT AS AFFIRMED BY THE COURT OF APPEALS AWARDING
P500,000.00 AS MORAL DAMAGES IS MANIFESTLY ABSURD, MISTAKEN AND UNJUST. 5
The petition is partly impressed with merit.
On the issue of whether petitioner Dassad Warehousing and Port Services, Inc. exercised the diligence of a good
father of a family in the selection and supervision of its employees, we find the assailed decision to be in full accord with
pertinent provisions of law and established jurisprudence.
Article 2176 of the Civil Code provides:
Whoever by act or omission causes damage to another, there being fault or negligence, is
obliged to pay for the damage done. Such fault or negligence, if there is no pre-existing contractual
relation between the parties, is called a quasi-delict and is governed by the provisions of this Chapter.
On the other hand, Article 2180, in pertinent part, states:
The obligation imposed by article 2176 is demandable not only for one’s own acts or omissions,
but also for those of persons for whom one is responsible . . .
Employers shall be liable for the damages caused by their employees and household helpers
acting within the scope of their assigned tasks, even though the former are not engaged in any business
or industry . . .
The responsibility treated of in this article shall cease when the persons herein mentioned prove
that they observed all the diligence of a good father of a family to prevent damage.
Based on the foregoing provisions, when an injury is caused by the negligence of an employee, there instantly
arises a presumption that there was negligence on the part of the employer either in the selection of his employee or in the
supervision over him after such selection. The presumption, however, may be rebutted by a clear showing on the part of the
employer that it exercised the care and diligence of a good father of a family in the selection and supervision of his
employee. Hence, to evade solidary liability for quasi-delict committed by an employee, the employer must adduce sufficient
proof that it exercised such degree of care. 6
How does an employer prove that he indeed exercised the diligence of a good father of a family in the selection and
supervision of his employee? The case of Metro Manila Transit Corporation v. Court of Appeals 7 is instructive:
In fine, the party, whether plaintiff or defendant, who asserts the affirmative of the issue has the
burden of presenting at the trial such amount of evidence required by law to obtain a favorable judgment
8 . . . In making proof in its or his case, it is paramount that the best and most complete evidence is
formally entered. 9
Coming now to the case at bar, while there is no rule which requires that testimonial evidence, to
hold sway, must be corroborated by documentary evidence, inasmuch as the witnesses’ testimonies
dwelt on mere generalities, we cannot consider the same as sufficiently persuasive proof that there was
observance of due diligence in the selection and supervision of employees. Petitioner’s attempt to prove
its “deligentissimi patris familias” in the selection and supervision of employees through oral evidence
must fail as it was unable to buttress the same with any other evidence, object or documentary, which
might obviate the apparent biased nature of the testimony. 10
Our view that the evidence for petitioner MMTC falls short of the required evidentiary quantum as
would convincingly and undoubtedly prove its observance of the diligence of a good father of a family has
its precursor in the underlying rationale pronounced in the earlier case of Central Taxicab Corp. vs. Ex-
Meralco Employees Transportation Co., et al., 11 set amidst an almost identical factual setting, where we
held that:
“The failure of the defendant company to produce in court any ‘record’ or other
documentary proof tending to establish that it had exercised all the diligence of a good father of a
family in the selection and supervision of its drivers and buses, notwithstanding the calls therefor
by both the trial court and the opposing counsel, argues strongly against its pretensions.
We are fully aware that there is no hard-and-fast rule on the quantum of evidence
needed to prove due observance of all the diligence of a good father of a family as would
constitute a valid defense to the legal presumption of negligence on the part of an employer or
master whose employee has by his negligence, caused damage to another. . . . (R)educing the
testimony of Albert to its proper proportion, we do not have enough trustworthy evidence left to
go by. We are of the considered opinion, therefore, that the believable evidence on the degree of
care and diligence that has been exercised in the selection and supervision of Roberto Leon y
Salazar, is not legally sufficient to overcome the presumption of negligence against the
defendant company.
The above-quoted ruling was reiterated in a recent case again involving the Metro Manila Transit Corporation, 12
thus:
In the selection of prospective employees, employers are required to examine them as to their
qualifications, experience, and service records. 13 On the other hand, with respect to the supervision of
employees, employers should formulate standard operating procedures, monitor their implementation,
and impose disciplinary measures for breaches thereof. To establish these factors in a trial involving the
issue of vicarious liability, employers must submit concrete proof, including documentary evidence.
In this case, MMTC sought to prove that it exercised the diligence of a good father of a family
with respect to the selection of employees by presenting mainly testimonial evidence on its hiring
procedure. According to MMTC, applicants are required to submit professional driving licenses,
certifications of work experience, and clearances from the National Bureau of Investigation; to undergo
tests of their driving skills, concentration, reflexes, and vision; and, to complete training programs on
traffic rules, vehicle maintenance, and standard operating procedures during emergency cases.
xxx xxx xxx
Although testimonies were offered that in the case of Pedro Musa all these precautions were
followed, the records of his interview, of the results of his examinations, and of his service were not
presented. . . [T]here is no record that Musa attended such training programs and passed the said
examinations before he was employed. No proof was presented that Musa did not have any record of
traffic violations. Nor were records of daily inspections, allegedly conducted by supervisors, ever
presented. . . The failure of MMTC to present such documentary proof puts in doubt the credibility of its
witnesses. AHTICD

Jurisprudentially, therefore, the employer must not merely present testimonial evidence to prove that he observed
the diligence of a good father of a family in the selection and supervision of his employee, but he must also support such
testimonial evidence with concrete or documentary evidence. The reason for this is to obviate the biased nature of the
employer’s testimony or that of his witnesses. 14
Applying the foregoing doctrines to the present case, we hold that petitioner Dassad Warehousing and Port
Services, Inc. failed to conclusively prove that it had exercised the requisite diligence of a good father of a family in the
selection and supervision of its employees.
Edilberto Duerme, the lone witness presented by Dassad Warehousing and Port Services, Inc. to support its
position that it had exercised the diligence of a good father of a family in the selection and supervision of its employees,
testified that he was the one who recommended petitioner Raymundo Secosa as a driver to Dassad Warehousing and Port
Services, Inc.; that it was his duty to scrutinize the capabilities of drivers; and that he believed petitioner to be physically and
mentally fit for he had undergone rigid training and attended the PPA safety seminar. 15
Petitioner Dassad Warehousing and Port Services, Inc. failed to support the testimony of its lone witness with
documentary evidence which would have strengthened its claim of due diligence in the selection and supervision of its
employees. Such an omission is fatal to its position, on account of which, Dassad can be rightfully held solidarily liable with
its co-petitioner Raymundo Secosa for the damages suffered by the heirs of Erwin Francisco.
However, we find that petitioner El Buenasenso Sy cannot be held solidarily liable with his co-petitioners. While it
may be true that Sy is the president of petitioner Dassad Warehousing and Port Services, Inc., such fact is not by itself
sufficient to hold him solidarily liable for the liabilities adjudged against his co-petitioners.
It is a settled precept in this jurisdiction that a corporation is invested by law with a personality separate from that of
its stockholders or members. 16 It has a personality separate and distinct from those of the persons composing it as well as
from that of any other entity to which it may be related. Mere ownership by a single stockholder or by another corporation of
all or nearly all of the capital stock of a corporation is not in itself sufficient ground for disregarding the separate corporate
personality. 17 A corporation’s authority to act and its liability for its actions are separate and apart from the individuals who
own it. 18
The so-called veil of corporation fiction treats as separate and distinct the affairs of a corporation and its officers
and stockholders. As a general rule, a corporation will be looked upon as a legal entity, unless and until sufficient reason to
the contrary appears. When the notion of legal entity is used to defeat public convenience, justify wrong, protect fraud, or
defend crime, the law will regard the corporation as an association of persons. 19 Also, the corporate entity may be
disregarded in the interest of justice in such cases as fraud that may work inequities among members of the corporation
internally, involving no rights of the public or third persons. In both instances, there must have been fraud and proof of it. For
the separate juridical personality of a corporation to be disregarded, the wrongdoing must be clearly and convincingly
established. 20 It cannot be presumed. 21
The records of this case are bereft of any evidence tending to show the presence of any grounds enumerated
above that will justify the piercing of the veil of corporate fiction such as to hold the president of Dassad Warehousing and
Port Services, Inc. solidarily liable with it.
The Isuzu cargo truck which ran over Erwin Francisco was registered in the name of Dassad Warehousing and Port
Services, Inc., and not in the name of El Buenasenso Sy. Raymundo Secosa is an employee of Dassad Warehousing and
Port Services, Inc. and not of El Buenasenso Sy. All these things, when taken collectively, point toward El Buenasenso Sy’s
exclusion from liability for damages arising from the death of Erwin Francisco.
Having both found Raymundo Secosa and Dassad Warehousing and Port Services, Inc. liable for negligence for
the death of Erwin Francisco on June 27, 1996, we now consider the question of moral damages which his parents, herein
respondents, are entitled to recover. Petitioners assail the award of moral damages of P500,000.00 for being manifestly
absurd, mistaken and unjust. We are not persuaded.
Under Article 2206, the “spouse, legitimate and illegitimate descendants and ascendants of the deceased may
demand moral damages for mental anguish for the death of the deceased.” The reason for the grant of moral damages has
been explained in this wise:
. . . the award of moral damages is aimed at a restoration, within the limits possible, of the
spiritual status quo ante; and therefore, it must be proportionate to the suffering inflicted. The intensity of
the pain experienced by the relatives of the victim is proportionate to the intensity of affection for him and
bears no relation whatsoever with the wealth or means of the offender.” 22
In the instant case, the spouses Francisco presented evidence of the searing pain that they felt when the premature
loss of their son was relayed to them. That pain was highly evident in the testimony of the father who was forever deprived
of a son, a son whose untimely death came at that point when the latter was nearing the culmination of every parent’s wish
to educate their children. The death of Francis has indeed left a void in the lives of the respondents. Antonio Francisco
testified on the effect of the death of his son, Francis, in this manner:
Q: (Atty. Balanag): What did you do when you learned that your son was killed on June 27,
1996?
A: (ANTONIO FRANCISCO): I boxed the door and pushed the image of St. Niño telling why this
happened to us.
Q: Mr. Witness, how did you feel when you learned of the untimely death of your son, Erwin
Suares (sic)?
A: Masakit po ang mawalan ng anak. It’s really hard for me, the thought that my son is dead.
xxx xxx xxx
Q: How did your family react to the death of Erwin Suarez Francisco?
A: All of my family and relatives were felt (sic) sorrow because they knew that my son is (sic)
good.
Q: We know that it is impossible to put money terms(s) [on] the life of [a] human, but since you
are now in court and if you were to ask this court how much would you and your family
compensate? (sic)
A: Even if they pay me millions, they cannot remove the anguish of my son (sic). 23
Moral damages are emphatically not intended to enrich a plaintiff at the expense of the defendant. They are
awarded to allow the former to obtain means, diversion or amusements that will serve to alleviate the moral suffering he has
undergone due to the defendant’s culpable action and must, perforce, be proportional to the suffering inflicted. 24 We have
previously held as proper an award of P500,000.00 as moral damages to the heirs of a deceased family member who died
in a vehicular accident. In our 2002 decision in Metro Manila Transit Corporation v. Court of Appeals, et al., 25 we affirmed
the award of moral damages of P500,000.00 to the heirs of the victim, a mother, who died from injuries she sustained when
a bus driven by an employee of the petitioner hit her. In the case at bar, we likewise affirm the portion of the assailed
decision awarding the moral damages.
Since the petitioners did not question the other damages adjudged against them by the court a quo, we affirm the
award of these damages to the respondents.
WHEREFORE, the petition is DENIED. The assailed decision is AFFIRMED with the MODIFICATION that petitioner
El Buenasenso Sy is ABSOLVED from any liability adjudged against his co-petitioners in this case.
Costs against petitioners. SO ORDERED.
||| (Secosa v. Heirs of Francisco, G.R. No. 160039, [June 29, 2004], 477 PHIL 317-330)

[G.R. Nos. 111810-11. June 16, 1995.]

JAMES YU and WILSON YOUNG, petitioners, vs. THE NATIONAL LABOR RELATIONS
COMMISSION, LABOR ARBITER DANIEL C. CUETO, TANDUAY DISTILLERY INC., FERNANDO
DURAN, EDUARDO PALIWAN, ROQUE ESTOCE AND RODRIGO SANTOS, respondents.

Ocampo, Quiroz, Mina & Associates for petitioners.


E.M.A. Cruz, Entero & Associates for private respondents.
Sycip, Salazar, Hernandez & Gatmaitan for Tanduay Distillery, Inc..

SYLLABUS
1. LABOR AND SOCIAL LEGISLATION; LABOR ARBITER; FINAL AND EXECUTORY DECISIONS THEREOF;
RULE AND EXCEPTION. — The decision dated May 24, 1989 is now final and executory, as only respondent TDI appealed
said decision and its appeal was later dismissed by respondent NLRC. It is fundamental that a final and executory decision
cannot be amended or corrected (First Integrated Bonding and Insurance Company, Inc. vs. Hernando, 199 SCRA 796
[1991]) except for clerical errors or mistakes (Maramba vs. Lozano, 20 SCRA 474 [1967]); Reyes vs. Court of Appeals, 189
SCRA 46 [1990]). A definitive judgment is no longer subject to change, revision, amendment, or reversal (Miranda vs. Court
of Appeals, 71 SCRA 295 l 1976]), and the court loses jurisdiction over it, except to order its execution (PY Eng Chong vs.
Herrera, 70 SCRA 130 [1976]).
2. ID.; ID.; ID.; WRIT OF EXECUTIONS MUST NOT GO BEYOND THE SCOPE OF THE JUDGMENT; NOT
COMPLIED WITH IN CASE AT BAR. — An examination of the dispositive portion of the decision shows that the same does
not in any manner obligate Tanduay Distillers, or even petitioners Yu and Young for that matter, to reinstate respondents.
Only TDI was held liable to reinstate respondents up to the time of change of ownership, and for separation benefits.
However, Labor Arbiter Cueto went beyond what was disposed by the decision and issued an order dated November 17,
1992 which required . . . Tanduay Distillers, Inc., Wilson Young and James Yu to immediately reinstate complainants
Fernando Duran, Rodrigo Santos, Roque Estoce and Eduardo Daliwan to their respective positions. Subsequently, a writ of
execution was issued on December 16, 1992 pursuant to the order of November 17, 1992. The order of execution dated
November 17, 1992 in effect amended the decision dated May 24, 1989 for the former orders petitioners and Tanduay
Distillers to reinstate private respondents employees whereas the decision dated May 24, 1989, as we have discussed
above, does not so decree. This cannot be done. It is beyond the power and competence of Labor Arbiter Cueto to amend a
final decision. The writ of execution must not go beyond the scope of the judgment. As Chief Justice Moran opined: "The
writ of execution must conform to the judgment which is to be executed, as it may not vary the terms of the judgment it
seeks to enforce. Nor may it go beyond the terms of the judgment, sought to be executed. Where the execution is not in
harmony with the judgment which gives it life and exceeds it, it has pro tanto no validity. To maintain otherwise would be to
ignore the constitutional provision against depriving a person of his property without due process of law" (Moran, Comments
on the Rules of Court, Vol. I, 1952 Ed., p. 809; cited in Villoria vs. Piccio, supra). (Gamboa's Incorporated vs. Court of
Appeals, 72 SCRA 131, 137-138 [1976].) The order of execution and the writ of execution ordering petitioners and Tanduay
Distillers to reinstate private respondents employees are, therefore, null and void. Neither may be said that petitioners and
Tanduay Distillers and the same as TDI, as seems to be the impression of respondents when they impleaded petitioners as
party respondents in their complaint for unfair labor practice, illegal lay off, and separation benefits. Such a stance is not
supported by the facts. The name of the company for whom the petitioners are working is Twin Ace Holdings Corporation.
As stated by the Solicitor General, Twin Ace is part of the Allied Bank Group although it conducts the rum business under
the name of Tanduay Distillers. The use of a similar sounding or almost identical name is an obvious device to capitalize on
the goodwill which Tanduay Rum has built over the years. Twin Ace or Tanduay Distillers, on one hand, and Tanduay
Distillery, Inc. (TDI), on the other, are distinct and separate corporations. There is nothing to suggest that the owners of TDI,
have any common relationship as to identify it with Allied Bank Group which runs Tanduay Distillers .
3. ID.; REINSTATEMENT; CANNOT BE COMPELLED AGAINST THE BUYING CORPORATION; CASE AT BAR.
— Labor Arbiter Cauton-Barcelona found the retrenchment effected by TDI illegal and ordered TDI to reinstate the
complainants and that if there is a change of management, then separation benefits would be paid. There is, however, no
order in the decision directing Twin Ace or Tanduay Distillers to hire or reinstate herein four individual respondents. The
letter of James Yu does not mention any reinstatement. It assures the president of the labor union that Tanduay Distillers
stood firm on its decision to hire employees with a clean slate on a probationary basis. The fact that the employees of the
former employer (TDI) would be hired on a probationary basis shows that there was no employer-employee relationship
between individual respondents and Twin Ace. Any one who joins the buyer corporation comes in as an outsider who is
newly hired and who starts on a probationary basis until he proves he deserves to be on a permanent status. His application
can be rejected in the exercise of the hiring authority's discretion. There is thus no legal basis for Labor Arbiter Cueto or the
NLRC to compel Twin Ace or Tanduay Distillers, or petitioners to "reinstate" the four individual respondents. The letter of
James Yu to the union president was a unilateral and gratuitous offer with no consideration. It refers to people who still have
to be hired. New hires had to be investigated or evaluated if they have "clean slates." Twin Ace or Tanduay Distillers and
petitioners are being compelled by public respondents to reinstate workers who were never their employees. There is no
showing that the sale of assets by TDI to Tanduay Distillers included a condition that employees of the former would be
absorbed by the latter. Employees of TDI had been terminated in their employment as of April 28, 1988. Petitioners state
that Twin Ace bought the assets of TDI after the employment of the individual respondents had been terminated. True,
Labor Arbiter Cauton-Barcelona declared the retrenchment program of TDI as illegal. This decision, however, did not
convert the individual respondents into employees of the firm which purchased the assets of the former employer. It merely
held TDI liable for the consequences of the illegal retrenchment. Labor Arbiter Cueto and the NLRC, therefore, committed
grave abuse of discretion when they read into the decision of Labor Arbiter Cauton-Barcelona something which did not
appear therein. And even assuming that Labor Arbiter Cauton-Barcelona formally included an order for the petitioners to
hire the individual respondents, there would be no factual or legal basis for such an order. An employer-employee relation is
created by contract, and cannot be forced upon either party simply upon order of a labor arbiter. The hiring of employees is
one of the recognized prerogatives of management.
4. ID.; ID.; RECEIPT OF SEPARATION PAY MILITATES AGAINST CLAIM THEREOF; CASE AT BAR. — Another
factor which militates against the claim for reinstatement of the individual respondents is their having received separation
pay as part of a compromise agreement in the course of their litigation with TDI. Rodrigo F. Santos received P20,282.03,
Roque Estoce, P20,092.50; Eduardo Daliwan, P19,973.40, and Fernando A. Duran, P25,702.00. These amounts
correspond to their entitlement to separation benefits. Having received separation pay from a former employer, how can
they compel, as a matter of right, another company to hire them on a supposed "reinstatement" basis? The orders executing
the earlier decision of Labor Arbiter Cauton-Barcelona and directing petitioners to immediately reinstate the four individual
respondents to their former positions are, thus, characterized by grave abuse of discretion. There are no "former positions"
to which individual respondents may be reinstated because they were never hired by Twin Ace/Tanduay Distillers and had
never worked for it.
5. COMMERCIAL LAW; CORPORATION; DOCTRINE OF PIERCING THE VEIL OF CORPORATE ENTITY;
WHEN APPLICABLE. — It is basic that a corporation is invested by law with a personality separate and distinct from those
of the persons composing it as well as from that of any other legal entity to which it may be related (Palay, Inc. et al. vs.
Clave, et al., 124 SCRA 641 [1983]). The genuine nature of the sale to Twin Ace is evidenced by the fact that Twin Ace was
only a subsequent interested buyer. At the time when termination notices were sent to its employees, TDI was negotiating
with the First Pacific Metro Corporation for the sale of its assets. Only after First Pacific gave up its efforts to acquire the
assets did Twin Ace or Tanduay Distillers come into the picture. Respondents-employees have not presented any proof as
to communality of ownership and management to support their contention that the two companies are one firm or closely
related. The doctrine of piercing the veil of corporate entity applies when the corporate fiction is used to defeat public
convenience, justify wrong, protect fraud, or defend crime or where a corporation is the mere alter ego or business conduit
of a person (Indophil Textile Mill Workers Union vs. Calica, 205 SCRA 697, 703 [1992]). To disregard the separate juridical
personality of a corporation, the wrong-doing must be clearly and convincingly established. It cannot be presumed (Del
Rosario vs. NLRC, 187 SCRA 777, 780 [1990]).
6. ID.; ID.; ID.; ID.; CASE AT BAR, A CASE OF. — The complaint for unfair labor practice, illegal lay off, and
separation benefits was filed against TDI. Only later when the manufacture and sale of Tanduay products was taken over by
Twin Ace or Tanduay Distillers were James Yu and Wilson Young impleaded. The corporation itself — Twin Ace or Tanduay
Distillers — was never made a party to the case. Another factor to consider is that TDI as a corporation or its shares of
stock were not purchased by Twin Ace. The buyer limited itself to purchasing most of the assets, equipment, and machinery
of TDI. Thus, Twin Ace or Tanduay Distillers did not take over the corporate personality of TDI although they manufacture
the same product at the same plant with the same equipment and machinery. Obviously, the trade name "Tanduay" went
with the sale because the new firm does business as Tanduay Distillers and its main product of rum is sold as Tanduay
Rum. There is no showing, however, that TDI itself was absorbed by Twin Ace or that it ceased to exist as a separate
corporation. In point of fact TDI is now herein a party respondent represented by its own counsel. Significantly, TDI in the
petition at hand has taken the side of its former employees and argues against Tanduay Distillers. In its memorandum filed
on January 9, 1995, TDI argues that it was not alone its liability which the arbiter recognized "but also of James Yu and
Wilson Young, representatives of Twin Ace and/or the Allied Bank Group doing business under the name 'TANDUAY
DISTILLERS,' to whom the business and assets of TDI were sold." If TDI and Tanduay Distillers are one and the same
group or one is a continuation of the other, the two would not be fighting each other in this case. TDI would not argue
strongly "that the petition for certiorari filed by James Yu and Wilson Young be dismissed for lack for merit." It is thus
obvious that the second corporation, Twin Ace or Tanduay Distillers, is an entity separate and distinct, from the first
corporation, TDI. The circumstances of this case are different from the earlier decisions of the Court in labor cases where
the veil of corporate fiction was pierced. In fine, the fiction of separate and distinct corporate entities cannot, in the instant
case, be disregarded and brushed aside, there being not the least indication that the second corporation is a dummy or
serves as a client of the first corporate entity. In the case at bench, since TDI and Twin Ace or Tanduay Distillers are two
separate and distinct entities, the order for Tanduay Distillers (and petitioners) to reinstate respondents-employees is
obviously without legal and factual basis.
DECISION

MELO, J p:

Before us is a petition for certiorari assailing the decision of public respondent National Labor Relations
Commission (NLRC) promulgated on August 25, 1993 in the cases of Fernando Duran, et al. vs. Tanduay Distillery,
Inc., docketed as NLRC NCR Case No. 00-04-01737-88, and Rodrigo Santos vs. Tanduay Distillery, Inc., docketed as
NLRC NCR Case No. 00-06-02546-88.
The relevant antecedent facts as gathered from the record are as follows:
Private respondents-employees Fernando Duran, Eduardo Paliwan, Roque Estoce, and Rodrigo Santos were
employees of respondent corporation Tanduay Distillery, Inc. (TDI).
On March 29, 1988, 22 employees of TDI, including private respondents employees, received a memorandum
from TDI terminating their services, for reason of retrenchment, effective 30 days from receipt thereof or not later than
the close of business hours on April 28, 1988.
On April 26, 1988, all 22 employees of TDI filed an application for the issuance of a temporary restraining order
against their retrenchment. The labor arbiter issued the restraining order the following day. However, due to the 20-day
lifetime of the temporary restraining order, and because of the on-going negotiations for the sale of TDI to the First
Pacific Metro Corporation, the retrenchment pushed through. Parenthetically, it should be mentioned that although all
22 employees were retrenched, the instant petition involves only the 4 individual respondents herein, namely, Fernando
Duran, Eduardo Paliwan, Roque Estoce, and Rodrigo Santos.
On June 14, 1988, the First Pacific Metro Corporation moved that it be dropped as a party to the case on the
ground that its projected purchase of the assets of TDI was not consummated. The participation of First Pacific was
later in effect held to be irrelevant (decision dated May 24, 1989; Annex G, pp. 50-58, Rollo). On June 1, 1988, or after
respondents-employees had ceased as such employees, a new buyer of TDI's assets, Twin Ace Holdings, Inc. took
over the business. Twin Ace assumed the business name Tanduay Distillers.
On August 8, 1988, the employees filed a motion to implead herein petitioners James Yu and Wilson Young,
doing business under the name and style of Tanduay Distillers, as party respondents in said cases. Petitioners filed an
opposition thereto, asserting that they are representatives of Tanduay Distillers an entity distinct and separate from TDI,
the previous owner, and that there is no employer-employee relationship between Tanduay Distillers and private
respondents. Respondents-employees filed a reply to the opposition stating that petitioner James Yu as officer-in-
charge of Tanduay Distillers had informed the president of TDI labor union of Tanduay Distillers' decision to hire
everybody with a clean slate on a probation basis.
On November 16, 1988, private respondents filed a motion for leave to file an amended complaint impleading
petitioners as respondents. Petitioners filed an opposition thereto reiterating the grounds they relied upon in their
opposition to private respondents' motion to implead. A reply was filed by private respondents, and a rejoinder was then
filed by petitioners. In turn, private respondents filed a sub-rejoinder.
Subsequently, an amended complaint was filed by private respondents against TDI and petitioners Yu and
Young "doing business under the name and style of Tanduay Distillers."
In her decision dated May 24, 1989, Labor Arbiter Daisy Cauton-Barcelona held:
In treating the motion to implied a motion to admit amended complaint with leave, the same [is]
hereby given due course and all pleadings filed by respondents James Yu and Wilson Young are hereby
treated as their responsive pleadings in the light of speedy disposition of justice and the basic rule that
administrative for a, such as this office are not governed by technical rules of proceedings.
(p. 52, Rollo).
In the same decision, it was disposed:
WHEREFORE, judgment is hereby rendered declaring that the retrenchment is illegal thereby
ordering respondent Tanduay Distillery, Inc., to reinstate the complainants to their former position with
backwages up to the time of the change of ownership, if one has taken place.
That in the event of change in management it (Tanduay Distillery, Inc.) is hereby ordered to pay
the complainants their respective separation benefits computed at the rate of one (1) month for every
year of service. This is without prejudice to the letter of Mr. James Yu as officer-in-charge of Tanduay
Distillers dated June 17, 1988 to the President of the Tanduay Distillery, Inc., Labor Union.
(pp. 57-58, Rollo.)
Only TDI appealed said decision to the National Labor Relations Commissions, but on June 18, 1991, said
commission promulgated an affirmance decision (p. 102, Rollo). TDI filed a motion for reconsideration, but the same
was denied on August 15, 1991.
Thereupon, private respondents-employees on September 16, 1991 filed a motion for execution (Annex Q, pp.
103-106, Rollo) praying that NLRC, through the labor arbiter, " [i]ssue the necessary writ for the execution of the entire
decision dated May 24, 1989, including the actual reinstatement of the complainants to their former position without loss
of seniority and benefits against Tanduay Distillery, Inc., and/or Tanduay Distillers, Inc., and/or Tanduay Distillers,
James Yu and Wilson Young."
On September 24, 1993, petitioners filed an opposition (Annex R, pp. 108-110, Rollo) to the motion for
execution on the ground that "the Motion for Execution is without any basis in so far as it prays for the issuance of a writ
of execution against respondent Tanduay Distillers, Inc., and respondents James Yu and Wilson Young." Respondents-
employees filed their reply thereto (Annex S, pp. 111-115, Rollo). and in turn petitioners filed their rejoinder (Annex T,
pp. 116-118, Rollo), to which private respondents filed their sur-rejoinder (Annex U, pp. 119-122, Rollo). On December
18, 1991, respondent TDI filed its comment on the motion for execution (Annex V, pp. 124-129, Rollo), while petitioners
on January 10, 1992, filed a joint comment (Annex W, pp. 130-132, Rollo) to private respondents' sur-rejoinder as well
to the comment filed by respondent TDI.
Subsequently, TDI filed a manifestation dated April 24, 1992 (Annex X, pp. 133-135, Rollo), stating —
2. At the hearing held on March 23, 1992, individual complainants, assisted by their counsel,
Atty. Noel Cruz, agreed to be paid the total amount of P86,049.83, in full satisfaction of the Company's
liability as stated in the dispositive portion of Labor Arbiter Barcelona's decision promulgated on May 24,
1989 and affirmed by the Second Division of the NLRC on June 18, 1991, which reads as follows:
WHEREFORE, judgment is hereby rendered declaring that the retrenchment is illegal
thereby ordering respondent Tanduay Distillery, Inc. to reinstate the complainants to their former
position with backwages up to the time of the change of ownership, if one has taken place.
That in the event of change in management it (Tanduay Distillery, Inc.) is hereby ordered
to pay the complainants their respective separation benefits computed at the rate of one (1)
month for every year of service. This is without prejudice to the letter of Mr. James Yu as officer-
in-charge of Tanduay Distillers dated June 17, 1988 to the president of the Tanduay Distillery,
Inc., Labor Union.
No costs.
SO ORDERED.
3. In accordance with the aforequoted decision, complainants shall be paid the amounts
appearing opposite their respective names:
Rodrigo F. Santos P20,282.03

Roque Estoce 20,092.50

Eduardo Daliwan 19,973.40

Fernando A. Duran 25,702.00

————

Total P86,049.83
========

4. The foregoing amounts shall be satisfied out of the cash bond deposited by the Company with
the Cashier of the NLRC. The excess amounting to P7,076.44 must revert to the Company.
(pp. 134-135, Rollo.)
On November 17, 1992, respondent NLRC, through Labor Arbiter Daniel C. Cueto, issued an order (Annex Z,
pp. 139-145, Rollo), resolving the motion for execution as follows:
Based on the foregoing considerations, this Branch finds the Motion for Writ of Execution filed by
the complainants meritorious and in order. Accordingly, let a Writ of Execution be issued against Tanduay
Distillers, Inc., Wilson Young and James Yu to immediately reinstate complainants Fernando Duran,
Rodrigo Santos, Roque Estoce and Eduardo Daliwan to their respective positions.
(p. 145, Rollo.)
Consequently, a writ of execution was issued by Labor Arbiter Cueto on December 16, 1992.
To stop the implementation of the writ of execution, petitioners filed a petition for certiorari (Annex AA, pp. 146-
158, Rollo) before respondent NLRC, praying that —
1. Immediately upon filing of the instant case, a temporary restraining order be issued, to wit:
a.) Enjoining and restraining the respondents from implementing the Order dated
November 17, 1992;
b.) Commanding the public respondent to desist from acting on the Order;
c.) Commanding the respondents to desist from committing any other act prejudicial to
the petitioners/appellants.
2. After the appropriate proceedings, a writ of preliminary injunction be issued so enjoining the
respondents;
3. After hearing on the merits, the Order dated November 17, 1992 be set aside and an
injunction be issued permanently enjoining the respondents from committing the aforesaid acts and to
comply strictly with the terms of the Decision and the NLRC;
4. Ordering the respondents, jointly and severally, to pay petitioner's fees in the amount of
P100,000.00 and to pay the costs of suit.
On August 25, 1993, respondent NLRC promulgated its decision, the dispositive portion of which reads:
In view of the foregoing premises, the petition/appeal with prayer for preliminary injunction is
hereby dismissed for lack of merit.
The petitioners-respondents are hereby directed to re-employ/re-hire respondents-complainants
immediately upon receipt of this decision.
(p. 36, Rollo.)
Thus the present petition where petitioners pray that —
1. Immediately upon filing of the instant case, a temporary restraining order be issued, to wit:
a.) Restraining and prohibiting the respondents from implementing the ORDER dated
November 17, 1992 and the NLRC Certiorari Decision.
b.) Commanding the respondents to desist from committing any other act prejudicial to
the petitioners.
2. After the appropriate proceedings, a writ of preliminary injunction be issued so enjoining the
respondents;
3. After appropriate proceedings, the ORDER dated November 17, 1992 and the injunction be
issued permanently enjoining the respondents from committing the aforesaid acts and to comply strictly
with the terms of the Arbiter Decision and the NLRC Decision;
4. Ordering the respondents, jointly and severally, to pay petitioners' attorney's fees in the
amount of P100,000.00 and to pay the costs of suit.
(pp. 26-27, Rollo.)
The issue posed by the present petition is whether respondent NLRC committed grave abuse of discretion in
holding petitioners Yu and Young liable under the decision dated May 24, 1989 which decreed that:
WHEREFORE, judgment is hereby rendered declaring that the retrenchment is illegal thereby
ordering respondent Tanduay Distillery, Inc., to reinstate the complainants to their former position with
backwages up to the time of the change of ownership, if one has taken place.
That in the event of change in management it (Tanduay Distillery, Inc.) is hereby ordered to pay
the complaints their respective separation benefits computed at the rate of one (1) month for every year
of service. This is without prejudice to the letter of Mr. James Yu as officer-in-charge of Tanduay Distillers
dated June 17, 1988 to the President of the Tanduay Distillery, Inc., Labor Union.
(pp. 57-58, Rollo.)
We hold that petitioners, for a number of reasons which we shall discuss below, may not be held answerable
and liable under the final judgment of Labor Arbiter Cauton-Barcelona.
1. Admittedly, the decision dated May 24, 1989 is now final and executory, as only respondent TDI appealed
said decision and its appeal was later dismissed by respondent NLRC. It is fundamental that a final and executory
decision cannot be amended or corrected (First Integrated Bonding and Insurance Company, Inc. vs. Hernando, 199
SCRA 796 [1991]) except for clerical errors or mistakes (Maramba vs. Lozano, 20 SCRA 474 [1967]); Reyes vs. Court
of Appeals, 189 SCRA 46 [1990]). A definitive judgment is no longer subject to change, revision, amendment, or
reversal (Miranda vs. Court of Appeals, 71 SCRA 295 [1976]), and the court loses jurisdiction over it, except to order its
execution (PY Eng Chong vs. Herrera, 70 SCRA 130 [1976]).
An examination of the aforequoted dispositive portion of the decision shows that the same does not in any
manner obligate Tanduay Distillers, or even petitioners Yu and Young for that matter, to reinstate respondents. Only
TDI was held liable to reinstate respondents up to the time of change of ownership, and for separation benefits.
However, Labor Arbiter Cueto went beyond what was disposed by the decision and issued an order dated
November 17, 1992 (Annex Z, petition, pp. 139-145, Rollo) which required
. . . Tanduay Distillers, Inc., Wilson Young and James Yu to immediately reinstate complainants
Fernando Duran, Rodrigo Santos, Roque Estoce and Eduardo Daliwan to their respective positions.
(p. 145, Rollo.)
Subsequently, a writ of execution was issued on December 16, 1992 pursuant to the order of November 17,
1992.
The order of execution dated November 17, 1992 in effect amended the decision dated May 24, 1989 for the
former orders petitioners and Tanduay Distillers to reinstate private respondents employees whereas the decision dated
May 24, 1989, as we have discussed above, does not so decree. This cannot be done. It is beyond the power and
competence of Labor Arbiter Cueto to amend a final decision. The writ of execution must not go beyond the scope of
the judgment.
As chief Justice Moran opined: "The writ of execution must conform to the judgment which is to
be executed, as it may not vary the terms of the judgment it seeks to enforce. Nor may it go beyond the
terms of the judgment sought to be executed. Where the execution is not in harmony with the judgment
which gives it life and exceeds it, it has pro tanto no validity. To maintain otherwise would be to ignore the
constitutional provision against depriving a person of his property without due process of law" (Moran,
Comments on the Rules of Court, Vol. I, 1952 Ed., p. 809; cited in Villoria vs. Piccio, supra).
(Gamboa's Incorporated vs. Court of Appeals, 72 SCRA 131, 137-138 [1976].)
The order of execution and the writ of execution ordering petitioners and Tanduay Distillers to reinstate private
respondents employees are, therefore, null and void.
2. Neither may be said that petitioners and Tanduay Distillers are one and the same as TDI, as seems to be the
impression of respondents when they impleaded petitioners as party respondents in their complaint for unfair labor
practice, illegal lay off, and separation benefits.
Such a stance is not supported by the facts. The name of the company for whom the petitioners are working is
Twin Ace Holdings Corporation. As stated by the Solicitor General, Twin Ace is part of the Allied Bank Group although it
conducts the rum business under the name of Tanduay Distillers. The use of a similar sounding or almost identical
name is an obvious device to capitalize on the goodwill which Tanduay Rum has built over the years. Twin Ace or
Tanduay Distillers, on one hand, and Tanduay Distillery, Inc. (TDI), on the other, are distinct and separate corporations.
There is nothing to suggest that the owners of TDI, have any common relationship as to identify it with Allied Bank
Group which runs Tanduay Distillers. The dissertation of the court in Diatagon Labor Federation Local 110 of the
ULGWP vs. Ople, et al. (101 SCRA 534 [1980]) is worthy of restatement, thusly:
We hold that the director of Labor Relations acted with grave abuse of discretion in treating the
two companies as a single bargaining unit. That ruling is arbitrary and untenable because the two
companies are indubitably distinct with separate juridical personalities.
The fact that their businesses are related and that the 236 employees of Georgia Pacific
International Corporation were originally employees of Lianga Bay Logging Co., Inc. is not a justification
for disregarding their separate personalities. Hence, the 236 employees, who are now attached to
Georgia Pacific International Corporation, should not be allowed to vote in the certification election at the
Lianga Bay Logging Co., Inc. They should vote at a separate certification election to determine the
collective bargaining representative of the employees of Georgia Pacific International Corporation.
(at pp. 540-541.)
It is basic that a corporation is invested by law with a personality separate and distinct from those of the
persons composing it as well as from that of any other legal entity to which it may be related (Palay, Inc., et al. vs.
Clave, et al., 124 SCRA 641 [1983])
The genuine nature of the sale to Twin Ace is evidenced by the fact that Twin Ace was only a subsequent
interested buyer. At the time when termination notices were sent to its employees, TDI was negotiating with the First
Pacific Metro Corporations for the sale of its assets. Only after First Pacific gave up its efforts to acquire the assets did
Twin Ace or Tanduay Distillers come into the picture. Respondents-employees have not presented any proof as to
communality of ownership and management to support their contention that the two companies are one firm or closely
related. The doctrine of piercing the veil of corporate entity applies when the corporate fiction is used to defeat public
convenience, justify wrong, protect fraud, or defend crime or where a corporation is the mere alter ego or business
conduit of a person (Indophil Textile Mill Workers Union vs. Calica, 205 SCRA 697, 703 [1992]). To disregard the
separate juridical personality of a corporation, the wrong-doing must be clearly and convincingly established. It cannot
be presumed (Del Rosario vs. NLRC, 187 SCRA 777, 780 [1990]).
The complaint for unfair labor practice, illegal lay off, and separation benefits was filed against TDI. Only later
when the manufacture and sale of Tanduay products was taken over by Twin Ace or Tanduay Distillers were James Yu
and Wilson Young impleaded.
The corporation itself — Twin Ace or Tanduay Distillers — was never made a party to the case.
Another factor to consider is that TDI as a corporation or its shares of stock were not purchased by Twin Ace.
The buyer limited itself to purchasing most of the assets, equipment, and machinery of TDI. Thus, Twin Ace or Tanduay
Distillers did not take over the corporate personality of TDI although they manufacture the same product at the same
plant with the same equipment and machinery. Obviously, the trade name "Tanduay" went with the sale because the
new firm does business as Tanduay Distillers and its main product of rum is sold as Tanduay Rum. There is no
showing, however, that TDI itself was absorbed by Twin Ace or that it ceased to exist as a separate corporation. In point
of fact TDI is now herein a party respondent represented by its own counsel.
Significantly, TDI in the petition at hand has taken the side of its former employees and argues against Tanduay
Distillers. In its memorandum filed on January 9, 1995, TDI argues that it was not alone its liability which the arbiter
recognized "but also of James Yu and Wilson Young, representatives of Twin Ace and/or the Allied Bank Group doing
business under the name 'TANDUAY DISTILLERS,' to whom the business and assets of DTI were sold." If TDI and
Tanduay Distillers are one and the same group or one is a continuation of the other, the two would not be fighting each
other in this case. TDI would not argue strongly "that the petition for certiorari filed by James Yu and Wilson Young be
dismissed for lack for merit." It is thus obvious that the second corporation, Twin Ace or Tanduay Distillers, is an entity
separate and distinct, from the first corporation, TDI. The circumstances of this case are different from the earlier
decisions of the Court in labor cases where the veil of corporate fiction was pierced.
In La Campana Coffee Factory, Inc. vs. Kaisahan ng Manggagawa sa La Campana (KKM), (93 Phil. 160
[1953], La Campana Coffee Factory, Inc. and La Campana Gaugau Packing were substantially owned by the same
person. They had one office, one management, and a single payroll for both business. The laborers of the gaugau
factory and the coffee factory were also interchangeable, the workers in one factory worked also in the other factory.
In Claparols vs. Court of Industrial Relations (65 SCRA 613 [1975], the Claparols Steel and Nail Plant, which
was ordered to pay its workers backwages, ceased operations on June 30, 1956 and was succeeded on the very next
day, July 1, 1957, by the Clarapols Steel Corporation. Both corporations were substantially owned and controlled by the
same person and there was no break or cessation in operations. Moreover, all the assets of the steel and nail plant
were transferred to the new corporation.
In fine, the friction of separate and distinct corporate entities cannot, in the instant case, be disregarded and
brushed aside, there being not the least indication that the second corporation is a dummy or serves as a client of the
first corporate entity.
In the case at bench, since TDI and Twin Ace or Tanduay Distillers are two separate and distinct entities, the
order for Tanduay Distillers (and petitioners) to reinstate respondents-employees is obviously without legal and factual
basis.
3. Nor could the order and writ to reinstate be anchored on the vague and seemingly uncalled for alternative
disposition in the Barcelona decision that —
. . . This is without prejudice to the letter of Mr. James Yu as officer-in-charge of Tanduay
Distillers dated June 16, 1988 to the President of the Tanduay Distillery, Inc. Labor Union.
The June 11, 1988 letter referred to was addressed to Benjamin C. Agaloos, president of the Tanduay Distillery
Labor Union by James Yu in his capacity as officer-in-charge of Tanduay Distillers.
It pertinently reads:
Please be informed that our company stands firm on its decision to hire everybody with a clean
slate effective June 1, 1988 on a probationary basis while those currently casual or contractual
employees shall retain the same employment status. In the same manner that the new company stood
firm on its decision to grant a 10% across-the-board increase to all employees, which in fact has been
received by employees concerned.
(p. 88, Rollo.)
We do not find in the decision of Labor Arbiter Cauton Barcelona or in the letter of James Yu what respondents
are trying to read into it. Labor Arbiter Cauton-Barcelona found the retrenchment effected by TDI illegal and ordered TDI
to reinstate the complainants and that if there is a change of management, then separation benefits would be paid.
There is, however, no order in the decision directing Twin Ace or Tanduay Distillers to hire or reinstate herein four
individual respondents.
The letter of James Yu does not mention any reinstatement. It assures the president of the labor union that
Tanduay Distillers stood firm on its decision to hire employees with a clean slate on a probationary basis. The fact that
the employees of the former employer (TDI) would be hired on a probationary basis shows that there was no employer-
employee relationship between individual respondents and Twin Ace. Any one who joins the buyer corporation comes in
as an outsider who is newly hired and who starts on a probationary basis until he proves he deserves to be on a
permanent status. His application can be rejected in the exercise of the hiring authority's discretion.
There is thus no legal basis for Labor Arbiter Cueto or the NLRC to compel Twin Ace or Tanduay Distillers, or
petitioners to "reinstate" the four individuals respondents. The letter of James Yu to the union president was a unilateral
and gratuitous offer with no consideration. It refers to people who still have to be hired. New hires had to be investigated
or evaluated if they have "clean slates." Twin Ace or Tanduay Distillers and petitioners are being compelled by public
respondents to reinstate workers who were never their employees. There is no showing that the sale of assets by TDI to
Tanduay Distillers included a condition that employees of the former would be absorbed by the latter.
Employees of TDI had been terminated in their employment as of April 28, 1988. Petitioners state that Twin Ace
bought the assets of TDI after the employment of the individual respondents had been terminated. True, Labor Arbiter
Cauton-Barcelona declared the retrenchment program of TDI as illegal. This decision, however, did not convert the
individual respondents into employees of the firm which purchased the assets of the former employer. It merely held
TDI liable for the consequences of the illegal retrenchment.
Labor Arbiter Cueto and the NLRC, therefore, committed grave abuse of discretion when they read into the
decision of Labor Arbiter Cauton-Barcelona something which did not appear there. And even assuming that Labor
Arbiter Cauton-Barcelona formally included an order for the petitioners to hire the individual respondents, there would
be no factual or legal basis for such an order. An employer-employee relation is created by contract, and cannot be
forced upon either party simply upon order of a labor arbiter. The hiring of employees is one of the recognized
prerogatives of management.
4. Another factor which militates against the claim for reinstatement of the individual respondents is their having
received separation pay as part of a compromise agreement in the course of their litigation with TDI. Rodrigo F. Santos
received P20,282.03; Roque Estoce, P20,092.50; Eduardo Daliwan, P19,973.40; and Fernando A. Duran, P25,702.00.
These amounts correspond to their entitlement to separation benefits. Having received separation pay from a former
employer, how can they compel, as a matter of right, another company to hire them on a supposed "reinstatement"
basis? The orders executing the earlier decision of Labor Arbiter Cauton-Barcelona and directing petitioners to
immediately reinstate the four individual respondents to their former positions are, thus, characterized by grave abuse of
discretion. There are no "former positions" to which individual respondents may be reinstated because they were never
hired by Twin Ace/Tanduay Distillers and had never worked for it.
WHEREFORE, the petition is hereby GRANTED. The questioned Order of the Labor Arbiter Daniel C. Cueto
dated November 17, 1992 and the decision of the National Labor Relations Commission upholding said order are set
aside as null and void. No special pronouncement is made as to costs. SO ORDERED.
||| (Yu v. National Labor Relations Commission, G.R. Nos. 111810-11, [June 16, 1995], 315 PHIL 107-128)

[G.R. No. L-33172. October 18, 1979.]


ERNESTO CEASE, CECILIA CEASE, MARION CEASE, TERESA CEASE-LACEBAL, and the F.L.
CEASE PLANTATION CO., INC. as Trustee of properties of the defunct TIAONG MILLING &
PLANTATION CO., petitioners, vs. HONORABLE COURT OF APPEALS, (Special Seventh Division),
HON. MANOLO L. MADDELA, Presiding Judge, Court of First Instance of Quezon, BENJAMIN
CEASE and FLORENCE CEASE, respondents.

DECISION

GUERRERO, J p:

Appeal by certiorari from the decision of the Court of Appeals in CA-G.R. No. 45474, entitled "Ernesto
Cease, et al. vs. Hon. Manolo L. Maddela, Judge of the Court of First Instance of Quezon, et al." 1 which
dismissed the petition for certiorari, mandamus, and prohibition instituted by the petitioners against the
respondent judge and the private respondents. cdll
The antecedents of the case, as found by the appellate court, are as follows:
"IT RESULTING: That the antecedents are not difficult to understand; sometime in June
1908, one Forrest L. Cease common predecessor in interest of the parties together with five (5)
other American citizens organized the Tiaong Milling and Plantation Company and in the course of
its corporate existence the company acquired various properties but at the same time all the other
original incorporates were bought out by Forrest L. Cease together with his children namely
Ernest, Cecilia, Teresita, Benjamin, Florence and one Bonifacia Tirante also considered a member
of the family; the charter of the company lapsed in June 1958; but whether there were steps to
liquidate it, the record is silent; on 13 August 1959, Forrest L. Cease died and by extrajudicial
partition of his shares, among the children, this was disposed of on 19 October 1959; it was here
where the trouble among them came to arise because it would appear that Benjamin and Florence
wanted an actual division while the other children wanted reincorporation; and proceeding on that,
these other children Ernesto, Teresita and Cecilia and aforementioned other stockholder Bonifacia
Tirante proceeded to incorporate themselves into the FL Cease Plantation Company and
registered it with the Securities and Exchange Commission on 9 December, 1959; apparently in
view of that, Benjamin and Florence for their part initiated a Special Proceeding No. 3893 of the
Court of First Instance of Tayabas for the settlement of the estate of Forest L. Cease on 21 April,
1960 and one month afterwards on 19 May, 1960 they filed Civil Case No. 6326 against Ernesto,
Teresita and Cecilia Cease together with Bonifacia Tirante asking that the Tiaong Milling and
Plantation Corporation be declared identical to FL Cease and that its properties be divided among
his children as his intestate heirs; this Civil Case was resisted by aforestated defendants and
notwithstanding efforts of the plaintiffs to have the properties placed under receivership, they were
not able to succeed because defendants filed a bond to remain as they have remained in
possession; after that and already during the pendency of Civil Case No. 6326 specifically on 21
May, 1961 apparently on the eve of the expiry of the three (3) year period provided by the law for
the liquidation of corporations, the board of liquidators of Tiaong Milling executed an assignment
and conveyance of properties and trust agreement in favor of FL Cease Plantation Co. Inc. as
trustee of the Tiaong Milling and Plantation Co. so that upon motion of the plaintiffs trial Judge
ordered that this alleged trustee be also included as party defendant; now this being the situation,
it will be remembered that there were thus two (2) proceedings pending in the Court of First
Instance of Quezon namely Civil Case No. 6326 and Special Proceeding No. 3893 but both of
these were assigned to the Honorable Respondent Judge Manolo L. Maddela, p. 43 and the case
was finally heard and submitted upon stipulation of facts pp. 34-110, rollo; and trial Judge by
decision dated 27 December 1969 held for the plaintiffs Benjamin and Florence, the decision
containing the following dispositive part:
"VIEWED IN THE LIGHT OF ALL THE FOREGOING, judgment is hereby rendered in
favor of plaintiffs and against the defendants declaring that:
1) The assets or properties of the defunct Tiaong Milling and Plantation Company now
appearing under the name of F.L. Cease Plantation Company as Trustee, is the estate also of the
deceased Forrest L. Cease and ordered divided share and share alike, among his six children the
plaintiffs and the defendants in accordance with Rule 69, Rules of Court;
2) The Resolution to Sell dated October 12, 1959 and the Transfer and Conveyance with
Trust Agreement is hereby set aside as improper and illegal for the purposes and effect that it was
intended and, therefore, null and void;
3) That F.L. Cease Plantation Company is removed as Trustee for interest against the
estate and essential to the protection of plaintiffs' rights and is hereby ordered to deliver and
convey all the properties and assets of the defunct Tiaong Milling now under its name, custody and
control to whomsoever be appointed as Receiver — disqualifying any of the parties herein — the
latter to act accordingly upon proper assumption of office; and
4) Special Proceedings No. 3893 for administration is terminated and dismissed; the
instant case to proceed but on issues of damages only and for such action inherently essential for
partition.
SO ORDERED.
Lucena City, December 27, 1969, pp. 122-a-123, rollo;"
upon receipt of that, defendants there filed a notice of appeal p. 129, rollo together with an appeal
bond and a record on appeal but the plaintiffs moved to dismiss the appeal on the ground that the
judgment was in fact interlocutory and not appealable p. 168 rollo and this position of defendants
was sustained by trial Judge, His Honor ruling that.
"IN VIEW OF THE FOREGOING, the appeal interposed by plaintiffs is hereby dismissed
as premature and the Record on Appeal is necessarily disapproved as improper at this stage of
the proceedings.
SO ORDERED.
Lucena City, April 27, 1970."
and so it was said defendants brought the matter first to the Supreme Court, on mandamus on 20
May, 1970 to compel the appeal and certiorari and prohibition to annul the order of 27 April, 1970
on the ground that the decision was "patently erroneous" p. 16, rollo; but the Supreme Court
remanded the case to this Court of Appeals by resolution of 27 May 1970, p. 173, and this Court of
Appeals on 1 July, 1970 p. 175 dismissed the petition so far as the mandamus was concerned
taking the view that the decision sought to be appealed dated 27 December, 1969 was
interlocutory and not appealable but on motion for reconsideration of petitioners and since there
was possible merit so far as its prayer for certiorari and prohibition was concerned, by resolution of
the Court on 19 August, 1970, p. 232, the petition was permitted to go ahead in that capacity; and
it is the position of petitioners that the decision of 27 December, 1969 as well as the order of 27
April, 1970 suffered of certain fatal defects, which respondents deny and on their part raise the
preliminary point that this Court of Appeals has no authority to give relief to petitioners because
not.
"in aid of its appellate jurisdiction,"
and that the questions presented cannot be raised for the first time before this Court of Appeals;"
Respondent Court of Appeals in its decision promulgated December 9, 1970 dismissed the petition with
costs against petitioners, hence the present petition to this Court on the following assignment of errors:
THE COURT OF APPEALS ERRED —
I. IN SANCTIONING THE WRONGFUL EXERCISE OF JURISDICTION BEYOND THE
LIMITS OF AUTHORITY CONFERRED BY LAW UPON THE LOWER COURT, WHEN IT
PROCEEDED TO HEAR, ADJUDGE AND ADJUDICATE —
(a) Special Proceedings No. 3893 for the settlement of the Estate of Forrest L.
Cease, simultaneously and concurrently with —
(b) Civil Case No. 6326, wherein the lower Court ordered Partition under Rule 69,
Rules of Court —
THE ISSUE OF LEGAL OWNERSHIP OF THE PROPERTIES COMMONLY INVOLVED IN BOTH
ACTIONS HAVING BEEN RAISED AT THE OUTSET BY THE TIAONG MILLING AND
PLANTATION COMPANY, AS THE REGISTERED OWNER OF SUCH PROPERTIES UNDER
ACT 496.
II. IN AFFIRMING — UNSUPPORTED BY ANY EVIDENCE WHATSOEVER NOR
CITATION OF ANY LAW TO JUSTIFY — THE UNWARRANTED CONCLUSION THAT SUBJECT
PROPERTIES, FOUND BY THE LOWER COURT AND THE COURT OF APPEALS AS
ACTUALLY REGISTERED IN THE NAME OF PETITIONER CORPORATION AND/OR ITS
PREDECESSOR IN INTEREST, THE TIAONG MILLING AND PLANTATION COMPANY,
DURING ALL THE 50 YEARS OF ITS CORPORATE EXISTENCE, "ARE ALSO PROPERTIES OF
THE ESTATE OF FOREST L. CEASE."
III. IN AFFIRMING THE ARBITRARY CONCLUSION OF THE LOWER COURT THAT ITS
DECISION OF DECEMBER 27, 1969 IS AN "INTERLOCUTORY DECISION." IN DISMISSING
THE PETITION FOR WRIT OF MANDAMUS, AND IN AFFIRMING THE MANIFESTLY UNJUST
JUDGMENT RENDERED WHICH CONTRADICTS THE FINDINGS OF ULTIMATE FACTS
THEREIN CONTAINED.
During the period that ensued after the filing in this Court of the respective briefs and the subsequent
submission of the case for decision, some incidents had transpired, the summary of which may be stated as
follows:
1. Separate from this present appeal, petitioners filed a petition for certiorari and prohibition in this Court,
docketed as G.R. No. L-35629 (Ernesto Cease, et al. vs. Hon. Manolo L. Maddela, et al.) which challenged the
order of respondent judge dated September 27, 1972 appointing his Branch Clerk of Court, Mr. Eleno M. Joyas,
as receiver of the properties subject of the appealed civil case, which order, petitioners saw as a virtual execution
of the lower court's judgment (p. 92, rollo). In Our resolution of November 13, 1972, issued in G.R. No. L-35629,
the petition was denied since respondent judge merely appointed an auxilliary receiver for the preservation of the
properties as well as for the protection of the interests of all parties in Civil Case No. 6326; but at the same time,
We expressed Our displeasure in the appointment of the branch clerk of court or any other court personnel for
that matter as receiver. (p. 102, rollo) LLjur
2. Meanwhile, sensing that the appointed receiver was making some attempts to take possession of the
properties, petitioners filed in this present appeal an urgent petition to restrain proceedings in the lower court. We
resolved the petition on January 29, 1975 by issuing a corresponding temporary restraining order enjoining the
court a quo from implementing its decision of December 27, 1969, more particularly, the taking over by a receiver
of the properties subject of the litigation, and private respondents Benjamin and Florence Cease from proceeding
or taking any action on the matter until further orders from this Court (pp. 99-100, rollo). Private respondents filed
a motion for reconsideration of Our resolution of January 29, 1975. After weighing the arguments of the parties
and taking note of Our resolution in G.R. No. L-35629 which upheld the appointment of a receiver, We issued
another resolution dated April 11, 1975 lifting effective immediately Our previous temporary restraining order
which enforced the earlier resolution of January 29, 1975 (pp. 140-141, rollo)
3. On February 6, 1976, private respondents filed an urgent petition to restrain proceedings below in view
of the precipitate replacement of the court appointed receiver Mayor Francisco Escueta (vice Mr. Eleno M. Joyas)
and the appointment of Mr. Guillermo Lagrosa on the eve of respondent Judge Maddela's retirement (p. 166,
rollo). The urgent petition was denied in Our resolution of February 18, 1976 (p. 176, rollo)
4. Several attempts at a compromise agreement failed to materialize. A Tentative Compromise
Agreement dated July 30, 1975 was presented to the Court on August 6, 1976 for the signature of the parties, but
respondents "unceremoniously" repudiated the same by leaving the courtroom without the permission of the court
(Court of First Instance of Quezon, Branch II) as a result of which respondents and their counsel were cited for
contempt (p. 195, 197, rollo); that respondents' reason for the repudiation appears to be petitioners' failure to
render an audited account of their administration covering the period from May 31, 1961 up to January 29, 1974,
plus the inclusion of a provision on waiver and relinquishment by respondents of whatever rights that may have
accrued to their favor by virtue of the lower court's decision and the affirmative decision of the appellate court.
We go now to the alleged errors committed by the respondent Court of Appeals.
As can be gleaned from petitioners' brief and the petition itself, two contentions underlie the first assigned
error. First, petitioners argue that there was an irregular and arbitrary termination and dismissal of the special
proceedings for judicial administration simultaneously ordered in the lower court's decision in Civil Case No. 6326
adjudicating the partition of the estate, without categorically resolving the opposition to the petition for
administration. Second, that the issue of ownership had been raised in the lower court when Tiaong Milling
asserted title over the properties registered in its corporate name adverse to Forrest L. Cease or his estate, and
that the said issue was erroneously disposed of by the trial court in the partition proceedings when it concluded
that the assets or properties of the defunct company is also the estate of the deceased proprietor.
The propriety of the dismissal and termination of the special proceedings for judicial administration must
be affirmed in spite of its rendition in another related case in view of the established jurisprudence which favors
partition when judicial administration becomes unnecessary. As observed by the Court of Appeals, the dismissal
at first glance is wrong, for the reason that what was actually heard was Civil Case No. 6326. The technical
consistency, however, if far less an importance than the reason behind the doctrinal rule against placing an
estate under administration. Judicial rulings consistently hold the view that where partition is possible, either
judicial or extrajudicial, the estate should not be burdened with an administration proceeding without good and
compelling reason. When the estate has no creditors or pending obligations to be paid, the beneficiaries in
interest are not bound to submit the property to judicial administration which is always long and costly, or to apply
for the appointment of an administrator by the court, especially when judicial administration is unnecessary and
superfluous. Thus —
"When a person dies without leaving pending obligations to be paid, his heirs, whether of
age or not, are bound to submit the property to a judicial administration, which is always long and
costly, or to apply for the appointment of an administrator by the court. It has been uniformly held
that in such case the judicial administration and the appointment of an administrator are
superfluous and unnecessary proceedings (Ilustre vs. Alaras Frondosa, 17 Phil., 321; Malahacan
vs. Ignacio, 19 Phil, 434; Bondad vs. Bondad, 34 Phil., 232; Baldemor vs. Malangyaon, 34 Phil.,
367; Fule vs. Fule, 46 Phil., 317)." Syllabus, Intestate estate of the deceased Luz Garcia. Pablo G.
Utulo vs. Leona Pasion Viuda de Garcia, 66 Phil. 302.
"Where the estate has no debts, recourse may be had to an administration proceeding
only if the heirs have good reasons for not resorting to an action for partition. Where partition is
possible, either in or out of court, the estate should not be burdened with an administration
proceeding without good and compelling reasons." (Intestate Estate of Mercado vs. Magtibay, 96
Phil. 383)
In the records of this case, We find no indication of any indebtedness of the estate. No creditor has come
up to charge the estate within the two-year period after the death of Forrest L. Cease, hence, the presumption
under Section 1, Rule 74 that the estate is free from creditors must apply. Neither has the status of the parties as
legal heirs, much less that of respondents, been raised as an issue. Besides, extant in the records is the
stipulation of the parties to submit the pleadings and contents of the administration proceedings for the
cognizance of the trial judge in adjudicating the civil case for partition (Respondents' Brief, p. 20, rollo). As
respondents observe, the parties in both cases are the same, so are the properties involved; that actual division
is the primary objective in both actions; the theory and defense of the respective parties are likewise common;
and that both cases have been assigned to the same respondent judge. We feel that the unifying effect of the
foregoing circumstances invites the wholesome exception to the structures of procedural rule, thus allowing,
instead, room for judicial flexibility. Respondent judge's dismissal of the administration proceedings then, is a
judicious move, appreciable in today's need for effective and speedy administration of justice. There being ample
reason to support the dismissal of the special proceedings in this appealed case, We cannot see in the records
any compelling reason why it may not be dismissed just the same even if considered in a separate action. This is
inevitably certain specially when the subject property has already been found appropriate for partition, thus
reducing the petition for administration to a mere unnecessary solicitation.
The second point raised by petitioners in their first assigned error is equally untenable. In effect,
petitioners argue that the action for partition should not have prospered in view of the repudiation of the co-
ownership by Tiaong Milling and Plantation Company when, as early in the trial court, it already asserted
ownership and corporate title over the properties adverse to the right of ownership of Forrest L. Cease or his
estate. We are not unmindful of the doctrine relied upon by petitioners in Rodriguez vs. Ravilan, 17 Phil. 63
wherein this Court held that in an action for partition, it is assumed that the parties by whom it is prosecuted are
all co-owners or co-proprietors of the property to be divided, and that the question of common ownership is not to
be argued, not the fact as to whether the intended parties are or are not the owners of the property in question,
but only as to how and in what manner and proportion the said property of common ownership shall be
distributed among the interested parties by order of the Court. Consistent with this dictum, it has been held that if
any party to a suit for partition denies the pro-indiviso character of the estate whose partition is sought, and
claims instead, exclusive title thereto, the action becomes one for recovery of property cognizable in the courts of
ordinary jurisdiction. 2
Petitioners' argument has only theoretical persuasion, to say the least, rather apparent than real. It must
be remembered that when Tiaong Milling adduced its defense and raised the issue of ownership, its corporate
existence already terminated through the expiration of its charter. It is clear in Section 77 of Act No. 1459
(Corporation Law) that upon the expiration of the charter period, the corporation ceases to exist and is dissolved
ipso facto except for purposes connected with the winding up and liquidation. The provision allows a three-year
period from expiration of the charter within which the entity gradually settles and closes its affairs, disposes and
convey its property and to divide its capital stock, but not for the purpose of continuing the business for which it
was established. At this terminal stage of its existence, Tiaong Milling may no longer persist to maintain adverse
title and ownership of the corporate assets as against the prospective distributees when at this time it merely
holds the property in trust, its assertion of ownership is not only a legal contradiction, but more so, to allow it to
maintain adverse interest would certainly thwart the very purpose of liquidation and the final distribution of the
assets to the proper parties. llcd
We agree with the Court of Appeals in its reasoning that substance is more important than form when it
sustained the dismissal of Special Proceedings No. 3893, thus —
"a) As to the dismissal of Special Proceedings No. 3893, of course, at first glance, this was
wrong, for the reason that the case that had been heard was Civil Case No. 6326; but what should
not be overlooked either is that respondent Judge was the same judge that had before him in his
own sala, said Special Proceedings No. 3893, p. 43 rollo, and the parties to the present Civil Case
No. 6326 had themselves asked respondent Judge to take judicial notice of the same and its
contents page 34, rollo; it is not difficult to see that when respondent Judge in par. 4 of the
dispositive part of his decision complained of, ordered that,
'4) Special Proceedings No. 3893 for administration is terminated and dismissed; the
instant case to proceed but on issues of damages only and for such action inherently
essential or partition. p. 123, rollo,
in truth and in fact, His Honor was issuing that order also within Civil Case No. 6326 but in
connection with Special Proceedings No. 3893; for substance is more important than form, the
contending parties in both proceedings being exactly the same, but not only this, let it not be
forgotten that when His Honor dismissed Special Proceedings No. 3893, that dismissal precisely
was a dismissal that petitioners herein had themselves sought and solicited from respondent
Judge as petitioners themselves aver in their present petition pp. 5-6, rollo: this Court must find
difficulty in reconciling petitioners' attack with the fact that it was they themselves that had insisted
on that dismissal; on the principle that not he who is favored but he who is hurt by a judicial order
is he only who should be heard to complain and especially since extraordinary legal remedies are
remedies in extremis granted to parties who have been the victims not merely of errors but of
grave wrongs, and it cannot be seen how one who got what he had asked could be heard to claim
that he had been the victim of a wrong, petitioners should not now complain of an order they had
themselves asked in order to attack such an order afterwards; if at all, perhaps, third parties,
creditors, the Bureau of Internal Revenue, might have been prejudiced, and could have had the
personality to attack that dismissal of Special Proceedings No. 3893, but not petitioners herein,
and it is not now for this Court of Appeals to protect said third persons who have not come to the
Court below or sought to intervene herein;"
On the second assigned error, petitioners argue that no evidence has been found to support the
conclusion that the registered properties of Tiaong Milling are also properties of the estate of Forrest L. Cease;
that on the contrary, said properties are registered under Act No. 496 in the name of Tiaong Milling as lawful
owner and possessor for the last 50 years of its corporate existence.
We do not agree. In reposing ownership to the estate of Forrest L. Cease, the trial court indeed found
strong support, one that is based on a well-entrenched principle of law. In sustaining respondents' theory of
"merger of Forrest L. Cease and the Tiaong Milling as one personality", or that "the company is only the business
conduit and alter ego of the deceased Forrest L. Cease and the registered properties of Tiaong Milling are
actually properties of Forrest L. Cease and should be divided equally, share and share alike among his six
children, . . .", the trial court did aptly apply the familiar exception to the general rule by disregarding the legal
fiction of distinct and separate corporate personality and regarding the corporation and the individual member
one and the same. In shredding the fictitious corporate veil, the trial judge narrated the undisputed factual
premise, thus:
"While the records showed that originally its incorporates were aliens, friends or third-
parties in relation of one to another, in the course of its existence, it developed into a close family
corporation. The Board of Directors and stockholders belong to one family the head of which
Forrest L. Cease always retained the majority stocks and hence the control and management of its
affairs. In fact, during the reconstruction of its records in 1947 before the Securities and Exchange
Commission only 9 nominal shares out of 300 appears in the name of his 3 eldest children then
and another person close to them. It is likewise noteworthy to observe that as his children increase
or perhaps become of age, he continued distributing his shares among them adding Florence,
Teresa and Marion until at the time of his death only 190 were left to his name. Definitely, only the
members of his family benefited from the Corporation.
"The accounts of the corporation and therefore its operation, as well as that of the family
appears to be instinguisable and apparently joined together. As admitted by the defendants
(Manifestation of Compliance with order of March 7, 1963 [Exhibit "21"] the corporation 'never' had
any account with any banking institution or if any account was carried in a bank on its behalf, it was
in the name of Mr. Forrest L. Cease. In brief, the operation of the Corporation is merged with those
of the majority stockholders, the latter using the former as his instrumentality and for the exclusive
benefits of all his family. From the foregoing indication, therefore, there is truth in plaintiff's
allegation that the corporation is only a business conduit of his father and an extension of his
personality, they are one and the same thing. Thus, the assets of the corporation are also the
estate of Forrest L. Cease, the father of the parties herein who are all legitimate children of full
blood."
A rich store of jurisprudence has established the rule known as the doctrine of disregarding or piercing
the veil of corporate fiction. Generally, a corporation is invested by law with a personality separate and distinct
from that of the persons composing it as well as from that of any other legal entity to which it may be related. By
virtue of this attribute, a corporation may not, generally, be made to answer for acts or liabilities of its
stockholders or those of the legal entities to which it may be connected, and vice versa. This separate and
distinct personality is, however, merely a fiction created by law for convenience and to promote the ends of
justice (Laguna Transportation Company vs. Social Security System, L-14606, April 28, 1960; La Campana
Coffee Factory, Inc. vs. Kaisahan ng mga Manggagawa sa La Campana, L-5677, May 25, 1953). For this reason,
it may not be used or invoked for ends subversive of the policy and purpose behind its creation (Emiliano Cano
Enterprises, Inc. vs. CIR, L-20502, Feb. 26, 1965) or which could not have been intended by law to which it owes
its being McConnel vs. Court of Appeals, L-10510, March 17, 1961, 1 SCRA 722). This is particularly true where
the fiction is used to defeat public convenience, justify wrong, protect fraud, defend crime (Yutivo Sons Hardware
Company vs. Court of Tax Appeals, L-13203, Jan. 28, 1961, 1 SCRA 160), confuse legitimate legal or judicial
issues (R.F. Sugay & Co. vs. Reyes, L-20451, Dec. 28, 1964), perpetrate deception or otherwise circumvent the
law (Gregorio Araneta, Inc. vs. Tuason de Paterno, L-2886, Aug. 22, 1952, 49 O.G. 721). This is likewise true
where the corporate entity is being used as an alter ego, adjunct, or business conduit for the sole benefit of the
stockholders or of another corporate entity (McConnel vs. Court of Appeals, supra; Commissioner of Internal
Revenue vs. Norton Harrison Co., L-7618, Aug. 31, 1964). cdrep
In any of these cases, the notion of corporate entity will be pierced or disregarded, and the corporation
will be treated merely as an association of persons or, where there are two corporations, they will be merged as
one, the one being merely regarded as part or the instrumentality of the other (Koppel [Phil.], Inc. vs. Yatco, 77
Phil. 496; Yutivo Sons Hardware Company vs. Court of Tax Appeals, supra).
So must the case at bar add to this jurisprudence. An indubitable deduction from the findings of the trial
court cannot but lead to the conclusion that the business of the corporation is largely, if not wholly, the personal
venture of Forrest L. Cease. There is not even a shadow of a showing that his children were subscribers or
purchasers of the stocks they own. Their participation as nominal shareholders emanated solely from Forrest L.
Cease's gratuitous dole out of his own shares to the benefit of his children and ultimately his family.
Were we sustain the theory of petitioners that the trial court acted in excess of jurisdiction or abuse of
discretion amounting to lack of jurisdiction in deciding Civil Case No. 6326 as a case for partition when the
defendant therein, Tiaong Milling and Plantation Company, Inc. as registered owner asserted ownership of the
assets and properties involved in the litigation, which theory must necessarily be based on the assumption that
said assets and properties of Tiaong Milling and Plantation Company, Inc. now appearing under the name of F. L.
Cease Plantation Company as Trustee are distinct and separate from the estate of Forrest L. Cease to which
petitioners and respondents as legal heirs of said Forrest L. Cease are equally entitled share and share alike,
then that legal fiction of separate corporate personality shall have been used to delay and ultimately deprive and
defraud the respondents of their successional rights to the estate of their deceased father. For Tiaong Milling and
Plantation Company shall have been able to extend its corporate existence beyond the period of its charter which
lapsed in June, 1958 under the guise and cover of F. L. Cease Plantation Company, Inc. as Trustee which would
be against the law, and said Trustee shall have been able to use the assets and properties for the benefit of the
petitioners, to the great prejudice and defraudation of private respondents. Hence, it becomes necessary and
imperative to pierce that corporate veil.
Under the third assigned error, petitioners claim that the decision of the lower court in the partition case is
not interlocutory but rather final for it consists of final and determinative dispositions of the contentions of the
parties. We find no merit in petitioners' stand.
Under the 1961 pronouncement and ruling of the Supreme Court in Vda. de Zaldarriaga vs. Enriquez, 1
SCRA 1188 (and the sequel case of Vda. de Zaldarriaga vs. Zaldarriaga 2 SCRA 356), the lower court's
dismissal of petitioners' proposed appeal from its December 27, 1969 judgment as affirmed by the Court of
Appeals on the ground of prematurity in that the judgment was not final but interlocutory was in order. As was
said in said case: prcd
"It is true that in Africa vs. Africa, 42 Phil. 934 and other cases it was held — contrary to
the rule laid down in Ron vs. Mojica, 8 Phil. 328; Rodriguez vs. Ravilan, 17 Phil. 63 — that in a
partition case where defendant relies on the defense of exclusive ownership, the action becomes
one for title and the decision or order directing partition is final, but the ruling to this effect has been
expressly reversed in the Fuentebella case which, in our opinion, expresses the correct view,
considering that a decision or order directing partition is not final because it leaves something more
to be done in the trial court for the complete disposition of the case, namely, the appointment of
commissioners, the proceedings to be had before them, the submission of their report which,
according to law, must be set for hearing. In fact, it is only after said hearing that the court may
render a final judgment finally disposing of the action (Rule 71, section 7, Rules of Court)." (1
SCRA at page 1193)
It should be noted, however, that the said ruling in Zaldarriaga as based on Fuentebella vs. Carrascoso,
XIV Lawyers Journal 305 (May 27, 1942), has been expressly abandoned by the Court in Miranda vs. Court of
Appeals, 71 SCRA 295; 331-333 (June 18, 1976) wherein Mr. Justice Teehankee, speaking for the Court, laid
down the following doctrine:
"The Court, however, deems it proper for the guidance of the bench and bar to now
declare as is clearly indicated from the compelling reasons and considerations herein above
stated:
— that the Court considers the better rule to be that stated in H. E. Heacock Co. vs.
American Trading Co., to wit, that where the primary purpose of a case is to ascertain and
determine who between plaintiff and defendant is the true owner and entitled to the exclusive use
of the disputed property, 'the judgment . . . rendered by the lower court [is] a judgment on the
merits as to those questions, and [that] the order of the court for an accounting was based upon,
and is incidental to the judgment on the merits. That is to say, that the judgment . . . [is] a final
judgment . . .; that in this kind of a case an accounting is a mere incident to the judgment; that an
appeal lies from the rendition of the judgment as rendered . . .' (as is widely held by a great number
of judges and members of the bar, as shown by the cases so decided and filed and still pending
with the Court) for the fundamental reasons therein stated that 'this is more in harmony with the
administration of justice and the spirit and intent of the [Rules]. If on appeal the judgment of the
lower court is affirmed, it would not in the least work an injustice to any of the legal rights of
[appellee]. On the other hand, if for any reason this court should reverse the judgment of the lower
court, the accounting would be a waste of time and money, and might work a material injury to the
[appellant]; and
— that accordingly, the contrary ruling in Fuentebella vs. Carrascoso which expressly
reversed the Heacock case and a line of similar decisions and ruled that such a decision for
recovery of property with accounting 'is not final but merely interlocutory and therefore not
appealable' and subsequent cases adhering to the same must be now in turn abandoned and set
aside.
"Fuentebella adopted instead the opposite line of conflicting decisions mostly in partition
proceedings and exemplified by Ron vs. Mojica, 8 Phil. 928 (under the old Code of Civil
Procedure) that an order for partition of real property is not final and appealable until after the
actual partition of the property as reported by the court-appointed commissioners and approved by
the court in its judgment accepting the report. It must be especially noted that such rule governing
partitions is now so expressly provided and spelled out in Rule 69 of the Rules of Court, with
special reference to Sections 1, 2, 3, 6, 7 and 11, to wit, that there must first be a preliminary order
for partition of the real estate (section 2) and where the par ties-co-owners cannot agree, the court-
appointed commissioners make a plan of actual partition which must first he passed upon and
accepted by the trial court and embodied in a judgment to be rendered by it (sections 6 and 11). In
partition cases, it must be further borne in mind that Rule 69, section 1 refers to 'a person having
the right to compel the partition of real estate,' so that the general rule of partition that an appeal
will not lie until the partition or distribution proceedings are terminated will not apply where
appellant claims exclusive ownership of the whole property and denies the adverse party's right to
any partition, as was the ruling in Villanueva vs. Capistrano and Africa vs. Africa supra,
Fuentebella's express reversal of these cases must likewise be deemed now also abandoned in
view of the Court's expressed preference for the rationale of the Heacock case.
"The Court's considered opinion is that imperative considerations of public policy and of
sound practice in the courts and adherence to the constitutional mandate of simplified, just, speedy
and inexpensive determination of every action call for considering such judgments for recovery of
property with accounting as final judgments which are duly appealable (and would therefore
become final and executory if not appealed within the reglementary period) with the accounting as
a mere incident of the judgment to be rendered during the course of the appeal as provided in Rule
39, section 4 or to be implemented at the execution stage upon final affirmance on appeal of the
judgment (as in Court of Industrial Relations unfair labor practice cases ordering the reinstatement
of the worker with accounting, computation and payment of his backwages less earnings
elsewhere during his layoff) and that the only reason given in Fuentebella for the contrary ruling,
viz, `the general harm that would follow from throwing the door open to multiplicity of appeals in a
single case' is of lesser import and consequence." (Emphasis copied)
The Miranda ruling has since then been applied as the new rule by a unanimous Court in Valdez vs.
Bagasao, 82 SCRA 22 (March 8, 1978).
If there were a valid genuine claim of exclusive ownership of the inherited properties on the part of
petitioners to respondents' action for partition, then under the Miranda ruling, petitioners would be sustained, for
as expressly held therein "the general rule of partition that an appeal will not lie until the partition or distribution
proceedings are terminated will not apply where appellant claims exclusive ownership of the whole property and
denies the adverse party's right to any partition."
But this question has now been rendered moot and academic for the very issue of exclusive ownership
claimed by petitioners to deny and defeat respondents' right to partition — which is the very core of their rejected
appeal — has been squarely resolved herein against them, as if the appeal had been given due course. The
Court has herein expressly sustained the trial court's findings, as affirmed by the Court of Appeals, that the
assets or properties of the defunct company constitute the estate of the deceased proprietor (supra at page 7)
and the defunct company's assertion of ownership of the properties is a legal contradiction and would but thwart
the liquidation and final distribution and partition of the properties among the parties hereof as children of their
deceased father Forrest L. Cease. There is therefore no further hindrance to effect the partition of the properties
among the parties in implementation of the appealed judgment. LexLib
One last consideration. Parties are brothers and sisters, legal heirs of their deceased father, Forrest L.
Cease. By all rights in law and jurisprudence, each is entitled to share and share alike in the estate, which the
trial court correctly ordained and sustained by the appellate court. Almost 20 years have lapsed since the filing of
Special Proceedings No. 3893 for the administration of the Estate of Forrest L. Cease and Civil Case No. 6326
for liquidation and partition of the assets of the defunct Tiaong Milling and Plantation Co., Inc. A succession of
receivers were appointed by the court to take, keep in possession, preserve and manage properties of the
corporation which at one time showed an income of P386,152.90 and expenses of P308,405:01 for the period
covering January 1, 1960 to August 31, 1967 as per Summary of Operations of Commissioner for Finance
appointed by the Court (Brief for Respondents, p. 38). In the meantime, ejectment cases were filed by and
against the heirs in connection with the properties involved, aggravating the already strained relations of the
parties. A prudent and practical realization of these circumstances ought and must constrain the parties to give
each one his due in law and with fairness and dispatch that their basic rights be enjoyed. And by remanding this
case to the court a quo for the actual partition of the properties, the substantial rights of everyone of the heirs
have not been impaired, for in fact, they have been preserved and maintained.
WHEREFORE, IN VIEW OF THE FOREGOING, the judgment appealed from is hereby AFFIRMED with
costs against the petitioners.
SO ORDERED.
||| (Cease v. Court of Appeals, G.R. No. L-33172, [October 18, 1979], 182 PHIL 61-80)

[G.R. No. 69259. January 26, 1988.]


DELPHER TRADES CORPORATION, and DELFIN PACHECO petitioners, vs. INTERMEDIATE
APPELLATE COURT and HYDRO PIPES PHILIPPINES, INC., respondents.

SYLLABUS

1. CORPORATION LAW; STOCKHOLDER; STOCK SUBSCRIPTION AS MEANS OF BECOMING A STOCKHOLDER IN A


CORPORATION. — After incorporation, one becomes a stockholder of a corporation by subscription or by purchasing stock
directly from the corporation or from individual owners thereof (Salmon, Dexter & Co. v. Unson, 47 Phil. 649, citing Bole v.
Fulton [1912], 233 Pa., 609). In the case at bar, in exchange for their properties, the Pachecos acquired 2,500 original
unissued no par value shares of stocks of the Delpher Trades Corporation. Consequently, the Pachecos became
stockholders of the corporation by subscription. "The essence of the stock subscription is an agreement to take and pay for
original unissued shares of a corporation, formed or to be formed." (Rohrlich 243, cited in Agbayani, Commentaries and
Jurisprudence on the Commercial Laws of the Philippines, Vol. III, 1980 Edition, p. 430).
2. ID.; SHARES OF STOCK; NO-PAR VALUE SHARES, CONSTRUED. — "A no-par value share does not purport to
represent any stated proportionate interest in the capital stock measured by value, but only an aliquot part of the whole
number of such shares of the issuing corporation. The holder of no-par shares may see from the certificate itself that he is
only an aliquot sharer in the assets of the corporation. But this character of proportionate interest is not hidden beneath a
false appearance of a given sum in money, as in the case of par value shares. The capital stock of a corporation issuing
only no-par value shares is not set forth by a stated amount of money, but instead is expressed to be divided into a stated
number of shares, such as, 1,000 shares. This indicates that a shareholder of 100 such shares is an aliquot sharer in the
assets of the corporation, no matter what value they may have, to the extent of 100/1,000 or 1/10. Thus, by removing the
par value of shares, the attention of persons interested in the financial condition of a corporation is focused upon the value
of assets and the amount of its debts." (Agbayani, Commentaries and Jurisprudence on the Commercial Laws of the
Philippines, Vol. III, 1980 Edition, p. 107)
3. ID.; INCORPORATION OF A CORPORATION; INVESTMENT IN ANOTHER WAY TO CHANGE NATURE OF
OWNERSHIP; CASE AT BAR. — It is to be stressed that by their ownership of the 2,500 no par shares of stock, the
Pachecos have control of the corporation. Their equity capital is 55% as against 45% of the other stockholders, who also
belong to the same family group. In effect, the Delpher Trades Corporation is a business conduit of the Pachecos. What
they really did was to invest their properties and change the nature of their ownership from unincorporated to incorporated
form by organizing Delpher Trades Corporation to take control of their properties and at the same time save on inheritance
taxes.
4. TAXATION; RESORT TO LEGAL MEANS TO DECREASE PAYMENT OF TAXES BY A TAXPAYER; RIGHT CANNOT
BE DOUBTED. — The records do not point to anything wrong or objectionable about this "estate planning" scheme resorted
to by the Pachecos. "The legal right of a taxpayer to decrease the amount of what otherwise could be his taxes or altogether
avoid them, by means which the law permits, cannot be doubted." (Liddell & Co., Inc. v. The collector of Internal Revenue, 2
SCRA 632 citing Gregory v. Helvering, 293 U.S. 465, 7 L. ed. 596).
5. CIVIL LAW; DEED OF EXCHANGE; NOT CONSIDERED A DEED OF SALE. — The "Deed of Exchange" of property
between the Pachecos and Delpher Trades Corporation cannot be considered a contract of sale. There was no transfer of
actual ownership interests by the Pachecos to a third party. The Pacheco family merely changed their ownership from one
form to another. The ownership remained in the same hands. Hence, the private respondent has no basis for its claim of a
right of first refusal under the lease contract.

DECISION

GUTIERREZ, JR., J p:

The petitioners question the decision of the Intermediate Appellate Court which sustained the private respondent's
contention that the deed of exchange whereby Delfin Pacheco and Pelagia Pacheco conveyed a parcel of land to Delpher
Trades Corporation in exchange for 2,500 shares of stock was actually a deed of sale which violated a right of first refusal
under a lease contract.
Briefly, the facts of the case are summarized as follows:
"In 1974, Delfin Pacheco and his sister, Pelagia Pacheco, were the owners of 27,169 square meters of
real estate identified as Lot. No. 1095, Malinta Estate, in the Municipality of Polo (now Valenzuela),
Province of Bulacan (now Metro Manila) which is covered by Transfer Certificate of Title No. T-4240 of
the Bulacan land registry.
"On April 3, 1974, the said co-owners leased to Construction Components International Inc. the same
property and providing that during the existence or after the term of this lease the lessor should he decide
to sell the property leased shall first offer the same to the lessee and the letter has the priority to buy
under similar conditions (Exhibits A to A-5)
"On August 3, 1974, lessee Construction Components International, Inc. assigned its rights and
obligations under the contract of lease in favor of Hydro Pipes Philippines, Inc. with the signed conformity
and consent of lessors Delfin Pacheco and Pelagia Pacheco (Exhs. B to B-6 inclusive)
"The contract of lease, as well as the assignment of lease were annotated at the back of the title, as per
stipulation of the parties (Exhs. A to D-3 inclusive)
"On January 3, 1976, a deed of exchange was executed between lessors Delfin and Pelagia Pacheco
and defendant Delpher Trades Corporation whereby the former conveyed to the latter the leased property
(TCT No. T-4240) together with another parcel of land also located in Malinta Estate, Valenzuela, Metro
Manila (TCT No. 4273) for 2,500 shares of stock of defendant corporation with a total value of
P1,500,000.00 (Exhs. C to C-5, inclusive)" (pp. 44-45, Rollo)
On the ground that it was not given the first option to buy the leased property pursuant to the proviso in the lease
agreement, respondent Hydro Pipes Philippines, Inc., filed an amended complaint for reconveyance of Lot. No. 1095 in its
favor under conditions similar to those whereby Delpher Trades Corporation acquired the property from Pelagia Pacheco
and Delphin Pacheco.
After trial, the Court of First Instance of Bulacan ruled in favor of the plaintiff. The dispositive portion of the decision reads:
"ACCORDINGLY, the judgment is hereby rendered declaring the valid existence of the plaintiff's
preferential right to acquire the subject property (right of first refusal) and ordering the defendants and all
persons deriving rights therefrom to convey the said property to plaintiff who may offer to acquire the
same at the rate of P14.00 per square meter, more or less, for Lot 1095 whose area is 27,169 square
meters only. Without pronouncement as to attorney's fees and costs. (Appendix I; Rec., pp. 246-247)."
(Appellant's Brief, pp. 1-2; p. 134, Rollo)
The lower court's decision was affirmed on appeal by the Intermediate Appellate Court.
The defendants-appellants, now the petitioners, filed a petition for certiorari to review the appellate court's decision.
We initially denied the petition but upon motion for reconsideration, we set aside the resolution denying the petition and
gave it due course.
The petitioners allege that:
"The denial of the petition will work great injustice to the petitioners, in that:
"1. Respondent Hydro Pipes Philippines, Inc. ('private respondent') will acquire from petitioners a parcel
of industrial land consisting of 27,169 square meters or 2.7 hectares (located right after the Valenzuela,
Bulacan exit of the toll expressway) for only P14/sq. meter, or a total of P380,366, although the prevailing
value thereof is approximately P300/sq. meter or P8.1 Million;
"2. Private respondent is allowed to exercise its right of first refusal even if there is no 'sale' or transfer of
actual ownership interests by petitioners to third parties; and
"3. Assuming arguendo that there has been a transfer of actual ownership interests, private respondent
will acquire the land not under 'similar conditions' by which it was transferred to petitioner Delpher Trades
Corporation, as provided in the same contractual provision invoked by private respondent." (pp. 251-252,
Rollo)
The resolution of the case hinges on whether or not the "Deed of Exchange" of the properties executed by the Pachecos on
the one hand and the Delpher Trades Corporation on the other was meant to be a contract of sale which, in effect,
prejudiced the private respondent's right of first refusal over the leased property included in the "deed of exchange."
Eduardo Neria, a certified public accountant and son-in-law of the late Pelagia Pacheco testified that Delpher Trades
Corporation is a family corporation; that the corporation was organized by the children of the two spouses (spouses Pelagia
Pacheco and Benjamin Hernandez and spouses Delfin Pacheco and Pilar Angeles) who owned in common the parcel of
land leased to Hydro Pipes Philippines in order to perpetuate their control over the property through the corporation and to
avoid taxes; that in order to accomplish this end, two pieces of real estate, including Lot No. 1095 which had been leased to
Hydro Pipes Philippines, were transferred to the corporation; that the leased property was transferred to the corporation by
virtue of a deed of exchange of property; that in exchange for these properties, Pelagia and Delfin acquired 2,500 unissued
no par value shares of stock which are equivalent to a 55% majority in the corporation because the other owners only
owned 2,000 shares; and that at the time of incorporation, he knew all about the contract of lease of Lot. No. 1095 to Hydro
Pipes Philippines. In the petitioners' motion for reconsideration, they refer to this scheme as "estate planning." (p. 252,
Rollo) LibLex

Under this factual backdrop, the petitioners contend that there was actually no transfer of ownership of the subject parcel of
land since the Pachecos remained in control of the property. Thus, the petitioners allege: "Considering that the beneficial
ownership and control of petitioner corporation remained in the hands of the original co-owners, there was no transfer of
actual ownership interests over the land when the same was transferred to petitioner corporation in exchange for the latter's
shares of stock. The transfer of ownership, if anything, was merely in form but not in substance. In reality, petitioner
corporation is a mere alter ego or conduit of the Pacheco co-owners; hence the corporation and the co-owners should be
deemed to be the same, there being in substance and in effect an identity of interest." (p. 254, Rollo)
The petitioners maintain that the Pachecos did not sell the property. They argue that there was no sale and that they
exchanged the land for shares of stocks in their own corporation. "Hence, such transfer is not within the letter, or even spirit
of the contract. There is a sale when ownership is transferred for a price certain in money or its equivalent (Art. 1468, Civil
Code) while there is a barter or exchange when one thing is given in consideration of another thing (Art. 1638, Civil Code)."
(pp. 254-255, Rollo)
On the other hand, the private respondent argues that Delpher Trades Corporation is a corporate entity separate and
distinct from the Pachecos. Thus, it contends that it cannot be said that Delpher Trades Corporation is the Pacheco's same
alter ego or conduit; that petitioner Delfin Pacheco, having treated Delpher Trades Corporation as such a separate and
distinct corporate entity, is not a party who may allege that this separate corporate existence should be disregarded. It
maintains that there was actual transfer of ownership interests over the leased property when the same was transferred to
Delpher Trades Corporation in exchange for the latter's shares of stock.
We rule for the petitioners.
After incorporation, one becomes a stockholder of a corporation by subscription or by purchasing stock directly from the
corporation or from individual owners thereof (Salmon, Dexter & Co. v. Unson, 47 Phil. 649, citing Bole v. Fulton [1912], 233
Pa., 609). In the case at bar, in exchange for their properties, the Pachecos acquired 2,500 original unissued no par value
shares of stocks of the Delpher Trades Corporation. Consequently, the Pachecos became stockholders of the corporation
by subscription. "The essence of the stock subscription is an agreement to take and pay for original unissued shares of a
corporation, formed or to be formed." (Rohrlich 243, cited in Agbayani, Commentaries and Jurisprudence on the
Commercial Laws of the Philippines, Vol. III, 1980 Edition, p. 430) It is significant that the Pachecos took no par value
shares in exchange for their properties.
"A no-par value share does not purport to represent any stated proportionate interest in the capital stock
measured by value, but only an aliquot part of the whole number of such shares of the issuing
corporation. The holder of no-par shares may see from the certificate itself that he is only an aliquot
sharer in the assets of the corporation. But this character of proportionate interest is not hidden beneath
a false appearance of a given sum in money, as in the case of par value shares. The capital stock of a
corporation issuing only no-par value shares is not set forth by a stated amount of money, but instead is
expressed to be divided into a stated number of shares, such as, 1,000 shares. This indicates that a
shareholder of 100 such shares is an aliquot sharer in the assets of the corporation, no matter what value
they may have, to the extent of 100/1,000 or 1/10. Thus, by removing the par value of shares, the
attention of persons interested in the financial condition of a corporation is focused upon the value of
assets and the amount of its debts." (Agbayani, Commentaries and Jurisprudence on the Commercial
Laws of the Philippines, Vol. III, 1980 Edition, p. 107)
Moreover, there was no attempt to state the true or current market value of the real estate. Land valued at P300.00 a
square meter was turned over to the family's corporation for only P14.00 a square meter. LexLib
It is to be stressed that by their ownership of the 2,500 no par shares of stock, the Pachecos have control of the corporation.
Their equity capital is 55% as against 45% of the other stockholders, who also belong to the same family group.
In effect, the Delpher Trades Corporation is a business conduit of the Pachecos. What they really did was to invest their
properties and change the nature of their ownership from unincorporated to incorporated form by organizing Delpher Trades
Corporation to take control of their properties and at the same time save on inheritance taxes.
As explained by Eduardo Neria:
xxx xxx xxx
ATTY. LINSANGAN:
"Q Mr. Neria, from the point of view of taxation, is there any benefit to the spouses
Hernandez and Pacheco in connection with their execution of a deed of exchange
on the properties for no par value shares of the defendant corporation?
"A Yes, sir.
COURT:
"Q What do you mean by 'point of view'?
"A To take advantage for both spouses and corporation in entering in the deed of
exchange.
ATTY. LINSANGAN:
"Q (What do you mean by 'point of view'?) What are these benefits to the spouses of this
deed of exchange?
"A Continuous control of the property, tax exemption benefits, and other inherent benefits
in a corporation.
"Q What are these advantages to the said spouses from the point of view of taxation in
entering in the deed of exchange?
"A Having fulfilled the conditions in the income tax law, providing for tax free exchange of
property, they were able to execute the deed of exchange free from income tax
and acquire a corporation.
"Q What provision in the income tax law are you referring to?
"A I refer to Section 35 of the National Internal Revenue Code under par. C-sub-par. (2)
Exceptions regarding the provision which I quote: 'No gain or loss shall also be
recognized if a person exchanges his property for stock in a corporation of which
as a result of such exchange said person alone or together with others not
exceeding four persons gains control of said corporation.'
"Q Did you explain to the spouses this benefit at the time you executed the deed of
exchange?
"A Yes, sir.
"Q You also, testified during the last hearing that the decision to have no par value share
in the defendant corporation was for the purpose of flexibility. Can you explain
flexibility in connection with the ownership of the property in question?
"A There is flexibility in using no par value shares as the value is determined by the board
of directors in increasing capitalization. The board can fix the value of the shares
equivalent to the capital requirements of the corporation.
"Q Now also from the point of taxation, is there any flexibility in the holding by the
corporation of the property in question?
"A Yes, since a corporation does not die it can continue to hold on to the property
indefinitely for a period of at least 50 years. On the other hand, if the property is
held by the spouse the property will be tied up in succession proceedings and the
consequential payments of estate and inheritance taxes when an owner dies.
"Q Now what advantage is this continuity in relation to ownership by a particular person of
certain properties in respect to taxation?
"A The property is not subjected to taxes on succession as the corporation does not die.
"Q So the benefit you are talking about are inheritance taxes?
"A Yes, sir." (pp. 3-5, tsn., December 15, 1981).
The records do not point to anything wrong or objectionable about this "estate planning" scheme resorted to by the
Pachecos. "The legal right of a taxpayer to decrease the amount of what otherwise could be his taxes or altogether avoid
them, by means which the law permits, cannot be doubted." (Liddell & Co., Inc. v. The Collector of Internal Revenue, 2
SCRA 632 citing Gregory v. Helvering, 293 U.S. 465, 7 L. ed. 596). LLjur
The "Deed of Exchange" of property between the Pachecos and Delpher Trades Corporation cannot be considered a
contract of sale. There was no transfer of actual ownership interests by the Pachecos to a third party. The Pacheco family
merely changed their ownership from one form to another. The ownership remained in the same hands. Hence, the private
respondent has no basis for its claim of a right of first refusal under the lease contract.
WHEREFORE, the instant petition is hereby GRANTED. The questioned decision and resolution of the then Intermediate
Appellate Court are REVERSED and SET ASIDE. The amended complaint in Civil Case No. 885-V-79 of the then Court of
First Instance of Bulacan is DISMISSED. No costs. SO ORDERED.
||| (Delpher Trades Corp. v. Intermediate Appellate Court, G.R. No. 69259, [January 26, 1988], 241 PHIL 358-366)

Garrett v. Southern Railway Company


278 F.2d 424 (6th Cir. 1960)
MARTIN, Circuit Judge.

In this court action, a wheel moulder in the employ of Lenoir Car Works, a Tennessee corporation, brought suit against the
Southern Railway Company for alleged injuries from silicosis, claimed to have been contracted from silica dust which
permeated the foundry. Upon completion of the trial, at which much evidence was received, the United States District Judge
entered judgment in favor of the defendant railway company. Plaintiff's appeal to this court is from that judgment. The
opinion of the District Court will be found in 173 F. Supp. 915, 918.
Appellant contends that the district court was in error in finding that the Lenoir Car Works was not the alter ego, adjunct,
subsidiary, agent or instrumentality of the appellee, Southern Railway Company, and consequently was not required to
respond in damages for the personal injuries sustained by appellant. Appellant insists, moreover, that the district court also
erred in finding that the Southern Railway Company did not exercise such control over Lenoir Car Works as to require a
determination that the railway company was the employer of appellant within the meaning of the Federal Employers' Liability
Act, 45 U.S.C.A. § 51 et seq.
In a carefully considered opinion, the United States District Court found that there was no evidence that the Southern
Railway Company dictated the management of the Lenoir Car Works, although it owned the entire capital stock of that
corporation. It was found that the evidence indicated that an individual, Henry Marius, was in full control of the operation of
the car works. He established prices and handled all negotiations and collective bargaining agreements. Lenoir paid local
taxes, had local counsel, and maintained workmen's compensation.
The district judge noted that "a substantial part of its requirements in the field of operation of Lenoir were bought
elsewhere." Lenoir sold substantial quantities of its product to other companies than Southern; it operated no rolling stock;
and had nothing to do with the transportation business.

The court found further that the facts did not reveal such intimacy and inseparability of control as would lead to the
conclusion that the Southern Railway Company and Lenoir Car Works were one and the same. Lenoir was not, in the words
of the Federal Employers' Liability Act, a "common carrier by railroad"; it was not performing the non-delegable duties of a
railroad; it was not the operator of a terminal; and it performed no switching or transportation functions at all. It was
distinctively a manufacturer, performing no common-carrier operations.
Upon the issue of whether or not the control by Southern of Lenoir was of such character as to constitute the latter a mere
adjunct of the former, the court held that no such control was evident. The following cases were cited: Kentucky Electric
Power Co. v. Norton Coal Mining Co., 6 Cir., 93 F.2d 923; Atlantic Coast Line v. Shields, 5 Cir., 220 F.2d 242, 246; Taylor v.
Standard Gas Electric Co., 10 Cir., 96 F.2d 693.

In Kentucky Electric Power Company v. Norton Coal Company, supra, 93 F.2d at page 926, this court said: "On the other
hand, it is likewise well settled that a corporation is ordinarily an entity, separate and apart from its stockholders, and mere
ownership of all the stock of one corporation by another, and the identity of officers of one with officers of another, are not
alone sufficient to create identity of corporate interest between the two companies or to create the relation of principal and
agent or to create a representative or fiduciary relationship between the two. If such stock ownership and potential control
be resorted to only for the purpose of normally participating in the affairs of the subsidiary corporation in a manner usual to
stockholders and not for the purpose of taking some unfair advantage of the subsidiary or using it as a mere adjunct to the
main corporation or as a subterfuge to justify wrongdoing, their identity as separate corporations will not be disregarded but
their respective rights when dealing with each other in respect to their separate property will be recognized and maintained.
The extent of stock ownership and mere potential control of one company over another has never been regarded as the
determining factor in the consideration of such cases. Something must be disclosed to indicate the exercise of undue
domination or influence resulting in an infringement upon the rights of the subservient corporation for the benefit of the
dominant one. Otherwise, the rights of the separate corporations in respect to their corporate property must be governed by
the rules applicable in ordinary cases. [Citing cases.]"
The district court concluded that, upon the principles of the Kentucky Electric Power Company case, the control of Southern
Railway Company over Lenoir Car Works was not such as constituted Lenoir an adjunct of Southern. We think this
conclusion was correct. Accordingly, the judgment of the district court dismissing the complaint herein is affirmed.
Garrett v. Southern Railway Company, 278 F.2d 424, (6th Cir. 1960)

[G.R. No. 151438. July 15, 2005.]

JARDINE DAVIES, INC., petitioner, vs. JRB REALTY, INC., respondent.

SYLLABUS

1. COMMERCIAL LAW; CORPORATION LAW; CORPORATIONS; DOCTRINE OF PIERCING THE VEIL OF


CORPORATE FICTION; WHEN APPLICABLE; RATIONALE BEHIND THE DOCTRINE. — It is an elementary and
fundamental principle of corporation law that a corporation is an artificial being invested by law with a personality
separate and distinct from its stockholders and from other corporations to which it may be connected. While a
corporation is allowed to exist solely for a lawful purpose, the law will regard it as an association of persons or in case of
two corporations, merge them into one, when this corporate legal entity is used as a cloak for fraud or illegality. This is
the doctrine of piercing the veil of corporate fiction which applies only when such corporate fiction is used to defeat
public convenience, justify wrong, protect fraud or defend crime. The rationale behind piercing a corporation's identity is
to remove the barrier between the corporation from the persons comprising it to thwart the fraudulent and illegal
schemes of those who use the corporate personality as a shield for undertaking certain proscribed activities.
2. ID.; ID.; ID.; ID.; REQUISITES; A SUBSIDIARY HAS AN INDEPENDENT AND SEPARATE JURIDICAL
PERSONALITY DISTINCT FROM THAT OF ITS PARENT COMPANY; CASE AT BAR. — While it is true that Aircon is
a subsidiary of the petitioner, it does not necessarily follow that Aircon's corporate legal existence can just be
disregarded. In Velarde v. Lopez, Inc., the Court categorically held that a subsidiary has an independent and separate
juridical personality, distinct from that of its parent company; hence, any claim or suit against the latter does not bind the
former, and vice versa. In applying the doctrine, the following requisites must be established: (1) control, not merely
majority or complete stock control; (2) such control must have been used by the defendant to commit fraud or wrong, to
perpetuate the violation of a statutory or other positive legal duty, or dishonest acts in contravention of plaintiff's legal
rights; and (3) the aforesaid control and breach of duty must proximately cause the injury or unjust loss complained of.
The records bear out that Aircon is a subsidiary of the petitioner only because the latter acquired Aircon's majority of
capital stock. It, however, does not exercise complete control over Aircon; nowhere can it be gathered that the petitioner
manages the business affairs of Aircon. Indeed, no management agreement exists between the petitioner and Aircon,
and the latter is an entirely different entity from the petitioner.
3. ID.; ID.; ID.; ID.; ABSENT FRAUD OR OTHER PUBLIC CONSIDERATIONS, THE EXISTENCE OF
INTERLOCKING DIRECTORS, CORPORATE OFFICERS AND SHAREHOLDERS DOES NOT JUSTIFY PIERCING
THE VEIL OF CORPORATE FICTION; CASE AT BAR. — The existence of interlocking directors, corporate officers and
shareholders, which the respondent court considered, is not enough justification to pierce the veil of corporate fiction, in
the absence of fraud or other public policy considerations. But even when there is dominance over the affairs of the
subsidiary, the doctrine of piercing the veil of corporate fiction applies only when such fiction is used to defeat public
convenience, justify wrong, protect fraud or defend crime. To warrant resort to this extraordinary remedy, there must be
proof that the corporation is being used as a cloak or cover for fraud or illegality, or to work injustice. Any piercing of the
corporate veil has to be done with caution. The wrongdoing must be clearly and convincingly established. It cannot just
be presumed. In the instant case, there is no evidence that Aircon was formed or utilized with the intention of defrauding
its creditors or evading its contracts and obligations. There was nothing fraudulent in the acts of Aircon in this case.
Aircon, as a manufacturing firm of air conditioners, complied with its obligation of providing two air conditioning units for
the second floor of the Blanco Center in good faith, pursuant to its contract with the respondent. Unfortunately, the
performance of the air conditioning units did not satisfy the respondent despite several adjustments and corrective
measures. In a Letter dated October 22, 1980, the respondent even conceded that Fedders Air Conditioning USA has
not yet perhaps perfected its technology of rotary compressors, and agreed to change the compressors with the semi-
hermetic type. Thus, Aircon substituted the units with serviceable ones which delivered the cooling temperature needed
for the law office. After enjoying ten (10) years of its cooling power, respondent cannot now complain about the
performance of these units, nor can it demand a replacement thereof.
4. CIVIL LAW; DAMAGES; ACTUAL OR COMPENSATORY DAMAGES; ACTUAL AMOUNT OF LOSS MUST
BE PROVED WITH A REASONABLE DEGREE OF CERTAINTY PREMISED UPON COMPETENT PROOF AND ON
THE BEST EVIDENCE OBTAINABLE BY THE INJURED PARTY; CASE AT BAR. — Moreover, it was reversible error
to award the respondent the amount of P556,551.55 representing the alleged 30% unsaved electricity costs and
P185,951.67 as maintenance cost without showing any basis for such award. To justify a grant of actual or
compensatory damages, it is necessary to prove with a reasonable degree of certainty, premised upon competent proof
and on the best evidence obtainable by the injured party, the actual amount of loss. The respondent merely based its
cause of action on Aircon's alleged representation that Fedders air conditioners with rotary compressors can save as
much as 30% on electricity compared to other brands. Offered in evidence were newspaper advertisements published
on April 12 and 26, 1981. The respondent then recorded its electricity consumption from October 21, 1981 up to April 3,
1995 and computed 30% thereof, which amounted to P556,551.55. The Court rules that this amount is highly
speculative and merely hypothetical, and for which the petitioner can not be held accountable.
5. ID.; ID.; ID.; MAINTENANCE COST CANNOT BE AWARDED ABSENT CONVINCING PROOF THEREOF.
— Likewise, there is no basis for the award of P185,951.67 representing maintenance cost. The respondent merely
submitted a schedule prepared by the respondent's accountant, listing the alleged repair costs from March 1987 up to
June 1994. Such evidence is self-serving and can not also be given probative weight, considering that there are no
proofs of receipts, vouchers, etc., which would substantiate the amounts paid for such services. Absent any more
convincing proof, the Court finds that the respondent's claims are without basis, and cannot, therefore, be awarded.
6. ID.; OBLIGATIONS AND CONTRACTS; CONTRACTS; PRIVITY OF CONTRACTS TAKE EFFECT ONLY
BETWEEN PARTIES, THEIR SUCCESSORS-IN-INTEREST, HEIRS AND ASSIGNS. — We sustain the petitioner's
separateness from that of Aircon in this case. It bears stressing that the petitioner was never a party to the contract.
Privity of contracts take effect only between parties, their successors-in-interest, heirs and assigns. The petitioner,
which has a separate and distinct legal personality from that of Aircon, cannot, therefore, be held liable.

DECISION

CALLEJO, SR., J p:
Before us is a petition for review of the Decision 1 of the Court of Appeals (CA) in CA-G.R. CV No. 54201
affirming in toto that of the Regional Trial Court (RTC) in Civil Case No. 90-237 for specific performance; and the
Resolution dated January 11, 2002 denying the motion for reconsideration thereof.
The facts are as follows:
In 1979-1980, respondent JRB Realty, Inc. built a nine-storey building, named Blanco Center, on its parcel of
land located at 119 Alfaro St., Salcedo Village, Makati City. An air conditioning system was needed for the Blanco Law
Firm housed at the second floor of the building. On March 13, 1980, the respondent's Executive Vice-President, Jose R.
Blanco, accepted the contract quotation of Mr. A.G. Morrison, President of Aircon and Refrigeration Industries, Inc.
(Aircon), for two (2) sets of Fedders Adaptomatic 30,000 kcal (Code: 10-TR) air conditioning equipment with a net total
selling price of P99,586.00. 2 Thereafter, two (2) brand new packaged air conditioners of 10 tons capacity each to
deliver 30,000 kcal or 120,000 BTUH 3 were installed by Aircon. When the units with rotary compressors were installed,
they could not deliver the desired cooling temperature. Despite several adjustments and corrective measures, the
respondent conceded that Fedders Air Conditioning USA's technology for rotary compressors for big capacity
conditioners like those installed at the Blanco Center had not yet been perfected. The parties thereby agreed to replace
the units with reciprocating/semi-hermetic compressors instead. In a Letter dated March 26, 1981, 4 Aircon stated that it
would be replacing the units currently installed with new ones using rotary compressors, at the earliest possible time.
Regrettably, however, it could not specify a date when delivery could be effected.
TempControl Systems, Inc. (a subsidiary of Aircon until 1987) undertook the maintenance of the units, inclusive
of parts and services. In October 1987, the respondent learned, through newspaper ads, 5 that Maxim Industrial and
Merchandising Corporation (Maxim, for short) was the new and exclusive licensee of Fedders Air Conditioning USA in
the Philippines for the manufacture, distribution, sale, installation and maintenance of Fedders air conditioners. The
respondent requested that Maxim honor the obligation of Aircon, but the latter refused. Considering that the ten-year
period of prescription was fast approaching, to expire on March 13, 1990, the respondent then instituted, on January 29,
1990, an action for specific performance with damages against Aircon & Refrigeration Industries, Inc., Fedders Air
Conditioning USA, Inc., Maxim Industrial & Merchandising Corporation and petitioner Jardine Davies, Inc. 6 The latter
was impleaded as defendant, considering that Aircon was a subsidiary of the petitioner. The respondent prayed that
judgment be rendered, as follows:
1. Ordering the defendants to jointly and severally at their account and expense deliver, install
and place in operation two brand new units of each 10-tons capacity Fedders unitary packaged air
conditioners with Fedders USA's technology perfected rotary compressors to always deliver 30,000
kcal or 120,000 BTUH to the second floor of the Blanco Center building at 119 Alfaro St., Salcedo
Village, Makati, Metro Manila;
2. Ordering defendants to jointly and severally reimburse plaintiff not only the sums of
P415,118.95 for unsaved electricity from 21st October 1981 to 7th January 1990 and P99,287.77 for
repair costs of the two service units from 7th March 1987 to 11th January 1990, with legal interest
thereon from the filing of this Complaint until fully reimbursed, but also like unsaved electricity costs
and like repair costs therefrom until Prayer No. 1 above shall have been complied with;
3. Ordering defendants to jointly and severally pay plaintiff's P150,000.00 attorney's fees and
other costs of litigation, as well as exemplary damages in an amount not less than or equal to Prayer 2
above; and
4. Granting plaintiff such other and further relief as shall be just and equitable in the premises.
7
Of the four defendants, only the petitioner filed its Answer. The court did not acquire jurisdiction over Aircon
because the latter ceased operations, as its corporate life ended on December 31, 1986. 8 Upon motion, defendants
Fedders Air Conditioning USA and Maxim were declared in default. 9
On May 17 1996, the RTC rendered its Decision, the dispositive portion of which reads:
WHEREFORE, judgment is hereby rendered ordering defendants Jardine Davies, Inc.,
Fedders Air Conditioning USA, Inc. and Maxim Industrial and Merchandising Corporation, jointly and
severally:
1. To deliver, install and place into operation the two (2) brand new units of Fedders
unitary packaged airconditioning units each of 10 tons capacity with rotary
compressors to deliver 30,000 kcal or 120,000 BTUH to the second floor of the
Blanco Center building, or to pay plaintiff the current price for two such units;
2. To reimburse plaintiff the amount of P556,551.55 as and for the unsaved electricity
bills from October 21, 1981 up to April 30, 1995; and another amount of
P185,951.67 as and for repair costs;
3. To pay plaintiff P50,000.00 as and for attorney's fees; and
4. Cost of suit. 10
The petitioner filed its notice of appeal with the CA, alleging that the trial court erred in holding it liable because
it was not a party to the contract between JRB Realty, Inc. and Aircon, and that it had a personality separate and
distinct from that of Aircon. HDCTAc
On March 23, 2000, the CA affirmed the trial court's ruling in toto; hence, this petition.
The petitioner raises the following assignment of errors:
I.
THE COURT OF APPEALS ERRED IN HOLDING JARDINE LIABLE FOR THE ALLEGED
CONTRACTUAL BREACH OF AIRCON SOLELY BECAUSE' THE LATTER WAS FORMERLY
JARDINE'S SUBSIDIARY.
II.
ASSUMING ARGUENDO THAT AIRCON MAY BE CONSIDERED AS JARDINE'S MERE
ALTER EGO, THE COURT OF APPEALS ERRED IN NOT DECLARING AIRCON'S OBLIGATION TO
DELIVER THE TWO (2) AIRCONDITIONING UNITS TO JRB AS HAVING BEEN SUBSTANTIALLY
COMPLIED WITH IN GOOD FAITH.
III.
ASSUMING ARGUENDO THAT AIRCON MAY BE CONSIDERED AS JARDINE'S MERE
ALTER EGO, THE COURT OF APPEALS ERRED IN NOT DECLARING JRB'S CAUSES OF ACTION
AS HAVING BEEN BARRED BY LACHES.
IV.
ASSUMING ARGUENDO THAT AIRCON MAY BE CONSIDERED AS JARDINE'S MERE
ALTER EGO, THE COURT OF APPEALS ERRED IN FINDING JRB ENTITLED TO RECOVER
ALLEGED UNSAVED ELECTRICITY EXPENSES.
V.
THE COURT OF APPEALS ERRED IN HOLDING JARDINE LIABLE TO PAY ATTORNEY'S
FEES.
VI.
THE COURT OF APPEALS ERRED IN NOT HOLDING JRB LIABLE TO JARDINE FOR
DAMAGES. 11
It is the well-settled rule that factual findings of the trial court, as affirmed by the CA, are accorded high respect,
even finality at times. However, considering that the factual findings of the CA and the RTC were based on speculation
and conjectures, unsupported by substantial evidence, the Court finds that the instant case falls under one of the
excepted instances. There is, thus, a need to correct the error.
The trial court ruled that Aircon was a subsidiary of the petitioner, and concluded, thus:
Plaintiff's documentary evidence shows that at the time it contracted with Aircon on March 13,
1980 (Exhibit "D") and on the date the revised agreement was reached on March 26, 1981, Aircon was
a subsidiary of Jardine. The phrase "A subsidiary of Jardine Davies, Inc." was printed on Aircon's
letterhead of its March 13, 1980 contract with plaintiff (Exhibit "D-1"), as well as the Aircon's letterhead
of Jardine's Director and Senior Vice-President A.G. Morrison and Aircon's President in his March 26,
1981 letter to plaintiff (Exhibit "J-2") confirming the revised agreement. Aircon's newspaper ads of April
12 and 26, 1981 and a press release on August 30, 1982 (Exhibits "E," "F" and "L") also show that
defendant Jardine publicly represented Aircon to be its subsidiary.
Records from the Securities and Exchange Commission (SEC) also reveal that as per
Jardine's December 31, 1986 and 1985 Financial Statements that "The company acts as general
manager of its subsidiaries" (Exhibit "P"). Jardine's Consolidated Balance Sheet as of December 31,
1979 filed with the SEC listed Aircon as its subsidiary by owning 94.35% of Aircon (Exhibit "P-1"). Also,
Aircon's reportorial General Information Sheet as of April 1980 and April 1981 filed with the SEC show
that Jardine was 94.34% owner of Aircon (Exhibits "Q" and "R") and that out of seven members of the
Board of Directors of Aircon, four (4) are also of Jardine. SEACTH
Defendant Jardine's witness, Atty. Fe delos Santos-Quiaoit admitted that defendant Aircon,
renamed Aircon & Refrigeration Industries, Inc. "is one of the subsidiaries of Jardine Davies" (TSN,
September 22, 1995, p. 12). She also testified that Jardine nominated, elected, and appointed the
controlling majority of the Board of Directors and the highest officers of Aircon (Ibid, pp. 10, 13-14).
The foregoing circumstances provide justifiable basis for this Court to disregard the fiction of
corporate entity and treat defendant Aircon as part of the instrumentality of co-defendant Jardine. 12
The respondent court arrived at the same conclusion basing its ruling on the following documents, to wit:
(a) Contract/Quotation #78-No. 80-1639 dated March 03, 1980 (Exh. D-1);
(b) Newspaper Advertisements (Exhs. E-1 and F-1);
(c) Letter dated March 26, 1981 of A.G. Morrison, President of Aircon, to Atty. J.R. Blanco
(Exh. J);
(d) News items of Bulletin Today dated August 30, 1982 (Exh. L);
(e) Balance Sheet of Jardine Davies, Inc. as of December 31, 1979 listing Aircon as one of its
subsidiaries (Exh. P);
(f) Financial Statement of Aircon as of December 31, 1982 and 1981 (Exh. S);

(g) Financial Statement of Aircon as of December 31, 1981 (Exh. S-1). 13


Applying the doctrine of piercing the veil of corporate fiction, both the respondent and trial courts conveniently
held the petitioner liable for the alleged omissions of Aircon, considering that the latter was its instrumentality or
corporate alter ego. The petitioner is now before us, reiterating its defense of separateness, and the fact that it is not a
party to the contract. DcaSIH
We find merit in the petition.
It is an elementary and fundamental principle of corporation law that a corporation is an artificial being invested
by law with a personality separate and distinct from its stockholders and from other corporations to which it may be
connected. While a corporation is allowed to exist solely for a lawful purpose, the law will regard it as an association of
persons or in case of two corporations, merge them into one, when this corporate legal entity is used as a cloak for
fraud or illegality. 14 This is the doctrine of piercing the veil of corporate fiction which applies only when such corporate
fiction is used to defeat public convenience, justify wrong, protect fraud or defend crime. 15 The rationale behind
piercing a corporation's identity is to remove the barrier between the corporation from the persons comprising it to
thwart the fraudulent and illegal schemes of those who use the corporate personality as a shield for undertaking certain
proscribed activities. 16
While it is true that Aircon is a subsidiary of the petitioner, it does not necessarily follow that Aircon's corporate
legal existence can just be disregarded. In Velarde v. Lopez, Inc., 17 the Court categorically held that a subsidiary has
an independent and separate juridical personality, distinct from that of its parent company; hence, any claim or suit
against the latter does not bind the former, and vice versa. In applying the doctrine, the following requisites must be
established: (1) control, not merely majority or complete stock control; (2) such control must have been used by the
defendant to commit fraud or wrong, to perpetuate the violation of a statutory or other positive legal duty, or dishonest
acts in contravention of plaintiff's legal rights; and (3) the aforesaid control and breach of duty must proximately cause
the injury or unjust loss complained of. 18
The records bear out that Aircon is a subsidiary of the petitioner only because the latter acquired Aircon's
majority of capital stock. It, however, does not exercise complete control over Aircon; nowhere can it be gathered that
the petitioner manages the business affairs of Aircon. Indeed, no management agreement exists between the petitioner
and Aircon, and the latter is an entirely different entity from the petitioner. 19
Jardine Davies, Inc., incorporated as early as June 28, 1946, 20 is primarily a financial and trading company. Its
Articles of Incorporation states among many others that the purposes for which the said corporation was formed, are as
follows:
(a) To carry on the business of merchants, commission merchants, brokers, factors,
manufacturers, and agents;
(b) Upon complying with the requirements of law applicable thereto, to act as agents of
companies and underwriters doing and engaging in any and all kinds of insurance business. 21
On the other hand, Aircon, incorporated on December 27, 1952, 22 is a manufacturing firm. Its Articles of
Incorporation states that its purpose is mainly —
To carry on the business of manufacturers of commercial and household appliances and
accessories of any form, particularly to manufacture, purchase, sell or deal in air conditioning and
refrigeration products of every class and description as well as accessories and parts thereof, or other
kindred articles; and to erect, or buy, lease, manage, or otherwise acquire manufactories, warehouses,
and depots for manufacturing, assemblage, repair and storing, buying, selling, and dealing in the
aforesaid appliances, accessories and products. . . . 23
The existence of interlocking directors, corporate officers and shareholders, which the respondent court
considered, is not enough justification to pierce the veil of corporate fiction, in the absence of fraud or other public policy
considerations. 24 But even when there is dominance over the affairs of the subsidiary, the doctrine of piercing the veil
of corporate fiction applies only when such fiction is used to defeat public convenience, justify wrong, protect fraud or
defend crime. 25 To warrant resort to this extraordinary remedy, there must be proof that the corporation is being used
as a cloak or cover for fraud or illegality, or to work injustice. 26 Any piercing of the corporate veil has to be done with
caution. 27 The wrongdoing must be clearly and convincingly established. It cannot just be presumed. 28
In the instant case, there is no evidence that Aircon was formed or utilized with the intention of defrauding its
creditors or evading its contracts and obligations. There was nothing fraudulent in the acts of Aircon in this case. Aircon,
as a manufacturing firm of air conditioners, complied with its obligation of providing two air conditioning units for the
second floor of the Blanco Center in good faith, pursuant to its contract with the respondent. Unfortunately, the
performance of the air conditioning units did not satisfy the respondent despite several adjustments and corrective
measures. In a Letter 29 dated October 22, 1980, the respondent even conceded that Fedders Air Conditioning USA
has not yet perhaps perfected its technology of rotary compressors, and agreed to change the compressors with the
semi-hermetic type. Thus, Aircon substituted the units with serviceable ones which delivered the cooling temperature
needed for the law office. After enjoying ten (10) years of its cooling power, respondent cannot now complain about the
performance of these units, nor can it demand a replacement thereof.
Moreover, it was reversible error to award the respondent the amount of P556,551.55 representing the alleged
30% unsaved electricity costs and P185,951.67 as maintenance cost without showing any basis for such award. To
justify a grant of actual or compensatory damages, it is necessary to prove with a reasonable degree of certainty,
premised upon competent proof and on the best evidence obtainable by the injured party, the actual amount of loss. 30
The respondent merely based its cause of action on Aircon's alleged representation that Fedders air conditioners with
rotary compressors can save as much as 30% on electricity compared to other brands. Offered in evidence were
newspaper advertisements published on April 12 and 26, 1981. The respondent then recorded its electricity
consumption from October 21, 1981 up to April 3, 1995 and computed 30% thereof, which amounted to P556,551.55.
The Court rules that this amount is highly speculative and merely hypothetical, and for which the petitioner can not be
held accountable.
First. The respondent merely relied on the newspaper advertisements showing the Fedders window-type air
conditioners, which are far different from the big capacity air conditioning units installed at Blanco Center.
Second. After such print advertisements, the respondent informed Aircon that it was going to install an electric
meter to register its electric consumption so as to determine the electric costs not saved by the presently installed units
with semi-hermetic compressors. Contrary to the allegations of the respondent that this was in pursuance to their
Revised Agreement, no proof was adduced that Aircon agreed to the respondent's proposition. It was a unilateral act on
the part of the respondent, which Aircon did not oblige or commit itself to pay.
Third. Needless to state, the amounts computed are mere estimates representing the respondent's self-serving
claim of unsaved electricity cost, which is too speculative and conjectural to merit consideration. No other proofs,
reports or bases of comparison showing that Fedders Air Conditioning USA could indeed cut down electricity cost by
30% were adduced.
Likewise, there is no basis for the award of P185,951.67 representing maintenance cost. The respondent
merely submitted a schedule 31 prepared by the respondent's accountant, listing the alleged repair costs from March
1987 up to June 1994. Such evidence is self-serving and can not also be given probative weight, considering that there
are no proofs of receipts, vouchers, etc., which would substantiate the amounts paid for such services. Absent any
more convincing proof, the Court finds that the respondent's claims are without basis, and cannot, therefore, be
awarded.
We sustain the petitioner's separateness from that of Aircon in this case. It bears stressing that the petitioner
was never a party to the contract. Privity of contracts take effect only between parties, their successors-in-interest, heirs
and assigns. 32 The petitioner, which has a separate and distinct legal personality from that of Aircon, cannot,
therefore, be held liable.
IN VIEW OF THE FOREGOING, the petition is GRANTED. The assailed decision of the Court of Appeals,
affirming the decision of the Regional Trial Court is REVERSED and SET ASIDE. The complaint of the respondent is
DISMISSED. Costs against the respondent. SO ORDERED.
||| (Jardine Davies Inc. v. JRB Realty Inc., G.R. No. 151438, [July 15, 2005], 502 PHIL 129-142)

[G.R. No. L-47673. October 10, 1946.]

KOPPEL (PHILIPPINES),INC., plaintiff-appellant, vs.ALFREDO L. YATCO, Collector of Internal


Revenue, defendant-appellee.

Padilla, Carlos & Fernando, for appellant.


Solicitor General Ozaeta, First Assistant Solicitor General Reyes and Solicitor Cañizares, for appellee.

SYLLABUS

1. CORPORATIONS; DISREGARD OF CORPORATE FICTION. — A corporation will be looked upon as a


legal entity as a general rule, and until sufficient reason to the contrary appears; but, when the notion of legal entity is
used to defeat public convenience, justify wrong, protect fraud, or defend crime, the law will regard the corporation as
an association of persons.
2. ID.;ID.;CONTROL BY ANOTHER CORPORATION. — The corporate entity is disregard where it is so
organized and controlled, and its affairs are so conducted, as to make it merely an instrumentality, agency, conduit or
adjunct of another corporation.
3. OBLIGATIONS AND CONTRACTS; SALE PERFECTION OF CONSENSUAL CONTRACT; LOCATION OF
PROPERTY AND PLACE OF DELIVERY IMMATERIAL; CASE AT BAR. — While it is true that when the contract was
perfected in the Philippines the pair of Atlas-Diesel Marine Engines were in Sweden and the agreement was to deliver
them C. I. F. Hongkong, the contract of sale being consensual — perfected by mere consent — (Civil Code, article
1445; 10 Manresa, 4th ed.,p. 11),the location of the property and the place of delivery did not matter in the question of
where the agreement was perfected.
4. ID.;ID.;PERFECTION OF, WHEN EXECUTED THROUGH CORRESPONDENCE. — Contracts executed
through correspondence are completed from the time an answer is made accepting the proposition or the conditions by
which the latter may be modified.
5. STATUTORY CONSTRUCTION; INTERPRETATION BY OFFICERS OF ADMINISTRATIVE BRANCHES
NOT BINDING ON COURTS; "STARE DECISIS";CASE AT BAR. — The ruling of the Secretary of Finance, Exhibit M,
was not binding upon the trial court, much less upon this tribunal, since the duty and power of interpreting the laws is
primarily a function of the judiciary. Plaintiff cannot be excused from abiding by this legal principle, nor can it properly be
heard to say that it relied on the Secretary's ruling and that, therefore, the courts should not now apply an interpretation
at variance therewith. The rule of stare decisis is undoubtedly entitled to more respect in the construction of statutes
than the interpretations given by officers of the administrative branches of the government, even those entrusted with
the administration of particular laws; and yet in Philippine Trust Co. and Smith, Bell & Co. vs.Mitchell (59 Phil.,30),this
court refused to follow its own doctrine laid down in a former case, saying: "More important than anything else is that
the court should be right."

DECISION

HILADO, J p:
This is an appeal by Koppel (Philippines),Inc.,from the judgment of the Court of First Instance of Manila in civil
case No. 51218 of said court dismissing said corporation's complaint for the recovery of the sum of P64,122.51 which it
had paid under protest to the Collector of Internal Revenue on October 30, 1936, as merchant sales tax. The main facts
of the case were stipulated in the court below as follows:
"AGREED STATEMENT OF FACTS
"Now come the plaintiff by attorney Eulogio P. Revilla and the defendant by the Solicitor General
and undersigned Assistant Attorney of the Bureau of Justice and, with leave of this Honorable Court,
hereby respectfully stipulated and agree to the following facts, to wit:
"I. That plaintiff is a corporation duly organized and existing under and by virtue of the laws of the
Philippines, with principal office therein at the City of Manila, the capital stock of the which is divided into
one thousand (1,000) shares of P100 each. The Koppel Industrial Car and Equipment Company, a
corporation organized and existing under the laws of the State of Pennsylvania, United States of
America, and not licensed to do business in the Philippines, owned nine hundred and ninety-five (995)
shares of the total capital stock of the plaintiff from the year 1928 up to and including the year 1936, and
the remaining five (5) shares only were and are owned one each by officers of the plaintiff corporation.
"II. That plaintiff, at all times material to this case, was and now is duly licensed to engage in
business as a merchant and commercial broker in the Philippines; and was and is the holder of the
corresponding merchant's and commercial broker's privilege tax receipts.
"III. That the defendant Collector of Internal Revenue is now Mr. Bibiano L. Meer in lieu of Mr.
Alfredo L. Yatco.
"IV. That during the period from January 1, 1929, up to and including December 31, 1932,
plaintiff transacted business in the Philippines in the following manner, with the exception of the
transactions which are described in paragraphs V and VI of this stipulation:
"'When a local buyer was interested in the purchase of railway materials, machinery, and
supplies, it asked for price quotations from plaintiff. A typical form of such request is attached hereto and
made a part hereof as Exhibit A. (Exhibit A represents typical transactions arising from written for
quotations, while Exhibit B to G, inclusive, are typical transactions arising from verbal requests for
quotation.) Plaintiff then cabled for the quotation desired from Koppel Industrial Car and Equipment
Company. A small of the pertinent cable is hereto attached and made a part hereof as Exhibit B. Koppel
Industrial Car and Equipment Company answered by cable quoting its cost price, usually A. C. I. F.
manila cost price, which was latter followed by a letter of confirmation. A sample of the said cable
quotation and of the letter of confirmation are hereto attached and made a part hereof as Exhibits C and
C-1. Plaintiff, however, quoted to the purchaser a selling price above the figures quoted by Koppel
Industrial Car and Equipment Company and made a part hereof as Exhibit D. On the basis of these
quotations, orders were placed by the local purchasers, copies of which orders are hereto attached as
Exhibits E and E-1.
"'A cable then sent to Koppel Industrial Car and Equipment Company giving instructions to ship
the merchandise to Manila forwarding the customer's order. Sample of said cable is hereto attached as
Exhibit F. The bills of lading were usually made to 'order' and indorsed in blank with notation to the effect
that the buyer be notified of the shipment of the goods covered in the bills of lading; commercial invoices
were issued by Koppel Industrial Car and Equipment Company in the names of the purchasers and
certificates of insurance were likewise issued in their names, or in the name of Koppel Industrial Car and
Equipment Company but indorsed in blank and attached to drafts drawn by Koppel Industrial Car and
Equipment Company on the purchasers, which were forwarded through foreign blanks to local. Samples
of the bills of lading are hereto attached as Exhibits F-1, I-1, I-2 and I-3. Bills of lading, Exhibits I-1, I-2
and I-3, may equally have been employed, but said Exhibits I-1, I-2 and I-3 have no connection with the
transaction covered by Exhibits B to G, inclusive. The purchasers secured the shipping papers by
arrangement with the banks, and thereupon received and cleared the shipments. If the merchandise were
of European origin, and if there was not sufficient time to forward the documents necessary for clearance,
through foreign banks to local banks, to the purchasers, the Koppel Industrial Car and Equipment
Company did, in many cases, send the documents directly from Europe to plaintiff with instructions to
turn these documents over to the purchasers. In many cases where sale was effected on the basis of C.
I. F. Manila, duty paid, plaintiff advanced the sums required for the payment of the duty, and these sums,
so advanced, were in every case reimbursed to plaintiff by Koppel Industrial Car and Equipment
Company. The price were payable by drafts agreed upon in each case and drawn by Koppel Industrial
Car and Equipment Company on the respective purchasers through local banks, and payments were
made to the banks by the purchasers on presentation and delivery to them of the above-mentioned
shipping documents or copies thereof. a sample of said drafts is hereto attached as Exhibit G. Plaintiff
received by way of compensation a percentage of the profits realized on the above transactions as fixed
in paragraph 6 of the plaintiff's contract with Koppel Industrial Car and Equipment Company, which
contract is hereto attached as Exhibit H, and suffered its corresponding share in the losses resulting from
some of the transactions.
"'That the total gross sales from January 1, 1929, up to and including December 31, 1932,
effected in the foregoing manner and under the above specified conditions, amount to P3,596,438.84.'
"V. That when a local sugar central was interested in the purchase of railway materials,
machinery and supplies, it secured quotations from, and placed the corresponding orders with, the
plaintiff in substantially the same manner as outlined in paragraph IV of this stipulation, with the only
difference that the purchase orders which were agreed to by the central and the plaintiff are similar to the
sample hereto attached and made a part hereof as Exhibit I. Typical samples of the bills of lading
covering the herein transaction are hereto attached and made a part hereto as Exhibits I-1, I-2 and I-3.
The value of the sales carried out in the manner mentioned in this paragraph is P133,964.98.
"VI. That sometime in February, 1929, Miguel J. Ossorio, of Manila, Philippines, placed an option
with Koppel Industrial Car and Equipment Company, through plaintiff, to purchase within three months a
pair of Atlas-Diesel Marine Engines. Koppel Industrial Car and Equipment Company purchased said
Diesel engines in Stockholm, Sweden, for $16,508.32. The supplies drew a draft for the amount of
$16,508.32 on the Koppel Industrial Car Equipment Company, which paid the amount covered by the
draft. Later, Miguel J. Ossorio definitely called the deal off, and as Koppel Industrial Car and Equipment
Company could not ship to or draw on said Mr. Miguel J. Ossorio, it in turn drew another draft on plaintiff
for the same amount at six months, with the understanding that Koppel Industrial Car Equipment
Company would reimburse plaintiff when said engines were disposed of. Plaintiff honored the draft and
debited the said sum of $16,508.32 to merchandise account. the engines were left stored at Stockholm,
Sweden. On April 1, 1930, a new local buyer, Mr. Cesar Barrios, of for $21.000 (P42,000) C. I. F.
Hongkong. The engines were shipped to Hongkong and a draft for $21,000 was drawn by Koppel
Industrial Car and Equipment Company on Mr. Cesar Barrios. After the draft was fully by Mr. Barrios,
Koppel Industrial Car and Equipment Company reimbursed plaintiff with cost price of $16,508.32 and
credited it with $1,152.95 as its share of the profit on the transaction. Exhibits J and J-1 are herewith
attached and made integral parts of this stipulation with particular reference to paragraph VI hereof.
"VIII. That plaintiff's share in the profits realized out of these transaction ascribed in paragraphs
IV, V and VI hereof totaling P3,772,403.82, amounts to P132,201.30; and that plaintiff within the time
provided by law returned the aforesaid amount of P132,201.30 for the purpose of the commercial
broker's 4 per cent tax and paid thereon the sum of P5,288.05 as such tax.
"VIII. That defendant demanded of the plaintiff the sum of P664,122.51 as the merchants' sales
tax of 1 1/2 per cent on the amount of P3,772,403.82, representing the total gross value of the sales
mentioned in paragraphs IV, V and VI hereof, including the 25 per cent surcharge for the late payment of
the said tax, which tax and surcharge were determined after the amount of P5,288.05 mentioned in
paragraph VI hereof was deducted.
"IX. That plaintiff, on October 30,1936, paid under protest said sum of P64,122.51 in order to
avoid further penalties, levy and distraint proceedings.
"X. That defendant, on November 10, 1936, overruled plaintiff's protest, and defendant has failed
and refused and still fails and refuses, notwithstanding demands by plaintiff, to return to the plaintiff said
sun of p64,122.51 or any part thereof.
xxx xxx xxx
"That the penalties hereby reserve the right to present additional evidence in support of their
respective contentions.
"Manila, Philippines, December 26, 1939.

(Sgd.) "ROMAN OZAETA


"Solicitor General

(Sgd.) "ANTONIO CAÑIZARES


"Assistant Attorney

(Sgd.) "E. P. REVILLA


"Attorney for the Plaintiff
"3rd Floor Perez Samanillo Bldg.,Manila"
Both parties adduced some oral evidence in clarification of or additional to their agreed statement of facts. A
preponderance of evidence has established, besides the facts thus stipulated, the following:
(a) The shares of stock of plaintiff corporation were and are all owned by Koppel Industrial Car and Equipment
Company of Pennsylvania, U. S. A.,except five which were necessary to qualify the Board of Directors of said plaintiff
corporation;
(b) In the transactions involved herein plaintiff corporation acted as the representative of Koppel Industrial Car
and Equipment Company only, and not as the agent of both the latter company and the respective local purchasers —
plaintiff's principal witness, A. H. Bishop, its resident Vice-President, in his testimony invariably referred to Koppel
Industrial Car and Equipment Co. as "our principal" (t. s. n.,pp. 10, 11, 12, 19, 75),except that at the bottom of page 10
to the top of page 11, the witness stated that they had "several principals" abroad but that "our principal abroad was, for
the years in question, Koppel Industrial Car and Equipment Company," and on page 68, he testified that what he
actually said was "...but our principal principal abroad" and not "our principal abroad" — as to which it is very significant
that neither witness nor any other gave the name of even a single other principal abroad of the plaintiff corporation;
(c) The plaintiff corporation bore alone incidental expenses as, for instance, cable expenses — not only those
of its own cables but also those of its "principal" (t.s.n.,pp. 52, 53);
(d) The plaintiff's "share in the profits" realized from the transactions in which it intervened was left virtually in
the hands of Koppel Industrial Car and Equipment Company (t.s.n.,P. 51);
(e) Where drafts were not paid by the purchasers, the local banks were instructed not to protest them but to
refer them to plaintiff which was fully empowered by Koppel Industrial Car and Equipment Company to instruct the
banks with regards to disposition of the drafts and documents (t.s.n.,p. 50; Exhibit G);
(f) Where the goods were of European origin consular invoices, bill of lading, and, in general, the documents
necessary for clearance were sent directly to plaintiff (t.s.n.,p. 14);
(g) If plaintiff had in stock the merchandise desired by local buyers, it immediately filled the orders of such local
buyers and made delivery in the Philippines without the necessity of cabling its principal in America either for price
quotations or confirmation of rejection of that agreed upon between it and the buyer (t.s.n.,pp. 39-43);
(h) Whenever the deliveries made by Koppel Industrial Car and Equipment Company were incomplete or
insufficient to fill the local buyers' orders, plaintiff used to make good the deficiencies by deliveries from its own local
stock, but in such cases it charged its principal only the actual costs of the merchandise thus delivered by it from its
stock and in such transactions plaintiff did not realize any profit (t.s.n.,pp. 53-54);
(i) The contracts of sale involved herein were all perfected in the Philippines.
Those described in paragraph IV of the agreed statement of facts went through the following process: (1)
"When a local buyer was interested in the purchase of railway materials, machinery, and supplies, it asked for price
quotations from plaintiff";(2) "Plaintiff then cabled for the quotation desired from Koppel Industrial Car and Equipment
Company";(3) "Plaintiff, however, quoted to the purchaser a selling price above the figures quoted by Koppel Industrial
Car and Equipment Company";(4) "On the basis of these quotations, orders were placed by the local purchasers ...."
Those described in paragraph V of said agreed statement of facts were translated "in substantially the same
manner as outlined in paragraph IV."
As to the single transaction described in paragraph VI of the same agreed statement of facts, discarding the
Ossorio option which anyway was called off, "On April 1, 1930, a new local buyer, Mr. Cesar Barrios, of
Iloilo,Philippines,was found and the same engines were sold to him for $21,000 (P42,000) C. I. F. Hongkong."
(Emphasis supplied.)
(j) Exhibit H contains the following paragraph:
"It is clearly understood that the intent of this contract is that the broker shall perform only the
functions of a broker a set forth above, and shall not take possession on any of the materials or
equipment applying to said orders or perform any acts or duties outside the scope of a broker; and in no
sense shall this contract be construed as granting to the broker the power to represent the principal as its
agent or to make commitments on its behalf."
The Court of First Instance held for the defendant and dismissed plaintiff's complaint with costs to it.
Upon this appeal, seven errors are signed to said judgment as follows:
"1. That the court a quo erred in not holding that appellant is a domestic corporation distinct and
separate from, and not a mere branch of Koppel Industrial Car and Equipment Co.;
"2. The court a quo erred in ignoring the ruling of the Secretary of Finance, dated January 31,
1931, Exhibit M;
"3. The court a quo erred in not holding that the character of a broker is determined by the nature
of the transaction and not by the basis or measure of his compensation;
"4. The court a quo erred in not holding that appellant acted as a commercial broker in the
transactions covered under paragraph IV of the agreed statement of facts;
"5. The court a quo erred in not holding that appellant acted as a commercial broker in the
transactions covered under paragraph V of the agreed statement of facts;
"6. The court a quo erred in not holding that appellant acted as a commercial broker in the sole
transaction covered under paragraph VI of the agreed statement of facts;
"7. The court a quo erred in dismissing appellant's complaint."
The lower court found and held that Koppel; (Philippines),Inc. is a mere dummy or branch ("hechura") of Koppel
Industrial Car and Equipment Company. The lower court did not deny legal personality to Koppel (Philippines),Inc. for
any and all purposes, but in effect its conclusion was that, in the transactions involved herein, the public interest and
convenience would be defeated and what would amount to a tax evasion perpetrated, unless resort is had to the
doctrine of "disregard of the corporate fiction."
I. In its first assignment of error appellant submits that the trial court erred in not holding that it is a domestic
corporation distinct and separate from and not a mere branch of Koppel Industrial Car and Equipment Company. It
contends that its corporate existence as a Philippine corporation can not be collaterally attacked and that the
Government is estopped from so doing. As stated above, the lower court did not deny legal personality to appellant for
any and all purposes, but held in effect that in the transactions involved in this case the public interest and convenience
would be defeated and what would amount to tax evasion perpetrated, unless resort is had to the doctrine of "disregard
of the corporate fiction." In other words, in looking through the corporate form to the ultimate person or corporation
behind that form, in the particular transactions which were involved in the case submitted to its determination and
judgment, the court did so in order to prevent the contravention of the local internal revenue laws, and the perpetration
of what would to a play evasion, inasmuch as it considered — and in our opinion, correctly — that appellant Koppel
(Philippines) Inc. u as a mere branch or agency or dummy ("hechura") of Koppel Industrial Car and Equipment Co. The
court did not hold that the corporate personality of Koppel (Philippines),Inc.,would also be disregarded in other cases or
for other purposes. It would have had no power to so hold. The courts' action in this regard must be confined to the
transactions involved in the case at bar "for the purpose of adjudging the rights and liabilities of the parties in the case.
They have no jurisdiction to do more." ( 1 Fletchel, Cyclopedia of Corporation, Permanent ed.,p. 134, section 41.)
A leading and much cited case puts it as follows:
"If any general rule can be laid down, in the present state of authority, it is that a corporation will
be looked upon as a legal entity as a general rule, and until sufficient reason to the contrary appears, but,
when the notion of legal entity is used to defeat public convenience, justify wrong, protect fraud, or
defend crime, the law will regard the corporation as an association of persons." (1 Fletcher Cyclopedia of
Corporation [Permanent Edition],pp. 135, 136; United States vs.Miwaukee Refrigeration Transit Co.,142
Fed.,247, 255, per Sanborn, J.)
In his second special defense appellee alleges "that the plaintiff as and is in fact a branch or subsidiary of
Koppel Industrial Car and Equipment Co.,a Pennsylvania corporation not licensed to do business in the Philippines but
actually doing business here through the plaintiff; that the said foreign corporation holds 995 of the 1,000 shares of the
plaintiff's capital stock, the remaining five shares being held by the officers of the plaintiff herein in order to permit the
incorporation thereof and to enable its aforesaid officers to act as directors of the plaintiff corporation; and that plaintiff
was organized as a Philippine corporation for the purpose of evading the payment by its parent foreign corporation of
merchants' sales tax on the transactions involved in this case and others of similar nature."
"By most courts the entity is normally regarded but is disregarded to prevent injustice, or the
distortion or hiding of the truth, or to let in a defense." (1 Fletcher, Cyclopedia of Corporation, Permanent
Edition, pp. 139 140; emphasis supplied.)
"Another rule is that, when the corporation is the mere alter ego, or business conduit of a person,
it may be disregarded." (1 Fletcher, Cyclopedia of Corporation, Permanent Edition, p. 136.)
Manifestly, the principle is the same whether the "person" be natural or artificial.
"A very numerous and growing class of cases wherein the corporate entity is disregarded is that
wherein (it is so organized and controlled, and its affairs are so conducted, as to make it merely an
instrumentality, agency, conduit or adjunct of another corporation)." (1 Fletcher, Cyclopedia of
Corporation, Permanent ed.,pp. 154, 155.)
"While we recognize the legal principle that a corporation does not lose its entity by the
ownership of the bulk or even the whole of its stock, by another corporation (Monongahela Co.
vs.Pittsburg Co.,196 Pa.,25; 46 Atl.,99; 79 Am. St. Rep.,685) yet it is equally well settled courts will look
beyond the mere artificial personality which incorporation confers, and if necessary to work out equitable
ends, will ignore corporate forms." (Colonial Trust Co. vs.Montello Brick Works, 172 Fed.,310.)
"Where it appears that two business enterprises are owned, conducted and controlled by the
same parties, both law and equity will, when necessary to protect the rights of third persons, disregard
the legal fiction that two corporations are distinct entities, and treat them as identical.';(Abney vs.Belmonb
Country Club Properties, Inc.,279 Pac.,829.)
"...the legal fiction of distinct corporate existence will be disregarded in a case where a
corporation is so organized and controlled and its affairs are so conducted, as to make it merely an
instrumentality or adjunct of another corporation." (Hanter vs.Baker Motor Vehicle Co.,190 Fed.,665.)
In United States vs.Lehigh Valley R. Co. (220 U. S.,257; 55 Law. ed.,458, 464),the Supreme Court of the
United States disregarded the artificial personality of the subsidiary coal company in order to avoid that the parent
corporation, the Lehigh Valley R. Co.,should be able, through the fiction of that personality, to evade the prohibition of
the Hepburn Act against the transportation by railroad companies of the articles and commodities described therein.
Chief Justice White, speaking for the court, said:
" ...Coming to discharge this duty is follows, in view of the express prohibitions of the
commodities clause, it must be held that while the right of a railroad company as a stockholder to use its
stock ownership for the purpose of a bona fide separate administration of the affairs of a corporation in
which it has a stock interest may not be denied, the use of such stock ownership in substance for the
purpose of destroying the entity of a producing, etc.,corporation, and of commingling its affairs in
administration with the affairs of the railroad company, so as to make the two corporations virtually one,
brings the railroad company so voluntarily acting as to such producing, etc.,corporation within the
prohibitions of the commodities clause. In other words, that by operation and effect of the commodities
clause there is a duty cast upon a railroad company proposing to carry in interstate the product of a
producing, etc.,corporation in which is has a stock interest, not to clause such power so as virtually to do
by indirection that which the commodities clause prohibits, — a duty which plainly would be violated by
the unnecessary commingling of the affairs of the producing company with its own, so as to cause them
to be one and inseparable."
Corroborative authorities can be cited in support of the same proposition, which we deem unnecessary to
mention here.
From the facts hereinabove stated, as established by a preponderance of the evidence, particularly those
narrated in paragraphs (a),(b),(c),(d),(e),(f),(h),(i),and (j) after the agreed statement of facts, we find that, in so far as the
sales involved herein are concerned, Koppel (Philippines),Inc.,and Koppel Industrial Car and Equipment Company are
to all intents and purposes one and the same; or, to use another mode of expression, that, as regards those transaction
s, the former corporation is a mere branch, subsidiary or agency of the latter. To our mind this is conclusively borne out
by the fact, among others, that the amount of the so-called "share in the profits of Koppel (Philippines) Inc.,was
ultimately left to the sole, underlined control of Koppel Industrial Car and Equipment Company. If, in their relations with
each other, Koppel (Philippines),Inc.,was considered and intended to function as a bona fide separate corporation, we
can not conceive how this arrangement could have been adopted, for if there was any factor in its business as to which
it would in that case naturally have been opposed to being thus controlled, it must have been precisely the amount of
profit which it could endeavor and hope to earn. No group of businessmen could be expected to organize a mercantile
corporation — the ultimate end of which could only be profit — if the amount of the profit were to be subjected to such a
unilateral control of another corporation, unless indeed the former has previously been designed by the incorporates to
serve as a mere subsidiary, branch or agency of the latter. Evidently, Koppel Industrial Car and Equipment Company
made use of its ownership of the overwhelming majority — 99.5% — of the capital stock of the local corporation to
control the operations of the latter to such an extent that it had the final say even as to how much should be allotted to
said local entity in the so-called sharing in the profits. We can not overlook the fact t at in the practical working of
corporate organizations of the class to which these two entities belong the holder or holders of the controlling part of the
capital stock of the corporation, particularly where the control is determined by the virtual ownership of the totality of the
shares, dominate not only the selection of the Board of Directors but, more often than not, also the action of that board.
Applying this to the instant case, we can not conceive how the Philippine corporation could effectively go against the
policies, decisions, and desires of the American corporation with regard to the scheme which was devised through the
instrumentality of the contract Exhibit H, as well as all the other details of the system which was adopted in order to
avoid paying the 1 1/2 per cent merchants' sales tax. Neither can we conceive how the Philippine corporation could
avoid following the directions of the American corporation in every other transaction where they had both to intervene, in
view of the fact that the American corporation held 99.5 per cent of the capital stock of the Philippine corporation. In the
present instance, we note that Koppel (Philippines),Inc.,was represented in the Philippines by its "resident Vice-
President." This fact necessarily leads to the inference that the corporation had at least a Vice-President, and
presumably also a President, who were not resident in the Philippines but in America, where the parent corporation is
domiciled. If Koppel (Philippines),Inc.,had been intended to operate as a regular domestic corporation in the Philippines,
where it was formed, the record and the evidence do not disclose any reason why all its officers should not reside and
perform their functions in the Philippines.
Other facts appearing from the evidence, and presently to be stated, strengthen our conclusion, because they
can only be explained if the local entity is considered as a mere subsidiary, branch or agency of the parent organization.
Plaintiff charged the parent corporation no more than actual cost — without profit whatsoever — for merchandise
allegedly of its own to complete deficiencies of shipments made by said parent corporation (t. s. n.,pp. 53, 54) — a fact
which could not conceivably have been the case if plaintiff had acted in such transactions as an entirely independent
entity doing business — for profit, of course — with the American concern. There has been no attempt even to explain,
if the latter situation really obtained, why these two corporations should have thus departed from the ordinary course of
business. Plaintiff was charged by the American corporation with the cost even of the latter's cable quotations — from
ought that appears from the evidence, this can only be comprehended by considering plaintiff as such a subsidiary,
branch or agency of the parent entity in which case it would be perfectly understandable that for convenient accounting
purposes and the easy determination of the profits or losses of the parent corporation's Philippine business, all
expenses of its business in the Philippines should be charged against the Philippine office and set off against its
receipts, thus, separating the accounts of said branch from those which the central organization might have, for
instance, in Sweden, and those which it might have in other countries. The reference to plaintiff by local banks, under a
standing instruction of the parent corporation, of unpaid drafts drawn on Philippine customers by said parent
corporation, whenever said customers dishonored the drafts, and the fact that the American corporation had previously
advised said banks that plaintiff in those cases was "fully empowered to instruct (the banks) with regard to the
disposition of the drafts and documents" (t.s.n.,p. 50),in the absence of any other satisfactory explanation naturally give
rise to the inference that plaintiff was a subsidiary, branch or agency of the American concern, rather than an
independent corporation acting as a broker. For, without such positive explanation, this delegation of power is indicative
of the relations between central and branch offices of the same business enterprise, with the latter acting under
instructions already given by the former. Far from disclosing a real separation between the two entities, particularly in
regard to the transactions in question, the evidence reveals such a coming and interlacing of their activities as to render
even incomprehensible certain accounting operations between them, except upon the basis that the Philippine
corporation was to all intents and purposes a mere subsidiary, branch, or agency of the American parent entity. Only
upon this basis can it be comprehended why it seems not to matter at all how much profit would be allocated to plaintiff,
or even that no profit at all be so allocated to it, at any given time or after any given period.
As already stated above, under the evidence the sales in the Philippines of the railway materials, machinery
and supplies imported here by Koppel Industrial Car and Equipment Company could have been as conveniently and
efficiently transacted and handled — if not more so — had said corporation merely established a branch or agency in
the Philippines and obtained license to do business locally; and if it had done so and said sales had been effected by
such branch or agency, there seems to be no dispute that the 1 1/2 per cent merchants' sales tax then in force would
have been collectible. So far as we can discover, there would be only one, but very important, difference between the
two schemes — a difference in tax liability amounting to the respectable sum of P64, 122.51 in this case. To allow the
taxpayer now to deny this tax liability on the ground that the sales were made through another and distinct corporation,
as alleged broker, when we have seen that this latter corporation is virtually owned by the former, or that they are
practically one and the same, is to sanction a circumvention of our laws, and permit a tax evasion of no mean
proportions and the consequent commission of a grave injustice to the Government. Not only this; it would allow the
taxpayer to do by indirection what the tax laws prohibited to be done directly (nonpayment of legitimate
taxes),paraphrasing the United States Supreme Court in United States vs.Lehigh Valley R. Co.,supra.
The act of one corporation crediting or debiting the other for certain items, expenses or even merchandise sold
or disposed of, is perfectly compatible with the idea of the domestic entity being or acting as a mere branch, agency or
subsidiary of the parent organization. Such operations were called for any way by the exigencies or convenience of the
entire business. Indeed, accounting operations such as these are inevitable, and have to be effected in the ordinary
course of business, wherever the home office of a business enterprise extends its trade to another land through a
branch office, or through another scheme amounting to the same thing.
If plaintiff were to act as broker in the Philippines for any other corporation, entity or person, distinct from
Koppel Industrial Car and Equipment Company, an entirely different question will arise, which, however, we are not
called upon, nor in a position, to decide.
As stated above, Exhibit H contains the following paragraph:
"It is clearly understood that the intent of this contract is that the broker shall perform only the
functions of a broker as set forth above, and shall not take possession of any of the materials or
equipment applying to said orders or perform any acts or duties outside the scope of a broker; and in no
sense shall this contract be construed as granting to the broker the power to represent the principal as it
agent or to make commitments on its behalf."
The foregoing paragraph, construed in the light of other facts noted elsewhere in this decision, betrays, we
think, a deliberate intent, through the medium of a scheme devised; with great care, to avoid the payment of precisely
the 1 1/2 per cent merchants' sales tax in force in the Philippines before, at the time of, and after, the making of the said
contract Exhibit H. If this were to be allowed, the payment of a tax, which directly could not have been avoided, could be
evaded by indirection,consideration being had of the aforementioned peculiar relations between the said American and
local corporations. Such evasion, involving as it would, a violation of the former Internal Revenue Law, would even fall
within the penal sanction of section 2741 of the Revised Administrative Code. which only goes to show the illegality of
the whole scheme. We are not here concerned with the impossibility. We are not here concerned with the impossibility
of collecting the merchants' sales tax, as a mere incidental consequence of transactions legal in themselves and
innocent in their purpose. We are dealing with a scheme the primary, not to say the sole, object of which is the evasion
of the payment of such tax. If is this aim of the scheme that makes it illegal.
We have said above that the contracts of the sale involved herein were all perfected in the Philippines. From
the facts stipulated in paragraph IV of the agreed statement of facts, it clearly appears that the Philippine purchasers
had to wait for Koppel Industrial Car and Equipment Company to communicate its cost prices to Koppel
(Philippines),Inc.,and for the latter to make the definite price quotations, before placing their orders, whenever such
price quotations from the American corporation were required. It is obvious that in those cases the contracts involved in
the orders thus placed by the said purchasers with Koppel (Philippines),Inc.,were perfected in the Philippines. In those
cases where no such price quotations from the American corporation were needed, of course, the sales were
immediately perfected locally. The sales effected in those cases described in paragraph V of the agreed statement of
facts were, as expressed therein, transacted "in substantially the same manner as outlined in paragraph IV." Even the
single transaction described in paragraph VI of the agreed statement of facts was also perfected in the Philippines,
because the contracting parties were here and the consent of each was given here. While it is true that when the
contract was thus perfected in the Philippines the pair of Atlas-Diesel Marine Engines were in Sweden and the
agreement was to deliver them C.I.F. Hongkong, the contract of sale being consensual — perfected by mere consent —
(Civil Code, article 1445; 10 Manresa, 4th ed.,p. 11),the location of the property and the place of delivery did not matter
in the question of where the agreement was perfected.
In said paragraph VI, we read the following, as indicating where the contract was perfected, considering
beforehand that one party, Koppel (Philippines) Inc.,which in contemplation of law, as to that transaction, was the same
Koppel Industrial Car Equipment Co.,was in the Philippines
" ...on April 1, 1930, a new local buyer, Mr. Cesar Barrios, of Iloilo, Philippines, was found and
the same engines were sold to him for $21,000 (42,000) C.I.F. Hongkong ..."(Emphasis supplied.)
Under the revenue law in force when the sales in question took place, the merchants' sales tax attached upon
the happening of the respective sales of the "commodities, goods, wares, and merchandise" involved, and we are
clearly of opinion that such "sales" took place upon the perfection of the corresponding contracts. If such perfection took
place in the Philippines, the merchants' sales tax then in force here attached to the transactions.
Even if we should consider that the Philippine buyers in the cases covered by paragraphs VI and V of the
agreed statement of facts, contracted with Koppel Industrial Car and Equipment Company, we will arrive at the same
final result. It can not be denied in that case that said American Corporation contracted through Koppel
(Philippines),Inc.,which was in the Philippines. The real transaction in each case of sale, in final effect, began with an
offer of sale from the seller, said American Corporation, through its agent, the local corporation, of the railway materials,
machinery, and supplies at the prices quoted, and perfected or completed by the acceptance of that offer by the local
buyers when latter, accepting those prices, placed their orders. The offer could not correctly be said to have been made
by the local buyers when they asked to have bound themselves to buy before knowing the prices. And even if we
should take into consideration the fact that the American corporation contracted, at least partly, through
correspondence, according to article 54 of the Code of Commerce, the respective contracts were completed from the
time of the acceptance by the local buyers, which happened in the Philippines.
"Contracts executed through correspondence shall be completed from the time an answer is
made accepting the proposition or the conditions by which the latter may be modified." (Code of
Commerce, article 54; emphasis supplied.)
"A contract is as a rule considered as entered into at the place where the offer is accepted, or
where the last act necessary to complete it is performed. So where delivery is regarded as essential to
the completion of the contract it is regarded as made at the place of delivery." (13 C. J.,580-81.,section
581.)
"(In the consensual contract of sale delivery is not needed for its perfection.)"
II. Appellant's second assignment of error can be summarily disposed of. It is clear that the ruling of the
Secretary of Finance, Exhibit M, was not binding upon the trial court, much less upon this tribunal the duty and power of
interpreting the laws is primarily a function of the judiciary. (Ortua vs.Singson Encarnacion, 59 Phil.,440, 444.) Plaintiff
cannot be excused from abiding by this legal principle, nor it properly be heard to say that it relied on the Secretary's
ruling and that, therefore, the courts should not now apply an interpretation at variance therewith. The rule of stare
decisis is undoubtedly entitled to more respect in the construction of statutes than the interpretations given by officers of
the administrative branches of the government, even those entrusted with the administration of particular laws. But this
court, in Philippine Trust Company and Smith, Bell & Co. vs.Mitchell (59 Phil.,30, 36),said:
" ...The rule of stare decisis is entitled to respect. Stability in the law, particularly in the business
field, is desirable. But idolatrous reverence for precedent, simply as precedent, no longer rules. More
Important than anything else is that the court should be right. ..."
III. In the view we take of the case, and after the disposition made above of the first assignment of error, it
becomes unnecessary to make any specific ruling on the third, fourth, fifth, sixth, and seventh assignments of error, all
of which are necessarily disposed of adversely to appellant's contention.
Wherefore, the judgment appealed from is affirmed, with costs of both instances against appellant. So ordered.
||| (Koppel (Phils.), Inc. v. Yatco, G.R. No. L-47673, [October 10, 1946], 77 PHIL 496-517)
[G.R. No. L-9687. June 30, 1961.]

LIDDELL & CO., INC., petitioner-appellant, vs. THE COLLECTOR OF INTERNAL REVENUE,
respondent-appellee.

Ozaeta, Lichauco & Picazo for petitioner-appellant.


Solicitor General for respondent-appellee.

SYLLABUS

1. JUDGES; DISQUALIFICATION; PARTICIPATION IN PRIOR PROCEEDINGS AS ADMINISTRATIVE


OFFICIAL. — The mere participation of a judge in prior proceeding relating to the subject in the capacity of an
administrative official does not disqualify him from acting as judge.
2. COURT OF TAX APPEALS; DECISION SIGNED AFTER 30 DAYS FROM SUBMISSION OF CASE, VALID.
— The requirement that cases brought before the Tax Court shall be decided within 30 days after the submission
thereof for decision is merely directory. Hence, decisions signed after the lapse of said period are valid.
3. CORPORATION LAW; WHEN CORPORATE FORM MAY BE IGNORED. — Where a corporation is a
dummy and serves no business purpose and is intended only as a blind, the corporate form may be ignored.
4. TAXATION; SALES TAX; WHEN TAXPAYER MAY NOT DENY TAX LIABILITY. — A taxpayer may not deny
tax liability on the ground that the sales were made through another and distinct corporation when it is proved that the
latter is virtually owned by the former or that they are practically one and the same corporation.
5. ID.; ID.; SURCHARGE WHEN NOT IMPOSABLE. — Where, as in the case at bar, the sales made by the
taxpayer to the corporation had been embodied in proper documents subject to inspection by the tax authorities, the
return filed on the basis of such sales and not on those to the public, cannot be said a false return and subject the
taxpayer to a surcharge. But penalty for late payment should be imposed.
6. ID.; ID.; DEFICIENCY SALES TAX, HOW COMPUTED. — Deficiency sales tax should be based on the
selling price to the public after deducting the tax paid on the original sales.

DECISION

BENGZON, C.J p:

Statement. This is an appeal from the decision of the Court of Tax Appeals imposing a tax deficiency liability of
P1,317,629.61 on Liddell & Co., Inc.
Said company lists down several issues which may be boiled to the following:
(a) Whether or not Judge Umali of the Tax Court below could validly participate in the
making of the decision;
(b) Whether or not Liddell & Co. Inc., and the Liddell Motors Inc. are (practically) identical
corporations, the latter being merely the alter ego of the former;
(c) Whether or not, granting the identical nature of the corporations, the assessment of
tax liability, including the surcharge thereon, by the Court of Tax Appeals, is
correct.
Undisputed Facts. The parties submitted a partial stipulation of facts, each reserving the right to present
additional evidence.
Said undisputed facts are substantially as follows:
The petitioner, Liddell & Co. Inc., (Liddell & Co. for short) is a domestic corporation established in
the Philippines on February 1, 1946, with an authorized capital of P100,000 divided into 1000 shares at
P100 each. Of this authorized capital, 196 shares valued at P19,600 were subscribed and paid by Frank
Liddell while the other four shares were in the name of Charles Kurz, E. J. Darras, Angel Manzano and
Julian Serrano at one share each. Its purpose was to engage in the business of importing and retailing
Oldsmobile and Chevrolet passenger cars and GMC and Chevrolet trucks.
On January 31, 1947, with the limited paid-in capital of P20,000, Liddell & Co. was able to
declare a 90% stock dividend after which declaration, Frank Liddell's holdings in the company increased
to 1,960 shares and the employees, Charles Kurz, E.J. Darras, Angel Manzano and Julian Serrano at 10
shares each. The declaration of stock dividend was followed by a resolution increasing the authorized
capital of the company to P1,000,000 which the Securities & Exchange Commission approved on March
3, 1947. Upon such approval, Frank Liddell subscribed to 3,000 additional shares, for which he paid into
the corporation P300,000 so that he had in his own name 4,960 shares.
On May 24, 1947, Frank Liddell, on one hand and Messrs. Kurz, Darras, Manzano and Serrano
on the other, executed an agreement (Exhibit A) which was further supplemented by two other
agreements (Exhibits B and C) dated May 24, 1947 and June 3, 1948, wherein Frank Liddell transferred
(On June 7, 1948) to various employees of Liddell & Co. shares of stock.
At the annual meeting of stockholders of Liddell & Co. held on March 9, 1948, a 100% stock
dividend was declared, thereby increasing the issued capital stock of said corporation to P1,000,000. The
stockholders also approved a resolution increasing the authorized capital stock from P1,000,000 to
P3,000,000 which increase was duly approved by the Securities and Exchange Commission on June 7,
1948. Frank Liddell subscribed to and paid 20% of the increase of P400,000. He paid 25% thereof in the
amount of P100,000 and the balance of P300,000 was merely debited to Frank Liddell-Drawing Account
and credited to Subscribed Capital Stock on December 31, 1948.
On March 8, 1949, stock dividends were again issued by Liddell & Co. and in accordance with
the agreements, Exhibits A, B, and C, the stocks of said company stood as follows:
Name No. of Shares Amount Percent

Frank Liddell 13,688 P1,368,800 72.00%

Irene Liddell 1 100 .01%

Mercedes Vecin 1 100 .01%

Charles Kurz 1,225 122,500 6.45%

E J. Darras 1,225 122,500 6.45%

Angel Manzano 1,150 115,000 6.06%

Julian Serrano 710 71,000


3.74%

E. Hasim 500 50,000 2.64%

G. W. Kernot 500 50,000 2.64%

——— ————— ————


19,000 P1,900,000 100.00%

===== ========= =======

On November 15, 1948, in accordance with a resolution of a special meeting of the Board of
Directors of Liddell & Co. stock dividends were again declared. As a result or said declaration and in
accordance with the agreements, Exhibits, A, B, and C, the stockholdings in the company appeared to
be:
Name No. of Shares Amount Percent

Frank Liddell 19,738 P1,973,800 65.791%

Irene Liddell 1 100 .003%

Mercedes Vecin 1 100 .003%

Charles Kurz 2,215 221,500 7.381%

E.J. Darras 2,215 221,500 7.381%

Angel Manzano 1,810 181,000 6.031%

Julian Serrano 1,700 170,000 5.670%

E. Hasim 830 83,000


2.770%

Kernot 1,490 149,000 4.970%

—— ——— ———

30,000 P3,000,000 100.00%

===== ======== ======

On the basis of the agreement Exhibit A, (May, 1947) 55% of the earnings available for dividends accrued to
Frank Liddell although at the time of the execution of said instrument, Frank Liddell owned all of the shares in said
corporation. 45% accrued to the employees, parties thereto: Kurz 12-1/2%; Darras 12-1/2%; A. Manzano 12-1/2% and
Julian Serrano 7-1/2%. The agreement Exhibit A was also made retroactive to 1946. Frank Liddell reserved the right to
reapportion the 45% dividends pertaining to the employees in the futurefor the purpose of including such other faithful
and efficient employees as he may subsequently designate. (As a matter of fact, Frank Liddell did so designate, two
additional employees namely: E. Hasim and G. W. Kernot). It was for such inclusion of future faithful employees that
Exhibits B-1 and C were executed. As per Exhibit C, dated May 13, 1948, the 45% given by Frank Liddell to his
employees was reapportioned as follows: C. Kurz — 12 %; E. J. Darras — 12%; A. Manzano — 12%; J. Serrano — 3-
1/2%; G. W. Kernot — 2%.
Exhibit B contains the employees' definition in detail of the manner by which they sought to prevent their
shareholdings from being transferred to others who may be complete strangers to the business of Liddell & Co.
From 1946 until November 22, 1948 when the purpose clause of the Articles of Incorporation of Liddell & Co.
Inc., was amended so as to limit its business activities to importations of automobiles and trucks, Liddell & Co. was
engaged in business as an importer and at the same time retailer of Oldsmobile and Chevrolet passenger cars and
GMC and Chevrolet trucks.
On December 20, 1948, the Liddell Motors, Inc. was organized and registered with the Securities and
Exchange Commission with an authorized capital stock of P100,000 of which P20,000 was subscribed and paid for as
follows: Irene Liddell, wife of Frank Liddell, 19,996 shares and Messrs, Marcial P. Lichauco, E. K. Bromwell, V. E. del
Rosario and Esmenia Silva, 1 share each.
At about the end of the year 1948, Messrs. Manzano, Kurz and Kernot resigned from their respective positions
in the Retail Dept. of Liddell & Co. and they were taken in and employed by Liddell Motors, Inc., Kurz as Manager-
Treasurer, Manzano as General Sales Manager for cars and Kernot as General Sales Manager for trucks.
Beginning January, 1949, Liddell & Co. stopped retailing cars and trucks; it conveyed them instead to Liddell
Motors, Inc. which in turn sold the vehicles to the public with a steep mark-up. Since then, Liddell & Co. paid sales taxes
on the basis of its sales to Liddell Motors, Inc. considering said sales as its original sales.
Upon review of the transactions between Liddell & Co. and Liddell Motors Inc., the Collector of Internal
Revenue determined that the latter was but an alter ego of Liddell & Co. Wherefore, he concluded, that for sales tax
purposes, those sales made by Liddell Motors, Inc. to the public were considered as the original sales of Liddell & Co.
Accordingly, the Collector of Internal Revenue assessed against Liddell & Co. a sales tax deficiency, including
surcharges, in the amount of P1,317,629.61. In the computation, the gross selling price of Liddell Motors, Inc. to the
general public from January 1, 1949 to September 15, 1950, was made the basis without deducting from the selling
price, the taxes already paid by Liddell & Co. in its sales to the Liddell Motors, Inc.
The Court of Tax Appeals upheld the position taken by the Collector of Internal Revenue.
A. Judge Umali: Appellant urges the disqualification of Judge Roman M. Umali to participate in the decision of
the instant case because he was Chief of the Law Division, then Acting Deputy Collector and later Chief Counsel of the
Bureau of Internal Revenue during the time when the assessment in question was made. 1 In refusing to disqualify
himself despite admission that he had held the aforementioned offices, Judge Umali stated that he had not in any way
participated, nor expressed any definite opinion, on any question raised by the parties when this case was presented for
resolution before the said bureau. Furthermore, after careful inspection of the records of the Bureau, he (Judge Umali
as well as the other members of the Court below), had not found any indication that he had expressed any opinion or
made any decision that would tend to disqualify him from participating in the consideration of the case in the Tax Court.
At this juncture, it is well to consider that petitioner did not question the truth of Judge Umali's statements. In
view thereof, this Tribunal is not inclined to disqualify said judge. Moreover, in furtherance of the presumption of a
judge's moral sense of responsibility this Court has adopted, and now here repeats, the ruling that the mere
participation of a judge in prior proceedings relating to the subject in the capacity of an administrative official does not
necessarily disqualify him from acting as judge. 2
Appellant also contends that Judge Umali signed the said decision contrary to the provision of Section 13,
Republic Act No. 1125, 3 that whereas the case was submitted for decision of the Court of Tax Appeals on July 12,
1955, and the decision of Associate Judge Luciano and Judge Nable were both signed on August 11, 1955 (that is, on
the last day of the 30-day period provided for in Section 13, Republic Act No. 1125). Judge Umali signed the decision
August 31, 1955 or 20 days after the lapse of the 30-day period allotted by law.
By analogy it may be said that inasmuch as in Republic Act No. 1125 (law creating the Court of Tax Appeals)
like the law governing the procedure in the Court of Industrial Relations, there is no provision invalidating decisions
rendered after the lapse of 30 days, the requirement of Section 13, Republic Act No. 1125 should be construed as
directory. 4
Besides as pointed out by appellee, the third paragraph of Section 13 of Republic Act No. 1125 (quoted in the
margin) 5 confirms this view, because in providing for two thirty-day periods, the law means that decisions may still be
rendered within the second period of thirty days (Judge Umali signed his decision within that period).
B. Identity of the two corporations: On the question whether or not Liddell Motors, Inc. is the alter ego of Liddell
& Co. Inc., we are fully convinced that Liddell & Co. is wholly owned by Frank Liddell. As of the time of its organization,
98% of the capital stock belonged to Frank Liddell. The 20% paid-up subscription with which the company began its
business was paid by him. The subsequent subscriptions to the capital stock were made by him and paid with his own
money.
These stipulations and conditions appear in Exhibit A: (1) that Frank Liddell had the authority to designate in the
future the employee who could receive earnings of the corporation; to apportion among the stockholders the share in
the profits; (2) that all certificates of stock in the names of the employees should be deposited with Frank Liddell duly
endorsed in blank by the employees concerned; (3) that each employee was required to sign an agreement with the
corporation to the effect that, upon his death or upon his retirement or separation for any cause whatsoever from the
corporation, the said corporation should, within a period of sixty days therefor, have the absolute and exclusive option to
purchase and acquire the whole of the stock interest of the employees so dying, resigning, retiring or separating.
These stipulations in our opinion attest to the fact that Frank Liddell owned Liddell & Co. Inc. They guarantee
his complete control over the corporation.
As to Liddell Motors Inc. we are fully persuaded that Frank Liddell also owned it. He supplied the original capital
funds. 6 It is not proven that his wife Irene, ostensibly the sole incorporator of Liddell Motors, Inc. had money of her own
to pay for her P20,000 initial subscription. 7 Her income in the United States in the years 1943 and 1944 and the
savings therefrom could not be enough to cover the amount of subscription, much less to operate an expensive trade
like the retail of motor vehicles. The alleged sale of her property in Oregon might have been true, but the money
received therefrom was never shown to have been saved or deposited so as to be still available at the time of the
organization of the Liddell Motors, Inc.
The evidence at hand also shows that Irene Liddell had scant participation in the affairs of Liddell Motors, Inc.
She could hardly be said to possess business experience. The income tax forms record no independent income of her
own. As a matter of fact, the checks that represented her salary and bonus from Liddell Motors, Inc. found their way into
the personal account of Frank Liddell. Her frequent absences from the country negate any active participation in the
affairs of the Motors company.
There are quite a series of conspicuous circumstances that militate against the separate and distinct personality
of Liddell Motors, Inc. from Liddell & Co. 8 We notice that the bulk of the business of Liddell & Co. was channelled
through Liddell Motors, Inc. On the other hand, Liddell Motors, Inc. pursued no activities except to secure cars, trucks,
and spare parts from Liddell & Co. Inc. and then sell them to the general public. These sales of vehicles by Liddell & Co.
to Liddell Motors, Inc. for the most part were shown to have taken place on the same day that Liddell Motors, Inc. sold
such vehicles to the public. We may even say that the cars and trucks merely touched the hands of Liddell Motors, Inc.
as a matter of formality.
During the first six months of 1949, Liddell & Co. issued ten (10) checks payable to Frank Liddell which were
deposited by Frank Liddell in his personal account with the Philippine National Bank. During this time also, he issued in
favor of Liddell Motors, Inc. six (6) checks drawn against his personal account with the same bank. The checks issued
by Frank Liddell to the Liddell Motors, Inc. were significantly for the most part issued on the same day when Liddell &
Co., Inc. issued the checks for Frank Liddell 9 and for the same amounts.
It is of course accepted that the mere fact that one or more corporations are owned and controlled by a single
stockholder is not of itself sufficient ground for disregarding separate corporate entities. Authorities 10 support the rule
that it is lawful to obtain a corporation charter, even with a single substantial stockholder, to engage in a specific activity,
and such activity may co-exist with other private activities of the stockholder. If the corporation is a substantial one,
conducted lawfully and without fraud on another, its separate identity is to be respected.
Accordingly, the mere fact that Liddell & Co. and Liddell Motors, Inc. are corporations owned and controlled by
Frank Liddell directly or indirectly is not by itself sufficient to justify the disregard of the separate corporate identity of
one from the other. There is however, in this instant case, a peculiar consequence of the organization and activities of
Liddell Motors, Inc.
Under the law in force at the time of its incorporation the sales tax on original sales of cars (sections 184, 185
and 186 of the National Internal Revenue Code), was progressive, i.e. 10% of the selling price of the car if it did not
exceed P5,000, and 15% of the price if more than P5,000 but not more than P7,000, etc. This progressive rate of the
sales tax naturally would tempt the taxpayer to employ a way of reducing the price of the first sale. And Liddell Motors,
Inc. was the medium created by Liddell & Co. to reduce the price and the tax liability.
Let us illustrate: a car with engine motor No. 212381 was sold by Liddell & Co., Inc. to Liddell Motors, Inc. on
January 17, 1948 for P4,546,000.00 including tax; the price of the car was P4,133,000.23, the tax paid being P413.22,
at 10%. And when this car was later sold (on the same day) by Liddell Motors, Inc. to P.V. Luistro for P5,500, no more
sales tax was paid. 11 In this price of P5,500 was included the P413.32 representing taxes paid by Liddell & Co. Inc. in
the sale to Liddell Motors, Inc. Deducting P413.32 representing taxes paid by Liddell & Co., Inc. the price of P5,500, the
balance of P5,087.68 would have been the net selling price of Liddell & Co., Inc. to the general public (had Liddell
Motors, Inc. not participated and intervened in the sale), and 15% sales tax would have been due. In this transaction,
P349.68 in the form of taxes was evaded. All the other transactions (numerous) examined in this light will inevitably
reveal that the Government coffers had been deprived of a sizeable amount of taxes.
As opined in the case of Gregory v. Helvering, 12 "the legal right of a taxpayer to decrease the amount of what
otherwise would be his taxes, or altogether avoid them, by means which the law permits, cannot be doubted." But, as
held in another case, 13 "where a corporation is a dummy, is unreal or a sham and serves no business purpose and is
intended only as a blind, the corporate form may be ignored for the law cannot countenance a form that is bald and
mischievous fiction."
Consistently with this view, the United States Supreme Court 14 held that "a taxpayer may gain advantage of
doing business thru a corporation if he pleases, but the revenue officers in proper cases, may disregard the separate
corporate entity where it serves but as a shield for tax evasion and treat the person who actually may take the benefits
of the transactions as the person accordingly taxable."
Thus we repeat: to allow a taxpayer to deny tax liability on the ground that the sales were made through
another and distinct corporation when it is proved that the latter is virtually owned by the former or that they are
practically one and the same is to sanction a circumvention of our tax laws. 15
C. Tax liability computation: In the Yutivo case: 16 the same question involving the computation of the alleged
deficiency sales tax has been raised. In accordance with our ruling in said case we hold as correctly stated by Judge
Nable in his concurring and dissenting opinion on this case, that the deficiency sales tax should be based on the selling
price obtained by Liddell Motors, Inc. to the public AFTER DEDUCTING THE TAX ALREADY PAID BY LIDDELL & CO.,
INC. in its sales to Liddell Motors, Inc.
On the imposition of the 50% surcharge by reason of fraud, we see that the transactions between Liddell
Motors, Inc. and Liddell & Co., Inc. have always been embodied in proper documents, constantly subject to inspection
by the tax authorities. Liddell & Co., Inc. have always made a full report of its income and receipts in its income tax
returns.
Paraphrasing our decision in the Yutivo case, we may now say, in filing its return on the basis of its sales to
Liddell Motors, Inc. and not on those by the latter to the public, it cannot be held that the Liddell & Co., deliberately
made a false return for the purpose of defrauding the government of its revenue, and should suffer a 50% surcharge.
But penalty for late payment (25%) should be imposed.
In view of the foregoing, the decision appealed from is hereby modified: Liddell & Co., Inc. is declared liable
only for the amount of P426,811.67 with 25% surcharge for late payment and 6% interest thereon from the time the
judgment becomes final.
As it appears, that, during the pendency of this litigation, appellant paid under protest to the Government the
total amount assessed by the Collector, the latter is hereby required to return the excess to the petitioner. No costs.
||| (Liddel & Co., Inc. v. Collector of Internal Revenue, G.R. No. L-9687, [June 30, 1961], 112 PHIL 524-536)

[G.R. No. L-5677. May 25, 1953.]

LA CAMPANA COFFEE FACTORY, INC., and TAN TONG, doing business under the trade name
"LA CAMPANA GAUGAU PACKING", petitioners, vs. KAISAHAN NG MGA MANGGAGAWA SA LA
CAMPANA (KKM) and THE COURT OF INDUSTRIAL RELATIONS, respondents.

Ceferino de los Santos, R., Ceferino de los Santos, Jr. and Manuel V. Roxas for petitioners.
Carlos E. Santiago for respondent union.
SYLLABUS

1. INDUSTRIAL DISPUTES; TWO FACTORIES OPERATING UNDER ONE MANAGEMENT; EFFECT OF


ONE OF THEM BEING A REGISTERED CORPORATION. — C Coffee Factory, Inc., and C Gaugau Packing are
operating as one business though with two trade names. The owner of the latter is T; and the former, though an
incorporated business, is in reality owned exclusively by T and his family. The two factories had but one office, one
management and one payroll until July 17, the day the case was certified to the Court of Industrial Relations, when the
person who was discharging the office of cashier for both branches of the business began preparing separate payrolls
for the two. And the laborers of the gaugau factory and the coffee factory were interchangeable, that is, the laborers
from the gaugau factory were sometimes transferred to the coffee factory and vice-versa. Held: The attempt to make
the two factories appear as two separate businesses, when in reality they are but one, is but a device to defeat the ends
of the law (the Act governing capital and labor relations) and should not be permitted to prevail. Although the coffee
factory is a corporation and, by legal fiction, an entity existing separate and apart from the persons composing it, that is,
T and his family, it is settled that this fiction of law, which has been introduced as a matter of convenience and to
subserve the ends of justice cannot be invoked to further an end subversive of that purpose. (13 Am. Jur., 160-162;
Annotation 1 A. L. R. 612, s. 34 A. L. R. 599.)
2. ID.; JURISDICTIONAL NUMBER OF LABORERS REQUIRED FOR A LABOR TO SUE IN THEIR BEHALF.
— Although the coffee factory has only 14 laborers and only five of these are members of the labor union, yet as the
gaugau factory has more than the jurisdiction number (31) required by law and the two factories are operating under
one single management, the industrial court has jurisdiction to try the case as against C Coffee Factory, Inc.
3. ID.; JURISDICTION ONCE ACQUIRED IS NOT LOST EVEN IF PERMIT OF LABOR UNION IS
SUSPENDED. — Once the Court of Industrial Relations has duly acquired jurisdiction over a case, such jurisdiction is
not lost even after the Department of Labor has suspended the permit of the petitioning labor union.

DECISION

REYES, J p:

Tan Tong, one of the herein petitioners, has since 1932 been engaged in the business of buying and selling
gaugau under the trade name La Campana Gaugau Packing with an establishment in Binondo, Manila, which was later
transferred to España Extension, Quezon City. But on July 6, 1950, Tan Tong, with himself and members of his family
as sole incorporators and stockholders, organized a family corporation known as La Campana Coffee Factory Co., Inc.,
with its principal office located in the same place as that of La Campana Gaugau Packing.
About a year before the formation of the corporation, or on July 11, 1949, Tan Tong had entered into a
collective bargaining agreement with the Philippine Legion of Organized Workers, known as PLOW for short, to which
the union of Tan Tong's employees headed by Manuel E. Sadde was then affiliated. Seceding, however, from the
PLOW, Tan Tong's employees later formed their own organization known as Kaisahan Ng Mga Manggagawa Sa La
Campana, one of the herein respondents, and applied for registration in the Department of Labor as an independent
entity. Pending consideration of this application, the Department gave the new organization legal standing by issuing it a
permit as an affiliate to the Kalipunan Ng Mga Kaisahang Manggagawa.
On July 19, 1951, the Kaisahan Ng Mga Manggagawa Sa La Campana, hereinafter to be referred to as the
respondent Kaisahan, which, as of that date, counted with 66 members — workers all of them of both La Campana
Gaugau Packing and La Campana Coffee Factory Co., Inc. — presented a demand for higher wages and more
privileges, the demand being addressed to La Campana Starch and Coffee Factory, by which name they sought to
designate, so it appears, the La Campana Gaugau Packing and the La Campana Coffee Factory Co., Inc. As the
demand was not granted and an attempt at settlement through the mediation of the Conciliation Service of the
Department of Labor had given no result, the said Department certified the dispute to the Court of Industrial Relations
on July 17, 1951, the case being there docketed as Case No. 584-V.
With the case already pending in the industrial court, the Secretary of Labor, on September 5, 1951, revoked
the Kalipunan Ng Mga Kaisahang Manggagawa's permit as a labor union on the strength of information received that it
was dominated by subversive elements, and, in consequence, on the 20th of the same month, also suspended the
permit of its affiliate, the respondent Kaisahan.
We have it from the court's order of January 15, 1952 which forms one of the annexes to the present petition,
that following the revocation of the Kaisahan's permit, "La Campana Gaugau and Coffee Factory" (obviously the
combined name of La Campana Gaugau Packing and La Campana Coffee Factory Co., Inc.) and the PLOW, which had
been allowed to intervene as a party having an interest in the dispute, filed separate motions for the dismissal of the
case on the following grounds:
"1. That the action is directed against two different entities with distinct personalities, with "La
Campana Starch Factory" and the "La Campana Coffee Factory, Inc."
"2. That the workers of the "La Campana Coffee Factory, Inc." are less than thirty-one;
"3. That the petitioning union has no legal capacity to sue, because its registration as an
organized union has been revoked by the Department of Labor on September 5, 1951, and
"4. That there is an existing valid contract between the respondent "La Campana Gaugau
Packing" and the intervenor PLOW, wherein the petitioner's members are contracting parties bound by
said contract."
Several hearings were held on the above motions, in the course of which ocular inspections were also made,
and on the basis of the evidence received and the facts observed in the ocular inspections, the Court of Industrial
Relations denied the said motions in its order of January 14, 1952, because it found as a fact that:
"A. While the coffee corporation is a family corporation with Mr. Tan Tong, his wife, and children
as the incorporators and stockholders (Exhibit 1), the La Campana Gaugau Packing is merely a business
name (Exhibit 4).
"B. According to the contract of lease (Exhibit 23), Mr. Tan Tong, proprietor and manager of the
La Campana Gaugau Factory, leased a space of 200 square meters in the bodega housing the gaugau
factory to his son Tan Keng Lim, manager of the La Campana Coffee Factory. But the lease was
executed only on September 1, 1951, while the dispute between the parties was pending before the
Court.
"C. There is only one entity La Campana Starch and Coffee Factory, as shown by the signboard
(Exhibit 1), the advertisement in the delivery trucks (Exhibit I-1), the packages of gaugau (Exhibit K), and
delivery forms (Exhibits J, J-1, and J-2).
"D. All the laborers working in the gaugau or in the coffee factory receive their pay from the same
person, the cashier, Miss Natividad Garcia, secretary of Mr. Tan Tong; and they are transferred from the
gaugau to the coffee and vice-versa as the management so requires.
"E. There has been only one payroll for the entire La Campana personnel and only one person
preparing the same — Miss Natividad Garcia, secretary of Mr. Tan Tong. But after the case at bar was
certified to this Court on July 17, 1951, the company began making separate payrolls for the coffee
factory (Exhibits M-2 and M-3, and for the gaugau factory (Exhibits O-2, O-3, and O-4). It is to be noted
that before July 21, 1951, the coffee payrolls all began with number "41-Maria Villanueva" with 24 or
more laborers (Exhibits M and M-1), whereas beginning July 21, 1951, the payrolls for the coffee factory
began with No. 1-Loreta Bernabe with only 14 laborers (Exhibits M-2 and M-3).
"F. During the ocular inspection made in the factory on August 26, 1951, the Court has found the
following:
'In the ground floor and second floor of the gau-gau factory there were hundreds of bags
of raw coffee behind the pile of gaugau sacks. There were also women employees working paper
wrappers for gaugau, and, in the same place there were about 3,000 cans to be used as
containers for coffee.
'The Court found out also that there were 16 trucks used both for the delivery of coffee
and gaugau. To show that those trucks carried both coffee and gaugau, the union president
invited the Court to examine the contents of delivery truck No. T-582 parked in a garage between
the gaugau building and the coffee factory, and upon examination, there were found inside the
said truck boxes of gaugau and cans of coffee,'"
and held that:
". . . there is only one management for the business of gau-gau and coffee with whom the laborers are
dealing regarding their work. Hence, the filing of action against the La Campana Starch and Coffee
Factory is proper and justified.
With regards to the alleged lack of personality, it is to be noted that before the certification of the
case to this Court on July 17, 1951, the petitioner Kaisahan ng Mga Manggagawa sa La Campana, had a
separate permit from the Department of Labor. This permit was suspended on September 30, 1951.
(Exhibit M-Intervenor, page 55, of the record). It is not true that, on July 17, 1951, when this case was
forwarded to this Court, the petitioner's permit, as an independent union, had not yet been issued, for the
very Exhibit MM-Intervenor regarding the permit, conclusively shows the preexistence of said permit."
(Annex G.)

Their motion for reconsideration of the above order having been denied, Tan Tong and La Campana Coffee
Factory, Inc. (same as La Campana Coffee Factory Co., Inc.), later joined by the PLOW, filed the present petition for
certiorari on the grounds that the Court of Industrial Relations had no jurisdiction to take cognizance of the case, for the
reason, according to them, "(1) that the petitioner La Campana Coffee Factory, Inc. has only 14 employees, only 5 of
whom are members of the respondent union and therefore the absence of the jurisdictional number (30) as provided by
sections 1 and 4 of Commonwealth Act No. 103; and, (2) that the suspension of respondent union's permit by the
Secretary of Labor has the effect of taking away the union's right to collective bargaining under section 2 of
Commonwealth Act No. 213 and, consequently, its personality to sue for and in behalf of its members."
As to the first ground, petitioners obviously do not question the fact that the number of employees of the La
Campana Gaugau Packing involved in the case is more than the jurisdictional number (31) required by law, but they do
contend that the industrial court has no jurisdiction to try the case as against La Campana Coffee Factory, Inc. because
the latter has allegedly only 14 laborers and only five of these are members of the respondent Kaisahan. This
contention loses force when it is noted that, as found by the industrial court — and this finding is conclusive upon us —
La Campana Gaugau Packing and La Campana Coffee Factory Co. Inc., are operating under one single management,
that is, as one business though with two trade names. True, the coffee factory is a corporation and, by legal fiction, an
entity existing separate and apart from the persons composing it, that is, Tan Tong and his family. But it is settled that
this fiction of law, which has been introduced as a matter of convenience and to subserve the ends of justice cannot be
invoked to further an end subversive of that purpose.
"Disregarding Corporate Entity. — The doctrine that a corporation is a legal entity existing
separate and apart from the persons composing it is a legal theory introduced for purposes of
convenience and to subserve the ends of justice. The concept cannot, therefore, be extended to a point
beyond its reason and policy, and when invoked in support of an end subversive of this policy, will be
disregarded by the courts. Thus, in an appropriate case and in furtherance of the ends of justice, a
corporation and the individual or individuals owning all its stocks and assets will be treated as identical,
the corporate entity being disregarded where used as a cloak or cover for fraud or illegality. (13 Am. Jur.,
160-161.)
" . . . A subsidiary or auxiliary corporation which is created by a parent corporation merely as an
agency for the latter may sometimes be regarded as identical with the parent corporation, especially if the
stockholders or officers of the two corporations are substantially the same or their system of operation
unified." (Ibid. 162; see Annotation 1 A. L. R. 612, s. 34 A. L. R. 599.)
In the present case Tan Tong appears to be the owner of the gaugau factory. And the coffee factory, though an
incorporated business, is in reality owned exclusively by Tan Tong and his family. As found by the Court of Industrial
Relations, the two factories have but one office, one management and one payroll, except after July 17, the day the
case was certified to the Court of Industrial Relations, when the person who was discharging the office of cashier for
both branches of the business began preparing separate payrolls for the two. And above all, it should not be overlooked
that, as also found by the industrial court, the laborers of the gaugau factory and the coffee factory were
interchangeable, that is, the laborers from the gaugau factory were sometimes transferred to the coffee factory and
vice-versa. In view of all these, the attempt to make the two factories appear as two separate businesses, when in
reality they are but one, is but a device to defeat the ends of the law (the Act governing capital and labor relations) and
should not be permitted to prevail.
The second point raised by petitioner is likewise without merit. In the first place, there being more than 30
laborers involved and the Secretary of Labor having certified the dispute to the Court of Industrial Relations, that court
duly acquired jurisdiction over the case (International Oil Factory vs. NLU, Inc. 73 Phil., 401; section 4, C. A. 108). This
jurisdiction was not lost when the Department of Labor suspended the permit of the respondent Kaisahan as a Labor
organization. For once jurisdiction is acquired by the Court of Industrial Relations it is retained until the case is
completely decided. (Manila Hotel Employees Association vs. Manila Hotel Co. et al., 73 Phil., 374.)
In view of the foregoing, the petition is denied, with costs against the petitioner.
||| (La Campana Coffee Factory, Inc. v. Kaisahan ng mga Manggagawa sa La Campana, G.R. No. L-5677, [May 25, 1953],
93 PHIL 160-167)
[G.R. No. 182770. September 17, 2014.]

WPM INTERNATIONAL TRADING, INC. and WARLITO P. MANLAPAZ, petitioners, vs. FE CORAZON
LABAYEN, respondent.

DECISION

BRION, J p:

We review in this petition for review on certiorari 1 the decision 2 dated September 28, 2007 and the resolution 3
dated April 28, 2008 of the Court of Appeals (CA) in CA-G.R. CV No. 68289 that affirmed with modification the decision 4 of
the Regional Trial Court (RTC), Branch 77, Quezon City.
The Factual Background
The respondent, Fe Corazon Labayen, is the owner of H.B.O. Systems Consultants, a management and consultant
firm. The petitioner, WPM International Trading, Inc. (WPM), is a domestic corporation engaged in the restaurant business,
while Warlito P. Manlapaz (Manlapaz) is its president.
Sometime in 1990, WPM entered into a management agreement with the respondent, by virtue of which the
respondent was authorized to operate, manage and rehabilitate Quickbite, a restaurant owned and operated by WPM. As
part of her tasks, the respondent looked for a contractor who would renovate the two existing Quickbite outlets in Divisoria,
Manila and Lepanto St., University Belt, Manila. Pursuant to the agreement, the respondent engaged the services of CLN
Engineering Services (CLN) to renovate Quickbite-Divisoria at the cost of P432,876.02.
On June 13, 1990, Quickbite-Divisoria's renovation was finally completed, and its possession was delivered to the
respondent. However, out of the P432,876.02 renovation cost, only the amount of P320,000.00 was paid to CLN, leaving a
balance of P112,876.02. SDAcaT
Complaint for Sum of Money (Civil Case No. Q-90-7013)
On October 19, 1990, CLN filed a complaint for sum of money and damages before the RTC against the
respondent and Manlapaz, which was docketed as Civil Case No. Q-90-7013. CLN later amended the complaint to exclude
Manlapaz as defendant. The respondent was declared in default for her failure to file a responsive pleading.
The RTC, in its January 28, 1991 decision, found the respondent liable to pay CLN actual damages in the amount
of P112,876.02 with 12% interest per annum from June 18, 1990 (the date of first demand) and 20% of the amount
recoverable as attorney's fees.
Complaint for Damages (Civil Case No. Q-92-13446)
Thereafter, the respondent instituted a complaint for damages against the petitioners, WPM and Manlapaz. The
respondent alleged that in Civil Case No. Q-90-7013, she was adjudged liable for a contract that she entered into for and in
behalf of the petitioners, to which she should be entitled to reimbursement; that her participation in the management
agreement was limited only to introducing Manlapaz to Engineer Carmelo Neri (Neri), CLN's general manager; that it was
actually Manlapaz and Neri who agreed on the terms and conditions of the agreement; that when the complaint for
damages was filed against her, she was abroad; and that she did not know of the case until she returned to the Philippines
and received a copy of the decision of the RTC.
In her prayer, the respondent sought indemnification in the amount of P112,876.60 plus interest at 12% per annum
from June 18, 1990 until fully paid; and 20% of the award as attorney's fees. She likewise prayed that an award of
P100,000.00 as moral damages and P20,000.00 as attorney's fees be paid to her.
In his defense, Manlapaz claims that it was his fellow incorporator/director Edgar Alcansaje who was in-charge with
the daily operations of the Quickbite outlets; that when Alcansaje left WPM, the remaining directors were compelled to hire
the respondent as manager; that the respondent had entered into the renovation agreement with CLN in her own personal
capacity; that when he found the amount quoted by CLN too high, he instructed the respondent to either renegotiate for a
lower price or to look for another contractor; that since the respondent had exceeded her authority as agent of WPM, the
renovation agreement should only bind her; and that since WPM has a separate and distinct personality, Manlapaz cannot
be made liable for the respondent's claim.
Manlapaz prayed for the dismissal of the complaint for lack of cause of action, and by way of counterclaim, for the
award of P350,000.00 as moral and exemplary damages and P50,000.00 attorney's fees.
The RTC, through an order dated March 2, 1993 declared WPM in default for its failure to file a responsive
pleading.
The Decision of the RTC
In its decision, the RTC held that the respondent is entitled to indemnity from Manlapaz. The RTC found that based
on the records, there is a clear indication that WPM is a mere instrumentality or business conduit of Manlapaz and as such,
WPM and Manlapaz are considered one and the same. The RTC also found that Manlapaz had complete control over WPM
considering that he is its chairman, president and treasurer at the same time. The RTC thus concluded that Manlapaz is
liable in his personal capacity to reimburse the respondent the amount she paid to CLN in connection with the renovation
agreement.
The petitioners appealed the RTC decision with the CA. There, they argued that in view of the respondent's act of
entering into a renovation agreement with CLN in excess of her authority as WPM's agent, she is not entitled to indemnity
for the amount she paid. Manlapaz also contended that by virtue of WPM's separate and distinct personality, he cannot be
made solidarily liable with WPM. AaCEDS
The Ruling of the Court of Appeals
On September 28, 2007, the CA affirmed, with modification on the award of attorney's fees, the decision of the
RTC. The CA held that the petitioners are barred from raising as a defense the respondent's alleged lack of authority to
enter into the renovation agreement in view of their tacit ratification of the contract.
The CA likewise affirmed the RTC ruling that WPM and Manlapaz are one and the same based on the following: (1)
Manlapaz is the principal stockholder of WPM; (2) Manlapaz had complete control over WPM because he concurrently held
the positions of president, chairman of the board and treasurer, in violation of the Corporation Code; (3) two of the four other
stockholders of WPM are employed by Manlapaz either directly or indirectly; (4) Manlapaz's residence is the registered
principal office of WPM; and (5) the acronym "WPM" was derived from Manlapaz's initials. The CA applied the principle of
piercing the veil of corporate fiction and agreed with the RTC that Manlapaz cannot evade his liability by simply invoking
WPM's separate and distinct personality.
After the CA's denial of their motion for reconsideration, the petitioners filed the present petition for review on
certiorari under Rule 45 of the Rules of Court.
The Petition
The petitioners submit that the CA gravely erred in sustaining the RTC's application of the principle of piercing the
veil of corporate fiction. They argue that the legal fiction of corporate personality could only be discarded upon clear and
convincing proof that the corporation is being used as a shield to avoid liability or to commit a fraud. Since the respondent
failed to establish that any of the circumstances that would warrant the piercing is present, Manlapaz claims that he cannot
be made solidarily liable with WPM to answer for damages allegedly incurred by the respondent.
The petitioners further argue that, assuming they may be held liable to reimburse to the respondent the amount she
paid in Civil Case No. Q-90-7013, such liability is only limited to the amount of P112,876.02, representing the balance of the
obligation to CLN, and should not include the twelve 12% percent interest, damages and attorney's fees.
The Issues
The core issues are: (1) whether WPM is a mere instrumentality, alter-ego, and business conduit of Manlapaz; and
(2) whether Manlapaz is jointly and severally liable with WPM to the respondent for reimbursement, damages and interest.
Our Ruling
We find merit in the petition.
We note, at the outset, that the question of whether a corporation is a mere instrumentality or alter-ego of another is
purely one of fact. 5 This is also true with respect to the question of whether the totality of the evidence adduced by the
respondent warrants the application of the piercing the veil of corporate fiction doctrine. 6
Generally, factual findings of the lower courts are accorded the highest degree of respect, if not finality. When
adopted and confirmed by the CA, these findings are final and conclusive and may not be reviewed on appeal, 7 save in
some recognized exceptions 8 among others, when the judgment is based on misapprehension of facts.
We have reviewed the records and found that the application of the principle of piercing the veil of corporate fiction
is unwarranted in the present case.
On the Application of the Principle of
Piercing the Veil of Corporate Fiction
The rule is settled that a corporation has a personality separate and distinct from the persons acting for and in its
behalf and, in general, from the people comprising it. 9 Following this principle, the obligations incurred by the corporate
officers, or other persons acting as corporate agents, are the direct accountabilities of the corporation they represent, and
not theirs. Thus, a director, officer or employee of a corporation is generally not held personally liable for obligations
incurred by the corporation; 10 it is only in exceptional circumstances that solidary liability will attach to them.
Incidentally, the doctrine of piercing the corporate veil applies only in three (3) basic instances, namely: a) when the
separate and distinct corporate personality defeats public convenience, as when the corporate fiction is used as a vehicle
for the evasion of an existing obligation; b) in fraud cases, or when the corporate entity is used to justify a wrong, protect a
fraud, or defend a crime; or c) is used in alter ego cases, i.e., where a corporation is essentially a farce, since it is a
mere alter ego or business conduit of a person, or where the corporation is so organized and controlled and its
affairs so conducted as to make it merely an instrumentality, agency, conduit or adjunct of another corporation. 11
Piercing the corporate veil based on the alter ego theory requires the concurrence of three elements, namely:
(1) Control, not mere majority or complete stock control, but complete domination, not only of finances but of policy
and business practice in respect to the transaction attacked so that the corporate entity as to this transaction had at the time
no separate mind, will or existence of its own;
(2) Such control must have been used by the defendant to commit fraud or wrong, to perpetuate the violation of a
statutory or other positive legal duty, or dishonest and unjust act in contravention of plaintiff's legal right; and
(3) The aforesaid control and breach of duty must have proximately caused the injury or unjust loss complained of.
DcITaC
The absence of any of these elements prevents piercing the corporate veil. 12
In the present case, the attendant circumstances do not establish that WPM is a mere alter ego of Manlapaz.
Aside from the fact that Manlapaz was the principal stockholder of WPM, records do not show that WPM was
organized and controlled, and its affairs conducted in a manner that made it merely an instrumentality, agency, conduit or
adjunct of Manlapaz. As held in Martinez v. Court of Appeals, 13 the mere ownership by a single stockholder of even all or
nearly all of the capital stocks of a corporation is not by itself a sufficient ground to disregard the separate corporate
personality. To disregard the separate juridical personality of a corporation, the wrongdoing must be clearly and
convincingly established. 14
Likewise, the records of the case do not support the lower courts' finding that Manlapaz had control or domination
over WPM or its finances. That Manlapaz concurrently held the positions of president, chairman and treasurer, or that the
Manlapaz's residence is the registered principal office of WPM, are insufficient considerations to prove that he had
exercised absolute control over WPM.
In this connection, we stress that the control necessary to invoke the instrumentality or alter ego rule is not majority
or even complete stock control but such domination of finances, policies and practices that the controlled corporation has,
so to speak, no separate mind, will or existence of its own, and is but a conduit for its principal. The control must be shown
to have been exercised at the time the acts complained of took place. Moreover, the control and breach of duty must
proximately cause the injury or unjust loss for which the complaint is made.
Here, the respondent failed to prove that Manlapaz, acting as president, had absolute control over WPM. Even
granting that he exercised a certain degree of control over the finances, policies and practices of WPM, in view of his
position as president, chairman and treasurer of the corporation, such control does not necessarily warrant piercing the veil
of corporate fiction since there was not a single proof that WPM was formed to defraud CLN or the respondent, or that
Manlapaz was guilty of bad faith or fraud.
On the contrary, the evidence establishes that CLN and the respondent knew and acted on the knowledge that they
were dealing with WPM for the renovation of the latter's restaurant, and not with Manlapaz. That WPM later reneged on its
monetary obligation to CLN, resulting to the filing of a civil case for sum of money against the respondent, does not
automatically indicate fraud, in the absence of any proof to support it.
This Court also observed that the CA failed to demonstrate how the separate and distinct personality of WPM was
used by Manlapaz to defeat the respondent's right for reimbursement. Neither was there any showing that WPM attempted
to avoid liability or had no property against which to proceed.
Since no harm could be said to have been proximately caused by Manlapaz for which the latter could be held
solidarily liable with WPM, and considering that there was no proof that WPM had insufficient funds, there was no sufficient
justification for the RTC and the CA to have ruled that Manlapaz should be held jointly and severally liable to the respondent
for the amount she paid to CLN. Hence, only WPM is liable to indemnify the respondent.
Finally, we emphasize that the piercing of the veil of corporate fiction is frowned upon and thus, must be done with
caution. 15 It can only be done if it has been clearly established that the separate and distinct personality of the corporation
is used to justify a wrong, protect fraud, or perpetrate a deception. The court must be certain that the corporate fiction was
misused to such an extent that injustice, fraud, or crime was committed against another, in disregard of its rights; it cannot
be presumed.
On the Award of Moral Damages
On the award of moral damages, we find the same in order in view of WPM's unjustified refusal to pay a just debt.
Under Article 2220 of the New Civil Code, 16 moral damages may be awarded in cases of a breach of contract where the
defendant acted fraudulently or in bad faith or was guilty of gross negligence amounting to bad faith.
In the present case, when payment for the balance of the renovation cost was demanded, WPM, instead of
complying with its obligation, denied having authorized the respondent to contract in its behalf and accordingly refused to
pay. Such cold refusal to pay a just debt amounts to a breach of contract in bad faith, as contemplated by Article 2220.
Hence, the CA's order to pay moral damages was in order. DSTCIa
WHEREFORE, in light of the foregoing, the decision dated September 28, 2007 of the Court of Appeals in CA-G.R.
CV No. 68289 is MODIFIED and that petitioner Warlito P. Manlapaz is ABSOLVED from any liability under the renovation
agreement. SO ORDERED.
||| (WPM International Trading, Inc. v. Labayen, G.R. No. 182770, [September 17, 2014])

[G.R. No. L-13119. September 22, 1959.]

RICARDO TANTONGCO, petitioner, vs. KAISAHAN NG MGA MANGGAGAWA SA LA CAMPANA


(KKM) AND THE HONORABLE COURT OF INDUSTRIAL RELATIONS, respondents.

Ernesto C. Estrella for petitioner.


Carlos E. Santiago for respondent Union.
Pedro M. Ligaya for respondent CIR.

SYLLABUS

1. CORPORATIONS; DISTINCT PERSONALITY OF A CORPORATION; JURISDICTION OF COURT OF


INDUSTRIAL RELATIONS NOT AFFECTED BY DEATH OF OWNER. — The death of an owner and manager of a
corporation, against which cases are pending in the Court of Industrial relations, does not deprive the latter of its
jurisdiction over the same. The party in those cases being the corporation and not the owner or manager personally, the
claims of the laborers therein, which are merely incidental to their demands for reinstatement for having been unjustly
dismissed, and for better working conditions, are not the claims contemplated by law to be submitted before the
administrator of the estate of a deceased person.
2. ID.; EXISTENCE AND OPERATION; ESTOPPEL. — Having admitted the existence and operation of the two
entities in a complaint filed for injunction by the said entities and on two other occasions, petitioner, who is now
estopped from denying the existence of the same.
3. COURT OF INDUSTRIAL RELATIONS; POWER TO ENFORCE ORDERS; CONTEMPT. — Petitioner's
contention that after he ceased to be administrator of the estate of the deceased owner and manager of the entities in
question, he cannot now be compelled to comply with the order of the Court of Industrial Relations to appear in the
contempt proceedings instituted against him is untenable. The jurisdiction and authority of said court as to compliance
with and violations of its orders are clearly defined in section 6 of Commonwealth Act No. 143.

DECISION

MONTEMAYOR, J p:

This is a petition for certiorari and prohibition with prayer for issuance of a writ of preliminary injunction to
prohibit respondent Court of Industrial Relations from proceeding with the hearing of the contempt proceedings
for which petitioner Ricardo Tantongco was cited to appear and present his evidence. The contempt proceedings
which petitioner seeks to stop are based on the order of the Court of Industrial Relations, dated September 30,
1957, which reads as follows:
"It appearing that the Order of this Court, in the above-entitled case, dated February 18,
1957 (folios 134-166), has become final and executory and the respondents have failed to comply
with the same, the said respondents, namely, the La Campana Starch and Coffee Factory or its
manager or the person who has charge of the management, and the administrator of the Estate of
Ramon Tantongco are hereby ordered to comply with said Order, within five days from receipt
hereof, particularly the following, to wit:
"(a) To reinstate the persons named in the said Order of February 18, 1957;
"(b) To deposit the amount of P65,534.01 with this Court.
"With respect to possible back wages from August 28, 1957 as mentioned in the petition
for contempt of August 30, 1957, the same shall first be determined.
"Failure to comply with this Order shall be directly dealt with accordingly."
It would appear that petitioner Ricardo Tantongco failed to comply with said order and so, as already
stated, he was cited to appear and to adduce evidence on his behalf to show why he should not be punished for
indirect contempt.
The facts in this case may be briefly narrated thus: Sometime in June, 1951, members of the Kaisahan
ng mga Manggagawa sa La Campana, a labor union to which were affiliated workers in the La Campana Starch
Factory and La Campana Coffee Factory, two separate entities but under one management, presented demands
for higher wages, and more privileges and benefits in connection with their work. When the management failed
and refused to grant the demands, the Department of Labor intervened; but failing to settle the controversy, it
certified the dispute to the Court of Industrial Relations on July 17, 1951, where it was docketed as Case No. 584-
V. On the theory that the laborers presenting the demands were only the ones working in the coffee factory, said
company filed through the management a motion to dismiss claiming that inasmuch as there were only 14 of
them in said factory, the Court of Industrial Relations had no jurisdiction to entertain and decide the case. The
motion was denied by the Court of Industrial Relations, which said:
". . . There was only one management for the business of gawgaw and coffee with whom
the laborers are dealing regarding their work. Hence, the filing of action against the La Campana
Starch and Coffee Factory is proper and justified."
The order of denial was appealed to this Tribunal through certiorari under G. R. No. L-5677. In disposing of the
case, we held:
"As to the first ground, petitioners obviously do not question the fact that the number of
employees La Campana Gaugau Packing involved in the case is more than the jurisdictional
number (31) required by law, but they contend that the industrial court has no jurisdiction to try the
case against La Campana Coffee Factory, Inc. because the latter has allegedly only 14 laborers
and only five of these are members of respondent Kaisahan. This contention loses force when it is
noted that, as found by the industrial court — and this finding is conclusive upon us — La
Campana Gaugau Packing and La Campana Coffee Factory Co. Inc., are operating under one
single management, that is, one business though with two trade names. True, the coffee factory is
a corporation, and, by legal fiction, an entity existing separate and apart from the persons
composing it, that is, Tan Tong and his family. But it is settled this fiction of law, which has been
introduced as a matter of convenience and to subserve the ends of justice cannot be invoked to
further an end subversive of that purpose.
". . . The attempt to make the two factories appear as two separate businesses, when in
reality they are but one is but a device to defeat the ends of the law (the Act governing capital and
labor relations) and should not be permitted to prevail." (La Campana Coffee Factory, et al. vs.
Kaisahan ng mga Manggagawa, etc. et al., 93 Phil., 160; 49 Off. Gaz., [6] 2300.)
Upon the return of the case to the Court of Industrial Relations, the latter proceeded with the hearing. In the
meantime incidental cases involving the same parties came up and were filed before the Court of Industrial
Relations in the following cases:
Case No. 584-V(1) — petition for contempt against the La Campana Starch and Coffee
Factory for having employed 21 new laborers in violation of the order of July 21, 1951, filed on July
25, 1951;
Case No. 584-V(2) — petition of La Campana for authority to dismiss Loreto Bernabe, filed
on July 25, 1951;
Case No. 584-V(3) — petition of Union to reinstate Bonifacio Calderon with backpay, filed
on August 3, 1951;
Case No. 584-V(5) — petition of Union to reinstate Marcelo Estrada and Exequiel Rapiz
with back pay and to punish officials of the company for contempt, filed on February 13, 1952; and
Case No. 584-V(6) — petition of union for reinstatement of Ibardolaza and seven other
member-laborers and to punish the officers of the company for contempt, filed on July 15, 1953.
These five cases were heard jointly. In the meantime Ramon Tantongco supposed to be the owner and manager
of the La Campana Starch Factory and the person in charge of the La Campana Coffee Factory died on May 16,
1956. On motion of the labor union, the Court of Industrial Relations ordered the inclusion as party respondent of
the administrator of the estate Ramon Tantongco who was Ricardo Tantongco.
Ricardo Tantongco, as administrator, under a special appearance filed a motion to dismiss all the cases
including the main case, that is to say, Cases No. 584-(V) to 584-V(6), on the ground that said cases involved
claims for sums of money and consequently should be filed before the probate court having jurisdiction over the
estate, pursuant to the provisions of Rule 3, Section 21, and Rule 88, Section 1 of the Rules of Court. On August
23, 1956, the Court of Industrial Relations denied the motion to dismiss and proceeded to hear the incidental
cases against the La Campana entities.
On June 12, 1956, a partial decision was rendered in the main case No. 584-V, which partial decision
was elevated to us and is still pending appeal. On February 18, 1957, the Court of Industrial Relations issued an
order in incidental Cases No. 584-V(1), V(2), V(3), V(5) and V(6), directing the "management of the respondent
company and or the administrator of the Estate of Ramon Tantongco", to reinstate the dismissed laborers
mentioned therein with back wages. This order of February 18, 1957, as well as the order directing the inclusion
of the administrator of the estate of Ramon Tantongco as additional respondent in the incidental cases, and the
order denying the petition of the administrator to dismiss said incidental cases were appealed to this tribunal
through certiorari. The appeal, however, was summarily dismissed by this Court in its resolution of June 12, 1957,
as follows:
"This Court, deliberating upon the allegations of the petition filed in case L-12355 (La
Campana Starch Coffee Factory et al. vs. Kaisahan ng Mga Manggagawa sa La Campana, KKM,
et al) for review, on certiorari of the decision of the Court of Industrial Relations referred to therein,
and finding that there is no merit in the petition, RESOLVES TO DISMISS the same."
The CIR order of February 18, 1957, in the incidental cases Nos. 584-V to V(6), having become final and
executory, the laborers involved reported for work on August 28, 1957, but they were not admitted by the
management. Consequently, the union filed a petition dated August 30, 1957, to hold respondents in said cases
for contempt. After hearing the CIR issued the order of September 30, 1957, subject of this petition, ordering "the
La Campana Starch and Coffee Factory or its manager or the person who has charge of its management and the
administrator of the estate of Ramon Tantongco" to "reinstate the persons named in the order of February 18,
1957" and "to deposit the amount of P65,534.01." For refusal or failure to comply with said order, petitioner
Ricardo Tantongco was required to appear before the attorney of the CIR in contempt proceedings. Petitioner
now seeks to prohibit the CIR from proceeding with the trial for contempt and to enjoin respondent CIR from
enforcing its order of September 30, 1957.
Petitioner contends that upon the death of Ramon Tantongco, the claims of the laborers should have
been dismissed and that said claims should have been filed with the probate court having jurisdiction over the
administration proceedings of the estate of Ramon Tantongco, pursuant to the provisions of Rule 3, Section 21 of
the Rules of Court and that the failure to file said claims with the administrator forever barred said claims as
provided in Rule 87, Section 5 of the Rules of Court, especially after the assets of the estate had been distributed
among the heirs, and petitioner had ceased to be the administrator of the estate. As already stated this same
question was raised by petitioner in G. R. No. L-12355, entitled "La Campana Starch and Coffee Factory and
Ricardo Tantongco, etc. vs. Kaisahan ng mga Manggagawa sa La Campana (KKM)," which, as already stated,
was summarily dismissed by this Court in a resolution dated June 12, 1957. Consequently, said question may not
again be raised in the present case. Furthermore, it may be recalled that both in the main case and in the
incidental cases No. 584-V to 584-V(6), Ramon Tantongco was never a party. The party there was the La
Campana Starch and Coffee Factory by which name it was sought to designate the two entities La Campana
Starch Packing and the La Campana Coffee Factory. Naturally, the claims contained in said cases were not the
claims contemplated by law to be submitted before the administrator. In other words the death of Ramon
Tantongco did not deprive the CIR of its jurisdiction over the cases aforementioned. Moreover, the money claims
of the laborers were merely incidental to their demands for reinstatement for having been unjustly dismissed, and
for better working conditions.
Petitioner, however, contends that in G.R. No. L-5677, we "pierced the veil of corporate existence", and
held that the La Campana Starch and Coffee Factory and its owner, Ramon Tantongco, were one; so that with
the death of Ramon, the La Campana entities ceased to exist, resulting in the loss of jurisdiction of the CIR to
enforce its order against said entities. The reason we applied the so-called "piercing the veil of corporate
existence" in G. R. No. L-5677 was to avoid the technicality therein advanced in order to defeat the jurisdiction of
the CIR. We there found that although there were ostensibly two separate companies or entities, they were
managed by the same person or persons and the workers in both were used interchangeably so that in order to
determine whether or not the CIR had jurisdiction, the number of workers in both entities, not in only one, was to
be considered. However, we still believe that although the family of Ramon Tantongco was practically the owner
of both the coffee factory and the starch factory, nevertheless these entities are separate from the personality of
Ramon. The coffee factory is a stock corporation and the shares are owned not only by Ramon but also by
others, such as petitioner Ricardo who not only is a stockholder and director and treasurer but also the manager
of the same. Furthermore, petitioner is now estopped from claiming that the two entities in question and Ramon
are one. Thus in Annex 3-CIR (par. 1 thereof) which is a complaint for injunction filed by La Campana Food
Products, et al and La Campana Starch Packing against the Consolidated Labor Organization of the Philippines,
in Civil Case No. P-2482 in the Court of First Instance of Rizal, petitioner admitted the existence and operation of
said entities; in Annex 4-CIR where petitioner appeared as General Manager representing the two entities in its
agreement with the La Campana Workers Union to resolve the dispute between the two entities and the laborers
in cases Nos. 1072-V and 1371-ULP, the existence of the two entities appears to have been admitted; and in
Annex 5-A-CIR, an answer to the complaint of La Campana Workers Union in case No. 1471-ULP (Annex 5-
CIR), petitioner admitted the allegation that said two factories were in existence and doing business with
petitioner as manager of the same.
In relation to the order of the CIR requiring petitioner to appear in the contempt proceedings instituted
against him, petitioner contends that after he ceased to be the administrator of the estate of Ramon Tantongco,
he may not now be compelled to comply with the order of the court. In answer, it is enough to bear in mind the
jurisdiction and authority of the CIR as to compliance with and violations of its orders under section 6,
Commonwealth Act No. 143, which we quote below:
". . . The Court or any Judge thereof shall have furthermore, all the inherent powers of a
court of justice provided in paragraph 5 of Rule 124 of the Supreme Court, as well as the power to
punish direct and indirect contempt as provided in Rule 64 of the same Court, under the same
procedure and penalties provided therein.
"Any violation of any order, award, or decision of the Court of Industrial Relations shall,
after such order, award or decision has become final, conclusive, and executory, constitute
contempt of court: . . .
"In case the employer (or landlord) committing any such violation or contempt is an
association or corporation, the manager or the person who has the charge of the management of
the business of the association or corporation and the officers or directors thereof who have
ordered or authorized the violation of contempt shall be liable. . . ."
In conclusion, we find and hold that the La Campana Starch and Food Products Company which stands
for the La Campana Starch and Coffee Factory are entities distinct from the personality of Ramon Tantongco;
that after the death of Ramon these two entities continued to exist and to operate under the management of
petitioner and that consequently he is the proper person and official to which the orders of the CIR are addressed
and who is in duty bound to comply with the same. We further find that the CIR acted within its jurisdiction in
issuing its order of September 30, 1957 and in requiring petitioner to appear to give his evidence if any in relation
with the contempt proceedings instituted against him.
In view of the foregoing, the petition for certiorari is hereby denied and the writ of preliminary injunction
dissolved, with costs.
||| (Tantongco v. Kaisahan ng mga Manggagawa sa La Campana, G.R. No. L-13119, [September 22, 1959], 106 PHIL 198-
207)

[A.M. No. R-181-P. July 31, 1987.]

ADELIO C. CRUZ, complainant, vs. QUITERIO L. DALISAY, Deputy Sheriff, RTC, Manila,
respondents.

RESOLUTION

FERNAN, J p:

In a sworn complaint dated July 23, 1984, Adelio C. Cruz charged Quiterio L. Dalisay, Senior Deputy Sheriff of Manila, with
"malfeasance in office, corrupt practices and serious irregularities" allegedly committed as follows:
1. Respondent sheriff attached and/or levied the money belonging to complainant Cruz when he was not himself the
judgment debtor in the final judgment of NLRC NCR Case No. 8-12389-91 sought to be enforced but rather the company
known as "Qualitrans Limousine Service, Inc.," a duly registered corporation; and,
2. Respondent likewise caused the service of the alias writ of execution upon complainant who is a resident of Pasay City,
despite knowledge that his territorial jurisdiction covers Manila only and does not extend to Pasay City.
In his Comments, respondent Dalisay explained that when he garnished complainant's cash deposit at the Philtrust bank,
he was merely performing a ministerial duty. While it is true that said writ was addressed to Qualitrans Limousine Service,
Inc., yet it is also a fact that complainant had executed an affidavit before the Pasay City assistant fiscal stating that he is
the owner/president of said corporation and, because of that declaration, the counsel for the plaintiff in the labor case
advised him to serve notice of garnishment on the Philtrust bank.
On November 12, 1984, this case was referred to the Executive Judge of the Regional Trial Court of Manila for
investigation, report and recommendation.
Prior to the termination of the proceedings, however, complainant executed an affidavit of desistance stating that he is no
longer interested in prosecuting the case against respondent Dalisay and that it was just a "misunderstanding" between
them. Upon respondent's motion, the Executive Judge issued an order dated May 29, 1986 recommending the dismissal of
the case.
It has been held that the desistance of complainant does not preclude the taking of disciplinary action against respondent.
Neither does it dissuade the Court from imposing the appropriate corrective sanction. One who holds a public position,
especially an office directly connected with the administration of justice and the execution of judgments, must at all times be
free from the appearance of impropriety. 1
We hold that respondent's actuation in enforcing a judgment against complainant who is not the judgment debtor in the case
calls for disciplinary action. Considering the ministerial nature of his duty in enforcing writs of execution, what is incumbent
upon him is to ensure that only that portion of a decision ordained or decreed in the dispositive part should be the subject of
execution. 2 No more, no less. That the title of the case specifically names complainant as one of the respondents is of no
moment as execution must conform to that directed in the dispositive portion and not in the title of the case.LibLex
The tenor of the NLRC judgment and the implementing writ is clear enough. It directed Qualitrans Limousine Service, Inc.,
to reinstate the discharged employees and pay them full backwages. Respondent, however, chose to "pierce the veil of
corporate entity" usurping a power belonging to the court and assumed improvidently that since the complainant is the
owner/president of Qualitrans Limousine Service, Inc., they are one and the same. It is a well-settled doctrine both in law
and in equity that as a legal entity, a corporation has a personality distinct and separate from its individual stockholders or
members. The mere fact that one is president of a corporation does not render the property he owns or possesses the
property of the corporation, since the president, as individual, and the corporation are separate entities. 3
Anent the charge that respondent exceeded his territorial jurisdiction, suffice it to say that the writ of execution sought to be
implemented was dated July 9, 1984, or prior to the issuance of Administrative Circular No. 12 which restrains a sheriff from
enforcing a court writ outside his territorial jurisdiction without first notifying in writing and seeking the assistance of the
sheriff of the place where execution shall take place.
ACCORDINGLY, we find Respondent Deputy Sheriff Quiterio L. Dalisay NEGLIGENT in the enforcement of the writ of
execution in NLRC Case No. 8-12389-91, and a fine equivalent to three [3] months salary is hereby imposed with a stern
warning that the commission of the same or similar offense in the future will merit a heavier penalty. Let a copy of this
Resolution be filed in the personal record of the respondent. SO ORDERED.
||| (Cruz v. Dalisay, A.M. No. R-181-P (Resolution), [July 31, 1987], 236 PHIL 520-523)

[G.R. No. 165442. August 25, 2010.]

NASECO GUARDS ASSOCIATION-PEMA (NAGA-PEMA), petitioner, vs. NATIONAL SERVICE


CORPORATION (NASECO), respondent.

DECISION

VILLARAMA, JR., J p:

This petition for review on certiorari under Rule 45 assails the Decision 1 dated May 27, 2004 of the Court of
Appeals (CA) in CA-G.R. SP No. 76667. The appellate court set aside the January 15, 2003 2 and March 11, 2003 3
Orders of the Department of Labor and Employment (DOLE) and ordered the latter to allow the parties to adduce
evidence in support of their respective positions.
The facts follow.
Respondent National Service Corporation (NASECO) is a wholly-owned subsidiary of the Philippine National
Bank (PNB) organized under the Corporation Code in 1975. It supplies security and manpower services to different
clients such as the Securities and Exchange Commission, the Philippine Deposit Insurance Corporation, Food Terminal
Incorporated, Forex Corporation and PNB. Petitioner NASECO Guards Association-PEMA (NAGA-PEMA) is the
collective bargaining representative of the regular rank and file security guards of respondent. NASECO Employees
Union-PEMA (NEMU-PEMA) is the collective bargaining representative of the regular rank and file (non-security)
employees of respondent such as messengers, janitors, typists, clerks and radio-telephone operators. 4
On December 2, 1993, respondent entered into a memorandum of agreement 5 with petitioner. The terms of
the agreement covered the monetary claims of the petitioner such as salary adjustments, conversion of salary scheme
under Republic Act (R.A.) No. 6758 6 to R.A. No. 6727, 7 signing bonus, leaves and other benefits. A year after,
petitioner demanded full negotiation for a collective bargaining agreement (CBA) with the respondent and submitted its
proposals thereto. TDcHCa
On June 8, 1995, petitioner and respondent agreed to sign a CBA on non-economic terms. 8
On September 24, 1996, petitioner filed a notice of strike because of respondent's refusal to bargain for
economic benefits in the CBA. Following conciliation hearings, the parties again commenced CBA negotiations and
started to resolve the issues on wage increase, productivity bonus, incentive bonus, allowances, and other benefits but
failed to reach an agreement.
Meanwhile, respondent and NEMU-PEMA entered into a CBA on non-economic terms. 9 Unfortunately, a
dispute among the leaders of NEMU-PEMA arose and at a certain point, leadership of the organization was unclear.
Hence, the negotiations concerning the economic terms of the CBA were put on hold until the internal dispute could be
resolved.
On April 29, 1997, petitioner filed a notice of strike before the National Conciliation and Mediation Board
(NCMB) against respondent and PNB due to a bargaining deadlock. The following day, NEMU-PEMA likewise filed a
notice of strike against respondent and PNB on the ground of unfair labor practices. 10 Efforts by the NCMB to
conciliate failed and pursuant to Article 263 (g) of the Labor Code, 11 as amended, then DOLE Secretary Cresenciano
B. Trajano assumed jurisdiction over the strike notices on June 25, 1998. 12
On November 19, 1999, then DOLE Secretary Bienvenido E. Laguesma issued a Resolution 13 directing
petitioner and respondent to execute a new CBA incorporating therein his dispositions regarding benefits of the
employees as to wage increase, productivity bonus, vacation and sick leave, medical allowances and signing bonus.
Respondent was further ordered to negotiate, for purposes of collective bargaining agreement, with NEMU-PEMA led
by its president, Ligaya Valencia. The charge of unfair labor practice against respondent and PNB was dismissed. 14
Respondent promptly filed a petition for certiorari before the CA questioning the DOLE Secretary's order and
arguing that the ruling of the DOLE Secretary in favor of the unions and awarding them monetary benefits totaling five
hundred thirty-one million four hundred forty-six thousand six hundred sixty-six and 67/100 (P531,446,666.67) was
inimical and deleterious to its financial standing and will result in closure and cessation of business for the company.
By Decision 15 dated March 19, 2001 (first CA Decision), the CA partly granted the petition and ruled that a
recomputation and reevaluation of the benefits awarded was in order. cCSTHA
WHEREFORE, the instant petition is partly GRANTED in that the case is remanded to the
Secretary of Labor for purposes of recomputation and reevaluation of the CBA benefits.
SO ORDERED. 16
In compliance with the CA directive, then DOLE Secretary Patricia A. Sto. Tomas conducted several
clarificatory hearings. On January 15, 2003, Secretary Sto. Tomas issued an Order which provides:
From the above, it is indubitable that the total cost to NASECO of our questioned award would
amount to only P322,725,000, not P531,446,666.67 as claimed by the company. Thus, our November
19, 1999 Order is hereby affirmed en toto.
WHEREFORE, judgment is hereby rendered:
1. [D]irecting NAGA-PEMA and NASECO to execute a new collective bargaining
agreement effective November 1, 1993, incorporating therein the dispositions
contained in our November 19, 1999 Order as well as all other items agreed
upon by the parties.
2. Ordering NASECO to negotiate with NEMA-PEMA for a new collective bargaining
agreement.
The charges of unfair labor practice against NASECO and PNB are dismissed for lack of merit.
SO ORDERED. 17
Respondent filed a motion for reconsideration with the DOLE Secretary which was denied on March 11, 2003.
Respondent thus filed a petition for certiorari with the CA arguing that the DOLE Secretary, in issuing the
January 15, 2003 Order deprived respondent of due process of law for there was no reevaluation that took place in the
DOLE. It also argued that the order merely recomputed the DOLE Secretary's initial award of P531,446,666.67 and
reduced it to P322,725,000.00, contrary to the ruling of the CA to recompute and reevaluate. Respondent claimed that
what the DOLE Secretary should have done was to let the parties introduce evidence to show the proper computation of
the monetary awards under the approved CBA. ITCcAD
In its second Decision dated May 27, 2004, the CA granted the petition, thus:
WHEREFORE, the orders dated 15 January 2003 and 11 March 2003 are hereby SET ASIDE
and the case remanded to the public respondent to allow the parties to adduce evidence in support of
their respective positions.
SO ORDERED. 18
A motion for reconsideration was filed by herein petitioner but the same was denied by the CA on September
22, 2004 19 finding no reason to reverse and set aside its earlier decision.
Petitioner now comes to this Court for relief by way of a petition for review on certiorari seeking to set aside and
reverse the May 27, 2004 Decision and the September 22, 2004 Resolution of the CA.
The main issue in this case is whether or not the respondent's right to due process was violated. A side issue
raised by the petitioner is whether or not PNB, being the undisputed owner of and exercising control over respondent,
should be made liable to pay the CBA benefits awarded to the petitioner.
Petitioner argues first that there was no violation of due process because respondent was never prohibited by
the DOLE Secretary to submit supporting documents when the instant case was pending on remand. Petitioner
contends that due process is properly observed when there is an opportunity to be heard, to present evidence and to
file pleadings, which was never denied to respondent.
Second, petitioner argues that the CA erred in stating that respondent was a company operating at a loss and
therefore cannot be expected to act generously and confer upon its employees additional benefits exceeding what is
mandated by law. It is the petitioner's position that based on the "no loss, no profit" policy of respondent with PNB,
respondent in truth has no "pocket" of its own and is, in effect, one (1) and the same with PNB with regard to financial
gains and/or liabilities. Thus, petitioners contend that the CBA benefits should be shouldered by PNB considering the
poor financial condition of respondent. To support such claim, petitioner submitted evidence 20 to show that PNB is in
superb financial condition and is very much capable of shouldering the CBA award. 21 TCcDaE
Respondent on the other hand maintains that the DOLE Secretary violated its right to due process when she
merely recomputed the CBA award instead of reevaluating the entire case and allowing it to present supporting
documents in accordance with the first CA decision. 22 It claims that the order of the CA to reevaluate included and
required a full assessment of the case together with reception of evidence such as financial statements, and the
omission of such is a violation of its right to due process.
As to the petitioner's argument that respondent and PNB are essentially the same when it comes to financial
condition, respondent contends that although a subsidiary, it has a separate and distinct personality from PNB with its
own charter. Hence, the issue of PNB's financial well-being is immaterial in this case.
The petition is partly meritorious.
In simple terms, the constitutional guarantee of due process requires that a litigant be given "a day in court." It
is the availability of the opportunity to be heard that determines whether or not due process was violated. A litigant may
or may not avail of the opportunity to be heard but as long as such was made available to him/her, there is no violation
of the due process clause. In the case of Lumiqued v. Exevea, 23 this Court declared that "[a]s long as a party was
given the opportunity to defend his interests in due course, he cannot be said to have been denied due process of law,
for this opportunity to be heard is the very essence of due process. Moreover, this constitutional mandate is deemed
satisfied if a person is granted an opportunity to seek reconsideration of the action or ruling complained of."
The respondent's right to due process in this case has not been denied. The order in the first CA decision to
recompute and reevaluate was satisfied when the DOLE Secretary reexamined their initial findings and adjusted the
awarded benefits. A reevaluation, contrary to what the respondent claims, is a process by which a person or office (in
this case the DOLE secretary) revisits its own initial pronouncement and makes another assessment of its findings. In
simple terms, to reevaluate is to take another look at a previous matter in issue. A reevaluation does not necessitate the
introduction of new materials for review nor does it require a full hearing for new arguments.
From a procedural standpoint, a reevaluation is a continuation of the original case and not a new proceeding.
Hence, the evidence, financial reports and other documents submitted by the parties in the course of the original
proceeding are to be visited and reviewed again. In this light, the respondent has been given the opportunity to be
heard by the DOLE Secretary.
Also, contrary to the claim of the respondent that it was barred by the DOLE Secretary to introduce supporting
documents during the recomputation and reevaluation, the records show that an Order by then Secretary of Labor
Patricia A. Sto. Tomas dated July 11, 2002 specifically allowed both parties to submit their respective computations as
regards the awarded benefits. To wit: aHSTID
WHEREFORE, the Bureau of Working Conditions is hereby directed to submit to this Office a
detailed computation of the CBA benefits indicated in the resolution of November 19, 2001 within twenty
(20) days from receipt of this Order. The parties may submit their own computations to the Bureau for
validation.
SO ORDERED. 24 (Italics supplied.)
It is thus inaccurate for the respondent to claim that it was denied due process because it had all the
opportunity to introduce any supporting document in the course of the recomputation and reevaluation of the DOLE
Secretary. Respondent admits that it did attach the financial statements and other documents in support of its alleged
financial incapacity to pay the CBA awarded benefits, the same evidence it had earlier submitted before the CA
(Memorandum in the first CA decision) in the motion for reconsideration of the DOLE Secretary's January 15, 2003
Order. 25 There is thus no showing that the DOLE Secretary denied respondent this basic constitutional right.
On the issue of liability, petitioner contends that PNB should be held liable to shoulder the CBA benefits
awarded to them by virtue of it being a company having full financial, managerial and functional control over respondent
as its subsidiary, and by reason of the unique "no loss, no profit" scheme implemented between respondent and PNB.
We are not persuaded.
Verily, what the petitioner is asking this Court to do is to pierce the veil of corporate fiction of respondent and
hold PNB (being the mother company) liable for the CBA benefits.
In Concept Builders, Inc. v. NLRC, 26 we explained the doctrine of piercing the corporate veil, as follows:
It is a fundamental principle of corporation law that a corporation is an entity separate and distinct
from its stockholders and from other corporations to which it may be connected. But, this separate and
distinct personality of a corporation is merely a fiction created by law for convenience and to promote
justice. So, when the notion of separate juridical personality is used to defeat public convenience, justify
wrong, protect fraud or defend crime, or is used as a device to defeat the labor laws, this separate
personality of the corporation may be disregarded or the veil of corporate fiction pierced. This is true
likewise when the corporation is merely an adjunct, a business conduit or an alter ego of another
corporation. SDEHIa
Also in Pantranco Employees Association (PEA-PTGWO) v. National Labor Relations Commission, 27 this
Court ruled:
Whether the separate personality of the corporation should be pierced hinges on obtaining facts
appropriately pleaded or proved. However, any piercing of the corporate veil has to be done with caution,
albeit the Court will not hesitate to disregard the corporate veil when it is misused or when necessary in
the interest of justice. After all, the concept of corporate entity was not meant to promote unfair
objectives.
Applying the doctrine to the case at bar, we find no reason to pierce the corporate veil of respondent and go
beyond its legal personality. Control, by itself, does not mean that the controlled corporation is a mere instrumentality or
a business conduit of the mother company. Even control over the financial and operational concerns of a subsidiary
company does not by itself call for disregarding its corporate fiction. There must be a perpetuation of fraud behind the
control or at least a fraudulent or illegal purpose behind the control in order to justify piercing the veil of corporate fiction.
Such fraudulent intent is lacking in this case.
Petitioner argues that the appreciation, analysis and inquiry of this case may go beyond the presentation of
respondent, and therefore must include the PNB, the bank being the undisputed whole owner of respondent and the
sole provider of funds for the company's operations and for the payment of wages and benefits of the employees, under
the "no loss, no profit" scheme. 28
We disagree. There is no showing that such "no loss, no profit" scheme between respondent and PNB was
implemented to defeat public convenience, justify wrong, protect fraud or defend crime, or is used as a device to defeat
the labor laws, nor does the scheme show that respondent is a mere business conduit or alter ego of PNB. Absent proof
of these circumstances, respondent's corporate personality cannot be pierced.
It is apparent that petitioner wants the Court to disregard the corporate personality of respondent and directly
go after PNB in order for it to collect the CBA benefits. On the same breath, however, petitioner argues that ultimately it
is PNB, by virtue of the "no loss, no profit" scheme, which shoulders and provides the funds for financial liabilities of
respondent including wages and benefits of employees. If such scheme was indeed true as the petitioner presents it,
then there was absolutely no need to pierce the veil of corporate fiction of respondent. Moreover, the Court notes the
pendency of a separate suit for absorption or regularization of NASECO employees filed by petitioner and NEMU-PEMA
against PNB and respondent, docketed as NLRC NCR Case No. 06-03944-96), which is still on appeal with the
National Labor Relations Commission (NLRC), as per manifestation by respondent. In the said case, petitioner
submitted for resolution by the labor tribunal the issues of whether PNB is the employer of NASECO's work force and
whether NASECO is a labor-only contractor. 29 CHDTIS
WHEREFORE, the petition is PARTLY GRANTED. The Decision dated May 27, 2004 and Resolution dated
September 22, 2004 in CA-G.R. SP No. 76667 are hereby REVERSED and SET ASIDE as to the order to remand the
case to the Secretary of Labor for introduction of supporting evidence. Accordingly, the Orders of the Secretary of Labor
dated January 15, 2003 and March 11, 2003 are REINSTATED and UPHELD.
No costs. SO ORDERED.
||| (NASECO Guards Association-PEMA v. National Service Corp., G.R. No. 165442, [August 25, 2010], 643 PHIL 316-327)

[G.R. No. 199687. March 24, 2014.]

PACIFIC REHOUSE CORPORATION, petitioner, vs. COURT OF APPEALS and EXPORT AND
INDUSTRY BANK, INC., respondents.

[G.R. No. 201537. March 24, 2014.]

PACIFIC REHOUSE CORPORATION, PACIFIC CONCORDE CORPORATION, MIZPAH HOLDINGS,


INC., FORUM HOLDINGS CORPORATION and EAST ASIA OIL COMPANY, INC., petitioners, vs.
EXPORT AND INDUSTRY BANK, INC., respondent.

DECISION

REYES, J p:

On the scales of justice precariously lie the right of a prevailing party to his victor's cup, no more, no less; and the
right of a separate entity from being dragged by the ball and chain of the vanquished party.
The facts of this case as garnered from the Decision 1 dated April 26, 2012 of the Court of Appeals (CA) in CA-G.R.
SP No. 120979 are as follows:
We trace the roots of this case to a complaint instituted with the Makati City Regional Trial Court
(RTC), Branch 66, against EIB Securities, Inc. (E-Securities) for unauthorized sale of 32,180,000 DMCI
shares of private respondents Pacific Rehouse Corporation, Pacific Concorde Corporation, Mizpah
Holdings, Inc., Forum Holdings Corporation, and East Asia Oil Company, Inc. In its October 18, 2005
Resolution, the RTC rendered judgment on the pleadings. The fallo reads:
WHEREFORE, premises considered, judgment is hereby rendered directing the
defendant [E-Securities] to return the plaintiffs' [private respondents herein] 32,180,000 DMCI
shares, as of judicial demand. cTACIa
On the other hand, plaintiffs are directed to reimburse the defendant the amount of
[P]10,942,200.00, representing the buy back price of the 60,790,000 KPP shares of stocks at
[P]0.18 per share.
SO ORDERED. . . .
The Resolution was ultimately affirmed by the Supreme Court and attained finality.
When the Writ of Execution was returned unsatisfied, private respondents moved for the
issuance of an alias writ of execution to hold Export and Industry Bank, Inc. liable for the judgment
obligation as E-Securities is "a wholly-owned controlled and dominated subsidiary of Export and Industry
Bank, Inc., and is[,] thus[,] a mere alter ego and business conduit of the latter. E-Securities opposed the
motion[,] arguing that it has a corporate personality that is separate and distinct from petitioner. On July
27, 2011, private respondents filed their (1) Reply attaching for the first time a sworn statement executed
by Atty. Ramon F. Aviado, Jr., the former corporate secretary of petitioner and E-Securities, to support
their alter ego theory; and (2) Ex-Parte Manifestation alleging service of copies of the Writ of Execution
and Motion for Alias Writ of Execution on petitioner.
On July 29, 2011, the RTC concluded that E-Securities is a mere business conduit or alter ego of
petitioner, the dominant parent corporation, which justifies piercing of the veil of corporate fiction. The trial
court brushed aside E-Securities' claim of denial of due process on petitioner as ". . . case records show
that notices regarding these proceedings had been tendered to the latter, which refused to even receive
them. Clearly, [petitioner] had been sufficiently put on notice and afforded the chance to give its side[,]
yet[,] it chose not to." Thus, the RTC disposed as follows:
WHEREFORE, . . .,
Let an Alias Writ of Execution be issued relative to the above-entitled case and pursuant
to the RESOLUTION dated October 18, 2005 and to this Order directing defendant EIB
Securities, Inc., and/or Export and Industry Bank, Inc., to fully comply therewith.
The Branch Sheriff of this Court is directed to cause the immediate implementation of the
given alias writ in accordance with the Order of Execution to be issued anew by the Branch Clerk
of Court.
SO ORDERED. . . .
With this development, petitioner filed an Omnibus Motion (Ex Abundanti Cautela) questioning
the alias writ because it was not impleaded as a party to the case. The RTC denied the motion in its
Order dated August 26, 2011 and directed the garnishment of P1,465,799,000.00, the total amount of the
32,180,000 DMCI shares at P45.55 per share, against petitioner and/or E-Securities. 2 . . . . (Citations
omitted)
The Regional Trial Court (RTC) ratiocinated that being one and the same entity in the eyes of the law, the service of
summons upon EIB Securities, Inc. (E-Securities) has bestowed jurisdiction over both the parent and wholly-owned
subsidiary. 3 The RTC cited the cases of Sps. Violago v. BA Finance Corp. et al. 4 and Arcilla v. Court of Appeals 5 where
the doctrine of piercing the veil of corporate fiction was applied notwithstanding that the affected corporation was not
brought to the court as a party. Thus, the RTC held in its Order 6 dated August 26, 2011: EHaCTA
WHEREFORE, premises considered, the Motion for Reconsideration with Motion to Inhibit filed
by defendant EIB Securities, Inc. is denied for lack of merit. The Omnibus Motion Ex Abundanti C[au]tela
is likewise denied for lack of merit.
Pursuant to Rule 39, Section 10 (a) of the Rules of Court, the Branch Clerk of Court or the
Branch Sheriff of this Court is hereby directed to acquire 32,180,000 DMCI shares of stock from the
Philippine Stock Exchange at the cost of EIB Securities, Inc. and Export and Industry Bank[,] Inc. and to
deliver the same to the plaintiffs pursuant to this Court's Resolution dated October 18, 2005.
To implement this Order, let GARNISHMENT issue against ALL THOSE HOLDING MONEYS,
PROPERTIES OF ANY AND ALL KINDS, REAL OR PERSONAL BELONGING TO OR OWNED BY
DEFENDANT EIB SECURITIES, INC. AND/OR EXPORT AND INDUSTRY BANK[,] INC., [sic] in such
amount as may be sufficient to acquire 32,180,000 DMCI shares of stock to the Philippine Stock
Exchange, based on the closing price of Php45.55 per share of DMCI shares as of August 1, 2011, the
date of the issuance of the Alias Writ of Execution, or the total amount of PhP1,465,799,000.00.
SO ORDERED. 7
CA-G.R. SP No. 120979
Export and Industry Bank, Inc. (Export Bank) filed before the CA a petition for certiorari with prayer for the issuance
of a temporary restraining order (TRO) 8 seeking the nullification of the RTC Order dated August 26, 2011 for having been
made with grave abuse of discretion amounting to lack or excess of jurisdiction. In its petition, Export Bank made reference
to several rulings 9 of the Court upholding the separate and distinct personality of a corporation.
In a Resolution 10 dated September 2, 2011, the CA issued a 60-day TRO enjoining the execution of the Orders of
the RTC dated July 29, 2011 and August 26, 2011, which granted the issuance of an alias writ of execution and ordered the
garnishment of the properties of E-Securities and/or Export Bank. The CA also set a hearing to determine the necessity of
issuing a writ of injunction, viz.:
Considering the amount ordered to be garnished from petitioner Export and Industry Bank, Inc.
and the fiduciary duty of the banking institution to the public, there is grave and irreparable injury that
may be caused to [Export Bank] if the assailed Orders are immediately implemented. We thus resolve to
GRANT the Temporary Restraining Order effective for a period of sixty (60) days from notice,
restraining/enjoining the Sheriff of the Regional Trial Court of Makati City or his deputies, agents,
representatives or any person acting in their behalf from executing the July 29, 2011 and August 26,
2011 Orders. [Export Bank] is DIRECTED to POST a bond in the sum of fifty million pesos
(P50,000,000.00) within ten (10) days from notice, to answer for any damage which private respondents
may suffer by reason of this Temporary Restraining Order; otherwise, the same shall automatically
become ineffective.
Let the HEARING be set on September 27, 2011 at 2:00 in the afternoon at the Paras Hall, Main
Building, Court of Appeals, to determine the necessity of issuing a writ of preliminary injunction. The
Division Clerk of Court is DIRECTED to notify the parties and their counsel with dispatch.
xxx xxx xxx
SO ORDERED. 11
Pacific Rehouse Corporation (Pacific Rehouse), Pacific Concorde Corporation, Mizpah Holdings, Inc., Forum
Holdings Corporation and East Asia Oil Company, Inc. (petitioners) filed their Comment 12 to Export Bank's petition and
proffered that the cases mentioned by Export Bank are inapplicable owing to their clearly different factual antecedents. The
petitioners alleged that unlike the other cases, there are circumstances peculiar only to E-Securities and Export Bank such
as: 499,995 out of 500,000 outstanding shares of stocks of E-Securities are owned by Export Bank; 13 Export Bank had
actual knowledge of the subject matter of litigation as the lawyers who represented E-Securities are also lawyers of Export
Bank. 14 As an alter ego, there is no need for a finding of fraud or illegality before the doctrine of piercing the veil of
corporate fiction can be applied. 15
After oral arguments before the CA, the parties were directed to file their respective memoranda. 16
On October 25, 2011, the CA issued a Resolution, 17 granting Export Bank's application for the issuance of a writ
of preliminary injunction, viz.: ADcSHC
WHEREFORE, finding [Export Bank's] application for the ancillary injunctive relief to be
meritorious, and it further appearing that there is urgency and necessity in restraining the same, a Writ of
Preliminary Injunction is hereby GRANTED and ISSUED against the Sheriff of the Regional Trial Court of
Makati City, Branch 66, or his deputies, agents, representatives or any person acting in their behalf from
executing the July 29, 2011 and August 26, 2011 Orders. Public respondents are ordered to CEASE and
DESIST from enforcing and implementing the subject orders until further notice from this Court. 18
The petitioners filed a Manifestation 19 and Supplemental Manifestation 20 challenging the above-quoted CA
resolution for lack of concurrence of Associate Justice Socorro B. Inting (Justice Inting), who was then on official leave.
On December 22, 2011, the CA, through a Special Division of Five, issued another Resolution, 21 which reiterated
the Resolution dated October 25, 2011 granting the issuance of a writ of preliminary injunction.
On January 2, 2012, one of the petitioners herein, Pacific Rehouse filed before the Court a petition for certiorari 22
under Rule 65, docketed as G.R. No. 199687, demonstrating its objection to the Resolutions dated October 25, 2011 and
December 22, 2011 of the CA.
On April 26, 2012, the CA rendered the assailed Decision 23 on the merits of the case, granting Export Bank's
petition. The CA disposed of the case in this wise:
We GRANT the petition. The Orders dated July 29, 2011 and August 26, 2011 of the Makati City
Regional Trial Court, Branch 66, insofar as [Export Bank] is concerned, are NULLIFIED. The Writ of
Preliminary Injunction (WPI) is rendered PERMANENT.
SO ORDERED. 24
The CA explained that the alter ego theory cannot be sustained because ownership of a subsidiary by the parent
company is not enough justification to pierce the veil of corporate fiction. There must be proof, apart from mere ownership,
that Export Bank exploited or misused the corporate fiction of E-Securities. The existence of interlocking incorporators,
directors and officers between the two corporations is not a conclusive indication that they are one and the same. 25 The
records also do not show that Export Bank has complete control over the business policies, affairs and/or transactions of E-
Securities. It was solely E-Securities that contracted the obligation in furtherance of its legitimate corporate purpose; thus,
any fall out must be confined within its limited liability. 26
The petitioners, without filing a motion for reconsideration, filed a Petition for Review 27 under Rule 45 docketed as
G.R. No. 201537, impugning the Decision dated April 26, 2012 of the CA.
Considering that G.R. Nos. 199687 and 201537 originated from the same set of facts, involved the same parties
and raised intertwined issues, the cases were then consolidated. 28
Issues
In précis, the issues for resolution of this Court are the following:
In G.R. No. 199687,
WHETHER THE CA COMMITTED GRAVE ABUSE OF DISCRETION IN GRANTING EXPORT
BANK'S APPLICATION FOR THE ISSUANCE OF A WRIT OF PRELIMINARY INJUNCTION. DASCIc
In G.R. No. 201537,
I.
WHETHER THE CA COMMITTED A REVERSIBLE ERROR IN RULING THAT EXPORT BANK MAY
NOT BE HELD LIABLE FOR A FINAL AND EXECUTORY JUDGMENT AGAINST E-SECURITIES IN
AN ALIAS WRIT OF EXECUTION BY PIERCING ITS VEIL OF CORPORATE FICTION; and
II.
WHETHER THE CA COMMITTED A REVERSIBLE ERROR IN RULING THAT THE ALTER EGO
DOCTRINE IS NOT APPLICABLE.
Ruling of the Court
G.R. No. 199687
The Resolution dated October 25, 2011 was initially challenged by the petitioners in its Manifestation 29 and
Supplemental Manifestation 30 due to the lack of concurrence of Justice Inting, which according to the petitioners rendered
the aforesaid resolution null and void.
To the petitioners' mind, Section 5, Rule VI of the Internal Rules of the CA (IRCA) 31 requires the submission of the
resolution granting an application for TRO or preliminary injunction to the absent Justice/s when they report back to work for
ratification, modification or recall, such that when the absent Justice/s do not agree with the issuance of the TRO or
preliminary injunction, the resolution is recalled and without force and effect. 32 Since the resolution which granted the
application for preliminary injunction appears short of the required number of consensus, owing to the absence of Justice
Inting's signature, the petitioners contest the validity of said resolution.
The petitioners also impugn the CA Resolution dated December 22, 2011 rendered by the Special Division of Five.
The petitioners maintain that pursuant to Batas Pambansa Bilang 129 33 and the IRCA, 34 such division is created only
when the three members of a division cannot reach a unanimous vote in deciding a case on the merits. 35 Furthermore, for
petitioner Pacific Rehouse, this Resolution is likewise infirm because the purpose of the formation of the Special Division of
Five is to decide the case on the merits and not to grant Export Bank's application for a writ of preliminary injunction. 36
We hold that the opposition to the CA resolutions is already nugatory because the CA has already rendered its
Decision on April 16, 2012, which disposed of the substantial merits of the case. Consequently, the petitioners' concern that
the Special Division of Five should have been created to resolve cases on the merits has already been addressed by the
rendition of the CA Decision dated April 16, 2012.
"It is well-settled that courts will not determine questions that have become moot and academic because there is no
longer any justiciable controversy to speak of. The judgment will not serve any useful purpose or have any practical legal
effect because, in the nature of things, it cannot be enforced." 37 In such cases, there is no actual substantial relief to which
the petitioners would be entitled to and which would be negated by the dismissal of the petition. 38 Thus, it would be futile
and pointless to address the issue in G.R. No. 199687 as this has become moot and academic.
G.R. No. 201537
The petitioners bewail that the certified true copy of the CA Decision dated April 26, 2012 along with its Certification
at the bottom portion were not signed by the Chairperson 39 of the Special Division of Five; thus, it is not binding upon the
parties. 40 The petitioners quoted this Court's pronouncement in Limkaichong v. Commission on Elections, 41 that a
decision must not only be signed by the Justices who took part in the deliberation, but must also be promulgated to be
considered a Decision. 42 aHIEcS
A cursory glance on a copy of the signature page 43 of the decision attached to the records would show that,
indeed, the same was not signed by CA Associate Justice Magdangal M. de Leon. However, it must be noted that the CA,
on May 7, 2012, issued a Resolution 44 explaining that due to inadvertence, copies of the decision not bearing the
signature of the Chairperson were sent to the parties on the same day of promulgation. The CA directed the Division Clerk
of Court to furnish the parties with copies of the signature page with the Chairperson's signature. Consequently, as the
mistake was immediately clarified and remedied by the CA, the lack of the Chairperson's signature on the copies sent to the
parties has already become a non-issue.
It must be emphasized that the instant cases sprang from Pacific Rehouse Corporation v. EIB Securities, Inc. 45
which was decided by this Court last October 13, 2010. Significantly, Export Bank was not impleaded in said case but was
unexpectedly included during the execution stage, in addition to E-Securities, against whom the writ of execution may be
enforced in the Order 46 dated July 29, 2011 of the RTC. In including Export Bank, the RTC considered E-Securities as a
mere business conduit of Export Bank. 47 Thus, one of the arguments interposed by the latter in its Opposition 48 that it
was never impleaded as a defendant was simply set aside.
This action by the RTC begs the question: may the RTC enforce the alias writ of execution against Export Bank?
The question posed before us is not novel.
The Court already ruled in Kukan International Corporation v. Reyes 49 that compliance with the recognized modes
of acquisition of jurisdiction cannot be dispensed with even in piercing the veil of corporate fiction, to wit:
The principle of piercing the veil of corporate fiction, and the resulting treatment of two related
corporations as one and the same juridical person with respect to a given transaction, is basically applied
only to determine established liability; it is not available to confer on the court a jurisdiction it has not
acquired, in the first place, over a party not impleaded in a case. Elsewise put, a corporation not
impleaded in a suit cannot be subject to the court's process of piercing the veil of its corporate
fiction. In that situation, the court has not acquired jurisdiction over the corporation and, hence, any
proceedings taken against that corporation and its property would infringe on its right to due process.
Aguedo Agbayani, a recognized authority on Commercial Law, stated as much:
"23. Piercing the veil of corporate entity applies to determination of liability not of
jurisdiction. . . .
This is so because the doctrine of piercing the veil of corporate fiction comes to
play only during the trial of the case after the court has already acquired jurisdiction over
the corporation. Hence, before this doctrine can be applied, based on the evidence presented,
it is imperative that the court must first have jurisdiction over the corporation. . . ." 50 (Citations
omitted)
From the preceding, it is therefore correct to say that the court must first and foremost acquire jurisdiction over the
parties; and only then would the parties be allowed to present evidence for and/or against piercing the veil of corporate
fiction. If the court has no jurisdiction over the corporation, it follows that the court has no business in piercing its veil of
corporate fiction because such action offends the corporation's right to due process.
"Jurisdiction over the defendant is acquired either upon a valid service of summons or the defendant's voluntary
appearance in court. When the defendant does not voluntarily submit to the court's jurisdiction or when there is no valid
service of summons, 'any judgment of the court which has no jurisdiction over the person of the defendant is null and void.'"
51 "The defendant must be properly apprised of a pending action against him and assured of the opportunity to present his
defenses to the suit. Proper service of summons is used to protect one's right to due process." 52 DCcTHa
As Export Bank was neither served with summons, nor has it voluntarily appeared before the court, the judgment
sought to be enforced against E-Securities cannot be made against its parent company, Export Bank. Export Bank has
consistently disputed the RTC jurisdiction, commencing from its filing of an Omnibus Motion 53 by way of special
appearance during the execution stage until the filing of its Comment 54 before the Court wherein it was pleaded that "RTC
[of] Makati[, Branch] 66 never acquired jurisdiction over Export [B]ank. Export [B]ank was not pleaded as a party in this
case. It was never served with summons by nor did it voluntarily appear before RTC [of] Makati[, Branch] 66 so as to be
subjected to the latter's jurisdiction." 55
In dispensing with the requirement of service of summons or voluntary appearance of Export Bank, the RTC applied
the cases of Violago and Arcilla. The RTC concluded that in these cases, the Court decided that the doctrine of piercing the
veil of corporate personality can be applied even when one of the affected parties has not been brought to the Court as a
party. 56
A closer perusal on the rulings of this Court in Violago and Arcilla, however, reveals that the RTC misinterpreted the
doctrines on these cases. We agree with the CA that these cases are not congruent to the case at bar. In Violago, Spouses
Pedro and Florencia Violago (Spouses Violago) filed a third party complaint against their cousin Avelino Violago (Avelino),
who is also the president of Violago Motor Sales Corporation (VMSC), for selling them a vehicle which was already sold to
someone else. VMSC was not impleaded as a third party defendant. Avelino contended that he was not a party to the
transaction personally, but VMSC. The Court ruled that "[t]he fact that VMSC was not included as defendant in [Spouses
Violago's] third party complaint does not preclude recovery by Spouses Violago from Avelino; neither would such non-
inclusion constitute a bar to the application of the piercing-of-the-corporate-veil doctrine." 57 It should be pointed out that
although VMSC was not made a third party defendant, the person who was found liable in Violago, Avelino, was properly
made a third party defendant in the first instance. The present case could not be any more poles apart from Violago,
because Export Bank, the parent company which was sought to be accountable for the judgment against E-Securities, is not
a party to the main case.
In Arcilla, meanwhile, Calvin Arcilla (Arcilla) obtained a loan in the name of Csar Marine Resources, Inc. (CMRI)
from Emilio Rodulfo. A complaint was then filed against Arcilla for non-payment of the loan. CMRI was not impleaded as a
defendant. The trial court eventually ordered Arcilla to pay the judgment creditor for such loan. Arcilla argued that he is not
personally liable for the adjudged award because the same constitutes a corporate liability which cannot even bind the
corporation as the latter is not a party to the collection suit. The Court made the succeeding observations:
[B]y no stretch of even the most fertile imagination may one be able to conclude that the challenged
Amended Decision directed Csar Marine Resources, Inc. to pay the amounts adjudged. By its clear
and unequivocal language, it is the petitioner who was declared liable therefor and consequently made
to pay. . . ., even if We are to assume arguendo that the obligation was incurred in the name of the
corporation, the petitioner would still be personally liable therefor because for all legal intents and
purposes, he and the corporation are one and the same. Csar Marine Resources, Inc. is nothing more
than his business conduit and alter ego. The fiction of a separate juridical personality conferred upon
such corporation by law should be disregarded. . . . . 58 (Citation omitted)
It is important to bear in mind that although CMRI was not a party to the suit, it was Arcilla, the defendant himself
who was found ultimately liable for the judgment award. CMRI and its properties were left untouched from the main case,
not only because of the application of the alter ego doctrine, but also because it was never made a party to that case.
AScTaD
The disparity between the instant case and those of Violago and Arcilla is that in said cases, although the
corporations were not impleaded as defendant, the persons made liable in the end were already parties thereto since the
inception of the main case. Consequently, it cannot be said that the Court had, in the absence of fraud and/or bad faith,
applied the doctrine of piercing the veil of corporate fiction to make a non-party liable. In short, liabilities attached only to
those who are parties. None of the non-party corporations (VMSC and CMRI) were made liable for the judgment award
against Avelino and Arcilla.
The Alter Ego Doctrine is not
applicable
"The question of whether one corporation is merely an alter ego of another is purely one of fact. So is the question
of whether a corporation is a paper company, a sham or subterfuge or whether petitioner adduced the requisite quantum of
evidence warranting the piercing of the veil of respondent's corporate entity." 59
As a rule, the parties may raise only questions of law under Rule 45, because the Supreme Court is not a trier of
facts. Generally, we are not duty-bound to analyze again and weigh the evidence introduced in and considered by the
tribunals below. 60 However, justice for all is of primordial importance that the Court will not think twice of reviewing the
facts, more so because the RTC and the CA arrived in contradicting conclusions.
"It is a fundamental principle of corporation law that a corporation is an entity separate and distinct from its
stockholders and from other corporations to which it may be connected. But, this separate and distinct personality of a
corporation is merely a fiction created by law for convenience and to promote justice. So, when the notion of separate
juridical personality is used to defeat public convenience, justify wrong, protect fraud or defend crime, or is used as a device
to defeat the labor laws, this separate personality of the corporation may be disregarded or the veil of corporate fiction
pierced. This is true likewise when the corporation is merely an adjunct, a business conduit or an alter ego of another
corporation." 61
"Where one corporation is so organized and controlled and its affairs are conducted so that it is, in fact, a mere
instrumentality or adjunct of the other, the fiction of the corporate entity of the "instrumentality" may be disregarded. The
control necessary to invoke the rule is not majority or even complete stock control but such domination of finances, policies
and practices that the controlled corporation has, so to speak, no separate mind, will or existence of its own, and is but a
conduit for its principal. It must be kept in mind that the control must be shown to have been exercised at the time the acts
complained of took place. Moreover, the control and breach of duty must proximately cause the injury or unjust loss for
which the complaint is made." 62
The Court has laid down a three-pronged control test to establish when the alter ego doctrine should be operative:
(1) Control, not mere majority or complete stock control, but complete domination, not only of finances
but of policy and business practice in respect to the transaction attacked so that the corporate entity as
to this transaction had at the time no separate mind, will or existence of its own;
(2) Such control must have been used by the defendant to commit fraud or wrong, to perpetuate the
violation of a statutory or other positive legal duty, or dishonest and unjust act in contravention of
plaintiff's legal right; and
(3) The aforesaid control and breach of duty must [have] proximately caused the injury or unjust loss
complained of. 63
The absence of any one of these elements prevents 'piercing the corporate veil' in applying the 'instrumentality' or
'alter ego' doctrine, the courts are concerned with reality and not form, with how the corporation operated and the individual
defendant's relationship to that operation. 64 Hence, all three elements should concur for the alter ego doctrine to be
applicable. DCcAIS
In its decision, the RTC maintained that the subsequently enumerated factors betray the true nature of E-Securities
as a mere alter ego of Export Bank:
1. Defendant EIB Securities, a subsidiary corporation 100% totally owned by Export and Industry
Bank, Inc., was only re-activated by the latter in 2002-2003 and the continuance of its operations was
geared for no other reason tha[n] to serve as the securities brokerage arm of said parent
corporation bank;
2. It was the parent corporation bank that provided and infused the fresh working cash capital
needed by defendant EIB Securities which prior thereto was non-operating and severely cash-strapped.
[This was so attested by the then Corporate Secretary of both corporations, Atty. Ramon Aviado, Jr., in
his submitted Sworn Statement which is deemed allowable "evidence on motion", under Sec. 7, Rule
133, Rules on Evidence; Bravo vs. Borja, 134 SCRA 438];
3. For effective control purposes, defendant EIB Securities and its operating office and staff are
all housed in Exportbank Plaza located at Chino Roces cor. Sen. Gil Puyat Avenue, Makati City which is
the same building w[h]ere the bank parent corporation has its headquarters;
4. As shown in the General Information Sheets annually filed with the S.E.C. from 2002 to 2011,
both defendant EIB Securities and the bank parent corporation share common key Directors and
corporate officers. Three of the 5-man Board of Directors of defendant EIB Securities are Directors of the
bank parent corporation, namely: Jaime C. Gonzales, Pauline C. Tan and Dionisio E. Carpio, Jr. In
addition, Mr. Gonzales is Chairman of the Board of both corporations, whereas Pauline C. Tan is
concurrently President/General Manager of EIB Securities, and Dionisio Carpio Jr., is not only director of
the bank, but also Director Treasurer of defendant EIB Securities;
5. As admitted by the bank parent corporation in its consolidated audited financial statements[,]
EIB Securities is a CONTROLLED SUBSIDIARY, and for which reason its financial condition and results
of operations are included and integrated as part of the group's consolidated financial statements,
examined and audited by the same auditing firm;
6. The lawyers handling the suits and legal matters of defendant EIB Securities are the same
lawyers in the Legal Department of the bank parent corporation. The Court notes that in [the] above-
entitled suit, the lawyers who at the start represented said defendant EIB Securities and filed all the
pleadings and filings in its behalf are also the lawyers in the Legal Services Division of the bank parent
corporation. They are Attys. Emmanuel A. Silva, Leonardo C. Bool, Riva Khristine E. Maala and Ma.
Esmeralda R. Cunanan, all of whom worked at the Legal Services Division of Export Industry Bank
located at 36/F, Exportbank Plaza, Don Chino Roces Avenue, cor. Sen. Gil Puyat Avenue, Makati City.
7. Finally[,] and this is very significant, the control and sway that the bank parent corporation held
over defendant EIB Securities was prevailing in June 2004 when the very act complained of in plaintiff's
Complaint took place, namely the unauthorized disposal of the 32,180,000 DMCI shares of stock. Being
then under the direction and control of the bank parent corporation, the unauthorized disposal of those
shares by defendant EIB Securities is attributable to, and the responsibility of the former. 65
All the foregoing circumstances, with the exception of the admitted stock ownership, were however not properly
pleaded and proved in accordance with the Rules of Court. 66 These were merely raised by the petitioners for the first time
in their Motion for Issuance of an Alias Writ of Execution 67 and Reply, 68 which the Court cannot consider. "Whether the
separate personality of the corporation should be pierced hinges on obtaining facts appropriately pleaded or proved." 69
DIETcH
Albeit the RTC bore emphasis on the alleged control exercised by Export Bank upon its subsidiary E-Securities,
"[c]ontrol, by itself, does not mean that the controlled corporation is a mere instrumentality or a business conduit of the
mother company. Even control over the financial and operational concerns of a subsidiary company does not by itself call
for disregarding its corporate fiction. There must be a perpetuation of fraud behind the control or at least a fraudulent or
illegal purpose behind the control in order to justify piercing the veil of corporate fiction. Such fraudulent intent is lacking in
this case." 70
Moreover, there was nothing on record demonstrative of Export Bank's wrongful intent in setting up a subsidiary, E-
Securities. If used to perform legitimate functions, a subsidiary's separate existence shall be respected, and the liability of
the parent corporation as well as the subsidiary will be confined to those arising in their respective business. 71 To justify
treating the sole stockholder or holding company as responsible, it is not enough that the subsidiary is so organized and
controlled as to make it "merely an instrumentality, conduit or adjunct" of its stockholders. It must further appear that to
recognize their separate entities would aid in the consummation of a wrong. 72
As established in the main case 73 and reiterated by the CA, the subject 32,180,000 DMCI shares which E-
Securities is obliged to return to the petitioners were originally bought at an average price of P0.38 per share and were sold
for an average price of P0.24 per share. The proceeds were then used to buy back 61,100,000 KPP shares earlier sold by
E-Securities. Quite unexpectedly however, the total amount of these DMCI shares ballooned to P1,465,799,000.00. 74 It
must be taken into account that this unexpected turnabout did not inure to the benefit of E-Securities, much less Export
Bank.
Furthermore, ownership by Export Bank of a great majority or all of stocks of E-Securities and the existence of
interlocking directorates may serve as badges of control, but ownership of another corporation, per se, without proof of
actuality of the other conditions are insufficient to establish an alter ego relationship or connection between the two
corporations, which will justify the setting aside of the cover of corporate fiction. The Court has declared that "mere
ownership by a single stockholder or by another corporation of all or nearly all of the capital stock of a corporation is not of
itself sufficient ground for disregarding the separate corporate personality." The Court has likewise ruled that the "existence
of interlocking directors, corporate officers and shareholders is not enough justification to pierce the veil of corporate fiction
in the absence of fraud or other public policy considerations." 75
While the courts have been granted the colossal authority to wield the sword which pierces through the veil of
corporate fiction, concomitant to the exercise of this power, is the responsibility to uphold the doctrine of separate entity,
when rightly so; as it has for so long encouraged businessmen to enter into economic endeavors fraught with risks and
where only a few dared to venture.
Hence, any application of the doctrine of piercing the corporate veil should be done with caution. A court should be
mindful of the milieu where it is to be applied. It must be certain that the corporate fiction was misused to such an extent that
injustice, fraud, or crime was committed against another, in disregard of its rights. The wrongdoing must be clearly and
convincingly established; it cannot be presumed. Otherwise, an injustice that was never unintended may result from an
erroneous application. 76
In closing, we understand that the petitioners are disgruntled at the turnout of this case — that they cannot enforce
the award due them on its entirety; however, the Court cannot supplant a remedy which is not sanctioned by our laws and
prescribed rules.
WHEREFORE, the petition in G.R. No. 199687 is hereby DISMISSED for having been rendered moot and
academic. The petition in G.R. No. 201537, meanwhile, is hereby DENIED for lack of merit. Consequently, the Decision
dated April 26, 2012 of the Court of Appeals in CA-G.R. SP No. 120979 is AFFIRMED. SO ORDERED.
||| (Pacific Rehouse Corp. v. Court of Appeals, G.R. Nos. 199687 & 201537, [March 24, 2014], 730 PHIL 325-353)

[G.R. No. 182729. September 29, 2010.]

KUKAN INTERNATIONAL CORPORATION, petitioner, vs. HON. AMOR REYES, in her capacity as
Presiding Judge of the Regional Trial Court of Manila, Branch 21, and ROMEO M. MORALES,
doing business under the name and style "RM Morales Trophies and Plaques," respondents.

DECISION

VELASCO, JR., J p:
The Case
This Petition for Review on Certiorari under Rule 45 seeks to nullify and reverse the January 23, 2008 Decision
1 and the April 16, 2008 Resolution 2 rendered by the Court of Appeals (CA) in CA-G.R. SP No. 100152. DETcAH
The assailed CA decision affirmed the March 12, 2007 3 and June 7, 2007 4 Orders of the Regional Trial Court
(RTC) of Manila, Branch 21, in Civil Case No. 99-93173, entitled Romeo M. Morales, doing business under the name
and style RM Morales Trophies and Plaques v. Kukan, Inc. In the said orders, the RTC disregarded the separate
corporate identities of Kukan, Inc. and Kukan International Corporation and declared them to be one and the same
entity. Accordingly, the RTC held Kukan International Corporation, albeit not impleaded in the underlying complaint of
Romeo M. Morales, liable for the judgment award decreed in a Decision dated November 28, 2002 5 in favor of Morales
and against Kukan, Inc.
The Facts
Sometime in March 1998, Kukan, Inc. conducted a bidding for the supply and installation of signages in a
building being constructed in Makati City. Morales tendered the winning bid and was awarded the PhP5 million contract.
Some of the items in the project award were later excluded resulting in the corresponding reduction of the contract price
to PhP3,388,502. Despite his compliance with his contractual undertakings, Morales was only paid the amount of
PhP1,976,371.07, leaving a balance of PhP1,412,130.93, which Kukan, Inc. refused to pay despite demands.
Shortchanged, Morales filed a Complaint 6 with the RTC against Kukan, Inc. for a sum of money, the case docketed as
Civil Case No. 99-93173 and eventually raffled to Branch 17 of the court.
Following the joinder of issues after Kukan, Inc. filed an answer with counterclaim, trial ensued. However,
starting November 2000, Kukan, Inc. no longer appeared and participated in the proceedings before the trial court,
prompting the RTC to declare Kukan, Inc. in default and paving the way for Morales to present his evidence ex parte.
On November 28, 2002, the RTC rendered a Decision finding for Morales and against Kukan, Inc., disposing as
follows:
WHEREFORE, consistent with Section 5, Rule 18 of the 1997 Rules of Civil Procedure, and by
preponderance of evidence, judgment is hereby rendered in favor of the plaintiff, ordering Kukan, Inc.:
1. to pay the sum of ONE MILLION TWO HUNDRED ONE THOUSAND SEVEN HUNDRED
TWENTY FOUR PESOS (P1,201,724.00) with legal interest at 12% per annum from
February 17, 1999 until full payment;
2. to pay the sum of FIFTY THOUSAND PESOS (P50,000.00) as moral damages;
3. to pay the sum of TWENTY THOUSAND PESOS (P20,000.00) as reasonable attorney's fees;
and
4. to pay the sum of SEVEN THOUSAND NINE HUNDRED SIXTY PESOS and SIX CENTAVOS
(P7,960.06) as litigation expenses.
For lack of factual foundation, the counterclaim is DISMISSED.
IT IS SO ORDERED. 7
After the above decision became final and executory, Morales moved for and secured a writ of execution 8
against Kukan, Inc. The sheriff then levied upon various personal properties found at what was supposed to be Kukan,
Inc.'s office at Unit 2205, 88 Corporate Center, Salcedo Village, Makati City. Alleging that it owned the properties thus
levied and that it was a different corporation from Kukan, Inc., Kukan International Corporation (KIC) filed an Affidavit of
Third-Party Claim. Notably, KIC was incorporated in August 2000, or shortly after Kukan, Inc. had stopped participating
in Civil Case No. 99-93173.
In reaction to the third party claim, Morales interposed an Omnibus Motion dated April 30, 2003. In it, Morales
prayed, applying the principle of piercing the veil of corporate fiction, that an order be issued for the satisfaction of the
judgment debt of Kukan, Inc. with the properties under the name or in the possession of KIC, it being alleged that both
corporations are but one and the same entity. KIC opposed Morales' motion. By Order of May 29, 2003 9 as reiterated
in a subsequent order, the court denied the omnibus motion.
In a bid to establish the link between KIC and Kukan, Inc., and thus determine the true relationship between the
two, Morales filed a Motion for Examination of Judgment Debtors dated May 4, 2005. In this motion Morales sought that
subpoena be issued against the primary stockholders of Kukan, Inc., among them Michael Chan, a.k.a. Chan Kai Kit.
This too was denied by the trial court in an Order dated May 24, 2005. 10
Morales then sought the inhibition of the presiding judge, Eduardo B. Peralta, Jr., who eventually granted the
motion. The case was re-raffled to Branch 21, presided by public respondent Judge Amor Reyes. AEITDH
Before the Manila RTC, Branch 21, Morales filed a Motion to Pierce the Veil of Corporate Fiction to declare KIC
as having no existence separate from Kukan, Inc. This time around, the RTC, by Order dated March 12, 2007, granted
the motion, the dispositive portion of which reads:
WHEREFORE, premises considered, the motion is hereby GRANTED. The Court hereby
declares as follows:
1. defendant Kukan, Inc. and newly created Kukan International Corp. as one and the
same corporation;
2. the levy made on the properties of Kukan International Corp. is hereby valid;
3. Kukan International Corp. and Michael Chan are jointly and severally liable to pay the
amount awarded to plaintiff pursuant to the decision of November [28], 2002
which has long been final and executory.
SO ORDERED.
From the above order, KIC moved but was denied reconsideration in another Order dated June 7, 2007.
KIC went to the CA on a petition for certiorari to nullify the aforesaid March 12 and June 7, 2007 RTC Orders.
On January 23, 2008, the CA rendered the assailed decision, the dispositive portion of which states:
WHEREFORE, premises considered, the petition is hereby DENIED and the assailed Orders
dated March 12, 2007 and June 7, 2007 of the court a quo are both AFFIRMED. No costs.
SO ORDERED. 11
The CA later denied KIC's motion for reconsideration in the assailed resolution.
Hence, the instant petition for review, with the following issues KIC raises for the Court's consideration:
1. There is no legal basis for the [CA] to resolve and declare that petitioner's Constitutional Right
to Due Process was not violated by the public respondent in rendering the Orders dated
March 12, 2007 and June 7, 2007 and in declaring petitioner to be liable for the judgment
obligations of the corporation "Kukan, Inc." to private respondent — as petitioner is a
stranger to the case and was never made a party in the case before the trial court nor
was it ever served a summons and a copy of the complaint.
2. There is no legal basis for the [CA] to resolve and declare that the Orders dated March 12,
2007 and June 7, 2007 rendered by public respondent declaring the petitioner liable to
the judgment obligations of the corporation "Kukan, Inc." to private respondent are valid
as said orders of the public respondent modify and/or amend the trial court's final and
executory decision rendered on November 28, 2002.
3. There is no legal basis for the [CA] to resolve and declare that the Orders dated March 12,
2007 and June 7, 2007 rendered by public respondent declaring the petitioner [KIC] and
the corporation "Kukan, Inc." as one and the same, and, therefore, the Veil of Corporate
Fiction between them be pierced — as the procedure undertaken by public respondent
which the [CA] upheld is not sanctioned by the Rules of Court and/or established
jurisprudence enunciated by this Honorable Supreme Court. 12
In gist, the issues to be resolved boil down to the question of, first, whether the trial court can, after the
judgment against Kukan, Inc. has attained finality, execute it against the property of KIC; second, whether the trial court
acquired jurisdiction over KIC; and third, whether the trial and appellate courts correctly applied, under the premises, the
principle of piercing the veil of corporate fiction.
The Ruling of the Court
The petition is meritorious.
First Issue: Against Whom Can a Final and
Executory Judgment Be Executed
The preliminary question that must be answered is whether or not the trial court can, after adjudging Kukan,
Inc. liable for a sum of money in a final and executory judgment, execute such judgment debt against the property of
KIC.
The poser must be answered in the negative.
In Carpio v. Doroja, 13 the Court ruled that the deciding court has supervisory control over the execution of its
judgment:
A case in which an execution has been issued is regarded as still pending so that all proceedings
on the execution are proceedings in the suit. There is no question that the court which rendered the
judgment has a general supervisory control over its process of execution, and this power carries with it
the right to determine every question of fact and law which may be involved in the execution. EaISTD
We reiterated the above holding in Javier v. Court of Appeals 14 in this wise: "The said branch has a general
supervisory control over its processes in the execution of its judgment with a right to determine every question of fact
and law which may be involved in the execution."
The court's supervisory control does not, however, extend as to authorize the alteration or amendment of a final
and executory decision, save for certain recognized exceptions, among which is the correction of clerical errors. Else,
the court violates the principle of finality of judgment and its immutability, concepts which the Court, in Tan v. Timbal, 15
defined:
As we held in Industrial Management International Development Corporation vs. NLRC:
It is an elementary principle of procedure that the resolution of the court in a given issue
as embodied in the dispositive part of a decision or order is the controlling factor as to settlement
of rights of the parties. Once a decision or order becomes final and executory, it is removed
from the power or jurisdiction of the court which rendered it to further alter or amend it. It
thereby becomes immutable and unalterable and any amendment or alteration which
substantially affects a final and executory judgment is null and void for lack of
jurisdiction, including the entire proceedings held for that purpose. An order of execution
which varies the tenor of the judgment or exceeds the terms thereof is a nullity. (Emphasis
supplied.)
Republic v. Tango 16 expounded on the same principle and its exceptions:
Deeply ingrained in our jurisprudence is the principle that a decision that has acquired finality
becomes immutable and unalterable. As such, it may no longer be modified in any respect even if
the modification is meant to correct erroneous conclusions of fact or law and whether it will be made by
the court that rendered it or by the highest court of the land. . . .
The doctrine of finality of judgment is grounded on the fundamental principle of public policy and
sound practice that, at the risk of occasional error, the judgment of courts and the award of quasi-judicial
agencies must become final on some definite date fixed by law. The only exceptions to the general rule
are the correction of clerical errors, the so-called nunc pro tunc entries which cause no prejudice to any
party, void judgments, and whenever circumstances transpire after the finality of the decision which
render its execution unjust and inequitable. None of the exceptions obtains here to merit the review
sought. (Emphasis added.)
So, did the RTC, in breach of the doctrine of immutability and inalterability of judgment, order the execution of
its final decision in a manner as would amount to its prohibited alteration or modification?
We repair to the dispositive portion of the final and executory RTC decision. Pertinently, it provides:
WHEREFORE, consistent with Section 5, Rule 18 of the 1997 Rules of Civil Procedure, and by
preponderance of evidence, judgment is hereby rendered in favor of the plaintiff, ordering Kukan, Inc.:
1. to pay the sum of ONE MILLION TWO HUNDRED ONE THOUSAND SEVEN
HUNDRED TWENTY FOUR PESOS (P1,201,724.00) with legal interest at 12%
per annum from February 17, 1999 until full payment;
2. to pay the sum of FIFTY THOUSAND PESOS (P50,000.00) as moral damages;
3. to pay the sum of TWENTY THOUSAND PESOS (P20,000.00) as reasonable
attorney's fees; and
4. to pay the sum of SEVEN THOUSAND NINE HUNDRED SIXTY PESOS and SIX
CENTAVOS (P7,960.06) as litigation expenses.
xxx xxx xxx (Emphasis supplied.)
As may be noted, the above decision, in unequivocal terms, directed Kukan, Inc. to pay the aforementioned
awards to Morales. Thus, making KIC, thru the medium of a writ of execution, answerable for the above judgment
liability is a clear case of altering a decision, an instance of granting relief not contemplated in the decision sought to be
executed. And the change does not fall under any of the recognized exceptions to the doctrine of finality and
immutability of judgment. It is a settled rule that a writ of execution must conform to the fallo of the judgment; as an
inevitable corollary, a writ beyond the terms of the judgment is a nullity. 17
Thus, on this ground alone, the instant petition can already be granted. Nonetheless, an examination of the
other issues raised by KIC would be proper. HSAcaE
Second Issue: Propriety of the RTC
Assuming Jurisdiction over KIC
The next issue turns on the validity of the execution the trial court authorized against KIC and its property, given
that it was neither made a party nor impleaded in Civil Case No. 99-93173, let alone served with summons. In other
words, did the trial court acquire jurisdiction over KIC?
In the assailed decision, the appellate court deemed KIC to have voluntarily submitted itself to the jurisdiction of
the trial court owing to its filing of four (4) pleadings adverted to earlier, namely: (a) the Affidavit of Third-Party Claim; 18
(b) the Comment and Opposition to Plaintiff's Omnibus Motion; 19 (c) the Motion for Reconsideration of the RTC Order
dated March 12, 2007; 20 and (d) the Motion for Leave to Admit Reply. 21 The CA, citing Section 20, Rule 14 of the
Rules of Court, stated that "the procedural rule on service of summons can be waived by voluntary submission to the
court's jurisdiction through any form of appearance by the party or its counsel." 22
We cannot give imprimatur to the appellate court's appreciation of the thrust of Sec. 20, Rule 14 of the Rules in
concluding that the trial court acquired jurisdiction over KIC.
Orion Security Corporation v. Kalfam Enterprises, Inc. 23 explains how courts acquire jurisdiction over the
parties in a civil case:
Courts acquire jurisdiction over the plaintiffs upon the filing of the complaint. On the other hand,
jurisdiction over the defendants in a civil case is acquired either through the service of summons
upon them or through their voluntary appearance in court and their submission to its authority.
(Emphasis supplied.)
In the fairly recent Palma v. Galvez, 24 the Court reiterated its holding in Orion Security Corporation, stating:
"[I]n civil cases, the trial court acquires jurisdiction over the person of the defendant either by the service of summons or
by the latter's voluntary appearance and submission to the authority of the former."
The court's jurisdiction over a party-defendant resulting from his voluntary submission to its authority is provided
under Sec. 20, Rule 14 of the Rules, which states:
Section 20. Voluntary appearance. — The defendant's voluntary appearance in the actions shall
be equivalent to service of summons. The inclusion in a motion to dismiss of other grounds aside from
lack of jurisdiction over the person of the defendant shall not be deemed a voluntary appearance.
To be sure, the CA's ruling that any form of appearance by the party or its counsel is deemed as voluntary
appearance finds support in the kindred Republic v. Ker & Co., Ltd. 25 and De Midgely v. Ferandos. 26
Republic and De Midgely, however, have already been modified if not altogether superseded 27 by La Naval
Drug Corporation v. Court of Appeals, 28 wherein the Court essentially ruled and elucidated on the current view in our
jurisdiction, to wit: "[A] special appearance before the court — challenging its jurisdiction over the person through a
motion to dismiss even if the movant invokes other grounds — is not tantamount to estoppel or a waiver by the movant
of his objection to jurisdiction over his person; and such is not constitutive of a voluntary submission to the jurisdiction of
the court." 29
In the instant case, KIC was not made a party-defendant in Civil Case No. 99-93173. Even if it is conceded that
it raised affirmative defenses through its aforementioned pleadings, KIC never abandoned its challenge, however
implicit, to the RTC's jurisdiction over its person. The challenge was subsumed in KIC's primary assertion that it was not
the same entity as Kukan, Inc. Pertinently, in its Comment and Opposition to Plaintiff's Omnibus Motion dated May 20,
2003, KIC entered its "special but not voluntary appearance" alleging therein that it was a different entity and has a
separate legal personality from Kukan, Inc. And KIC would consistently reiterate this assertion in all its pleadings, thus
effectively resisting all along the RTC's jurisdiction of its person. It cannot be overemphasized that KIC could not file
before the RTC a motion to dismiss and its attachments in Civil Case No. 99-93173, precisely because KIC was neither
impleaded nor served with summons. Consequently, KIC could only assert and claim through its affidavits, comments,
and motions filed by special appearance before the RTC that it is separate and distinct from Kukan, Inc.
Following La Naval Drug Corporation, 30 KIC cannot be deemed to have waived its objection to the court's lack
of jurisdiction over its person. It would defy logic to say that KIC unequivocally submitted itself to the jurisdiction of the
RTC when it strongly asserted that it and Kukan, Inc. are different entities. In the scheme of things obtaining, KIC had
no other option but to insist on its separate identity and plead for relief consistent with that position.
Third Issue: Piercing the
Veil of Corporate Fiction
The third and main issue in this case is whether or not the trial and appellate courts correctly applied the
principle of piercing the veil of corporate entity — called also as disregarding the fiction of a separate juridical
personality of a corporation — to support a conclusion that Kukan, Inc. and KIC are but one and the same corporation
with respect to the contract award referred to at the outset. This principle finds its context on the postulate that a
corporation is an artificial being invested with a personality separate and distinct from those of the stockholders and
from other corporations to which it may be connected or related. 31
In Pantranco Employees Association (PEA-PTGWO) v. National Labor Relations Commission, 32 the Court
revisited the subject principle of piercing the veil of corporate fiction and wrote:
Under the doctrine of "piercing the veil of corporate fiction," the court looks at the corporation as
a mere collection of individuals or an aggregation of persons undertaking business as a group,
disregarding the separate juridical personality of the corporation unifying the group. Another formulation
of this doctrine is that when two business enterprises are owned, conducted and controlled by the
same parties, both law and equity will, when necessary to protect the rights of third parties,
disregard the legal fiction that two corporations are distinct entities and treat them as identical or
as one and the same.
Whether the separate personality of the corporation should be pierced hinges on obtaining
facts appropriately pleaded or proved. However, any piercing of the corporate veil has to be done with
caution, albeit the Court will not hesitate to disregard the corporate veil when it is misused or when
necessary in the interest of justice. . . . (Emphasis supplied.)
The same principle was the subject and discussed in Rivera v. United Laboratories, Inc.:
While a corporation may exist for any lawful purpose, the law will regard it as an association of
persons or, in case of two corporations, merge them into one, when its corporate legal entity is
used as a cloak for fraud or illegality. This is the doctrine of piercing the veil of corporate fiction.
The doctrine applies only when such corporate fiction is used to defeat public convenience,
justify wrong, protect fraud, or defend crime, or when it is made as a shield to confuse the
legitimate issues, or where a corporation is the mere alter ego or business conduit of a person, or where
the corporation is so organized and controlled and its affairs are so conducted as to make it merely an
instrumentality, agency, conduit or adjunct of another corporation.
To disregard the separate juridical personality of a corporation, the wrongdoing must be
established clearly and convincingly. It cannot be presumed. 33 (Emphasis supplied.)
Now, as before the appellate court, petitioner KIC maintains that the RTC violated its right to due process when,
in the execution of its November 28, 2002 Decision, the court authorized the issuance of the writ against KIC for Kukan,
Inc.'s judgment debt, albeit KIC has never been a party to the underlying suit. As a counterpoint, Morales argues that
KIC's specific concern on due process and on the validity of the writ to execute the RTC's November 28, 2002 Decision
would be mooted if it were established that KIC and Kukan, Inc. are indeed one and the same corporation. HDCAaS
Morales' contention is untenable.
The principle of piercing the veil of corporate fiction, and the resulting treatment of two related corporations as
one and the same juridical person with respect to a given transaction, is basically applied only to determine established
liability; 34 it is not available to confer on the court a jurisdiction it has not acquired, in the first place, over a party not
impleaded in a case. Elsewise put, a corporation not impleaded in a suit cannot be subject to the court's process of
piercing the veil of its corporate fiction. In that situation, the court has not acquired jurisdiction over the corporation and,
hence, any proceedings taken against that corporation and its property would infringe on its right to due process.
Aguedo Agbayani, a recognized authority on Commercial Law, stated as much:
23. Piercing the veil of corporate entity applies to determination of liability not of jurisdiction. . . .
This is so because the doctrine of piercing the veil of corporate fiction comes to play only
during the trial of the case after the court has already acquired jurisdiction over the corporation.
Hence, before this doctrine can be applied, based on the evidence presented, it is imperative that the
court must first have jurisdiction over the corporation. 35 . . . (Emphasis supplied.)
The implication of the above comment is twofold: (1) the court must first acquire jurisdiction over the corporation
or corporations involved before its or their separate personalities are disregarded; and (2) the doctrine of piercing the
veil of corporate entity can only be raised during a full-blown trial over a cause of action duly commenced involving
parties duly brought under the authority of the court by way of service of summons or what passes as such service.
The issue of jurisdiction or the lack of it over KIC has already been discussed. Anent the matter of the time and
manner of raising the principle in question, it is undisputed that no full-blown trial involving KIC was had when the RTC
disregarded the corporate veil of KIC. The reason for this actuality is simple and undisputed: KIC was not impleaded in
Civil Case No. 99-93173 and that the RTC did not acquire jurisdiction over it. It was dragged to the case after it reacted
to the improper execution of its properties and veritably hauled to court, not thru the usual process of service of
summons, but by mere motion of a party with whom it has no privity of contract and after the decision in the main case
had already become final and executory. As to the propriety of a plea for the application of the principle by mere motion,
the following excerpts are instructive:
Generally, a motion is appropriate only in the absence of remedies by regular pleadings,
and is not available to settle important questions of law, or to dispose of the merits of the case. A
motion is usually a proceeding incidental to an action, but it may be a wholly distinct or independent
proceeding. A motion in this sense is not within this discussion even though the relief demanded is
denominated an "order."
A motion generally relates to procedure and is often resorted to in order to correct errors which
have crept in along the line of the principal action's progress. Generally, where there is a procedural
defect in a proceeding and no method under statute or rule of court by which it may be called to the
attention of the court, a motion is an appropriate remedy. In many jurisdictions, the motion has replaced
the common-law pleas testing the sufficiency of the pleadings, and various common-law writs, such as
writ of error coram nobis and audita querela. In some cases, a motion may be one of several remedies
available. For example, in some jurisdictions, a motion to vacate an order is a remedy alternative to an
appeal therefrom.
Statutes governing motions are given a liberal construction. 36 (Emphasis supplied.)
The bottom line issue of whether Morales can proceed against KIC for the judgment debt of Kukan, Inc. —
assuming hypothetically that he can, applying the piercing the corporate veil principle — resolves itself into the question
of whether a mere motion is the appropriate vehicle for such purpose.
Verily, Morales espouses the application of the principle of piercing the corporate veil to hold KIC liable on
theory that Kukan, Inc. was out to defraud him through the use of the separate and distinct personality of another
corporation, KIC. In net effect, Morales' adverted motion to pierce the veil of corporate fiction dated January 3, 2007
stated a new cause of action, i.e., for the liability of judgment debtor Kukan, Inc. to be borne by KIC on the alleged
identity of the two corporations. This new cause of action should be properly ventilated in another complaint and
subsequent trial where the doctrine of piercing the corporate veil can, if appropriate, be applied, based on the evidence
adduced. Establishing the claim of Morales and the corresponding liability of KIC for Kukan, Inc.'s indebtedness could
hardly be the subject, under the premises, of a mere motion interposed after the principal action against Kukan, Inc.
alone had peremptorily been terminated. After all, a complaint is one where the plaintiff alleges causes of action.
TaEIcS
In any event, the principle of piercing the veil of corporate fiction finds no application to the instant case.
As a general rule, courts should be wary of lifting the corporate veil between corporations, however related.
Philippine National Bank v. Andrada Electric Engineering Company 37 explains why:
A corporation is an artificial being created by operation of law. . . . It has a personality separate
and distinct from the persons composing it, as well as from any other legal entity to which it may be
related. This is basic.
Equally well-settled is the principle that the corporate mask may be removed or the corporate veil
pierced when the corporation is just an alter ego of a person or of another corporation. For reasons of
public policy and in the interest of justice, the corporate veil will justifiably be impaled only when it
becomes a shield for fraud, illegality or inequity committed against third persons.
Hence, any application of the doctrine of piercing the corporate veil should be done with
caution. A court should be mindful of the milieu where it is to be applied. It must be certain that the
corporate fiction was misused to such an extent that injustice, fraud, or crime was committed
against another, in disregard of its rights. The wrongdoing must be clearly and convincingly
established; it cannot be presumed. Otherwise, an injustice that was never unintended may result
from an erroneous application.
This Court has pierced the corporate veil to ward off a judgment credit, to avoid inclusion of
corporate assets as part of the estate of the decedent, to escape liability arising from a debt, or to
perpetuate fraud and/or confuse legitimate issues either to promote or to shield unfair objectives or to
cover up an otherwise blatant violation of the prohibition against forum-shopping. Only in these and
similar instances may the veil be pierced and disregarded. (Emphasis supplied.)
In fine, to justify the piercing of the veil of corporate fiction, it must be shown by clear and convincing proof that
the separate and distinct personality of the corporation was purposefully employed to evade a legitimate and binding
commitment and perpetuate a fraud or like wrongdoings. To be sure, the Court has, on numerous occasions, 38 applied
the principle where a corporation is dissolved and its assets are transferred to another to avoid a financial liability of the
first corporation with the result that the second corporation should be considered a continuation and successor of the
first entity.
In those instances when the Court pierced the veil of corporate fiction of two corporations, there was a
confluence of the following factors:
1. A first corporation is dissolved;
2. The assets of the first corporation is transferred to a second corporation to avoid a financial
liability of the first corporation; and
3. Both corporations are owned and controlled by the same persons such that the second
corporation should be considered as a continuation and successor of the first
corporation.
In the instant case, however, the second and third factors are conspicuously absent. There is, therefore, no
compelling justification for disregarding the fiction of corporate entity separating Kukan, Inc. from KIC. In applying the
principle, both the RTC and the CA miserably failed to identify the presence of the abovementioned factors. Consider:
The RTC disregarded the separate corporate personalities of Kukan, Inc. and KIC based on the following
premises and arguments:
While it is true that a corporation has a separate and distinct personality from its stockholder,
director and officers, the law expressly provides for an exception. When Michael Chan, the Managing
Director of defendant Kukan, Inc. (majority stockholder of the newly formed corporation [KIC]) confirmed
the award to plaintiff to supply and install interior signages in the Enterprise Center he (Michael Chan,
Managing Director of defendant Kukan, Inc.) knew that there was no sufficient corporate funds to pay its
obligation/account, thus implying bad faith on his part and fraud in contracting the obligation. Michael
Chan neither returned the interior signages nor tendered payment to the plaintiff. This circumstance may
warrant the piercing of the veil of corporation fiction. Having been guilty of bad faith in the
management of corporate matters the corporate trustee, director or officer may be held
personally liable. . . .
Since fraud is a state of mind, it need not be proved by direct evidence but may be inferred from
the circumstances of the case. . . . [A]nd the circumstances are: the signature of Michael Chan, Managing
Director of Kukan, Inc. appearing in the confirmation of the award sent to the plaintiff; signature of Chan
Kai Kit, a British National appearing in the Articles of Incorporation and signature of Michael Chan also a
British National appearing in the Articles of Incorporation [of] Kukan International Corp. give the
impression that they are one and the same person, that Michael Chan and Chan Kai Kit are both majority
stockholders of Kukan International Corp. and Kukan, Inc. holding 40% of the stocks; that Kukan
International Corp. is practically doing the same kind of business as that of Kukan, Inc. 39 (Emphasis
supplied.)
As is apparent from its disquisition, the RTC brushed aside the separate corporate existence of Kukan, Inc. and
KIC on the main argument that Michael Chan owns 40% of the common shares of both corporations, obviously
oblivious that overlapping stock ownership is a common business phenomenon. It must be remembered, however, that
KIC's properties were the ones seized upon levy on execution and not that of Kukan, Inc. or of Michael Chan for that
matter. Mere ownership by a single stockholder or by another corporation of a substantial block of shares of a
corporation does not, standing alone, provide sufficient justification for disregarding the separate corporate personality.
40 For this ground to hold sway in this case, there must be proof that Chan had control or complete dominion of Kukan
and KIC's finances, policies, and business practices; he used such control to commit fraud; and the control was the
proximate cause of the financial loss complained of by Morales. The absence of any of the elements prevents the
piercing of the corporate veil. 41 And indeed, the records do not show the presence of these elements.
On the other hand, the CA held:
In the present case, the facts disclose that Kukan, Inc. entered into a contractual obligation . . .
worth more than three million pesos although it had only Php5,000.00 paid-up capital; [KIC] was
incorporated shortly before Kukan, Inc. suddenly ceased to appear and participate in the trial; [KIC's]
purpose is related and somewhat akin to that of Kukan, Inc.; and in [KIC] Michael Chan, a.k.a. Chan Kai
Kit, holds forty percent of the outstanding stocks, while he formerly held the same amount of stocks in
Kukan, Inc. These would lead to the inescapable conclusion that Kukan, Inc. committed fraudulent
representation by awarding to the private respondent the contract with full knowledge that it was
not in a position to comply with the obligation it had assumed because of inadequate paid-up
capital. It bears stressing that shareholders should in good faith put at the risk of the business,
unencumbered capital reasonably adequate for its prospective liabilities. The capital should not be
illusory or trifling compared with the business to be done and the risk of loss. HIETAc
Further, it is clear that [KIC] is a continuation and successor of Kukan, Inc. Michael Chan, a.k.a.
Chan Kai Kit has the largest block of shares in both business enterprises. The emergence of the former
was cleverly timed with the hasty withdrawal of the latter during the trial to avoid the financial liability that
was eventually suffered by the latter. The two companies have a related business purpose. Considering
these circumstances, the obvious conclusion is that the creation of Kukan International
Corporation served as a device to evade the obligation incurred by Kukan, Inc. and yet profit from
the goodwill attained by the name "Kukan" by continuing to engage in the same line of business
with the same list of clients. 42 (Emphasis supplied.)
Evidently, the CA found the meager paid-up capitalization of Kukan, Inc. and the similarity of the business
activities in which both corporations are engaged as a jumping board to its conclusion that the creation of KIC "served
as a device to evade the obligation incurred by Kukan, Inc." The appellate court, however, left a gaping hole by failing to
demonstrate that Kukan, Inc. and its stockholders defrauded Morales. In fine, there is no showing that the incorporation,
and the separate and distinct personality, of KIC was used to defeat Morales' right to recover from Kukan, Inc. Judging
from the records, no serious attempt was made to levy on the properties of Kukan, Inc. Morales could not, thus, validly
argue that Kukan, Inc. tried to avoid liability or had no property against which to proceed.
Morales further contends that Kukan, Inc.'s closure is evidenced by its failure to file its 2001 General
Information Sheet (GIS) with the Securities and Exchange Commission. However, such fact does not necessarily mean
that Kukan, Inc. had altogether ceased operations, as Morales would have this Court believe, for it is stated on the face
of the GIS that it is only upon a failure to file the corporate GIS for five (5) consecutive years that non-operation shall
be presumed.
The fact that Kukan, Inc. entered into a PhP3.3 million contract when it only had a paid-up capital of PhP5,000
is not an indication of the intent on the part of its management to defraud creditors. Paid-up capital is merely seed
money to start a corporation or a business entity. As in this case, it merely represented the capitalization upon
incorporation in 1997 of Kukan, Inc. Paid-up capitalization of PhP5,000 is not and should not be taken as a reflection
of the firm's capacity to meet its recurrent and long-term obligations. It must be borne in mind that the equity portion
cannot be equated to the viability of a business concern, for the best test is the working capital which consists of the
liquid assets of a given business relating to the nature of the business concern.
Neither should the level of paid-up capital of Kukan, Inc. upon its incorporation be viewed as a badge of fraud,
for it is in compliance with Sec. 13 of the Corporation Code, 43 which only requires a minimum paid-up capital of
PhP5,000.
The suggestion that KIC is but a continuation and successor of Kukan, Inc., owned and controlled as they are
by the same stockholders, stands without factual basis. It is true that Michael Chan, a.k.a. Chan Kai Kit, owns 40% of
the outstanding capital stock of both corporations. But such circumstance, standing alone, is insufficient to establish
identity. There must be at least a substantial identity of stockholders for both corporations in order to consider this factor
to be constitutive of corporate identity.
It would not avail Morales any to rely 44 on General Credit Corporation v. Alsons Development and Investment
Corporation. 45 General Credit Corporation is factually not on all fours with the instant case. There, the common
stockholders of the corporations represented 90% of the outstanding capital stock of the companies, unlike here where
Michael Chan merely represents 40% of the outstanding capital stock of both KIC and Kukan, Inc., not even a majority
of it. In that case, moreover, evidence was adduced to support the finding that the funds of the second corporation
came from the first. Finally, there was proof in General Credit Corporation of complete control, such that one
corporation was a mere dummy or alter ego of the other, which is absent in the instant case.
Evidently, the aforementioned case relied upon by Morales cannot justify the application of the principle of
piercing the veil of corporate fiction to the instant case. As shown by the records, the name Michael Chan, the similarity
of business activities engaged in, and incidentally the word "Kukan" appearing in the corporate names provide the
nexus between Kukan, Inc. and KIC. As illustrated, these circumstances are insufficient to establish the identity of KIC
as the alter ego or successor of Kukan, Inc.
It bears reiterating that piercing the veil of corporate fiction is frowned upon. Accordingly, those who seek to
pierce the veil must clearly establish that the separate and distinct personalities of the corporations are set up to justify
a wrong, protect fraud, or perpetrate a deception. In the concrete and on the assumption that the RTC has validly
acquired jurisdiction over the party concerned, Morales ought to have proved by convincing evidence that Kukan, Inc.
was collapsed and thereafter KIC purposely formed and operated to defraud him. Morales has not to us discharged his
burden. SICDAa
WHEREFORE, the petition is hereby GRANTED. The CA's January 23, 2008 Decision and April 16, 2008
Resolution in CA-G.R. SP No. 100152 are hereby REVERSED and SET ASIDE. The levy placed upon the personal
properties of Kukan International Corporation is hereby ordered lifted and the personal properties ordered returned to
Kukan International Corporation. The RTC of Manila, Branch 21 is hereby directed to execute the RTC Decision dated
November 28, 2002 against Kukan, Inc. with reasonable dispatch.
No costs. SO ORDERED.
||| (Kukan International Corporation v. Reyes, G.R. No. 182729, [September 29, 2010], 646 PHIL 210-243)

[G.R. No. 167530. March 13, 2013.]

PHILIPPINE NATIONAL BANK, petitioner, vs. HYDRO RESOURCES CONTRACTORS


CORPORATION, respondent.

[G.R. No. 167561. March 13, 2013.]

ASSET PRIVATIZATION TRUST, petitioner, vs. HYDRO RESOURCES CONTRACTORS


CORPORATION, respondent.

[G.R. No. 167603. March 13, 2013.]


DEVELOPMENT BANK OF THE PHILIPPINES, petitioner, vs. HYDRO RESOURCES CONTRACTORS
CORPORATION, respondent.

DECISION

LEONARDO-DE CASTRO, J p:

These petitions for review on certiorari 1 assail the Decision 2 dated November 30, 2004 and the Resolution 3
dated March 22, 2005 of the Court of Appeals in CA-G.R. CV No. 57553. The said Decision affirmed the Decision 4 dated
November 6, 1995 of the Regional Trial Court (RTC) of Makati City, Branch 62, granting a judgment award of
P8,370,934.74, plus legal interest, in favor of respondent Hydro Resources Contractors Corporation (HRCC) with the
modification that the Privatization and Management Office (PMO), successor of petitioner Asset Privatization Trust (APT), 5
has been held solidarily liable with Nonoc Mining and Industrial Corporation (NMIC) 6 and petitioners Philippine National
Bank (PNB) and Development Bank of the Philippines (DBP), while the Resolution denied reconsideration separately
prayed for by PNB, DBP, and APT.
Sometime in 1984, petitioners DBP and PNB foreclosed on certain mortgages made on the properties of
Marinduque Mining and Industrial Corporation (MMIC). As a result of the foreclosure, DBP and PNB acquired substantially
all the assets of MMIC and resumed the business operations of the defunct MMIC by organizing NMIC. 7 DBP and PNB
owned 57% and 43% of the shares of NMIC, respectively, except for five qualifying shares. 8 As of September 1984, the
members of the Board of Directors of NMIC, namely, Jose Tengco, Jr., Rolando Zosa, Ruben Ancheta, Geraldo Agulto, and
Faustino Agbada, were either from DBP or PNB. 9
Subsequently, NMIC engaged the services of Hercon, Inc., for NMIC's Mine Stripping and Road Construction
Program in 1985 for a total contract price of P35,770,120. After computing the payments already made by NMIC under the
program and crediting the NMIC's receivables from Hercon, Inc., the latter found that NMIC still has an unpaid balance of
P8,370,934.74. 10 Hercon, Inc. made several demands on NMIC, including a letter of final demand dated August 12, 1986,
and when these were not heeded, a complaint for sum of money was filed in the RTC of Makati, Branch 136 seeking to hold
petitioners NMIC, DBP, and PNB solidarily liable for the amount owing Hercon, Inc. 11 The case was docketed as Civil
Case No. 15375. EHScCA
Subsequent to the filing of the complaint, Hercon, Inc. was acquired by HRCC in a merger. This prompted the
amendment of the complaint to substitute HRCC for Hercon, Inc. 12
Thereafter, on December 8, 1986, then President Corazon C. Aquino issued Proclamation No. 50 creating the APT
for the expeditious disposition and privatization of certain government corporations and/or the assets thereof. Pursuant to
the said Proclamation, on February 27, 1987, DBP and PNB executed their respective deeds of transfer in favor of the
National Government assigning, transferring and conveying certain assets and liabilities, including their respective stakes in
NMIC. 13 In turn and on even date, the National Government transferred the said assets and liabilities to the APT as trustee
under a Trust Agreement. 14 Thus, the complaint was amended for the second time to implead and include the APT as a
defendant.
In its answer, 15 NMIC claimed that HRCC had no cause of action. It also asserted that its contract with HRCC was
entered into by its then President without any authority. Moreover, the said contract allegedly failed to comply with laws,
rules and regulations concerning government contracts. NMIC further claimed that the contract amount was manifestly
excessive and grossly disadvantageous to the government. NMIC made counterclaims for the amounts already paid to
Hercon, Inc. and attorney's fees, as well as payment for equipment rental for four trucks, replacement of parts and other
services, and damage to some of NMIC's properties. 16
For its part, DBP's answer 17 raised the defense that HRCC had no cause of action against it because DBP was
not privy to HRCC's contract with NMIC. Moreover, NMIC's juridical personality is separate from that of DBP. DBP further
interposed a counterclaim for attorney's fees. 18
PNB's answer 19 also invoked lack of cause of action against it. It also raised estoppel on HRCC's part and laches
as defenses, claiming that the inclusion of PNB in the complaint was the first time a demand for payment was made on it by
HRCC. PNB also invoked the separate juridical personality of NMIC and made counterclaims for moral damages and
attorney's fees. 20
APT set up the following defenses in its answer: 21 lack of cause of action against it, lack of privity between
Hercon, Inc. and APT, and the National Government's preferred lien over the assets of NMIC. 22
After trial, the RTC of Makati rendered a Decision dated November 6, 1995 in favor of HRCC. It pierced the
corporate veil of NMIC and held DBP and PNB solidarily liable with NMIC: IaHSCc
On the issue of whether or not there is sufficient ground to pierce the veil of corporate fiction, this
Court likewise finds for the plaintiff.
From the documentary evidence adduced by the plaintiff, some of which were even adopted by
defendants and DBP and PNB as their own evidence (Exhibits "I", "I-1", "I-2", "I-3", "I-4", "I-5", "I-5-A", "I-
5-B", "I-5-C", "I-5-D" and submarkings, inclusive), it had been established that except for five (5)
qualifying shares, [NMIC] is owned by defendants DBP and PNB, with the former owning 57% thereof,
and the latter 43%. As of September 24, 1984, all the members of [NMIC]'s Board of Directors, namely,
Messrs. Jose Tengco, Jr., Rolando M. Zosa, Ruben Ancheta, Geraldo Agulto, and Faustino Agbada are
either from DBP or PNB (Exhibits "I-5", "I-5-C", "I-5-D").
The business of [NMIC] was then also being conducted and controlled by both DBP and PNB. In
fact, it was Rolando M. Zosa, then Governor of DBP, who was signing and entering into contracts with
third persons, on behalf of [NMIC].
In this jurisdiction, it is well-settled that "where it appears that the business enterprises are
owned, conducted and controlled by the same parties, both law and equity will, when necessary to
protect the rights of third persons, disregard legal fiction that two (2) corporations are distinct entities, and
treat them as identical." (Phil. Veterans Investment Development Corp. vs. CA, 181 SCRA 669).
From all indications, it appears that [NMIC] is a mere adjunct, business conduit or alter ego of
both DBP and PNB. Thus, the DBP and PNB are jointly and severally liable with [NMIC] for the latter's
unpaid obligations to plaintiff. 23
Having found DBP and PNB solidarily liable with NMIC, the dispositive portion of the Decision of the trial court
reads:
WHEREFORE, in view of the foregoing, judgment is hereby rendered in favor of the plaintiff
HYDRO RESOURCES CONTRACTORS CORPORATION and against the defendant[s] NONOC
MINING AND INDUSTRIAL CORPORATION, DEVELOPMENT BANK OF THE PHILIPPINES and
PHILIPPINE NATIONAL BANK, ordering the aforenamed defendants, to pay the plaintiff jointly and
severally, the sum of P8,370,934.74 plus legal interest thereon from date of demand, and attorney's fees
equivalent to 25% of the judgment award. aIcSED
The complaint against APT is hereby dismissed. However, APT, as trustee of NONOC MINING
AND INDUSTRIAL CORPORATION is directed to ensure compliance with this Decision. 24
DBP and PNB filed their respective appeals in the Court of Appeals. Both insisted that it was wrong for the RTC to
pierce the veil of NMIC's corporate personality and hold DBP and PNB solidarily liable with NMIC. 25
The Court of Appeals rendered the Decision dated November 30, 2004, affirmed the piercing of the veil of the
corporate personality of NMIC and held DBP, PNB, and APT solidarily liable with NMIC. In particular, the Court of Appeals
made the following findings:
In the case before Us, it is indubitable that [NMIC] was owned by appellants DBP and PNB to the
extent of 57% and 43% respectively; that said two (2) appellants are the only stockholders, with the
qualifying stockholders of five (5) consisting of its own officers and included in its charter merely to
comply with the requirement of the law as to number of incorporators; and that the directorates of DBP,
PNB and [NMIC] are interlocked.
xxx xxx xxx
We find it therefore correct for the lower court to have ruled that:
"From all indications, it appears that [NMIC] is a mere adjunct, business conduit or alter
ego of both DBP and PNB. Thus, the DBP and PNB are jointly and severally liable with [NMIC]
for the latter's unpaid obligation to plaintiff." 26 (Citation omitted.)
The Court of Appeals then concluded that, "in keeping with the concept of justice and fair play," the corporate veil of
NMIC should be pierced, ratiocinating:
For to treat [NMIC] as a separate legal entity from DBP and PNB for the purpose of securing beneficial
contracts, and then using such separate entity to evade the payment of a just debt, would be the height
of injustice and iniquity. Surely that could not have been the intendment of the law with respect to
corporations. . . . . 27
The dispositive portion of the Decision of the Court of Appeals reads: CDcaSA
WHEREFORE, premises considered, the Decision appealed from is hereby MODIFIED. The
judgment in favor of appellee Hydro Resources Contractors Corporation in the amount of P8,370,934.74
with legal interest from date of demand is hereby AFFIRMED, but the dismissal of the case as against
Assets Privatization Trust is REVERSED, and its successor the Privatization and Management Office is
INCLUDED as one of those jointly and severally liable for such indebtedness. The award of attorney's
fees is DELETED.
All other claims and counter-claims are hereby DISMISSED.
Costs against appellants. 28
The respective motions for reconsideration of DBP, PNB, and APT were denied. 29
Hence, these consolidated petitions. 30
All three petitioners assert that NMIC is a corporate entity with a juridical personality separate and distinct from both
PNB and DBP. They insist that the majority ownership by DBP and PNB of NMIC is not a sufficient ground for disregarding
the separate corporate personality of NMIC because NMIC was not a mere adjunct, business conduit or alter ego of DBP
and PNB. According to them, the application of the doctrine of piercing the corporate veil is unwarranted as nothing in the
records would show that the ownership and control of the shareholdings of NMIC by DBP and PNB were used to commit
fraud, illegality or injustice. In the absence of evidence that the stock control by DBP and PNB over NMIC was used to
commit some fraud or a wrong and that said control was the proximate cause of the injury sustained by HRCC, resort to the
doctrine of "piercing the veil of corporate entity" is misplaced. 31
DBP and PNB further argue that, assuming they may be held solidarily liable with NMIC to pay NMIC's exclusive
and separate corporate indebtedness to HRCC, such liability of the two banks was transferred to and assumed by the
National Government through the APT, now the PMO, under the respective deeds of transfer both dated February 27, 1997
executed by DBP and PNB pursuant to Proclamation No. 50 dated December 8, 1986 and Administrative Order No. 14
dated February 3, 1987. 32
For its part, the APT contends that, in the absence of an unqualified assumption by the National Government of all
liabilities incurred by NMIC, the National Government through the APT could not be held liable for NMIC's contractual
liability. The APT asserts that HRCC had not sufficiently shown that the APT is the successor-in-interest of all the liabilities
of NMIC, or of DBP and PNB as transferors, and that the adjudged liability is included among the liabilities assigned and
transferred by DBP and PNB in favor of the National Government. 33
HRCC counters that both the RTC and the CA correctly applied the doctrine of "piercing the veil of corporate
fiction." It claims that NMIC was the alter ego of DBP and PNB which owned, conducted and controlled the business of
NMIC as shown by the following circumstances: NMIC was owned by DBP and PNB, the officers of DBP and PNB were
also the officers of NMIC, and DBP and PNB financed the operations of NMIC. HRCC further argues that a parent
corporation may be held liable for the contracts or obligations of its subsidiary corporation where the latter is a mere agency,
instrumentality or adjunct of the parent corporation. 34 ICcaST
Moreover, HRCC asserts that the APT was properly held solidarily liable with DBP, PNB, and NMIC because the
APT assumed the obligations of DBP and PNB as the successor-in-interest of the said banks with respect to the assets and
liabilities of NMIC. 35 As trustee of the Republic of the Philippines, the APT also assumed the responsibility of the Republic
pursuant to the following provision of Section 2.02 of the respective deeds of transfer executed by DBP and PNB in favor of
the Republic:
SECTION 2. TRANSFER OF BANK'S LIABILITIES. —
xxx xxx xxx
2.02 With respect to the Bank's liabilities which are contingent and those liabilities where the Bank's
creditors consent to the transfer thereof is not obtained, said liabilities shall remain in the books of the
BANK with the GOVERNMENT funding the payment thereof. 36
After a careful review of the case, this Court finds the petitions impressed with merit.
A corporation is an artificial entity created by operation of law. It possesses the right of succession and such
powers, attributes, and properties expressly authorized by law or incident to its existence. 37 It has a personality separate
and distinct from that of its stockholders and from that of other corporations to which it may be connected. 38 As a
consequence of its status as a distinct legal entity and as a result of a conscious policy decision to promote capital
formation, 39 a corporation incurs its own liabilities and is legally responsible for payment of its obligations. 40 In other
words, by virtue of the separate juridical personality of a corporation, the corporate debt or credit is not the debt or credit of
the stockholder. 41 This protection from liability for shareholders is the principle of limited liability. 42
Equally well-settled is the principle that the corporate mask may be removed or the corporate veil pierced when the
corporation is just an alter ego of a person or of another corporation. For reasons of public policy and in the interest of
justice, the corporate veil will justifiably be impaled only when it becomes a shield for fraud, illegality or inequity committed
against third persons. 43
However, the rule is that a court should be careful in assessing the milieu where the doctrine of the corporate veil
may be applied. Otherwise an injustice, although unintended, may result from its erroneous application. 44 Thus, cutting
through the corporate cover requires an approach characterized by due care and caution: CAcIES
Hence, any application of the doctrine of piercing the corporate veil should be done with
caution. A court should be mindful of the milieu where it is to be applied. It must be certain that the
corporate fiction was misused to such an extent that injustice, fraud, or crime was committed
against another, in disregard of its rights. The wrongdoing must be clearly and convincingly
established; it cannot be presumed. . . . . 45 (Emphases supplied; citations omitted.)
Sarona v. National Labor Relations Commission 46 has defined the scope of application of the doctrine of piercing
the corporate veil:
The doctrine of piercing the corporate veil applies only in three (3) basic areas, namely: 1) defeat
of public convenience as when the corporate fiction is used as a vehicle for the evasion of an existing
obligation; 2) fraud cases or when the corporate entity is used to justify a wrong, protect fraud, or defend
a crime; or 3) alter ego cases, where a corporation is merely a farce since it is a mere alter ego or
business conduit of a person, or where the corporation is so organized and controlled and its affairs are
so conducted as to make it merely an instrumentality, agency, conduit or adjunct of another corporation.
(Citation omitted.)
Here, HRCC has alleged from the inception of this case that DBP and PNB (and the APT as assignee of DBP and
PNB) should be held solidarily liable for using NMIC as alter ego. 47 The RTC sustained the allegation of HRCC and
pierced the corporate veil of NMIC pursuant to the alter ego theory when it concluded that NMIC "is a mere adjunct,
business conduit or alter ego of both DBP and PNB." 48 The Court of Appeals upheld such conclusion of the trial court. 49
In other words, both the trial and appellate courts relied on the alter ego theory when they disregarded the separate
corporate personality of NMIC.
In this connection, case law lays down a three-pronged test to determine the application of the alter ego theory,
which is also known as the instrumentality theory, namely:
(1) Control, not mere majority or complete stock control, but complete domination, not only of finances
but of policy and business practice in respect to the transaction attacked so that the corporate entity as
to this transaction had at the time no separate mind, will or existence of its own; aASEcH
(2) Such control must have been used by the defendant to commit fraud or wrong, to perpetuate the
violation of a statutory or other positive legal duty, or dishonest and unjust act in contravention of
plaintiff's legal right; and
(3) The aforesaid control and breach of duty must have proximately caused the injury or unjust loss
complained of. 50 (Emphases omitted.)
The first prong is the "instrumentality" or "control" test. This test requires that the subsidiary be completely under the
control and domination of the parent. 51 It examines the parent corporation's relationship with the subsidiary. 52 It inquires
whether a subsidiary corporation is so organized and controlled and its affairs are so conducted as to make it a mere
instrumentality or agent of the parent corporation such that its separate existence as a distinct corporate entity will be
ignored. 53 It seeks to establish whether the subsidiary corporation has no autonomy and the parent corporation, though
acting through the subsidiary in form and appearance, "is operating the business directly for itself." 54
The second prong is the "fraud" test. This test requires that the parent corporation's conduct in using the subsidiary
corporation be unjust, fraudulent or wrongful. 55 It examines the relationship of the plaintiff to the corporation. 56 It
recognizes that piercing is appropriate only if the parent corporation uses the subsidiary in a way that harms the plaintiff
creditor. 57 As such, it requires a showing of "an element of injustice or fundamental unfairness." 58
The third prong is the "harm" test. This test requires the plaintiff to show that the defendant's control, exerted in a
fraudulent, illegal or otherwise unfair manner toward it, caused the harm suffered. 59 A causal connection between the
fraudulent conduct committed through the instrumentality of the subsidiary and the injury suffered or the damage incurred by
the plaintiff should be established. The plaintiff must prove that, unless the corporate veil is pierced, it will have been treated
unjustly by the defendant's exercise of control and improper use of the corporate form and, thereby, suffer damages. 60
To summarize, piercing the corporate veil based on the alter ego theory requires the concurrence of three
elements: control of the corporation by the stockholder or parent corporation, fraud or fundamental unfairness imposed on
the plaintiff, and harm or damage caused to the plaintiff by the fraudulent or unfair act of the corporation. The absence of
any of these elements prevents piercing the corporate veil. 61
This Court finds that none of the tests has been satisfactorily met in this case. aCHDST
In applying the alter ego doctrine, the courts are concerned with reality and not form, with how the corporation
operated and the individual defendant's relationship to that operation. 62 With respect to the control element, it refers not to
paper or formal control by majority or even complete stock control but actual control which amounts to "such domination of
finances, policies and practices that the controlled corporation has, so to speak, no separate mind, will or existence of its
own, and is but a conduit for its principal." 63 In addition, the control must be shown to have been exercised at the time the
acts complained of took place. 64
Both the RTC and the Court of Appeals applied the alter ego theory and penetrated the corporate cover of NMIC
based on two factors: (1) the ownership by DBP and PNB of effectively all the stocks of NMIC, and (2) the alleged
interlocking directorates of DBP, PNB and NMIC. 65 Unfortunately, the conclusion of the trial and appellate courts that the
DBP and PNB fit the alter ego theory with respect to NMIC's transaction with HRCC on the premise of complete stock
ownership and interlocking directorates involved a quantum leap in logic and law exposing a gap in reason and fact.
While ownership by one corporation of all or a great majority of stocks of another corporation and their interlocking
directorates may serve as indicia of control, by themselves and without more, however, these circumstances are insufficient
to establish an alter ego relationship or connection between DBP and PNB on the one hand and NMIC on the other hand,
that will justify the puncturing of the latter's corporate cover. This Court has declared that "mere ownership by a single
stockholder or by another corporation of all or nearly all of the capital stock of a corporation is not of itself sufficient ground
for disregarding the separate corporate personality." 66 This Court has likewise ruled that the "existence of interlocking
directors, corporate officers and shareholders is not enough justification to pierce the veil of corporate fiction in the absence
of fraud or other public policy considerations." 67
True, the findings of fact of the Court of Appeals are conclusive and cannot be reviewed on appeal to this Court,
provided they are borne out of the record or are based on substantial evidence. 68 It is equally true that the question of
whether one corporation is merely an alter ego of another is purely one of fact. So is the question of whether a corporation
is a paper company, a sham or subterfuge or whether the requisite quantum of evidence has been adduced warranting the
piercing of the veil of corporate personality. 69 Nevertheless, it has been held in Sarona v. National Labor Relations
Commission 70 that this Court has the power to resolve a question of fact, such as whether a corporation is a mere alter
ego of another entity or whether the corporate fiction was invoked for fraudulent or malevolent ends, if the findings in the
assailed decision are either not supported by the evidence on record or based on a misapprehension of facts. aTAEHc
In this case, nothing in the records shows that the corporate finances, policies and practices of NMIC were
dominated by DBP and PNB in such a way that NMIC could be considered to have no separate mind, will or existence of its
own but a mere conduit for DBP and PNB. On the contrary, the evidence establishes that HRCC knew and acted on the
knowledge that it was dealing with NMIC, not with NMIC's stockholders. The letter proposal of Hercon, Inc., HRCC's
predecessor-in-interest, regarding the contract for NMIC's mine stripping and road construction program was addressed to
and accepted by NMIC. 71 The various billing reports, progress reports, statements of accounts and communications of
Hercon, Inc./HRCC regarding NMIC's mine stripping and road construction program in 1985 concerned NMIC and NMIC's
officers, without any indication of or reference to the control exercised by DBP and/or PNB over NMIC's affairs, policies and
practices. 72
HRCC has presented nothing to show that DBP and PNB had a hand in the act complained of, the alleged undue
disregard by NMIC of the demands of HRCC to satisfy the unpaid claims for services rendered by HRCC in connection with
NMIC's mine stripping and road construction program in 1985. On the contrary, the overall picture painted by the evidence
offered by HRCC is one where HRCC was dealing with NMIC as a distinct juridical person acting through its own corporate
officers. 73
Moreover, the finding that the respective boards of directors of NMIC, DBP, and PNB were interlocking has no
basis. HRCC's Exhibit "I-5," 74 the initial General Information Sheet submitted by NMIC to the Securities and Exchange
Commission, relied upon by the trial court and the Court of Appeals may have proven that DBP and PNB owned the stocks
of NMIC to the extent of 57% and 43%, respectively. However, nothing in it supports a finding that NMIC, DBP, and PNB
had interlocking directors as it only indicates that, of the five members of NMIC's board of directors, four were nominees of
either DBP or PNB and only one was a nominee of both DBP and PNB. 75 Only two members of the board of directors of
NMIC, Jose Tengco, Jr. and Rolando Zosa, were established to be members of the board of governors of DBP and none
was proved to be a member of the board of directors of PNB. 76 No director of NMIC was shown to be also sitting
simultaneously in the board of governors/directors of both DBP and PNB.
In reaching its conclusion of an alter ego relationship between DBP and PNB on the one hand and NMIC on the
other hand, the Court of Appeals invoked Sibagat Timber Corporation v. Garcia, 77 which it described as "a case under a
similar factual milieu." 78 However, in Sibagat Timber Corporation, this Court took care to enumerate the circumstances
which led to the piercing of the corporate veil of Sibagat Timber Corporation for being the alter ego of Del Rosario & Sons
Logging Enterprises, Inc. Those circumstances were as follows: holding office in the same building, practical identity of the
officers and directors of the two corporations and assumption of management and control of Sibagat Timber Corporation by
the directors/officers of Del Rosario & Sons Logging Enterprises, Inc. cCaEDA
Here, DBP and PNB maintain an address different from that of NMIC. 79 As already discussed, there was
insufficient proof of interlocking directorates. There was not even an allegation of similarity of corporate officers. Instead of
evidence that DBP and PNB assumed and controlled the management of NMIC, HRCC's evidence shows that NMIC
operated as a distinct entity endowed with its own legal personality. Thus, what obtains in this case is a factual backdrop
different from, not similar to, Sibagat Timber Corporation.
In relation to the second element, to disregard the separate juridical personality of a corporation, the wrongdoing or
unjust act in contravention of a plaintiff's legal rights must be clearly and convincingly established; it cannot be presumed.
Without a demonstration that any of the evils sought to be prevented by the doctrine is present, it does not apply. 80
In this case, the Court of Appeals declared:
We are not saying that PNB and DBP are guilty of fraud in forming [NMIC], nor are we implying
that [NMIC] was used to conceal fraud. . . . . 81
Such a declaration clearly negates the possibility that DBP and PNB exercised control over NMIC which DBP and
PNB used "to commit fraud or wrong, to perpetuate the violation of a statutory or other positive legal duty, or dishonest and
unjust act in contravention of plaintiff's legal rights." It is a recognition that, even assuming that DBP and PNB exercised
control over NMIC, there is no evidence that the juridical personality of NMIC was used by DBP and PNB to commit a fraud
or to do a wrong against HRCC.
There being a total absence of evidence pointing to a fraudulent, illegal or unfair act committed against HRCC by
DBP and PNB under the guise of NMIC, there is no basis to hold that NMIC was a mere alter ego of DBP and PNB. As this
Court ruled in Ramoso v. Court of Appeals: 82
As a general rule, a corporation will be looked upon as a legal entity, unless and until sufficient
reason to the contrary appears. When the notion of legal entity is used to defeat public convenience,
justify wrong, protect fraud, or defend crime, the law will regard the corporation as an association of
persons. Also, the corporate entity may be disregarded in the interest of justice in such cases as fraud
that may work inequities among members of the corporation internally, involving no rights of the public or
third persons. In both instances, there must have been fraud, and proof of it. For the separate juridical
personality of a corporation to be disregarded, the wrongdoing must be clearly and convincingly
established. It cannot be presumed. ScAaHE
As regards the third element, in the absence of both control by DBP and PNB of NMIC and fraud or fundamental
unfairness perpetuated by DBP and PNB through the corporate cover of NMIC, no harm could be said to have been
proximately caused by DBP and PNB on HRCC for which HRCC could hold DBP and PNB solidarily liable with NMIC.
Considering that, under the deeds of transfer executed by DBP and PNB, the liability of the APT as transferee of
the rights, titles and interests of DBP and PNB in NMIC will attach only if DBP and PNB are held liable, the APT incurs no
liability for the judgment indebtedness of NMIC. Even HRCC recognizes that "as assignee of DBP and PNB's loan
receivables," the APT simply "stepped into the shoes of DBP and PNB with respect to the latter's rights and obligations" in
NMIC. 83 As such assignee, therefore, the APT incurs no liability with respect to NMIC other than whatever liabilities may
be imputable to its assignors, DBP and PNB.
Even under Section 2.02 of the respective deeds of transfer executed by DBP and PNB which HRCC invokes, the
APT cannot be held liable. The contingent liability for which the National Government, through the APT, may be held liable
under the said provision refers to contingent liabilities of DBP and PNB. Since DBP and PNB may not be held solidarily
liable with NMIC, no contingent liability may be imputed to the APT as well. Only NMIC as a distinct and separate legal
entity is liable to pay its corporate obligation to HRCC in the amount of P8,370,934.74, with legal interest thereon from date
of demand.
As trustee of the assets of NMIC, however, the APT should ensure compliance by NMIC of the judgment against it.
The APT itself acknowledges this. 84
WHEREFORE, the petitions are hereby GRANTED.
The complaint as against Development Bank of the Philippines, the Philippine National Bank, and the Asset
Privatization Trust, now the Privatization and Management Office, is DISMISSED for lack of merit. The Asset Privatization
Trust, now the Privatization and Management Office, as trustee of Nonoc Mining and Industrial Corporation, now the
Philnico Processing Corporation, is DIRECTED to ensure compliance by the Nonoc Mining and Industrial Corporation, now
the Philnico Processing Corporation, with this Decision. AaIDHS
SO ORDERED.
||| (Philippine National Bank v. Hydro Resources Contractors Corp., G.R. Nos. 167530, 167561 & 167603, [March 13, 2013],
706 PHIL 297-319)

[G.R. No. 156759. June 5, 2013.]

ALLEN A. MACASAET, NICOLAS V. QUIJANO, JR., ISAIAS ALBANO, LILY REYES, JANET BAY,
JESUS R. GALANG, AND RANDY HAGOS, petitioners, vs. FRANCISCO R. CO, JR., respondent.

DECISION

BERSAMIN, J p:

To warrant the substituted service of the summons and copy of the complaint, the serving officer must first attempt
to effect the same upon the defendant in person. Only after the attempt at personal service has become futile or impossible
within a reasonable time may the officer resort to substituted service.
The Case
Petitioners-defendants in a suit for libel brought by respondent — appeal the decision promulgated on March 8,
2002 1 and the resolution promulgated on January 13, 2003, 2 whereby the Court of Appeals (CA) respectively dismissed
their petition for certiorari, prohibition and mandamus and denied their motion for reconsideration. Thereby, the CA upheld
the order the Regional Trial Court (RTC), Branch 51, in Manila had issued on March 12, 2001 denying their motion to
dismiss because the substituted service of the summons and copies of the complaint on each of them had been valid and
effective. 3
Antecedents
On July 3, 2000, respondent, a retired police officer assigned at the Western Police District in Manila, sued
AbanteTonite, a daily tabloid of general circulation; its Publisher Allen A. Macasaet; its Managing Director Nicolas V.
Quijano; its Circulation Manager Isaias Albano; its Editors Janet Bay, Jesus R. Galang and Randy Hagos; and its
Columnist/Reporter Lily Reyes (petitioners), claiming damages because of an allegedly libelous article petitioners published
in the June 6, 2000 issue of AbanteTonite. The suit, docketed as Civil Case No. 00-97907, was raffled to Branch 51 of the
RTC, which in due course issued summons to be served on each defendant, including AbanteTonite, at their business
address at Monica Publishing Corporation, 301-305 3rd Floor, BF Condominium Building, Solana Street corner A. Soriano
Street, Intramuros, Manila. 4 TSaEcH
In the morning of September 18, 2000, RTC Sheriff Raul Medina proceeded to the stated address to effect the
personal service of the summons on the defendants. But his efforts to personally serve each defendant in the address were
futile because the defendants were then out of the office and unavailable. He returned in the afternoon of that day to make a
second attempt at serving the summons, but he was informed that petitioners were still out of the office. He decided to
resort to substituted service of the summons, and explained why in his sheriff's return dated September 22, 2000, 5 to wit:
SHERIFF'S RETURN
This is to certify that on September 18, 2000, I caused the service of summons together with
copies of complaint and its annexes attached thereto, upon the following:
1. Defendant Allen A. Macasaet, President/Publisher of defendant AbanteTonite, at Monica
Publishing Corporation, Rooms 301-305 3rd Floor, BF Condominium Building, Solana corner A. Soriano
Streets, Intramuros, Manila, thru his secretary Lu-Ann Quijano, a person of sufficient age and discretion
working therein, who signed to acknowledge receipt thereof. That effort (sic) to serve the said summons
personally upon said defendant were made, but the same were ineffectual and unavailing on the ground
that per information of Ms. Quijano said defendant is always out and not available, thus, substituted
service was applied;
2. Defendant Nicolas V. Quijano, at the same address, thru his wife Lu-Ann Quijano, who signed
to acknowledge receipt thereof. That effort (sic) to serve the said summons personally upon said
defendant were made, but the same were ineffectual and unavailing on the ground that per information of
(sic) his wife said defendant is always out and not available, thus, substituted service was applied;
3. Defendants Isaias Albano, Janet Bay, Jesus R. Galang, Randy Hagos and Lily Reyes, at the
same address, thru Rene Esleta, Editorial Assistant of defendant AbanteTonite, a person of sufficient age
and discretion working therein who signed to acknowledge receipt thereof. That effort (sic) to serve the
said summons personally upon said defendants were made, but the same were ineffectual and
unavailing on the ground that per information of (sic) Mr. Esleta said defendants is (sic) always roving
outside and gathering news, thus, substituted service was applied.
Original copy of summons is therefore, respectfully returned duly served.
Manila, September 22, 2000. IaEHSD
On October 3, 2000, petitioners moved for the dismissal of the complaint through counsel's special appearance in
their behalf, alleging lack of jurisdiction over their persons because of the invalid and ineffectual substituted service of
summons. They contended that the sheriff had made no prior attempt to serve the summons personally on each of them in
accordance with Section 6 and Section 7, Rule 14 of the Rules of Court. They further moved to drop AbanteTonite as a
defendant by virtue of its being neither a natural nor a juridical person that could be impleaded as a party in a civil action.
At the hearing of petitioners' motion to dismiss, Medina testified that he had gone to the office address of petitioners
in the morning of September 18, 2000 to personally serve the summons on each defendant; that petitioners were out of the
office at the time; that he had returned in the afternoon of the same day to again attempt to serve on each defendant
personally but his attempt had still proved futile because all of petitioners were still out of the office; that some competent
persons working in petitioners' office had informed him that Macasaet and Quijano were always out and unavailable, and
that Albano, Bay, Galang, Hagos and Reyes were always out roving to gather news; and that he had then resorted to
substituted service upon realizing the impossibility of his finding petitioners in person within a reasonable time.
On March 12, 2001, the RTC denied the motion to dismiss, and directed petitioners to file their answers to the
complaint within the remaining period allowed by the Rules of Court, 6 relevantly stating:
Records show that the summonses were served upon Allen A. Macasaet, President/Publisher of
defendant AbanteTonite, through Lu-Ann Quijano; upon defendants Isaias Albano, Janet Bay, Jesus R.
Galang, Randy Hagos and Lily Reyes, through Rene Esleta, Editorial Assistant of defendant
AbanteTonite (p. 12, records). It is apparent in the Sheriff's Return that on several occasions, efforts to
served (sic) the summons personally upon all the defendants were ineffectual as they were always out
and unavailable, so the Sheriff served the summons by substituted service.
Considering that summonses cannot be served within a reasonable time to the persons of all the
defendants, hence substituted service of summonses was validly applied. Secretary of the President who
is duly authorized to receive such document, the wife of the defendant and the Editorial Assistant of the
defendant, were considered competent persons with sufficient discretion to realize the importance of the
legal papers served upon them and to relay the same to the defendants named therein (Sec. 7, Rule 14,
1997 Rules of Civil Procedure).
WHEREFORE, in view of the foregoing, the Motion to Dismiss is hereby DENIED for lack of
merit.
Accordingly, defendants are directed to file their Answers to the complaint within the period still
open to them, pursuant to the rules.
SO ORDERED.
Petitioners filed a motion for reconsideration, asserting that the sheriff had immediately resorted to substituted
service of the summons upon being informed that they were not around to personally receive the summons, and that
AbanteTonite, being neither a natural nor a juridical person, could not be made a party in the action.
On June 29, 2001, the RTC denied petitioners' motion for reconsideration. 7 It stated in respect of the service of
summons, as follows:
The allegations of the defendants that the Sheriff immediately resorted to substituted service of
summons upon them when he was informed that they were not around to personally receive the same is
untenable. During the hearing of the herein motion, Sheriff Raul Medina of this Branch of the Court
testified that on September 18, 2000 in the morning, he went to the office address of the defendants to
personally serve summons upon them but they were out. So he went back to serve said summons upon
the defendants in the afternoon of the same day, but then again he was informed that the defendants
were out and unavailable, and that they were always out because they were roving around to gather
news. Because of that information and because of the nature of the work of the defendants that they are
always on field, so the sheriff resorted to substituted service of summons. There was substantial
compliance with the rules, considering the difficulty to serve the summons personally to them because of
the nature of their job which compels them to be always out and unavailable. Additional matters regarding
the service of summons upon defendants were sufficiently discussed in the Order of this Court dated
March 12, 2001.
Regarding the impleading of AbanteTonite as defendant, the RTC held, viz.:
"AbanteTonite" is a daily tabloid of general circulation. People all over the country could buy a
copy of "AbanteTonite" and read it, hence, it is for public consumption. The persons who organized said
publication obviously derived profit from it. The information written on the said newspaper will affect the
person, natural as well as juridical, who was stated or implicated in the news. All of these facts imply that
"AbanteTonite" falls within the provision of Art. 44 (2 or 3), New Civil Code. Assuming arguendo that
"AbanteTonite" is not registered with the Securities and Exchange Commission, it is deemed a
corporation by estoppels considering that it possesses attributes of a juridical person, otherwise it cannot
be held liable for damages and injuries it may inflict to other persons. ICHAaT
Undaunted, petitioners brought a petition for certiorari, prohibition, mandamus in the CA to nullify the orders of the
RTC dated March 12, 2001 and June 29, 2001.
Ruling of the CA
On March 8, 2002, the CA promulgated its questioned decision, 8 dismissing the petition for certiorari, prohibition,
mandamus, to wit:
We find petitioners' argument without merit. The rule is that certiorari will prosper only if there is a
showing of grave abuse of discretion or an act without or in excess of jurisdiction committed by the
respondent Judge. A judicious reading of the questioned orders of respondent Judge would show that the
same were not issued in a capricious or whimsical exercise of judgment. There are factual bases and
legal justification for the assailed orders. From the Return, the sheriff certified that "effort to serve the
summons personally . . . were made, but the same were ineffectual and unavailing . . . .
and upholding the trial court's finding that there was a substantial compliance with the rules that allowed the substituted
service.
Furthermore, the CA ruled:
Anent the issue raised by petitioners that "AbanteTonite is neither a natural or juridical person
who may be a party in a civil case," and therefore the case against it must be dismissed and/or dropped,
is untenable.
The respondent Judge, in denying petitioners' motion for reconsideration, held that:
xxx xxx xxx
AbanteTonite's newspapers are circulated nationwide, showing ostensibly its being a corporate
entity, thus the doctrine of corporation by estoppel may appropriately apply.
An unincorporated association, which represents itself to be a corporation, will be estopped from
denying its corporate capacity in a suit against it by a third person who relies in good faith on such
representation. IaECcH
There being no grave abuse of discretion committed by the respondent Judge in the exercise of
his jurisdiction, the relief of prohibition is also unavailable.
WHEREFORE, the instant petition is DENIED. The assailed Orders of respondent Judge are
AFFIRMED.
SO ORDERED. 9
On January 13, 2003, the CA denied petitioners' motion for reconsideration. 10
Issues
Petitioners hereby submit that:
1. THE COURT OF APPEALS COMMITTED AN ERROR OF LAW IN HOLDING THAT THE
TRIAL COURT ACQUIRED JURISDICTION OVER HEREIN PETITIONERS.
2. THE COURT OF APPEALS COMMITTED REVERSIBLE ERROR BY SUSTAINING THE
INCLUSION OF ABANTE TONITE AS PARTY IN THE INSTANT CASE. 11
Ruling
The petition for review lacks merit.
Jurisdiction over the person, or jurisdiction in personam — the power of the court to render a personal judgment or
to subject the parties in a particular action to the judgment and other rulings rendered in the action — is an element of due
process that is essential in all actions, civil as well as criminal, except in actions in rem or quasi in rem. Jurisdiction over the
defendantin an action in rem or quasi in rem is not required, and the court acquires jurisdiction over an action as long as it
acquires jurisdiction over the res that is the subject matter of the action. The purpose of summons in such action is not the
acquisition of jurisdiction over the defendant but mainly to satisfy the constitutional requirement of due process. 12
The distinctions that need to be perceived between an action in personam, on the one hand, and an action in rem
or quasi in rem, on the other hand, are aptly delineated in Domagas v. Jensen, 13 thusly: HATICc
The settled rule is that the aim and object of an action determine its character. Whether a
proceeding is in rem, or in personam, or quasi in rem for that matter, is determined by its nature and
purpose, and by these only. A proceeding in personam is a proceeding to enforce personal rights and
obligations brought against the person and is based on the jurisdiction of the person, although it may
involve his right to, or the exercise of ownership of, specific property, or seek to compel him to control or
dispose of it in accordance with the mandate of the court. The purpose of a proceeding in personam is to
impose, through the judgment of a court, some responsibility or liability directly upon the person of the
defendant. Of this character are suits to compel a defendant to specifically perform some act or actions to
fasten a pecuniary liability on him. An action in personam is said to be one which has for its object a
judgment against the person, as distinguished from a judgment against the prop[er]ty to determine its
state. It has been held that an action in personam is a proceeding to enforce personal rights or
obligations; such action is brought against the person. As far as suits for injunctive relief are concerned, it
is well-settled that it is an injunctive act in personam. In Combs v. Combs, the appellate court held that
proceedings to enforce personal rights and obligations and in which personal judgments are rendered
adjusting the rights and obligations between the affected parties is in personam. Actions for recovery of
real property are in personam. TaDSCA
On the other hand, a proceeding quasi in rem is one brought against persons seeking to subject
the property of such persons to the discharge of the claims assailed. In an action quasi in rem, an
individual is named as defendant and the purpose of the proceeding is to subject his interests therein to
the obligation or loan burdening the property. Actions quasi in rem deal with the status, ownership or
liability of a particular property but which are intended to operate on these questions only as between the
particular parties to the proceedings and not to ascertain or cut off the rights or interests of all possible
claimants. The judgments therein are binding only upon the parties who joined in the action.
As a rule, Philippine courts cannot try any case against a defendant who does not reside and is not found in the
Philippines because of the impossibility of acquiring jurisdiction over his person unless he voluntarily appears in court; but
when the case is an action in rem or quasi in rem enumerated in Section 15, Rule 14 of the Rules of Court, Philippine courts
have jurisdiction to hear and decide the case because they have jurisdiction over the res, and jurisdiction over the person of
the non-resident defendant is not essential. In the latter instance, extraterritorial service of summons can be made upon the
defendant, and such extraterritorial service of summons is not for the purpose of vesting the court with jurisdiction, but for
the purpose of complying with the requirements of fair play or due process, so that the defendant will be informed of the
pendency of the action against him and the possibility that property in the Philippines belonging to him or in which he has an
interest may be subjected to a judgment in favor of the plaintiff, and he can thereby take steps to protect his interest if he is
so minded. On the other hand, when the defendant in an action in personam does not reside and is not found in the
Philippines, our courts cannot try the case against him because of the impossibility of acquiring jurisdiction over his person
unless he voluntarily appears in court. 14 aSTHDc
As the initiating party, the plaintiff in a civil action voluntarily submits himself to the jurisdiction of the court by the act
of filing the initiatory pleading. As to the defendant, the court acquires jurisdiction over his person either by the proper
service of the summons, or by a voluntary appearance in the action. 15
Upon the filing of the complaint and the payment of the requisite legal fees, the clerk of court forthwith issues the
corresponding summons to the defendant. 16 The summons is directed to the defendant and signed by the clerk of court
under seal. It contains the name of the court and the names of the parties to the action; a direction that the defendant
answers within the time fixed by the Rules of Court; and a notice that unless the defendant so answers, the plaintiff will take
judgment by default and may be granted the relief applied for. 17 To be attached to the original copy of the summons and all
copies thereof is a copy of the complaint (and its attachments, if any) and the order, if any, for the appointment of a guardian
ad litem. 18
The significance of the proper service of the summons on the defendant in an action in personam cannot be
overemphasized. The service of the summons fulfills two fundamental objectives, namely: (a) to vest in the court jurisdiction
over the person of the defendant; and (b) to afford to the defendant the opportunity to be heard on the claim brought against
him. 19 As to the former, when jurisdiction in personam is not acquired in a civil action through the proper service of the
summons or upon a valid waiver of such proper service, the ensuing trial and judgment are void. 20 If the defendant
knowingly does an act inconsistent with the right to object to the lack of personal jurisdiction as to him, like voluntarily
appearing in the action, he is deemed to have submitted himself to the jurisdiction of the court. 21 As to the latter, the
essence of due process lies in the reasonable opportunity to be heard and to submit any evidence the defendant may have
in support of his defense. With the proper service of the summons being intended to afford to him the opportunity to be
heard on the claim against him, he may also waive the process. 22 In other words, compliance with the rules regarding the
service of the summons is as much an issue of due process as it is of jurisdiction. 23
Under the Rules of Court, the service of the summons should firstly be effected on the defendant himself whenever
practicable. Such personal service consists either in handing a copy of the summons to the defendant in person, or, if the
defendant refuses to receive and sign for it, in tendering it to him. 24 The rule on personal service is to be rigidly enforced in
order to ensure the realization of the two fundamental objectives earlier mentioned. If, for justifiable reasons, the defendant
cannot be served in person within a reasonable time, the service of the summons may then be effected either (a) by leaving
a copy of the summons at his residence with some person of suitable age and discretion then residing therein, or (b) by
leaving the copy at his office or regular place of business with some competent person in charge thereof. 25 The latter
mode of service is known as substituted service because the service of the summons on the defendant is made through his
substitute. DHESca
It is no longer debatable that the statutory requirements of substituted service must be followed strictly, faithfully
and fully, and any substituted service other than that authorized by statute is considered ineffective. 26 This is because
substituted service, being in derogation of the usual method of service, is extraordinary in character and may be used only
as prescribed and in the circumstances authorized by statute. 27 Only when the defendant cannot be served personally
within a reasonable time may substituted service be resorted to. Hence, the impossibility of prompt personal service should
be shown by stating the efforts made to find the defendant himself and the fact that such efforts failed, which statement
should be found in the proof of service or sheriff's return. 28 Nonetheless, the requisite showing of the impossibility of
prompt personal service as basis for resorting to substituted service may be waived by the defendant either expressly or
impliedly. 29
There is no question that Sheriff Medina twice attempted to serve the summons upon each of petitioners in person
at their office address, the first in the morning of September 18, 2000 and the second in the afternoon of the same date.
Each attempt failed because Macasaet and Quijano were "always out and not available" and the other petitioners were
"always roving outside and gathering news." After Medina learned from those present in the office address on his second
attempt that there was no likelihood of any of petitioners going to the office during the business hours of that or any other
day, he concluded that further attempts to serve them in person within a reasonable time would be futile. The circumstances
fully warranted his conclusion. He was not expected or required as the serving officer to effect personal service by all means
and at all times, considering that he was expressly authorized to resort to substituted service should he be unable to effect
the personal service within a reasonable time. In that regard, what was a reasonable time was dependent on the
circumstances obtaining. While we are strict in insisting on personal service on the defendant, we do not cling to such
strictness should the circumstances already justify substituted service instead. It is the spirit of the procedural rules, not their
letter, that governs. 30 SCDaET
In reality, petitioners' insistence on personal service by the serving officer was demonstrably superfluous. They had
actually received the summonses served through their substitutes, as borne out by their filing of several pleadings in the
RTC, including an answer with compulsory counterclaim ad cautelam and a pre-trial brief ad cautelam. They had also
availed themselves of the modes of discovery available under the Rules of Court. Such acts evinced their voluntary
appearance in the action.
Nor can we sustain petitioners' contention that Abante Tonite could not be sued as a defendant due to its not being
either a natural or a juridical person. In rejecting their contention, the CA categorized Abante Tonite as a corporation by
estoppel as the result of its having represented itself to the reading public as a corporation despite its not being
incorporated. Thereby, the CA concluded that the RTC did not gravely abuse its discretion in holding that the non-
incorporation of Abante Tonite with the Securities and Exchange Commission was of no consequence, for, otherwise,
whoever of the public who would suffer any damage from the publication of articles in the pages of its tabloids would be left
without recourse. We cannot disagree with the CA, considering that the editorial box of the daily tabloid disclosed that
although Monica Publishing Corporation had published the tabloid on a daily basis, nothing in the box indicated that Monica
Publishing Corporation had owned Abante Tonite.
WHEREFORE, the Court AFFIRMS the decision promulgated on March 8, 2002; and ORDERS petitioners to pay
the costs of suit. SO ORDERED.
||| (Macasaet v. Co, Jr., G.R. No. 156759, [June 5, 2013], 710 PHIL 167-183)

[G.R. No. 197530. July 9, 2014.]

ABOITIZ EQUITY VENTURES, INC., petitioner, vs. VICTOR S. CHIONGBIAN, BENJAMIN D.


GOTHONG, and CARLOS A. GOTHONG LINES, INC. (CAGLI), respondents.

DECISION

LEONEN, J p:

This is a petition for review on certiorari with an application for the issuance of a temporary restraining order and/or
writ of preliminary injunction under Rule 45 of the Rules of Court. This petition prays that the assailed orders dated May 5,
2011 1 and June 24, 2011 2 of the Regional Trial Court, Cebu City, Branch 10 in Civil Case No. CEB-37004 be nullified and
set aside and that judgment be rendered dismissing with prejudice the complaint 3 dated July 20, 2010 filed by respondents
Carlos A. Gothong Lines, Inc. ("CAGLI") and Benjamin D. Gothong.
On January 8, 1996, Aboitiz Shipping Corporation ("ASC"), principally owned by the Aboitiz family, CAGLI,
principally owned by the Gothong family, and William Lines, Inc. ("WLI"), principally owned by the Chiongbian family,
entered into an agreement (the "Agreement"), 4 whereby ASC and CAGLI would transfer their shipping assets to WLI in
exchange for WLI's shares of stock. 5 WLI, in turn, would run their merged shipping businesses and, henceforth, be known
as WG&A, Inc. ("WG&A"). 6
Sec. 11.06 of the Agreement required all disputes arising out of or in connection with the Agreement to be settled
by arbitration:
11.06 Arbitration
All disputes arising out of or in connection with this Agreement including any issue as to this
Agreement's validity or enforceability, which cannot be settled amicably among the
parties, shall be finally settled by arbitration in accordance with the Arbitration Law
(Republic Act No. 876) by an arbitration tribunal composed of four (4) arbitrators. Each of
the parties shall appoint one (1) arbitrator, the three (3) to appoint the fourth arbitrator
who shall act as Chairman. Any award by the arbitration tribunal shall be final and
binding upon the parties and shall be enforced by judgment of the Courts of Cebu or
Metro Manila. 7
Among the attachments to the Agreement was Annex SL-V. 8 This was a letter dated January 8, 1996, from WLI,
through its President (herein respondent) Victor S. Chiongbian addressed to CAGLI, through its Chief Executive Officer Bob
D. Gothong and Executive Vice President for Engineering (herein respondent) Benjamin D. Gothong. On its second page,
Annex SL-V bore the signatures of Bob D. Gothong and respondent Benjamin D. Gothong by way of a conforme on behalf
of CAGLI. IEAaST
Annex SL-V confirmed WLI's commitment to acquire certain inventories of CAGLI. These inventories would have a
total aggregate value of, at most, PhP400 million, "as determined after a special examination of the [i]nventories". 9 Annex
SL-V also specifically stated that such acquisition was "pursuant to the Agreement". 10
The entirety of Annex SL-V's substantive portion reads:
We refer to the Agreement dated January 8, 1996 (the "Agreement") among William Lines, Inc.
("Company C"), Aboitiz Shipping Corporation ("Company A") and Carlos A. Gothong Lines, Inc.
("Company B") regarding the transfer of various assets of Company A and Company B to Company C in
exchange for shares of capital stock of Company C. Terms defined in the Agreement are used herein as
therein defined.
This will confirm our commitment to acquire certain spare parts and materials inventory (the
"Inventories") of Company B pursuant to the Agreement.
The total aggregate value of the Inventories to be acquired shall not exceed P400 Million as
determined after a special examination of the Inventories as performed by SGV & Co. to be completed on
or before the Closing Date under the agreed procedures determined by the parties.
Subject to documentation acceptable to both parties, the Inventories to be acquired shall be
determined not later than thirty (30) days after the Closing Date and the payments shall be made in equal
quarterly instalments over a period of two years with the first payment due on March 31, 1996. 11
Pursuant to Annex SL-V, inventories were transferred from CAGLI to WLI. These inventories were assessed to
have a value of PhP514 million, which was later adjusted to PhP558.89 million. 12 Of the total amount of PhP558.89 million,
"CAGLI was paid the amount of PhP400 Million." 13 In addition to the payment of PhP400 million, petitioner Aboitiz Equity
Ventures ("AEV") noted that WG&A shares with a book value of PhP38.5 million were transferred to CAGLI. 14
As there was still a balance, in 2001, CAGLI sent WG&A (the renamed WLI) demand letters "for the return of or the
payment for the excess [i]nventories." 15 AEV alleged that to satisfy CAGLI's demand, WLI/WG&A returned inventories
amounting to PhP120.04 million. 16 As proof of this, AEV attached copies of delivery receipts signed by CAGLI's
representatives as Annex "K" of the present petition. 17
Sometime in 2002, the Chiongbian and Gothong families decided to leave the WG&A enterprise and sell their
interest in WG&A to the Aboitiz family. As such, a share purchase agreement 18 ("SPA") was entered into by petitioner AEV
and the respective shareholders groups of the Chiongbians and Gothongs. In the SPA, AEV agreed to purchase the
Chiongbian group's 40.61% share and the Gothong group's 20.66% share in WG&A's issued and outstanding stock. 19
Section 6.5 of the SPA provided for arbitration as the mode of settling any dispute arising from the SPA. It reads:
6.5 Arbitration. Should there be any dispute arising between the parties relating to this Agreement
including the interpretation or performance hereof which cannot be resolved by agreement of the
parties within fifteen (15) days after written notice by a party to another, such matter shall then be
finally settled by arbitration in Cebu City in accordance with the Philippine Arbitration Law. Substantive
aspects of the dispute shall be settled by applying the laws of the Philippines. The decision of the
arbitrators shall be final and binding upon the parties hereto and the expense of arbitration (including
without limitation the award of attorney's fees to the prevailing party) shall be paid as the arbitrators
shall determine. 20 SHECcT
Section 6.8 of the SPA further provided that the Agreement (of January 8, 1996) shall be deemed terminated except
its Annex SL-V. It reads:
6.8 Termination of Shareholders Agreement. The Buyer and the Sellers hereby agree that on Closing,
the Agreement among Aboitiz Shipping Corporation, Carlos A. Gothong Lines, Inc. and William Lines,
Inc. dated January 8, 1996, as the same has been amended from time to time (the "Shareholders'
Agreement") shall all be considered terminated, except with respect to such rights and obligations that
the parties to the Shareholders' Agreement have under a letter dated January 8, 1996 (otherwise
known as "SL-V") from William Lines, Inc. to Carlos A. Gothong Lines, Inc. regarding certain spare
parts and materials inventory, which rights and obligations shall survive through the date prescribed by
the applicable statute of limitations. 21
As part of the SPA, the parties entered into an Escrow Agreement 22 whereby ING Bank N.V.-Manila Branch was
to take custody of the shares subject of the SPA. 23 Section 14.7 of the Escrow Agreement provided that all disputes arising
from it shall be settled through arbitration:
14.7 All disputes, controversies or differences which may arise by and among the parties hereto out of,
or in relation to, or in connection with this Agreement, or for the breach thereof shall be finally settled by
arbitration in Cebu City in accordance with the Philippine Arbitration Law. The award rendered by the
arbitrator(s) shall be final and binding upon the parties concerned. However, notwithstanding the
foregoing provision, the parties reserve the right to seek redress before the regular court and avail of
any provisional remedies in the event of any misconduct, negligence, fraud or tortuous acts which arise
from any extra-contractual conduct that affects the ability of a party to comply with his obligations and
responsibilities under this Agreement. 24
As a result of the SPA, AEV became a stockholder of WG&A. Subsequently, WG&A was renamed Aboitiz Transport
Shipping Corporation ("ATSC"). 25
Petitioner AEV alleged that in 2008, CAGLI resumed making demands despite having already received PhP120.04
million worth of excess inventories. 26 CAGLI initially made its demand to ATSC (the renamed WLI/WG&A) through a letter
27 dated February 14, 2008. As alleged by AEV, however, CAGLI subsequently resorted to a "shotgun approach" 28 and
directed its subsequent demand letters to AEV 29 as well as to FCLC 30 (a company related to respondent Chiongbian).
AEV responded to CAGLI's demands through several letters. 31 In these letters, AEV rebuffed CAGLI's demands
noting that: (1) CAGLI already received the excess inventories; (2) it was not a party to CAGLI's claim as it had a personality
distinct from WLI/WG&A/ATSC; and (3) CAGLI's claim was already barred by prescription. HEASaC
In a reply-letter 32 dated May 5, 2008, CAGLI claimed that it was unaware of the delivery to it of the excess
inventories and asked for copies of the corresponding delivery receipts. 33 CAGLI threatened that unless it received proof of
payment or return of excess inventories having been made on or before March 31, 1996, it would pursue arbitration. 34
In letters written for AEV (the first dated October 16, 2008 by Aboitiz and Company, Inc.'s Associate General
Counsel Maria Cristina G. Gabutina 35 and the second dated October 27, 2008 by SyCip Salazar Hernandez and
Gatmaitan), 36 it was noted that the excess inventories were delivered to GT Ferry Warehouse. 37 Attached to these letters
were a listing and/or samples 38 of the corresponding delivery receipts. In these letters it was also noted that the amount of
excess inventories delivered (PhP120.04 million) was actually in excess of the value of the supposedly unreturned
inventories (PhP119.89 million). 39 Thus, it was pointed out that it was CAGLI which was liable to return the difference
between PhP120.04 million and PhP119.89 million. 40
Its claims not having been satisfied, CAGLI filed on November 6, 2008 the first of two applications for arbitration
("first complaint") 41 against respondent Chiongbian, ATSC, ASC, and petitioner AEV, before the Cebu City Regional Trial
Court, Branch 20. The first complaint was docketed as Civil Case No. CEB-34951.
In response, AEV filed a motion to dismiss 42 dated February 5, 2009. AEV argued that CAGLI failed to state a
cause of action as there was no agreement to arbitrate between CAGLI and AEV. 43 Specifically, AEV pointed out that: (1)
AEV was never a party to the January 8, 1996 Agreement or to its Annex SL-V; 44 (2) while AEV is a party to the SPA and
Escrow Agreement, CAGLI's claim had no connection to either agreement; (3) the unsigned and unexecuted SPA attached
to the complaint cannot be a source of any right to arbitrate; 45 and (4) CAGLI did not say how WLI/WG&A/ATSC's
obligation to return the excess inventories can be charged to AEV.
On December 4, 2009, the Cebu City Regional Trial Court, Branch 20 issued an order 46 dismissing the first
complaint with respect to AEV. It sustained AEV's assertion that there was no agreement binding AEV and CAGLI to
arbitrate CAGLI's claim. 47 Whether by motion for reconsideration, appeal or other means, CAGLI did not contest this
dismissal.
On February 26, 2010, the Cebu City Regional Trial Court, Branch 20 issued an order 48 directing the parties
remaining in the first complaint (after the discharge of AEV) to proceed with arbitration.
The February 26, 2010 order notwithstanding, CAGLI filed a notice of dismissal 49 dated July 8, 2010, withdrawing
the first complaint. In an order 50 dated August 13, 2010, the Cebu City Regional Trial Court, Branch 20 allowed this
withdrawal.
ATSC (the renamed WLI/WG&A) filed a motion for reconsideration 51 dated September 20, 2010 to the allowance
of CAGLI's notice of dismissal. This motion was denied in an order 52 dated April 15, 2011. HTCAED
On September 1, 2010, while the first complaint was still pending (n.b., it was only on April 15, 2011 that the Cebu
City Regional Trial Court, Branch 20 denied ATSC's motion for reconsideration assailing the allowance of CAGLI's notice of
disallowance), CAGLI, now joined by respondent Benjamin D. Gothong, filed a second application for arbitration ("second
complaint") 53 before the Cebu City Regional Trial Court, Branch 10. The second complaint was docketed as Civil Case No.
CEB-37004 and was also in view of the return of the same excess inventories subject of the first complaint.
On October 28, 2010, AEV filed a motion to dismiss 54 the second complaint on the following grounds: 55 (1) forum
shopping; (2) failure to state a cause of action; (3) res judicata; and (4) litis pendentia.
In the first of the two (2) assailed orders dated May 5, 2011, 56 the Cebu City Regional Trial Court, Branch 10
denied AEV's motion to dismiss. cCaDSA
On the matter of litis pendentia, the Regional Trial Court, Branch 10 noted that the first complaint was dismissed
with respect to AEV on December 4, 2009, while the second complaint was filed on September 1, 2010. As such, the first
complaint was no longer pending at the time of the filing of the second complaint. 57 On the matter of res judicata, the trial
court noted that the dismissal without prejudice of the first complaint "[left] the parties free to litigate the matter in a
subsequent action, as though the dismiss[ed] action had not been commenced". 58 It added that since litis pendentia and
res judicata did not exist, CAGLI could not be charged with forum shopping. 59 On the matter of an agreement to arbitrate,
the Regional Trial Court, Branch 10 pointed to the SPA as "clearly express[ing] the intention of the parties to bring to
arbitration process all disputes, if amicable settlement fails". 60 It further dismissed AEV's claim that it was not a party to the
SPA, as "already touching on the merits of the case" 61 and therefore beyond its duty "to determine if they should proceed
to arbitration or not". 62
In the second assailed order 63 dated June 24, 2011, the Cebu City Regional Trial Court, Branch 10 denied AEV's
motion for reconsideration.
Aggrieved, AEV filed the present petition. 64 AEV asserts that the second complaint is barred by res judicata and
litis pendentia and that CAGLI engaged in blatant forum shopping. 65 It insists that it is not bound by an agreement to
arbitrate with CAGLI and that, even assuming that it may be required to arbitrate, it is being ordered to do so under terms
that are "manifestly contrary to the . . . agreements on which CAGLI based its demand for arbitration". 66
For resolution are the following issues:
I. Whether the complaint in Civil Case No. CEB-37004 constitutes forum shopping and/or is
barred by res judicata and/or litis pendentia
II. Whether petitioner, Aboitiz Equity Ventures, Inc., is bound by an agreement to arbitrate with
Carlos A. Gothong Lines, Inc., with respect to the latter's claims for unreturned
inventories delivered to William Lines, Inc./WG&A, Inc./Aboitiz Transport System
Corporation
AEV availed of the wrong
remedy in seeking relief from
this court
Before addressing the specific matters raised by the present petition, we emphasize that AEV is in error in seeking
relief from this court via a petition for review on certiorari under Rule 45 of the Rules of Court. As such, we are well in a
position to dismiss the present petition outright. Nevertheless, as the actions of the Cebu City Regional Trial Court, Branch
10 are tainted with grave abuse of discretion amounting to lack or excess of jurisdiction, this court treats the present Rule 45
petition as a Rule 65 petition and gives it due course. TEHIaD
A petition for review on certiorari under Rule 45 is a mode of appeal. This is eminently clear from the very title and
from the first section of Rule 45 (as amended by A.M. No. 07-7-12-SC):
Rule 45
APPEAL BY CERTIORARI TO THE SUPREME COURT
SECTION 1. Filing of petition with Supreme Court. — A party desiring to appeal by certiorari from a
judgment, final order or resolution of the Court of Appeals, the Sandiganbayan, the Court of Tax
Appeals, the Regional Trial Court or other courts, whenever authorized by law, may file with the
Supreme Court a verified petition for review on certiorari. The petition may include an application for a
writ of preliminary injunction or other provisional remedies and shall raise only questions of law, which
must be distinctly set forth. The petitioner may seek the same provisional remedies by verified motion
filed in the same action or proceeding at any time during its pendency. (Emphasis supplied)
Further, it is elementary that an appeal may only be taken from a judgment or final order that completely disposes
of the case. 67 As such, no appeal may be taken from an interlocutory order 68 (i.e., "one which refers to something
between the commencement and end of the suit which decides some point or matter but it is not the final decision of the
whole controversy"). 69 As explained in Sime Darby Employees Association v. NLRC, 70 "[a]n interlocutory order is not
appealable until after the rendition of the judgment on the merits for a contrary rule would delay the administration of justice
and unduly burden the courts". 71
An order denying a motion to dismiss is interlocutory in character. Hence, it may not be the subject of an appeal.
The interlocutory nature of an order denying a motion to dismiss and the remedies for assailing such an order were
discussed in Douglas Lu Ym v. Nabua: 72
An order denying a motion to dismiss is an interlocutory order which neither terminates nor finally
disposes of a case, as it leaves something to be done by the court before the case is finally decided on
the merits. As such, the general rule is that the denial of a motion to dismiss cannot be questioned in a
special civil action for certiorari which is a remedy designed to correct errors of jurisdiction and not errors
of judgment. Neither can a denial of a motion to dismiss be the subject of an appeal unless and until a
final judgment or order is rendered. In order to justify the grant of the extraordinary remedy of certiorari,
the denial of the motion to dismiss must have been tainted with grave abuse of discretion amounting to
lack or excess of jurisdiction. 73 (Emphasis supplied)
Thus, where a motion to dismiss is denied, the proper recourse is for the movant to file an answer. 74 Nevertheless,
where the order denying the motion to dismiss is tainted with grave abuse of discretion amounting to lack or excess of
jurisdiction, the movant may assail such order via a Rule 65 (i.e., certiorari, prohibition, and/or mandamus) petition. This is
expressly recognized in the third paragraph of Rule 41, Section 1 of the Rules of Court. 75 Following the enumeration in the
second paragraph of Rule 41, Section 1 of the instances when an appeal may not be taken, the third paragraph specifies
that "[in] any of the foregoing circumstances, the aggrieved party may file an appropriate special civil action as provided in
Rule 65". 76
Per these rules, AEV is in error for having filed what it itself calls a "Petition for Review on Certiorari [Appeal by
Certiorari under Rule 45 of the Rules of Court]". 77 Since AEV availed of the improper remedy, this court is well in a position
to dismiss the present petition.
Nevertheless, there have been instances when a petition for review on certiorari under Rule 45 was treated by this
court as a petition for certiorari under Rule 65. As explained in China Banking Corporation v. Asian Construction and
Development Corporation: 78 CHTcSE
[I]n many instances, the Court has treated a petition for review on certiorari under Rule 45 as a petition
for certiorari under Rule 65 of the Rules of Court, such as in cases where the subject of the recourse
was one of jurisdiction, or the act complained of was perpetrated by a court with grave abuse of
discretion amounting to lack or excess of jurisdiction. 79
In this case, the May 5, 2011 and June 24, 2011 orders of the Cebu City Regional Trial Court, Branch 10 in Civil
Case No. CEB-37004 are assailed for having denied AEV's motion to dismiss despite: first, the second complaint having
been filed in a manner constituting forum shopping; second, the prior judgment on the merits made in Civil Case No. CEB-
34951, thereby violating the principle of res judicata; and third, the (then) pendency of Civil Case No. CEB-34951 with
respect to the parties that, unlike AEV, were not discharged from the case, thereby violating the principle of litis pendentia.
The same orders are assailed for having allowed CAGLI's application for arbitration to continue despite supposedly clear
and unmistakable evidence that AEV is not bound by an agreement to arbitrate with CAGLI.
As such, the Cebu City, Regional Trial Court, Branch 10's orders are assailed for having been made with grave
abuse of discretion amounting to lack or excess of jurisdiction in that the Cebu City Regional Trial Court, Branch 10 chose to
continue taking cognizance of the second complaint, despite there being compelling reasons for its dismissal and the Cebu
City, Regional Trial Court Branch 20's desistance. Conformably, we treat the present petition as a petition for certiorari
under Rule 65 of the Rules of Court and give it due course.
The complaint in Civil Case
No. CEB-37004 constitutes
forum shopping and is barred
by res judicata
The concept of and rationale against forum shopping were explained by this court in Top Rate Construction &
General Services, Inc. v. Paxton Development Corporation: 80
FORUM SHOPPING is committed by a party who institutes two or more suits in different courts,
either simultaneously or successively, in order to ask the courts to rule on the same or related causes or
to grant the same or substantially the same reliefs, on the supposition that one or the other court would
make a favorable disposition or increase a party's chances of obtaining a favorable decision or action. It
is an act of malpractice for it trifles with the courts, abuses their processes, degrades the administration
of justice and adds to the already congested court dockets. What is critical is the vexation brought upon
the courts and the litigants by a party who asks different courts to rule on the same or related causes and
grant the same or substantially the same reliefs and in the process creates the possibility of conflicting
decisions being rendered by the different fora upon the same issues, regardless of whether the court in
which one of the suits was brought has no jurisdiction over the action. 81 SDEHCc
Equally settled is the test for determining forum shopping. As this court explained in Yap v. Chua: 82
To determine whether a party violated the rule against forum shopping, the most important factor
to ask is whether the elements of litis pendentia are present, or whether a final judgment in one case will
amount to res judicata in another; otherwise stated, the test for determining forum shopping is whether in
the two (or more) cases pending, there is identity of parties, rights or causes of action, and reliefs sought.
83
Litis pendentia "refers to that situation wherein another action is pending between the same parties for the same
cause of action, such that the second action becomes unnecessary and vexatious". 84 It requires the concurrence of three
(3) requisites: "(1) the identity of parties, or at least such as representing the same interests in both actions; (2) the identity
of rights asserted and relief prayed for, the relief being founded on the same facts; and (3) the identity of the two cases such
that judgment in one, regardless of which party is successful, would amount to res judicata in the other". 85
In turn, prior judgment or res judicata bars a subsequent case when the following requisites concur: "(1) the former
judgment is final; (2) it is rendered by a court having jurisdiction over the subject matter and the parties; (3) it is a judgment
or an order on the merits; (4) there is — between the first and the second actions — identity of parties, of subject matter,
and of causes of action". 86
Applying the cited concepts and requisites, we find that the complaint in Civil Case No. CEB-37004 is barred by res
judicata and constitutes forum shopping.
First, between the first and second complaints, there is identity of parties. The first complaint was brought by CAGLI
as the sole plaintiff against Victor S. Chiongbian, ATSC, and AEV as defendants. In the second complaint, CAGLI was
joined by Benjamin D. Gothong as (co-)plaintiff. As to the defendants, ATSC was deleted while Chiongbian and AEV were
retained.
While it is true that the parties to the first and second complaints are not absolutely identical, this court has clarified
that, for purposes of forum shopping, "[a]bsolute identity of parties is not required [and that it] is enough that there is
substantial identity of parties". 87
Even as the second complaint alleges that Benjamin D. Gothong "is . . . suing in his personal capacity", 88 Gothong
failed to show any personal interest in the reliefs sought by the second complaint. Ultimately, what is at stake in the second
complaint is the extent to which CAGLI may compel AEV and Chiongbian to arbitrate in order that CAGLI may then recover
the value of its alleged unreturned inventories. This claim for recovery is pursuant to the agreement evinced in Annex SL-V.
Annex SL-V was entered into by CAGLI and not by Benjamin D. Gothong. While it is true that Benjamin D. Gothong, along
with Bob D. Gothong, signed Annex SL-V, he did so only in a representative, and not in a personal, capacity. As such,
Benjamin D. Gothong cannot claim any right that personally accrues to him on account of Annex SL-V. From this, it follows
that Benjamin D. Gothong is not a real party in interest — "one who stands to be benefitted or injured by the judgment in the
suit or the party entitled to the avails of the suit" 89 — and that his inclusion in the second complaint is an unnecessary
superfluity.
Second, there is identity in subject matter and cause of action. There is identity in subject matter as both complaints
are applications for the same relief. There is identity in cause of action as both complaints are grounded on the right to be
paid for or to receive the value of excess inventories (and the supposed corresponding breach thereof) as spelled out in
Annex SL-V. THAECc
The first and second complaints are both applications for arbitration and are founded on the same instrument —
Annex SL-V. Moreover, the intended arbitrations in both complaints cater to the same ultimate purpose, i.e., that CAGLI
may recover the value of its supposedly unreturned inventories earlier delivered to WLI/WG&A/ATSC.
In both complaints, the supposed propriety of compelling the defendants to submit themselves to arbitration are
anchored on the same bases: (1) Section 6.8 of the SPA, which provides that the January 8, 1996 Agreement shall be
deemed terminated but that the rights and obligations arising from Annex SL-V shall continue to subsist; 90 (2) Section 6.5
of the SPA, which requires arbitration as the mode for settling disputes relating to the SPA; 91 and, (3) defendants' refusal
to submit themselves to arbitration vis-a-vis Republic Act No. 876, which provides that "[a] party aggrieved by the failure,
neglect or refusal of another to perform under an agreement in writing providing for arbitration may petition the court for an
order directing that such arbitration proceed in the manner provided for in such agreement". 92
Both complaints also rely on the same factual averments: 93
1. that ASC, CAGLI, and WLI entered into an agreement on January 8, 1996;
2. that under Annex SL-V of the Agreement, WLI/WG&A "committed to acquire certain
[inventories], the total aggregate value of which shall not exceed PhP400 Million"; 94
3. that after examination, it was ascertained that the value of the transferred inventories
exceeded PhP400 million;
4. that pursuant to Annex SL-V, WG&A paid CAGLI PhP400 million but that the former failed to
return or pay for spare parts representing a value in excess of PhP400 million;
5. "[t]hat on August 31, 2001, [CAGLI] wrote the WG&A through its AVP Materials Management,
Ms. Concepcion M. Magat, asking for the return of the excess spare parts"; 95
6. that on September 5, 2001, WG&A's Ms. Magat replied that the matter is beyond her authority
level and that she must elevate it to higher management;
7. that several communications demanding the return of the excess spare parts were sent to
WG&A but these did not elicit any response; and
8. "[t]hat the issue of excess spare parts, was taken over by events, when on July 31, 2002", 96
the Chiongbians and Gothongs entered into an Escrow Agreement with AEV.
Third, the order dated December 4, 2009 of the Cebu City Regional Trial Court, Branch 20, which dismissed the
first complaint with respect to AEV, attained finality when CAGLI did not file a motion for reconsideration, appealed, or, in
any other manner, questioned the order.
Fourth, the parties did not dispute that the December 4, 2009 order was issued by a court having jurisdiction over
the subject matter and the parties. Specifically as to jurisdiction over the parties, jurisdiction was acquired over CAGLI as
plaintiff when it filed the first complaint and sought relief from the Cebu City Regional Trial Court, Branch 20; jurisdiction
over defendants AEV, ATSC, and Victor S. Chiongbian was acquired with the service of summons upon them.
Fifth, the dismissal of the first complaint with respect to AEV was a judgment on the merits. As explained in
Cabreza, Jr. v. Cabreza: 97 IESDCH
A judgment may be considered as one rendered on the merits "when it determines the rights and
liabilities of the parties based on the disclosed facts, irrespective of formal, technical or dilatory
objections"; or when the judgment is rendered "after a determination of which party is right, as
distinguished from a judgment rendered upon some preliminary or formal or merely technical point". 98
Further, as this court clarified in Mendiola v. Court of Appeals, 99 "[i]t is not necessary . . . that there [be] a trial" 100
in order that a judgment be considered as one on the merits.
Prior to issuing the December 4, 2009 order dismissing the first complaint with respect to AEV, the Cebu City
Regional Trial Court, Branch 20 allowed the parties the full opportunity to establish the facts and to ventilate their arguments
relevant to the complaint. Specifically, the Cebu City Regional Trial Court, Branch 20 admitted: 1) AEV's motion to dismiss;
101 2) CAGLI's opposition to the motion to dismiss; 102 3) AEV's reply and opposition; 103 4) CAGLI's rejoinder; 104 and
5) AEV's sur-rejoinder. 105
Following these, the Cebu City Regional Trial Court, Branch 20 arrived at the following findings and made a
definitive determination that CAGLI had no right to compel AEV to subject itself to arbitration with respect to CAGLI's claims
under Annex SL-V:
After going over carefully the contentions and arguments of both parties, the court has found that
no contract or document exists binding CAGLI and AEV to arbitrate the former's claim. The WLI Letter
upon which the claim is based confirms only the commitment of William Lines, Inc. (WLI) to purchase
certain material inventories from CAGLI. It does not involve AEV. The court has searched in vain for any
agreement or document showing that said commitment was passed on to and assumed by AEV. Such
agreement or document, if one exists, being an actionable document, should have been attached to the
complaint. While the Agreement of January 8, 1996 and the Share Purchase Agreement provide for
arbitration of disputes, they refer to disputes arising from or in connection with the Agreements
themselves. No reference is made, as included therein, to the aforesaid commitment of WLI or to any
claim that CAGLI may pursue based thereon or relative thereto. Section 6.8 of the Share Purchase
Agreement, cited by plaintiff CAGLI, does not incorporate therein, expressly or impliedly, the WLI
commitment above-mentioned. It only declares that the rights and obligations of the parties under the
WLI Letter shall survive even after the termination of the Shareholder's Agreement. It does not speak of
arbitration. Finally, the complaint does not allege the existence of a contract obliging CAGLI and AEV to
arbitrate CAGLI's claim under the WLI Letter. Consequently, there is no legal or factual basis for the
present complaint for application for arbitration. 106 (Emphasis supplied)
In the assailed order dated May 5, 2011, the Cebu City Regional Trial Court, Branch 10 made much of the Cebu
City Regional Trial Court, Branch 20's pronouncement in the latter's December 4, 2009 order that "the [first] complaint fails
to state a cause of action". 107 Based on this, the Cebu City Regional Trial Court, Branch 10 concluded that the dismissal
of the first complaint was one made without prejudice, thereby "leav[ing] the parties free to litigate the matter in a
subsequent action, as though the dismissal [sic] action had not been commenced". 108
The Cebu City Regional Trial Court, Branch 10 is in serious error. In holding that the second complaint was not
barred by res judicata, the Cebu City Regional Trial Court, Branch 10 ignored established jurisprudence.
Referring to the earlier cases of Manalo v. Court of Appeals 109 and Mendiola v. Court of Appeals, 110 this court
emphasized in Luzon Development Bank v. Conquilla 111 that dismissal for failure to state a cause of action may very well
be considered a judgment on the merits and, thereby, operate as res judicata on a subsequent case:
[E]ven a dismissal on the ground of "failure to state a cause of action" may operate as res judicata on a
subsequent case involving the same parties, subject matter, and causes of action, provided that the
order of dismissal actually ruled on the issues raised. What appears to be essential to a judgment on
the merits is that it be a reasoned decision, which clearly states the facts and the law on which it is
based. 112 (Emphasis supplied)
To reiterate, the Cebu City Regional Trial Court, Branch 20 made a definitive determination that CAGLI had no
right to compel AEV to subject itself to arbitration vis-a-vis CAGLI's claims under Annex SL-V. This determination was
arrived at after due consideration of the facts established and the arguments advanced by the parties. Accordingly, the
Cebu City Regional Trial Court, Branch 20's December 4, 2009 order constituted a judgment on the merits and operated as
res judicata on the second complaint.
In sum, the requisites for res judicata have been satisfied and the second complaint should, thus, have been
dismissed. From this, it follows that CAGLI committed an act of forum shopping in filing the second complaint. CAGLI
instituted two suits in two regional trial court branches, albeit successively and not simultaneously. It asked both branches to
rule on the exact same cause and to grant the exact same relief. CAGLI did so after it had obtained an unfavorable decision
(at least with respect to AEV) from the Cebu City Regional Trial Court, Branch 20. These circumstances afford the
reasonable inference that the second complaint was filed in the hopes of a more favorable ruling. ECTIcS
Notwithstanding our pronouncements sustaining AEV's allegations that CAGLI engaged in forum shopping and that
the second complaint was barred by res judicata, we find that at the time of the filing of the second complaint, AEV had
already been discharged from the proceedings relating to the first complaint. Thus, as between AEV and CAGLI, the first
complaint was no longer pending at the time of the filing of the second complaint. Accordingly, the second complaint could
not have been barred by litis pendentia.
There is no agreement
binding AEV to arbitrate
with CAGLI on the latter's
claims arising from Annex
SL-V
For arbitration to be proper, it is imperative that it be grounded on an agreement between the parties. This was
adequately explained in Ormoc Sugarcane Planters' Association, Inc. v. Court of Appeals: 113
Section 2 of R.A. No. 876 (the Arbitration Law) pertinently provides:
Sec. 2. Persons and matters subject to arbitration. — Two or more persons or parties
may submit to the arbitration of one or more arbitrators any controversy existing between them at
the time of the submission and which may be the subject of an action, or the parties to any
contract may in such contract agree to settle by arbitration a controversy thereafter arising
between them. Such submission or contract shall be valid, enforceable and irrevocable, save
upon such grounds as exist at law for the revocation of any contract. . . . (Emphasis ours)
The foregoing provision speaks of two modes of arbitration: (a) an agreement to submit to
arbitration some future dispute, usually stipulated upon in a civil contract between the parties, and known
as an agreement to submit to arbitration, and (b) an agreement submitting an existing matter of difference
to arbitrators, termed the submission agreement. Article XX of the milling contract is an agreement to
submit to arbitration because it was made in anticipation of a dispute that might arise between the parties
after the contract's execution.
Except where a compulsory arbitration is provided by statute, the first step toward the settlement
of a difference by arbitration is the entry by the parties into a valid agreement to arbitrate. An agreement
to arbitrate is a contract, the relation of the parties is contractual, and the rights and liabilities of the
parties are controlled by the law of contracts. In an agreement for arbitration, the ordinary elements of a
valid contract must appear, including an agreement to arbitrate some specific thing, and an agreement to
abide by the award, either in express language or by implication. 114 (Emphasis supplied)
In this petition, not one of the parties — AEV, CAGLI, Victor S. Chiongbian, and Benjamin D. Gothong — has
alleged and/or shown that the controversy is properly the subject of "compulsory arbitration [as] provided by statute". 115
Thus, the propriety of compelling AEV to submit itself to arbitration must necessarily be founded on contract.
Four (4) distinct contracts have been cited in the present petition:
1. The January 8, 1996 Agreement in which ASC, CAGLI, and WLI merged their shipping
enterprises, with WLI (subsequently renamed WG&A) as the surviving entity. Section
11.06 of this Agreement provided for arbitration as the mechanism for settling all
disputes arising out of or in connection with the Agreement.
2. Annex SL-V of the Agreement between CAGLI and WLI (and excluded ASC and any other
Aboitiz-controlled entity), and which confirmed WLI's commitment to acquire certain
inventories, worth not more than PhP400 million, of CAGLI. Annex SL-V stated that the
acquisition was "pursuant to the Agreement". 116 It did not contain an arbitration clause.
3. The September 23, 2003 Share Purchase Agreement or SPA in which AEV agreed to
purchase the Chiongbian and Gothong groups' shares in WG&A's issued and
outstanding stock. Section 6.5 of the SPA provided for arbitration as the mode of settling
any dispute arising from the SPA. Section 6.8 of the SPA further provided that the
Agreement of January 8, 1996 shall be deemed terminated except its Annex SL-V.
4. The Escrow Agreement whereby ING Bank N.V.-Manila Branch was to take custody of the
shares subject of the SPA. Section 14.7 of the Escrow Agreement provided that all
disputes arising from it shall be settled via arbitration. DaCEIc
The obligation for WLI to acquire certain inventories of CAGLI and which is the subject of the present petition was
contained in Annex SL-V. It is therefore this agreement which deserves foremost consideration. As to this particular
agreement, these points must be underscored: first, that it has no arbitration clause; second, Annex SL-V is only between
WLI and CAGLI.
On the first point, it is clear, pursuant to this court's pronouncements in Ormoc Sugarcane Planters' Association,
that neither WLI nor CAGLI can compel arbitration under Annex SL-V. Plainly, there is no agreement to arbitrate.
It is of no moment that Annex SL-V states that it was made "pursuant to the Agreement" or that Section 11.06 of the
January 8, 1996 Agreement provides for arbitration as the mode of settling disputes arising out of or in connection with the
Agreement.
For one, to say that Annex SL-V was made "pursuant to the Agreement" is merely to acknowledge: (1) the factual
context in which Annex SL-V was executed and (2) that it was that context that facilitated the agreement embodied in it.
Absent any other clear or unequivocal pronouncement integrating Annex SL-V into the January 8, 1996 Agreement, it would
be too much of a conjecture to jump to the conclusion that Annex SL-V is governed by the exact same stipulations which
govern the January 8, 1996 Agreement.
Likewise, a reading of the Agreement's arbitration clause will reveal that it does not contemplate disputes arising
from Annex SL-V.
Section 11.06 of the January 8, 1996 Agreement requires the formation of an arbitration tribunal composed of four
(4) arbitrators. Each of the parties — WLI, CAGLI, and ASC — shall appoint one (1) arbitrator, and the fourth arbitrator, who
shall act as chairman, shall be appointed by the three (3) arbitrators appointed by the parties. From the manner by which
the arbitration tribunal is to be constituted, the necessary implication is that the arbitration clause is applicable to three-party
disputes — as will arise from the tripartite January 8, 1996 Agreement — and not to two-party disputes as will arise from the
two-party Annex SL-V.
From the second point — that Annex SL-V is only between WLI and CAGLI — it necessarily follows that none but
WLI/WG&A/ATSC and CAGLI are bound by the terms of Annex SL-V. It is elementary that contracts are characterized by
relativity or privity, that is, that "[c]ontracts take effect only between the parties, their assigns and heirs". 117 As such, one
who is not a party to a contract may not seek relief for such contract's breach. Likewise, one who is not a party to a contract
may not be held liable for breach of any its terms. HAEIac
While the principle of privity or relativity of contracts acknowledges that contractual obligations are transmissible to
a party's assigns and heirs, AEV is not WLI's successor-in-interest. In the period relevant to this petition, the transferee of
the inventories transferred by CAGLI pursuant to Annex SL-V assumed three (3) names: (1) WLI, the original name of the
entity that survived the merger under the January 8, 1996 Agreement; (2) WG&A, the name taken by WLI in the wake of the
Agreement; and (3) ATSC, the name taken by WLI/WG&A in the wake of the SPA. As such, it is now ATSC that is liable
under Annex SL-V.
Pursuant to the January 8, 1996 Agreement, the Aboitiz group (via ASC) and the Gothong group (via CAGLI)
became stockholders of WLI/WG&A, along with the Chiongbian group (which initially controlled WLI). This continued until,
pursuant to the SPA, the Gothong group and the Chiongbian group transferred their shares to AEV. With the SPA, AEV
became a stockholder of WLI/WG&A, which was subsequently renamed ATSC. Nonetheless, AEV's status as ATSC's
stockholder does not subject it to ATSC's obligations.
It is basic that a corporation has a personality separate and distinct from that of its individual stockholders. Thus, a
stockholder does not automatically assume the liabilities of the corporation of which he is a stockholder. As explained in
Philippine National Bank v. Hydro Resources Contractors Corporation: 118
A corporation is an artificial entity created by operation of law. It possesses the right of
succession and such powers, attributes, and properties expressly authorized by law or incident to its
existence. It has a personality separate and distinct from that of its stockholders and from that of other
corporations to which it may be connected. As a consequence of its status as a distinct legal entity and
as a result of a conscious policy decision to promote capital formation, a corporation incurs its own
liabilities and is legally responsible for payment of its obligations. In other words, by virtue of the separate
juridical personality of a corporation, the corporate debt or credit is not the debt or credit of the
stockholder. This protection from liability for shareholders is the principle of limited liability. 119
In fact, even the ownership by a single stockholder of all or nearly all the capital stock of a corporation is not, in and
of itself, a ground for disregarding a corporation's separate personality. As explained in Secosa v. Heirs of Francisco: 120
It is a settled precept in this jurisdiction that a corporation is invested by law with a personality
separate from that of its stockholders or members. It has a personality separate and distinct from those of
the persons composing it as well as from that of any other entity to which it may be related. Mere
ownership by a single stockholder or by another corporation of all or nearly all of the capital stock of a
corporation is not in itself sufficient ground for disregarding the separate corporate personality. A
corporation's authority to act and its liability for its actions are separate and apart from the individuals who
own it.
The so-called veil of corporation fiction treats as separate and distinct the affairs of a corporation
and its officers and stockholders. As a general rule, a corporation will be looked upon as a legal entity,
unless and until sufficient reason to the contrary appears. When the notion of legal entity is used to
defeat public convenience, justify wrong, protect fraud, or defend crime, the law will regard the
corporation as an association of persons. Also, the corporate entity may be disregarded in the interest of
justice in such cases as fraud that may work inequities among members of the corporation internally,
involving no rights of the public or third persons. In both instances, there must have been fraud and proof
of it. For the separate juridical personality of a corporation to be disregarded, the wrongdoing must be
clearly and convincingly established. It cannot be presumed. 121 (Emphasis supplied)
AEV's status as ATSC's stockholder is, in and of itself, insufficient to make AEV liable for ATSC's obligations.
Moreover, the SPA does not contain any stipulation which makes AEV assume ATSC's obligations. It is true that Section
6.8 of the SPA stipulates that the rights and obligations arising from Annex SL-V are not terminated. But all that Section 6.8
does is recognize that the obligations under Annex SL-V subsist despite the termination of the January 8, 1996 Agreement.
At no point does the text of Section 6.8 support the position that AEV steps into the shoes of the obligor under Annex SL-V
and assumes its obligations. HAEDIS
Neither does Section 6.5 of the SPA suffice to compel AEV to submit itself to arbitration. While it is true that Section
6.5 mandates arbitration as the mode for settling disputes between the parties to the SPA, Section 6.5 does not
indiscriminately cover any and all disputes which may arise between the parties to the SPA. Rather, Section 6.5 is limited to
"dispute[s] arising between the parties relating to this Agreement [i.e., the SPA]". 122 To belabor the point, the obligation
which is subject of the present dispute pertains to Annex SL-V, not to the SPA. That the SPA, in Section 6.8, recognizes the
subsistence of Annex SL-V is merely a factual recognition. It does not create new obligations and does not alter or modify
the obligations spelled out in Annex SL-V.
AEV was drawn into the present controversy on account of its having entered into the SPA. This SPA made AEV a
stockholder of WLI/WG&A/ATSC. Even then, AEV retained a personality separate and distinct from WLI/WG&A/ATSC. The
SPA did not render AEV personally liable for the obligations of the corporation whose stocks it held.
The obligation animating CAGLI's desire to arbitrate is rooted in Annex SL-V. Annex SL-V is a contract entirely
different from the SPA. It created distinct obligations for distinct parties. AEV was never a party to Annex SL-V. Rather than
pertaining to AEV, Annex SL-V pertained to a different entity: WLI (renamed WG&A then renamed ATSC). AEV is, thus, not
bound by Annex SL-V.
On one hand, Annex SL-V does not stipulate that disputes arising from it are to be settled via arbitration. On the
other hand, the SPA requires arbitration as the mode for settling disputes relating to it and recognizes the subsistence of the
obligations under Annex SL-V. But as a separate contract, the mere mention of Annex SL-V in the SPA does not suffice to
place Annex SL-V under the ambit of the SPA or to render it subject to the SPA's terms, such as the requirement to
arbitrate. cSIHCA
WHEREFORE, the petition is GRANTED. The assailed orders dated May 5, 2011 and June 24, 2011 of the
Regional Trial Court, Cebu City, Branch 10 in Civil Case No. CEB-37004 are declared VOID. The Regional Trial Court,
Cebu City, Branch 10 is ordered to DISMISS Civil Case No. CEB-37004. SO ORDERED.
||| (Aboitiz Equity Ventures, Inc. v. Chiongbian, G.R. No. 197530, [July 9, 2014], 738 PHIL 773-810)

[G.R. No. 181416. November 11, 2013.]


MEDICAL PLAZA MAKATI CONDOMINIUM CORPORATION, petitioner, vs. ROBERT H. CULLEN,
respondent.

DECISION

PERALTA, J p:

This is a petition for review on certiorari under Rule 45 of the Rules of Court assailing the Court of Appeals (CA)
Decision 1 dated July 10, 2007 and Resolution 2 dated January 25, 2008 in CA-G.R. CV No. 86614. The assailed decision
reversed and set aside the September 9, 2005 Order 3 of the Regional Trial Court (RTC) of Makati, Branch 58 in Civil Case
No. 03-1018; while the assailed resolution denied the separate motions for reconsideration filed by petitioner Medical Plaza
Makati Condominium Corporation (MPMCC) and Meridien Land Holding, Inc. (MLHI).
The factual and procedural antecedents are as follows:
Respondent Robert H. Cullen purchased from MLHI condominium Unit No. 1201 of the Medical Plaza Makati
covered by Condominium Certificate of Title No. 45808 of the Register of Deeds of Makati. Said title was later cancelled and
Condominium Certificate of Title No. 64218 was issued in the name of respondent.
On September 19, 2002, petitioner, through its corporate secretary, Dr. Jose Giovanni E. Dimayuga, demanded
from respondent payment for alleged unpaid association dues and assessments amounting to P145,567.42. Respondent
disputed this demand claiming that he had been religiously paying his dues shown by the fact that he was previously
elected president and director of petitioner. 4 Petitioner, on the other hand, claimed that respondent's obligation was a
carry-over of that of MLHI. 5 Consequently, respondent was prevented from exercising his right to vote and be voted for
during the 2002 election of petitioner's Board of Directors. 6 Respondent thus clarified from MLHI the veracity of petitioner's
claim, but MLHI allegedly claimed that the same had already been settled. 7 This prompted respondent to demand from
petitioner an explanation why he was considered a delinquent payer despite the settlement of the obligation. Petitioner
failed to make such explanation. Hence, the Complaint for Damages 8 filed by respondent against petitioner and MLHI, the
pertinent portions of which read: CSIHDA
xxx xxx xxx
6. Thereafter, plaintiff occupied the said condominium unit no. 1201 and religiously paid all the
corresponding monthly contributions/association dues and other assessments imposed on the same. For
the years 2000 and 2001, plaintiff served as President and Director of the Medical Plaza Makati
Condominium Corporation;
7. Nonetheless, on September 19, 2002, plaintiff was shocked/surprised to [receive] a letter from
the incumbent Corporate Secretary of the defendant Medical Plaza Makati, demanding payment of
alleged unpaid association dues and assessments arising from plaintiff's condominium unit no. 1201. The
said letter further stressed that plaintiff is considered a delinquent member of the defendant Medical
Plaza Makati. . . .;
8. As a consequence, plaintiff was not allowed to file his certificate of candidacy as director.
Being considered a delinquent, plaintiff was also barred from exercising his right to vote in the election of
new members of the Board of Directors . . .;
9. . . . Again, prior to the said election date, . . . counsel for the defendant [MPMCC] sent a
demand letter to plaintiff, anent the said delinquency, explaining that the said unpaid amount is a carry-
over from the obligation of defendant Meridien. . . .;
10. Verification with the defendant [MPMCC] resulted to the issuance of a certification stating that
Condominium Unit 1201 has an outstanding unpaid obligation in the total amount of P145,567.42 as of
November 30, 2002, which again, was attributed by defendant [MPMCC] to defendant Meridien. . . .;
DIETcC
11. Due to the seriousness of the matter, and the feeling that defendant Meridien made false
representations considering that it fully warranted to plaintiff that condominium unit 1201 is free and clear
from all liens and encumbrances, the matter was referred to counsel, who accordingly sent a letter to
defendant Meridien, to demand for the payment of said unpaid association dues and other assessments
imposed on the condominium unit and being claimed by defendant [MPMCC]. . . .;
12. . . . defendant Meridien claimed however, that the obligation does not exist considering that
the matter was already settled and paid by defendant Meridien to defendant [MPMCC]. . . .;
13. Plaintiff thus caused to be sent a letter to defendant [MPMCC] . . . . The said letter . . . sought
an explanation on the fact that, as per the letter of defendant Meridien, the delinquency of unit 1201 was
already fully paid and settled, contrary to the claim of defendant [MPMCC]. . . .;
14. Despite receipt of said letter on April 24, 2003, and to date however, no explanation was
given by defendant [MPMCC], to the damage and prejudice of plaintiff who is again obviously being
barred from voting/participating in the election of members of the board of directors for the year 2003;
15. Clearly, defendant [MPMCC] acted maliciously by insisting that plaintiff is a delinquent
member when in fact, defendant Meridien had already paid the said delinquency, if any. The branding of
plaintiff as delinquent member was willfully and deceitfully employed so as to prevent plaintiff from
exercising his right to vote or be voted as director of the condominium corporation;
16. Defendant [MPMCC]'s ominous silence when confronted with claim of payment made by
defendant Meridien is tantamount to admission that indeed, plaintiff is not really a delinquent member;
CScTED
17. Accordingly, as a direct and proximate result of the said acts of defendant [MPMCC], plaintiff
experienced/suffered from mental anguish, moral shock, and serious anxiety. Plaintiff, being a doctor of
medicine and respected in the community further suffered from social humiliation and besmirched
reputation thereby warranting the grant of moral damages in the amount of P500,000.00 and for which
defendant [MPMCC] should be held liable;
18. By way of example or correction for the public good, and as a stern warning to all similarly
situated, defendant [MPMCC] should be ordered to pay plaintiff exemplary damages in the amount of
P200,000.00;
[19]. As a consequence, and so as to protect his rights and interests, plaintiff was constrained to
hire the services of counsel, for an acceptance fee of P100,000.00 plus P2,500.00 per every court
hearing attended by counsel;
[20]. In the event that the claim of defendant [MPMCC] turned out to be true, however, the herein
defendant Meridien should be held liable instead, by ordering the same to pay the said delinquency of
condominium unit 1201 in the amount of P145,567.42 as of November 30, 2002 as well as the above
damages, considering that the non-payment thereof would be the proximate cause of the damages
suffered by plaintiff; 9
Petitioner and MLHI filed their separate motions to dismiss the complaint on the ground of lack of jurisdiction. 10
MLHI claims that it is the Housing and Land Use Regulatory Board (HLURB) which is vested with the exclusive jurisdiction
to hear and decide the case. Petitioner, on the other hand, raises the following specific grounds for the dismissal of the
complaint: (1) estoppel as respondent himself approved the assessment when he was the president; (2) lack of jurisdiction
as the case involves an intra-corporate controversy; (3) prematurity for failure of respondent to exhaust all intra-corporate
remedies; and (4) the case is already moot and academic, the obligation having been settled between petitioner and MLHI.
11 AIaSTE
On September 9, 2005, the RTC rendered a Decision granting petitioner's and MLHI's motions to dismiss and,
consequently, dismissing respondent's complaint.
The trial court agreed with MLHI that the action for specific performance filed by respondent clearly falls within the
exclusive jurisdiction of the HLURB. 12 As to petitioner, the court held that the complaint states no cause of action,
considering that respondent's obligation had already been settled by MLHI. It, likewise, ruled that the issues raised are intra-
corporate between the corporation and member. 13
On appeal, the CA reversed and set aside the trial court's decision and remanded the case to the RTC for further
proceedings. Contrary to the RTC conclusion, the CA held that the controversy is an ordinary civil action for damages which
falls within the jurisdiction of regular courts. 14 It explained that the case hinged on petitioner's refusal to confirm MLHI's
claim that the subject obligation had already been settled as early as 1998 causing damage to respondent. 15 Petitioner's
and MLHI's motions for reconsideration had also been denied. 16
Aggrieved, petitioner comes before the Court based on the following grounds:
I.
THE COURT A QUO HAS DECIDED A QUESTION OF SUBSTANCE, NOT THERETOFORE
DETERMINED BY THE SUPREME COURT, OR HAS DECIDED IT IN A WAY NOT IN ACCORD WITH
LAW OR WITH THE APPLICABLE DECISIONS OF THE SUPREME COURT WHEN IT DECLARED
THE INSTANT CASE AN ORDINARY ACTION FOR DAMAGES INSTEAD OF AN INTRA-CORPORATE
CONTROVERSY COGNIZABLE BY A SPECIAL COMMERCIAL COURT. cEHSIC
II.
THE COURT A QUO HAS DECIDED THE INSTANT CASE IN A WAY NOT IN ACCORD WITH
LAW OR WITH THE APPLICABLE DECISIONS OF THE SUPREME COURT WHEN IT TOOK
COGNIZANCE OF THE APPEAL WHILE RAISING ONLY PURE QUESTIONS OF LAW. 17
The petition is meritorious.
It is a settled rule that jurisdiction over the subject matter is determined by the allegations in the complaint. It is not
affected by the pleas or the theories set up by the defendant in an answer or a motion to dismiss. Otherwise, jurisdiction
would become dependent almost entirely upon the whims of the defendant. 18 Also illuminating is the Court's
pronouncement in Go v. Distinction Properties Development and Construction, Inc.: 19
Basic as a hornbook principle is that jurisdiction over the subject matter of a case is conferred by
law and determined by the allegations in the complaint which comprise a concise statement of the
ultimate facts constituting the plaintiff's cause of action. The nature of an action, as well as which court or
body has jurisdiction over it, is determined based on the allegations contained in the complaint of the
plaintiff, irrespective of whether or not the plaintiff is entitled to recover upon all or some of the claims
asserted therein. The averments in the complaint and the character of the relief sought are the ones
to be consulted. Once vested by the allegations in the complaint, jurisdiction also remains vested
irrespective of whether or not the plaintiff is entitled to recover upon all or some of the claims asserted
therein. . . . 20
Based on the allegations made by respondent in his complaint, does the controversy involve intra-corporate issues
as would fall within the jurisdiction of the RTC sitting as a special commercial court or an ordinary action for damages within
the jurisdiction of regular courts? AIECSD
In determining whether a dispute constitutes an intra-corporate controversy, the Court uses two tests, namely, the
relationship test and the nature of the controversy test. 21
An intra-corporate controversy is one which pertains to any of the following relationships: (1) between the
corporation, partnership or association and the public; (2) between the corporation, partnership or association and the State
insofar as its franchise, permit or license to operate is concerned; (3) between the corporation, partnership or association
and its stockholders, partners, members or officers; and (4) among the stockholders, partners or associates themselves. 22
Thus, under the relationship test, the existence of any of the above intra-corporate relations makes the case intra-corporate.
23
Under the nature of the controversy test, "the controversy must not only be rooted in the existence of an intra-
corporate relationship, but must as well pertain to the enforcement of the parties' correlative rights and obligations under the
Corporation Code and the internal and intra-corporate regulatory rules of the corporation." 24 In other words, jurisdiction
should be determined by considering both the relationship of the parties as well as the nature of the question involved. 25
Applying the two tests, we find and so hold that the case involves intra-corporate controversy. It obviously arose
from the intra-corporate relations between the parties, and the questions involved pertain to their rights and obligations
under the Corporation Code and matters relating to the regulation of the corporation. 26
Admittedly, petitioner is a condominium corporation duly organized and existing under Philippine laws, charged with
the management of the Medical Plaza Makati. Respondent, on the other hand, is the registered owner of Unit No. 1201 and
is thus a stockholder/member of the condominium corporation. Clearly, there is an intra-corporate relationship between the
corporation and a stockholder/member. HDIaST
The nature of the action is determined by the body rather than the title of the complaint. Though denominated as an
action for damages, an examination of the allegations made by respondent in his complaint shows that the case principally
dwells on the propriety of the assessment made by petitioner against respondent as well as the validity of petitioner's act in
preventing respondent from participating in the election of the corporation's Board of Directors. Respondent contested the
alleged unpaid dues and assessments demanded by petitioner.
The issue is not novel. The nature of an action involving any dispute as to the validity of the assessment of
association dues has been settled by the Court in Chateau de Baie Condominium Corporation v. Moreno. 27 In that case,
respondents therein filed a complaint for intra-corporate dispute against the petitioner therein to question how it calculated
the dues assessed against them, and to ask an accounting of association dues. Petitioner, however, moved for the
dismissal of the case on the ground of lack of jurisdiction alleging that since the complaint was against the owner/developer
of a condominium whose condominium project was registered with and licensed by the HLURB, the latter has the exclusive
jurisdiction. In sustaining the denial of the motion to dismiss, the Court held that the dispute as to the validity of the
assessments is purely an intra-corporate matter between petitioner and respondent and is thus within the exclusive
jurisdiction of the RTC sitting as a special commercial court. More so in this case as respondent repeatedly questioned his
characterization as a delinquent member and, consequently, petitioner's decision to bar him from exercising his rights to
vote and be voted for. These issues are clearly corporate and the demand for damages is just incidental. Being corporate in
nature, the issues should be threshed out before the RTC sitting as a special commercial court. The issues on damages can
still be resolved in the same special commercial court just like a regular RTC which is still competent to tackle civil law
issues incidental to intra-corporate disputes filed before it. 28
Moreover, Presidential Decree No. 902-A enumerates the cases over which the Securities and Exchange
Commission (SEC) exercises exclusive jurisdiction:
xxx xxx xxx
b) Controversies arising out of intra-corporate or partnership relations, between and among
stockholders, members or associates; between any or all of them and the corporation, partnership or
association of which they are stockholders, members, or associates, respectively; and between such
corporation, partnership or association and the State insofar as it concerns their individual franchise or
right to exist as such entity; and
c) Controversies in the election or appointment of directors, trustees, officers, or managers of
such corporations, partnerships, or associations. 29 cITAaD
To be sure, this action partakes of the nature of an intra-corporate controversy, the jurisdiction over which pertains
to the SEC. Pursuant to Section 5.2 of Republic Act No. 8799, otherwise known as the Securities Regulation Code, the
jurisdiction of the SEC over all cases enumerated under Section 5 of Presidential Decree No. 902-A has been transferred to
RTCs designated by this Court as Special Commercial Courts. 30 While the CA may be correct that the RTC has
jurisdiction, the case should have been filed not with the regular court but with the branch of the RTC designated as a
special commercial court. Considering that the RTC of Makati City, Branch 58 was not designated as a special commercial
court, it was not vested with jurisdiction over cases previously cognizable by the SEC. 31 The CA, therefore, gravely erred
in remanding the case to the RTC for further proceedings.
Indeed, Republic Act (RA) No. 9904, or the Magna Carta for Homeowners and Homeowners' Associations,
approved on January 7, 2010 and became effective on July 10, 2010, empowers the HLURB to hear and decide inter-
association and/or intra-association controversies or conflicts concerning homeowners' associations. However, we cannot
apply the same in the present case as it involves a controversy between a condominium unit owner and a condominium
corporation. While the term association as defined in the law covers homeowners' associations of other residential real
property which is broad enough to cover a condominium corporation, it does not seem to be the legislative intent. A
thorough review of the deliberations of the bicameral conference committee would show that the lawmakers did not intend
to extend the coverage of the law to such kind of association. We quote hereunder the pertinent portion of the Bicameral
Conference Committee's deliberation, to wit:
THE CHAIRMAN (SEN. ZUBIRI). Let's go back, Mr. Chair, very quickly on homeowners.
THE ACTING CHAIRMAN (REP. ZIALCITA). Ang sa akin lang, I think our views are similar,
Your Honor, Senator Zubiri, the entry of the condominium units might just complicate the whole matters.
So we'd like to put it on record that we're very much concerned about the plight of the Condominium Unit
Homeowners' Association. But this could very well be addressed on a separate bill that I'm willing to co-
sponsor with the distinguished Senator Zubiri, to address in the Condominium Act of the Philippines,
rather than address it here because it might just create a red herring into the entire thing and it will just
complicate matters, hindi ba?
THE CHAIRMAN (SEN. ZUBIRI). I also agree with you although I sympathize with them —
although we sympathize with them and we feel that many times their rights have been also violated by
abusive condominium corporations. However, there are certain things that we have to reconcile. There
are certain issues that we have to reconcile with this version.
In the Condominium Code, for example, they just raised a very peculiar situation under the
Condominium Code — Condominium Corporation Act. It's five years the proxy, whereas here, it's three
years. So there would already be violation or there will be already a problem with their version and our
version. Sino ang matutupad doon? Will it be our version or their version?
So I agree that has to be studied further. And because they have a law pertaining to the
condominium housing units, I personally feel that it would complicate matters if we include them.
Although I agree that they should be looked after and their problems be looked into. cTEICD
Probably we can ask our staff, Your Honor, to come up already with the bill although we have no
more time. Hopefully we can tackle this again on the 15th Congress. But I agree with the sentiments and
the inputs of the Honorable Chair of the House panel.
May we ask our resource persons to also probably give comments?
Atty. Dayrit.
MR. DAYRIT. Yes I agree with you. There are many, I think, practices in their provisions in the
Condominium Law that may be conflicting with this version of ours.
For instance, in the case of, let's say, the condominium, the so-called common areas and/or
maybe so called open spaces that they may have, especially common areas, they are usually owned by
the condominium corporation. Unlike a subdivision where the open spaces and/or the common areas are
not necessarily owned by the association. Because sometimes — generally these are donated to the
municipality or to the city. And it is only when the city or municipality gives the approval or the conformity
that this is donated to the homeowners' association. But generally, under PD [Presidential Decree] 957,
it's donated. In the Condominium Corporation, hindi. Lahat ng mga open spaces and common areas like
corridors, the function rooms and everything, are owned by the corporation. So that's one main issue that
can be conflicting.
THE CHAIRMAN (SEN. ZUBIRI). I'll just ask for a one-minute suspension so we can talk.
THE ACTING CHAIRMAN (REP. ZIALCITA). Unless you want to put a catchall phrase like what
we did in the Senior Citizen's Act. Something like, to the extent — paano ba iyon? To the extent that it is
practicable and applicable, the rights and benefits of the homeowners, are hereby extended to the —
mayroon kaming ginamit na phrase eh. . . to the extent that it be practicable and applicable to the unit
homeowners, is hereby extended, something like that. It's a catchall phrase. But then again, it might
create a. . .
MR. JALANDONI. It will become complicated. There will be a lot of conflict of laws between the
two laws.
THE ACTING CHAIRMAN (REP. ZIALCITA). Kaya nga eh. At saka, I don't know. I think the —
mayroon naman silang protection sa ano eh, di ba? Buyers decree doon sa Condominium Act. I'm sure
there are provisions there eh. Huwag na lang, huwag na lang.
MR. JALANDONI. Mr. Chairman, I think it would be best if your previous comments that you'd be
supporting an amendment. I think that would be — Well, that would be the best course of action with all
due respect.
THE ACTING CHAIRMAN (REP. ZIALCITA). Yeah. Okay. Thank you. So iyon na lang final
proposal naming 'yung catchall phrase, "With respect to the. . ." 32
xxx xxx xxx
THE CHAIRMAN (SEN. ZUBIRI). . . . And so, what is their final decision on the definition of
homeowners?
THE ACTING CHAIRMAN (REP. ZIALCITA). We stick to the original, Mr. Chairman. We'll just
open up a whole can of worms and a whole new ball game will come into play. Besides, I am not
authorized, neither are you, by our counterparts to include the condominium owners. IEAHca
THE CHAIRMAN (SEN. ZUBIRI). Basically that is correct. We are not authorized by the Senate
nor — because we have discussed this lengthily on the floor, actually, several months on the floor. And
we don't have the authority as well for other Bicam members to add a provision to include a separate
entity that has already their legal or their established Republic Act tackling on that particular issue. But we
just like to put on record, we sympathize with the plight of our friends in the condominium associations
and we will just guarantee them that we will work on an amendment to the Condominium Corporation
Code. So with that — we skipped, that is correct, we have to go back to homeowners' association
definition, Your Honor, because we had skipped it altogether. So just quickly going back to Page 7
because there are amendments to the definition of homeowners. If it is alright with the House Panel,
adopt the opening phrase of Subsection 7 of the Senate version as opening phrase of Subsection 10 of
the reconciled version.
xxx xxx xxx 33
To be sure, RA 4726 or the Condominium Act was enacted to specifically govern a condominium. Said law
sanctions the creation of the condominium corporation which is especially formed for the purpose of holding title to the
common area, in which the holders of separate interests shall automatically be members or shareholders, to the exclusion
of others, in proportion to the appurtenant interest of their respective units. 34 The rights and obligations of the
condominium unit owners and the condominium corporation are set forth in the above Act.
Clearly, condominium corporations are not covered by the amendment. Thus, the intra-corporate dispute between
petitioner and respondent is still within the jurisdiction of the RTC sitting as a special commercial court and not the HLURB.
The doctrine laid down by the Court in Chateau de Baie Condominium Corporation v. Moreno 35 which in turn cited Wack
Wack Condominium Corporation, et al. v. CA 36 is still a good law.
WHEREFORE, we hereby GRANT the petition and REVERSE the Court of Appeals Decision dated July 10, 2007
and Resolution dated January 25, 2008 in CA-G.R. CV No. 86614. The Complaint before the Regional Trial Court of Makati
City, Branch 58, which is not a special commercial court, docketed as Civil Case No. 03-1018 is ordered DISMISSED for
lack of jurisdiction. Let the case REMANDED to the Executive Judge of the Regional Trial Court of Makati City for re-raffle
purposes among the designated special commercial courts.
SO ORDERED.
||| (Medical Plaza Makati Condominium Corp. v. Cullen, G.R. No. 181416, [November 11, 2013], 720 PHIL 732-749)

[G.R. No. 176579. June 28, 2011.]

WILSON P. GAMBOA, petitioner, vs. FINANCE SECRETARY MARGARITO B. TEVES, FINANCE


UNDERSECRETARY JOHN P. SEVILLA, AND COMMISSIONER RICARDO ABCEDE OF THE
PRESIDENTIAL COMMISSION ON GOOD GOVERNMENT (PCGG) IN THEIR CAPACITIES AS CHAIR
AND MEMBERS, RESPECTIVELY, OF THE PRIVATIZATION COUNCIL, CHAIRMAN ANTHONI
SALIM OF FIRST PACIFIC CO., LTD. IN HIS CAPACITY AS DIRECTOR OF METRO PACIFIC ASSET
HOLDINGS INC., CHAIRMAN MANUEL V. PANGILINAN OF PHILIPPINE LONG DISTANCE
TELEPHONE COMPANY (PLDT) IN HIS CAPACITY AS MANAGING DIRECTOR OF FIRST PACIFIC
CO., LTD., PRESIDENT NAPOLEON L. NAZARENO OF PHILIPPINE LONG DISTANCE TELEPHONE
COMPANY, CHAIR FE BARIN OF THE SECURITIES EXCHANGE COMMISSION, and PRESIDENT
FRANCIS LIM OF THE PHILIPPINE STOCK EXCHANGE, respondents.

PABLITO V. SANIDAD and ARNO V. SANIDAD, petitioners-in-intervention.

DECISION

CARPIO, J p:

The Case
This is an original petition for prohibition, injunction, declaratory relief and declaration of nullity of the sale of
shares of stock of Philippine Telecommunications Investment Corporation (PTIC) by the government of the Republic of
the Philippines to Metro Pacific Assets Holdings, Inc. (MPAH), an affiliate of First Pacific Company Limited (First
Pacific).
The Antecedents
The facts, according to petitioner Wilson P. Gamboa, a stockholder of Philippine Long Distance Telephone
Company (PLDT), are as follows: 1
On 28 November 1928, the Philippine Legislature enacted Act No. 3436 which granted PLDT a franchise and
the right to engage in telecommunications business. In 1969, General Telephone and Electronics Corporation (GTE), an
American company and a major PLDT stockholder, sold 26 percent of the outstanding common shares of PLDT to
PTIC. In 1977, Prime Holdings, Inc. (PHI) was incorporated by several persons, including Roland Gapud and Jose
Campos, Jr. Subsequently, PHI became the owner of 111,415 shares of stock of PTIC by virtue of three Deeds of
Assignment executed by PTIC stockholders Ramon Cojuangco and Luis Tirso Rivilla. In 1986, the 111,415 shares of
stock of PTIC held by PHI were sequestered by the Presidential Commission on Good Government (PCGG). The
111,415 PTIC shares, which represent about 46.125 percent of the outstanding capital stock of PTIC, were later
declared by this Court to be owned by the Republic of the Philippines. 2
In 1999, First Pacific, a Bermuda-registered, Hong Kong-based investment firm, acquired the remaining 54
percent of the outstanding capital stock of PTIC. On 20 November 2006, the Inter-Agency Privatization Council (IPC) of
the Philippine Government announced that it would sell the 111,415 PTIC shares, or 46.125 percent of the outstanding
capital stock of PTIC, through a public bidding to be conducted on 4 December 2006. Subsequently, the public bidding
was reset to 8 December 2006, and only two bidders, Parallax Venture Fund XXVII (Parallax) and Pan-Asia Presidio
Capital, submitted their bids. Parallax won with a bid of P25.6 billion or US$510 million. TAESDH
Thereafter, First Pacific announced that it would exercise its right of first refusal as a PTIC stockholder and buy
the 111,415 PTIC shares by matching the bid price of Parallax. However, First Pacific failed to do so by the 1 February
2007 deadline set by IPC and instead, yielded its right to PTIC itself which was then given by IPC until 2 March 2007 to
buy the PTIC shares. On 14 February 2007, First Pacific, through its subsidiary, MPAH, entered into a Conditional Sale
and Purchase Agreement of the 111,415 PTIC shares, or 46.125 percent of the outstanding capital stock of PTIC, with
the Philippine Government for the price of P25,217,556,000 or US$510,580,189. The sale was completed on 28
February 2007.
Since PTIC is a stockholder of PLDT, the sale by the Philippine Government of 46.125 percent of PTIC shares
is actually an indirect sale of 12 million shares or about 6.3 percent of the outstanding common shares of PLDT. With
the sale, First Pacific's common shareholdings in PLDT increased from 30.7 percent to 37 percent, thereby
increasing the common shareholdings of foreigners in PLDT to about 81.47 percent. This violates Section 11,
Article XII of the 1987 Philippine Constitution which limits foreign ownership of the capital of a public utility to not more
than 40 percent. 3
On the other hand, public respondents Finance Secretary Margarito B. Teves, Undersecretary John P. Sevilla,
and PCGG Commissioner Ricardo Abcede allege the following relevant facts:
On 9 November 1967, PTIC was incorporated and had since engaged in the business of investment holdings.
PTIC held 26,034,263 PLDT common shares, or 13.847 percent of the total PLDT outstanding common shares. PHI, on
the other hand, was incorporated in 1977, and became the owner of 111,415 PTIC shares or 46.125 percent of the
outstanding capital stock of PTIC by virtue of three Deeds of Assignment executed by Ramon Cojuangco and Luis Tirso
Rivilla. In 1986, the 111,415 PTIC shares held by PHI were sequestered by the PCGG, and subsequently declared by
this Court as part of the ill-gotten wealth of former President Ferdinand Marcos. The sequestered PTIC shares were
reconveyed to the Republic of the Philippines in accordance with this Court's decision 4 which became final and
executory on 8 August 2006.
The Philippine Government decided to sell the 111,415 PTIC shares, which represent 6.4 percent of the
outstanding common shares of stock of PLDT, and designated the Inter-Agency Privatization Council (IPC), composed
of the Department of Finance and the PCGG, as the disposing entity. An invitation to bid was published in seven
different newspapers from 13 to 24 November 2006. On 20 November 2006, a pre-bid conference was held, and the
original deadline for bidding scheduled on 4 December 2006 was reset to 8 December 2006. The extension was
published in nine different newspapers.
During the 8 December 2006 bidding, Parallax Capital Management LP emerged as the highest bidder with a
bid of P25,217,556,000. The government notified First Pacific, the majority owner of PTIC shares, of the bidding results
and gave First Pacific until 1 February 2007 to exercise its right of first refusal in accordance with PTIC's Articles of
Incorporation. First Pacific announced its intention to match Parallax's bid.
On 31 January 2007, the House of Representatives (HR) Committee on Good Government conducted a public
hearing on the particulars of the then impending sale of the 111,415 PTIC shares. Respondents Teves and Sevilla were
among those who attended the public hearing. The HR Committee Report No. 2270 concluded that: (a) the auction of
the government's 111,415 PTIC shares bore due diligence, transparency and conformity with existing legal procedures;
and (b) First Pacific's intended acquisition of the government's 111,415 PTIC shares resulting in First Pacific's
100% ownership of PTIC will not violate the 40 percent constitutional limit on foreign ownership of a public
utility since PTIC holds only 13.847 percent of the total outstanding common shares of PLDT. 5 On 28 February
2007, First Pacific completed the acquisition of the 111,415 shares of stock of PTIC.
Respondent Manuel V. Pangilinan admits the following facts: (a) the IPC conducted a public bidding for the sale
of 111,415 PTIC shares or 46 percent of the outstanding capital stock of PTIC (the remaining 54 percent of PTIC shares
was already owned by First Pacific and its affiliates); (b) Parallax offered the highest bid amounting to P25,217,556,000;
(c) pursuant to the right of first refusal in favor of PTIC and its shareholders granted in PTIC's Articles of Incorporation,
MPAH, a First Pacific affiliate, exercised its right of first refusal by matching the highest bid offered for PTIC shares on
13 February 2007; and (d) on 28 February 2007, the sale was consummated when MPAH paid IPC P25,217,556,000
and the government delivered the certificates for the 111,415 PTIC shares. Respondent Pangilinan denies the other
allegations of facts of petitioner. HcaDIA
On 28 February 2007, petitioner filed the instant petition for prohibition, injunction, declaratory relief, and
declaration of nullity of sale of the 111,415 PTIC shares. Petitioner claims, among others, that the sale of the 111,415
PTIC shares would result in an increase in First Pacific's common shareholdings in PLDT from 30.7 percent to 37
percent, and this, combined with Japanese NTT DoCoMo's common shareholdings in PLDT, would result to a total
foreign common shareholdings in PLDT of 51.56 percent which is over the 40 percent constitutional limit. 6 Petitioner
asserts:
If and when the sale is completed, First Pacific's equity in PLDT will go up from 30.7 percent to
37.0 percent of its common — or voting-stockholdings, . . . . Hence, the consummation of the sale will put
the two largest foreign investors in PLDT — First Pacific and Japan's NTT DoCoMo, which is the world's
largest wireless telecommunications firm, owning 51.56 percent of PLDT common equity. . . . With the
completion of the sale, data culled from the official website of the New York Stock Exchange
(www.nyse.com) showed that those foreign entities, which own at least five percent of common equity,
will collectively own 81.47 percent of PLDT's common equity. . . .
. . . as the annual disclosure reports, also referred to as Form 20-K reports . . . which PLDT
submitted to the New York Stock Exchange for the period 2003-2005, revealed that First Pacific and
several other foreign entities breached the constitutional limit of 40 percent ownership as early as
2003. . . ." 7
Petitioner raises the following issues: (1) whether the consummation of the then impending sale of 111,415
PTIC shares to First Pacific violates the constitutional limit on foreign ownership of a public utility; (2) whether public
respondents committed grave abuse of discretion in allowing the sale of the 111,415 PTIC shares to First Pacific; and
(3) whether the sale of common shares to foreigners in excess of 40 percent of the entire subscribed common capital
stock violates the constitutional limit on foreign ownership of a public utility. 8
On 13 August 2007, Pablito V. Sanidad and Arno V. Sanidad filed a Motion for Leave to Intervene and Admit
Attached Petition-in-Intervention. In the Resolution of 28 August 2007, the Court granted the motion and noted the
Petition-in-Intervention. caIACE
Petitioners-in-intervention "join petitioner Wilson Gamboa . . . in seeking, among others, to enjoin and/or nullify
the sale by respondents of the 111,415 PTIC shares to First Pacific or assignee." Petitioners-in-intervention claim that,
as PLDT subscribers, they have a "stake in the outcome of the controversy . . . where the Philippine Government is
completing the sale of government owned assets in [PLDT], unquestionably a public utility, in violation of the nationality
restrictions of the Philippine Constitution."
The Issue
This Court is not a trier of facts. Factual questions such as those raised by petitioner, 9 which indisputably
demand a thorough examination of the evidence of the parties, are generally beyond this Court's jurisdiction. Adhering
to this well-settled principle, the Court shall confine the resolution of the instant controversy solely on the threshold and
purely legal issue of whether the term "capital" in Section 11, Article XII of the Constitution refers to the total common
shares only or to the total outstanding capital stock (combined total of common and non-voting preferred shares) of
PLDT, a public utility.
The Ruling of the Court
The petition is partly meritorious.
Petition for declaratory relief treated as petition for mandamus
At the outset, petitioner is faced with a procedural barrier. Among the remedies petitioner seeks, only the
petition for prohibition is within the original jurisdiction of this court, which however is not exclusive but is concurrent with
the Regional Trial Court and the Court of Appeals. The actions for declaratory relief, 10 injunction, and annulment of
sale are not embraced within the original jurisdiction of the Supreme Court. On this ground alone, the petition could
have been dismissed outright.
While direct resort to this Court may be justified in a petition for prohibition, 11 the Court shall nevertheless
refrain from discussing the grounds in support of the petition for prohibition since on 28 February 2007, the questioned
sale was consummated when MPAH paid IPC P25,217,556,000 and the government delivered the certificates for the
111,415 PTIC shares.
However, since the threshold and purely legal issue on the definition of the term "capital" in Section 11, Article
XII of the Constitution has far-reaching implications to the national economy, the Court treats the petition for declaratory
relief as one for mandamus. 12
In Salvacion v. Central Bank of the Philippines, 13 the Court treated the petition for declaratory relief as one for
mandamus considering the grave injustice that would result in the interpretation of a banking law. In that case, which
involved the crime of rape committed by a foreign tourist against a Filipino minor and the execution of the final judgment
in the civil case for damages on the tourist's dollar deposit with a local bank, the Court declared Section 113 of Central
Bank Circular No. 960, exempting foreign currency deposits from attachment, garnishment or any other order or
process of any court, inapplicable due to the peculiar circumstances of the case. The Court held that "injustice would
result especially to a citizen aggrieved by a foreign guest like accused . . . " that would "negate Article 10 of the Civil
Code which provides that 'in case of doubt in the interpretation or application of laws, it is presumed that the lawmaking
body intended right and justice to prevail.'" The Court therefore required respondents Central Bank of the Philippines,
the local bank, and the accused to comply with the writ of execution issued in the civil case for damages and to release
the dollar deposit of the accused to satisfy the judgment. ICHcTD
In Alliance of Government Workers v. Minister of Labor, 14 the Court similarly brushed aside the procedural
infirmity of the petition for declaratory relief and treated the same as one for mandamus. In Alliance, the issue was
whether the government unlawfully excluded petitioners, who were government employees, from the enjoyment of
rights to which they were entitled under the law. Specifically, the question was: "Are the branches, agencies,
subdivisions, and instrumentalities of the Government, including government owned or controlled corporations included
among the four 'employers' under Presidential Decree No. 851 which are required to pay their employees . . . a
thirteenth (13th) month pay . . .?" The Constitutional principle involved therein affected all government employees,
clearly justifying a relaxation of the technical rules of procedure, and certainly requiring the interpretation of the assailed
presidential decree.
In short, it is well-settled that this Court may treat a petition for declaratory relief as one for mandamus if the
issue involved has far-reaching implications. As this Court held in Salvacion:
The Court has no original and exclusive jurisdiction over a petition for declaratory relief.
However, exceptions to this rule have been recognized. Thus, where the petition has far-reaching
implications and raises questions that should be resolved, it may be treated as one for
mandamus. 15 (Emphasis supplied)
In the present case, petitioner seeks primarily the interpretation of the term "capital" in Section 11, Article XII of
the Constitution. He prays that this Court declare that the term "capital" refers to common shares only, and that such
shares constitute "the sole basis in determining foreign equity in a public utility." Petitioner further asks this Court to
declare any ruling inconsistent with such interpretation unconstitutional.
The interpretation of the term "capital" in Section 11, Article XII of the Constitution has far-reaching implications
to the national economy. In fact, a resolution of this issue will determine whether Filipinos are masters, or second class
citizens, in their own country. What is at stake here is whether Filipinos or foreigners will have effective control of the
national economy. Indeed, if ever there is a legal issue that has far-reaching implications to the entire nation, and to
future generations of Filipinos, it is the threshhold legal issue presented in this case. CAIHTE
The Court first encountered the issue on the definition of the term "capital" in Section 11, Article XII of the
Constitution in the case of Fernandez v. Cojuangco, docketed as G.R. No. 157360. 16 That case involved the same
public utility (PLDT) and substantially the same private respondents. Despite the importance and novelty of the
constitutional issue raised therein and despite the fact that the petition involved a purely legal question, the Court
declined to resolve the case on the merits, and instead denied the same for disregarding the hierarchy of courts. 17
There, petitioner Fernandez assailed on a pure question of law the Regional Trial Court's Decision of 21 February 2003
via a petition for review under Rule 45. The Court's Resolution, denying the petition, became final on 21 December
2004.
The instant petition therefore presents the Court with another opportunity to finally settle this purely legal issue
which is of transcendental importance to the national economy and a fundamental requirement to a faithful adherence
to our Constitution. The Court must forthwith seize such opportunity, not only for the benefit of the litigants, but more
significantly for the benefit of the entire Filipino people, to ensure, in the words of the Constitution, "a self-reliant and
independent national economy effectively controlled by Filipinos." 18 Besides, in the light of vague and confusing
positions taken by government agencies on this purely legal issue, present and future foreign investors in this country
deserve, as a matter of basic fairness, a categorical ruling from this Court on the extent of their participation in the
capital of public utilities and other nationalized businesses.
Despite its far-reaching implications to the national economy, this purely legal issue has remained unresolved
for over 75 years since the 1935 Constitution. There is no reason for this Court to evade this ever recurring fundamental
issue and delay again defining the term "capital," which appears not only in Section 11, Article XII of the Constitution,
but also in Section 2, Article XII on co-production and joint venture agreements for the development of our natural
resources, 19 in Section 7, Article XII on ownership of private lands, 20 in Section 10, Article XII on the reservation of
certain investments to Filipino citizens, 21 in Section 4 (2), Article XIV on the ownership of educational institutions, 22
and in Section 11 (2), Article XVI on the ownership of advertising companies. 23
Petitioner has locus standi
There is no dispute that petitioner is a stockholder of PLDT. As such, he has the right to question the subject
sale, which he claims to violate the nationality requirement prescribed in Section 11, Article XII of the Constitution. If the
sale indeed violates the Constitution, then there is a possibility that PLDT's franchise could be revoked, a dire
consequence directly affecting petitioner's interest as a stockholder. DCIEac
More importantly, there is no question that the instant petition raises matters of transcendental importance to
the public. The fundamental and threshold legal issue in this case, involving the national economy and the economic
welfare of the Filipino people, far outweighs any perceived impediment in the legal personality of the petitioner to bring
this action.
In Chavez v. PCGG, 24 the Court upheld the right of a citizen to bring a suit on matters of transcendental
importance to the public, thus:
In Tañada v. Tuvera, the Court asserted that when the issue concerns a public right and the
object of mandamus is to obtain the enforcement of a public duty, the people are regarded as the
real parties in interest; and because it is sufficient that petitioner is a citizen and as such is
interested in the execution of the laws, he need not show that he has any legal or special interest
in the result of the action. In the aforesaid case, the petitioners sought to enforce their right to be
informed on matters of public concern, a right then recognized in Section 6, Article IV of the 1973
Constitution, in connection with the rule that laws in order to be valid and enforceable must be published
in the Official Gazette or otherwise effectively promulgated. In ruling for the petitioners' legal standing, the
Court declared that the right they sought to be enforced 'is a public right recognized by no less than the
fundamental law of the land.'
Legaspi v. Civil Service Commission, while reiterating Tañada, further declared that 'when a
mandamus proceeding involves the assertion of a public right, the requirement of personal
interest is satisfied by the mere fact that petitioner is a citizen and, therefore, part of the general
'public' which possesses the right.'
Further, in Albano v. Reyes, we said that while expenditure of public funds may not have been
involved under the questioned contract for the development, management and operation of the Manila
International Container Terminal, 'public interest [was] definitely involved considering the important
role [of the subject contract] . . . in the economic development of the country and the magnitude
of the financial consideration involved.' We concluded that, as a consequence, the disclosure
provision in the Constitution would constitute sufficient authority for upholding the petitioner's standing.
(Emphasis supplied) DIHETS
Clearly, since the instant petition, brought by a citizen, involves matters of transcendental public importance,
the petitioner has the requisite locus standi.
Definition of the Term "Capital" in
Section 11, Article XII of the 1987 Constitution
Section 11, Article XII (National Economy and Patrimony) of the 1987 Constitution mandates the Filipinization
of public utilities, to wit:
Section 11. No franchise, certificate, or any other form of authorization for the operation of
a public utility shall be granted except to citizens of the Philippines or to corporations or
associations organized under the laws of the Philippines, at least sixty per centum of whose
capital is owned by such citizens; nor shall such franchise, certificate, or authorization be exclusive in
character or for a longer period than fifty years. Neither shall any such franchise or right be granted
except under the condition that it shall be subject to amendment, alteration, or repeal by the Congress
when the common good so requires. The State shall encourage equity participation in public utilities by
the general public. The participation of foreign investors in the governing body of any public utility
enterprise shall be limited to their proportionate share in its capital, and all the executive and managing
officers of such corporation or association must be citizens of the Philippines. (Emphasis supplied)
The above provision substantially reiterates Section 5, Article XIV of the 1973 Constitution, thus:
Section 5. No franchise, certificate, or any other form of authorization for the operation of
a public utility shall be granted except to citizens of the Philippines or to corporations or
associations organized under the laws of the Philippines at least sixty per centum of the capital of
which is owned by such citizens, nor shall such franchise, certificate, or authorization be exclusive in
character or for a longer period than fifty years. Neither shall any such franchise or right be granted
except under the condition that it shall be subject to amendment, alteration, or repeal by the National
Assembly when the public interest so requires. The State shall encourage equity participation in public
utilities by the general public. The participation of foreign investors in the governing body of any public
utility enterprise shall be limited to their proportionate share in the capital thereof. (Emphasis supplied)
THAICD
The foregoing provision in the 1973 Constitution reproduced Section 8, Article XIV of the 1935 Constitution,
viz.:
Section 8. No franchise, certificate, or any other form of authorization for the operation of
a public utility shall be granted except to citizens of the Philippines or to corporations or other
entities organized under the laws of the Philippines sixty per centum of the capital of which is
owned by citizens of the Philippines, nor shall such franchise, certificate, or authorization be exclusive
in character or for a longer period than fifty years. No franchise or right shall be granted to any individual,
firm, or corporation, except under the condition that it shall be subject to amendment, alteration, or repeal
by the Congress when the public interest so requires. (Emphasis supplied)

Father Joaquin G. Bernas, S.J., a leading member of the 1986 Constitutional Commission, reminds us that the
Filipinization provision in the 1987 Constitution is one of the products of the spirit of nationalism which gripped the 1935
Constitutional Convention. 25 The 1987 Constitution "provides for the Filipinization of public utilities by requiring that
any form of authorization for the operation of public utilities should be granted only to 'citizens of the Philippines or to
corporations or associations organized under the laws of the Philippines at least sixty per centum of whose capital is
owned by such citizens.' The provision is [an express] recognition of the sensitive and vital position of public
utilities both in the national economy and for national security." 26 The evident purpose of the citizenship
requirement is to prevent aliens from assuming control of public utilities, which may be inimical to the national interest.
27 This specific provision explicitly reserves to Filipino citizens control of public utilities, pursuant to an overriding
economic goal of the 1987 Constitution: to "conserve and develop our patrimony" 28 and ensure "a self-reliant and
independent national economy effectively controlled by Filipinos." 29
Any citizen or juridical entity desiring to operate a public utility must therefore meet the minimum nationality
requirement prescribed in Section 11, Article XII of the Constitution. Hence, for a corporation to be granted authority to
operate a public utility, at least 60 percent of its "capital" must be owned by Filipino citizens.
The crux of the controversy is the definition of the term "capital." Does the term "capital" in Section 11, Article
XII of the Constitution refer to common shares or to the total outstanding capital stock (combined total of common and
non-voting preferred shares)?
Petitioner submits that the 40 percent foreign equity limitation in domestic public utilities refers only to common
shares because such shares are entitled to vote and it is through voting that control over a corporation is exercised.
Petitioner posits that the term "capital" in Section 11, Article XII of the Constitution refers to "the ownership of common
capital stock subscribed and outstanding, which class of shares alone, under the corporate set-up of PLDT, can vote
and elect members of the board of directors." It is undisputed that PLDT's non-voting preferred shares are held mostly
by Filipino citizens. 30 This arose from Presidential Decree No. 217, 31 issued on 16 June 1973 by then President
Ferdinand Marcos, requiring every applicant of a PLDT telephone line to subscribe to non-voting preferred shares to
pay for the investment cost of installing the telephone line. 32
Petitioners-in-intervention basically reiterate petitioner's arguments and adopt petitioner's definition of the term
"capital." 33 Petitioners-in-intervention allege that "the approximate foreign ownership of common capital stock of
PLDT . . . already amounts to at least 63.54% of the total outstanding common stock," which means that foreigners
exercise significant control over PLDT, patently violating the 40 percent foreign equity limitation in public utilities
prescribed by the Constitution.
Respondents, on the other hand, do not offer any definition of the term "capital" in Section 11, Article XII of the
Constitution. More importantly, private respondents Nazareno and Pangilinan of PLDT do not dispute that more than 40
percent of the common shares of PLDT are held by foreigners.
In particular, respondent Nazareno's Memorandum, consisting of 73 pages, harps mainly on the procedural
infirmities of the petition and the supposed violation of the due process rights of the "affected foreign common
shareholders." Respondent Nazareno does not deny petitioner's allegation of foreigners' dominating the common
shareholdings of PLDT. Nazareno stressed mainly that the petition "seeks to divest foreign common shareholders
purportedly exceeding 40% of the total common shareholdings in PLDT of their ownership over their shares."
Thus, "the foreign natural and juridical PLDT shareholders must be impleaded in this suit so that they can be heard." 34
Essentially, Nazareno invokes denial of due process on behalf of the foreign common shareholders.
While Nazareno does not introduce any definition of the term "capital," he states that "among the factual
assertions that need to be established to counter petitioner's allegations is the uniform interpretation by
government agencies (such as the SEC), institutions and corporations (such as the Philippine National Oil
Company-Energy Development Corporation or PNOC-EDC) of including both preferred shares and common
shares in "controlling interest" in view of testing compliance with the 40% constitutional limitation on foreign
ownership in public utilities." 35
Similarly, respondent Manuel V. Pangilinan does not define the term "capital" in Section 11, Article XII of the
Constitution. Neither does he refute petitioner's claim of foreigners holding more than 40 percent of PLDT's common
shares. Instead, respondent Pangilinan focuses on the procedural flaws of the petition and the alleged violation of the
due process rights of foreigners. Respondent Pangilinan emphasizes in his Memorandum (1) the absence of this
Court's jurisdiction over the petition; (2) petitioner's lack of standing; (3) mootness of the petition; (4) non-availability of
declaratory relief; and (5) the denial of due process rights. Moreover, respondent Pangilinan alleges that the issue
should be whether "owners of shares in PLDT as well as owners of shares in companies holding shares in PLDT may
be required to relinquish their shares in PLDT and in those companies without any law requiring them to surrender their
shares and also without notice and trial."
Respondent Pangilinan further asserts that "Section 11, [Article XII of the Constitution] imposes no
nationality requirement on the shareholders of the utility company as a condition for keeping their shares in
the utility company." According to him, "Section 11 does not authorize taking one person's property (the shareholder's
stock in the utility company) on the basis of another party's alleged failure to satisfy a requirement that is a condition
only for that other party's retention of another piece of property (the utility company being at least 60% Filipino-owned to
keep its franchise)." 36
The OSG, representing public respondents Secretary Margarito Teves, Undersecretary John P. Sevilla,
Commissioner Ricardo Abcede, and Chairman Fe Barin, is likewise silent on the definition of the term "capital." In its
Memorandum 37 dated 24 September 2007, the OSG also limits its discussion on the supposed procedural defects of
the petition, i.e., lack of standing, lack of jurisdiction, non-inclusion of interested parties, and lack of basis for injunction.
The OSG does not present any definition or interpretation of the term "capital" in Section 11, Article XII of the
Constitution. The OSG contends that "the petition actually partakes of a collateral attack on PLDT's franchise as a
public utility," which in effect requires a "full-blown trial where all the parties in interest are given their day in court." 38
Respondent Francisco Ed Lim, impleaded as President and Chief Executive Officer of the Philippine Stock
Exchange (PSE), does not also define the term "capital" and seeks the dismissal of the petition on the following
grounds: (1) failure to state a cause of action against Lim; (2) the PSE allegedly implemented its rules and required all
listed companies, including PLDT, to make proper and timely disclosures; and (3) the reliefs prayed for in the petition
would adversely impact the stock market.
In the earlier case of Fernandez v. Cojuangco, petitioner Fernandez who claimed to be a stockholder of record
of PLDT, contended that the term "capital" in the 1987 Constitution refers to shares entitled to vote or the common
shares. Fernandez explained thus: DHATcE
The forty percent (40%) foreign equity limitation in public utilities prescribed by the Constitution
refers to ownership of shares of stock entitled to vote, i.e., common shares, considering that it is through
voting that control is being exercised. . . .
Obviously, the intent of the framers of the Constitution in imposing limitations and restrictions on
fully nationalized and partially nationalized activities is for Filipino nationals to be always in control of the
corporation undertaking said activities. Otherwise, if the Trial Court's ruling upholding respondents'
arguments were to be given credence, it would be possible for the ownership structure of a public utility
corporation to be divided into one percent (1%) common stocks and ninety-nine percent (99%) preferred
stocks. Following the Trial Court's ruling adopting respondents' arguments, the common shares can be
owned entirely by foreigners thus creating an absurd situation wherein foreigners, who are supposed to
be minority shareholders, control the public utility corporation.
xxx xxx xxx
Thus, the 40% foreign ownership limitation should be interpreted to apply to both the beneficial
ownership and the controlling interest.
xxx xxx xxx
Clearly, therefore, the forty percent (40%) foreign equity limitation in public utilities prescribed by
the Constitution refers to ownership of shares of stock entitled to vote, i.e., common shares. Furthermore,
ownership of record of shares will not suffice but it must be shown that the legal and beneficial ownership
rests in the hands of Filipino citizens. Consequently, in the case of petitioner PLDT, since it is already
admitted that the voting interests of foreigners which would gain entry to petitioner PLDT by the
acquisition of SMART shares through the Questioned Transactions is equivalent to 82.99%, and the
nominee arrangements between the foreign principals and the Filipino owners is likewise admitted, there
is, therefore, a violation of Section 11, Article XII of the Constitution. EIDATc
Parenthetically, the Opinions dated February 15, 1988 and April 14, 1987 cited by the Trial Court
to support the proposition that the meaning of the word "capital" as used in Section 11, Article XII of the
Constitution allegedly refers to the sum total of the shares subscribed and paid-in by the shareholder and
it allegedly is immaterial how the stock is classified, whether as common or preferred, cannot stand in the
face of a clear legislative policy as stated in the FIA which took effect in 1991 or way after said opinions
were rendered, and as clarified by the above-quoted Amendments. In this regard, suffice it to state that
as between the law and an opinion rendered by an administrative agency, the law indubitably prevails.
Moreover, said Opinions are merely advisory and cannot prevail over the clear intent of the framers of the
Constitution.
In the same vein, the SEC's construction of Section 11, Article XII of the Constitution is at best
merely advisory for it is the courts that finally determine what a law means. 39
On the other hand, respondents therein, Antonio O. Cojuangco, Manuel V. Pangilinan, Carlos A. Arellano,
Helen Y. Dee, Magdangal B. Elma, Mariles Cacho-Romulo, Fr. Bienvenido F. Nebres, Ray C. Espinosa, Napoleon L.
Nazareno, Albert F. Del Rosario, and Orlando B. Vea, argued that the term "capital" in Section 11, Article XII of the
Constitution includes preferred shares since the Constitution does not distinguish among classes of stock, thus:
16. The Constitution applies its foreign ownership limitation on the corporation's "capital," without
distinction as to classes of shares. . . .
In this connection, the Corporation Code — which was already in force at the time the present
(1987) Constitution was drafted — defined outstanding capital stock as follows:
Section 137. Outstanding capital stock defined. — The term "outstanding capital stock",
as used in this Code, means the total shares of stock issued under binding subscription
agreements to subscribers or stockholders, whether or not fully or partially paid, except treasury
shares.
Section 137 of the Corporation Code also does not distinguish between common and preferred
shares, nor exclude either class of shares, in determining the outstanding capital stock (the "capital") of a
corporation. Consequently, petitioner's suggestion to reckon PLDT's foreign equity only on the basis of
PLDT's outstanding common shares is without legal basis. The language of the Constitution should be
understood in the sense it has in common use.
xxx xxx xxx
17. But even assuming that resort to the proceedings of the Constitutional Commission is
necessary, there is nothing in the Record of the Constitutional Commission (Vol. III) — which petitioner
misleadingly cited in the Petition . . . — which supports petitioner's view that only common shares should
form the basis for computing a public utility's foreign equity. AaITCS
xxx xxx xxx
18. In addition, the SEC — the government agency primarily responsible for implementing the
Corporation Code, and which also has the responsibility of ensuring compliance with the Constitution's
foreign equity restrictions as regards nationalized activities . . . — has categorically ruled that both
common and preferred shares are properly considered in determining outstanding capital stock and the
nationality composition thereof. 40
We agree with petitioner and petitioners-in-intervention. The term "capital" in Section 11, Article XII of the
Constitution refers only to shares of stock entitled to vote in the election of directors, and thus in the present case only
to common shares, 41 and not to the total outstanding capital stock comprising both common and non-voting preferred
shares.
The Corporation Code of the Philippines 42 classifies shares as common or preferred, thus:
Sec. 6. Classification of shares. — The shares of stock of stock corporations may be divided into
classes or series of shares, or both, any of which classes or series of shares may have such rights,
privileges or restrictions as may be stated in the articles of incorporation: Provided, That no share may
be deprived of voting rights except those classified and issued as "preferred" or "redeemable"
shares, unless otherwise provided in this Code: Provided, further, That there shall always be a class
or series of shares which have complete voting rights. Any or all of the shares or series of shares may
have a par value or have no par value as may be provided for in the articles of incorporation: Provided,
however, That banks, trust companies, insurance companies, public utilities, and building and loan
associations shall not be permitted to issue no-par value shares of stock.
Preferred shares of stock issued by any corporation may be given preference in the distribution
of the assets of the corporation in case of liquidation and in the distribution of dividends, or such other
preferences as may be stated in the articles of incorporation which are not violative of the provisions of
this Code: Provided, That preferred shares of stock may be issued only with a stated par value. The
Board of Directors, where authorized in the articles of incorporation, may fix the terms and conditions of
preferred shares of stock or any series thereof: Provided, That such terms and conditions shall be
effective upon the filing of a certificate thereof with the Securities and Exchange Commission.
Shares of capital stock issued without par value shall be deemed fully paid and non-assessable
and the holder of such shares shall not be liable to the corporation or to its creditors in respect thereto:
Provided; That shares without par value may not be issued for a consideration less than the value of five
(P5.00) pesos per share: Provided, further, That the entire consideration received by the corporation for
its no-par value shares shall be treated as capital and shall not be available for distribution as dividends.
TcCEDS
A corporation may, furthermore, classify its shares for the purpose of insuring compliance with
constitutional or legal requirements.
Except as otherwise provided in the articles of incorporation and stated in the certificate of stock,
each share shall be equal in all respects to every other share.
Where the articles of incorporation provide for non-voting shares in the cases allowed by this
Code, the holders of such shares shall nevertheless be entitled to vote on the following matters:
1. Amendment of the articles of incorporation;
2. Adoption and amendment of by-laws;
3. Sale, lease, exchange, mortgage, pledge or other disposition of all or substantially all
of the corporate property;
4. Incurring, creating or increasing bonded indebtedness;
5. Increase or decrease of capital stock;
6. Merger or consolidation of the corporation with another corporation or other
corporations;
7. Investment of corporate funds in another corporation or business in accordance with
this Code; and caIETS
8. Dissolution of the corporation.
Except as provided in the immediately preceding paragraph, the vote necessary to approve a
particular corporate act as provided in this Code shall be deemed to refer only to stocks with voting rights.
Indisputably, one of the rights of a stockholder is the right to participate in the control or management of the
corporation. 43 This is exercised through his vote in the election of directors because it is the board of directors that
controls or manages the corporation. 44 In the absence of provisions in the articles of incorporation denying voting
rights to preferred shares, preferred shares have the same voting rights as common shares. However, preferred
shareholders are often excluded from any control, that is, deprived of the right to vote in the election of directors and on
other matters, on the theory that the preferred shareholders are merely investors in the corporation for income in the
same manner as bondholders. 45 In fact, under the Corporation Code only preferred or redeemable shares can be
deprived of the right to vote. 46 Common shares cannot be deprived of the right to vote in any corporate meeting, and
any provision in the articles of incorporation restricting the right of common shareholders to vote is invalid. 47
Considering that common shares have voting rights which translate to control, as opposed to preferred shares
which usually have no voting rights, the term "capital" in Section 11, Article XII of the Constitution refers only to common
shares. However, if the preferred shares also have the right to vote in the election of directors, then the term "capital"
shall include such preferred shares because the right to participate in the control or management of the corporation is
exercised through the right to vote in the election of directors. In short, the term "capital" in Section 11, Article XII of
the Constitution refers only to shares of stock that can vote in the election of directors.
This interpretation is consistent with the intent of the framers of the Constitution to place in the hands of Filipino
citizens the control and management of public utilities. As revealed in the deliberations of the Constitutional
Commission, "capital" refers to the voting stock or controlling interest of a corporation, to wit:
MR. NOLLEDO.
In Sections 3, 9 and 15, the Committee stated local or Filipino equity and foreign equity; namely,
60-40 in Section 3, 60-40 in Section 9 and 2/3-1/3 in Section 15.
MR. VILLEGAS.
That is right.
MR. NOLLEDO.
In teaching law, we are always faced with this question: "Where do we base the equity
requirement, is it on the authorized capital stock, on the subscribed capital stock, or on the paid-
up capital stock of a corporation"? Will the Committee please enlighten me on this? ITESAc
MR. VILLEGAS.
We have just had a long discussion with the members of the team from the UP Law Center who
provided us a draft. The phrase that is contained here which we adopted from the UP draft
is "60 percent of voting stock."
MR. NOLLEDO.
That must be based on the subscribed capital stock, because unless declared delinquent, unpaid
capital stock shall be entitled to vote.
MR. VILLEGAS.
That is right.
MR. NOLLEDO.
Thank you.
With respect to an investment by one corporation in another corporation, say, a corporation with
60-40 percent equity invests in another corporation which is permitted by the Corporation Code,
does the Committee adopt the grandfather rule?
MR. VILLEGAS.
Yes, that is the understanding of the Committee.
MR. NOLLEDO.
Therefore, we need additional Filipino capital?
MR. VILLEGAS.
Yes. 48
xxx xxx xxx
MR. AZCUNA.
May I be clarified as to that portion that was accepted by the Committee.
MR. VILLEGAS.
The portion accepted by the Committee is the deletion of the phrase "voting stock or controlling
interest." ESTAIH
MR. AZCUNA.
Hence, without the Davide amendment, the committee report would read: "corporations or
associations at least sixty percent of whose CAPITAL is owned by such citizens."
MR. VILLEGAS.
Yes.
MR. AZCUNA.
So if the Davide amendment is lost, we are stuck with 60 percent of the capital to be owned by
citizens.
MR. VILLEGAS.
That is right.
MR. AZCUNA.
But the control can be with the foreigners even if they are the minority. Let us say 40
percent of the capital is owned by them, but it is the voting capital, whereas, the Filipinos
own the nonvoting shares. So we can have a situation where the corporation is controlled
by foreigners despite being the minority because they have the voting capital. That is the
anomaly that would result here.
MR. BENGZON.
No, the reason we eliminated the word "stock" as stated in the 1973 and 1935
Constitutions is that according to Commissioner Rodrigo, there are associations that do
not have stocks. That is why we say "CAPITAL."
MR. AZCUNA.
We should not eliminate the phrase "controlling interest."
MR. BENGZON.
In the case of stock corporations, it is assumed. 49 (Emphasis supplied)
Thus, 60 percent of the "capital" assumes, or should result in, "controlling interest" in the corporation.
Reinforcing this interpretation of the term "capital," as referring to controlling interest or shares entitled to vote, is the
definition of a "Philippine national" in the Foreign Investments Act of 1991, 50 to wit:
SEC. 3. Definitions. — As used in this Act:
a. The term "Philippine national" shall mean a citizen of the Philippines; or a domestic
partnership or association wholly owned by citizens of the Philippines; or a corporation organized
under the laws of the Philippines of which at least sixty percent (60%) of the capital stock
outstanding and entitled to vote is owned and held by citizens of the Philippines; or a corporation
organized abroad and registered as doing business in the Philippines under the Corporation Code of
which one hundred percent (100%) of the capital stock outstanding and entitled to vote is wholly owned
by Filipinos or a trustee of funds for pension or other employee retirement or separation benefits, where
the trustee is a Philippine national and at least sixty percent (60%) of the fund will accrue to the benefit of
Philippine nationals: Provided, That where a corporation and its non-Filipino stockholders own stocks in a
Securities and Exchange Commission (SEC) registered enterprise, at least sixty percent (60%) of the
capital stock outstanding and entitled to vote of each of both corporations must be owned and held by
citizens of the Philippines and at least sixty percent (60%) of the members of the Board of Directors of
each of both corporations must be citizens of the Philippines, in order that the corporation, shall be
considered a "Philippine national." (Emphasis supplied) TDcHCa
In explaining the definition of a "Philippine national," the Implementing Rules and Regulations of the Foreign
Investments Act of 1991 provide:
b. "Philippine national" shall mean a citizen of the Philippines or a domestic partnership or
association wholly owned by the citizens of the Philippines; or a corporation organized under the laws
of the Philippines of which at least sixty percent [60%] of the capital stock outstanding and
entitled to vote is owned and held by citizens of the Philippines; or a trustee of funds for pension or
other employee retirement or separation benefits, where the trustee is a Philippine national and at least
sixty percent [60%] of the fund will accrue to the benefit of the Philippine nationals; Provided, that where
a corporation its non-Filipino stockholders own stocks in a Securities and Exchange Commission [SEC]
registered enterprise, at least sixty percent [60%] of the capital stock outstanding and entitled to vote of
both corporations must be owned and held by citizens of the Philippines and at least sixty percent [60%]
of the members of the Board of Directors of each of both corporation must be citizens of the Philippines,
in order that the corporation shall be considered a Philippine national. The control test shall be applied for
this purpose.
Compliance with the required Filipino ownership of a corporation shall be determined on
the basis of outstanding capital stock whether fully paid or not, but only such stocks which are
generally entitled to vote are considered.
For stocks to be deemed owned and held by Philippine citizens or Philippine nationals,
mere legal title is not enough to meet the required Filipino equity. Full beneficial ownership of the
stocks, coupled with appropriate voting rights is essential. Thus, stocks, the voting rights of
which have been assigned or transferred to aliens cannot be considered held by Philippine
citizens or Philippine nationals. ITaESD
Individuals or juridical entities not meeting the aforementioned qualifications are
considered as non-Philippine nationals. (Emphasis supplied)
Mere legal title is insufficient to meet the 60 percent Filipino-owned "capital" required in the Constitution. Full
beneficial ownership of 60 percent of the outstanding capital stock, coupled with 60 percent of the voting rights, is
required. The legal and beneficial ownership of 60 percent of the outstanding capital stock must rest in the hands of
Filipino nationals in accordance with the constitutional mandate. Otherwise, the corporation is "considered as non-
Philippine national[s]."
Under Section 10, Article XII of the Constitution, Congress may "reserve to citizens of the Philippines or to
corporations or associations at least sixty per centum of whose capital is owned by such citizens, or such higher
percentage as Congress may prescribe, certain areas of investments." Thus, in numerous laws Congress has reserved
certain areas of investments to Filipino citizens or to corporations at least sixty percent of the "capital" of which is
owned by Filipino citizens. Some of these laws are: (1) Regulation of Award of Government Contracts or R.A. No. 5183;
(2) Philippine Inventors Incentives Act or R.A. No. 3850; (3) Magna Carta for Micro, Small and Medium Enterprises or
R.A. No. 6977; (4) Philippine Overseas Shipping Development Act or R.A. No. 7471; (5) Domestic Shipping
Development Act of 2004 or R.A. No. 9295; (6) Philippine Technology Transfer Act of 2009 or R.A. No. 10055; and (7)
Ship Mortgage Decree or P.D. No. 1521. Hence, the term "capital" in Section 11, Article XII of the Constitution is also
used in the same context in numerous laws reserving certain areas of investments to Filipino citizens.
To construe broadly the term "capital" as the total outstanding capital stock, including both common and non-
voting preferred shares, grossly contravenes the intent and letter of the Constitution that the "State shall develop a self-
reliant and independent national economy effectively controlled by Filipinos." A broad definition unjustifiably
disregards who owns the all-important voting stock, which necessarily equates to control of the public utility.
We shall illustrate the glaring anomaly in giving a broad definition to the term "capital." Let us assume that a
corporation has 100 common shares owned by foreigners and 1,000,000 non-voting preferred shares owned by
Filipinos, with both classes of share having a par value of one peso (P1.00) per share. Under the broad definition of the
term "capital," such corporation would be considered compliant with the 40 percent constitutional limit on foreign equity
of public utilities since the overwhelming majority, or more than 99.999 percent, of the total outstanding capital stock is
Filipino owned. This is obviously absurd.
In the example given, only the foreigners holding the common shares have voting rights in the election of
directors, even if they hold only 100 shares. The foreigners, with a minuscule equity of less than 0.001 percent, exercise
control over the public utility. On the other hand, the Filipinos, holding more than 99.999 percent of the equity, cannot
vote in the election of directors and hence, have no control over the public utility. This starkly circumvents the intent of
the framers of the Constitution, as well as the clear language of the Constitution, to place the control of public utilities in
the hands of Filipinos. It also renders illusory the State policy of an independent national economy effectively
controlled by Filipinos. HCEaDI
The example given is not theoretical but can be found in the real world, and in fact exists in the present case.
Holders of PLDT preferred shares are explicitly denied of the right to vote in the election of directors. PLDT's
Articles of Incorporation expressly state that "the holders of Serial Preferred Stock shall not be entitled to vote at
any meeting of the stockholders for the election of directors or for any other purpose or otherwise participate in
any action taken by the corporation or its stockholders, or to receive notice of any meeting of stockholders." 51
On the other hand, holders of common shares are granted the exclusive right to vote in the election of directors.
PLDT's Articles of Incorporation 52 state that "each holder of Common Capital Stock shall have one vote in respect of
each share of such stock held by him on all matters voted upon by the stockholders, and the holders of Common
Capital Stock shall have the exclusive right to vote for the election of directors and for all other purposes." 53
In short, only holders of common shares can vote in the election of directors, meaning only common
shareholders exercise control over PLDT. Conversely, holders of preferred shares, who have no voting rights in the
election of directors, do not have any control over PLDT. In fact, under PLDT's Articles of Incorporation, holders of
common shares have voting rights for all purposes, while holders of preferred shares have no voting right for any
purpose whatsoever.
It must be stressed, and respondents do not dispute, that foreigners hold a majority of the common shares of
PLDT. In fact, based on PLDT's 2010 General Information Sheet (GIS), 54 which is a document required to be
submitted annually to the Securities and Exchange Commission, 55 foreigners hold 120,046,690 common shares of
PLDT whereas Filipinos hold only 66,750,622 common shares. 56 In other words, foreigners hold 64.27% of the total
number of PLDT's common shares, while Filipinos hold only 35.73%. Since holding a majority of the common shares
equates to control, it is clear that foreigners exercise control over PLDT. Such amount of control unmistakably exceeds
the allowable 40 percent limit on foreign ownership of public utilities expressly mandated in Section 11, Article XII of the
Constitution.
Moreover, the Dividend Declarations of PLDT for 2009, 57 as submitted to the SEC, shows that per share the
SIP 58 preferred shares earn a pittance in dividends compared to the common shares. PLDT declared dividends for the
common shares at P70.00 per share, while the declared dividends for the preferred shares amounted to a measly P1.00
per share. 59 So the preferred shares not only cannot vote in the election of directors, they also have very little and
obviously negligible dividend earning capacity compared to common shares.
As shown in PLDT's 2010 GIS, 60 as submitted to the SEC, the par value of PLDT common shares is P5.00
per share, whereas the par value of preferred shares is P10.00 per share. In other words, preferred shares have twice
the par value of common shares but cannot elect directors and have only 1/70 of the dividends of common shares.
Moreover, 99.44% of the preferred shares are owned by Filipinos while foreigners own only a minuscule 0.56% of the
preferred shares. 61 Worse, preferred shares constitute 77.85% of the authorized capital stock of PLDT while common
shares constitute only 22.15%. 62 This undeniably shows that beneficial interest in PLDT is not with the non-voting
preferred shares but with the common shares, blatantly violating the constitutional requirement of 60 percent Filipino
control and Filipino beneficial ownership in a public utility.
The legal and beneficial ownership of 60 percent of the outstanding capital stock must rest in the hands of
Filipinos in accordance with the constitutional mandate. Full beneficial ownership of 60 percent of the outstanding
capital stock, coupled with 60 percent of the voting rights, is constitutionally required for the State's grant of authority to
operate a public utility. The undisputed fact that the PLDT preferred shares, 99.44% owned by Filipinos, are non-voting
and earn only 1/70 of the dividends that PLDT common shares earn, grossly violates the constitutional requirement of
60 percent Filipino control and Filipino beneficial ownership of a public utility.
In short, Filipinos hold less than 60 percent of the voting stock, and earn less than 60 percent of the
dividends, of PLDT. This directly contravenes the express command in Section 11, Article XII of the Constitution that
"[n]o franchise, certificate, or any other form of authorization for the operation of a public utility shall be granted except
to . . . corporations . . . organized under the laws of the Philippines, at least sixty per centum of whose capital is
owned by such citizens . . . ." IaCHTS
To repeat, (1) foreigners own 64.27% of the common shares of PLDT, which class of shares exercises the sole
right to vote in the election of directors, and thus exercise control over PLDT; (2) Filipinos own only 35.73% of PLDT's
common shares, constituting a minority of the voting stock, and thus do not exercise control over PLDT; (3) preferred
shares, 99.44% owned by Filipinos, have no voting rights; (4) preferred shares earn only 1/70 of the dividends that
common shares earn; 63 (5) preferred shares have twice the par value of common shares; and (6) preferred shares
constitute 77.85% of the authorized capital stock of PLDT and common shares only 22.15%. This kind of ownership and
control of a public utility is a mockery of the Constitution.
Incidentally, the fact that PLDT common shares with a par value of P5.00 have a current stock market value of
P2,328.00 per share, 64 while PLDT preferred shares with a par value of P10.00 per share have a current stock market
value ranging from only P10.92 to P11.06 per share, 65 is a glaring confirmation by the market that control and
beneficial ownership of PLDT rest with the common shares, not with the preferred shares.
Indisputably, construing the term "capital" in Section 11, Article XII of the Constitution to include both voting and
non-voting shares will result in the abject surrender of our telecommunications industry to foreigners, amounting to a
clear abdication of the State's constitutional duty to limit control of public utilities to Filipino citizens. Such an
interpretation certainly runs counter to the constitutional provision reserving certain areas of investment to Filipino
citizens, such as the exploitation of natural resources as well as the ownership of land, educational institutions and
advertising businesses. The Court should never open to foreign control what the Constitution has expressly reserved to
Filipinos for that would be a betrayal of the Constitution and of the national interest. The Court must perform its solemn
duty to defend and uphold the intent and letter of the Constitution to ensure, in the words of the Constitution, "a self-
reliant and independent national economy effectively controlled by Filipinos."
Section 11, Article XII of the Constitution, like other provisions of the Constitution expressly reserving to
Filipinos specific areas of investment, such as the development of natural resources and ownership of land, educational
institutions and advertising business, is self-executing. There is no need for legislation to implement these self-
executing provisions of the Constitution. The rationale why these constitutional provisions are self-executing was
explained in Manila Prince Hotel v. GSIS, 66 thus:
. . . Hence, unless it is expressly provided that a legislative act is necessary to enforce a
constitutional mandate, the presumption now is that all provisions of the constitution are self-executing. If
the constitutional provisions are treated as requiring legislation instead of self-executing, the legislature
would have the power to ignore and practically nullify the mandate of the fundamental law. This can be
cataclysmic. That is why the prevailing view is, as it has always been, that —
. . . in case of doubt, the Constitution should be considered self-executing rather than
non-self-executing. . . . Unless the contrary is clearly intended, the provisions of the
Constitution should be considered self-executing, as a contrary rule would give the
legislature discretion to determine when, or whether, they shall be effective. These
provisions would be subordinated to the will of the lawmaking body, which could make them
entirely meaningless by simply refusing to pass the needed implementing statute. (Emphasis
supplied) TAaEIc
In Manila Prince Hotel, even the Dissenting Opinion of then Associate Justice Reynato S. Puno, later Chief
Justice, agreed that constitutional provisions are presumed to be self-executing. Justice Puno stated:
Courts as a rule consider the provisions of the Constitution as self-executing, rather than as
requiring future legislation for their enforcement. The reason is not difficult to discern. For if they are not
treated as self-executing, the mandate of the fundamental law ratified by the sovereign people can
be easily ignored and nullified by Congress. Suffused with wisdom of the ages is the unyielding
rule that legislative actions may give breath to constitutional rights but congressional inaction
should not suffocate them.
Thus, we have treated as self-executing the provisions in the Bill of Rights on arrests, searches
and seizures, the rights of a person under custodial investigation, the rights of an accused, and the
privilege against self-incrimination. It is recognized that legislation is unnecessary to enable courts to
effectuate constitutional provisions guaranteeing the fundamental rights of life, liberty and the protection
of property. The same treatment is accorded to constitutional provisions forbidding the taking or
damaging of property for public use without just compensation. (Emphasis supplied)
Thus, in numerous cases, 67 this Court, even in the absence of implementing legislation, applied directly the
provisions of the 1935, 1973 and 1987 Constitutions limiting land ownership to Filipinos. In Soriano v. Ong Hoo, 68 this
Court ruled:
. . . As the Constitution is silent as to the effects or consequences of a sale by a citizen of his
land to an alien, and as both the citizen and the alien have violated the law, none of them should have a
recourse against the other, and it should only be the State that should be allowed to intervene and
determine what is to be done with the property subject of the violation. We have said that what the State
should do or could do in such matters is a matter of public policy, entirely beyond the scope of judicial
authority. (Dinglasan, et al. vs. Lee Bun Ting, et al., 6 G.R. No. L-5996, June 27, 1956.) While the
legislature has not definitely decided what policy should be followed in cases of violations
against the constitutional prohibition, courts of justice cannot go beyond by declaring the
disposition to be null and void as violative of the Constitution. . . . (Emphasis supplied) IScaAE
To treat Section 11, Article XII of the Constitution as not self-executing would mean that since the 1935
Constitution, or over the last 75 years, not one of the constitutional provisions expressly reserving specific areas of
investments to corporations, at least 60 percent of the "capital" of which is owned by Filipinos, was enforceable. In
short, the framers of the 1935, 1973 and 1987 Constitutions miserably failed to effectively reserve to Filipinos specific
areas of investment, like the operation by corporations of public utilities, the exploitation by corporations of mineral
resources, the ownership by corporations of real estate, and the ownership of educational institutions. All the
legislatures that convened since 1935 also miserably failed to enact legislations to implement these vital constitutional
provisions that determine who will effectively control the national economy, Filipinos or foreigners. This Court cannot
allow such an absurd interpretation of the Constitution.
This Court has held that the SEC "has both regulatory and adjudicative functions." 69 Under its regulatory
functions, the SEC can be compelled by mandamus to perform its statutory duty when it unlawfully neglects to perform
the same. Under its adjudicative or quasi-judicial functions, the SEC can be also be compelled by mandamus to hear
and decide a possible violation of any law it administers or enforces when it is mandated by law to investigate such
violation.
Under Section 17 (4) 70 of the Corporation Code, the SEC has the regulatory function to reject or disapprove
the Articles of Incorporation of any corporation where "the required percentage of ownership of the capital stock to
be owned by citizens of the Philippines has not been complied with as required by existing laws or the
Constitution." Thus, the SEC is the government agency tasked with the statutory duty to enforce the nationality
requirement prescribed in Section 11, Article XII of the Constitution on the ownership of public utilities. This Court, in a
petition for declaratory relief that is treated as a petition for mandamus as in the present case, can direct the SEC to
perform its statutory duty under the law, a duty that the SEC has apparently unlawfully neglected to do based on the
2010 GIS that respondent PLDT submitted to the SEC.
Under Section 5 (m) of the Securities Regulation Code, 71 the SEC is vested with the "power and function" to
"suspend or revoke, after proper notice and hearing, the franchise or certificate of registration of corporations,
partnerships or associations, upon any of the grounds provided by law." The SEC is mandated under Section 5
(d) of the same Code with the "power and function" to "investigate . . . the activities of persons to ensure
compliance" with the laws and regulations that SEC administers or enforces. The GIS that all corporations are required
to submit to SEC annually should put the SEC on guard against violations of the nationality requirement prescribed in
the Constitution and existing laws. This Court can compel the SEC, in a petition for declaratory relief that is treated as a
petition for mandamus as in the present case, to hear and decide a possible violation of Section 11, Article XII of the
Constitution in view of the ownership structure of PLDT's voting shares, as admitted by respondents and as stated in
PLDT's 2010 GIS that PLDT submitted to SEC.
WHEREFORE, we PARTLY GRANT the petition and rule that the term "capital" in Section 11, Article XII of the
1987 Constitution refers only to shares of stock entitled to vote in the election of directors, and thus in the present case
only to common shares, and not to the total outstanding capital stock (common and non-voting preferred shares).
Respondent Chairperson of the Securities and Exchange Commission is DIRECTED to apply this definition of the term
"capital" in determining the extent of allowable foreign ownership in respondent Philippine Long Distance Telephone
Company, and if there is a violation of Section 11, Article XII of the Constitution, to impose the appropriate sanctions
under the law. aDSSO ORDERED.
||| (Gamboa v. Teves, G.R. No. 176579, [June 28, 2011], 668 PHIL 1-118)
[G.R. No. 176579. October 9, 2012.]

HEIRS OF WILSON P. GAMBOA, * petitioners, vs. FINANCE SECRETARY MARGARITO B. TEVES,


FINANCE UNDERSECRETARY JOHN P. SEVILLA, AND COMMISSIONER RICARDO ABCEDE OF
THE PRESIDENTIAL COMMISSION ON GOOD GOVERNMENT (PCGG) IN THEIR CAPACITIES AS
CHAIR AND MEMBERS, RESPECTIVELY, OF THE PRIVATIZATION COUNCIL, CHAIRMAN
ANTHONI SALIM OF FIRST PACIFIC CO., LTD. IN HIS CAPACITY AS DIRECTOR OF METRO
PACIFIC ASSET HOLDINGS, INC., CHAIRMAN MANUEL V. PANGILINAN OF PHILIPPINE LONG
DISTANCE TELEPHONE COMPANY (PLDT) IN HIS CAPACITY AS MANAGING DIRECTOR OF
FIRST PACIFIC CO., LTD., PRESIDENT NAPOLEON L. NAZARENO OF PHILIPPINE LONG
DISTANCE TELEPHONE COMPANY, CHAIR FE BARIN OF THE SECURITIES AND EXCHANGE
COMMISSION, and PRESIDENT FRANCIS LIM OF THE PHILIPPINE STOCK
EXCHANGE, respondents.

PABLITO V. SANIDAD and ARNO V. SANIDAD, petitioners-in-Intervention.

RESOLUTION

CARPIO, J p:

This resolves the motions for reconsideration of the 28 June 2011 Decision filed by (1) the Philippine Stock
Exchange's (PSE) President, 1 (2) Manuel V. Pangilinan (Pangilinan), 2 (3) Napoleon L. Nazareno (Nazareno), 3 and
(4) the Securities and Exchange Commission (SEC) 4 (collectively, movants).
The Office of the Solicitor General (OSG) initially filed a motion for reconsideration on behalf of the
SEC, 5 assailing the 28 June 2011 Decision. However, it subsequently filed a Consolidated Comment on behalf of the
State, 6 declaring expressly that it agrees with the Court's definition of the term "capital" in Section 11, Article XII of the
Constitution. During the Oral Arguments on 26 June 2012, the OSG reiterated its position consistent with the Court's 28
June 2011 Decision.
We deny the motions for reconsideration.
I.
Far-reaching implications of the legal issue justify
treatment of petition for declaratory relief as one for mandamus.
As we emphatically stated in the 28 June 2011 Decision, the interpretation of the term "capital" in Section 11,
Article XII of the Constitution has far-reaching implications to the national economy. In fact, a resolution of this issue will
determine whether Filipinos are masters, or second-class citizens, in their own country. What is at stake here is whether
Filipinos or foreigners will have effective controlof the Philippine national economy. Indeed, if ever there is a legal
issue that has far-reaching implications to the entire nation, and to future generations of Filipinos, it is the threshold
legal issue presented in this case. TcDAHS
Contrary to Pangilinan's narrow view, the serious economic consequences resulting in the interpretation of the
term "capital" in Section 11, Article XII of the Constitution undoubtedly demand an immediate adjudication of this
issue. Simply put, the far-reaching implications of this issue justify the treatment of the petition as one
for mandamus. 7
In Luzon Stevedoring Corp. v. Anti-Dummy Board, 8 the Court deemed it wise and expedient to resolve the
case although the petition for declaratory relief could be outrightly dismissed for being procedurally defective. There,
appellant admittedly had already committed a breach of the Public Service Act in relation to the Anti-Dummy Law since
it had been employing non-American aliens long before the decision in a prior similar case. However, the main issue
in Luzon Stevedoring was of transcendental importance, involving the exercise or enjoyment of rights, franchises,
privileges, properties and businesses which only Filipinos and qualified corporations could exercise or enjoy under the
Constitution and the statutes. Moreover, the same issue could be raised by appellant in an appropriate action. Thus,
in Luzon Stevedoring the Court deemed it necessary to finally dispose of the case for the guidance of all concerned,
despite the apparent procedural flaw in the petition.
The circumstances surrounding the present case, such as the supposed procedural defect of the petition and
the pivotal legal issue involved, resemble those in Luzon Stevedoring. Consequently, in the interest of substantial
justice and faithful adherence to the Constitution, we opted to resolve this case for the guidance of the public and all
concerned parties.
II.
No change of any long-standing rule;
thus, no redefinition of the term "capital."
Movants contend that the term "capital" in Section 11, Article XII of the Constitution has long been settled and
defined to refer to the total outstanding shares of stock, whether voting or non-voting. In fact, movants claim that the
SEC, which is the administrative agency tasked to enforce the 60-40 ownership requirement in favor of Filipino citizens
in the Constitution and various statutes, has consistently adopted this particular definition in its numerous opinions.
Movants point out that with the 28 June 2011 Decision, the Court in effect introduced a "new" definition or "midstream
redefinition" 9 of the term "capital" in Section 11, Article XII of the Constitution. HCaIDS
This is egregious error.
For more than 75 years since the 1935 Constitution, the Court has not interpreted or defined the term "capital"
found in various economic provisions of the 1935, 1973 and 1987 Constitutions. There has never been a judicial
precedent interpreting the term "capital" in the 1935, 1973 and 1987 Constitutions, until now. Hence, it is patently wrong
and utterly baseless to claim that the Court in defining the term "capital" in its 28 June 2011 Decision modified,
reversed, or set aside the purported long-standing definition of the term "capital," which supposedly refers to the total
outstanding shares of stock, whether voting or non-voting. To repeat, until the present case there has never been a
Court ruling categorically defining the term "capital" found in the various economic provisions of the 1935, 1973 and
1987 Philippine Constitutions.
The opinions of the SEC, as well as of the Department of Justice (DOJ), on the definition of the term "capital"
as referring to both voting and non-voting shares (combined total of common and preferred shares) are, in the first
place, conflicting and inconsistent. There is no basis whatsoever to the claim that the SEC and the DOJ have
consistently and uniformly adopted a definition of the term "capital" contrary to the definition that this Court adopted in
its 28 June 2011 Decision.
In DOJ Opinion No. 130, s. 1985, 10 dated 7 October 1985, the scope of the term "capital" in Section 9, Article
XIV of the 1973 Constitution was raised, that is, whether the term "capital" includes "both preferred and common
stocks." The issue was raised in relation to a stock-swap transaction between a Filipino and a Japanese corporation,
both stockholders of a domestic corporation that owned lands in the Philippines. Then Minister of Justice Estelito P.
Mendoza ruled that the resulting ownership structure of the corporation would be unconstitutional because 60% of the
voting stock would be owned by Japanese while Filipinos would own only 40% of the voting stock, although when the
non-voting stock is added, Filipinos would own 60% of the combined voting and non-voting stock. This ownership
structure is remarkably similar to the current ownership structure of PLDT. Minister Mendoza ruled:
xxx xxx xxx
Thus, the Filipino group still owns sixty (60%) of the entire subscribed capital stock (common and
preferred) while the Japanese investors control sixty percent (60%) of the common (voting) shares.
It is your position that . . . since Section 9, Article XIV of the Constitution uses the word
"capital," which is construed "to include both preferred and common shares" and "that where the
law does not distinguish, the courts shall not distinguish."
xxx xxx xxx
In light of the foregoing jurisprudence, it is my opinion that the stock-swap transaction in
question may not be constitutionally upheld. While it may be ordinary corporate practice to classify
corporate shares into common voting shares and preferred non-voting shares, any arrangement which
attempts to defeat the constitutional purpose should be eschewed. Thus, the resultant equity
arrangement which would place ownership of 60% 11 of the common (voting) shares in the
Japanese group, while retaining 60% of the total percentage of common and preferred shares in
Filipino hands would amount to circumvention of the principle of control by Philippine
stockholders that is implicit in the 60% Philippine nationality requirement in the Constitution.
(Emphasis supplied)
In short, Minister Mendoza categorically rejected the theory that the term "capital" in Section 9, Article XIV of
the 1973 Constitution includes "both preferred and common stocks" treated as the same class of shares regardless of
differences in voting rights and privileges. Minister Mendoza stressed that the 60-40 ownership requirement in favor of
Filipino citizens in the Constitution is not complied with unless the corporation "satisfies the criterion of beneficial
ownership" and that in applying the same "the primordial consideration is situs of control."
On the other hand, in Opinion No. 23-10 dated 18 August 2010, addressed to Castillo Laman Tan Pantaleon &
San Jose, then SEC General Counsel Vernette G. Umali-Paco applied the Voting Control Test, that is, using only the
voting stock to determine whether a corporation is a Philippine national. The Opinion states:
Applying the foregoing, particularly the Control Test, MLRC is deemed as a Philippine national
because: (1) sixty percent (60%) of its outstanding capital stock entitled to vote is owned by a
Philippine national, the Trustee; and (2) at least sixty percent (60%) of the ERF will accrue to the benefit
of Philippine nationals. Still pursuant to the Control Test, MLRC's investment in 60% of BFDC's
outstanding capital stock entitled to vote shall be deemed as of Philippine nationality, thereby
qualifying BFDC to own private land. EcICSA
Further, under, and for purposes of, the FIA, MLRC and BFDC are both Philippine nationals,
considering that: (1) sixty percent (60%) of their respective outstanding capital stock entitled to vote is
owned by a Philippine national (i.e., by the Trustee, in the case of MLRC; and by MLRC, in the case of
BFDC); and (2) at least 60% of their respective board of directors are Filipino citizens. (Boldfacing and
italicization supplied)
Clearly, these DOJ and SEC opinions are compatible with the Court's interpretation of the 60-40 ownership
requirement in favor of Filipino citizens mandated by the Constitution for certain economic activities. At the same time,
these opinions highlight the conflicting, contradictory, and inconsistent positions taken by the DOJ and the SEC on the
definition of the term "capital" found in the economic provisions of the Constitution.
The opinions issued by SEC legal officers do not have the force and effect of SEC rules and regulations
because only the SEC en banc can adopt rules and regulations. As expressly provided in Section 4.6 of the Securities
Regulation Code, 12 the SEC cannot delegate to any of its individual Commissioner or staff the power to adopt any rule
or regulation. Further, under Section 5.1 of the same Code, it is the SEC as a collegial body, and not any of its
legal officers, that is empowered to issue opinions and approve rules and regulations. Thus:
4.6. The Commission may, for purposes of efficiency, delegate any of its functions to any department
or office of the Commission, an individual Commissioner or staff member of the Commission except its
review or appellate authority and its power to adopt, alter and supplement any rule or regulation.
The Commission may review upon its own initiative or upon the petition of any interested party any
action of any department or office, individual Commissioner, or staff member of the Commission.
SEC. 5. Powers and Functions of the Commission. — 5.1. The Commission shall act with transparency
and shall have the powers and functions provided by this Code, Presidential Decree No. 902-A, the
Corporation Code, the Investment Houses Law, the Financing Company Act and other existing laws.
Pursuant thereto the Commission shall have, among others, the following powers and
functions:DcTAIH
xxx xxx xxx
(g) Prepare, approve, amend or repeal rules, regulations and orders, and issue opinions and
provide guidance on and supervise compliance with such rules, regulations and orders;
xxx xxx xxx (Emphasis supplied)
Thus, the act of the individual Commissioners or legal officers of the SEC in issuing opinions that have the
effect of SEC rules or regulations is ultra vires. Under Sections 4.6 and 5.1 (g) of the Code, only the SEC en banc can
"issue opinions" that have the force and effect of rules or regulations. Section 4.6 of the Code bars the SEC en
banc from delegating to any individual Commissioner or staff the power to adopt rules or regulations. In short, any
opinion of individual Commissioners or SEC legal officers does not constitute a rule or regulation of the SEC.
The SEC admits during the Oral Arguments that only the SEC en banc, and not any of its individual
commissioners or legal staff, is empowered to issue opinions which have the same binding effect as SEC rules and
regulations, thus:
JUSTICE CARPIO:
So, under the law, it is the Commission En Banc that can issue an SEC Opinion, correct?
COMMISSIONER GAITE: 13
That's correct, Your Honor.
JUSTICE CARPIO:
Can the Commission En Banc delegate this function to an SEC officer? SECIcT
COMMISSIONER GAITE:
Yes, Your Honor, we have delegated it to the General Counsel.
JUSTICE CARPIO:
It can be delegated. What cannot be delegated by the Commission En Banc to a commissioner
or an individual employee of the Commission?
COMMISSIONER GAITE:
Novel opinions that [have] to be decided by the En Banc . . .
JUSTICE CARPIO:
What cannot be delegated, among others, is the power to adopt or amend rules and regulations,
correct?
COMMISSIONER GAITE:
That's correct, Your Honor.
JUSTICE CARPIO:
So, you combine the two (2), the SEC officer, if delegated that power, can issue an opinion
but that opinion does not constitute a rule or regulation, correct?
COMMISSIONER GAITE:
Correct, Your Honor.
JUSTICE CARPIO:
So, all of these opinions that you mentioned they are not rules and regulations,
correct? cDAISC
COMMISSIONER GAITE:
They are not rules and regulations.
JUSTICE CARPIO:
If they are not rules and regulations, they apply only to that particular situation and will not
constitute a precedent, correct?
COMMISSIONER GAITE:
Yes, Your Honor. 14 (Emphasis supplied)
Significantly, the SEC en banc, which is the collegial body statutorily empowered to issue rules and opinions on
behalf of the SEC, has adopted even the Grandfather Rule in determining compliance with the 60-40 ownership
requirement in favor of Filipino citizens mandated by the Constitution for certain economic activities. This prevailing
SEC ruling, which the SEC correctly adopted to thwart any circumvention of the required Filipino "ownership and
control," is laid down in the 25 March 2010 SEC en banc ruling in Redmont Consolidated Mines, Corp. v. McArthur
Mining, Inc., et al., 15 to wit:
The avowed purpose of the Constitution is to place in the hands of Filipinos the exploitation of
our natural resources. Necessarily, therefore, the Rule interpreting the constitutional provision
should not diminish that right through the legal fiction of corporate ownership and control. But the
constitutional provision, as interpreted and practiced via the 1967 SEC Rules, has favored foreigners
contrary to the command of the Constitution. Hence, the Grandfather Rule must be applied to
accurately determine the actual participation, both direct and indirect, of foreigners in a
corporation engaged in a nationalized activity or business.
Compliance with the constitutional limitation(s) on engaging in nationalized activities must be
determined by ascertaining if 60% of the investing corporation's outstanding capital stock is owned by
"Filipino citizens", or as interpreted, by natural or individual Filipino citizens. If such investing corporation
is in turn owned to some extent by another investing corporation, the same process must be observed.
One must not stop until the citizenships of the individual or natural stockholders of layer after layer of
investing corporations have been established, the very essence of the Grandfather Rule. EScHDA
Lastly, it was the intent of the framers of the 1987 Constitution to adopt the Grandfather
Rule. In one of the discussions on what is now Article XII of the present Constitution, the framers made
the following exchange:
MR. NOLLEDO.
In Sections 3, 9 and 15, the Committee stated local or Filipino equity and foreign equity;
namely, 60-40 in Section 3, 60-40 in Section 9, and 2/3-1/3 in Section 15.
MR. VILLEGAS.
That is right.
MR. NOLLEDO.
In teaching law, we are always faced with the question: 'Where do we base the equity
requirement, is it on the authorized capital stock, on the subscribed capital stock, or on
the paid-up capital stock of a corporation'? Will the Committee please enlighten me on
this?
MR. VILLEGAS.
We have just had a long discussion with the members of the team from the UP Law
Center who provided us a draft. The phrase that is contained here which we adopted
from the UP draft is '60 percent of voting stock.'
MR. NOLLEDO.
That must be based on the subscribed capital stock, because unless declared
delinquent, unpaid capital stock shall be entitled to vote.
MR. VILLEGAS.
That is right.
MR. NOLLEDO.
Thank you. With respect to an investment by one corporation in another corporation,
say, a corporation with 60-40 percent equity invests in another corporation which is
permitted by the Corporation Code, does the Committee adopt the grandfather rule?
MR. VILLEGAS.
Yes, that is the understanding of the Committee.
MR. NOLLEDO.
Therefore, we need additional Filipino capital?
MR. VILLEGAS.
Yes. (Boldfacing and underscoring supplied; italicization in the original)
This SEC en banc ruling conforms to our 28 June 2011 Decision that the 60-40 ownership requirement in favor
of Filipino citizens in the Constitution to engage in certain economic activities applies not only to voting control of the
corporation, but also to the beneficial ownership of the corporation. Thus, in our 28 June 2011 Decision we stated:
Mere legal title is insufficient to meet the 60 percent Filipino-owned "capital" required in the
Constitution. Full beneficial ownership of 60 percent of the outstanding capital stock, coupled with
60 percent of the voting rights, is required. The legal and beneficial ownership of 60 percent of the
outstanding capital stock must rest in the hands of Filipino nationals in accordance with the constitutional
mandate. Otherwise, the corporation is "considered as non-Philippine national[s]." (Emphasis
supplied) ADEaHT
Both the Voting Control Test and the Beneficial Ownership Test must be applied to determine whether a
corporation is a "Philippine national."
The interpretation by legal officers of the SEC of the term "capital," embodied in various opinions which
respondents relied upon, is merely preliminary and an opinion only of such officers. To repeat, any such opinion does
not constitute an SEC rule or regulation. In fact, many of these opinions contain a disclaimer which expressly states:
". . . the foregoing opinion is based solely on facts disclosed in your query and relevant only to the particular issue
raised therein and shall not be used in the nature of a standing rule binding upon the Commission in other cases
whether of similar or dissimilar circumstances." 16 Thus, the opinions clearly make a caveatthat they do not
constitute binding precedents on any one, not even on the SEC itself.
Likewise, the opinions of the SEC en banc, as well as of the DOJ, interpreting the law are neither conclusive
nor controlling and thus, do not bind the Court. It is hornbook doctrine that any interpretation of the law that
administrative or quasi-judicial agencies make is only preliminary, never conclusive on the Court. The power to make a
final interpretation of the law, in this case the term "capital" in Section 11, Article XII of the 1987 Constitution, lies with
this Court, not with any other government entity.
In his motion for reconsideration, the PSE President cites the cases of National Telecommunications
Commission v. Court of Appeals 17 and Philippine Long Distance Telephone Company v. National Telecommunications
Commission 18 in arguing that the Court has already defined the term "capital" in Section 11, Article XII of the 1987
Constitution. 19
The PSE President is grossly mistaken. In both cases of National Telecommunications v. Court of
Appeals 20 and Philippine Long Distance Telephone Company v. National Telecommunications Commission, 21 the
Court did not define the term "capital" as found in Section 11, Article XII of the 1987 Constitution. In fact, these two
cases never mentioned, discussed or cited Section 11, Article XII of the Constitution or any of its economic
provisions, and thus cannot serve as precedent in the interpretation of Section 11, Article XII of the
Constitution. These two cases dealt solely with the determination of the correct regulatory fees under Section 40 (e)
and (f) of the Public Service Act, to wit: SEIDAC
(e) For annual reimbursement of the expenses incurred by the Commission in the supervision of other
public services and/or in the regulation or fixing of their rates, twenty centavos for each one hundred
pesos or fraction thereof, of the capital stock subscribed or paid, or if no shares have been issued,
of the capital invested, or of the property and equipment whichever is higher.
(f) For the issue or increase of capital stock, twenty centavos for each one hundred pesos or fraction
thereof, of the increased capital. (Emphasis supplied)
The Court's interpretation in these two cases of the terms "capital stock subscribed or paid," "capital stock" and
"capital" does not pertain to, and cannot control, the definition of the term "capital" as used in Section 11, Article XII of
the Constitution, or any of the economic provisions of the Constitution where the term "capital" is found. The definition of
the term "capital" found in the Constitution must not be taken out of context. A careful reading of these two cases
reveals that the terms "capital stock subscribed or paid," "capital stock" and "capital" were defined solely to determine
the basis for computing the supervision and regulation fees under Section 40 (e) and (f) of the Public Service Act.
III.
Filipinization of Public Utilities
The Preamble of the 1987 Constitution, as the prologue of the supreme law of the land, embodies the ideals
that the Constitution intends to achieve. 22 The Preamble reads:
We, the sovereign Filipino people, imploring the aid of Almighty God, in order to build a just and
humane society, and establish a Government that shall embody our ideals and aspirations, promote the
common good, conserve and develop our patrimony, and secure to ourselves and our posterity, the
blessings of independence and democracy under the rule of law and a regime of truth, justice, freedom,
love, equality, and peace, do ordain and promulgate this Constitution. (Emphasis supplied)
Consistent with these ideals, Section 19, Article II of the 1987 Constitution declares as State policy the
development of a national economy "effectively controlled" by Filipinos: DHIcET
Section 19. The State shall develop a self-reliant and independent national economy effectively
controlled by Filipinos.
Fortifying the State policy of a Filipino-controlled economy, the Constitution decrees:
Section 10. The Congress shall, upon recommendation of the economic and planning agency, when
the national interest dictates, reserve to citizens of the Philippines or to corporations or associations at
least sixty per centum of whose capital is owned by such citizens, or such higher percentage as
Congress may prescribe, certain areas of investments. The Congress shall enact measures that will
encourage the formation and operation of enterprises whose capital is wholly owned by Filipinos.
In the grant of rights, privileges, and concessions covering the national economy and patrimony,
the State shall give preference to qualified Filipinos.
The State shall regulate and exercise authority over foreign investments within its national
jurisdiction and in accordance with its national goals and priorities. 23
Under Section 10, Article XII of the 1987 Constitution, Congress may "reserve to citizens of the Philippines or to
corporations or associations at least sixty per centum of whose capital is owned by such citizens, or such higher
percentage as Congress may prescribe, certain areas of investments." Thus, in numerous laws Congress has reserved
certain areas of investments to Filipino citizens or to corporations at least sixty percent of the "capital" of which is
owned by Filipino citizens. Some of these laws are: (1) Regulation of Award of Government Contracts or R.A. No. 5183;
(2) Philippine Inventors Incentives Act or R.A. No. 3850; (3) Magna Carta for Micro, Small and Medium Enterprises or
R.A. No. 6977; (4) Philippine Overseas Shipping Development Act or R.A. No. 7471; (5) Domestic Shipping
Development Act of 2004 or R.A. No. 9295; (6) Philippine Technology Transfer Act of 2009 or R.A. No. 10055; and (7)
Ship Mortgage Decree or P.D. No. 1521. ATCaDE
With respect to public utilities, the 1987 Constitution specifically ordains:
Section 11. No franchise, certificate, or any other form of authorization for the operation of a
public utility shall be granted except to citizens of the Philippines or to corporations or
associations organized under the laws of the Philippines, at least sixty per centum of whose
capital is owned by such citizens; nor shall such franchise, certificate, or authorization be exclusive
in character or for a longer period than fifty years. Neither shall any such franchise or right be granted
except under the condition that it shall be subject to amendment, alteration, or repeal by the Congress
when the common good so requires. The State shall encourage equity participation in public utilities by
the general public. The participation of foreign investors in the governing body of any public utility
enterprise shall be limited to their proportionate share in its capital, and all the executive and managing
officers of such corporation or association must be citizens of the Philippines. (Emphasis supplied)
This provision, which mandates the Filipinization of public utilities, requires that any form of authorization for the
operation of public utilities shall be granted only to "citizens of the Philippines or to corporations or associations
organized under the laws of the Philippines at least sixty per centum of whose capital is owned by such citizens." "The
provision is [an express] recognition of the sensitive and vital position of public utilities both in the national
economy and for national security." 24
The 1987 Constitution reserves the ownership and operation of public utilities exclusively to (1) Filipino citizens,
or (2) corporations or associations at least 60 percent of whose "capital" is owned by Filipino citizens. Hence, in the
case of individuals, only Filipino citizens can validly own and operate a public utility. In the case of corporations or
associations, at least 60 percent of their "capital" must be owned by Filipino citizens. In other words, under Section
11, Article XII of the 1987 Constitution, to own and operate a public utility a corporation's capital must at least
be 60 percent owned by Philippine nationals. IcDCaT
IV.
Definition of "Philippine National"
Pursuant to the express mandate of Section 11, Article XII of the 1987 Constitution, Congress enacted Republic
Act No. 7042 or theForeign Investments Act of 1991 (FIA), as amended, which defined a "Philippine national" as
follows:
SEC. 3. Definitions. — As used in this Act:
a. The term "Philippine national" shall mean a citizen of the Philippines; or a domestic
partnership or association wholly owned by citizens of the Philippines; or a corporation organized
under the laws of the Philippines of which at least sixty percent (60%) of the capital stock
outstanding and entitled to vote is owned and held by citizens of the Philippines; or a corporation
organized abroad and registered as doing business in the Philippines under the Corporation Code of
which one hundred percent (100%) of the capital stock outstanding and entitled to vote is wholly owned
by Filipinos or a trustee of funds for pension or other employee retirement or separation benefits, where
the trustee is a Philippine national and at least sixty percent (60%) of the fund will accrue to the benefit of
Philippine nationals: Provided, That where a corporation and its non-Filipino stockholders own stocks in a
Securities and Exchange Commission (SEC) registered enterprise, at least sixty percent (60%) of the
capital stock outstanding and entitled to vote of each of both corporations must be owned and held by
citizens of the Philippines and at least sixty percent (60%) of the members of the Board of Directors of
each of both corporations must be citizens of the Philippines, in order that the corporation, shall be
considered a "Philippine national." (Boldfacing, italicization and underscoring supplied)
Thus, the FIA clearly and unequivocally defines a "Philippine national" as a Philippine citizen, or a domestic
corporation at least "60% of the capital stock outstanding and entitled to vote" is owned by Philippine citizens.
The definition of a "Philippine national" in the FIA reiterated the meaning of such term as provided in its
predecessor statute, Executive Order No. 226 or the Omnibus Investments Code of 1987, 25 which was issued by then
President Corazon C. Aquino. Article 15 of this Code states:
Article 15. "Philippine national" shall mean a citizen of the Philippines or a diplomatic partnership or
association wholly-owned by citizens of the Philippines; or a corporation organized under the laws
of the Philippines of which at least sixty per cent (60%) of the capital stock outstanding and
entitled to vote is owned and held by citizens of the Philippines; or a trustee of funds for pension
or other employee retirement or separation benefits, where the trustee is a Philippine national and at
least sixty per cent (60%) of the fund will accrue to the benefit of Philippine nationals: Provided, That
where a corporation and its non-Filipino stockholders own stock in a registered enterprise, at least sixty
per cent (60%) of the capital stock outstanding and entitled to vote of both corporations must be owned
and held by the citizens of the Philippines and at least sixty per cent (60%) of the members of the
Board of Directors of both corporations must be citizens of the Philippines in order that the corporation
shall be considered a Philippine national. (Boldfacing, italicization and underscoring supplied) aDCIHE
Under Article 48 (3) 26 of the Omnibus Investments Code of 1987, "no corporation . . . which is not a 'Philippine
national' . . . shall do business . . . in the Philippines . . . without first securing from the Board of Investments a written
certificate to the effect that such business or economic activity . . . would not conflict with the Constitution or laws of the
Philippines." 27 Thus, a "non-Philippine national" cannot own and operate a reserved economic activity like a public
utility. This means, of course, that only a "Philippine national" can own and operate a public utility.
In turn, the definition of a "Philippine national" under Article 15 of the Omnibus Investments Code of 1987 was a
reiteration of the meaning of such term as provided in Article 14 of the Omnibus Investments Code of 1981, 28 to wit:
Article 14. "Philippine national" shall mean a citizen of the Philippines; or a domestic partnership
or association wholly owned by citizens of the Philippines; or a corporation organized under the laws
of the Philippines of which at least sixty per cent (60%) of the capital stock outstanding and
entitled to vote is owned and held by citizens of the Philippines; or a trustee of funds for pension or
other employee retirement or separation benefits, where the trustee is a Philippine national and at least
sixty per cent (60%) of the fund will accrue to the benefit of Philippine nationals: Provided, That where a
corporation and its non-Filipino stockholders own stock in a registered enterprise, at least sixty per cent
(60%) of the capital stock outstanding and entitled to vote of both corporations must be owned and held
by the citizens of the Philippines and at least sixty per cent (60%) of the members of the Board of
Directors of both corporations must be citizens of the Philippines in order that the corporation shall be
considered a Philippine national. (Boldfacing, italicization and underscoring supplied)
Under Article 69 (3) of the Omnibus Investments Code of 1981, "no corporation . . . which is not a 'Philippine
national' . . . shall do business . . . in the Philippines . . . without first securing a written certificate from the Board of
Investments to the effect that such business or economic activity . . . would not conflict with the Constitution or laws of
the Philippines." 29 Thus, a "non-Philippine national" cannot own and operate a reserved economic activity like a public
utility. Again, this means that only a "Philippine national" can own and operate a public utility.
Prior to the Omnibus Investments Code of 1981, Republic Act No. 5186 30 or the Investment Incentives Act,
which took effect on 16 September 1967, contained a similar definition of a "Philippine national," to wit:
(f) "Philippine National" shall mean a citizen of the Philippines; or a partnership or association wholly
owned by citizens of the Philippines; or a corporation organized under the laws of the Philippines
of which at least sixty per cent of the capital stock outstanding and entitled to vote is owned
and held by citizens of the Philippines; or a trustee of funds for pension or other employee
retirement or separation benefits, where the trustee is a Philippine National and at least sixty per cent
of the fund will accrue to the benefit of Philippine Nationals: Provided, That where a corporation and its
non-Filipino stockholders own stock in a registered enterprise, at least sixty per cent of the capital stock
outstanding and entitled to vote of both corporations must be owned and held by the citizens of the
Philippines and at least sixty per cent of the members of the Board of Directors of both corporations
must be citizens of the Philippines in order that the corporation shall be considered a Philippine
National. (Boldfacing, italicization and underscoring supplied) IEHDAT
Under Section 3 of Republic Act No. 5455 or the Foreign Business Regulations Act, which took effect on 30
September 1968, if the investment in a domestic enterprise by non-Philippine nationals exceeds 30% of its outstanding
capital stock, such enterprise must obtain prior approval from the Board of Investments before accepting such
investment. Such approval shall not be granted if the investment "would conflict with existing constitutional provisions
and laws regulating the degree of required ownership by Philippine nationals in the enterprise." 31 A "non-Philippine
national" cannot own and operate a reserved economic activity like a public utility. Again, this means that only a
"Philippine national" can own and operate a public utility.
The FIA, like all its predecessor statutes, clearly defines a "Philippine national" as a Filipino citizen, or
a domestic corporation "at least sixty percent (60%) of the capital stock outstanding and entitled to vote" is
owned by Filipino citizens. A domestic corporation is a "Philippine national" only if at least 60% of its voting stock is
owned by Filipino citizens. This definition of a "Philippine national" is crucial in the present case because the FIA
reiterates and clarifies Section 11, Article XII of the 1987 Constitution, which limits the ownership and operation of public
utilities to Filipino citizens or to corporations or associations at least 60% Filipino-owned.
The FIA is the basic law governing foreign investments in the Philippines, irrespective of the nature of business
and area of investment. The FIA spells out the procedures by which non-Philippine nationals can invest in the
Philippines. Among the key features of this law is the concept of a negative list or the Foreign Investments Negative
List. 32 Section 8 of the law states:
SEC. 8. List of Investment Areas Reserved to Philippine Nationals [Foreign Investment Negative
List]. — The Foreign Investment Negative List shall have two [2] component lists: A and B:
a. List A shall enumerate the areas of activities reserved to Philippine nationals by mandate of
the Constitution and specific laws.
b. List B shall contain the areas of activities and enterprises regulated pursuant to law: SEcADa
1. which are defense-related activities, requiring prior clearance and authorization from the Department
of National Defense [DND] to engage in such activity, such as the manufacture, repair, storage and/or
distribution of firearms, ammunition, lethal weapons, military ordinance, explosives, pyrotechnics and
similar materials; unless such manufacturing or repair activity is specifically authorized, with a
substantial export component, to a non-Philippine national by the Secretary of National Defense; or
2. which have implications on public health and morals, such as the manufacture and distribution of
dangerous drugs; all forms of gambling; nightclubs, bars, beer houses, dance halls, sauna and steam
bathhouses and massage clinics. (Boldfacing, underscoring and italicization supplied)
Section 8 of the FIA enumerates the investment areas "reserved to Philippine nationals." Foreign Investment
Negative List A consists of "areas of activities reserved to Philippine nationals by mandate of the Constitution
and specific laws," where foreign equity participation in any enterprise shall be limited to the maximum
percentage expressly prescribed by the Constitution and other specific laws. In short, to own and operate a
public utility in the Philippines one must be a "Philippine national" as defined in the FIA. The FIA is abundant
notice to foreign investors to what extent they can invest in public utilities in the Philippines.
To repeat, among the areas of investment covered by the Foreign Investment Negative List A is the ownership
and operation of public utilities, which the Constitution expressly reserves to Filipino citizens and to corporations at least
60% owned by Filipino citizens.In other words, Negative List A of the FIA reserves the ownership and operation of
public utilities only to "Philippine nationals," defined in Section 3 (a) of the FIA as "(1) a citizen of the
Philippines; . . . or (3) a corporation organized under the laws of the Philippines of which at least sixty percent
(60%) of the capital stock outstanding and entitled to vote is owned and held by citizens of the Philippines; or
(4) a corporation organized abroad and registered as doing business in the Philippines under the Corporation Code of
which one hundred percent (100%) of the capital stock outstanding and entitled to vote is wholly owned by Filipinos or a
trustee of funds for pension or other employee retirement or separation benefits, where the trustee is a Philippine
national and at least sixty percent (60%) of the fund will accrue to the benefit of Philippine nationals." ASDCaI
Clearly, from the effectivity of the Investment Incentives Act of 1967 to the adoption of the Omnibus
Investments Code of 1981, to the enactment of the Omnibus Investments Code of 1987, and to the passage of the
present Foreign Investments Act of 1991, or for more than four decades, the statutory definition of the term
"Philippine national" has been uniform and consistent: it means a Filipino citizen, or a domestic corporation at
least 60% of the voting stock is owned by Filipinos. Likewise, these same statutes have uniformly and
consistently required that only "Philippine nationals" could own and operate public utilities in the Philippines.
The following exchange during the Oral Arguments is revealing:
JUSTICE CARPIO:
Counsel, I have some questions. You are aware of the Foreign Investments Act of 1991, . . .?
And the FIA of 1991 took effect in 1991, correct? That's over twenty (20) years ago, correct?
COMMISSIONER GAITE:
Correct, Your Honor.
JUSTICE CARPIO:
And Section 8 of the Foreign Investments Act of 1991 states that []only Philippine nationals can
own and operate public utilities[], correct?
COMMISSIONER GAITE:
Yes, Your Honor.
JUSTICE CARPIO:
And the same Foreign Investments Act of 1991 defines a "Philippine national" either as a citizen
of the Philippines, or if it is a corporation at least sixty percent (60%) of the voting stock is owned
by citizens of the Philippines, correct? aIcTCS
COMMISSIONER GAITE:
Correct, Your Honor.
JUSTICE CARPIO:
And, you are also aware that under the predecessor law of the Foreign Investments Act of 1991,
the Omnibus Investments Act of 1987, the same provisions apply: . . . only Philippine nationals
can own and operate a public utility and the Philippine national, if it is a corporation, . . . sixty
percent (60%) of the capital stock of that corporation must be owned by citizens of the
Philippines, correct?
COMMISSIONER GAITE:
Correct, Your Honor.
JUSTICE CARPIO:
And even prior to the Omnibus Investments Act of 1987, under the Omnibus Investments Act of
1981, the same rules apply: . . . only a Philippine national can own and operate a public utility
and a Philippine national, if it is a corporation, sixty percent (60%) of its . . . voting stock, must be
owned by citizens of the Philippines, correct?
COMMISSIONER GAITE:
Correct, Your Honor.
JUSTICE CARPIO:
And even prior to that, under [the] 1967 Investments Incentives Act and the Foreign Company
Act of 1968, the same rules applied, correct? ITCHSa
COMMISSIONER GAITE:
Correct, Your Honor.
JUSTICE CARPIO:
So, for the last four (4) decades, . . ., the law has been very consistent — only a Philippine
national can own and operate a public utility, and a Philippine national, if it is a
corporation, . . . at least sixty percent (60%) of the voting stock must be owned by citizens
of the Philippines, correct?
COMMISSIONER GAITE:
Correct, Your Honor. 33 (Emphasis supplied)
Government agencies like the SEC cannot simply ignore Sections 3 (a) and 8 of the FIA which categorically
prescribe that certain economic activities, like the ownership and operation of public utilities, are reserved to
corporations "at least sixty percent (60%) of the capital stock outstanding and entitled to vote is owned and held by
citizens of the Philippines." Foreign Investment Negative List A refers to "activities reserved to Philippine nationals by
mandate of the Constitution and specific laws." The FIA is the basic statute regulating foreign investments in the
Philippines. Government agencies tasked with regulating or monitoring foreign investments, as well as counsels of
foreign investors, should start with the FIA in determining to what extent a particular foreign investment is allowed in the
Philippines. Foreign investors and their counsels who ignore the FIA do so at their own peril. Foreign investors and their
counsels who rely on opinions of SEC legal officers that obviously contradict the FIA do so also at their own peril.
Occasional opinions of SEC legal officers that obviously contradict the FIA should immediately raise a red flag.
There are already numerous opinions of SEC legal officers that cite the definition of a "Philippine national" in Section 3
(a) of the FIA in determining whether a particular corporation is qualified to own and operate a nationalized or partially
nationalized business in the Philippines. This shows that SEC legal officers are not only aware of, but also rely on and
invoke, the provisions of the FIA in ascertaining the eligibility of a corporation to engage in partially nationalized
industries. The following are some of such opinions:
1. Opinion of 23 March 1993, addressed to Mr. Francis F. How;
2. Opinion of 14 April 1993, addressed to Director Angeles T. Wong of the Philippine Overseas
Employment Administration;
3. Opinion of 23 November 1993, addressed to Messrs. Dominador Almeda and Renato S.
Calma; acADIT
4. Opinion of 7 December 1993, addressed to Roco Bunag Kapunan Migallos & Jardeleza;
5. SEC Opinion No. 49-04, addressed to Romulo Mabanta Buenaventura Sayoc & De Los Angeles;
6. SEC-OGC Opinion No. 17-07, addressed to Mr. Reynaldo G. David; and
7. SEC-OGC Opinion No. 03-08, addressed to Attys. Ruby Rose J. Yusi and Rudyard S. Arbolado.
The SEC legal officers' occasional but blatant disregard of the definition of the term "Philippine national" in the
FIA signifies their lack of integrity and competence in resolving issues on the 60-40 ownership requirement in favor of
Filipino citizens in Section 11, Article XII of the Constitution.
The PSE President argues that the term "Philippine national" defined in the FIA should be limited and
interpreted to refer to corporations seeking to avail of tax and fiscal incentives under investment incentives laws and
cannot be equated with the term "capital" in Section 11, Article XII of the 1987 Constitution. Pangilinan similarly
contends that the FIA and its predecessor statutes do not apply to "companies which have not registered and obtained
special incentives under the schemes established by those laws."
Both are desperately grasping at straws. The FIA does not grant tax or fiscal incentives to any enterprise. Tax
and fiscal incentives to investments are granted separately under the Omnibus Investments Code of 1987, not under
the FIA. In fact, the FIA expressly repealed Articles 44 to 56 of Book II of the Omnibus Investments Code of 1987, which
articles previously regulated foreign investments in nationalized or partially nationalized industries.
The FIA is the applicable law regulating foreign investments in nationalized or partially nationalized industries.
There is nothing in the FIA, or even in the Omnibus Investments Code of 1987 or its predecessor statutes, that states,
expressly or impliedly, that the FIA or its predecessor statutes do not apply to enterprises not availing of tax and fiscal
incentives under the Code. The FIA and its predecessor statutes apply to investments in all domestic enterprises,
whether or not such enterprises enjoy tax and fiscal incentives under the Omnibus Investments Code of 1987 or its
predecessor statutes. The reason is quite obvious — mere non-availment of tax and fiscal incentives by a non-
Philippine national cannot exempt it from Section 11, Article XII of the Constitution regulating foreign
investments in public utilities. In fact, the Board of Investments' Primer on Investment Policies in the
Philippines, 34 which is given out to foreign investors, provides: SDIACc
PART III.FOREIGN INVESTMENTS WITHOUT INCENTIVES. —
Investors who do not seek incentives and/or whose chosen activities do not qualify for incentives, (i.e.,
the activity is not listed in the IPP, and they are not exporting at least 70% of their production) may go
ahead and make the investments without seeking incentives.They only have to be guided by the
Foreign Investments Negative List (FINL).
The FINL clearly defines investment areas requiring at least 60% Filipino ownership. All other areas
outside of this list are fully open to foreign investors. (Emphasis supplied)
V.
Right to elect directors, coupled with beneficial ownership,
translates to effective control.
The 28 June 2011 Decision declares that the 60 percent Filipino ownership required by the Constitution to
engage in certain economic activities applies not only to voting control of the corporation, but also to the beneficial
ownership of the corporation. To repeat, we held:
Mere legal title is insufficient to meet the 60 percent Filipino-owned "capital" required in the
Constitution. Full beneficial ownership of 60 percent of the outstanding capital stock, coupled with
60 percent of the voting rights, is required. The legal and beneficial ownership of 60 percent of the
outstanding capital stock must rest in the hands of Filipino nationals in accordance with the constitutional
mandate. Otherwise, the corporation is "considered as non-Philippine national[s]." (Emphasis supplied)
This is consistent with Section 3 of the FIA which provides that where 100% of the capital stock is held by "a
trustee of funds for pension or other employee retirement or separation benefits," the trustee is a Philippine national if
"at least sixty percent (60%) of the fund will accrue to the benefit of Philippine nationals." Likewise, Section 1 (b) of the
Implementing Rules of the FIA provides that "for stocks to be deemed owned and held by Philippine citizens or
Philippine nationals, mere legal title is not enough to meet the required Filipino equity. Full beneficial ownership of
the stocks, coupled with appropriate voting rights, is essential."
Since the constitutional requirement of at least 60 percent Filipino ownership applies not only to voting control
of the corporation but also to the beneficial ownership of the corporation, it is therefore imperative that such requirement
apply uniformly and across the board to all classes of shares, regardless of nomenclature and category, comprising the
capital of a corporation. Under the Corporation Code, capital stock 35 consists of all classes of shares issued to
stockholders, that is, common shares as well as preferred shares, which may have different rights, privileges or
restrictions as stated in the articles of incorporation. 36
The Corporation Code allows denial of the right to vote to preferred and redeemable shares, but disallows
denial of the right to vote in specific corporate matters. Thus, common shares have the right to vote in the election of
directors, while preferred shares may be denied such right. Nonetheless, preferred shares, even if denied the right to
vote in the election of directors, are entitled to vote on the following corporate matters: (1) amendment of articles of
incorporation; (2) increase and decrease of capital stock; (3) incurring, creating or increasing bonded indebtedness; (4)
sale, lease, mortgage or other disposition of substantially all corporate assets; (5) investment of funds in another
business or corporation or for a purpose other than the primary purpose for which the corporation was organized; (6)
adoption, amendment and repeal of by-laws; (7) merger and consolidation; and (8) dissolution of
corporation. 37 HESAIT
Since a specific class of shares may have rights and privileges or restrictions different from the rest of the
shares in a corporation, the 60-40 ownership requirement in favor of Filipino citizens in Section 11, Article XII of the
Constitution must apply not only to shares with voting rights but also to shares without voting rights. Preferred shares,
denied the right to vote in the election of directors, are anyway still entitled to vote on the eight specific corporate
matters mentioned above. Thus, if a corporation, engaged in a partially nationalized industry, issues a mixture of
common and preferred non-voting shares, at least 60 percent of the common shares and at least 60 percent of
the preferred non-voting shares must be owned by Filipinos. Of course, if a corporation issues only a single class
of shares, at least 60 percent of such shares must necessarily be owned by Filipinos. In short, the 60-40 ownership
requirement in favor of Filipino citizens must apply separately to each class of shares, whether common,
preferred non-voting, preferred voting or any other class of shares. This uniform application of the 60-40
ownership requirement in favor of Filipino citizens clearly breathes life to the constitutional command that the ownership
and operation of public utilities shall be reserved exclusively to corporations at least 60 percent of whose capital is
Filipino-owned. Applying uniformly the 60-40 ownership requirement in favor of Filipino citizens to each class of shares,
regardless of differences in voting rights, privileges and restrictions, guarantees effective Filipino control of public
utilities, as mandated by the Constitution.
Moreover, such uniform application to each class of shares insures that the "controlling interest" in public
utilities always lies in the hands of Filipino citizens. This addresses and extinguishes Pangilinan's worry that foreigners,
owning most of the non-voting shares, will exercise greater control over fundamental corporate matters requiring two-
thirds or majority vote of all shareholders.
VI.
Intent of the framers of the Constitution
While Justice Velasco quoted in his Dissenting Opinion 38 a portion of the deliberations of the Constitutional
Commission to support his claim that the term "capital" refers to the total outstanding shares of stock, whether voting or
non-voting, the following excerpts of the deliberations reveal otherwise. It is clear from the following exchange that the
term "capital" refers to controlling interest of a corporation, thus: TcIAHS
MR. NOLLEDO.
In Sections 3, 9 and 15, the Committee stated local or Filipino equity and foreign equity; namely,
60-40 in Section 3, 60-40 in Section 9 and 2/3-1/3 in Section 15.
MR. VILLEGAS.
That is right.
MR. NOLLEDO.
In teaching law, we are always faced with this question: "Where do we base the equity
requirement, is it on the authorized capital stock, on the subscribed capital stock, or on the paid-
up capital stock of a corporation"? Will the Committee please enlighten me on this?
MR. VILLEGAS.
We have just had a long discussion with the members of the team from the UP Law Center who
provided us a draft. The phrase that is contained here which we adopted from the UP draft
is "60 percent of voting stock."
MR. NOLLEDO.
That must be based on the subscribed capital stock, because unless declared delinquent, unpaid
capital stock shall be entitled to vote.
MR. VILLEGAS.
That is right.
MR. NOLLEDO.
Thank you.
With respect to an investment by one corporation in another corporation, say, a corporation with
60-40 percent equity invests in another corporation which is permitted by the Corporation Code,
does the Committee adopt the grandfather rule?
MR. VILLEGAS.
Yes, that is the understanding of the Committee.
MR. NOLLEDO.
Therefore, we need additional Filipino capital? cDTCIA
MR. VILLEGAS.
Yes. 39
xxx xxx xxx
MR. AZCUNA.
May I be clarified as to that portion that was accepted by the Committee.
MR. VILLEGAS.
The portion accepted by the Committee is the deletion of the phrase "voting stock or controlling
interest."
MR. AZCUNA.
Hence, without the Davide amendment, the committee report would read: "corporations or
associations at least sixty percent of whose CAPITAL is owned by such citizens."
MR. VILLEGAS.
Yes.
MR. AZCUNA.
So if the Davide amendment is lost, we are stuck with 60 percent of the capital to be owned by
citizens.
MR. VILLEGAS.
That is right.
MR. AZCUNA.
But the control can be with the foreigners even if they are the minority. Let us say 40
percent of the capital is owned by them, but it is the voting capital, whereas, the Filipinos
own the nonvoting shares. So we can have a situation where the corporation is controlled
by foreigners despite being the minority because they have the voting capital. That is the
anomaly that would result here. TAHcCI
MR. BENGZON.
No, the reason we eliminated the word "stock" as stated in the 1973 and 1935
Constitutions is that according to Commissioner Rodrigo, there are associations that do
not have stocks. That is why we say "CAPITAL."
MR. AZCUNA.
We should not eliminate the phrase "controlling interest."
MR. BENGZON.
In the case of stock corporations, it is assumed. 40 (Boldfacing and underscoring supplied)
Thus, 60 percent of the "capital" assumes, or should result in, a "controlling interest" in the corporation.
The use of the term "capital" was intended to replace the word "stock" because associations without stocks can
operate public utilities as long as they meet the 60-40 ownership requirement in favor of Filipino citizens prescribed in
Section 11, Article XII of the Constitution. However, this did not change the intent of the framers of the Constitution to
reserve exclusively to Philippine nationals the "controlling interest" in public utilities.
During the drafting of the 1935 Constitution, economic protectionism was "the battle-cry of the nationalists in the
Convention." 41The same battle-cry resulted in the nationalization of the public utilities. 42 This is also the same intent
of the framers of the 1987 Constitution who adopted the exact formulation embodied in the 1935 and 1973 Constitutions
on foreign equity limitations in partially nationalized industries.
The OSG, in its own behalf and as counsel for the State, 43 agrees fully with the Court's interpretation of the
term "capital." In its Consolidated Comment, the OSG explains that the deletion of the phrase "controlling interest" and
replacement of the word "stock" with the term "capital" were intended specifically to extend the scope of the entities
qualified to operate public utilities to include associations without stocks. The framers' omission of the phrase
"controlling interest" did not mean the inclusion of all shares of stock, whether voting or non-voting. The OSG reiterated
essentially the Court's declaration that the Constitution reserved exclusively to Philippine nationals the ownership and
operation of public utilities consistent with the State's policy to "develop a self-reliant and independent national
economy effectively controlled by Filipinos." cDTIAC
As we held in our 28 June 2011 Decision, to construe broadly the term "capital" as the total outstanding capital
stock, treated as asingle class regardless of the actual classification of shares, grossly contravenes the intent and letter
of the Constitution that the "State shall develop a self-reliant and independent national economy effectively
controlled by Filipinos." We illustrated the glaring anomaly which would result in defining the term "capital" as the total
outstanding capital stock of a corporation, treated as a single class of shares regardless of the actual classification of
shares, to wit:
Let us assume that a corporation has 100 common shares owned by foreigners and 1,000,000
non-voting preferred shares owned by Filipinos, with both classes of share having a par value of one
peso (P1.00) per share. Under the broad definition of the term "capital," such corporation would be
considered compliant with the 40 percent constitutional limit on foreign equity of public utilities since the
overwhelming majority, or more than 99.999 percent, of the total outstanding capital stock is Filipino
owned. This is obviously absurd.
In the example given, only the foreigners holding the common shares have voting rights in the
election of directors, even if they hold only 100 shares. The foreigners, with a minuscule equity of less
than 0.001 percent, exercise control over the public utility. On the other hand, the Filipinos, holding more
than 99.999 percent of the equity, cannot vote in the election of directors and hence, have no control over
the public utility. This starkly circumvents the intent of the framers of the Constitution, as well as the clear
language of the Constitution, to place the control of public utilities in the hands of Filipinos. . . .
Further, even if foreigners who own more than forty percent of the voting shares elect an all-Filipino board of
directors, this situation does not guarantee Filipino control and does not in any way cure the violation of the
Constitution. The independence of the Filipino board members so elected by such foreign shareholders is highly
doubtful. As the OSG pointed out, quoting Justice George Sutherland's words in Humphrey's Executor v. US, 44 ". . . it
is quite evident that one who holds his office only during the pleasure of another cannot be depended upon to maintain
an attitude of independence against the latter's will." Allowing foreign shareholders to elect a controlling majority of the
board, even if all the directors are Filipinos, grossly circumvents the letter and intent of the Constitution and defeats the
very purpose of our nationalization laws. TICAcD
VII.
Last sentence of Section 11, Article XII of the Constitution
The last sentence of Section 11, Article XII of the 1987 Constitution reads:
The participation of foreign investors in the governing body of any public utility enterprise shall be
limited to their proportionate share in its capital, and all the executive and managing officers of such
corporation or association must be citizens of the Philippines.
During the Oral Arguments, the OSG emphasized that there was never a question on the intent of the framers
of the Constitution to limit foreign ownership, and assure majority Filipino ownership and control of public utilities. The
OSG argued, "while the delegates disagreed as to the percentage threshold to adopt, . . . the records show they clearly
understood that Filipino control of the public utility corporation can only be and is obtained only through the election of a
majority of the members of the board."
Indeed, the only point of contention during the deliberations of the Constitutional Commission on 23 August
1986 was the extent of majority Filipino control of public utilities. This is evident from the following exchange:
THE PRESIDENT.
Commissioner Jamir is recognized.
MR. JAMIR.
Madam President, my proposed amendment on lines 20 and 21 is to delete the phrase "two
thirds of whose voting stock or controlling interest," and instead substitute the words "SIXTY
PERCENT OF WHOSE CAPITAL" so that the sentence will read: "No franchise, certificate, or
any other form of authorization for the operation of a public utility shall be granted except to
citizens of the Philippines or to corporations or associations organized under the laws of the
Philippines at least SIXTY PERCENT OF WHOSE CAPITAL is owned by such
citizens." SEcTHA
xxx xxx xxx
THE PRESIDENT:
Will Commissioner Jamir first explain?
MR. JAMIR.
Yes, in this Article on National Economy and Patrimony, there were two previous sections in
which we fixed the Filipino equity to 60 percent as against 40 percent for foreigners. It is only in
this Section 15 with respect to public utilities that the committee proposal was increased to two-
thirds. I think it would be better to harmonize this provision by providing that even in the case of
public utilities, the minimum equity for Filipino citizens should be 60 percent.
MR. ROMULO.
Madam President.
THE PRESIDENT.
Commissioner Romulo is recognized.
MR. ROMULO.
My reason for supporting the amendment is based on the discussions I have had with
representatives of the Filipino majority owners of the international record carriers, and the
subsequent memoranda they submitted to me. . . .
Their second point is that under the Corporation Code, the management and control of a
corporation is vested in the board of directors, not in the officers but in the board of directors. The
officers are only agents of the board. And they believe that with 60 percent of the equity, the
Filipino majority stockholders undeniably control the board. Only on important corporate acts can
the 40-percent foreign equity exercise a veto, . . . . DcHSEa
xxx xxx xxx 45
MS. ROSARIO BRAID.
Madam President.
THE PRESIDENT.
Commissioner Rosario Braid is recognized.
MS. ROSARIO BRAID.
Yes, in the interest of equal time, may I also read from a memorandum by the spokesman of the
Philippine Chamber of Communications on why they would like to maintain the present equity, I
am referring to the 66 2/3. They would prefer to have a 75-25 ratio but would settle for 66 2/3. . . .
xxx xxx xxx
THE PRESIDENT.
Just to clarify, would Commissioner Rosario Braid support the proposal of two-thirds rather than
the 60 percent?
MS. ROSARIO BRAID.
I have added a clause that will put management in the hands of Filipino citizens.
xxx xxx xxx 46
While they had differing views on the percentage of Filipino ownership of capital, it is clear that the framers of
the Constitution intended public utilities to be majority Filipino-owned and controlled. To ensure that Filipinos control
public utilities, the framers of the Constitution approved, as additional safeguard, the inclusion of the last sentence of
Section 11, Article XII of the Constitution commanding that "[t]he participation of foreign investors in the governing body
of any public utility enterprise shall be limited to their proportionate share in its capital, and all the executive and
managing officers of such corporation or association must be citizens of the Philippines." In other words, the last
sentence of Section 11, Article XII of the Constitution mandates that (1) the participation of foreign investors in the
governing body of the corporation or association shall be limited to their proportionate share in the capital of such entity;
and (2) all officers of the corporation or association must be Filipino citizens.
Commissioner Rosario Braid proposed the inclusion of the phrase requiring the managing officers of the
corporation or association to be Filipino citizens specifically to prevent management contracts, which were designed
primarily to circumvent the Filipinization of public utilities, and to assure Filipino control of public utilities, thus:
MS. ROSARIO BRAID.
. . . They also like to suggest that we amend this provision by adding a phrase which states:
"THE MANAGEMENT BODY OF EVERY CORPORATION OR ASSOCIATION SHALL IN ALL
CASES BE CONTROLLED BY CITIZENS OF THE PHILIPPINES." I have with me their position
paper.
THE PRESIDENT.
The Commissioner may proceed. TDcHCa
MS. ROSARIO BRAID.
The three major international record carriers in the Philippines, which Commissioner Romulo
mentioned — Philippine Global Communications, Eastern Telecommunications, Globe Mackay
Cable — are 40-percent owned by foreign multinational companies and 60-percent owned by
their respective Filipino partners. All three, however, also have management contracts with these
foreign companies — Philcom with RCA, ETPI with Cable and Wireless PLC, and GMCR with
ITT. Up to the present time, the general managers of these carriers are foreigners. While the
foreigners in these common carriers are only minority owners, the foreign multinationals are the
ones managing and controlling their operations by virtue of their management contracts and by
virtue of their strength in the governing bodies of these carriers. 47
xxx xxx xxx
MR. OPLE.
I think a number of us have agreed to ask Commissioner Rosario Braid to propose an
amendment with respect to the operating management of public utilities, and in this amendment,
we are associated with Fr. Bernas, Commissioners Nieva and Rodrigo. Commissioner Rosario
Braid will state this amendment now.
Thank you.
MS. ROSARIO BRAID.
Madam President. ADCSEa
THE PRESIDENT.
This is still on Section 15.
MS. ROSARIO BRAID.
Yes.
MR. VILLEGAS.
Yes, Madam President.
xxx xxx xxx
MS. ROSARIO BRAID.
Madam President, I propose a new section to read: 'THE MANAGEMENT BODY OF EVERY
CORPORATION OR ASSOCIATION SHALL IN ALL CASES BE CONTROLLED BY CITIZENS
OF THE PHILIPPINES."
This will prevent management contracts and assure control by Filipino citizens. Will the
committee assure us that this amendment will insure that past activities such as management
contracts will no longer be possible under this amendment?
xxx xxx xxx
FR. BERNAS.
Madam President.
THE PRESIDENT.
Commissioner Bernas is recognized.
FR. BERNAS.
Will the committee accept a reformulation of the first part?
MR. BENGZON.
Let us hear it.
FR. BERNAS.
The reformulation will be essentially the formula of the 1973 Constitution which reads: "THE
PARTICIPATION OF FOREIGN INVESTORS IN THE GOVERNING BODY OF ANY PUBLIC
UTILITY ENTERPRISE SHALL BE LIMITED TO THEIR PROPORTIONATE SHARE IN THE
CAPITAL THEREOF AND . . ."
MR. VILLEGAS.
"ALL THE EXECUTIVE AND MANAGING OFFICERS OF SUCH CORPORATIONS AND
ASSOCIATIONS MUST BE CITIZENS OF THE PHILIPPINES."
MR. BENGZON.
Will Commissioner Bernas read the whole thing again?
FR. BERNAS.
"THE PARTICIPATION OF FOREIGN INVESTORS IN THE GOVERNING BODY OF ANY
PUBLIC UTILITY ENTERPRISE SHALL BE LIMITED TO THEIR PROPORTIONATE SHARE IN
THE CAPITAL THEREOF . . ." I do not have the rest of the copy. CDaSAE
MR. BENGZON.
"AND ALL THE EXECUTIVE AND MANAGING OFFICERS OF SUCH CORPORATIONS OR
ASSOCIATIONS MUST BE CITIZENS OF THE PHILIPPINES." Is that correct?
MR. VILLEGAS.
Yes.
MR. BENGZON.
Madam President, I think that was said in a more elegant language. We accept the amendment.
Is that all right with Commissioner Rosario Braid?
MS. ROSARIO BRAID.
Yes.
xxx xxx xxx
MR. DE LOS REYES.
The governing body refers to the board of directors and trustees.
MR. VILLEGAS.
That is right.
MR. BENGZON.
Yes, the governing body refers to the board of directors.
MR. REGALADO.
It is accepted.
MR. RAMA.
The body is now ready to vote, Madam President.
VOTING
xxx xxx xxx
The results show 29 votes in favor and none against; so the proposed amendment is approved.
xxx xxx xxx
THE PRESIDENT.
All right. Can we proceed now to vote on Section 15?
MR. RAMA.
Yes, Madam President.
THE PRESIDENT.
Will the chairman of the committee please read Section 15?
MR. VILLEGAS.
The entire Section 15, as amended, reads: "No franchise, certificate, or any other form of
authorization for the operation of a public utility shall be granted except to citizens of the
Philippines or to corporations or associations organized under the laws of the Philippines at least
60 PERCENT OF WHOSE CAPITAL is owned by such citizens." May I request Commissioner
Bengzon to please continue reading. SCHcaT
MR. BENGZON.
"THE PARTICIPATION OF FOREIGN INVESTORS IN THE GOVERNING BODY OF ANY
PUBLIC UTILITY ENTERPRISE SHALL BE LIMITED TO THEIR PROPORTIONATE SHARE IN
THE CAPITAL THEREOF AND ALL THE EXECUTIVE AND MANAGING OFFICERS OF SUCH
CORPORATIONS OR ASSOCIATIONS MUST BE CITIZENS OF THE PHILIPPINES."
MR. VILLEGAS.
"NOR SHALL SUCH FRANCHISE, CERTIFICATE OR AUTHORIZATION BE EXCLUSIVE IN
CHARACTER OR FOR A PERIOD LONGER THAN TWENTY-FIVE YEARS RENEWABLE FOR
NOT MORE THAN TWENTY-FIVE YEARS. Neither shall any such franchise or right be granted
except under the condition that it shall be subject to amendment, alteration, or repeal by
Congress when the common good so requires. The State shall encourage equity participation in
public utilities by the general public."
VOTING
xxx xxx xxx
The results show 29 votes in favor and 4 against; Section 15, as amended, is
approved. 48 (Emphasis supplied)
The last sentence of Section 11, Article XII of the 1987 Constitution, particularly the provision on the limited
participation of foreign investors in the governing body of public utilities, is a reiteration of the last sentence of Section 5,
Article XIV of the 1973 Constitution, 49signifying its importance in reserving ownership and control of public utilities to
Filipino citizens.
VIII.
The undisputed facts
There is no dispute, and respondents do not claim the contrary, that (1) foreigners own 64.27% of the common
shares of PLDT, which class of shares exercises the sole right to vote in the election of directors, and thus foreigners
control PLDT; (2) Filipinos own only 35.73% of PLDT's common shares, constituting a minority of the voting stock, and
thus Filipinos do not control PLDT; (3) preferred shares, 99.44% owned by Filipinos, have no voting rights; (4) preferred
shares earn only 1/70 of the dividends that common shares earn;50 (5) preferred shares have twice the par value of
common shares; and (6) preferred shares constitute 77.85% of the authorized capital stock of PLDT and common
shares only 22.15%. cSTCDA
Despite the foregoing facts, the Court did not decide, and in fact refrained from ruling on the question of
whether PLDT violated the 60-40 ownership requirement in favor of Filipino citizens in Section 11, Article XII of the 1987
Constitution. Such question indisputably calls for a presentation and determination of evidence through a hearing, which
is generally outside the province of the Court's jurisdiction, but well within the SEC's statutory powers. Thus, for obvious
reasons, the Court limited its decision on the purely legal and threshold issue on the definition of the term "capital" in
Section 11, Article XII of the Constitution and directed the SEC to apply such definition in determining the exact
percentage of foreign ownership in PLDT.
IX.
PLDT is not an indispensable party;
SEC is impleaded in this case.
In his petition, Gamboa prays, among others:
xxx xxx xxx
5. For the Honorable Court to issue a declaratory relief that ownership of common or voting
shares is the sole basis in determining foreign equity in a public utility and that any other government
rulings, opinions, and regulations inconsistent with this declaratory relief be declared unconstitutional and
a violation of the intent and spirit of the 1987 Constitution;
6. For the Honorable Court to declare null and void all sales of common stocks to foreigners in
excess of 40 percent of the total subscribed common shareholdings; and
7. For the Honorable Court to direct the Securities and Exchange Commission and Philippine
Stock Exchange to require PLDT to make a public disclosure of all of its foreign shareholdings and
their actual and real beneficial owners.
Other relief(s) just and equitable are likewise prayed for. (Emphasis supplied)
As can be gleaned from his prayer, Gamboa clearly asks this Court to compel the SEC to perform its statutory
duty to investigate whether "the required percentage of ownership of the capital stock to be owned by citizens of the
Philippines has been complied with [by PLDT] as required by . . . the Constitution." 51 Such plea clearly negates SEC's
argument that it was not impleaded.
Granting that only the SEC Chairman was impleaded in this case, the Court has ample powers to order the
SEC's compliance with its directive contained in the 28 June 2011 Decision in view of the far-reaching implications of
this case. In Domingo v. Scheer, 52 the Court dispensed with the amendment of the pleadings to implead the Bureau of
Customs considering (1) the unique backdrop of the case; (2) the utmost need to avoid further delays; and (3) the issue
of public interest involved. The Court held: aIcDCH
The Court may be curing the defect in this case by adding the BOC as party-petitioner. The
petition should not be dismissed because the second action would only be a repetition of the first.
In Salvador, et al. v. Court of Appeals, et al., we held that this Court has full powers, apart from that
power and authority which is inherent, to amend the processes, pleadings, proceedings and decisions by
substituting as party-plaintiff the real party-in-interest. The Court has the power to avoid delay in the
disposition of this case, to order its amendment as to implead the BOC as party-respondent.
Indeed, it may no longer be necessary to do so taking into account the unique backdrop in this
case, involving as it does an issue of public interest. After all, the Office of the Solicitor General has
represented the petitioner in the instant proceedings, as well as in the appellate court, and maintained
the validity of the deportation order and of the BOC's Omnibus Resolution. It cannot, thus, be claimed by
the State that the BOC was not afforded its day in court, simply because only the petitioner, the
Chairperson of the BOC, was the respondent in the CA, and the petitioner in the instant recourse.
In Alonso v. Villamor, we had the occasion to state:
There is nothing sacred about processes or pleadings, their forms or contents.
Their sole purpose is to facilitate the application of justice to the rival claims of
contending parties. They were created, not to hinder and delay, but to facilitate and promote,
the administration of justice. They do not constitute the thing itself, which courts are always
striving to secure to litigants. They are designed as the means best adapted to obtain that thing.
In other words, they are a means to an end. When they lose the character of the one and
become the other, the administration of justice is at fault and courts are correspondingly remiss
in the performance of their obvious duty. 53 (Emphasis supplied)
In any event, the SEC has expressly manifested 54 that it will abide by the Court's decision and defer to
the Court's definition of the term "capital" in Section 11, Article XII of the Constitution. Further, the SEC entered
its special appearance in this case and argued during the Oral Arguments, indicating its submission to the
Court's jurisdiction. It is clear, therefore, that there exists no legal impediment against the proper and
immediate implementation of the Court's directive to the SEC. DEcTIS
PLDT is an indispensable party only insofar as the other issues, particularly the factual questions, are
concerned. In other words, PLDT must be impleaded in order to fully resolve the issues on (1) whether the sale of
111,415 PTIC shares to First Pacific violates the constitutional limit on foreign ownership of PLDT; (2) whether the sale
of common shares to foreigners exceeded the 40 percent limit on foreign equity in PLDT; and (3) whether the total
percentage of the PLDT common shares with voting rights complies with the 60-40 ownership requirement in favor of
Filipino citizens under the Constitution for the ownership and operation of PLDT. These issues indisputably call for an
examination of the parties' respective evidence, and thus are clearly within the jurisdiction of the SEC. In short, PLDT
must be impleaded, and must necessarily be heard, in the proceedings before the SEC where the factual issues will be
thoroughly threshed out and resolved.
Notably, the foregoing issues were left untouched by the Court. The Court did not rule on the factual
issues raised by Gamboa, except the single and purely legal issue on the definition of the term "capital" in Section 11,
Article XII of the Constitution. The Court confined the resolution of the instant case to this threshold legal issue in
deference to the fact-finding power of the SEC.
Needless to state, the Court can validly, properly, and fully dispose of the fundamental legal issue in this case
even without the participation of PLDT since defining the term "capital" in Section 11, Article XII of the Constitution does
not, in any way, depend on whether PLDT was impleaded. Simply put, PLDT is not indispensable for a complete
resolution of the purely legal question in this case. 55In fact, the Court, by treating the petition as one
for mandamus, 56 merely directed the SEC to apply the Court's definition of the term "capital" in Section 11, Article XII
of the Constitution in determining whether PLDT committed any violation of the said constitutional provision. The
dispositive portion of the Court's ruling is addressed not to PLDT but solely to the SEC, which is the
administrative agency tasked to enforce the 60-40 ownership requirement in favor of Filipino citizens in Section
11, Article XII of the Constitution.
Since the Court limited its resolution on the purely legal issue on the definition of the term "capital" in Section
11, Article XII of the 1987 Constitution, and directed the SEC to investigate any violation by PLDT of the 60-40
ownership requirement in favor of Filipino citizens under the Constitution, 57 there is no deprivation of PLDT's property
or denial of PLDT's right to due process, contrary to Pangilinan and Nazareno's misimpression. Due process will be
afforded to PLDT when it presents proof to the SEC that it complies, as it claims here, with Section 11, Article XII of the
Constitution. AHaDSI
X.
Foreign Investments in the Philippines
Movants fear that the 28 June 2011 Decision would spell disaster to our economy, as it may result in a sudden
flight of existing foreign investors to "friendlier" countries and simultaneously deterring new foreign investors to our
country. In particular, the PSE claims that the 28 June 2011 Decision may result in the following: (1) loss of more than
P630 billion in foreign investments in PSE-listed shares; (2) massive decrease in foreign trading transactions; (3) lower
PSE Composite Index; and (4) local investors not investing in PSE-listed shares. 58
Dr. Bernardo M. Villegas, one of the amici curiae in the Oral Arguments, shared movants' apprehension.
Without providing specific details, he pointed out the depressing state of the Philippine economy compared to our
neighboring countries which boast of growing economies. Further, Dr. Villegas explained that the solution to our
economic woes is for the government to "take-over" strategic industries, such as the public utilities sector, thus:
JUSTICE CARPIO:
I would like also to get from you Dr. Villegas if you have additional information on whether this
high FDI 59 countries in East Asia have allowed foreigners . . . control [of] their public utilities, so
that we can compare apples with apples.
DR. VILLEGAS:
Correct, but let me just make a comment. When these neighbors of ours find an industry
strategic, their solution is not to "Filipinize" or "Vietnamize" or "Singaporize." Their solution is to
make sure that those industries are in the hands of state enterprises. So, in these
countries, nationalization means the government takes over. And because their
governments are competent and honest enough to the public, that is the
solution. . . . 60 (Emphasis supplied) cEAaIS
If government ownership of public utilities is the solution, then foreign investments in our public utilities serve no
purpose. Obviously, there can never be foreign investments in public utilities if, as Dr. Villegas claims, the "solution is to
make sure that those industries are in the hands of state enterprises." Dr. Villegas's argument that foreign investments
in telecommunication companies like PLDT are badly needed to save our ailing economy contradicts his own theory
that the solution is for government to take over these companies. Dr. Villegas is barking up the wrong tree since State
ownership of public utilities and foreign investments in such industries are diametrically opposed concepts, which
cannot possibly be reconciled.
In any event, the experience of our neighboring countries cannot be used as argument to decide the present
case differently for two reasons. First, the governments of our neighboring countries have, as claimed by Dr. Villegas,
taken over ownership and control of their strategic public utilities like the telecommunications industry. Second, our
Constitution has specific provisions limiting foreign ownership in public utilities which the Court is sworn to uphold
regardless of the experience of our neighboring countries.
In our jurisdiction, the Constitution expressly reserves the ownership and operation of public utilities to Filipino
citizens, or corporations or associations at least 60 percent of whose capital belongs to Filipinos. Following Dr.
Villegas's claim, the Philippines appears to be more liberal in allowing foreign investors to own 40 percent of public
utilities, unlike in other Asian countries whose governments own and operate such industries.
XI.
Prospective Application of Sanctions
In its Motion for Partial Reconsideration, the SEC sought to clarify the reckoning period of the application and
imposition of appropriate sanctions against PLDT if found violating Section 11, Article XII of the Constitution.
As discussed, the Court has directed the SEC to investigate and determine whether PLDT violated Section 11,
Article XII of the Constitution. Thus, there is no dispute that it is only after the SEC has determined PLDT's violation, if
any exists at the time of the commencement of the administrative case or investigation, that the SEC may impose the
statutory sanctions against PLDT. In other words, once the 28 June 2011 Decision becomes final, the SEC shall impose
the appropriate sanctions only if it finds after due hearing that, at the start of the administrative case or investigation,
there is an existing violation of Section 11, Article XII of the Constitution. Under prevailing jurisprudence, public utilities
that fail to comply with the nationality requirement under Section 11, Article XII and the FIA can cure their deficiencies
prior to the start of the administrative case or investigation. 61 CTAIHc
XII.
Final Word
The Constitution expressly declares as State policy the development of an economy "effectively
controlled" by Filipinos. Consistent with such State policy, the Constitution explicitly reserves the ownership and
operation of public utilities to Philippine nationals, who are defined in the Foreign Investments Act of 1991 as Filipino
citizens, or corporations or associations at least 60 percent of whose capital with voting rights belongs to Filipinos.
The FIA's implementing rules explain that "[f]or stocks to be deemed owned and held by Philippine citizens or Philippine
nationals, mere legal title is not enough to meet the required Filipino equity. Full beneficial ownership of the stocks,
coupled with appropriate voting rights is essential." In effect, the FIA clarifies, reiterates and confirms the
interpretation that the term "capital" in Section 11, Article XII of the 1987 Constitution refers to shares with voting
rights, as well as with full beneficial ownership. This is precisely because the right to vote in the election of
directors, coupled with full beneficial ownership of stocks, translates to effective control of a corporation.
Any other construction of the term "capital" in Section 11, Article XII of the Constitution contravenes the letter
and intent of the Constitution. Any other meaning of the term "capital" openly invites alien domination of economic
activities reserved exclusively to Philippine nationals. Therefore, respondents' interpretation will ultimately result in
handing over effective control of our national economy to foreigners in patent violation of the Constitution, making
Filipinos second-class citizens in their own country.
Filipinos have only to remind themselves of how this country was exploited under the Parity Amendment, which
gave Americans the same rights as Filipinos in the exploitation of natural resources, and in the ownership and control of
public utilities, in the Philippines. To do this the 1935 Constitution, which contained the same 60 percent Filipino
ownership and control requirement as the present 1987 Constitution, had to be amended to give Americans parity rights
with Filipinos. There was bitter opposition to the Parity Amendment 62and many Filipinos eagerly awaited its expiration.
In late 1968, PLDT was one of the American-controlled public utilities that became Filipino-controlled when the
controlling American stockholders divested in anticipation of the expiration of the Parity Amendment on 3 July
1974. 63 No economic suicide happened when control of public utilities and mining corporations passed to Filipinos'
hands upon expiration of the Parity Amendment. AHSEaD
Movants' interpretation of the term "capital" would bring us back to the same evils spawned by the Parity
Amendment, effectively giving foreigners parity rights with Filipinos, but this time even without any amendment
to the present Constitution. Worse, movants' interpretation opens up our national economy to effective control not
only by Americans but also by all foreigners, be they Indonesians, Malaysians or Chinese, even in the absence of
reciprocal treaty arrangements. At least the Parity Amendment, as implemented by the Laurel-Langley Agreement,
gave the capital-starved Filipinos theoretical parity — the same rights as Americans to exploit natural resources, and to
own and control public utilities, in the United States of America. Here, movants' interpretation would effectively mean
a unilateral opening up of our national economy to all foreigners, without any reciprocal arrangements. That would
mean that Indonesians, Malaysians and Chinese nationals could effectively control our mining companies and public
utilities while Filipinos, even if they have the capital, could not control similar corporations in these countries.
The 1935, 1973 and 1987 Constitutions have the same 60 percent Filipino ownership and control requirement
for public utilities like PLDT. Any deviation from this requirement necessitates an amendment to the Constitution as
exemplified by the Parity Amendment. This Court has no power to amend the Constitution for its power and duty is only
to faithfully apply and interpret the Constitution.
WHEREFORE, we DENY the motions for reconsideration WITH FINALITY. No further pleadings shall be
entertained. SO ORDERED.
||| (Heirs of Gamboa v. Teves, G.R. No. 176579 (Resolution), [October 9, 2012], 696 PHIL 276-485)

[G.R. No. 195580. April 21, 2014.]

NARRA NICKEL MINING AND DEVELOPMENT CORP.,TESORO MINING AND DEVELOPMENT,


INC.,and MCARTHUR MINING, INC., petitioners,vs.REDMONT CONSOLIDATED MINES
CORP., respondent.

DECISION

VELASCO, JR., J p:

Before this Court is a Petition for Review on Certiorari under Rule 45 filed by Narra Nickel and Mining Development
Corp. (Narra), Tesoro Mining and Development, Inc. (Tesoro), and McArthur Mining, Inc. (McArthur), which seeks to reverse
the October 1, 2010 Decision 1and the February 15, 2011 Resolution of the Court of Appeals (CA).
The Facts
Sometime in December 2006, respondent Redmont Consolidated Mines Corp. (Redmont),a domestic corporation
organized and existing under Philippine laws, took interest in mining and exploring certain areas of the province of Palawan.
After inquiring with the Department of Environment and Natural Resources (DENR),it learned that the areas where it wanted
to undertake exploration and mining activities where already covered by Mineral Production Sharing Agreement (MPSA)
applications of petitioners Narra, Tesoro and McArthur.
Petitioner McArthur, through its predecessor-in-interest Sara Marie Mining, Inc. (SMMI),filed an application for an
MPSA and Exploration Permit (EP) with the Mines and Geo-Sciences Bureau (MGB),Region IV-B, Office of the Department
of Environment and Natural Resources (DENR).Subsequently, SMMI was issued MPSA-AMA-IVB-153 covering an area of
over 1,782 hectares in Barangay Sumbiling, Municipality of Bataraza, Province of Palawan and EPA-IVB-44 which includes
an area of 3,720 hectares in Barangay Malatagao, Bataraza, Palawan. The MPSA and EP were then transferred to
Madridejos Mining Corporation (MMC) and, on November 6, 2006, assigned to petitioner McArthur. 2
Petitioner Narra acquired its MPSA from Alpha Resources and Development Corporation and Patricia Louise
Mining & Development Corporation (PLMDC) which previously filed an application for an MPSA with the MGB, Region IV-B,
DENR on January 6, 1992. Through the said application, the DENR issued MPSA-IV-1-12 covering an area of 3.277
hectares in barangays Calategas and San Isidro, Municipality of Narra, Palawan. Subsequently, PLMDC conveyed,
transferred and/or assigned its rights and interests over the MPSA application in favor of Narra.
Another MPSA application of SMMI was filed with the DENR Region IV-B, labeled as MPSA-AMA-IVB-154
(formerly EPA-IVB-47) over 3,402 hectares in Barangays Malinao and Princesa Urduja, Municipality of Narra, Province of
Palawan. SMMI subsequently conveyed, transferred and assigned its rights and interest over the said MPSA application to
Tesoro.
On January 2, 2007, Redmont filed before the Panel of Arbitrators (POA) of the DENR three (3) separate petitions
for the denial of petitioners' applications for MPSA designated as AMA-IVB-153, AMA-IVB-154 and MPSA IV-1-12. CSHEAI
In the petitions, Redmont alleged that at least 60% of the capital stock of McArthur, Tesoro and Narra are owned
and controlled by MBMI Resources, Inc. (MBMI),a 100% Canadian corporation. Redmont reasoned that since MBMI is a
considerable stockholder of petitioners, it was the driving force behind petitioners' filing of the MPSAs over the areas
covered by applications since it knows that it can only participate in mining activities through corporations which are deemed
Filipino citizens. Redmont argued that given that petitioners' capital stocks were mostly owned by MBMI, they were likewise
disqualified from engaging in mining activities through MPSAs, which are reserved only for Filipino citizens.
In their Answers, petitioners averred that they were qualified persons under Section 3 (aq) of Republic Act No. (RA)
7942 or thePhilippine Mining Act of 1995 which provided:
Sec. 3 Definition of Terms. — As used in and for purposes of this Act, the following terms,
whether in singular or plural, shall mean:
xxx xxx xxx
(aq) "Qualified person" means any citizen of the Philippines with capacity to contract, or a corporation,
partnership, association, or cooperative organized or authorized for the purpose of engaging in mining,
with technical and financial capability to undertake mineral resources development and duly registered
in accordance with law at least sixty per cent (60%) of the capital of which is owned by citizens of the
Philippines: Provided, That a legally organized foreign-owned corporation shall be deemed a qualified
person for purposes of granting an exploration permit, financial or technical assistance agreement or
mineral processing permit.
Additionally, they stated that their nationality as applicants is immaterial because they also applied for Financial or
Technical Assistance Agreements (FTAA) denominated as AFTA-IVB-09 for McArthur, AFTA-IVB-08 for Tesoro and AFTA-
IVB-07 for Narra, which are granted to foreign-owned corporations. Nevertheless, they claimed that the issue on
nationality should not be raised since McArthur, Tesoro and Narra are in fact Philippine Nationals as 60% of their
capital is owned by citizens of the Philippines. They asserted that though MBMI owns 40% of the shares of PLMC
(which owns 5,997 shares of Narra), 3 40% of the shares of MMC (which owns 5,997 shares of McArthur) 4 and 40% of the
shares of SLMC (which, in turn, owns 5,997 shares of Tesoro), 5 the shares of MBMI will not make it the owner of at least
60% of the capital stock of each of petitioners. They added that the best tool used in determining the nationality of a
corporation is the "control test," embodied in Sec. 3 of RA 7042 or the Foreign Investments Act of 1991. They also
claimed that the POA of DENR did not have jurisdiction over the issues in Redmont's petition since they are not enumerated
in Sec. 77 of RA 7942. Finally, they stressed that Redmont has no personality to sue them because it has no pending claim
or application over the areas applied for by petitioners.
On December 14, 2007, the POA issued a Resolution disqualifying petitioners from gaining MPSAs. It held:
[I]t is clearly established that respondents are not qualified applicants to engage in mining
activities. On the other hand, [Redmont] having filed its own applications for an EPA over the areas
earlier covered by the MPSA application of respondents may be considered if and when they are
qualified under the law. The violation of the requirements for the issuance and/or grant of permits over
mining areas is clearly established thus, there is reason to believe that the cancellation and/or revocation
of permits already issued under the premises is in order and open the areas covered to other qualified
applicants.
xxx xxx xxx
WHEREFORE, the Panel of Arbitrators finds the Respondents, McArthur Mining, Inc.,Tesoro
Mining and Development, Inc.,and Narra Nickel Mining and Development Corp. as, DISQUALIFIED for
being considered as Foreign Corporations. Their Mineral Production Sharing Agreement (MPSA) are
hereby ...DECLARED NULL AND VOID. 6
The POA considered petitioners as foreign corporations being "effectively controlled" by MBMI, a 100% Canadian
company and declared their MPSAs null and void. In the same Resolution, it gave due course to Redmont's EPAs.
Thereafter, on February 7, 2008, the POA issued an Order 7 denying the Motion for Reconsideration filed by petitioners.
Aggrieved by the Resolution and Order of the POA, McArthur and Tesoro filed a joint Notice of Appeal 8 and
Memorandum of Appeal 9with the Mines Adjudication Board (MAB) while Narra separately filed its Notice of Appeal 10 and
Memorandum of Appeal. 11
In their respective memorandum, petitioners emphasized that they are qualified persons under the law. Also,
through a letter, they informed the MAB that they had their individual MPSA applications converted to FTAAs. McArthur's
FTAA was denominated as AFTA-IVB-0912 on May 2007, while Tesoro's MPSA application was converted to AFTA-IVB-
08 13 on May 28, 2007, and Narra's FTAA was converted to AFTA-IVB-07 14 on March 30, 2006. DHESca
Pending the resolution of the appeal filed by petitioners with the MAB, Redmont filed a Complaint 15 with the
Securities and Exchange Commission (SEC), seeking the revocation of the certificates for registration of petitioners on the
ground that they are foreign-owned or controlled corporations engaged in mining in violation of Philippine laws. Thereafter,
Redmont filed on September 1, 2008 a Manifestation and Motion to Suspend Proceeding before the MAB praying for the
suspension of the proceedings on the appeals filed by McArthur, Tesoro and Narra.
Subsequently, on September 8, 2008, Redmont filed before the Regional Trial Court of Quezon City, Branch 92
(RTC) a Complaint 16 for injunction with application for issuance of a temporary restraining order (TRO) and/or writ of
preliminary injunction, docketed as Civil Case No. 08-63379. Redmont prayed for the deferral of the MAB proceedings
pending the resolution of the Complaint before the SEC.
But before the RTC can resolve Redmont's Complaint and applications for injunctive reliefs, the MAB issued an
Order on September 10, 2008, finding the appeal meritorious. It held:
WHEREFORE, in view of the foregoing, the Mines Adjudication Board hereby REVERSES and
SETS ASIDE the Resolution dated 14 December 2007 of the Panel of Arbitrators of Region IV-B
(MIMAROPA) in POA-DENR Case Nos. 2001-01, 2007-02 and 2007-03, and its Order dated 07 February
2008 denying the Motions for Reconsideration of the Appellants. The Petition filed by Redmont
Consolidated Mines Corporation on 02 January 2007 is hereby ordered DISMISSED. 17
Belatedly, on September 16, 2008, the RTC issued an Order 18 granting Redmont's application for a TRO and
setting the case for hearing the prayer for the issuance of a writ of preliminary injunction on September 19, 2008.
Meanwhile, on September 22, 2008, Redmont filed a Motion for Reconsideration 19 of the September 10, 2008
Order of the MAB. Subsequently, it filed a Supplemental Motion for Reconsideration 20 on September 29, 2008.
Before the MAB could resolve Redmont's Motion for Reconsideration and Supplemental Motion for
Reconsideration, Redmont filed before the RTC a Supplemental Complaint 21 in Civil Case No. 08-63379.
On October 6, 2008, the RTC issued an Order 22 granting the issuance of a writ of preliminary injunction enjoining
the MAB from finally disposing of the appeals of petitioners and from resolving Redmont's Motion for Reconsideration and
Supplement Motion for Reconsideration of the MAB's September 10, 2008 Resolution.
On July 1, 2009, however, the MAB issued a second Order denying Redmont's Motion for Reconsideration and
Supplemental Motion for Reconsideration and resolving the appeals filed by petitioners.
Hence, the petition for review filed by Redmont before the CA, assailing the Orders issued by the MAB. On October
1, 2010, the CA rendered a Decision, the dispositive of which reads:
WHEREFORE, the Petition is PARTIALLY GRANTED. The assailed Orders, dated September
10, 2008 and July 1, 2009 of the Mining Adjudication Board are reversed and set aside. The findings of
the Panel of Arbitrators of the Department of Environment and Natural Resources that respondents
McArthur, Tesoro and Narra are foreign corporations is upheld and, therefore, the rejection of their
applications for Mineral Product Sharing Agreement should be recommended to the Secretary of the
DENR.
With respect to the applications of respondents McArthur, Tesoro and Narra for Financial or
Technical Assistance Agreement (FTAA) or conversion of their MPSA applications to FTAA, the matter
for its rejection or approval is left for determination by the Secretary of the DENR and the President of the
Republic of the Philippines.
SO ORDERED. 23
In a Resolution dated February 15, 2011, the CA denied the Motion for Reconsideration filed by petitioners.
After a careful review of the records, the CA found that there was doubt as to the nationality of petitioners when it
realized that petitioners had a common major investor, MBMI, a corporation composed of 100% Canadians. Pursuant to the
first sentence of paragraph 7 of Department of Justice (DOJ) Opinion No. 020, Series of 2005, adopting the 1967 SEC
Rules which implemented the requirement of theConstitution and other laws pertaining to the exploitation of natural
resources, the CA used the "grandfather rule" to determine the nationality of petitioners. It provided:
Shares belonging to corporations or partnerships at least 60% of the capital of which is owned by
Filipino citizens shall be considered as of Philippine nationality, but if the percentage of Filipino
ownership in the corporation or partnership is less than 60%,only the number of shares
corresponding to such percentage shall be counted as of Philippine nationality.Thus, if 100,000
shares are registered in the name of a corporation or partnership at least 60% of the capital stock or
capital, respectively, of which belong to Filipino citizens, all of the shares shall be recorded as owned by
Filipinos. But if less than 60%,or say, 50% of the capital stock or capital of the corporation or partnership,
respectively, belongs to Filipino citizens, only 50,000 shares shall be recorded as belonging to
aliens. 24 (emphasis supplied)
In determining the nationality of petitioners, the CA looked into their corporate structures and their corresponding
common shareholders. Using the grandfather rule, the CA discovered that MBMI in effect owned majority of the common
stocks of the petitioners as well as at least 60% equity interest of other majority shareholders of petitioners through joint
venture agreements. The CA found that through a "web of corporate layering, it is clear that one common controlling
investor in all mining corporations involved ...is MBMI." 25Thus, it concluded that petitioners McArthur, Tesoro and Narra
are also in partnership with, or privies-in-interest of, MBMI.
Furthermore, the CA viewed the conversion of the MPSA applications of petitioners into FTAA applications
suspicious in nature and, as a consequence, it recommended the rejection of petitioners' MPSA applications by the
Secretary of the DENR.
With regard to the settlement of disputes over rights to mining areas, the CA pointed out that the POA has
jurisdiction over them and that it also has the power to determine the of nationality of petitioners as a prerequisite of
the Constitution prior the conferring of rights to "co-production, joint venture or production-sharing agreements" of the state
to mining rights. However, it also stated that the POA's jurisdiction is limited only to the resolution of the dispute and not on
the approval or rejection of the MPSAs. It stipulated that only the Secretary of the DENR is vested with the power to
approve or reject applications for MPSA.
Finally, the CA upheld the findings of the POA in its December 14, 2007 Resolution which considered petitioners
McArthur, Tesoro and Narra as foreign corporations. Nevertheless, the CA determined that the POA's declaration that the
MPSAs of McArthur, Tesoro and Narra are void is highly improper. cCaEDA
While the petition was pending with the CA, Redmont filed with the Office of the President (OP) a petition dated
May 7, 2010 seeking the cancellation of petitioners' FTAAs. The OP rendered a Decision 26 on April 6, 2011, wherein it
canceled and revoked petitioners' FTAAs for violating and circumventing the "Constitution . . .[,] the Small Scale Mining Law
and Environmental Compliance Certificate as well as Sections 3 and 8 of the Foreign Investment Act and E.O. 584." 27 The
OP, in affirming the cancellation of the issued FTAAs, agreed with Redmont stating that petitioners committed violations
against the abovementioned laws and failed to submit evidence to negate them. The Decision further quoted the December
14, 2007 Order of the POA focusing on the alleged misrepresentation and claims made by petitioners of being domestic or
Filipino corporations and the admitted continued mining operation of PMDC using their locally secured Small Scale Mining
Permit inside the area earlier applied for an MPSA application which was eventually transferred to Narra. It also agreed with
the POA's estimation that the filing of the FTAA applications by petitioners is a clear admission that they are "not capable of
conducting a large scale mining operation and that they need the financial and technical assistance of a foreign entity in
their operation, that is why they sought the participation of MBMI Resources, Inc." 28 The Decision further quoted:
The filing of the FTAA application on June 15, 2007, during the pendency of the case only
demonstrate the violations and lack of qualification of the respondent corporations to engage in mining.
The filing of the FTAA application conversion which is allowed foreign corporation of the earlier MPSA is
an admission that indeed the respondent is not Filipino but rather of foreign nationality who is disqualified
under the laws. Corporate documents of MBMI Resources, Inc. furnished its stockholders in their head
office in Canada suggest that they are conducting operation only through their local counterparts. 29
The Motion for Reconsideration of the Decision was further denied by the OP in a Resolution 30 dated July 6, 2011.
Petitioners then filed a Petition for Review on Certiorari of the OP's Decision and Resolution with the CA, docketed as CA-
G.R. SP No. 120409. In the CA Decision dated February 29, 2012, the CA affirmed the Decision and Resolution of the OP.
Thereafter, petitioners appealed the same CA decision to this Court which is now pending with a different division.
Thus, the instant petition for review against the October 1, 2010 Decision of the CA. Petitioners put forth the
following errors of the CA:
I.
The Court of Appeals erred when it did not dismiss the case for mootness despite the fact that the
subject matter of the controversy, the MPSA Applications, have already been converted into FTAA
applications and that the same have already been granted.
II.
The Court of Appeals erred when it did not dismiss the case for lack of jurisdiction considering that the
Panel of Arbitrators has no jurisdiction to determine the nationality of Narra, Tesoro and McArthur.
III.
The Court of Appeals erred when it did not dismiss the case on account of Redmont's willful forum
shopping.
IV.
The Court of Appeals' ruling that Narra, Tesoro and McArthur are foreign corporations based on the
"Grandfather Rule" is contrary to law, particularly the express mandate of the Foreign Investments Act
of 1991, as amended, and the FIA Rules.
V.
The Court of Appeals erred when it applied the exceptions to the res inter alios acta rule.
VI.
The Court of Appeals erred when it concluded that the conversion of the MPSA Applications into FTAA
Applications were of "suspicious nature" as the same is based on mere conjectures and surmises
without any shred of evidence to show the same. 31
We find the petition to be without merit.
This case not moot and academic
The claim of petitioners that the CA erred in not rendering the instant case as moot is without merit.
Basically, a case is said to be moot and/or academic when it "ceases to present a justiciable controversy by virtue
of supervening events, so that a declaration thereon would be of no practical use or value." 32 Thus, the courts "generally
decline jurisdiction over the case or dismiss it on the ground of mootness." 33
The "mootness" principle, however, does accept certain exceptions and the mere raising of an issue of "mootness"
will not deter the courts from trying a case when there is a valid reason to do so. In David v. Macapagal-Arroyo (David),the
Court provided four instances where courts can decide an otherwise moot case, thus:
1.) There is a grave violation of the Constitution;
2.) The exceptional character of the situation and paramount public interest is involved;
3.) When constitutional issue raised requires formulation of controlling principles to guide the bench, the
bar, and the public; and caTIDE
4.) The case is capable of repetition yet evading review. 34
All of the exceptions stated above are present in the instant case. We of this Court note that a grave violation of
the Constitution, specifically Section 2 of Article XII, is being committed by a foreign corporation right under our country's
nose through a myriad of corporate layering under different, allegedly, Filipino corporations. The intricate corporate layering
utilized by the Canadian company, MBMI, is of exceptional character and involves paramount public interest since it
undeniably affects the exploitation of our Country's natural resources. The corresponding actions of petitioners during the
lifetime and existence of the instant case raise questions as what principle is to be applied to cases with similar issues. No
definite ruling on such principle has been pronounced by the Court; hence, the disposition of the issues or errors in the
instant case will serve as a guide "to the bench, the bar and the public." 35 Finally, the instant case is capable of repetition
yet evading review, since the Canadian company, MBMI, can keep on utilizing dummy Filipino corporations through various
schemes of corporate layering and conversion of applications to skirt the constitutional prohibition against foreign mining in
Philippine soil.
Conversion of MPSA applications to FTAA applications
We shall discuss the first error in conjunction with the sixth error presented by petitioners since both involve the
conversion of MPSA applications to FTAA applications. Petitioners propound that the CA erred in ruling against them since
the questioned MPSA applications were already converted into FTAA applications; thus, the issue on the prohibition relating
to MPSA applications of foreign mining corporations is academic. Also, petitioners would want us to correct the CA's finding
which deemed the aforementioned conversions of applications as suspicious in nature, since it is based on mere
conjectures and surmises and not supported with evidence.
We disagree.
The CA's analysis of the actions of petitioners after the case was filed against them by respondent is on point. The
changing of applications by petitioners from one type to another just because a case was filed against them, in truth, would
raise not a few sceptics' eyebrows. What is the reason for such conversion? Did the said conversion not stem from the case
challenging their citizenship and to have the case dismissed against them for being "moot"? It is quite obvious that it is
petitioners' strategy to have the case dismissed against them for being "moot."
Consider the history of this case and how petitioners responded to every action done by the court or appropriate
government agency: on January 2, 2007, Redmont filed three separate petitions for denial of the MPSA applications of
petitioners before the POA. On June 15, 2007, petitioners filed a conversion of their MPSA applications to FTAAs. The
POA, in its December 14, 2007 Resolution, observed this suspect change of applications while the case was pending before
it and held:
The filing of the Financial or Technical Assistance Agreement application is a clear admission
that the respondents are not capable of conducting a large scale mining operation and that they need the
financial and technical assistance of a foreign entity in their operation that is why they sought the
participation of MBMI Resources, Inc. The participation of MBMI in the corporation only proves the fact
that it is the Canadian company that will provide the finances and the resources to operate the mining
areas for the greater benefit and interest of the same and not the Filipino stockholders who only have a
less substantial financial stake in the corporation.
xxx xxx xxx
...The filing of the FTAA application on June 15, 2007,during the pendency of the case only
demonstrate the violations and lack of qualification of the respondent corporations to engage in
mining. The filing of the FTAA application conversion which is allowed foreign corporation of the
earlier MPSA is an admission that indeed the respondent is not Filipino but rather of foreign
nationality who is disqualified under the laws.Corporate documents of MBMI Resources, Inc.
furnished its stockholders in their head office in Canada suggest that they are conducting operation only
through their local counterparts. 36
On October 1, 2010, the CA rendered a Decision which partially granted the petition, reversing and setting aside the
September 10, 2008 and July 1, 2009 Orders of the MAB. In the said Decision, the CA upheld the findings of the POA of the
DENR that the herein petitioners are in fact foreign corporations thus a recommendation of the rejection of their MPSA
applications were recommended to the Secretary of the DENR. With respect to the FTAA applications or conversion of the
MPSA applications to FTAAs, the CA deferred the matter for the determination of the Secretary of the DENR and the
President of the Republic of the Philippines. 37
In their Motion for Reconsideration dated October 26, 2010, petitioners prayed for the dismissal of the petition
asserting that on April 5, 2010, then President Gloria Macapagal-Arroyo signed and issued in their favor FTAA No. 05-
2010-IVB, which rendered the petition moot and academic. However, the CA, in a Resolution dated February 15, 2011
denied their motion for being a mere "rehash of their claims and defenses." 38 Standing firm on its Decision, the CA
affirmed the ruling that petitioners are, in fact, foreign corporations. On April 5, 2011, petitioners elevated the case to us via
a Petition for Review on Certiorari under Rule 45, questioning the Decision of the CA. Interestingly, the OP rendered a
Decision dated April 6, 2011, a day after this petition for review was filed, cancelling and revoking the FTAAs, quoting the
Order of the POA and stating that petitioners are foreign corporations since they needed the financial strength of MBMI, Inc.
in order to conduct large scale mining operations. The OP Decision also based the cancellation on the misrepresentation of
facts and the violation of the "Small Scale Mining Law and Environmental Compliance Certificate as well as Sections 3 and
8 of the Foreign Investment Act and E.O. 584." 39 On July 6, 2011, the OP issued a Resolution, denying the Motion for
Reconsideration filed by the petitioners.
Respondent Redmont, in its Comment dated October 10, 2011, made known to the Court the fact of the OP's
Decision and Resolution. In their Reply, petitioners chose to ignore the OP Decision and continued to reuse their old
arguments claiming that they were granted FTAAs and, thus, the case was moot. Petitioners filed a Manifestation and
Submission dated October 19, 2012, 40 wherein they asserted that the present petition is moot since, in a remarkable turn
of events, MBMI was able to sell/assign all its shares/interest in the "holding companies" to DMCI Mining Corporation
(DMCI),a Filipino corporation and, in effect, making their respective corporations fully-Filipino owned.
Again, it is quite evident that petitioners have been trying to have this case dismissed for being "moot." Their final
act, wherein MBMI was able to allegedly sell/assign all its shares and interest in the petitioner "holding companies" to
DMCI, only proves that they were in factnot Filipino corporations from the start. The recent divesting of interest by MBMI will
not change the stand of this Court with respect to the nationality of petitioners prior the suspicious change in their corporate
structures. The new documents filed by petitioners are factual evidence that this Court has no power to verify.
The only thing clear and proved in this Court is the fact that the OP declared that petitioner corporations have
violated several mining laws and made misrepresentations and falsehood in their applications for FTAA which lead to the
revocation of the said FTAAs, demonstrating that petitioners are not beyond going against or around the law using shifty
actions and strategies. Thus, in this instance, we can say that their claim of mootness is moot in itself because their defense
of conversion of MPSAs to FTAAs has been discredited by the OP Decision.
Grandfather test
The main issue in this case is centered on the issue of petitioners' nationality, whether Filipino or foreign. In their
previous petitions, they had been adamant in insisting that they were Filipino corporations, until they submitted their
Manifestation and Submission dated October 19, 2012 where they stated the alleged change of corporate ownership to
reflect their Filipino ownership. Thus, there is a need to determine the nationality of petitioner corporations.
Basically, there are two acknowledged tests in determining the nationality of a corporation: the control test and the
grandfather rule. Paragraph 7 of DOJ Opinion No. 020, Series of 2005, adopting the 1967 SEC Rules which implemented
the requirement of the Constitutionand other laws pertaining to the controlling interests in enterprises engaged in the
exploitation of natural resources owned by Filipino citizens, provides:
Shares belonging to corporations or partnerships at least 60% of the capital of which is owned by
Filipino citizens shall be considered as of Philippine nationality, but if the percentage of Filipino ownership
in the corporation or partnership is less than 60%,only the number of shares corresponding to such
percentage shall be counted as of Philippine nationality. Thus, if 100,000 shares are registered in the
name of a corporation or partnership at least 60% of the capital stock or capital, respectively, of which
belong to Filipino citizens, all of the shares shall be recorded as owned by Filipinos. But if less than
60%,or say, 50% of the capital stock or capital of the corporation or partnership, respectively, belongs to
Filipino citizens, only 50,000 shares shall be counted as owned by Filipinos and the other 50,000 shall be
recorded as belonging to aliens.
The first part of paragraph 7, DOJ Opinion No. 020, stating "shares belonging to corporations or partnerships at
least 60% of the capital of which is owned by Filipino citizens shall be considered as of Philippine nationality," pertains to
the control test or the liberal rule. On the other hand, the second part of the DOJ Opinion which provides, "if the percentage
of the Filipino ownership in the corporation or partnership is less than 60%, only the number of shares corresponding to
such percentage shall be counted as Philippine nationality," pertains to the stricter, more stringent grandfather rule. TaEIcS
Prior to this recent change of events, petitioners were constant in advocating the application of the "control test"
under RA 7042, as amended by RA 8179, otherwise known as the Foreign Investments Act (FIA), rather than using the
stricter grandfather rule. The pertinent provision under Sec. 3 of the FIA provides:
SECTION 3. Definitions. — As used in this Act:
a.) The term Philippine national shall mean a citizen of the Philippines; or a domestic partnership
or association wholly owned by the citizens of the Philippines; a corporation organized under the laws of
the Philippines of which at least sixty percent (60%) of the capital stock outstanding and entitled to vote is
wholly owned by Filipinos or a trustee of funds for pension or other employee retirement or separation
benefits, where the trustee is a Philippine national and at least sixty percent (60%) of the fund will accrue
to the benefit of Philippine nationals: Provided, That were a corporation and its non-Filipino
stockholders own stocks in a Securities and Exchange Commission (SEC) registered enterprise,
at least sixty percent (60%) of the capital stock outstanding and entitled to vote of each of both
corporations must be owned and held by citizens of the Philippines and at least sixty percent
(60%) of the members of the Board of Directors, in order that the corporation shall be considered
a Philippine national.(emphasis supplied)
The grandfather rule, petitioners reasoned, has no leg to stand on in the instant case since the definition of a
"Philippine National" under Sec. 3 of the FIA does not provide for it. They further claim that the grandfather rule "has been
abandoned and is no longer the applicable rule." 41 They also opined that the last portion of Sec. 3 of the FIA admits the
application of a "corporate layering" scheme of corporations. Petitioners claim that the clear and unambiguous wordings of
the statute preclude the court from construing it and prevent the court's use of discretion in applying the law. They said that
the plain, literal meaning of the statute meant the application of the control test is obligatory.
We disagree. "Corporate layering" is admittedly allowed by the FIA; but if it is used to circumvent
the Constitution and pertinent laws, then it becomes illegal. Further, the pronouncement of petitioners that the grandfather
rule has already been abandoned must be discredited for lack of basis.
Art. XII, Sec. 2 of the Constitution provides:
Sec. 2. All lands of the public domain, waters, minerals, coal, petroleum and other mineral oils,
all forces of potential energy, fisheries, forests or timber, wildlife, flora and fauna, and other natural
resources are owned by the State. With the exception of agricultural lands, all other natural resources
shall not be alienated. The exploration, development, and utilization of natural resources shall be under
the full control and supervision of the State. The State may directly undertake such activities, or it
may enter into co-production, joint venture or production-sharing agreements with Filipino
citizens, or corporations or associations at least sixty per centum of whose capital is owned by
such citizens.Such agreements may be for a period not exceeding twenty-five years, renewable for not
more than twenty-five years, and under such terms and conditions as may be provided by law.
xxx xxx xxx
The President may enter into agreements with Foreign-owned corporations involving either
technical or financial assistance for large-scale exploration, development, and utilization of minerals,
petroleum, and other mineral oils according to the general terms and conditions provided by law, based
on real contributions to the economic growth and general welfare of the country. In such agreements, the
State shall promote the development and use of local scientific and technical resources. (emphasis
supplied)
The emphasized portion of Sec. 2 which focuses on the State entering into different types of agreements for the
exploration, development, and utilization of natural resources with entities who are deemed Filipino due to 60 percent
ownership of capital is pertinent to this case, since the issues are centered on the utilization of our country's natural
resources or specifically, mining. Thus, there is a need to ascertain the nationality of petitioners since, as the Constitution so
provides, such agreements are only allowed corporations or associations "at least 60 percent of such capital is owned by
such citizens." The deliberations in the Records of the 1986 Constitutional Commission shed light on how a citizenship of a
corporation will be determined:
Mr. BENNAGEN:
Did I hear right that the Chairman's interpretation of an independent national economy is freedom from
undue foreign control? What is the meaning of undue foreign control?
MR. VILLEGAS:
Undue foreign control is foreign control which sacrifices national sovereignty and the welfare of the
Filipino in the economic sphere.
MR. BENNAGEN:
Why does it have to be qualified still with the word "undue"? Why not simply freedom from foreign
control? I think that is the meaning of independence, because as phrased, it still allows for
foreign control.
MR. VILLEGAS:
It will now depend on the interpretation because if, for example, we retain the 60/40 possibility in the
cultivation of natural resources, 40 percent involves some control; not total control, but some
control.
MR. BENNAGEN:
In any case, I think in due time we will propose some amendments.
MR. VILLEGAS:
Yes. But we will be open to improvement of the phraseology.
Mr. BENNAGEN:
Yes.
Thank you, Mr. Vice-President.
xxx xxx xxx
MR. NOLLEDO:
In Sections 3, 9 and 15, the Committee stated local or Filipino equity and foreign equity; namely, 60-40 in
Section 3, 60-40 in Section 9, and 2/3-1/3 in Section 15.
MR. VILLEGAS:
That is right.
MR. NOLLEDO:
In teaching law, we are always faced with the question: 'Where do we base the equity requirement, is it
on the authorized capital stock, on the subscribed capital stock, or on the paid-up capital stock of
a corporation'? Will the Committee please enlighten me on this?
MR. VILLEGAS:
We have just had a long discussion with the members of the team from the UP Law Center who provided
us with a draft. The phrase that is contained here which we adopted from the UP draft is '60
percent of the voting stock.'
MR. NOLLEDO:
That must be based on the subscribed capital stock, because unless declared delinquent, unpaid capital
stock shall be entitled to vote.
MR. VILLEGAS:
That is right.
MR. NOLLEDO:
Thank you.
With respect to an investment by one corporation in another corporation, say, a corporation with
60-40 percent equity invests in another corporation which is permitted by the Corporation
Code, does the Committee adopt the grandfather rule?
MR. VILLEGAS:
Yes, that is the understanding of the Committee.
MR. NOLLEDO:
Therefore, we need additional Filipino capital?
MR. VILLEGAS:
Yes.42 (emphasis supplied)
It is apparent that it is the intention of the framers of the Constitution to apply the grandfather rule in cases where
corporate layering is present. Elementary in statutory construction is when there is conflict between the Constitution and a
statute, the Constitution will prevail. In this instance, specifically pertaining to the provisions under Art. XII of
the Constitution on National Economy and Patrimony, Sec. 3 of the FIA will have no place of application. As decreed by the
honorable framers of our Constitution, the grandfather rule prevails and must be applied.
Likewise, paragraph 7, DOJ Opinion No. 020, Series of 2005 provides:
The above-quoted SEC Rules provide for the manner of calculating the Filipino interest in a
corporation for purposes, among others, of determining compliance with nationality requirements (the
'Investee Corporation').Such manner of computation is necessary since the shares in the Investee
Corporation may be owned both by individual stockholders ('Investing Individuals') and by corporations
and partnerships ('Investing Corporation').The said rules thus provide for the determination of nationality
depending on the ownership of the Investee Corporation and, in certain instances, the Investing
Corporation.
Under the above-quoted SEC Rules, there are two cases in determining the nationality of the
Investee Corporation. The first case is the 'liberal rule',later coined by the SEC as the Control Test in its
30 May 1990 Opinion, and pertains to the portion in said Paragraph 7 of the 1967 SEC Rules which
states, '(s)hares belonging to corporations or partnerships at least 60% of the capital of which is owned
by Filipino citizens shall be considered as of Philippine nationality.' Under the liberal Control Test, there is
no need to further trace the ownership of the 60% (or more) Filipino stockholdings of the Investing
Corporation since a corporation which is at least 60% Filipino-owned is considered as Filipino.
The second case is the Strict Rule or the Grandfather Rule Proper and pertains to the portion in
said Paragraph 7 of the 1967 SEC Rules which states, "but if the percentage of Filipino ownership in the
corporation or partnership is less than 60%,only the number of shares corresponding to such percentage
shall be counted as of Philippine nationality." Under the Strict Rule or Grandfather Rule Proper, the
combined totals in the Investing Corporation and the Investee Corporation must be traced
(i.e.,"grandfathered") to determine the total percentage of Filipino ownership.
Moreover, the ultimate Filipino ownership of the shares must first be traced to the level of the
Investing Corporation and added to the shares directly owned in the Investee Corporation ....
xxx xxx xxx
In other words, based on the said SEC Rule and DOJ Opinion, the Grandfather Rule or the
second part of the SEC Rule applies only when the 60-40 Filipino-foreign equity ownership is in
doubt (i.e.,in cases where the joint venture corporation with Filipino and foreign stockholders with less
than 60% Filipino stockholdings [or 59%] invests in other joint venture corporation which is either 60-40%
Filipino-alien or the 59% less Filipino).Stated differently, where the 60-40 Filipino-foreign equity
ownership is not in doubt, the Grandfather Rule will not apply.(emphasis supplied) CTacSE
After a scrutiny of the evidence extant on record, the Court finds that this case calls for the application of the
grandfather rule since, as ruled by the POA and affirmed by the OP, doubt prevails and persists in the corporate ownership
of petitioners. Also, as found by the CA, doubt is present in the 60-40 Filipino equity ownership of petitioners Narra,
McArthur and Tesoro, since their common investor, the 100% Canadian corporation — MBMI, funded them. However,
petitioners also claim that there is "doubt" only when the stockholdings of Filipinos are less than 60%. 43
The assertion of petitioners that "doubt" only exists when the stockholdings are less than 60% fails to convince this
Court. DOJOpinion No. 20, which petitioners quoted in their petition, only made an example of an instance where "doubt" as
to the ownership of the corporation exists. It would be ludicrous to limit the application of the said word only to the instances
where the stockholdings of non-Filipino stockholders are more than 40% of the total stockholdings in a corporation. The
corporations interested in circumventing our laws would clearly strive to have "60% Filipino Ownership" at face value. It
would be senseless for these applying corporations to state in their respective articles of incorporation that they have less
than 60% Filipino stockholders since the applications will be denied instantly. Thus, various corporate schemes and
layerings are utilized to circumvent the application of the Constitution.
Obviously, the instant case presents a situation which exhibits a scheme employed by stockholders to circumvent
the law, creating a cloud of doubt in the Court's mind. To determine, therefore, the actual participation, direct or indirect, of
MBMI, the grandfather rule must be used.
McArthur Mining, Inc.
To establish the actual ownership, interest or participation of MBMI in each of petitioners' corporate structure, they
have to be "grandfathered."
As previously discussed, McArthur acquired its MPSA application from MMC, which acquired its application from
SMMI. McArthur has a capital stock of ten million pesos (PhP10,000,000) divided into 10,000 common shares at one
thousand pesos (PhP1,000) per share, subscribed to by the following: 44
Name Nationality Number of Amount Amount Paid
Shares Subscribed

Madridejos Mining Filipino 5,997 PhP5,997,000.00 PhP825,000.00


Corporation
MBMI Resources, Canadian 3,998 PhP3,998,000.00 PhP1,878,174.60
Inc.
Lauro L. Salazar Filipino 1 PhP1,000.00 PhP1,000.00
Fernando B. Filipino 1 PhP1,000.00 PhP1,000.00
Esguerra
Manuel A. Agcaoili Filipino 1 PhP1,000.00 PhP1,000.00
Michael T. Mason American 1 PhP1,000.00 PhP1,000.00
Kenneth Cawkell Canadian 1 PhP1,000.00 PhP1,000.00
––––––– ––––––––––––– ––––––––––––––
Total 10,000 PhP10,000,000.00 PhP2,708,174.60
====== ============ ============
(emphasis supplied)
Interestingly, looking at the corporate structure of MMC, we take note that it has a similar structure and
composition as McArthur. In fact, it would seem that MBMI is also a major investor and "controls" 45 MBMI and also,
similar nominal shareholders were present,i.e.,Fernando B. Esguerra (Esguerra),Lauro L. Salazar (Salazar),Michael T.
Mason (Mason) and Kenneth Cawkell (Cawkell):
Madridejos Mining Corporation
Name Nationality Number of Amount Amount Paid
Shares Subscribed

Olympic Mines & Filipino 6,663 PhP6,663,000.00 PhP0


Development Corp.
MBMI Resources, Canadian 3,331 PhP3,331,000.00 PhP2,803,900.00
Inc.
Amanti Limson Filipino 1 PhP1,000.00 PhP1,000.00
Fernando B. Filipino 1 PhP1,000.00 PhP1,000.00
Esguerra
Lauro Salazar Filipino 1 PhP1,000.00 PhP1,000.00
Emmanuel G. Filipino 1 PhP1,000.00 PhP1,000.00
Hernando
Michael T. Mason American 1 PhP1,000.00 PhP1,000.00
Kenneth Cawkell Canadian 1 PhP1,000.00 PhP1,000.00
––––––– –––––––––––––––– ––––––––––––––––
Total 10,000 PhP10,000,000.00 PhP2,809,900.00
====== ============== ==============
(emphasis supplied)
Noticeably, Olympic Mines & Development Corporation (Olympic) did not pay any amount with respect to the
number of shares they subscribed to in the corporation, which is quite absurd since Olympic is the major stockholder in
MMC. MBMI's 2006 Annual Report sheds light on why Olympic failed to pay any amount with respect to the number of
shares it subscribed to. It states that Olympic entered into joint venture agreements with several Philippine companies,
wherein it holds directly and indirectly a 60% effective equity interest in the Olympic Properties. 46 Quoting the said Annual
report:
On September 9, 2004, the Company and Olympic Mines & Development Corporation ("Olympic")
entered into a series of agreements including a Property Purchase and Development Agreement (the
Transaction Documents) with respect to three nickel laterite properties in Palawan, Philippines (the
"Olympic Properties").The Transaction Documents effectively establish a joint venture between
the Company and Olympic for purposes of developing the Olympic Properties. The Company
holds directly and indirectly an initial 60% interest in the joint venture. Under certain
circumstances and upon achieving certain milestones, the Company may earn up to a 100%
interest,subject to a 2.5% net revenue royalty. 47 (emphasis supplied)
Thus, as demonstrated in this first corporation, McArthur, when it is "grandfathered," company layering was utilized
by MBMI to gain control over McArthur. It is apparent that MBMI has more than 60% or more equity interest in McArthur,
making the latter a foreign corporation.
Tesoro Mining and Development, Inc.
Tesoro, which acquired its MPSA application from SMMI, has a capital stock of ten million pesos (PhP10,000,000)
divided into ten thousand (10,000) common shares at PhP1,000 per share, as demonstrated below:
Name Nationality Number of Amount Amount Paid
Shares Subscribed

Sara Marie Filipino 5,997 PhP5,997,000.00 PhP825,000.00


Mining, Inc.
MBMI Canadian 3,998 PhP3,998,000.00 PhP1,878,174.60
Resources, Inc.
Lauro L. Salazar Filipino 1 PhP1,000.00 PhP1,000.00
Fernando B. Filipino 1 PhP1,000.00 PhP1,000.00
Esguerra
Manuel A. Filipino 1 PhP1,000.00 PhP1,000.00
Agcaoili
Michael T. Mason American 1 PhP1,000.00 PhP1,000.00
Kenneth Cawkell Canadian 1 PhP1,000.00 PhP1,000.00
–––––– –––––––––––––––– –––––––––––––––
Total 10,000 PhP10,000,000.00 PhP2,708,174.60
===== ============== =============
(emphasis supplied)
Except for the name "Sara Marie Mining, Inc.," the table above shows exactly the same figures as the corporate
structure of petitioner McArthur, down to the last centavo. All the other shareholders are the same: MBMI, Salazar,
Esguerra, Agcaoili, Mason and Cawkell. The figures under "Nationality," "Number of Shares," "Amount Subscribed," and
"Amount Paid" are exactly the same. Delving deeper, we scrutinize SMMI's corporate structure:
Name Nationality Number of Amount Amount Paid
Shares Subscribed

Olympic Mines & Filipino 6,663 PhP6,663,000.00 PhP0


Development Corp.
MBMI Resources, Canadian 3,331 PhP3,331,000.00 PhP2,794,000.00
Inc.
Amanti Limson Filipino 1 PhP1,000.00 PhP1,000.00
Fernando B. Filipino 1 PhP1,000.00 PhP1,000.00
Esguerra
Lauro Salazar Filipino 1 PhP1,000.00 PhP1,000.00
Emmanuel G. Filipino 1 PhP1,000.00 PhP1,000.00
Hernando
Michael T. Mason American 1 PhP1,000.00 PhP1,000.00
Kenneth Cawkell Canadian 1 PhP1,000.00 PhP1,000.00
–––––– ––––––––––––– ––––––––––––––
Total 10,000 PhP10,000,000.00 PhP2,809,900.00
====== ============= ============
(emphasis supplied)
After subsequently studying SMMI's corporate structure, it is not farfetched for us to spot the glaring similarity
between SMMI and MMC's corporate structure. Again, the presence of identical stockholders, namely: Olympic, MBMI,
Amanti Limson (Limson),Esguerra, Salazar, Hernando, Mason and Cawkell. The figures under the headings "Nationality,"
"Number of Shares," "Amount Subscribed," and "Amount Paid" are exactly the same except for the amount paid by MBMI
which now reflects the amount of two million seven hundred ninety four thousand pesos (PhP2,794,000).Oddly, the total
value of the amount paid is two million eight hundred nine thousand nine hundred pesos (PhP2,809,900).
Accordingly, after "grandfathering" petitioner Tesoro and factoring in Olympic's participation in SMMI's corporate
structure, it is clear that MBMI is in control of Tesoro and owns 60% or more equity interest in Tesoro. This makes petitioner
Tesoro a non-Filipino corporation and, thus, disqualifies it to participate in the exploitation, utilization and development of
our natural resources.
Narra Nickel Mining and Development Corporation
Moving on to the last petitioner, Narra, which is the transferee and assignee of PLMDC's MPSA application, whose
corporate structure's arrangement is similar to that of the first two petitioners discussed. The capital stock of Narra is ten
million pesos (PhP10,000,000),which is divided into ten thousand common shares (10,000) at one thousand pesos
(PhP1,000) per share, shown as follows:ACHEaI
Name Nationality Number of Amount Amount Paid
Shares Subscribed

Patricia Louise Filipino 5,997 PhP5,997,000.00 PhP1,677,000.00


Mining &
Development
Corp.
MBMI Canadian 3,998 PhP3,996,000.00 PhP1,116,000.00
Resources, Inc.
Higinio C. Filipino 1 PhP1,000.00 PhP1,000.00
Mendoza, Jr.
Henry E. Filipino 1 PhP1,000.00 PhP1,000.00
Fernandez
Manuel A. Filipino 1 PhP1,000.00 PhP1,000.00
Agcaoili
Ma. Elena A. Filipino 1 PhP1,000.00 PhP1,000.00
Bocalan
Bayani H. Agabin Filipino 1 PhP1,000.00 PhP1,000.00
Robert L. American 1 PhP1,000.00 PhP1,000.00
McCurdy
Kenneth Cawkell Canadian 1 PhP1,000.00 PhP1,000.00
––––––– ––––––––––––– ––––––––––––––
Total 10,000 PhP10,000,000.00 PhP2,800,000.00
====== ============= =============
Again, MBMI, along with other nominal stockholders, i.e.,Mason, Agcaoili and Esguerra, is present in this corporate
structure.
Patricia Louise Mining & Development Corporation
Using the grandfather method, we further look and examine PLMDC's corporate structure:
Name Nationality Number of Amount Amount Paid
Shares Subscribed

Palawan Alpha Filipino 6,596 PhP6,596,000.00 PhP0


South Resources
Development
Corporation
MBMI Resources, Canadian 3,396 PhP3,396,000.00 PhP2,796,000.00
Inc.
Higinio C. Filipino 1 PhP1,000.00 PhP1,000.00
Mendoza, Jr.
Fernando B. Filipino 1 PhP1,000.00 PhP1,000.00
Esguerra
Henry E. Filipino 1 PhP1,000.00 PhP1,000.00
Fernandez
Lauro L. Salazar Filipino 1 PhP1,000.00 PhP1,000.00
Manuel A. Agcaoili Filipino 1 PhP1,000.00 PhP1,000.00
Bayani H. Agabin Filipino 1 PhP1,000.00 PhP1,000.00
Michael T. Mason American 1 PhP1,000.00 PhP1,000.00
Kenneth Cawkell Canadian 1 PhP1,000.00 PhP1,000.00
––––––– ––––––––––––– ––––––––––––––
Total 10,000 PhP10,000,000.00 PhP2,708,174.60
====== ============= ============
(emphasis supplied)
Yet again, the usual players in petitioners' corporate structures are present. Similarly, the amount of money paid by
the 2nd tier majority stock holder, in this case, Palawan Alpha South Resources and Development Corp. (PASRDC),is zero.
Studying MBMI's Summary of Significant Accounting Policies dated October 31, 2005 explains the reason behind
the intricate corporate layering that MBMI immersed itself in:
JOINT VENTURES The Company's ownership interests in various mining ventures engaged in
the acquisition, exploration and development of mineral properties in
the Philippines is described as follows:
(a) Olympic Group
The Philippine companies holding the Olympic Property, and the ownership and interests therein, are
as follows:
Olympic-Philippines (the "Olympic Group"
Sara Marie Mining Properties Ltd. ("Sara Marie") — 33.3%
Tesoro Mining & Development, Inc. (Tesoro) — 60.0%
Pursuant to the Olympic joint venture agreement the Company holds directly and indirectly an
effective equity interest in the Olympic Property of 60.0%.Pursuant to a shareholders' agreement,
the Company exercises joint control over the companies in the Olympic Group.
(b) Alpha Group
The Philippine companies holding the Alpha Property, and the ownership interests therein, are as
follows:
Alpha-Philippines (the "Alpha Group")
Patricia Louise Mining Development Inc. ("Patricia") — 34.0%
Narra Nickel Mining & Development Corporation (Narra) — 60.4%
Under a joint venture agreement the Company holds directly and indirectly an effective equity
interest in the Alpha Property of 60.4%.Pursuant to a shareholders' agreement, the Company
exercises joint control over the companies in the Alpha Group.48(emphasis supplied)
Concluding from the above-stated facts, it is quite safe to say that petitioners McArthur, Tesoro and Narra are not
Filipino since MBMI, a 100% Canadian corporation, owns 60% or more of their equity interests. Such conclusion is derived
from grandfathering petitioners' corporate owners, namely: MMI, SMMI and PLMDC. Going further and adding to the
picture, MBMI's Summary of Significant Accounting Policies statement — regarding the "joint venture" agreements that it
entered into with the "Olympic" and "Alpha" groups — involves SMMI, Tesoro, PLMDC and Narra. Noticeably, the
ownership of the "layered" corporations boils down to MBMI, Olympic or corporations under the "Alpha" group wherein
MBMI has joint venture agreements with, practically exercising majority control over the corporations mentioned. In effect,
whether looking at the capital structure or the underlying relationships between and among the corporations, petitioners are
NOT Filipino nationals and must be considered foreign since 60% or more of their capital stocks or equity interests are
owned by MBMI.
Application of the res inter alios acta rule
Petitioners question the CA's use of the exception of the res inter alios acta or the "admission by co-partner or
agent" rule and "admission by privies" under the Rules of Court in the instant case, by pointing out that statements made by
MBMI should not be admitted in this case since it is not a party to the case and that it is not a "partner" of petitioners.
Secs. 29 and 31, Rule 130 of the Revised Rules of Court provide:
Sec. 29. Admission by co-partner or agent. — The act or declaration of a partner or agent of the
party within the scope of his authority and during the existence of the partnership or agency, may be
given in evidence against such party after the partnership or agency is shown by evidence other than
such act or declaration itself. The same rule applies to the act or declaration of a joint owner, joint debtor,
or other person jointly interested with the party.
Sec. 31. Admission by privies. — Where one derives title to property from another, the act,
declaration, or omission of the latter, while holding the title, in relation to the property, is evidence against
the former.
Petitioners claim that before the above-mentioned Rule can be applied to a case, "the partnership relation must be
shown, and that proof of the fact must be made by evidence other than the admission itself." 49 Thus, petitioners assert that
the CA erred in finding that a partnership relationship exists between them and MBMI because, in fact, no such partnership
exists.
Partnerships vs. joint venture agreements
Petitioners claim that the CA erred in applying Sec. 29, Rule 130 of the Rules by stating that "by entering into a joint
venture, MBMI have a joint interest" with Narra, Tesoro and McArthur. They challenged the conclusion of the CA which
pertains to the close characteristics of "partnerships" and "joint venture agreements." Further, they asserted that before this
particular partnership can be formed, it should have been formally reduced into writing since the capital involved is more
than three thousand pesos (PhP3,000).Being that there is no evidence of written agreement to form a partnership between
petitioners and MBMI, no partnership was created.
We disagree.
A partnership is defined as two or more persons who bind themselves to contribute money, property, or industry to
a common fund with the intention of dividing the profits among themselves. 50 On the other hand, joint ventures have been
deemed to be "akin" to partnerships since it is difficult to distinguish between joint ventures and partnerships. Thus: IEDHAT
[T]he relations of the parties to a joint venture and the nature of their association are so similar
and closely akin to a partnership that it is ordinarily held that their rights, duties, and liabilities are to be
tested by rules which are closely analogous to and substantially the same, if not exactly the same, as
those which govern partnership. In fact, it has been said that the trend in the law has been to blur the
distinctions between a partnership and a joint venture, very little law being found applicable to one that
does not apply to the other.51
Though some claim that partnerships and joint ventures are totally different animals, there are very few rules that
differentiate one from the other; thus, joint ventures are deemed "akin" or similar to a partnership. In fact, in joint venture
agreements, rules and legal incidents governing partnerships are applied. 52
Accordingly, culled from the incidents and records of this case, it can be assumed that the relationships entered
between and among petitioners and MBMI are no simple "joint venture agreements." As a rule, corporations are prohibited
from entering into partnership agreements; consequently, corporations enter into joint venture agreements with other
corporations or partnerships for certain transactions in order to form "pseudo partnerships." Obviously, as the intricate web
of "ventures" entered into by and among petitioners and MBMI was executed to circumvent the legal prohibition against
corporations entering into partnerships, then the relationship created should be deemed as "partnerships," and the laws on
partnership should be applied. Thus, a joint venture agreement between and among corporations may be seen as similar to
partnerships since the elements of partnership are present.
Considering that the relationships found between petitioners and MBMI are considered to be partnerships, then the
CA is justified in applying Sec. 29, Rule 130 of the Rules by stating that "by entering into a joint venture, MBMI have a joint
interest" with Narra, Tesoro and McArthur.
Panel of Arbitrators' jurisdiction
We affirm the ruling of the CA in declaring that the POA has jurisdiction over the instant case. The POA has
jurisdiction to settle disputes over rights to mining areas which definitely involve the petitions filed by Redmont against
petitioners Narra, McArthur and Tesoro. Redmont, by filing its petition against petitioners, is asserting the right of Filipinos
over mining areas in the Philippines against alleged foreign-owned mining corporations. Such claim constitutes a "dispute"
found in Sec. 77 of RA 7942:
Within thirty (30) days, after the submission of the case by the parties for the decision, the panel
shall have exclusive and original jurisdiction to hear and decide the following:
(a) Disputes involving rights to mining areas
(b) Disputes involving mineral agreements or permits
We held in Celestial Nickel Mining Exploration Corporation v. Macroasia Corp.: 53
The phrase "disputes involving rights to mining areas" refers to any adverse claim, protest, or
opposition to an application for mineral agreement. The POA therefore has the jurisdiction to resolve any
adverse claim, protest, or opposition to a pending application for a mineral agreement filed with the
concerned Regional Office of the MGB. This is clear from Secs. 38 and 41 of the DENR AO 96-40, which
provide:
Sec. 38.
xxx xxx xxx
Within thirty (30) calendar days from the last date of publication/posting/radio
announcements, the authorized officer(s) of the concerned office(s) shall issue a certification(s)
that the publication/posting/radio announcement have been complied with.Any adverse claim,
protest, opposition shall be filed directly, within thirty (30) calendar days from the last
date of publication/posting/radio announcement, with the concerned Regional Office or
through any concerned PENRO or CENRO for filing in the concerned Regional Office for
purposes of its resolution by the Panel of Arbitrators pursuant to the provisions of this
Act and these implementing rules and regulations. Upon final resolution of any adverse
claim, protest or opposition, the Panel of Arbitrators shall likewise issue a certification to
that effect within five (5) working days from the date of finality of resolution thereof.
Where there is no adverse claim, protest or opposition, the Panel of Arbitrators shall
likewise issue a Certification to that effect within five working days therefrom.
xxx xxx xxx
No Mineral Agreement shall be approved unless the requirements under this
Section are fully complied with and any adverse claim/protest/opposition is finally
resolved by the Panel of Arbitrators.
Sec. 41.
xxx xxx xxx
Within fifteen (15) working days from the receipt of the Certification issued by the
Panel of Arbitrators as provided in Section 38 hereof, the concerned Regional Director
shall initially evaluate the Mineral Agreement applications in areas outside Mineral
reservations. He/She shall thereafter endorse his/her findings to the Bureau for further
evaluation by the Director within fifteen (15) working days from receipt of forwarded
documents. Thereafter, the Director shall endorse the same to the secretary for
consideration/approval within fifteen working days from receipt of such endorsement.
In case of Mineral Agreement applications in areas with Mineral Reservations, within
fifteen (15) working days from receipt of the Certification issued by the Panel of Arbitrators as
provided for in Section 38 hereof, the same shall be evaluated and endorsed by the Director to
the Secretary for consideration/approval within fifteen days from receipt of such endorsement.
(emphasis supplied) ACcDEa
It has been made clear from the aforecited provisions that the "disputes involving rights to mining
areas" under Sec. 77(a) specifically refer only to those disputes relative to the applications for a mineral
agreement or conferment of mining rights.
The jurisdiction of the POA over adverse claims, protest, or oppositions to a mining right
application is further elucidated by Secs. 219 and 43 of DENR AO 95-936, which read:
Sec. 219. Filing of Adverse Claims/Conflicts/Oppositions. — Notwithstanding the
provisions of Sections 28, 43 and 57 above, any adverse claim, protest or opposition
specified in said sections may also be filed directly with the Panel of Arbitrators within the
concerned periods for filing such claim, protest or opposition as specified in said Sections.
Sec. 43. Publication/Posting of Mineral Agreement. —
xxx xxx xxx
The Regional Director or concerned Regional Director shall also cause the posting of the
application on the bulletin boards of the Bureau, concerned Regional office(s) and in the
concerned province(s) and municipality(ies),copy furnished the barangayswhere the proposed
contract area is located once a week for two (2) consecutive weeks in a language generally
understood in the locality. After forty-five (45) days from the last date of publication/posting has
been made and no adverse claim, protest or opposition was filed within the said forty-five (45)
days, the concerned offices shall issue a certification that publication/posting has been made and
that no adverse claim, protest or opposition of whatever nature has been filed. On the other
hand, if there be any adverse claim, protest or opposition, the same shall be filed within
forty-five (45) days from the last date of publication/posting, with the Regional Offices
concerned, or through the Department's Community Environment and Natural Resources
Officers (CENRO) or Provincial Environment and Natural Resources Officers (PENRO),to
be filed at the Regional Office for resolution of the Panel of Arbitrators.However previously
published valid and subsisting mining claims are exempted from posted/posting required under
this Section.
No mineral agreement shall be approved unless the requirements under this
section are fully complied with and any opposition/adverse claim is dealt with in writing
by the Director and resolved by the Panel of Arbitrators.(Emphasis supplied.)
It has been made clear from the aforecited provisions that the "disputes involving rights to mining
areas" under Sec. 77(a) specifically refer only to those disputes relative to the applications for a mineral
agreement or conferment of mining rights.
The jurisdiction of the POA over adverse claims, protest, or oppositions to a mining right
application is further elucidated by Secs. 219 and 43 of DENRO AO 95-936, which reads:
Sec. 219. Filing of Adverse Claims/Conflicts/Oppositions.— Notwithstanding the
provisions of Sections 28, 43 and 57 above, any adverse claim, protest or opposition specified in
said sections may also be filed directly with the Panel of Arbitrators within the concerned periods
for filing such claim, protest or opposition as specified in said Sections.
Sec. 43. Publication/Posting of Mineral Agreement Application.—
xxx xxx xxx
The Regional Director or concerned Regional Director shall also cause the posting of the
application on the bulletin boards of the Bureau, concerned Regional office(s) and in the
concerned province(s) and municipality(ies),copy furnished the barangayswhere the proposed
contract area is located once a week for two (2) consecutive weeks in a language generally
understood in the locality. After forty-five (45) days from the last date of publication/posting has
been made and no adverse claim, protest or opposition was filed within the said forty-five (45)
days, the concerned offices shall issue a certification that publication/posting has been made and
that no adverse claim, protest or opposition of whatever nature has been filed. On the other
hand, if there be any adverse claim, protest or opposition, the same shall be filed within
forty-five (45) days from the last date of publication/posting, with the Regional offices
concerned, or through the Department's Community Environment and Natural Resources
Officers (CENRO) or Provincial Environment and Natural Resources Officers (PENRO),to
be filed at the Regional Office for resolution of the Panel of Arbitrators.However, previously
published valid and subsisting mining claims are exempted from posted/posting required under
this Section.
No mineral agreement shall be approved unless the requirements under this
section are fully complied with and any opposition/adverse claim is dealt with in writing
by the Director and resolved by the Panel of Arbitrators.(Emphasis supplied.)
These provisions lead us to conclude that the power of the POA to resolve any adverse claim,
opposition, or protest relative to mining rights under Sec. 77(a) of RA 7942 is confined only to adverse
claims, conflicts and oppositions relating to applications for the grant of mineral rights. POA's
jurisdiction is confined only to resolutions of such adverse claims, conflicts and oppositions and
it has no authority to approve or reject said applications. Such power is vested in the DENR
Secretary upon recommendation of the MGB Director. Clearly, POA's jurisdiction over "disputes
involving rights to mining areas" has nothing to do with the cancellation of existing mineral
agreements.(emphasis ours)
Accordingly, as we enunciated in Celestial, the POA unquestionably has jurisdiction to resolve disputes over MPSA
applications subject of Redmont's petitions. However, said jurisdiction does not include either the approval or rejection of
the MPSA applications, which is vested only upon the Secretary of the DENR. Thus, the finding of the POA, with respect to
the rejection of petitioners' MPSA applications being that they are foreign corporation, is valid.
Justice Marvic Mario Victor F. Leonen, in his Dissent, asserts that it is the regular courts, not the POA, that has
jurisdiction over the MPSA applications of petitioners.
This postulation is incorrect.
It is basic that the jurisdiction of the court is determined by the statute in force at the time of the commencement of
the action. 54
Sec. 19, Batas Pambansa Blg. 129 or "The Judiciary Reorganization Act of 1980" reads:
Sec. 19. Jurisdiction in Civil Cases. — Regional Trial Courts shall exercise exclusive original
jurisdiction:
1. In all civil actions in which the subject of the litigation is incapable of pecuniary estimation.
On the other hand, the jurisdiction of POA is unequivocal from Sec. 77 of RA 7942:
Section 77. Panel of Arbitrators. —
...Within thirty (30) days, after the submission of the case by the parties for the decision, the
panel shall have exclusive and original jurisdiction to hear and decide the following:
(c) Disputes involving rights to mining areas SEIcHa
(d) Disputes involving mineral agreements or permits
It is clear that POA has exclusive and original jurisdiction over any and all disputes involving rights to mining areas.
One such dispute is an MPSA application to which an adverse claim, protest or opposition is filed by another interested
applicant. In the case at bar, the dispute arose or originated from MPSA applications where petitioners are asserting their
rights to mining areas subject of their respective MPSA applications. Since respondent filed 3 separate petitions for the
denial of said applications, then a controversy has developed between the parties and it is POA's jurisdiction to resolve said
disputes.
Moreover, the jurisdiction of the RTC involves civil actions while what petitioners filed with the DENR Regional
Office or any concerned DENRE or CENRO are MPSA applications. Thus POA has jurisdiction.
Furthermore, the POA has jurisdiction over the MPSA applications under the doctrine of primary jurisdiction. Euro-
med Laboratories v. Province of Batangas 55 elucidates:
The doctrine of primary jurisdiction holds that if a case is such that its determination requires the
expertise, specialized training and knowledge of an administrative body, relief must first be obtained in an
administrative proceeding before resort to the courts is had even if the matter may well be within their
proper jurisdiction.
Whatever may be the decision of the POA will eventually reach the court system via a resort to the CA and to this
Court as a last recourse.
Selling of MBMI's shares to DMCI
As stated before, petitioners' Manifestation and Submission dated October 19, 2012 would want us to declare the
instant petition moot and academic due to the transfer and conveyance of all the shareholdings and interests of MBMI to
DMCI, a corporation duly organized and existing under Philippine laws and is at least 60% Philippine-owned. 56 Petitioners
reasoned that they now cannot be considered as foreign-owned; the transfer of their shares supposedly cured the "defect"
of their previous nationality. They claimed that their current FTAA contract with the State should stand since "even wholly-
owned foreign corporations can enter into an FTAA with the State." 57 Petitioners stress that there should no longer be any
issue left as regards their qualification to enter into FTAA contracts since they are qualified to engage in mining activities in
the Philippines. Thus, whether the "grandfather rule" or the "control test" is used, the nationalities of petitioners cannot be
doubted since it would pass both tests.
The sale of the MBMI shareholdings to DMCI does not have any bearing in the instant case and said fact should be
disregarded. The manifestation can no longer be considered by us since it is being tackled in G.R. No. 202877 pending
before this Court. Thus, the question of whether petitioners, allegedly a Philippine-owned corporation due to the sale of
MBMI's shareholdings to DMCI, are allowed to enter into FTAAs with the State is a non-issue in this case.
In ending, the "control test" is still the prevailing mode of determining whether or not a corporation is a Filipino
corporation, within the ambit of Sec. 2, Art. II of the 1987 Constitution, entitled to undertake the exploration, development
and utilization of the natural resources of the Philippines. When in the mind of the Court there is doubt, based on the
attendant facts and circumstances of the case, in the 60-40 Filipino-equity ownership in the corporation, then it may apply
the "grandfather rule."
WHEREFORE,premises considered, the instant petition is DENIED.The assailed Court of Appeals Decision dated
October 1, 2010 and Resolution dated February 15, 2011 are hereby AFFIRMED. SO ORDERED.
||| (Narra Nickel Mining & Development Corp. v. Redmont Consolidated Mines Corp., G.R. No. 195580, [April 21, 2014], 733
PHIL 365-490)

[G.R. No. 195580. January 28, 2015.]

NARRA NICKEL MINING AND DEVELOPMENT CORP., TESORO MINING AND DEVELOPMENT,
INC., and McARTHUR MINING, INC., petitioners, vs. REDMONT CONSOLIDATED MINES
CORP., respondent.

RESOLUTION

VELASCO, JR., J p:

Before the Court is the Motion for Reconsideration of its April 21, 2014 Decision, which denied the Petition for
Review on Certiorariunder Rule 45 jointly interposed by petitioners Narra Nickel and Mining Development Corp. (Narra),
Tesoro Mining and Development, Inc. (Tesoro), and McArthur Mining, Inc. (McArthur), and affirmed the October 1, 2010
Decision and February 15, 2011 Resolution of the Court of Appeals (CA) in CA-G.R. SP No. 109703.
Very simply, the challenged Decision sustained the appellate court's ruling that petitioners, being foreign
corporations, are not entitled to Mineral Production Sharing Agreements (MPSAs). In reaching its conclusion, this Court
upheld with approval the appellate court's finding that there was doubt as to petitioners' nationality since a 100% Canadian-
owned firm, MBMI Resources, Inc. (MBMI), effectively owns 60% of the common stocks of the petitioners by owning equity
interest of petitioners' other majority corporate shareholders.
In a strongly worded Motion for Reconsideration dated June 5, 2014, petitioners-movants argued, in the main, that
the Court's Decision was not in accord with law and logic. In its September 2, 2014 Comment, on the other hand,
respondent Redmont Consolidated Mines Corp. (Redmont) countered that petitioners' motion for reconsideration is nothing
but a rehash of their arguments and should, thus, be denied outright for being pro-forma. Petitioners have interposed on
September 30, 2014 their Reply to the respondent's Comment.
After considering the parties' positions, as articulated in their respective submissions, We resolve to deny the
motion for reconsideration.
I.
The case has not been rendered moot and academic
Petitioners have first off criticized the Court for resolving in its Decision a substantive issue, which, as argued, has
supposedly been rendered moot by the fact that petitioners' applications for MPSAs had already been converted to an
application for a Financial Technical Assistance Agreement (FTAA), as petitioners have in fact been granted an FTAA.
Further, the nationality issue, so petitioners presently claim, had been rendered moribund by the fact that MBMI had already
divested itself and sold all its shareholdings in the petitioners, as well as in their corporate stockholders, to a Filipino
corporation — DMCI Mining Corporation (DMCI).
As a counterpoint, respondent Redmont avers that the present case has not been rendered moot by the supposed
issuance of an FTAA in petitioners' favor as this FTAA was subsequently revoked by the Office of the President (OP) and is
currently a subject of a petition pending in the Court's First Division. Redmont likewise contends that the supposed sale of
MBMI's interest in the petitioners and in their "holding companies" is a question of fact that is outside the Court's province to
verify in a Rule 45 certiorari proceedings. In any case, assuming that the controversy has been rendered moot, Redmont
claims that its resolution on the merits is still justified by the fact that petitioners have violated a constitutional provision, the
violation is capable of repetition yet evading review, and the present case involves a matter of public concern.
Indeed, as the Court clarified in its Decision, the conversion of the MPSA application to one for FTAAs and the
issuance by the OP of an FTAA in petitioners' favor are irrelevant. The OP itself has already cancelled and revoked the
FTAA thus issued to petitioners. Petitioners curiously have omitted this critical fact in their motion for reconsideration.
Furthermore, the supposed sale by MBMI of its shares in the petitioner-corporations and in their holding companies is not
only a question of fact that this Court is without authority to verify, it also does not negate any violation of the Constitutional
provisions previously committed before any such sale.
We can assume for the nonce that the controversy had indeed been rendered moot by these two events. As this
Court has time and again declared, the "moot and academic" principle is not a magical formula that automatically dissuades
courts in resolving a case. 1 The Court may still take cognizance of an otherwise moot and academic case, if it finds that (a)
there is a grave violation of the Constitution; (b) the situation is of exceptional character and paramount public interest is
involved; (c) the constitutional issue raised requires formulation of controlling principles to guide the bench, the bar, and the
public; and (d) the case is capable of repetition yet evading review. 2 The Court's April 21, 2014 Decision explained in some
detail that all four (4) of the foregoing circumstances are present in the case. If only to stress a point, we will do so again.
First, allowing the issuance of MPSAs to applicants that are owned and controlled by a 100% foreign-owned
corporation, albeit through an intricate web of corporate layering involving alleged Filipino corporations, is tantamount to
permitting a blatant violation of Section 2, Article XII of the Constitution. The Court simply cannot allow this breach and
inhibit itself from resolving the controversy on the facile pretext that the case had already been rendered academic.
Second, the elaborate corporate layering resorted to by petitioners so as to make it appear that there is compliance
with the minimum Filipino ownership in the Constitution is deftly exceptional in character. More importantly, the case is of
paramount public interest, as the corporate layering employed by petitioners was evidently designed to circumvent the
constitutional caveat allowing only Filipino citizens and corporations 60%-owned by Filipino citizens to explore, develop, and
use the country's natural resources.
Third, the facts of the case, involving as they do a web of corporate layering intended to go around the Filipino
ownership requirement in the Constitution and pertinent laws, require the establishment of a definite principle that will
ensure that the Constitutional provision reserving to Filipino citizens or "corporations at least sixty per centum of whose
capital is owned by such citizens" be effectively enforced and complied with. The case, therefore, is an opportunity to
establish a controlling principle that will "guide the bench, the bar, and the public."
Lastly, the petitioners' actions during the lifetime and existence of the instant case that gave rise to the present
controversy are capable of repetition yet evading review because, as shown by petitioners' actions, foreign corporations can
easily utilize dummy Filipino corporations through various schemes and stratagems to skirt the constitutional prohibition
against foreign mining in Philippine soil.
II.
The application of the Grandfather Rule is justified by the circumstances of the case to determine the
nationality of petitioners.
To petitioners, the Court's application of the Grandfather Rule to determine their nationality is erroneous and
allegedly without basis in the Constitution, the Foreign Investments Act of 1991 (FIA), the Philippine Mining Act of
1995, 3 and the Rules issued by the Securities and Exchange Commission (SEC). These laws and rules supposedly
espouse the application of the Control Test in verifying the Philippine nationality of corporate entities for purposes of
determining compliance with Sec. 2, Art. XII of the Constitution that only "corporations or associations at least sixty per
centum of whose capital is owned by such [Filipino] citizens" may enjoy certain rights and privileges, like the exploration and
development of natural resources.
The application of the Grandfather Rule in the
present case does not eschew the Control Test.
Clearly, petitioners have misread, and failed to appreciate the clear import of, the Court's April 21, 2014 Decision.
Nowhere in that disposition did the Court foreclose the application of the Control Test in determining which corporations
may be considered as Philippine nationals. Instead, to borrow Justice Leonen's term, the Court used the Grandfather Rule
as a "supplement" to the Control Test so that the intent underlying the averted Sec. 2, Art. XII of the Constitution be given
effect. The following excerpts of the April 21, 2014 Decision cannot be clearer:
In ending, the "control test" is still the prevailing mode of determining whether or not a
corporation is a Filipino corporation, within the ambit of Sec. 2, Art. XII of the 1987 Constitution,
entitled to undertake the exploration, development and utilization of the natural resources of the
Philippines. When in the mind of the Court, there is doubt, based on the attendant facts and
circumstances of the case, in the 60-40 Filipino equity ownership in the corporation, then it may apply
the "grandfather rule."(emphasis supplied)
With that, the use of the Grandfather Rule as a "supplement" to the Control Test is not proscribed by
the Constitution or thePhilippine Mining Act of 1995.
The Grandfather Rule implements the intent of
the Filipinization provisions of the Constitution.
To reiterate, Sec. 2, Art. XII of the Constitution reserves the exploration, development, and utilization of natural
resources to Filipinocitizens and "corporations or associations at least sixty per centum of whose capital is owned by such
citizens." Similarly, Section 3 (aq) of the Philippine Mining Act of 1995 considers a "corporation . . . registered in accordance
with law at least sixty per cent of the capital of which is owned by citizens of the Philippines" as a person qualified to
undertake a mining operation. Consistent with this objective, the Grandfather Rule was originally conceived to look into
the citizenship of the individuals who ultimately own and control the shares of stock of a corporation for purposes of
determining compliance with the constitutional requirement of Filipino ownership. It cannot, therefore, be denied that the
framers of the Constitution have not foreclosed the Grandfather Rule as a tool in verifying the nationality of corporations for
purposes of ascertaining their right to participate in nationalized or partly nationalized activities. The following excerpts from
the Record of the 1986 Constitutional Commission suggest as much:
MR. NOLLEDO:
In Sections 3, 9 and 15, the Committee stated local or Filipino equity and foreign equity; namely, 60-40 in
Section 3, 60-40 in Section 9, and 2/3-1/3 in Section 15.
MR. VILLEGAS:
That is right.
xxx xxx xxx
MR. NOLLEDO:
Thank you.
With respect to an investment by one corporation in another corporation, say, a corporation with 60-40
percent equity invests in another corporation which is permitted by the Corporation Code, does
the Committee adopt the grandfather rule?
MR. VILLEGAS:
Yes, that is the understanding of the Committee.
As further defined by Dean Cesar Villanueva, the Grandfather Rule is "the method by which the percentage of
Filipino equity in a corporation engaged in nationalized and/or partly nationalized areas of activities, provided for under
the Constitution and other nationalization laws, is computed, in cases where corporate shareholders are present,
by attributing the nationality of the second or even subsequent tier of ownership to determine the nationality of the
corporate shareholder." 4 Thus, to arrive at the actual Filipino ownership and control in a corporation, both the direct and
indirect shareholdings in the corporation are determined.
This concept of stock attribution inherent in the Grandfather Rule to determine the ultimate ownership in a
corporation is observed by the Bureau of Internal Revenue (BIR) in applying Section 127 (B) 5 of the National Internal
Revenue Code on taxes imposed on closely held corporations, in relation to Section 96 of the Corporation Code 6 on close
corporations. Thus, in BIR Ruling No. 148-10, Commissioner Kim Henares held:
In the case of a multi-tiered corporation, the stock attribution rule must be allowed to run
continuously along the chain of ownership until it finally reaches the individual
stockholders. This is in consonance with the "grandfather rule" adopted in the Philippines under
Section 96 of the Corporation Code (Batas Pambansa Blg. 68) which provides that notwithstanding the
fact that all the issued stock of a corporation are held by not more than twenty persons, among others, a
corporation is nonetheless not to be deemed a close corporation when at least two thirds of its voting
stock or voting rights is owned or controlled by another corporation which is not a close corporation. 7
In SEC-OGC Opinion No. 10-31 dated December 9, 2010 (SEC Opinion 10-31), the SEC applied the Grandfather
Rule even if the corporation engaged in mining operation passes the 60-40 requirement of the Control Test, viz.:
You allege that the structure of MML's ownership in PHILSAGA is as follows: (1) MML owns 40%
equity in MEDC, while the 60% is ostensibly owned by Philippine individual citizens who are actually
MML's controlled nominees; (2) MEDC, in turn, owns 60% equity in MOHC, while MML owns the
remaining 40%; (3) Lastly, MOHC owns 60% of PHILSAGA, while MML owns the remaining 40%. You
provide the following figure to illustrate this structure:
xxx xxx xxx
We note that the Constitution and the statute use the concept "Philippine citizens." Article III,
Section 1 of the Constitution provides who are Philippine citizens: . . . This enumeration is exhaustive. In
other words, there can be no other Philippine citizens other than those falling within the enumeration
provided by the Constitution. Obviously, only natural persons are susceptible of citizenship. Thus, for
purposes of the Constitutional and statutory restrictions on foreign participation in the exploitation of
mineral resources, a corporation investing in a mining joint venture can never be considered as a
Philippine citizen.
The Supreme Court En Banc confirms this [in] . . . Pedro R. Palting, vs. San Jose Petroleum,
[Inc.]. The Court held that a corporation investing in another corporation engaged in a nationalized
activity cannot be considered as a citizen for purposes of the Constitutional provision restricting foreign
exploitation of natural resources:
xxx xxx xxx
Accordingly, we opine that we must look into the citizenship of the individual stockholders, i.e.,
natural persons, of that investor-corporation in order to determine if the Constitutional and statutory
restrictions are complied with. If the shares of stock of the immediate investor corporation is in turn held
and controlled by another corporation, then we must look into the citizenship of the individual
stockholders of the latter corporation. In other words, if there are layers of intervening corporations
investing in a mining joint venture, we must delve into the citizenship of the individual
stockholders of each corporation. This is the strict application of the grandfather rule, which the
Commission has been consistently applying prior to the 1990s.
Indeed, the framers of the Constitution intended for the "grandfather rule" to apply in case
a 60%-40% Filipino-Foreign equity corporation invests in another corporation engaging in an
activity where the Constitution restricts foreign participation.
xxx xxx xxx
Accordingly, under the structure you represented, the joint mining venture is 87.04% foreign
owned, while it is only 12.96% owned by Philippine citizens. Thus, the constitutional requirement of 60%
ownership by Philippine citizens is violated. (emphasis supplied)
Similarly, in the eponymous Redmont Consolidated Mines Corporation v. McArthur Mining, Inc., et al., 8 the SEC en
banc applied the Grandfather Rule despite the fact that the subject corporations ostensibly have satisfied the 60-40 Filipino
equity requirement. The SEC en banc held that to attain the Constitutional objective of reserving to Filipinos the
utilization of natural resources, one should not stop where the percentage of the capital stock is 60%. Thus:
[D]oubt, we believe, exists in the instant case because the foreign investor, MBMI,
provided practically all the funds of the remaining appellee-corporations. The records disclose that:
(1) Olympic Mines and Development Corporation ("OMDC"), a domestic corporation, and MBMI
subscribed to 6,663 and 3,331 shares, respectively, out of the authorized capital stock of Madridejos;
however, OMDC paid nothing for this subscription while MBMI paid P2,803,900.00 out of its total
subscription cost of P3,331,000.00; (2) Palawan Alpha South Resource Development Corp. ("Palawan
Alpha"), also a domestic corporation, and MBMI subscribed to 6,596 and 3,996 shares, respectively, out
of the authorized capital stock of Patricia Louise; however, Palawan Alpha paid nothing for this
subscription while MBMI paid P2,796,000.00 out of its total subscription cost of P3,996,000.00; (3)
OMDC and MBMI subscribed to 6,663 and 3,331 shares, respectively, out of the authorized capital stock
of Sara Marie; however, OMDC paid nothing for this subscription while MBMI paid P2,794,000.00 out of
its total subscription cost of P3,331,000.00; and (4) Falcon Ridge Resources Management Corp. ("Falcon
Ridge"), another domestic corporation, and MBMI subscribed to 5,997 and 3,998 shares, respectively,
out of the authorized capital stock of San Juanico; however, Falcon Ridge paid nothing for this
subscription while MBMI paid P2,500,000.00 out of its total subscription cost of P3,998,000.00. Thus,
pursuant to the afore-quoted DOJ Opinion, the Grandfather Rule must be used.
xxx xxx xxx
The avowed purpose of the Constitution is to place in the hands of Filipinos the
exploitation of our natural resources. Necessarily, therefore, the Rule interpreting the
constitutional provision should not diminish that right through the legal fiction of corporate
ownership and control. But the constitutional provision, as interpreted and practiced via the 1967 SEC
Rules, has favored foreigners contrary to the command of the Constitution. Hence, the Grandfather
Rule must be applied to accurately determine the actual participation, both direct and indirect, of
foreigners in a corporation engaged in a nationalized activity or business.
The method employed in the Grandfather Rule of attributing the shareholdings of a given corporate shareholder to
the second or even the subsequent tier of ownership hews with the rule that the "beneficial ownership" of corporations
engaged in nationalized activities must reside in the hands of Filipino citizens. Thus, even if the 60-40 Filipino equity
requirement appears to have been satisfied, the Department of Justice (DOJ), in its Opinion No. 144, S. of 1977, stated
that an agreement that may distort the actual economic or beneficial ownership of a mining corporation may be
struck down as violative of the constitutional requirement, viz.:
In this connection, you raise the following specific questions:
1. Can a Philippine corporation with 30% equity owned by foreigners enter into a mining service
contract with a foreign company granting the latter a share of not more than 40% from the proceeds of
the operations?
xxx xxx xxx
By law, a mining lease may be granted only to a Filipino citizen, or to a corporation or
partnership registered with the [SEC] at least 60% of the capital of which is owned by Filipino
citizens and possessing . . . . The sixty percent Philippine equity requirement in mineral resource
exploitation . . . is intended to insure, among other purposes, the conservation of indigenous
natural resources, for Filipino posterity . . . . I think it is implicit in this provision, even if it refers merely
to ownership of stock in the corporation holding the mining concession, that beneficial ownership of the
right to dispose, exploit, utilize, and develop natural resources shall pertain to Filipino citizens,
and that the nationality requirement is not satisfied unless Filipinos, are the principal
beneficiaries in the exploitation of the country's natural resources. This criterion of beneficial
ownership is tacitly adopted in Section 44 of P.D. No. 463, above-quoted, which limits the service fee in
service contracts to 40% of the proceeds of the operation, thereby implying that the 60-40 benefit-sharing
ratio is derived from the 60-40 equity requirement in the Constitution.
xxx xxx xxx
It is obvious that while payments to a service contractor may be justified as a service fee, and
therefore, properly deductible from gross proceeds, the service contract could be employed as a
means of going about or circumventing the constitutional limit on foreign equity participation and
the obvious constitutional policy to insure that Filipinos retain beneficial ownership of our
mineral resources. Thus, every service contract scheme has to be evaluated in its entirety, on a case to
case basis, to determine reasonableness of the total "service fee" . . . like the options available to the
contractor to become equity participant in the Philippine entity holding the concession, or to acquire rights
in the processing and marketing stages. . . . (emphasis supplied)
The "beneficial ownership" requirement was subsequently used in tandem with the "situs of control" to determine
the nationality of a corporation in DOJ Opinion No. 84, S. of 1988, through the Grandfather Rule, despite the fact that both
the investee and investor corporations purportedly satisfy the 60-40 Filipino equity requirement: 9
This refers to your request for opinion on whether or not there may be an investment in real
estate by a domestic corporation (the investing corporation) seventy percent (70%) of the capital stock of
which is owned by another domestic corporation with at least 60%-40% Filipino-Foreign Equity, while the
remaining thirty percent (30%) of the capital stock is owned by a foreign corporation.
xxx xxx xxx
This Department has had the occasion to rule in several opinions that it is implicit in the
constitutional provisions, even if it refers merely to ownership of stock in the corporation holding the land
or natural resource concession, that the nationality requirement is not satisfied unless it meets the
criterion of beneficial ownership, i.e., Filipinos are the principal beneficiaries in the exploration of
natural resources (Op. No. 144, s. 1977; Op. No. 130, s. 1985), and that in applying the same "the
primordial consideration is situs of control, whether in a stock or non-stock corporation" (Op. No.
178, s. 1974). As stated in the Register of Deeds vs. Ung Sui Si Temple (97 Phil. 58), obviously to insure
that corporations and associations allowed to acquire agricultural land or to exploit natural resources
"shall be controlled by Filipinos." Accordingly, any arrangement which attempts to defeat the
constitutional purpose should be eschewed (Op. No. 130, s. 1985).
We are informed that in the registration of corporations with the [SEC], compliance with the sixty
per centum requirement is being monitored by SEC under the "Grandfather Rule" a method by which the
percentage of Filipino equity in corporations engaged in nationalized and/or partly nationalized areas of
activities provided for under the Constitution and other national laws is accurately computed, and the
diminution if said equity prevented (SEC Memo, S. 1976). The "Grandfather Rule" is applied
specifically in cases where the corporation has corporate stockholders with alien stockholdings,
otherwise, if the rule is not applied, the presence of such corporate stockholders could diminish
the effective control of Filipinos.
Applying the "Grandfather Rule" in the instant case, the result is as follows: . . . the total foreign
equity in the investing corporation is 58% while the Filipino equity is only 42%, in the investing
corporation, subject of your query, is disqualified from investing in real estate, which is a nationalized
activity, as it does not meet the 60%-40% Filipino-Foreign equity requirement under the Constitution.
This pairing of the concepts "beneficial ownership" and the "situs of control" in determining what constitutes
"capital" has been adopted by this Court in Heirs of Gamboa v. Teves. 10 In its October 9, 2012 Resolution, the Court
clarified, thus:
This is consistent with Section 3 of the FIA which provides that where 100% of the capital stock
is held by "a trustee of funds for pension or other employee retirement or separation benefits," the trustee
is a Philippine national if "at least sixty percent (60%) of the fund will accrue to the benefit of Philippine
nationals." Likewise, Section 1(b) of the Implementing Rules of the FIA provides that "for stocks to be
deemed owned and held by Philippine citizens or Philippine nationals, mere legal title is not enough to
meet the required Filipino equity. Full beneficial ownership of the stocks, coupled with appropriate
voting rights, is essential." (emphasis supplied)
In emphasizing the twin requirements of "beneficial ownership" and "control" in determining compliance with the
required Filipino equity in Gamboa, the en banc Court explicitly cited with approval the SEC en banc's application
in Redmont Consolidated Mines, Corp. v. McArthur Mining, Inc., et al. of the Grandfather Rule, to wit:
Significantly, the SEC en banc, which is the collegial body statutorily empowered to issue rules
and opinions on behalf of SEC, has adopted the Grandfather Rule in determining compliance with the
60-40 ownership requirement in favor of Filipino citizens mandated by the Constitution for certain
economic activities. This prevailing SEC ruling, which the SEC correctly adopted to thwart any
circumvention of the required Filipino "ownership and control," is laid down in the 25 March 2010
SEC en banc ruling in Redmont Consolidated Mines, Corp. v. McArthur Mining, Inc., et al. . . . (emphasis
supplied)
Applying Gamboa, the Court, in Express Investments III Private Ltd. v. Bayantel Communications, Inc., 11 denied
the foreign creditors' proposal to convert part of Bayantel's debts to common shares of the company at a rate of
77.7%. Supposedly, the conversion of the debts to common shares by the foreign creditors would be done, both
directly and indirectly, in order to meet the control test principle under the FIA. Under the proposed structure, the
foreign creditors would own 40% of the outstanding capital stock of the telecommunications company on a direct basis,
while the remaining 40% of shares would be registered to a holding company that shall retain, on a direct basis, the other
60% equity reserved for Filipino citizens. Nonetheless, the Court found the proposal non-compliantwith the
Constitutional requirement of Filipino ownership as the proposed structure would give more than 60% of the ownership
of the common shares of Bayantel to the foreign corporations, viz.:
In its Rehabilitation Plan, among the material financial commitments made by respondent
Bayantel is that its shareholders shall relinquish the agreed-upon amount of common stock[s] as payment
to Unsecured Creditors as per the Term Sheet. Evidently, the parties intend to convert the
unsustainable portion of respondent's debt into common stocks, which have voting rights. If we
indulge petitioners on their proposal, the Omnibus Creditors which are foreign corporations, shall
have control over 77.7% of Bayantel, a public utility company. This is precisely the scenario
proscribed by the Filipinization provision of the Constitution. Therefore, the Court of Appeals acted
correctly in sustaining the 40% debt-to-equity ceiling on conversion. (emphasis supplied)
As shown by the quoted legislative enactments, administrative rulings, opinions, and this Court's decisions, the
Grandfather Rule not only finds basis, but more importantly, it implements the Filipino equity requirement, in
the Constitution.
Application of the Grandfather
Rule with the Control Test.
Admittedly, an ongoing quandary obtains as to the role of the Grandfather Rule in determining compliance with the
minimum Filipino equity requirement vis-à-vis the Control Test. This confusion springs from the erroneous assumption that
the use of one method forecloses the use of the other.
As exemplified by the above rulings, opinions, decisions and this Court's April 21, 2014 Decision, the Control Test
can be, as it has been, applied jointly with the Grandfather Rule to determine the observance of foreign ownership
restriction in nationalized economic activities. The Control Test and the Grandfather Rule are not, as it were, incompatible
ownership-determinant methods that can only be applied alternative to each other. Rather, these methods can, if
appropriate, be used cumulatively in the determination of the ownership and control of corporations engaged in
fully or partly nationalized activities, as the mining operation involved in this case or the operation of public utilities as
in Gamboa or Bayantel.
The Grandfather Rule, standing alone, should not be used to determine the Filipino ownership and control in a
corporation, as it could result in an otherwise foreign corporation rendered qualified to perform nationalized or partly
nationalized activities. Hence, it is only when the Control Test is first complied with that the Grandfather Rule may be
applied. Put in another manner, if the subject corporation's Filipino equity falls below the threshold 60%, the corporation is
immediately considered foreign-owned, in which case, the need to resort to the Grandfather Rule disappears.
On the other hand, a corporation that complies with the 60-40 Filipino to foreign equity requirement can be
considered a Filipino corporation if there is no doubt as to who has the "beneficial ownership" and "control" of the
corporation. In that instance, there is no need for a dissection or further inquiry on the ownership of the corporate
shareholders in both the investing and investee corporation or the application of the Grandfather Rule. 12 As a corollary
rule, even if the 60-40 Filipino to foreign equity ratio is apparently met by the subject or investee corporation, a resort to the
Grandfather Rule is necessary if doubt exists as to the locus of the "beneficial ownership" and "control." In this
case, a further investigation as to the nationality of the personalities with the beneficial ownership and control of the
corporate shareholders in both the investing and investee corporations is necessary.
As explained in the April 21, 2012 Decision, the "doubt" that demands the application of the Grandfather Rule in
addition to or in tandem with the Control Test is not confined to, or more bluntly, does not refer to the fact that the apparent
Filipino ownership of the corporation's equity falls below the 60% threshold. Rather, "doubt" refers to various indicia that
the "beneficial ownership" and "control" of the corporation do not in fact reside in Filipino shareholders but in
foreign stakeholders. As provided in DOJ Opinion No. 165, Series of 1984, which applied the pertinent provisions of
the Anti-Dummy Law in relation to the minimum Filipino equity requirement in the Constitution, "significant indicators of the
dummy status" have been recognized in view of reports "that some Filipino investors or businessmen are being utilized or
[are] allowing themselves to be used as dummies by foreign investors" specifically in joint ventures for national resource
exploitation. These indicators are:
1. That the foreign investors provide practically all the funds for the joint investment undertaken
by these Filipino businessmen and their foreign partner;
2. That the foreign investors undertake to provide practically all the technological support for the
joint venture;
3. That the foreign investors, while being minority stockholders, manage the company and
prepare all economic viability studies.
Thus, In the Matter of the Petition for Revocation of the Certificate of Registration of Linear Works Realty
Development Corporation, 13the SEC held that when foreigners contribute more capital to an enterprise, doubt exists
as to the actual control and ownership of the subject corporation even if the 60% Filipino equity threshold is
met. Hence, the SEC in that one ordered a further investigation,viz.:
. . . The [SEC Enforcement and Prosecution Department (EPD)] maintained that the basis for
determining the level of foreign participation is the number of shares subscribed, regardless of the par
value. Applying such an interpretation, the EPD rules that the foreign equity participation in Linearworks
Realty Development Corporation amounts to 26.41% of the corporation's capital stock since the amount
of shares subscribed by foreign nationals is 1,795 only out of the 6,795 shares. Thus, the subject
corporation is compliant with the 40% limit on foreign equity participation. Accordingly, the EPD
dismissed the complaint, and did not pursue any investigation against the subject corporation.
xxx xxx xxx
. . . [I]n this respect we find no error in the assailed order made by the EPD. The EPD did not err
when it did not take into account the par value of shares in determining compliance with the constitutional
and statutory restrictions on foreign equity.
However, we are aware that some unscrupulous individuals employ schemes to
circumvent the constitutional and statutory restrictions on foreign equity. In the present case, the
fact that the shares of the Japanese nationals have a greater par value but only have similar
rights to those held by Philippine citizens having much lower par value, is highly suspicious. This
is because a reasonable investor would expect to have greater control and economic rights than
other investors who invested less capital than him. Thus, it is reasonable to suspect that there may
be secret arrangements between the corporation and the stockholders wherein the Japanese nationals
who subscribed to the shares with greater par value actually have greater control and economic
rights contrary to the equality of shares based on the articles of incorporation.
With this in mind, we find it proper for the EPD to investigate the subject corporation. The EPD is
advised to avail of the Commission's subpoena powers in order to gather sufficient evidence, and file the
necessary complaint.
As will be discussed, even if at first glance the petitioners comply with the 60-40 Filipino to foreign equity
ratio, doubt exists in the present case that gives rise to a reasonable suspicion that the Filipino shareholders do not
actually have the requisite number of control and beneficial ownership in petitioners Narra, Tesoro, and McArthur. Hence, a
further investigation and dissection of the extent of the ownership of the corporate shareholders through the Grandfather
Rule is justified.
Parenthetically, it is advanced that the application of the Grandfather Rule is impractical as tracing the
shareholdings to the point when natural persons hold rights to the stocks may very well lead to an investigation ad
infinitum. Suffice it to say in this regard that, while the Grandfather Rule was originally intended to trace the shareholdings to
the point where natural persons hold the shares, the SEC had already set up a limit as to the number of corporate layers the
attribution of the nationality of the corporate shareholders may be applied.
In a 1977 internal memorandum, the SEC suggested applying the Grandfather Rule on two (2) levels of corporate
relations for publicly-held corporations or where the shares are traded in the stock exchanges, and to three (3) levels for
closely held corporations or the shares of which are not traded in the stock exchanges. 14 These limits comply with the
requirement in Palting v. San Jose Petroleum, Inc. 15that the application of the Grandfather Rule cannot go beyond the
level of what is reasonable.
A doubt exists as to the extent of control and
beneficial ownership of MBMI over the petitioners
and their investing corporate stockholders.
In the Decision subject of this recourse, the Court applied the Grandfather Rule to determine the matter of true
ownership and control over the petitioners as doubt exists as to the actual extent of the participation of MBMI in the equity of
the petitioners and their investing corporations.
We considered the following membership and control structures and like nuances:
Tesoro
Supposedly Filipino corporation Sara Marie Mining, Inc. (Sara Marie) holds 59.97% of the 10,000 common shares
of petitioner Tesoro while the Canadian-owned company, MBMI, holds 39.98% of its shares.
Name Nationality Number Amount Amount Paid
of Shares Subscribed
Sara Marie Mining, Inc. Filipino 5,997 P5,997,000.00 P825,000.00
MBMI Resources, Inc. 16 Canadian 3,998 P3,998,000.00 P1,878,174.60
Lauro L. Salazar Filipino 1 P1,000.00 P1,000.00
Fernando B. Esguerra Filipino 1 P1,000.00 P1,000.00
Manuel A. Agcaoili Filipino 1 P1,000.00 P1,000.00
Michael T. Mason American 1 P1,000.00 P1,000.00
Kenneth Cawkel Canadian 1 P1,000.00 P1,000.00
–––––––– ––––––––––––– –––––––––––––
Total 10,000 P10,000,000.00 P2,708,174.60
======= ============ ============
In turn, the Filipino corporation Olympic Mines & Development Corp. (Olympic) holds 66.63% of Sara Marie's
shares while the same Canadian company MBMI holds 33.31% of Sara Marie's shares. Nonetheless, it is admitted that
Olympic did not pay a single peso for its shares. On the contrary, MBMI paid for 99% of the paid-up capital of Sara Marie.

Name Nationality Number Amount Amount Paid


of Shares Subscribed

Olympic Mines & Filipino 6,663 P6,663,000.00 P0.00


Development Corp. 17
MBMI Resources, Inc. Canadian 3,331 P3,331,000.00 P2,794,000.00
Amanti Limson Filipino 1 P1,000.00 P1,000.00
Fernando B. Esguerra Filipino 1 P1,000.00 P1,000.00
Lauro Salazar Filipino 1 P1,000.00 P1,000.00
Emmanuel G. Hernando Filipino 1 P1,000.00 P1,000.00
Michael T. Mason American 1 P1,000.00 P1,000.00
Kenneth Cawkel Canadian 1 P1,000.00 P1,000.00
–––––––– ––––––––––––– –––––––––––––
Total 10,000 P10,000,000.00 P2,800,000.00
======= ============ ============
The fact that MBMI had practically provided all the funds in Sara Marie and Tesoro creates serious doubt as
to the true extent of its (MBMI) control and ownership over both Sara Marie and Tesoro since, as observed by the
SEC, "a reasonable investor would expect to have greater control and economic rights than other investors who invested
less capital than him." The application of the Grandfather Rule is clearly called for, and as shown below, the Filipinos'
control and economic benefits in petitioner Tesoro (through Sara Marie) fall below the threshold 60%, viz.:
Filipino participation in petitioner Tesoro: 40.01%
66.67
––––– (Filipino equity in Sara Marie) x 59.97 (Sara Marie's share in Tesoro) = 39.98%
100

39.98% + .03% (shares of individual Filipino shareholders [SHs] in Tesoro) = 40.01%


========
Foreign participation in petitioner Tesoro: 59.99%

(Foreign equity in Sara Marie) x 59.97 (Sara Marie's share in Tesoro) =

8% (MBMI's direct participation in Tesoro) + .02% (shares of foreign individual SHs in Tesoro) =

With only 40.01% Filipino ownership in petitioner Tesoro, as compared to 59.99% foreign ownership of its shares, it
is clear that petitioner Tesoro does not comply with the minimum Filipino equity requirement imposed in Sec. 2, Art. XII of
the Constitution. Hence, the appellate court's observation that Tesoro is a foreign corporation not entitled to an MPSA is apt.
McArthur
Petitioner McArthur follows the corporate layering structure of Tesoro, as 59.97% of its 10,000 common shares is
owned by supposedly Filipino Madridejos Mining Corporation (Madridejos), while 39.98% belonged to the Canadian MBMI.
Name Nationality Number Amount Amount Paid
of Shares Subscribed
Madridejos Mining Filipino 5,997 P5,997,000.00 P825,000.00
Corporation
MBMI Resources, Inc. 18 Canadian 3,998 P3,998,000.00 P1,878,174.60
Lauro L. Salazar Filipino 1 P1,000.00 P1,000.00
Fernando B. Esguerra Filipino 1 P1,000.00 P1,000.00
Manuel A. Agcaoili Filipino 1 P1,000.00 P1,000.00
Michael T. Mason American 1 P1,000.00 P1,000.00
Kenneth Cawkel Canadian 1 P1,000.00 P1,000.00
–––––––– ––––––––––––– –––––––––––––
Total 10,000 P10,000,000.00 P2,708,174.60
======= ============ ============
In turn, 66.63% of Madridejos' shares were held by Olympic while 33.31% of its shares belonged to MBMI. Yet
again, Olympic did not contribute to the paid-up capital of Madridejos and it was MBMI that provided 99.79% of the paid-up
capital of Madridejos.
Name Nationality Number Amount Amount Paid
of Shares Subscribed

Olympic Mines & Filipino 6,663 P6,663,000.00 P0.00


Development Corp. 19
MBMI Resources, Inc. Canadian 3,331 P3,331,000.00 P2,803,900.00
Amanti Limson Filipino 1 P1,000.00 P1,000.00
Fernando B. Esguerra Filipino 1 P1,000.00 P1,000.00
Lauro Salazar Filipino 1 P1,000.00 P1,000.00
Emmanuel G. Hernando Filipino 1 P1,000.00 P1,000.00
Michael T. Mason American 1 P1,000.00 P1,000.00
Kenneth Cawkel Canadian 1 P1,000.00 P1,000.00
–––––––– ––––––––––––– –––––––––––––
Total 10,000 P10,000,000.00 P2,809,900.00
======= ============ ============
Again, the fact that MBMI had practically provided all the funds in Madridejos and McArthur creates serious
doubt as to the true extent of its control and ownership of MBMI over both Madridejos and McArthur. The
application of the Grandfather Rule is clearly called for, and as will be shown below, MBMI, along with the other foreign
shareholders, breached the maximum limit of 40% ownership in petitioner McArthur, rendering the petitioner disqualified to
an MPSA:
Filipino participation in petitioner McArthur: 40.01%
66.67
––––– (Filipino equity in Madridejos) x 59.97 (Madridejos' share in McArthur) = 39.98%
100

39.98% + .03% (shares of individual Filipino SHs in McArthur) = 40.01%


============
Foreign participation in petitioner McArthur: 59.99%

(Foreign equity in Madridejos) x 59.97 (Madridejos' share in McArthur) =

% (MBMI's direct participation in McArthur) + .02% (shares of foreign individual SHs in McArthur) =

As with petitioner Tesoro, with only 40.01% Filipino ownership in petitioner McArthur, as compared to 59.99%
foreign ownership of its shares, it is clear that petitioner McArthur does not comply with the minimum Filipino equity
requirement imposed in Sec. 2, Art. XII of theConstitution. Thus, the appellate court did not err in holding that petitioner
McArthur is a foreign corporation not entitled to an MPSA.
Narra
As for petitioner Narra, 59.97% of its shares belonged to Patricia Louise Mining & Development Corporation
(PLMDC), while Canadian MBMI held 39.98% of its shares.
Name Nationality Number Amount Amount Paid
of Shares Subscribed

Patricia Lousie Mining Filipino 5,997 P5,997,000.00 P1,677,000.00


and Development Corp.
MBMI Resources, Inc. 20 Canadian 3,996 P3,996,000.00 P1,116,000.00
Higinio C. Mendoza, Jr. Filipino 1 P1,000.00 P1,000.00
Henry E. Fernandez Filipino 1 P1,000.00 P1,000.00
Ma. Elena A. Bocalan Filipino 1 P1,000.00 P1,000.00
Michael T. Mason American 1 P1,000.00 P1,000.00
Robert L. McCurdy Canadian 1 P1,000.00 P1,000.00
Manuel A. Agcaoili Filipino 1 P1,000.00 P1,000.00
Bayani H. Agabin Filipino 1 P1,000.00 P1,000.00
–––––––– ––––––––––––– –––––––––––––
Total 10,000 P10,000,000.00 P2,800,000.00
======= ============ ============
Yet again, PASRDC did not pay for any of its subscribed shares, while MBMI contributed 99.75% of PLMDC's paid-
up capital. This factcreates serious doubt as to the true extent of MBMI's control and ownership over both PLMDC
and Narra since "a reasonable investor would expect to have greater control and economic rights than other investors who
invested less capital than him." Thus, the application of the Grandfather Rule is justified. And as will be shown, it is clear
that the Filipino ownership in petitioner Narra falls below the limit prescribed in both the Constitution and the Philippine
Mining Act of 1995.
Filipino participation in petitioner Narra: 39.64%
66.02
––––– (Filipino equity in PLMDC) x 59.97 (PLMDC's share in Narra) = 39.59%
100

39.59% + .05% (shares of individual Filipino SHs in McArthur) = 39.64%


=========
Foreign participation in petitioner Narra: 60.36%

– (Foreign equity in PLMDC) x 59.97 (PLMDC's share in Narra) =

9.96% (MBMI's direct participation in Narra) + .02% (shares of foreign individual SHs in McArthur) =

With 60.36% foreign ownership in petitioner Narra, as compared to only 39.64% Filipino ownership of its shares, it
is clear that petitioner Narra does not comply with the minimum Filipino equity requirement imposed in Section 2, Article XII
of the Constitution. Hence, the appellate court did not err in holding that petitioner McArthur is a foreign corporation not
entitled to an MPSA.
It must be noted that the foregoing determination and computation of petitioners' Filipino equity composition was
based on theircommon shareholdings, not preferred or redeemable shares. Section 6 of the Corporation Code of the
Philippines explicitly provides that "no share may be deprived of voting rights except those classified as 'preferred' or
'redeemable' shares." Further, as Justice Leonen puts it, there is "no indication that any of the shares . . . do not have voting
rights, [thus] it must be assumed that all such shares have voting rights." 22 It cannot therefore be gainsaid that the
foregoing computation hewed with the pronouncements of Gamboa, as implemented bySEC Memorandum Circular No. 8,
Series of 2013, (SEC Memo No. 8) 23 Section 2 of which states:
Section 2. All covered corporations shall, at all times, observe the constitutional or statutory
requirement. For purposes of determining compliance therewith, the required percentage of Filipino
ownership shall be applied to BOTH (a) the total outstanding shares of stock entitled to vote in the
election of directors; AND (b) the total number of outstanding shares of stock, whether or not entitled to
vote in the election of directors.
In fact, there is no indication that herein petitioners issued any other class of shares besides the 10,000 common
shares. Neither is it suggested that the common shares were further divided into voting or non-voting common shares.
Hence, for purposes of this case, items a) and b) in SEC Memo No. 8 both refer to the 10,000 common shares of each of
the petitioners, and there is no need to separately apply the 60-40 ratio to any segment or part of the said common shares.
III.
In mining disputes, the POA has jurisdiction to pass upon the nationality of applications for MPSAs
Petitioners also scoffed at this Court's decision to uphold the jurisdiction of the Panel of Arbitrators (POA) of the
Department of Environment and Natural Resources (DENR) since the POA's determination of petitioners' nationalities is
supposedly beyond its limited jurisdiction, as defined in Gonzales v. Climax Mining Ltd. 24 and Philex Mining Corp. v.
Zaldivia. 25
The April 21, 2014 Decision did not dilute, much less overturn, this Court's pronouncements in
either Gonzales or Philex Mining that POA's jurisdiction "is limited only to mining disputes which raise questions of fact," and
not judicial questions cognizable by regular courts of justice. However, to properly recognize and give effect to the
jurisdiction vested in the POA by Section 77 of the Philippine Mining Act of 1995, 26 and in parallel with this Court's ruling
in Celestial Nickel Mining Exploration Corporation v. Macroasia Corp., 27 the Court has recognized in its Decision that in
resolving disputes "involving rights to mining areas" and "involving mineral agreements or permits," the POA has jurisdiction
to make a preliminary finding of the required nationality of the corporate applicant in order to determine its right to a mining
area or a mineral agreement.
There is certainly nothing novel or aberrant in this approach. In ejectment and unlawful detainer cases, where the
subject of inquiry is possession de facto, the jurisdiction of the municipal trial courts to make a preliminary adjudication
regarding ownership of the real property involved is allowed, but only for purposes of ruling on the determinative issue of
material possession.
The present case arose from petitioners' MPSA applications, in which they asserted their respective rights to the
mining areas each applied for. Since respondent Redmont, itself an applicant for exploration permits over the same mining
areas, filed petitions for the denial of petitioners' applications, it should be clear that there exists a controversy between the
parties and it is POA's jurisdiction to resolve the said dispute. POA's ruling on Redmont's assertion that petitioners are
foreign corporations not entitled to MPSA is but a necessary incident of its disposition of the mining dispute presented
before it, which is whether the petitioners are entitled to MPSAs.
Indeed, as the POA has jurisdiction to entertain "disputes involving rights to mining areas," it necessarily follows
that the POA likewise wields the authority to pass upon the nationality issue involving petitioners, since the resolution of this
issue is essential and indispensable in the resolution of the main issue, i.e., the determination of the petitioners' right to the
mining areas through MPSAs.
WHEREFORE, We DENY the motion for reconsideration WITH FINALITY. No further pleadings shall be
entertained. Let entry of judgment be made in due course. SO ORDERED.
||| (Narra Nickel Mining and Development Corp. v. Redmont Consolidated Mines Corp., G.R. No. 195580 (Resolution),
[January 28, 2015])

You might also like